You are on page 1of 686

Official Guide 2019

Official Guide 2019 Official Guide 2019


The definitive guide from the makers of the NMATTM exam
Book + Online

Here’s what you will find inside: Book + Online


„ Information about the exam format and structure
„ Extensive review of concepts and question types of each section, including tips
The ONLY
and strategies for success
source of real
TM
„ Past NMAT
TM
questions with answer explanations
NMAT
questions
„ Practice questions with answer explanations
from past
Never before seen: 240 NMATTM questions

TM
exams
„ Other official prep materials from the house of GMAC: NMAT by GMAC Official
Prep Online Practice Exam offerings including an NMAT by GMAC™ Official Prep
Practice Exam 4 (Official Guide Access Code) exclusively with the purchase of
the Official Guide through access code provided inside. This edition includes
TM
Visit nmat.org.in, the official website of the NMAT by GMAC exam, to learn more, 240 never-before-seen questions
register and purchase official prep materials. „ Discussion of concepts of each section with relevant examples
Cover Design: Wiley Book „ 480 questions from past NMAT by GMAC
TM
exams including
+ 240 new questions, with answer keys & explanations
ISBN 978-81-265-9818-2

„ 625 practice questions with answer keys & explanations

Online
9 788126 598182

Price: `899/- Graduate Management Admission Council nmat.org.in


Official Guide 2019
Access the Free NMAT by GMACTM Official Practice
Exam 4 (Official Guide Access Code) GMAC

1. Visit www.nmat.org.in
2. Go to NMAT by GMAC™ Official Guide page in the
PREPARATION tab and use the access code
3. To get access code for the free Practice Exam 4,
please write to us at srehman@wiley.com, with a
copy of the Amazon's invoice to verify purchase

Terms and Conditions:


• Code can be used only once.

Note: This access code offers the original purchaser access to the free NMAT by GMAC TM Official
Practice Exam 4 (Official Guide Access Code) on www.nmat.org.in under the Preparation tab. The
online practice exam is available until the earlier of 30th April 2020 or the publication of the 2020
Official Guide practice exam. GMAC and Wiley are not responsible for providing access to the
online companion for any customer who purchases or borrows a used or pirated copy of this book.
This code will only provide you with access to the free online Official Guide practice exam and will not
work for other Wiley or GMAC password-protected websites or services.
NMAT by GMAC™ Official Guide 2019
Actual NMAT by GMACTM Test Questions Copyright © 2019 by the Graduate Management Admission
Council®.
All rights reserved.
Practice Test Questions & Content © 2019 by Wiley. All rights reserved.
Published by Wiley India Pvt. Ltd., 4436/7, Ansari Road, Daryaganj, New Delhi - 110002.
No part of this publication may be reproduced, stored in a retrieval system or transmitted in any form or by
any means, electronic, mechanical, photocopying, recording, scanning or otherwise, except as permitted
under Sections 107 or 108 of the 1976 United States Copyright Act, without either the prior written
permission of the Publisher, or authorization through payment of the appropriate per-copy fee to the
Copyright Clearance Center, 222 Rosewood Drive, Danvers, MA 01923, (978) 750-8400, fax (978) 646-8600,
or on the web at www.copyright.com. Requests to the Publisher for permission should be addressed to the
Permissions Department, John Wiley & Sons, Inc., 111 River Street, Hoboken, NJ 07030, (201) 748-6011,
fax (201) 748-6008, or online at http://www.wiley.com/go/permissions.
The publisher and the author make no representations or warranties with respect to the accuracy or
completeness of the contents of this work and specifically disclaim all warranties, including without
limitation warranties of fitness for a particular purpose. No warranty may be created or extended by sales or
promotional materials. The advice and strategies contained herein may not be suitable for every situation.
This work is sold with the understanding that the publisher is not engaged in rendering legal, accounting, or
other professional services. If professional assistance is required, the services of a competent professional
person should be sought. Neither the publisher nor the author shall be liable for damages arising here from.
The fact that an organization or Website is referred to in this work as a citation and/or a potential source of
further information does not mean that the author or the publisher endorses the information the
organization or Website may provide or recommendations it may make. Further, readers should be aware
that Internet Websites listed in this work may have changed or disappeared between when this work was
written and when it is read.
Trademarks: Wiley, the Wiley logo, and related trademarks are trademarks or registered trademarks of John
Wiley & Sons, Inc. and/or its affiliates. NMAT by GMAC™ © 2015 Graduate Management Admission
Council (GMAC). All rights reserved. NMAT™ is a registered trademark of Graduate Management Global
Connection (India) Private Limited (GMGC), a subsidiary of Graduate Management Admission Council
(GMAC). NMAT by GMAC™ is a trademark, and the GMAC logo, GMAC®, GMAT®, Graduate
Management Admission Council®, and Graduate Management Admission Test® are registered trademarks
of the Graduate Management Admission Council (GMAC) in the United States and other countries. All
other trademarks are the property of their respective owners. John Wiley & Sons, Inc., is not associated with
any product or vendor mentioned in this book.
John Wiley & Sons, Inc., also publishes its books in a variety of electronic formats and by print-on-demand.
Not all content that is available in standard print versions of this book may appear or be packaged in all
book formats. If you have purchased a version of this book that did not include media that is referenced by
or accompanies a standard print version, you may request this media by visiting
http://booksupport.wiley.com. For more information about Wiley products, visit us at www.wiley.com.
ISBN: 978-81-265-9818-2
ISBN: 978-81-265-8868-8 (ebk)

10 9 8 7 6 5 4 3 2 1
Table of Contents Letter from President and CEO, GMAC™ 5
How to Use NMAT BY GMAC™ Official Guide 2019 7

1.0 What is the NMAT by GMAC™ Exam? 8


1.0 What is the NMAT by GMAC™ Exam? 9
1.1 Why take the NMAT by GMAC™ Exam? 9
1.2 NMAT by GMAC™ Exam Format 13
1.3 NMAT by GMAC™ Exam Structure and Features 13
1.4 Review Exam 13
1.5 NMAT by GMAC™ Scores 14
1.6 About GMAC™ 15

2.0 Quantitative Skills Review 16


2.0 Quantitative Skills Review 17
2.1 Top Tips to Prepare for Quantitative Skills 18
2.2 Arithmetic 20
2.3 What is Measured? 21
2.4 Overall Test Taking Strategies 21
2.5 Algebra 76
2.6 What is Measured? 77
2.7 Overall Test Taking Strategies 77
2.8 Geometry and Mensuration 86
2.9 What is Measured? 87
2.10 Overall Test Taking Strategies 87
2.11 Modern Math 110
2.12 What is Measured? 111
2.13 Overall Test Taking Strategies 111
2.14 Data Interpretation 142
2.15 What is Measured? 143
2.16 Overall Test Taking Strategies 143
2.17 Data Sufficiency 154
2.18 What is Measured? 155
2.19 Overall Test Taking Strategies 155

3.0 Quantitative Skills Practice 162


3.1 Practice Questions 163
3.2 Answers and Explanations 216

4.0 Language Skills Review 278


4.0 Language Skills Review 279
4.1 Top Tips to Prepare for Language Skills 280
4.2 Vocabulary 282
4.3 What is Measured? 283
4.4 Overall Test Taking Strategies 283
4.5 Grammar 346
4.6 What is Measured? 347
4.7 Overall Test Taking Strategies 347
4.8 Comprehension 368

Prelim Pages.indd 4 01/05/2019 4:20:36 PM


4.9 What is Measured? 369
4.10 Overall Test Taking Strategies 369

5.0 Language Skills Practice 386


5.1 Practice Questions 387
5.2 Answers and Explanations 445

6.0 Logical Reasoning Review 506


6.0 Logical Reasoning Review 507
6.1 Top Tips to Prepare for Logical Reasoning 508
6.2 Verbal Reasoning 510
6.3 What is Measured? 511
6.4 Overall Test Taking Strategies 511
6.5 Analytical Reasoning 542
6.6 What is Measured? 543
6.7 Overall Test Taking Strategies 543

7.0 Logical Reasoning Practice 580


7.1 Practice Questions 581
7.2 Answers and Explanations 634

Prelim Pages.indd 5 01/05/2019 4:20:37 PM


Dear Future NMAT by GMAC™ Test Taker and Business Leader,

Preparing for the NMAT by GMAC™ exam is an important step in your graduate management education
journey. The NMAT by GMAC™ Official Guide 2019, is designed to help you study and achieve your
personal best.

To help you succeed, this Official Guide includes 240 never-before-seen NMAT™ questions included from
past NMAT™ exams. You will find guidance on exam format, structure, and question type, as well as
discussion of concepts within each section. Our over 60 years of research and experience shows that proper
preparation and using the right materials are essential to achieving a competitive score on any high-stakes
exam.

The Graduate Management Admission Council (GMAC) is a global, non-profit association of leading
graduate business schools. Founded in 1953, we are actively committed to advancing the art and science
of admissions by convening and representing the industry and offering best-in-class products and services
for schools and students. GMAC™ owns and administers the Graduate Management Admission Test™
(GMAT™) exam, the NMAT by GMAC™ exam, and the Executive Assessment. GMAC™ also manages
mba.com, the most frequently visited site for information about graduate management education.

Our mission is to connect the talent and aspirations of students globally with the opportunities provided
through business and management education, and our official prep materials are designed to help you make
that connection. Using this Official Guide to prepare for the NMAT by GMAC™ exam is an important step
toward gaining admission to a high-quality business or management school or programme of your choice.

I applaud your commitment to studying for the NMAT by GMAC™ exam and wish you the best success
throughout your education and career.

Sangeet Chowfla

President and CEO


Graduate Management Admission Council®
Owners of the GMAT™ and the NMAT by GMAC™ Exams

Prelim Pages.indd 6 01/05/2019 4:20:37 PM




Prelim Pages.indd 7 01/05/2019 4:20:37 PM


HOW TO USE
NMAT BY GMAC™ OFFICIAL GUIDE 2019
NMAT by GMAC™ Official Guide 2019 has been designed with the following focus to help you study
and achieve your personal best in the NMAT by GMAC™ exam:

Understand the NMAT by GMACTM Exam Format


Chapter 1 provides you in-depth information on the NMAT by GMAC™ exam format and structure,
scoring pattern, and other key details.

Learn the Key Concepts


The book takes a structured approach to acing the NMAT by GMAC™ exam by first introducing you to
the key theoretical concepts tested in each of the three sections—Language Skills, Quantitative Skills and
Logical Reasoning.
Therefore, Chapters 2, 4 and 6 are designed to provide you in-depth understanding of the key concepts and
strategies for acing each section of the exam.

Apply the Key Concepts


As the next step, the book provides a practice chapter after each of the three theory-based chapters. This
helps you immediately test your learning for maximum knowledge retention.
Hence, Chapters 3, 5 and 7 are designed to provide you extensive practice with the help of past years' real
NMAT questions as well as other practice questions.

Get Additional Practice through NMAT by GMAC™ Official Guide Practice Exam 4
(Official Guide Access Code)
As a bonus feature, the purchase of NMAT by GMAC™ Official Guide provides you free access to online
NMAT by GMAC™ Official Guide Practice Exam 4 (Official Guide Access Code).

Prelim Pages.indd 8 01/05/2019 4:20:37 PM


1.0   What is the NMAT by
GMAC Exam?
TM

Book 1.indb 8 30/04/2019 4:46:11 PM


1.0 What is the NMAT by GMACTM Exam?

1.0 What is the NMAT by GMACTM Exam?


The NMAT by GMACTM exam opens doors to the leading management programmes in India. The exam
is brought to you by the Graduate Management Admission Council (GMAC) the organisation that owns
the GMAT® exam. The NMAT by GMACTM enables schools to recruit top talent with the comfort that
the test measures the skills needed to succeed in their programmes.
Since becoming a computer-based test (CBT) in 2009, the NMAT by GMACTM has been at the forefront
of measurement within high stakes testing. The NMAT by GMACTM exam is used for securing admissions
to some of the leading graduate business programmes in India.
In 2019, the NMAT by GMACTM exam will build on these strengths by enhancing access to the exam.
Schools will have access to one of the more diverse test taking populations in India with some of the
highest proportions of women and non-engineers taking the exam. Prior to taking their exam, test takers
will be able to access practice material to enhance their preparations.

1.1 Why take the NMAT by GMACTM Exam?


The NMAT by GMACTM exam has been, and will continue to be, a symbol of quality—enabling test
takers to access graduate management education and a career of choice, while providing schools with access
to top talent.

Accepted as an admissions criterion by the leading management institutions in


India, South Africa, Philippines & Nigeria
Today the NMAT by GMACTM is accepted by many leading institutions and this list is growing. Pursuing
a management degree at one of these universities makes the pursuit of your dreams that much easier.
For more information, please refer to the NMAT by GMACTM Accepting Schools section or visit www.
nmat.org.in.

Measures the skills needed to succeed in business school


When entering business school, you want to know that you have what it takes to succeed and graduate
with flying colours. Having been selected using an assessment that measures the skills needed to succeed in
business school gives you the confidence that you can handle the curriculum and have a fulfilling b-school
experience.

Designed to give you the confidence to perform to the best of your ability
When appearing for any test, nerves are always a contributing factor in how well you perform on the day of
the test. The NMAT by GMACTM exam was designed to allay those nerves. Here are some ways that drive
that confidence:
1. Taking the test multiple times: You can take the NMAT by GMACTM three times in a given
admissions cycle. This gives you the confidence that you have more than a single opportunity to
perform to your very best.
2. Choosing the section order: The NMAT by GMACTM allows you to choose the order in which you
respond to each of the sections on the test. This allows you to play to your strengths.
3. Revisiting your answers: You can review your answers at the end of each section so you always have
the opportunity to revise your original response if you're not sure.

Book 1.indb 9 30/04/2019 4:46:11 PM


NMAT by GMAC™ Official Guide 2019

4. No negative marking: Sometimes you’re just not sure of the right answer on the test and are running
out of time. Well, you can make an informed guess instead of skipping a question without the fear of a
penalty for a wrong answer. The system of no negative marking is in line with global practices.
5. Does not require any computer typing skills: The NMAT by GMACTM user interface is designed
to minimise reliance on computer skills. You do not need to have prior computer experience to use
CBT. No typing skills are needed. You will use the computer mouse to highlight and confirm the
selected response as each question appears on the screen. At the beginning of the test, you will receive
a brief introductory tutorial that will instruct you on how to use the computer to answer questions and
review responses.

Designed for your convenience


We know that you are busy and the task of registering for the test, taking the test and applying to school
can be time consuming and stressful. So we’ve made it more convenient for you:
1. Choose a date, time and location of your convenience: A long testing window allows you to choose a
convenient test date. The 75-day test window opens the first week of October and extends through the
third week of December.
2. Self-schedule: You can choose your test appointment date, time and location in real-time based on
the availability of seats at each test centre.
3. Register from your mobile: This feature allows you to register anytime and anywhere.
4. Choose from multiple locations: You can choose from any of 48 conveniently located testing centres
to take the exam.
5. Send scores to up to 5 programmes for free: You can choose to select the list of programmes you
want to send your scores to before or after taking the exam. Up to 5 programmes are included in your
registration fee. However, you will be able to send your scores to more than 5 programmes before or
after you register for an additional fee.
6. Apply to business schools after reviewing results: You can view your results and then decide to apply
to a business school or retake the test depending on your comfort with the results you’ve obtained.
7. You may now send your NMAT score to the leading B-Schools in India, South AFrica, Philippines,
Nigeria. Please check nmat.org.in for updated list of NMAT accepting school countries.

10

Book 1.indb 10 30/04/2019 4:46:11 PM


Book 1.indb 11 30/04/2019 4:46:11 PM
1.2  NMAT by GMAC Exam
TM

Format

Book 1.indb 12 30/04/2019 4:46:11 PM


1.0 What is the NMAT by GMACTM Exam? 1.2 NMAT by GMACTM Exam Format

1.2 NMAT by GMACTM Exam Format


The NMAT by GMACTM exam is computer-delivered and gives each candidate a randomly generated
test. The number of questions, difficulty levels, and the time limit for each of the sections is predetermined
and, under normal administration conditions, are the same regardless of when and where the test is
administered. Candidates have a choice of selecting the order/sequence of the test sections.

1.3 NMAT by GMACTM Exam Structure and Features


The NMAT by GMACTM exam has three sections targeted at measuring different skills—Language Skills,
Quantitative Skills, and Logical Reasoning.

Each candidate will receive an exam with 120 items divided into the following sections:

Language Skills 32 questions 22 minutes


Quantitative Skills 48 questions 60 minutes
Logical Reasoning 40 questions 38 minutes
Total 120 questions 120 minutes

Each section is further divided into subsections.


Candidates have a choice of selecting the order/sequence of the test sections.
The three sections of the NMAT by GMACTM are individually timed. Test takers must answer questions
and review answers of each section within the allotted time. Test takers must review their answers before
moving to another section of the exam, as once the candidate closes out of a section, all answers are final.

1.4 Review Exam


At the end of each section, candidates will be presented with a Review screen to identify the questions that
they have left incomplete or flagged for review while attempting the test. Candidates can go directly to
specific questions by clicking on the question number on the Review screen.
Each of the three sections has individual section timings and candidates have to answer and review the
questions of the particular section within the allotted time.
In case a candidate completes the section before the section time expires, the balance time is added to the
next section(s). The next section has its own time allocation and candidates must complete the section
within the allotted time.

13

Book 1.indb 13 30/04/2019 4:46:11 PM


NMAT by GMAC™ Official Guide 2019

1.5 NMAT by GMACTM Scores


The NMAT by GMACTM exam scaled scores by section and test total scaled score will be announced
within 4-5 weeks after the candidate has tested. The percentile scores will be calculated after all candidates
have taken their exam(s) and announced in January of every year on the NMAT by GMACTM website at
www.nmat.org.in.

What is included in your score report?


Your scores will include a scaled score for each section, a total score as well as a percentile ranking. The
following table lists the range of scores possible on the different sections of the NMAT by GMACTM
exam.

Section Score Range


Quantitative Skills 0-144
Language Skills 0-96
Logical Reasoning Skills 0-120
Total 0-360

The final percentile rankings for this year will be calculated after all the candidates have taken their exam(s)
and announced in January of every year on the NMAT by GMACTM website at www.nmat.org.in.

Scaled scores and percentile scores


Scaled scores—All of the test forms are equated to account for any differences in overall difficulty and the
raw test scores are placed on a common scale. This process ensures that scores are comparable across test
forms such that the scores are not impacted by different candidates getting different test forms.
Percentile scores—The percentile rank or score shows how a candidate performed compared to all other
candidates who took the exam (for example, all candidates who took the NMAT by GMACTM exam in
a testing year). For example, if a candidate is said to be at the 75th percentile, the candidate scored better
than 75 percent of the candidates who took the NMAT by GMACTM in the testing year.
Candidates will not be sent any results directly. GMACTM will publish scaled scores every 4-5 weeks and
the final percentile results will be posted on the NMAT by GMACTM website. Once the scores have been
published, you will receive an email indicating that your scores are available. You will be able to access by
logging into the website www. nmat.org.in. At the same time, your scores will be sent to the schools that
you elected to share your scores.

14

Book 1.indb 14 30/04/2019 4:46:11 PM


1.0 What is the NMAT by GMACTM Exam? 1.2 NMAT by GMACTM Exam Format

1.6 About GMAC TM


The Graduate Management Admission Council (GMAC) is a global non-profit organisation comprised
of leading graduate business schools around the world. GMAC™ is the owner and administrator of the
GMAT® and NMAT by GMACTM exams.
GMAC™believes that business and management are critical to the economic and social well-being of people
worldwide. We advocate for graduate management education and its value to individuals and society.
GMAC™strives to increase access in order to grow and diversify the candidate pool; develop assessments to
meet global management education needs; and extend the value to schools with our products, services, and
industry knowledge.
GMAC™ meets the needs of management schools, and current and prospective students, through a wide
array of products, services, and programmes designed to open doors for students, professionals, and
graduate management programmes worldwide. GMAC® also serves as the leading source of research and
information about quality graduate management education.

In various Practice Sections of this book, several questions are marked as Real NMAT Question. Real
NMAT Question denotes that this question has appeared in an NMAT™ (now NMAT by GMAC™)
exam in the past.

15

Book 1.indb 15 30/04/2019 4:46:11 PM


2.0  Quantitative Skills Review

Book 1.indb 16 30/04/2019 4:46:11 PM


2.0  Quantitative Skills Review 

2.0 Quantitative Skills Review


This section provides a comprehensive review of the key mathematical concepts assessed in the NMAT by
GMACTM exam. These concepts are elucidated with the help of several examples. It is recommended that
you review the concepts and practice the different examples to build proficiencies.
The first section, ‘Arithmetic’, includes the following topics:
1. Number System
2. Averages
3. Percentages
4. Ratio and Proportion
5. Exponents
6. Profit, Loss and Discount
7. Alligations and Mixtures
8. Time, Speed and Distance
9. Time and Work
10. Clocks
11. Calendars
The second section, ‘Algebra’, includes the following topics:
1. Equations
2. Inequalities
3. Logarithm
The third section, ‘Geometry and Mensuration’, includes the following topics:
1. Lines and Angles
2. Triangles
3. Quadrilaterals
4. Circles
5. Mensuration
The fourth section, ‘Modern Math’, includes the following topics:
1. Permutation and Combination
2. Probability
3. Progression
4. Set Theory
5. Coordinate Geometry
The fifth section includes ‘Data Interpretation’

The sixth section includes ‘Data Sufficiency’

17

Book 1.indb 17 30/04/2019 4:46:12 PM


NMAT by GMAC™ Official Guide 2019

2.1 Top Tips to Prepare for Quantitative Skills


1. You will get 48 questions in the Quantitative Skills section on the NMAT by GMAC™ exam that
you will have to attempt in 60 minutes. This means you have roughly 1.25 minutes per question.
2. This section will test you on the four important areas of Math—Arithmetic, Algebra, Geometry and
Modern Math.
3. Make sure you have looked at all of the questions before the time allotted for the section runs
out. Evaluate each question on the basis of the time you think it would take you to solve it and its
difficulty level and attempt the easier ones first since the marks are the same for each question.
4. Time is the most important resource on the NMAT by GMAC™ exam. If you feel you can answer a
question correctly but it will take you 2 minutes or more to do so, flag and skip the question, and come
back to it at the end if you have time left.
5. Do not assume that questions should always be answered using the long methods you learnt in school.
Most of the NMAT by GMAC™ quant questions can be solved faster using short cuts and tricks,
some of which you will learn in this book.
6. Start with the basics. Make sure you are clear on fundamental number properties, formulae and
number operations concepts such as odds and evens, prime numbers, Least Common Multiples
(LCM) and Highest Common Factors (HCF), etc., before moving on to the more advanced concepts.
7. Given the time constraints, your mental math will have to be quite good to arrive at the answer
quickly. So, make sure you remember multiplication tables and square of numbers up to 20. It also
1
helps to know some common fractions and their percentage equivalents such as 12.5% = , etc.
8
8. For Data Sufficiency questions, focus on the sufficiency aspect and not on the actual answer.
9. Read the question carefully so that you understand it properly before answering. Do not assume
anything. A slight misinterpretation of the question can take you to an incorrect answer.
10. Take a quick look at the options before you attempt to solve a question. In some questions, it may be
faster and easier to simply back-solve from the answer choices.

18

Book 1.indb 18 30/04/2019 4:46:12 PM


Book 1.indb 19 30/04/2019 4:46:12 PM
2.2 Arithmetic

Book 1.indb 20 30/04/2019 4:46:12 PM


2.0  Quantitative Skills Review  2.2  Arithmetic

2.3 What is Measured?


The arithmetic section comprises topics such as averages, profit and loss, ratio and proportion, percentages,
time, speed and distance, number systems, simple and compound interest, mixtures, calendars, etc.
Basic arithmetic questions test your ability to interpret and solve problems of a mathematical nature, using
such operations as addition, subtraction, division and multiplication, and in a variety of problem formats
and situations.
While most of the concepts in arithmetic are quite simple, the NMAT by GMAC™ will not always test
you on straightforward concepts; rather, it will mix up two or more topics, so you may see a question that
combines percentages and ratio and proportion.
The arithmetic section will also test your mental math skills as you will be required to make quick
calculations in your head.

2.4 Overall Test Taking Strategies


• Read the question carefully.
• Look at the options once before you start solving the question.
• Be aware of the common mathematical operations between odd and even numbers.
• Remember the divisibility rules for common numbers, frequently tested squares and cubes and
multiplication tables up to 20.
• Be clear about the equivalent of common fractions as percentage and also about decimal terms.
• Try to pick numbers and back-solve from the answer choices.
The next few sections will provide you with in-depth strategies for approaching each topic.

21

Book 1.indb 21 30/04/2019 4:46:12 PM


NMAT by GMAC™ Official Guide 2019

1 Number System
Introduction
In order to understand quantitative skills, it is important to understand numbers as they are the basic
building blocks of entire mathematics. This unit is about understanding numbers and their basic properties.

Understanding numbers
While numbers can be divided into various types, for the NMAT by GMAC™, you need to be conversant
with the following types of numbers.

Natural and whole numbers


Numbers from 0, 1, 2, 3, so on are known as whole numbers. Natural numbers do not include 0.

Rational and Irrational Numbers


p
The real numbers expressed in the form of (where q ≠ 0) are called rational numbers. They include
q
fractions, integers, decimals (terminating and recurring), natural numbers and whole numbers. 3 27 , – 4,
2
2.56, , 1.33 etc. are the examples of rational numbers.
5
p
The real numbers that cannot be expressed in the form of   (where q ≠ 0) are called irrational numbers.
q
They include non-terminating and non-recurring decimal numbers. 3 6 , 2 5, − 8, 1.67834569012…, etc.

are the examples of irrational numbers.

Integers and fractions


Numbers -1, 0, 1, 2, etc., which have no fractional part, are called integers. Integers include the counting
numbers (1, 2, 3,…), their negative counterparts (−1, −2, −3,…) and 0.

Important Learning:  0 as a number is neither a negative integer nor a positive integer. Also, 0 and all
positive integers are called non-negative integers.

A fraction is a quantity that represents a part of a whole. It has two parts—a numerator and a denominator.
There are two types of fractions:
A
1. Proper fractions: Fractions of the form , where A and B are integers and A is less than B, are
B
2 4 7
called proper fractions. For example, , , and so on.
3 7 11
A
2. Improper fractions: Fractions of the form , where A and B are integers and A is greater than B,
B

are called improper fractions. For example, 5 , 8 , 11 and so on.


3 7 9

22

Book 1.indb 22 30/04/2019 4:46:14 PM


2.0  Quantitative Skills Review  2.2  Arithmetic

Important concepts about fractions


1. If the numerator is increased while keeping the denominator constant, the fraction increases in value
and vice versa.
2. If the denominator is increased while keeping the numerator constant, the fraction decreases in value
and vice versa.
3. A quick method of comparing two positive fractions is to multiply the numerator of the first fraction
with the denominator of the second and vice versa. If the product on the left side is larger, then the
left fraction is greater and if the product on the right side is larger, then the right fraction is greater.
3 5
For example, compare and . On cross-multiplying, we get 3 × 11 = 33 and 5 × 7 = 35
7 11
5
Since 35 > 33, the fraction that corresponds to 35, that is , is greater.
11
Prime and composite numbers
A number which has exactly two different factors, that is, 1 and the number itself is a prime number
(3, 11, 19, etc.) and a number having more than two different factors is a composite number (4, 12, 20, etc.).
Since the number 1 has only one factor that is 1 itself, it is neither a prime number nor a composite
number.

Important Learning:  1 as a number is neither prime nor composite.

Some properties and observations on prime numbers are:


1.
2 is the smallest prime number.
2.
2 is the only even prime number; all other prime numbers are odd.
3.
There are 25 prime numbers when counted from 1 to 100.
4.
Every prime number, except 2 and 3, can be expressed as 1 more than a multiple of 6 or 1 less than a
multiple of 6 (that is, 6N + 1 or 6N − 1).

Odd and even numbers


All the numbers which are completely divisible by 2 are called even numbers (e.g. 2, 4, 6, 8, 10) and
numbers which are not divisible by 2 are called odd numbers (e.g. 1, 3, 5, 7, 9, 11).
0 as a number will be treated as neither even nor odd, until and unless specified otherwise. Therefore, we
have
1. Five odd digits: 1, 3, 5, 7, 9
2. Four even digits: 2, 4, 6, 8

23

Book 1.indb 23 30/04/2019 4:46:14 PM


NMAT by GMAC™ Official Guide 2019

Important operations with odd and even numbers

Function Result
Even + Even Even
Even + Odd Odd
Odd + Odd Even
Even - Even Even
Even - Odd Odd
Odd - Odd Even
Even × Even Even
Even × Odd Even
Odd × Odd Odd
Even ÷ Even Anything (even, odd or not an integer)
Even ÷ Odd Even or not an integer
Odd ÷ Even Not an integer
Odd ÷ Odd Odd or not an integer

Example 1

If x is an odd number, which of the following must be even?


(A) x2
(B) x2 + 2
(C) (x + 1)
2
(D) 2x + 1
(E) 2x + 2

Solution

Note that the question says ‘must be even’, so there cannot be any exceptions. We know that any
number multiplied by an even number gives an even product, and also the sum of two even numbers is
an even number. Thus, option (E) should be our answer.
A lot of you who tried to solve this question by picking an odd number and plugging it into the
answer choices might have marked (C) as the answer. But 5 is also an odd number; try plugging 5 in
option (C) and see what happens.
The correct answer is E.

Decimals
Decimals are numbers that fall in between integers. They express a part-to-whole relationship in terms of
place value. For example, 1.2 is a decimal. The integers 1 and 2 are not decimals. An integer written as 1.0,
however, is considered a decimal.

24

Book 1.indb 24 30/04/2019 4:46:14 PM


2.0  Quantitative Skills Review  2.2  Arithmetic

Digits and place value


There are 10 digits that make up all numbers: 0, 1, 2, 3, 4, 5, 6, 7, 8 and 9. For example, the three-digit
number 412 consists of the digits 4, 1 and 2.

Every digit in a given number has a particular place value. The place value depends upon the digit’s location
relative to the decimal point.

Place value of digits in a number

6 7 8 9 1 0 2 3 . 8 3 4
H
U
N T
T O
D E T
E N
R N T H H
N E H
E H U O
U T
D T O N U
M M N T O E
H U D S
I I D E N N
T O S DECIMAL R A
L L R N E T
H U A E N
L L E S S H
O S N T D
I I D S
U A D H T
O O S
S N S S H
N N
A D S
S
N S
D
S

Divisibility rules for numbers


Now, let us look at the divisibility rules for some important numbers that will help you make quick
calculations.
1. Divisibility rule for 2: A number is divisible by 2 if its last digit is 0 or even, that is, 0, 2, 4, 6 or 8. All
even numbers are divisible by 2. For example, the last digit of 15646790 is 0; therefore, it is divisible
by 2.
2. Divisibility rule for 3: A number is divisible by 3 if the sum of all the digits of the number is divisible
by 3. For example, let us take the number 4,689. Now, the sum of the digits will be 4 + 6 + 8 + 9 = 27;
and since 27 is divisible by 3, the number 4,689 is also divisible by 3.
3. Divisibility rule for 4: A number is divisible by 4 if the number formed by its last 2 digits is divisible
by 4. Let us take the example of 4,689. The number formed by the last 2 digits is 89, which is not
divisible by 4. So, the given number is not divisible by 4. Also, since 89 divided by 4 gives a remainder
of 1, so 4,689 when divided by 4 will also give a remainder of 1.
4. Divisibility rule for 5: A number is divisible by 5 if its last digit is 0 or 5. For example, the last digit of
567899239645 is 5; therefore, it is divisible by 5.
5. Divisibility rule for 6: A number is divisible by 6 if it is divisible by both 2 and 3. For example, the
last digit of 1236 is 6. It is even; hence, it is divisible by 2. The sum of its digits (1 + 2 + 3 + 6) is 12,
which is divisible by 3. So, the given number is divisible by 6.

25

Book 1.indb 25 30/04/2019 4:46:14 PM


NMAT by GMAC™ Official Guide 2019

6. Divisibility rule for 7: If the difference of twice of last digit of a number and the number formed by
its remaining digits is either 0 or a multiple of 7, then the number is divisible by 7 (Repeat this process
until we get a smaller number whose divisibility is known to us).
Let us check whether 1456 is divisible by 7 or not.
145 – (6 × 2) = 133
13 – (3 × 2) = 7
Therefore, 1456 is divisible by 7.
7. Divisibility rule for 8: A number is divisible by 8 if the number formed by its last three digits is
divisible by 8. For example, the last three digits of 123696 are 696. 696 is divisible by 8; therefore,
123696 is divisible by 8.
8. Divisibility rule for 9: A number is divisible by 9 if the sum of all its digits is divisible by 9. For
example, the sum of all the digits of 6794568 is 45 (6 + 7 + 9 + 4 + 5 + 6 + 8) and 45 is divisible by 9.
Therefore, 6794568 is divisible by 9.
9. Divisibility rule for 10: A number is divisible by 10 if its last digit is 0. For example, the last digit of
27798870 is 0. Therefore, it is divisible by 10.
10. Divisibility rule for 11: If the difference between the sum of digits of a number at odd place and the
sum of digits of that number at even place is either 0 or a multiple of 11, then that number is divisible
by 11.
For example, the difference between the sum of digits of 23452 at odd place (2 + 4 + 2 = 8) and at
even place (3 + 5 = 8) is 0 (8 – 8). Therefore, 23452 is divisible by 11.
11. For divisibility of other composite numbers: Any number is divisible by a composite number
N (where N = m × p) if it is divisible by both m and p. There is a condition that m and p must be
co-prime.
For example, any number is divisible by 88 if it is divisible by both 8 and 11, but, not if it is divisible
by both 4 and 22.
12. Divisibility rule for 12: A number is divisible by 12 if it is divisible by both 3 and 4. For example, the
sum of all the digits of 7896 is 30 that is divisible by 3 and the number formed by the last two digits is
96 that is divisible by 4. So, 7896 is divisible by 12.
Example 2
If the number 2546bc is completely divisible by 3, find the possible values of b + c.
(A) 2
(B) 3
(C) 5
(D) 13
(E) 17
Solution
For a number to be divisible by 3, the sum of all the digits should be divisible by 3.
Now, 2 + 5 + 4 + 6 + b + c should be divisible by 3.
17 + b + c must be divisible by 3.
Therefore, b + c must be a (multiple of 3) + 1.
That is, 1, 4, 7, 10, 13, 16, 19.
b + c can take the mentioned nine values. 13 is the only value available in the options.
The correct answer is D.

26

Book 1.indb 26 30/04/2019 4:46:14 PM


2.0  Quantitative Skills Review  2.2  Arithmetic

Example 3

A number Q gives a remainder of 5 when divided by 7. Find the remainder when 2Q is divided by 7.
(A) 2
(B) 3
(C) 5
(D) 7
(E) 9
Solution

2Q when divided by 7 will give a remainder of 2 × 5, that is, 10. This can be divided by 7 and the
required remainder will be 3.
The other way of approaching this question is to pick a number that satisfies the criteria given in the
question, that is, it should leave a remainder of 5 when divided by 7. Let us pick Q as 12. So, when
2Q, that is, 24 is divided by 7, the remainder is 3, which is your answer. You can try this with any other
  38
number, such as 19, the result will remain the same  = 3  .
 7 
The correct answer is B.

Factors, Multiples, Least Common Multiple and Highest Common Factor


Factors and multiples
A factor is a number that is able to completely divide a number greater than or equal to it. For example, 2 is
a factor of 4 and 3 is a factor of 15, but 2 is not a factor of 15.
On the other hand, a multiple is a number that may be divided by another number a certain number of
times without leaving a remainder. So, 4 is a multiple of 2, 15 is a multiple of 3, but 15 is not a multiple of 2
since dividing 15 by 2 will leave a remainder of 1.

Factor foundation rule


If a is a factor of b and b is a factor of c, then a is also a factor of c. For example, 3 is a factor of 9 and 9 is a
factor of 81. Therefore, 3 is also a factor of 81.

Prime factorisation
Prime factorisation is a way to express any number as a product of prime numbers. For example, the prime
factorisation of 30 is 2 × 3 × 5. Prime factorisation is useful in answering questions about divisibility.
Example 4

Given that 1,176 = 2p × 3q × 7r, find the value of p + q + r.

(A) 6
(B) 8
(C) 9
(D) 10
(E) 12
Solution

The given number can be written as below:

27

Book 1.indb 27 30/04/2019 4:46:14 PM


NMAT by GMAC™ Official Guide 2019

1176 = 4 × 294 = 4 × 3 × 98 = 4 × 3 × 2 × 49 = 23 × 31 × 72
Since 1176 = 2p × 3q × 7r, therefore, p = 3, q = 1 and r = 2
Hence, p + q + r = 3 + 1 + 2 = 6
The correct answer is A.

Highest Common Factor (HCF)


We already have an understanding of what is a factor. The Highest Common Factor or HCF is the highest
common factor among all the factors of a set of given numbers. It is also known as the Greatest Common
Factor (GCF) or the Greatest Common Divisor (GCD). For example, the HCF of 24 and 40 will be 8,
since 8 is the largest number that can completely divide both 24 and 40.

Least Common Multiple (LCM)


The Least Common Multiple or LCM is the least common multiple of any set of given numbers. LCM
refers to the smallest multiple of two (or more) integers. Multiples will be equal to or larger than the given
integers. The LCM of 6 and 15 is 30, because 30 is the smallest number that both 6 and 15 go into.
Properties of HCF and LCM
Following are properties of HCF and LCM:
1.
For any set of given numbers, the LCM is always a multiple of the HCF.
2.
For two given numbers, product of the numbers is equal to the product of their HCF and LCM.
(This property will also hold true for 3 numbers, 4 numbers and so on, provided none of them have a
common factor other than 1.)
3.
For a set of given fractions

LCM of the numerator


LCM of fractions =
HCF of the denominator
HCF of the numerator
HCF of fractions =
LCM of the denominator
Before doing this, we need to bring the fractions to the smallest or lowest form.
Example 5

What is the smallest integer which is greater than 1 and which leaves a remainder of 2 when divided
by any of the integers 3, 5 and 7?

(A) 18
(B) 38
(C) 105
(D) 107
(E) 213
Solution

You start by finding out the smallest number that is divisible by 2, 5 and 7 (which will be their LCM)
and add 2 to the result.
The LCM of 3, 5 and 7 is 105, so our desired answer is 105 + 2 = 107.
The correct answer is D.

28

Book 1.indb 28 30/04/2019 4:46:15 PM


2.0  Quantitative Skills Review  2.2  Arithmetic

Example 6

What minimum number must be subtracted from 247 so that the number is divisible by both 6
and 7?
(A) 35
(B) 36
(C) 37
(D) 38
(E) 39
Solution

LCM of 6 and 7 is 42.


We need to find out a multiple of 42 closest to 247.
42 × 5, that is, 210 is the multiple of 42 closest to 247 and so the value to be subtracted from 247 so
that the number left is completely divisible by both 6 and 7 = 247 − 210 = 37.
The correct answer is C.

Word Problems on Numbers and Variables


There are a few things that need to be known before we look at various problems based on numbers and
variables:
1. A two-digit number will always be written as 10x + y and not as xy. Similarly, a three-digit number
will be written as 100x + 10y + z. But the digits of a two-digit number will be called x and y.
2. The sum of a two-digit number and the number obtained by interchanging the digits is always
divisible by 11.
(10x + y) + (10y + x) = 11x + 11y = 11(x + y)
3. The difference of a two digit number and the number obtained by interchanging the digits is always
divisible by 9.
4. Any two two-digit numbers can be added to make a maximum sum of 198.
Example 7

The sum of a two-digit number and the number obtained by reversing the digits is a multiple of 88. If
the difference of the digits at the ten’s place and unit’s place is 6, find the digit at the ten’s place of the
number.
(A) 1
(B) 3
(C) 5
(D) 6
(E) 7
Solution

Let the two-digit number be 10x + y and so the number obtained by reversing the digits is 10y + x.
Their sum will be 11x + 11y, that is, 11(x + y).
If 11(x + y) is a multiple of 88, then x + y is a multiple of 8, that is, x + y can be either 8 or 16.

29

Book 1.indb 29 30/04/2019 4:46:15 PM


NMAT by GMAC™ Official Guide 2019

Also, y - x = 6
Thus, one possible result is y = 7 and x = 1. The other result is y = 11 and x = 5, which is not possible.
The correct answer is E.

VBODMAS
VBODMAS stands for Vinculum Brackets, Of, Division, Multiplication, Addition and Subtraction. This
acronym helps us remember the sequence in which to carry out arithmetic operations.

Order of Operations
V - Vinculum or Bar
The bar you see on top of the values like X must be solved first.
B - Brackets
Parts of calculation inside the brackets are always done first after the vinculum or bar.
O - Orders
Solve for orders if there is any, like powers, square roots or cube roots.
DM – Divide or multiply before addition or subtraction

AS – Do addition and subtraction in the last (from left to right).

Note: ‘Of ’ is used as a multiply operand in simplification.

While performing simplification, always ensure that the brackets are opened in the following order of
priority, starting from (i)

i. Bar bracket ‘----’


ii. Circular bracket ( )
iii. Curly bracket { }
iv. Square bracket [ ]
Let’s take an example to simplify a statement using the BODMAS rule:

Number line and absolute value

.
−5 −4 −3 −2 −1 0 1 2 3 3.5 4 5

Number line
A number line is a line on which all real numbers can be placed as per their value. Each point on the
number line corresponds to a real number. For example, in the figure, above, the number 3.5 corresponds to
a point on the number line which is halfway between 3 and 4.
The absolute value of a point is its distance from 0 on the number line. A positive number is already in the
same form as that number’s absolute value. For a negative number, remove the negative sign to get that
number’s absolute value. For example, the absolute value of −2 is 2. The absolute value is denoted by two
vertical parallel lines.

30

Book 1.indb 30 30/04/2019 4:46:15 PM


2.0  Quantitative Skills Review  2.2  Arithmetic

Important Learning:  The absolute value of a number cannot be negative.

Largest Power of a Number in a Factorial


For Prime Numbers

Largest power of a prime number ‘p’ in factorial of any natural number ‘N’

N  N   N   N  N
=   +  2  +  3  +  4  +………+  n  , where N ≥ p n .
 p  p  p  p  p 
For any non-negative integer X, [X] denotes the greatest integer less than or equal to X.
For example
[3.15] = 3
Example 8

Find the largest power of 3 in 100!


(A) 46
(B) 47
(C) 48
(D) 49
(E) 50
Solution

100  100  100  100 


Largest power of 3 in 100! =  + 2 + 3 + 4
 3   3   3   3 

Note that here we cannot take  5  as 35 > 100.


100

 3 

∴ Largest power of 3 in 100! = 33 + 11 + 3 + 1 = 48

The correct answer is C.

For Composite Numbers

In order to find the largest power of a composite number in N! follow the steps given below.
(1) Factorise the given number into its prime factors.
(2) Find the largest power of largest prime factor of given number in N!.
The largest power of largest prime factor in N! is the largest power of given number in N!.

31

Book 1.indb 31 30/04/2019 4:46:16 PM


NMAT by GMAC™ Official Guide 2019

Example 9

Find the largest power of 30 in 50!


(A) 10
(B) 11
(C) 12
(D) 13
(E) 14
Solution

30 = 2 × 3 × 5
Since largest prime factor of 30 is 5, therefore, largest power of 5 in 50! is the largest power of
30 in 50!.

Largest power of 5 in 50! =   +  2  = 10 + 2 = 12


50 50

 5   5 
∴ Largest power of 30 in 50! = 12
The correct answer is C.

Find units digit of pq

Unit's place of pq depends on unit's place digit of ‘p’ and the divisibility of power ‘q’. Consider powers of 2,
as we know, 21 =2, 22 = 4, 23 = 8, 24 = 16, 25 = 32, 26 = 64, 27 = 128 and so on.
The units place digit for powers of 2 repeat in an order: 2, 4, 8, 6. So, the cyclicity of number 2 is 4
(that means the pattern repeats after 4 occurrences) and the cycle pattern is 2, 4, 8, 6. From this, you can see
that to find the units place digit of powers of 2, you have to divide the exponent by 4.
Shortcuts to solve problems related to units place digit of pq
1. Case 1: If ‘q’ is a multiple of 4
If ‘p’ is an even number, that is: 2, 4, 6 or 8, then the units place digit is 6
If ‘p’ is an odd number, that is: 1, 3, 7 or 9, then the units place digit is 1
2. Case 2: If ‘q’ is not a multiple of 4
Let ‘r’ be the remainder when ‘q’ is divided by 4, then units place of pq will be equal to units place of pr.

32

Book 1.indb 32 30/04/2019 4:46:17 PM


2.0  Quantitative Skills Review  2.2  Arithmetic

Cyclicity of numbers up to 9:

Number ^1 ^2 ^3 ^4 Cyclicity
2 2 4 8 6 4
3 3 9 7 1 4
4 4 6 4 6 2
5 5 5 5 5 1
6 6 6 6 6 1
7 7 9 3 1 4
8 8 4 2 6 4
9 9 1 9 1 2

Let us take some examples to understand it clearly.


(a) The last digit for (163)11 is the same as that of 311, that is, 34×2+3 which is the same as the last digit for
33, that is, last digit for 27 and so our last digit for (163)11 is 7.
(b) Last digit for (29)19 is the same as that of 919, that is, 92×9+1 which is the same as the last digit for
91 = 9.

2 Averages
Introduction
An average is typically the central value of a set of numbers.
For a set of ‘n’ values x1, x2,……xn, the average is given by the following algebraic expression:

x1 + x 2 + .... + x n
xavg =
n
Therefore, the formula for the average of a set of values can be expressed by the following formula:

Sum of all given values


Average =
Number of values
Example 1

The average of five consecutive integers is 20. What is the average of the first 3 of these integers?
(A) 15
(B) 17
(C) 18
(D) 19
(E) 21
Solution

We know that the average of consecutive integers is always the middle value. So, if the average is 20,
the integers are 18, 19, 20, 21, 22.

33

Book 1.indb 33 30/04/2019 4:46:17 PM


NMAT by GMAC™ Official Guide 2019

So, the first 3 integers in this list are 18, 19, 20 whose average will again be the middle value, that
is, 19.
The correct answer is D.

Properties of average
• If each number in a set of numbers is increased by ‘p’, then their average is also increased by ‘p’.
• If each number in a set of numbers is decreased by ‘p’, then their average is also decreased by ‘p’.
• Similarly, if each number in a set of numbers is multiplied or divided by ‘p’, then their average also gets
multiplied or divided by the same number ‘p’.

Average of two different groups


Let us consider there are two groups; group 1 and group 2 with respective averages ‘a’ and ‘b’. If the number
of total items in group 1 and group 2 are ‘m’ and ‘n’ respectively, then the combined average of the two
groups is given by the following expression:

ma + nb
Combined Average =
m+n

Change in average on deletion of an item


Let us consider that a value ‘x’ is deleted from a set of ‘n’ values with average ‘a’,. On deletion of an item the
average of the remaining values may either increase or decrease which depends on the value of the deleted
item. Therefore, it can be defined under two cases:

Case 1
If the average is increased by µ, then the deleted value ‘x’ is given by
x = a – (n – 1) µ
Example 2

The average of a set of five values is 12. If one number is deleted, the average of the set is increased by
0.6. What is the value of the deleted number?

(A) 8.2
(B) 8.8
(C) 9.2
(D) 9.6
(E) 10.2
Solution
Here, the original average (a) = 12
Total number of items (n) = 5
Decrease in the original average (µ) = 0.6
Therefore, the deleted value (x) = a – (n – 1) µ = 12 – (5 – 1)0.6 = 9.6
The correct answer is D.

34

Book 1.indb 34 30/04/2019 4:46:17 PM


2.0  Quantitative Skills Review  2.2  Arithmetic

Case 2
If the average is decreases by µ, then the deleted value ‘x’ is given by
x = a + (n – 1) µ
Example 3

The average of a set of five values is 12. If one number is deleted, the average of the set is decreased by
0.6. What is the value of the deleted number?

(A) 14.2
(B) 14.4
(C) 16.2
(D) 19.6
(E) 20.2
Solution

Here, the original average (a) = 12


Total number of items (n) = 5
Decrease in the original average (µ) = 0.6
Therefore, the deleted value (x) = a + (n – 1) µ = 12 + (5 – 1) 0.6 = 14.4
The correct answer is B.

Change in Average on Addition of an Item


Let us consider that a value ‘x’ is added to a set of ‘n’ values with the average ‘a’. On addition of an item, the
average may either increase or decrease which depends on the value of the added item. Therefore, it can be
defined under two cases:

Case 1

If the average is increased by µ, then the added value ‘x’ is given by


x = a + (n + 1) µ
Example 4

The average weight of a class of 13 students is 62.875 kg. When a new student joins the class, the
average weight increases to 62.985 kg. What is the weight of the new student?
(A) 64.415 Kg
(B) 65.825 Kg
(C) 66.545 Kg
(D) 67.215 Kg
(E) 69.615 Kg

Solution

Original average (a) = 62.875 kg


Increase in average weight (µ) = 62.985 – 62.875 = 0.11

35

Book 1.indb 35 30/04/2019 4:46:17 PM


NMAT by GMAC™ Official Guide 2019

Number of students (n) = 13


Therefore, the weight of the new students can be calculated using the formula:
x = a + (n + 1) µ
x = 62.875 + (13 + 1)0.11 = 64.415 kg
The correct answer is A.

Case 2

If the average is decreased by µ, then the added value ‘x’ is given by


x = a – (n + 1) µ
Example 5

Virat Kohli has an average score of 54 in the last 15 matches. After the last match his average
becomes 53. What was Virat’s score in the last match?
(A) 30 runs
(B) 34 runs
(C) 38 runs
(D) 42 runs
(E) 46 runs
Solution

Using the formula discussed above, we get.


Virat’s score in the last match = 54 – (15 + 1)1 = 38 runs
The correct answer is C.

3 Percentages
Introduction
Percentage is an important topic in the NMAT by GMAC™, not only because you may be asked
questions specifically related to percentages, but also because the knowledge of percentages helps you to
easily understand and attempt problems from other areas in arithmetic.
Also, the understanding of percentages forms an important aspect of data interpretation (DI), in which
questions require you to calculate percentage values, growth rates and other percentage changes.
So, what is a percentage? Any value expressed on a base of 100 or over a base of 100 is called percentage
and is represented as % (cent represents the base 100).
A fraction is another way in which the value of a particular percentage can be represented. Therefore, one
can say that percentages and fractions are equivalent and can be converted into one another as per the need.
1 1 1
For example, 25% is the same as , 33.33% is the same as , 50% is the same as and so on.
4 3 2

36

Book 1.indb 36 30/04/2019 4:46:18 PM


2.0  Quantitative Skills Review  2.2  Arithmetic

1. To convert a percentage into a fraction, divide the percentage by 100.


20 1
For example, 20% is same as = .
100 5
2. Similarly, to convert a fraction into a percentage, multiply the fraction by 100.

2 2
= × 100 = 40%
For example,
5 5
Percentage equivalent of fractions
We need to know the percentage equivalent of fractions in order to enhance our understanding of
percentages as a concept and to help in quick calculations.
1  1 1
For example, the percentage equivalent of will be   × 100 = 50%. Instead of saying or half of any
2  2  2
value, we can also say that we are calculating 50% of the given value.
1
Students must know the percentage equivalent of fractions up to .
20
Let us look at some of these values:

1 1 1 1 1
= 100% = 50% = 33.33% = 25% = 20%
1 2 3 4 5

1 1
1
= 16.67%
1
= 14.28% 1
= 12.5% = 11.11% = 10%
6 7 8 9 10

Percentage Change
Final value − Initial value
• Percentage Change = × 100
Initial value
Percentage Increase/Decrease

Actual increase
• Percentage increase = × 100
Initial quantity
Actual decrease
• Percentage decrease = × 100
Initial quantity

• If the price of a commodity increases by R%, then the reduction in consumption so as not to change

 R 
the expenditure is:  × 100  %
 (100 + R) 
• If the price of a commodity decreases by R%, then the increase in consumption so as not to change
 R 
the expenditure is:  × 100  %
 (100 − R) 

37

Book 1.indb 37 30/04/2019 4:46:20 PM


NMAT by GMAC™ Official Guide 2019

Example 1

If the price of a commodity increases by 20%, then by what percentage the consumption should be
reduced so as not to increase the expenditure?
(A) 12.5%
(B) 14.28%
(C) 16.67%
(D) 18.12%
(E) 20.25%
Solution
20 20
Required percentage = × 100 = × 100 = 16.67%
100 + 20 120

The correct answer is C.

Results on Population
Let the population of a town be P now and suppose it increases at the rate of R% per annum, then:
n
R 
• Population after n years = P  1 + 
 100 

P
• Population n years ago = n
 R 
 1+ 
 100 

Results on Depreciation
Let the present value of a machine be P. Suppose it depreciates at the rate of R% per annum. Then:

n
R 
• Value of the machine after n years = P  1 − 
 100 
P
• Value of the machine n years ago = n
 R 
 1− 
 100 

Successive Percentage Change


If any quantity is increased by x%, then y% and later on z%, then the overall or effective percentage
increase is:
  100 + x   100 + y   100 + z  
  100   100   100  − 1  × 100
    

Successive Percentage Change Formula


When a number increases by A% and then by B%, the overall percentage increase is equal to

 AB 
A+B+ %
 100 
38

Book 1.indb 38 30/04/2019 4:46:20 PM


2.0  Quantitative Skills Review  2.2  Arithmetic

Example 2

A number is increased by 20% and then the increased number is again increased by 10%. What is the
total increment in the number?
(A) 30%
(B) 31%
(C) 32%
(D) 33%
(E) 34%
Solution
20 × 10
Required percentage = 20 + 10 + = 20 + 10 + 2 = 32%
100
The correct answer is C.

Application of Successive Percentage Change Formula


We can use successive percentage change formula to solve percentage-related problems where the product
of two quantities equals the third quantity. For example,
Length × Breadth = Area
Price × Quantity purchased = Expenditure

Multiplication factor
R
To find the value of R% of a number, we multiply that number by . If we want to find out 35% of a
100
35
given number, we need to multiply the number by or 0.35.
100
(100 + R)
To increase a number by R%, we multiply the number by , and to decrease a number by R%, we
100
(100 − R)
multiply the number by .
100
We need to understand that finding out R% of a number and increasing or decreasing a given number by
R% are different operations.

The factor with which we multiply a number in order to (a) find the value of certain percentage of a given
number, (b) increase the value of a number by a particular percentage or (c) decrease the value of a number
by a particular percentage is called the multiplication factor.
(100 + 20) 120
For example, if we have to increase 120 by 20%, we need to multiply 120 by or or 1.2. In
100 100
this case, 1.2 is the multiplication factor. The result is 120 × 1.2 = 144. Therefore, if we increase 120 by 20%,
the final result will be 144.
Let us look at the multiplication factor for some cases:
(100 + 17)
1. To increase a number by 17%, the multiplication factor will be = 1.17.
100

39

Book 1.indb 39 30/04/2019 4:46:21 PM


NMAT by GMAC™ Official Guide 2019

(100 - 11)
2. To decrease a number by 11%, the multiplication factor will be = 0.89.
100
(100 + 34)
3. To increase a number by 34%, the multiplication factor will be = 1.34.
100
(100 - 30)
4. To decrease a number by 30%, the multiplication factor will be = 0.7.
100
5. To find 40% of a number, the multiplication factor will be 0.4.

Important Learning:  If A is 20% more than B, then B will not be 20% less than A.

Base and base change


In percentages, it is very important to understand the base on which the change is happening. For example,
let us take two numbers, say 40 and 50, and carry out the following operations:
1. Determine what percent of 40 is 50.
 50 
For this, we need to express 50 as a percentage of 40, that is,   × 100 = 125%. Therefore, 50 is
40
125% of the given value 40.
2. Express 40 as a percent of 50.
For this, we need to find out 40 as a percentage of the given base value, that is, 50. Therefore,
 40 
  × 100 = 80%
50

Do not commit this common error:  There are two things that a student must appreciate.
50% is what percentage of 40% and 50% is how much more than 40% are two different problems. In this
example, we are being asked about how much more is 50% than 40%.
Many students make the mistake of saying that the required answer is 10%, which is the difference
between the given values 50 and 40. The percentage affixed after the given values probably creates this
confusion. What if the two values were 50 km/h and 40 km/h, that is, speed, or 50 kg and 40 kg, that is,
weight?

Points to Remember
Some points to remember while resolving percentage related problems are listed as follows:
• A% of B = B% of A
For example, 20% of 80 = 80% of 20 = 16
 k 
• If percentage increase in initial quantity is k%, then the new value =  + 1  × Initial quantity.
 100 
• If new quantity becomes k times the old quantity, then the percentage increase is (k – 1) × 100%. For
example, if a quantity becomes 5 times of its initial value, then the percentage increase is 400%.

40

Book 1.indb 40 30/04/2019 4:46:22 PM


2.0  Quantitative Skills Review  2.2  Arithmetic

k
• If A is k% more than B, then B is × 100 % less than A.
(100 + k )
k
• If A is k% less than B, then B is × 100 % more than A.
(100 − k )
• If the price of a commodity increases by k%, then to keep expenditure constant, decrease in
k
consumption is × 100 %.
(100 + k )
• If the price of a commodity decreases by k%, then to keep expenditure constant, increase in
k
consumption is × 100%.
(100 − k )
1
• If increase in price isof the original price, then decrease in quantity purchased so that expenditure
x
1
remains unchanged is equal to of the original quantity and vice versa; where x is a natural
number. x + 1

1
For example, if price increases by 16.67% or , then in order to keep the expenditure unchanged, the
6
1
quantity has to be decreased by or 14.28% of its original value.
7
1
• For the same expenditure, if increase in the quantity purchased is of the original quantity, then
x
1
decrease in price is equal to of the original price and vice versa; where x is a natural number.
x+1
1
For example, if for the same expenditure, increase in quantity purchased is 20% or of the original
5
1
quantity, then decrease in price is equal to 16.67% or of original price.
6
Example 3

In a class having 60% girls, 40% of the students qualified in a test. If 50% of the girls qualified, find
the number of boys who did not qualify in the test as a percentage of the total strength of the class?
(A) 10%
(B) 30%
(C) 45%
(D) 55%
(E) 90%
Solution

Therefore, as a percentage of the total strength of the class, 30% of the boys have not qualified. Let the
total number of students in the class be 100. Then the number of girls = 60 and number of boys = 40.
Total number of students who qualified = 40% of 100 = 40. Of those 40 students, 30 are girls (since
50% of the girls have qualified and 50% of 60 = 30). So, 10 boys have qualified, which means that 30
boys have not qualified.
The correct answer is B.

41

Book 1.indb 41 30/04/2019 4:46:22 PM


NMAT by GMAC™ Official Guide 2019

Interest
Interest is an additional amount that a person gets against investment of capital. Interest earnings can be of
two types: Simple Interest (SI) and Compound Interest (CI). Let us look at and understand both of them.

Simple Interest (SI)


The basic formula for Simple interest is
P×R×T
SI =
100
Where P is the Principal, R is the rate % per annum and T is the time period (in years) of investment.

Compound Interest (CI)


Compound interest is calculated on the principal amount and also on the accumulated interest of previous
periods. This compounding effect can make a big difference to the total interest payable on a loan.
The following basic formula is used for calculating compound interest. The formula calculates the amount,
using which the interest can be calculated, that is
n
 R 
A = P1 + 
 100 
Where A is the amount, P is the Principal, R is the rate % applicable and n is the number of periods.
Compound Interest calculation if interest is payable more than once a year
If interest is paid before completion of the year, then amount
np
 R 
A = P1 +
 100p 
Where p = number of times interest is paid in 1 year and n is number of years. So, if interest is paid semi-
annually, quarterly or monthly, the value of p is 2, 4 and 12 respectively.

Important Learning: 

• In the first period, SI and CI are equal. In all the other periods after the first period, the CI is greater
than the SI.
• In simple interest, the total rate of interest applicable is the sum of all the respective rates applicable.
• In compound interest, the total rate of interest applicable is the successive effect of the respective rates
given.
• If an amount becomes N times itself in T years at SI, then the required rate of Interest
(N − 1)
R= × 100%
T
• Difference between compound interest and simple interest
2
a. For Two years, CI – SI =  R 
P 
 100 
 R 2  300 + R
b. For Three years, CI – SI = P  2 
×
 100  100

CI 200 + R
• Ratio of CI and SI for two years, =
SI 200
42

Book 1.indb 42 30/04/2019 4:46:22 PM


2.0  Quantitative Skills Review  2.2  Arithmetic

Example 4

The simple interest for 10 years is Rs. 6,000. The compound interest for 2 years is Rs. 1,400. Find the
rate of interest and the principal.
(A) 30%, 1,800
(B) 33.33%, 1,600
(C) 33.33%, 1,800
(D) 35%, 1,500
(E) 66.66%, 1,600
Solution
SI for 10 years is Rs. 6,000.
Therefore, SI for 1 year will be Rs. 600.
SI for 2 years will be Rs. 1,200.
CI for 2 years is Rs. 1,400.
Difference = Rs. 200. This is because of interest received on the first period’s interest.

Therefore,
R
200 = 600 ×
100
Therefore,
200
R= = 33.33%
6

Also, interest for the first period is Rs. 600, rate is 33.33% and time period is 1 year. Therefore,
P × 33.33 × 1
600 = ⇒ P = Rs. 1, 800
100

The correct answer is C.

Partnership
Partnership is defined as a legal agreement between two or more persons who agree to share profits or
losses incurred by a business entity. Each person in the partnership is called a partner.
If the partnership incurs losses, then partners also share losses; and vice versa.
If IR = Investment Ratio
And, TR = The ratio of time periods of different partners,
Then, Profit Sharing Ratio (PSR) or Loss Sharing Ratio (LSR) is calculated as the product of the
IR and TR.
Therefore,
PSR or LSR = IR × TR

43

Book 1.indb 43 30/04/2019 4:46:23 PM


NMAT by GMAC™ Official Guide 2019

Important Cases Related to Partnership:

1. If there are two partners who invest I1 and I2 for the same period of time, then the PSR or LSR
between partners 1 and 2 is calculated as:

Profit ( or loss ) of Partner 1 I1


=
Profit ( or loss ) of Partner 2 I 2

2. If there are two partners who invest I1 and I2 amounts for time periods t1 and t2, then the PSR or
LSR of partners 1 and 2 is calculated as:

Profit ( or loss ) of Partner 1 I1 t 1


=
Profit ( or loss ) of Partner 2 I 2 t 2
3. The share of each partner in a two-person (say Partner 1 and Partner 2) partnership, when they invest
I1 and I2 amounts for the same duration of time, is calculated as:
I1
Partner 1 = ´ P (or L)
I1 + I 2

I2
Partner 2 = × P (or L)
I1 + I 2
4. The share of partners in a three-person (say Partner 1, Partner 2 and Partner 3) partnership, when they
invest I1, I2 and I3 amounts for the same duration of time, is calculated as:

I1
Partner 1 = × P (or L)
I1 + I 2 + I 3

I2
Partner 2 = × P (or L)
I1 + I 2 + I 3

I3
Partner 3 = × P (or L)
I1 + I 2 + I 3

Example 5

A started a business with a capital of Rs. 5,000. Three months later, B joined with a capital of
Rs. 7,000. After another 3 months, A invested Rs. 1,000 more while B withdrew Rs. 2,000. Two
months later, C joined with a capital of Rs. 5,000. In what ratio should the profits get divided at the
end of the year?
(A) 22:4:17
(B) 22:17:8
(C) 22:17:4
(D) 66:51:20
(E) 66:17:20

44

Book 1.indb 44 30/04/2019 4:46:23 PM


2.0  Quantitative Skills Review  2.2  Arithmetic

Solution

As per the problem:


Equivalent contribution of A = 5,000 × 6 + 6,000 × 6 = 6,6000
Equivalent contribution of B = 7,000 × 3 + 5,000 × 6 = 51,000
Equivalent contribution of C = 5,000 × 4 = 20,000
Therefore, the required ratio = 66:51:20

The correct answer is D.

4 Ratio and Proportion


Introduction
Ratio is a comparison between two or more similar quantities having the same dimensions; therefore, ratio
happens to be a dimensionless quantity.
For example, when we mention that the speed of two persons A and B is in the ratio 2:3, we do not ask whether
the speed is in km/h or m/s, as the ratio is a simple comparison between two similar variables or values.
A ratio and fraction are synonymous yet different entities. When we say that a:b is 2:3, we are talking about
the ratio. Ratios are used to make comparisons, but when we need to find the individual contributions or
values, fractions are required for the same.
Therefore, if a:b is 4:3, we understand that for every value of 4 that a gets, b will get a value of 3, and so a
gets a value of 4 for every 7 that they get together. This is known as a fraction.
4 4 3
Fraction of a is , that is , and fraction of b is , that is 3 .
( 4 + 3) 7 ( 3 + 4) 7

Important Characteristics and Formulae Related to Ratios


Some of the basic properties of ratios are listed below:
1. If both the antecedent and the consequent are multiplied or divided by the same number (except 0),
the ratio will remain the same, i.e.
a ka a a/k
= or that, =
b kb b b/k
2. Duplicate ratio refers to the ratio of the squares of the antecedent and the consequent. Duplicate ratio
of x : y = x2 : y2.
3. Triplicate ratio refers to the ratio of the cubes of the antecedent and the consequent. Triplicate ratio of
x : y = x3 : y3.
4. Sub-duplicate ratio refers to the ratio of the square roots of the antecedent and the consequent.

Sub-duplicate ratio of x : y = x : y.

5. Sub-triplicate ratio refers to the ratio of the cube roots of the antecedent and the consequent.
Sub-triplicate ratio of x : y = 3 x : 3 y.

45

Book 1.indb 45 30/04/2019 4:46:24 PM


NMAT by GMAC™ Official Guide 2019

6. Inverse ratio is derived by interchanging the positions of the antecedent and the consequent. Inverse
ratio of x : y = y : x.
7. If the denominator of two ratios is same, then the ratio with larger numerator is greater than the ratio
with the smaller numerator.
a c e g
8. If we are given that = = = = k, then
b d f h
a+c+e+g
k=
b+d+f+h

a c a c a+c
9. If = , then = =
b d b d b+d

Important Learning:  Ratio between two quantities or variables is only a comparative measure. It does
not tell you anything about the actual values. For calculating actual values, you need to use fractions.

Example 1

The sum of the ages of the five members in a family is 124 years. If the ages of the children are in the
ratio 3:4:5 while the combined age of their parents is 76, find the age of the youngest child.
(A) 8
(B) 12
(C) 13
(D) 14
(E) 15
Solution

Combined age of the three children = 124 - 76 = 48 years


3
Age of the youngest child will be: × 48 = 12 years
12

The correct answer is B.

Proportion and variation


Proportion is directly connected to ratios. Basically, a proportion is a statement that tells us that two ratios
are equal. It can be written in two ways:
a c
1. Two equal fractions, = or
b d
2. Using a colon, a:b = c:d
When two ratios are equal, the cross-multiplication of the ratios is also equal. For example,
2 6
= ⇒ 2 × 27 = 6 × 9
9 27

46

Book 1.indb 46 30/04/2019 4:46:24 PM


2.0  Quantitative Skills Review  2.2  Arithmetic

Important Characteristics and Formulae Related to Proportions


Some of the basic properties of proportions are listed below:
a c
1. Since = , the product of extremes is equal to the product of means, i.e. ad = bc.
b d
2. If a proportion is such that a : x :: x : b, then x is called the mean proportional or the second
proportional of a and b.
3. If a proportion is such that a : b :: b : x, then x is called the third proportional to a and b.

a c a+b c+d
4. Componendo rule: If = , then =
b d b d
a c a–b c−d
5. Dividendo rule: If = , then =
b d b d
a c a+b c+d
6. Componendo and Dividendo (C and D) rule: If = , then =
b d a−b c−d
a c a a+c a−c c
7.
If = , then = = =
b d b b+d b−d d
a c b d
8. Invertendo rule: If = , then =
b d a c
a c a b
9. Alternendo rule: If = , then =
b d c d
Example 2

Three solutions having milk and water in the ratio 2:3, 3:1 and 4:5, respectively, were mixed in the
ratio 2:3:4. Find the ratio of milk to water in the resultant mixture.
(A) 869:751
(B) 219:341
(C) 420:519
(D) 531:622
(E) 640:729
Solution

Let the solutions added be 2, 3 and 4 L, respectively.

Then, the quantity of milk in the solution is as below:


2 3 4 4 9 16
= 2× +3× +4× = + +
5 4 9 5 4 9
144 + 405 + 320 869
= =
180 180

And, the quantity of water in the solution is as below:

47

Book 1.indb 47 30/04/2019 4:46:25 PM


NMAT by GMAC™ Official Guide 2019

869 1620 - 869 751


=9- = =
180 180 180
Therefore, ratio of milk to water = 869:751
The correct answer is A.

Direct proportionality
Y is said to be directly proportional to X if Y increases as X increases and Y decreases as X decreases. Here, Y
is called the dependent variable, while X is called the independent variable.
We can write the relation in the form Y = KX, where K is called the constant of proportionality.
Applications of direct proportionality
1.
Distance covered is directly proportional to speed if time of travel is constant.
2.
Amount of work done is directly proportional to the number of people if the number of days is
constant.

Inverse proportionality
Y is said to be inversely proportional to X if Y decreases as X increases and Y increases as X decreases.
K
We can write the relation in the form Y = , where K is called the constant of proportionality.
X
Applications of inverse proportionality

1.
Time taken is inversely proportional to speed if distance is constant.
2.
Number of days is inversely proportional to the number of people if the amount of work done is
constant.

Age-related problems
Problems based on ages are a simple application of the concept of ratios. In all problems of ages, we need to
follow the instructions given in the problem keeping the time shift in consideration.
Important Points
1.
We can take the unknown variable as the current age of the persons in the question or their age a
few years earlier or a few years later. The answer will be the same, provided we keep the time shift in
consideration.
2.
The difference between the ages of two persons will always be the same whether the calculation is
done today, a few years earlier or a few years later.
3.
If the average age of a family of n members is x today, after three years, the average age of the family
will be x + 3.
Example 3

Three years ago, the ratio of the ages of a father and a son was 6:1. After 3 years, the ratio will be
36:11. Find the present age of the son.
(A) 3 years
(B) 5 years
(C) 8 years
(D) 11 years
(E) 17 years

48

Book 1.indb 48 30/04/2019 4:46:25 PM


2.0  Quantitative Skills Review  2.2  Arithmetic

Solution

Let the ages of father and son three years ago be 6x and x. Today their ages will be 6x + 3 and x + 3
and after 3 years their ages will be 6x + 6 and x + 6.
Now,
6 x + 6 36
+
x + 6 11
x +1 6
⇒ =
x + 6 11
⇒ 5 x = 25
⇒x=5
Present age of the son will be x + 3, that is, 5 + 3 = 8 years
The correct answer is C.

5 Exponents
Introduction
In the term x2, x is called the base and 2 is called the exponent. An exponent basically refers to the number
of times the base is multiplied by itself. For example, in 53, 5 is multiplied by itself 3 times, that is, 5 × 5 × 5.
A number multiplied twice by itself is called the square of that number; so, 42 is 4 squared.
A number multiplied thrice by itself is called the cube of that number; so, 43 is 4 cubed.

Rules of exponents
Exponent questions on the NMAT by GMAC™ will primarily test your knowledge of what rules to
follow when working with exponents.
Let’s look at these rules:
1. Multiplying Powers with the same Base: To multiply powers with the same base, keep the base and
add the exponents.
bm × bn = bm+n
For example: 35 × 38 = 3 5+8 = 313
2. Dividing Powers with the same Base: To divide powers with the same base, keep
the base and subtract the exponents.

bm
n
= bm-n
b
For example:
610
3
= 610- 3 = 67
6
3. Raising a Power to a Power: To raise a power to a power, keep the base and multiply the exponents.
(bm)n = bmn

49

Book 1.indb 49 30/04/2019 4:46:26 PM


NMAT by GMAC™ Official Guide 2019

For example:
(95)4 = 95 . 4 = 920

4. Power of a Product Property: To simplify a power of a product, find the power of each factor and
multiply.
(ab)m = am . bm

For example: (2 . 5)5 = 25 . 55


5. Division of Powers Rule: When you are dividing two powers with the same base, subtract the second
exponent from the first to give you the exponent of the answer.
(am ÷ an = a(m–n))

Important Learning:  Any Number Raised to the Power 0 is 1


For any number a ≠ 0, a0 = 1.

6. Negative Exponents Rule: There are a few special exponent properties that deal with exponents that
are not positive. Following are the rules of negative exponents:
1
a- m =
am
1
Rules of Negative Exponents: -m
= am
a
−m
 a bm
  =
 b am

7. A non-zero number raised to the power of zero is equal to 1


20 = 1

8. An exponential expression with base 0 and a positive exponent yields 0, regardless of the exponent.
012 = 0

9. An exponential expression with base 1 yields 1, regardless of the exponent


112 = 1

10. An exponential expression with base −1 yields 1 when the exponent is even and −1 when the exponent
is odd
(−1)15 = −1 and (−1)16 = 1

11. If the exponent is a fraction, the numerator reflects what power to raise the base to and the
denominator reflects which root to take
4 2/3 = Cube root(4)2 = 3 4 2

50

Book 1.indb 50 30/04/2019 4:46:26 PM


2.0  Quantitative Skills Review  2.2  Arithmetic

Roots/Radicals
Radical is the opposite of an exponent (in a sense). Radical is basically another name for a root. For
example, 25 means what number (or numbers), when multiplied by itself twice, will yield 25? The answer
is, of course, 5.
Perfect square roots will yield an integer. For example, 25 = 5. Imperfect square roots do not yield an

integer. 30 is not an integer, but it is between 25 and 36 , or between 5 and 6.

Simplifying roots
In the case of positive terms, roots can be combined or split apart if the operation between the terms is
multiplication or division.
4×9 = 4 × 9

Note:  If the operation between the terms is addition or subtraction, you cannot separate or combine the
roots! 4 + 9 does not equal 4 + 9.

Important Learning: 32 × 34 = 36. But, 32 + 34 ≠ 36



Powers of 10
The exponent of a power of 10 simply tells us the number of zeroes that number would contain if it were
fully written.
107 = 1,00,00,000 (seven zeros)
When multiplying a number by a power of 10, move the decimal point to the right the same number of
places as that power.
0.00035 × 104 = 3.5
When dividing a number by a power of 10, move the decimal point to the left the same number of places as
that power.
0.35 ÷ 103 = 0.00035
Fractional Exponents
Fractional exponents are related to roots or radicals.
If n is a positive integer, then al/n is the nth root of a.
If a is positive, it is the positive number b such that bn = a.
If a is negative, then:
1. If n is odd, a1/n is the negative number b such that bn = a.
2. If n is even, a1/n is undefined. al/n is also written n
a.

51

Book 1.indb 51 30/04/2019 4:46:27 PM


NMAT by GMAC™ Official Guide 2019

6 Profit, Loss and Discounts


Introduction
Profit is an additional amount that a customer pays in return for buying an item that is owned by or sold by
someone else.
Profit = SP − CP
where SP is the selling price, that is, the price at which the item is sold, and CP is the cost price, that is, the
price at which it was originally manufactured or purchased by the seller.
When profit is expressed as a percentage of CP, it is known as profit %. Therefore,

(SP - CP)
Profit % = × 100
CP
Sometimes, the product is sold at a price lower than the CP.  This is called loss.
Loss can be written either as (CP − SP), in which case it has a positive sign or (SP − CP), in which case it
has a negative sign. So, loss = SP − CP.
(SP - CP)
Loss % = × 100
CP

Important Learning:  Profit or loss % is always expressed as a percentage of the cost price.

Example 1

50 kg of a product is sold and the profit generated is equal to the cost price of 20 kg of the product.
Find the profit percentage made.
(A) 20%
(B) 25%
(C) 35%
(D) 40%
(E) 45%
Solution
As per the problem, we have:
SP of 50 kg - CP of 50 kg = CP of 20 kg
Or, CP of 70 kg = SP of 50 kg

70 - 50
Therefore, × 100 = 40%
50

The correct answer is D.

52

Book 1.indb 52 30/04/2019 4:46:27 PM


2.0  Quantitative Skills Review  2.2  Arithmetic

False Weights
If an item is claimed to be sold at cost price using false weights, then the overall percentage profit is given
by

æ Claimed weight of item ö


Percentage Profit = ç - 1 ÷ ´ 100
è Actual weight of item ø
Example 2

A dishonest dealer claims to sell his good at cost price but uses a false weight, which reads 1000 gm
for 800 gm. What is his net profit percentage?
(A) 20%
(B) 25%
(C) 35%
(D) 40%
(E) 45%
Solution
1000 ö 200
Required percentage = æç - 1÷ ´ 100 = ´ 100 = 25%
è 800 ø 800
The correct answer is B.

Discount
A discount is a reduction in the marked (or list) price of an article. “25% discount” means a reduction of
25% in the marked price of an article. For example, if the marked price of an article is ` 400, it is sold for
` 300, i.e. ` 100 less than the marked price.
Let us define the following:
Marked Price (or List price)

The marked price (M.P.) of an article is the price at which the article is listed for sale. Since this price is
written (marked) on the article, so it is called the marked price.
Net Selling Price (S.P.)

In case of discount selling, the price of the article obtained by subtracting discount from the marked price is
called the Net Selling price or Selling price (S.P.).
Discount = Marked Price − Selling Price
Discount
Discount Percentage = × 100
Marked Price
Successive Discounts
When a discount of a% is followed by another discount of b%, then
ab
Total discount = a + b -
100

53

Book 1.indb 53 30/04/2019 4:46:28 PM


NMAT by GMAC™ Official Guide 2019

Important Points and Formulae


1. While the mark-up is always calculated as a percentage of the cost price, discount % is always
calculated as a percentage of the marked price.
2. If two items are sold for Rs. x each, the first one at a profit of P% and the other at a loss of P%, then
 P2 
the overall loss will be =  %.
 100 
3. When the CP and SP are either both increased or both decreased by the same percentage, there is no
change in the existing profit % or loss %.
 AB 
4. If two discounts are A% and B%, then the single equivalent discount =  A + B − %
 100 
5. If a person wants to make a profit of A% after giving a discount of B%, then the Marked Price,
100 + A
MP = CP ×
100 − B

7 Alligations and Mixtures


Introduction
In this chapter, we will cover the following two important topics and a variety of problems based on them:
1.
Alligation
2.
Replacement of part of a solution

Alligation
Alligation is the mathematical process of finding out the resultant value when we mix two or more things
having a common quality or property. This common quality or property of the constituents being mixed is
called ‘VALUE’.
Mathematically, there are three ways to look at an alligation problem and these are as follows:
1. Alligation is the process of weighted average presented in a different way, and therefore whatever is
being discussed related to an alligation problem can also be calculated using the weighted average
rule.
Weighted average rule:
Weighted average of x = w1x1 + w2x2 +…+ wnxn
Where,
w = Relative weight
x = Value
2. If two things are being mixed, one of them is lower in VALUE. This constituent is called ‘CHEAP’.
The other one, which is higher in VALUE, is called ‘DEAR’. The resultant obtained after the mixing
is called the ‘MEAN VALUE’.


Rule of Alligation
Amount or quantity of CHEAP = Value of DEAR - MEAN VALUE
Amount or quantity of DEAR MEAN VALUE - Value of CHEAP

54

Book 1.indb 54 30/04/2019 4:46:28 PM


2.0  Quantitative Skills Review  2.2  Arithmetic

3. Write down all the given information in a structured form.


We will consider an example to illustrate all the three ways:
Example 1


4 L of a 60% milk solution is mixed with 6 L of a 40% milk solution. Find the percentage of milk in
the resultant mixture.
(A) 40%
(B) 42%
(C) 44%
(D) 46%
(E) 48%
Solution

Using weighted average:


4 × 60% + 6 × 40% = 10 × y %
240 + 240 = 10 y
10 y = 480 ⇒ y = 48%

The correct answer is E.


Using the rule of alligation:  We need to understand that in order to use the rule of alligation, we need to
first identify the ‘CHEAP’ and ‘DEAR’ solution. In this example, the constituent having 60% milk will
be higher in quality and, therefore, called as DEAR. The quantity of DEAR is 4 L, while the quantity of
CHEAP is 6 L. So,
6 60 − x
=
4 x − 40
3 60 − x
=
2 x − 40
3x − 120 = 120 − 2 x
5x = 240 ⇒ x = 48%
Using structured form:  It has been found that students either spend too much time or make mistakes in
identifying ‘CHEAP’ and ‘DEAR’, and therefore, a third way to solve the problem is discussed as follows.
Start by quickly drawing a triangle of arrows representing the stated problem and attach quantities and
percentages as shown below.
4L 6L
60% M 40% M

x% M
On the left-hand side, take the ratio in which the two constituents are being added, that is, 4 L and 6 L.
After that, follow the arrow in the same direction and the right-hand side of the equation will be the
differences taken, as shown

55

Book 1.indb 55 30/04/2019 4:46:29 PM


NMAT by GMAC™ Official Guide 2019

4L 40 − x
=
6L x − 60
2 40 − x
=
3 x − 60
2x − 120 = 120 − 3 x
5x = 240 ⇒ x = 48%

A working like the one shown above may seem to be not directly aligned to the concept, but it helps a
student in calculating a problem easily. We will refer to this method as the structured form for ease of
understanding.

Important Learning:  If two things are being added, it is easier to form the structure and use the rule
of alligation to solve the problem. However, if more than two things are being added, it is better and
easier to use the concept of weighted average to solve the problem.

8 Time, Speed and Distance


Introduction
The basic concept of time, speed and distance is the relation between the three variables. The speed of a
body is the distance covered by the body per unit time, that is,
Distance covered
Speed =
Time taken

Most of the questions from this topic on the NMAT by GMAC™ will require you to work with some
form of the above equation.

Basic concepts
The basic concepts that form the fundamentals of time, speed and distance are:
1. In any problem, one has to ensure that speed, distance and time are in the same dimensions. The
popular dimensions used for speed are km/h and m/s. If required, one can also convert one dimension
into the other.
5
• To convert km/h into m/s, multiply km/h by and to convert m/s to km/h, multiply m/s
18
18
by .
5
2. Distance covered is directly proportional to the speed if time of travel is constant.
d1 d 2
For constant time of travel, =
s1 s 2

3. Time taken is inversely proportional to the speed if distance is constant.


For constant distance, s1 × t1 = s2 × t2
Total distance travelled
4. Average speed =
Total time

56

Book 1.indb 56 30/04/2019 4:46:29 PM


2.0  Quantitative Skills Review  2.2  Arithmetic

Important Learning:  When the total time of travel is divided into parts and the speed is different for
each part, the average speed can be expressed as the weighted average of the individual speeds.

Example 1

A person travels from his home to his office every day covering the same distance. If he has to
decrease the time taken today by 6.66%, by what percentage should the speed be increased?
(A) 7.14%
(B) 8.24%
(C) 9.33%
(D) 10.22%
(E) 11.34%
Solution

1
We know that 6.66% is the percentage equivalent of .
15
For constant distance, speed and time taken are inversely proportional.

Initial Final
Time 15 14
Speed 14 15

15 - 14
Percentage change in speed = × 100 = 7.14%
14
The correct answer is A.

5. Relative speed: Relative speed is defined as the speed of a body with respect to another body. The two
possible cases of relative motion are:
• Same direction: If two bodies are moving in the same direction, the relative speed is the
difference in their speeds and is always expressed as a positive value. Therefore, if two bodies are
moving at x m/s and y m/s, then
Relative speed = |x − y| m/s

The use of modulus sign above (| | pipe) indicates that the relative speed will always be positive,
irrespective of the values of x and y.
• Opposite direction: If two bodies are moving in the opposite direction, the relative speed is the
sum of their speeds. Therefore, if two bodies are moving in the opposite direction at x m/s and
y m/s, then
Relative speed = (x + y) m/s

57

Book 1.indb 57 30/04/2019 4:46:30 PM


NMAT by GMAC™ Official Guide 2019

Problems related to trains


Questions based on trains are commonly asked on the NMAT by GMAC™, and therefore, it is
important to be familiar with them.
Consider that a train of length l is travelling at a speed of u m/s. Then, it is important to understand the
following:
1. Time taken by the train to cross a man standing on a platform
 l
t= s
 u

2. Time taken by the train to cross a man moving in the same direction at v m/s:
 l 
t= s
 u - v 

3. Time taken by the train to cross a man moving in the opposite direction at v m/s:
 l 
t= s
 u + v 

4. Time taken by the train to cross a platform of length P:


 l + P
t= s
 u 
5. Time taken by two trains of lengths l1 and l2 and speeds u and v m/s to cross each other while
travelling in the opposite direction:
l +l 
t= 1 2s
 u+v

Example 2

Two trains start from A and B and move towards B and A, respectively. The train from A starts at 6
a.m. and reaches B at 2 p.m., whereas the train from B starts at 8 a.m. and reaches A at 2 p.m. In how
much time after 8 a.m. will the two trains meet?

8
(A) 1 h
9
15
(B) h
7

(C) 2 1 h
6

(D) 2 4 h
7

(E) 3 1 h
6

58

Book 1.indb 58 30/04/2019 4:46:30 PM


2.0  Quantitative Skills Review  2.2  Arithmetic

Solution

In this example, the starting time for the two trains is different. Also, the time taken for travelling the
same distance is different, which in turn would mean that their speeds are different.
Train from A starts at 6 a.m. and reaches B at 2 p.m., that is, a total travel time of 8 h.
Train from B starts at 8 a.m. and reaches A at 2 p.m., that is, a total travel time of 6 h.
Let the total distance be 24 km, and so the speed of the two trains will be 3 km/h and 4 km/h,
respectively.
First, we need to make their reference starting time same, that is, we will calculate the distance
travelled by the train from A for 2 h (that is from 6 a.m. to 8 a.m.).
Distance covered by the train from A in the first 2 h = 2 × 3 = 6 km
Distance between the two trains at 8 a.m. = 24 - 6 = 18 km
Relative speed = 3 + 4 = 7 km/h
4
Time taken to meet after 8 a.m. = 18 = 2 7 h
7
The correct answer D.

Example 3

Two trains start from A and B and travel towards B and A, respectively. Their speeds are 80 km/h and
95 km/h, respectively. When they meet, it is found that one train has travelled 165 km more than the
other. What is the distance of A to B?
(A) 1,650 km
(B) 1,700 km
(C) 1,825 km
(D) 1,890 km
(E) 1,925 km
Solution
Let both the trains travel for t h each before meeting.
Distance covered by train A = 80t and distance covered by train B = 95t
Now, 95 t - 80 t = 165

15 t = 165 ⇒ t = 11 h
that is, they have both travelled for 11 h before meeting.
Distance AB = Distance covered by A + Distance covered by B
Distance AB = 80 × 11 + 95 × 11 = 1,925 km
The correct answer is E.

Problems on boats and streams


The concept of boats and streams is exactly opposite to the concept of relative speed.
Let u m/s be the speed of the boat in still water and v m/s be the speed of the river/stream.

59

Book 1.indb 59 30/04/2019 4:46:31 PM


NMAT by GMAC™ Official Guide 2019

We will talk about two situations here:


1. Speed of the boat against the direction of the stream: Speed upstream = (u − v) m/s
2. Speed of the boat in the direction of the stream: Speed downstream = (u + v) m/s
Speed downstream + Speed upstream = 2u
Therefore,
Speed downstream + Speed upstream
u=
2
Speed downstream - Speed upstream
v=
2

Example 4

A man can row at 5 km/h in still water. If the speed of the stream is 1 km/h and it takes 1 h to row to
a place and come back, how far is the place?
(A) 2.4 km
(B) 2.5 km
(C) 3 km
(D) 3.6 km
(E) 4.2 km
Solution
Let the place be d kilometres away. Then
d d
+ =1
6 4
(2d + 3d)
⇒ =1
12
⇒ d = 2.4 km

The correct answer is A.

Problems on circular motion


1.
In case of circular motion, if two people are travelling in the same direction, they will meet for the first
time after the start when the faster person covers one more complete round than the slower person.
2.
In case of circular motion, if two people are travelling in the opposite direction, they will meet for the
first time after the start when they would have together covered one complete round.
3.
In case of both the above mentioned points, the two people will again meet at the starting point after
a time, which is equal to the LCM of the time taken by each to complete one round.

Example 5

The length of a circular track is 1200 metres. Two people are running around it at 100 m/min and 200
m/min respectively in the same direction. When will both meet at the starting point for the first time?
(A) After 10 minutes
(B) After 12 minutes
(C) After 14 minutes
(D) After 16 minutes
(E) After 18 minutes

60

Book 1.indb 60 30/04/2019 4:46:31 PM


2.0  Quantitative Skills Review  2.2  Arithmetic

Solution
1200
Time taken by the first person for one complete round = = 12 minutes
100

Time taken by the second person for one complete round = 1200 = 6 minutes
200
The time of their meeting at the starting point for the first time will be the
LCM of 12 and 6 = 12 minutes
So, after 12 minutes these people will be together at the starting point.
The correct answer is B.

Races and Games


A race is a competition in which the at least two competitors cover a given distance in the least time. The
point from where a race starts is called the starting point, the winning point or goal is the point where a
race finishes and the gaps between these two points is called distance.
Important terms used in the questions from Races and Games:

(i) ‘P gives Q a start of x metres’: This statement implies that, while P starts the race from starting point,
whereas, Q starts x metres ahead of P. To cover a race of 100 metres in this case, P will have to cover
100 metres while Q will have to cover only (100 - x).

(ii) ‘P beats Q by x m’: This statement implies that in the same time, while P reached the winning point,
whereas, Q is behind P by x m. To cover a race of 100 metres in this case, P has covered 100 metres
while Q has covered only (100 - x )

(iii) P can give Q a start of t minutes: This statement implies that P will start ‘t’ minutes after Q starts
from the starting point.

(iv) P beats Q x metres and t minutes: This statement implies that P and Q start from the starting point
at the same instant, but while P reaches the finishing point, Q is behind by x metres, and, Q takes ‘t’
minutes compared to P to complete the race. So, Q covers remaining x metres in extra ‘t’ minutes. This
gives the speed of Q as x/t.

(v) Dead Heat: A dead heat situation is when all participants reach the finishing point at the same instant
of time.

Example 6

Ratio of speed’s of Salim and Sanjeev is 3:4. Both participates in a race of 500 m. Salim has a start of
140 m. Then, Salim wins by how many metres?
(A) 18 m
(B) 20 m
(C) 22 m
(D) 24 m
(E) 26 m

61

Book 1.indb 61 30/04/2019 4:46:31 PM


NMAT by GMAC™ Official Guide 2019

Solution

To win the race, total distance covered by Salim = 500 m – 140 m = 360 m
While Salim covers 3 m, Sanjeev covers 4 m.
While Salim covers 360 m, Sanjeev covers 480 m.
Therefore, when Salim reaches the winning post, Sanjeev covers 480 m.
Hence, Salim wins by 20 m.
The correct answer is B.

9 Time and Work


Introduction
Time and work problems are important because there is a certain relationship between the number of
persons doing the work, number of days or time taken by them to complete the work and the amount of
work that is done.
The problems of time and work can primarily be divided into two types. The first type is the problem where
individuals work with different efficiencies either alone or in combination to complete a task. The second
type is where group efficiencies are involved. Such types of problems are also known as problems that use
the chain rule.
The NMAT by GMAC™ will test you on both of these types of problems.

Problems involving individual efficiencies


In such questions, the rates at which some individuals complete a work alone is given and you are required
to calculate the rate at which they can complete the work together (or vice versa). The basic formula for
solving such problems is
1 1 1
+ =
a b c

where a and b are the time it takes the two individuals to complete a job, while working alone and c is
the number of hours it takes them to complete the job working together. Let us look at an illustration to
understand this concept better.
Example 1

A can do a work in 20 days. B can do the same work in 30 days. In how many days can A and B do
the work together?
(A) 8 days
(B) 10 days
(C) 12 days
(D) 14 days
(E) 16 days

62

Book 1.indb 62 30/04/2019 4:46:31 PM


2.0  Quantitative Skills Review  2.2  Arithmetic

Solution

Unitary method:  We have been solving such problems using the unitary method.
1
A can do a work in 20 days. Therefore, in 1 day, A will be able to do of the work.
20
1
B can do the same work in 30 days. Therefore, in 1 day, B will be able to do of the work.
30
 1 1 5
Both of them together can do  +  of the work in 1 day = of the work in 1 day.
20 30 60
60
Therefore, the entire work will be completed in days, that is, 12 days.
5

The correct answer is C.

LCM method 

An easier method to do such problems is to understand and use a method called the LCM method.
Let the total work be 60 units, where a unit is a simple measurement of work.
Total work = 60 units
A can to do 60 units in 20 days, that is, 3 units per day.
B can to do 60 units in 30 days, that is, 2 units per day.
Together, they are able to do 3 + 2, that is, 5 units per day.
60
Therefore, they will finish the work in , that is, 12 days.
5
With a little practice, the LCM method can be effectively used to solve a problem.

Important Learning:  While using the LCM method, it is not necessary to use the LCM of the
individual time taken to do the problem. One can take any convenient value and solve the problem.

Pipes and cisterns


Problems on pipes and cisterns are common and popular applications of work-related problems. The only
difference being that in the case of pipes and cisterns there can be a contribution that is against the work to
be completed. In such a situation, the particular contribution(s) needs to be taken with a negative sign.
Basic Concepts
1
If inlet pipe fills the cistern in x hours and outlet pipe empties in y hours, then:
1 x
1 x
• Part of cistern filled in one hour when only inlet pipe is opened is .
x 1
1
• Part of cistern emptied in one hour when only outlet pipe is opened
1
is . y
y
y 1 1
• When both inlet and outlet pipes are opened, part of cistern filled or emptied
 1 1is ± -  .
± -   x y
 1 1
±  -  x y 
Rules that give shortcut formulae for question related to pipe and cistern:
x y
1.
Working alone, if two pipes can fill or empty the cisterns in x hours and y hours respectively, and then
 xy 
working together they will fill or empty the entire cistern in   hours .
 x+y
63

Book 1.indb 63 30/04/2019 4:46:32 PM


NMAT by GMAC™ Official Guide 2019

2.
Working alone, if three pipes can fill or empty the cisterns in x hours, y hours and z hours respectively,
æ xyz ö
and then working together they will fill or empty the entire cistern in ç ÷ hours .
è xy + yz + zx ø
3.
Working alone, if one pipe can fill a cistern in x hours and other can empty the cistern in y hours
 xy 
respectively, and then working together they will fill or empty the entire cistern in ±   hours .
 x−y
4.
If two pipes can fill a cistern in x h ours and y hours respectively and third can empty in z hours, then
 xyz 
working together they can fill the cistern in  hours .
 yz + xz − xy 

Example 2

Pipe 1 can normally fill a tank alone in 40 days but takes 20 days more due to a leak. In how much
time can the leak alone empty half the tank?
(A) 25
(B) 35
(C) 55
(D) 60
(E) 70
Solution
Let the capacity of the tank be 120 units.
Therefore, pipe 1 can fill the tank in 40 days filling 3 units/day.
And pipe 1 due to leak can fill the tank in 60 days filling 2 units/day.
That is, the leak can empty 1 unit/day.
Therefore, half the tank, that is, 60 units, will get emptied in 60 days.
The correct answer is D.

Example 3

A pipe can fill a tank alone in 12 h while a leak can empty it in 16 h. If the efficiency of the pipe is
increased by 25% while that of the leak is halved, find the time taken to fill 75% of the tank.

(A) 36 h
7

(B) 48 h
7

(C) 60 h
7

(D) 72 h
7

(E) 83 h
7

64

Book 1.indb 64 30/04/2019 4:46:33 PM


2.0  Quantitative Skills Review  2.2  Arithmetic

Solution

Let the total work be 48 units.


Therefore, pipe can fill the tank in 12 hours filling 4 units per hour.
And, the leak can empty the tank in 16 hours emptying 3 units per hour.
New efficiency of the pipe = 5 units
New efficiency of the leak = 1.5 units
Effective efficiency = 5 - 1.5 = 3.5 units

75% of the tank has to be filled = 3 of 48 = 36 units


4
Time required = 36 = 72 h.
3.5 7
The correct answer is D.

Problems involving group efficiencies


We will now look at problems where people with the same efficiencies are working in groups. Let us try to
understand this with the help of an example.
Example 4
A and B can do a work in 20 and 25 days, respectively. With the help of C and D, they finish the
same work in 5 days. If the efficiency of C is half that of A, find the total time taken by D to finish the
work alone.

(A) 100
17
(B) 12
(C) 200
17
(D) 15
(E) 20
Solution

Let the total work be 100 units.


A: 20 days 5 units/day
B: 25 days 4 units/day
A + B + C + D: 5 days 20 units/day
It means C and D can do 11 units per day. Since the efficiency of C is half of A, C will be able to do
2.5 units per day. It means D would be doing the remaining 8.5 units per day.

Total time taken by D to finish the work alone = 100 = 200 days.
8.5 17
The correct answer is C.

Example 5

Three boys can do the same work as one woman. If a work is completed by 36 boys in 28 days working
9 h every day, how many women must be required to complete the same work in 7 days working 6 h
every day?

65

Book 1.indb 65 30/04/2019 4:46:33 PM


NMAT by GMAC™ Official Guide 2019

(A) 36 women
(B) 48 women
(C) 54 women
(D) 66 women
(E) 72 women
Solution

Given that 36 boys will be equivalent to 12 women.


12 women × 28 days × 9 h = y women × 7 days × 6 h
Therefore, y = 72 women

The correct answer is E.

10 Clocks
Introduction
The concept of clocks is the same as the concept of circular motion. Just like in circular motion, in case of
clocks, the hour hand and the minute hand are continuously racing against each other around a circle.

Concept of clocks
The hour hand travels 5 min or 30° in 1 h. Similarly, the minute hand travels 60 min or 360° in 1 h.
Therefore, in one hour, the minute hand moves 55 min or 330° more than the hour hand. This can also be
called the relative speed between the minute hand and the hour hand.

Concept of clocks

Actual Relative
Movement
Time (min) Movement
  Minute Hand Hour Hand  
60 60 min 5 min 55 min
60 360° 30° 330°

The information presented in the table above is the aspect on which all problems of clocks are based and is
to be used as per the problem type. Different types of problems based on clocks are illustrated as follows.

Type 1

Example 1

At what time between 4 pm and 5 pm will the two hands of a clock coincide?
(A) 19.81 min past 4 pm
(B) 21.81 min past 4 pm
(C) 23.81 min past 4 pm
(D) 25.81 min past 4 pm
(E) 26.81 min past 4 pm
66

Book 1.indb 66 30/04/2019 4:46:33 PM


2.0  Quantitative Skills Review  2.2  Arithmetic

Solution

Usually, the reaction to a problem like this will be 4.20 pm, but please remember that in case of clocks
the answer has to be accurate because all the options given will be close to 4.20 pm. Therefore, an
approximate answer will not be sufficient. We need to accurately determine the time in this case.
We will start our problem from 4 pm. At 4 pm, the minute hand is behind the hour hand by 20 min.
Therefore, to coincide, the minute hand has to take a lead of 20 min over the hour hand, or the minute
hand has to cover 20 min more or 20 min relative to the hour hand.
As per the concept given in table above, the minute hand is able to cover a lead of 55 min in 60 min
of actual time.
Therefore, 11 min more can be covered in 12 min of actual time. And so, 1 min more can be covered
12
in an actual time of min of actual time.
11
(20 × 12)  240 
Therefore, 20 min can be covered in or  = 21.81 min past 4 pm.
11  11 

The correct answer is B.

 o not commit this common error:  The time is not 4.21 pm and 81 s. It is 0.81 of a minute, that is,
D
0.81 × 60 = approximately 48 s.
Therefore, the time between 4 pm and 5 pm when the two hands of a clock coincide will be
approximately 4.21 pm and 48 s.

Type 2

Example 2

What is the angle between the two hands of a clock at 4.40 pm?
(A) 90°
(B) 100°
(C) 105°
(D) 110°
(E) 115°
Solution

In a problem like this, we will take a reference time at which the position of both the hands can
be exactly determined. This reference time should be as close to the given time as possible. In this
example, we will take the reference time as 4.30 pm.
Now at 4.30 pm (the time of reference), the angle between the two hands of the clock is 45° with the
minute hand ahead of the hour hand.
As the problem is about the angle between the two hands of the clock, we will use the relative speed
in degree terms.

67

Book 1.indb 67 30/04/2019 4:46:33 PM


NMAT by GMAC™ Official Guide 2019

The minute hand covers 360° in 60 min or 6°/min.


The hour hand covers 30° in 60 min or 0.5°/min.
Therefore, in every minute, the minute hand is able to cover 5.5° more than the hour hand.
At 4.30 pm, the minute hand is ahead by 45°. In the next 10 min, that is, up to 4.40 pm, it will further
extend this angle by 5.5 × 10, that is, 55°.
Therefore, the angle between the two hands at 4.40 pm = 45° + 55° = 100°.

The correct answer is B.

Type 3

Example 3

At what time between 3 pm and 4 pm will the angle between the two hands of a clock be equal to
50°?
(A) 7 and 3/11 min past 3 pm
(B) 25 and 5/11 min past 3 pm
(C) 27 and 2/11 min past 3 pm
(D) Both A and B
(E) None of the above
Solution

At 3 pm, the angle between the two hands of a clock is 90°. When the minute hand reduces this lead
by 40°, the angle will be 50°.
Also, when the minute hand covers the existing gap of 90° and takes a further lead of 50°, the angle
between the two hands will become equal to 50°.
Therefore, an example like the one above will have two possible answers (times) at which the angle
between the two hands becomes equal to 50°.

(a) The first position will be when the minute hand moves 40° more than the hour hand.
5.5° more will be covered by the minute hand in 1 min.
1 10  2 
1° more will be covered in = =   min.
5.5 55  11

( 40 × 2) 80 3
Therefore, 40° more will be covered in = min past 3 pm = 7 and min past 3 pm.
11 11 11

The second position will be when the minute hand moves 90° + 50° = 140° more than the hour
(b) 
hand.
(140 × 2) 280 5
140° more will be covered in = = 25 and min past 3 pm.
11 11 11
The correct answer is D.

68

Book 1.indb 68 30/04/2019 4:46:34 PM


2.0  Quantitative Skills Review  2.2  Arithmetic

Important Learning:  Will there always be two answers to an example like the one above? It will
depend on the angle asked in the question being more than the angle existing at the reference point or
less than the angle existing at the reference point.
If the angle asked in the question is less than the angle existing at the reference point, there will be two
answers; if the angle asked in the question is more than the angle existing at the reference point, there
will be only one position and therefore one answer.

Other important concepts of clocks


1.
The two hands of a clock coincide once every hour, but in 12 h they coincide only 11 times. This is
because of a single common position between 11 am and 1 pm. Between 11 am and 1 pm , the two
hands meet only once, and that is at 12 pm.
2.
The hands are at right angles twice in every hour, but in 12 h they are at right angles only 22 times.
This is because of one common position between 2 pm and 4 pm and another common position
between 8 pm and 10 pm. Between 2 pm and 4 pm there must be four right-angle positions. There are
only three right-angle positions because the right angle at 3 pm is common to and counted in both
2 pm to 3 pm and 3 pm to 4 pm. Similarly, the right-angle position at 9 pm is common to both 8 pm
to 9 pm and 9 pm to 10 pm.
3.
The hands point in the opposite direction once in every hour, but in 12 h they are opposite only 11
times. This is because the opposite position of 6 pm is common to and counted in both 5 pm to 6 pm
and 6 pm to 7 pm.
4.
The hands are said to be in the same straight line whenever they are coincident or opposite to each
other.
5.
If the two hands of a clock coincide once, after how much time will they again coincide? If they
coincide once, they will coincide again when the minute hand takes a lead of 60 min on the hour
hand.
As per the concept, the minute hand takes a lead of 55 min in 60 min of actual time.

 60  12
Therefore, to cover 1 min, it will require   or min of actual time.
 55  11

To cover 60 min more, it will require 60 ×  12  = 720 = 65 5 min of actual time.


 
11 11 11

5
Important Learning:  The two hands of every correct clock will coincide after every 65 min
11
of actual time—another reason for the two hands to coincide only 11 times in 12 h.
720 5
or 65 min is a standard value for every correct clock and therefore will be used in the problems
11 11
on incorrect clocks.

69

Book 1.indb 69 30/04/2019 4:46:35 PM


NMAT by GMAC™ Official Guide 2019

Concepts of incorrect clocks


An incorrect clock can either be a fast clock which gains time or a slow clock which loses time.

Fast clock
5
The two hands of a clock coincide every x minutes where x is less than 65 + min. This is an example of
11
a fast clock and therefore it will gain time.
[(65 + 5/11) - x]
Time gained per minute =
x
Using this equation, the time gained can be calculated per hour or per day as the case may be.

Slow Clock
5
The two hands of a clock coincide every x minutes where x is greater than 65 + min. This is an example
11
of a slow clock and therefore it will lose time.
[x - (65 + 5/11)]
Time lost per minute =
x
Some problems based on incorrect clocks are discussed as follows.

Type 4

Example 4

The two hands of an incorrect clock coincide after every 65 min. How much time does the clock gain
or lose in one day?
(A) 9.069 min
(B) 10.069 min
(C) 11.069 min
(D) 12.069 min
(E) 13.069 min
Solution

Time gained by the clock in one minute


[(720/11) - 65] 720 - 715
=
65 65 × 11

Time gained by the clock in one day


5
× 60 × 24 = 10.069 min gain in 1 day
65 × 11

The correct answer is B.

70

Book 1.indb 70 30/04/2019 4:46:35 PM


2.0  Quantitative Skills Review  2.2  Arithmetic

11 Calendars
Introduction
The topic of calendar includes concepts such as odd days and leap year and finding the day of the week for
a given date.

Concept of odd days


An ordinary year has 365 days, that is, 52 weeks and one odd day. This means, out of the 365 days in an
ordinary year, 364 days will get converted into 52 weeks and one day will remain. This one day is referred to
as 1 odd day.

Whenever we will have the case of certain number of days in our calculations, we will convert the days into
weeks by dividing the total number of days by 7 and the remainder will be the number of odd days.
How the day of the week for the same date shifts from one year to the next

The concept of odd days means that when we proceed from one year to the next and the year is ordinary,
that is, it has 365 days, the day will get shifted by one day.
As an example, if the 24th of May 2014 is a Saturday, then 24th of May 2015 will be a Sunday while 24th
of May 2013 would have been a Friday and so on.
A leap year has 366 days, that is, 52 weeks and two odd days. This means that when we proceed from one
year to the next and the year has the effect of leap (29th February being included), and the year has 366
days, the day would get shifted by two days.
As an example, if the 24th of May 2015 is a Sunday, then 24th of May 2016 would be a Tuesday, that is, a
shift of two days.

 o not commit this common error:  Normally, students make the mistake of only looking at the
D
years (from which year to which year) to decide whether the shift will be one day or two days. This is a
wrong approach. When calculating the above, we need to look at not only the years but also the date from
which we are moving to the other given date. If the 29th of February falls between the two dates, there
will be a shift of two days, otherwise there will be a shift of one day.

Definition of a leap year


Normally, we define a leap year by saying that a year if divisible by 4 is a leap year, and if not, then an
ordinary year. This definition is unfortunately not complete. The correct definition of a leap year is as
follows:
An end of the century year (the last year of a century, e.g. 1900, 2000, 2100, etc.) is a leap year only if
divisible by 400. For all the other years, check the divisibility by 4, and if the year is divisible by 4 it is said
to be a leap year and will have 366 days.
This would mean that the year 1900 in spite of being divisible by 4 was not a leap year and the year 2100
will also not be a leap year.

71

Book 1.indb 71 30/04/2019 4:46:35 PM


NMAT by GMAC™ Official Guide 2019

Important Learning:  Do you know why an end of the century year has to be divisible by 400 to
be a leap year? This is because the revolution of the Earth around the Sun, which is normally said
to be 365.25 days, which is what leads to an additional day being added in February, is actually
365.242 days.
Therefore, when we take 365.25 days in each year in our calculations, we are introducing an error of
approximately 0.008 days in our calendar every year.
This error of 0.008 days does not seem very significant when viewed alone, but when its effect is taken
for 400 consecutive years, it will introduce an error of 3 complete days in our calendar.
Therefore, it was decided that if the above mentioned error is to be corrected, then we need to add 3
days less in every 400 years, and so the 100th, 200th and 300th year are not leap years in spite of being
divisible by 4. This error correction has also meant that after every 400 years, our calendar repeats itself
for the next 400 years.

Counting odd days


1st January AD from where our calendar started was a Monday, and hence the reason for our week starting
on a Monday, and Saturday and Sunday being called as weekends.
Therefore, if we are calculating from 1st January AD and after converting into weeks, whenever we have 1
odd day left, it would be a Monday. If there are 2 odd days left, then the first one would be a Monday, the
second a Tuesday and so on. So, after converting into weeks,
First odd day = Monday
Second odd day = Tuesday
Third odd day = Wednesday
Fourth odd day = Thursday
Fifth odd day = Friday
Sixth odd day = Saturday
Seventh or zeroth odd day = Sunday

Concept of total odd days in 100, 200, 300 and 400 years
The total number of odd days form the basis of these calculations:
1.
If we take 100 consecutive years from 1st January AD, there will be 24 leap years (remember the
100th year will not be a leap year) and 76 ordinary years.
24 leap years = 24 × 2 = 48 odd days, that is, 42 days being converted into 6 weeks and 6 odd days.
76 ordinary years = 76 × 1 = 76 odd days, that is, 70 days being converted into 10 weeks and
6 odd days.
Total = 6 + 6 = 12 odd days, that is one week and 5 odd days.
So, 100 consecutive years from 1st January AD will give 5 odd days.
Similarly, 200 consecutive years from 1st January AD = 10 odd days, that is, 3 odd days.
2.
3.
300 consecutive years will be 15 odd days, that is, 1 odd day.
But, 400 consecutive years = 20 + 1, that is, 21 odd days, that is, 0 odd days. (This is because the 400th
4.
year will be a leap year and contribute 1 extra day.)

72

Book 1.indb 72 30/04/2019 4:46:35 PM


2.0  Quantitative Skills Review  2.2  Arithmetic

Also, any multiple of 400 consecutive years will always give 0 odd days. This is used along with the
other four concepts to calculate a day if a date is given.
5. Also, 100 consecutive years will have 5 odd days, that is, the last day of 100 years will be a Friday.
Similarly, the last day of the 200th, 300th and the 400th years will be Wednesday, Monday and
Sunday, respectively.
Any two years will have the same calendar if they are both of the same type (that is, both ordinary or
both leap) and the first days of both the years are the same.

Important Learning:  The last day of a century will definitely be one day out of Friday, Wednesday,
Monday or Sunday. Continuing with the same logic, the first day of a new century will be a Saturday,
Thursday, Tuesday or Monday.

Problem types based on calendars


Type 1

Example 1

What was the day on 24th May 2014?


(A) Tuesday
(B) Wednesday
(C) Thursday
(D) Friday
(E) Saturday
Solution

In such questions, it would always depend on whether we have a reference point or not. As we do not
have a reference point in this case, we will start our calculations from 1st January AD.
The first 2,000 years = 0 odd days
Next 13 years will have:
3 leap years × 2 odd days = 6 odd days
10 ordinary years × 1 odd day = 10 odd days = 3 odd days
For the year 2014: 
January: 31 days, 3 odd days
February: 28 days, 0 odd days
March: 31 days, 3 odd days
April: 30 days, 2 odd days
May: 24 days, 24 odd days = 3 odd days
Total number of odd days for the year 2014 = 3 + 0 + 3 + 2 + 3 = 11 odd days = 4 odd days
Total odd days = 6 + 3 + 4 = 13 odd days = 6 odd days
Thus, 24th May 2014 will be a Saturday.
The correct answer is E.

73

Book 1.indb 73 30/04/2019 4:46:36 PM


NMAT by GMAC™ Official Guide 2019

Example 2

If 31 March 2017 is a Saturday, find the day of the week on 1 January 2014.
(A) Wednesday
(B) Friday
(C) Thursday
(D) Monday
(E) Tuesday
Solution

The day of the week on 1 January 2014 can be determined as:


31 March 2017 is a Saturday.
31 March 2016 will be a Friday.
31 March 2015 will be a Wednesday.
31 March 2014 will be a Tuesday.
3 March 2014 (28 days before) will be a Tuesday.
28 February 2014 will be a Saturday.
31 January 2014 will be a Saturday.
3 January 2014 will be a Saturday.
So, 1 January 2014 will be a Thursday.
The correct answer is C.

74

Book 1.indb 74 30/04/2019 4:46:36 PM


Book 1.indb 75 30/04/2019 4:46:36 PM
2.5 Algebra

Book 1.indb 76 30/04/2019 4:46:36 PM


2.0  Quantitative Skills Review  2.5  Algebra

2.6 What is Measured?


The Algebra section will test you on topics such as equations and inequalities. You will be required to solve
linear equations with one unknown and two unknowns, calculate the roots of a quadratic equation and
work with the concept of absolute value.
Algebra is also tested indirectly in word problems as most of these questions will require you to make an
equation and solve it.
You will also be tested on the expression of common algebraic identities such as (a + b)2, (a - b)2 and so on.

2.7 Overall Test Taking Strategies


• As algebra is mostly formula-driven, you should be familiar with all the commonly used formulae.
• If you make any change to one side of an equation or inequality, remember to make that same change
to the other side as well.
• Remember to reverse the inequality sign when changing a positive number into a negative number or
vice versa.
• A lot of the algebra questions are best tackled through the answer choices. Instead of finding the
actual answer, it may be faster to back-solve from the answer choices, especially if you are able to
eliminate one or two choices at first glance.
The next few sections will provide you with in-depth strategies for approaching each topic.

77

Book 1.indb 77 30/04/2019 4:46:36 PM


NMAT by GMAC™ Official Guide 2019

1 Equations
Introduction
While arithmetic primarily deals with numbers, the basic building block of algebra is a variable which does
not have a fixed value.
A variable along with a constant forms the basis of algebra. A variable by itself or a constant by itself or a
variable in combination with a constant forms a term in algebra. For example, x3 will be a term, 8 is also a
term, 5x2 is also a term and so on.
When many terms combine together, they form an expression. For example, x3 − 5x2 + 11 is an expression
in variable x. Please note that an expression is different from an equation. While an expression does not
include the ‘=’ sign, an equation will always include an ‘=’ sign. Moreover, an expression always has a value,
while an equation has roots or a solution.

Basis of classification
Algebraic expressions can be classified in the following ways:
1. Number of terms: The first basis of classification of algebraic expressions is based on the number of
terms in the expression.
• An expression having a single term is called a monomial, for example, 5x2y. Please note that the
number of variables does not make any difference as long as the term is single.
• An expression having two terms is called a binomial. For example, 3x + 5.
• An expression having more than two terms is called a polynomial. For example, 5x + 2y - 6.

2. Degree of the expression: Before we understand this, we need to understand the definition of degree.
Degree is defined as the highest or maximum sum of the powers of all the variables in any term of the
expression.
For example,
3x3 + 2x2yz − 7y2 + 5yz - 15x + 17

• The degree of this expression will be 4 because in the term 2x2yz, the power of x = 2, power of
y = 1, power of z = 1. Hence the degree will be 2 + 1 + 1 = 4.
• An expression of degree 1 is called linear.
• An expression of degree 2 is called quadratic.
• An expression of degree 3 is called cubic and so on.

78

Book 1.indb 78 30/04/2019 4:46:36 PM


2.0  Quantitative Skills Review  2.5  Algebra

Important Learning: 

• (a + b)2 = a2 + 2ab + b2 = (a – b)2 + 4ab


• (a - b)2 = a2 – 2ab + b2 = (a + b)2 – 4ab
• a2 – b2 = (a + b)(a – b)
2 2
• a2 + b2 = (a + b) + (a - b)
2
• (a + b)2 – (a – b)2 = 4ab
• (a + b)3 = a3 + b3 + 3ab (a + b)
• (a – b)3 = a3 – b3 – 3ab (a – b)
• a3 + b3 = (a + b) (a2 – ab + b2)
• a3 – b3 = (a – b) (a2 + ab + b2)
(a + b) 2 – (a – b) 2
• = ab
4
• (a + b + c)2 = a2 + b2 + c2 + 2(ab + bc + ac)

Linear equations
As stated earlier, an equation of degree 1 is called a linear equation. In this type of equation, all the variables
are raised to the first power only (there are no squares, cubes, etc.). For example,
x + 10 = 25
In order to solve linear equations, we try to isolate the variable whose value we are trying to find by
bringing it on one side of the equation and taking all other values to the other side of the equation. So, in
the above equation
x + 10 - 25 = 0
x = 25 - 10 (isolating x by taking 10 to the other side)
x = 15 (the solution to the equation)

We can always verify whether we have got the correct answer by replacing the value of x (that we have
arrived at) in the original equation and checking if it holds true.

Important Learning:  To solve a linear equation, you just need to isolate the variable on one side and
take all the numbers on the other side. Remember to reverse the signs when changing sides.

Example 1

In colony A, there are 12 houses with an average of 4 members per house, while in colony B, there are
20 houses with an average of Y members per house. If the two colonies together have an average of
3.5 members per house, find Y.

79

Book 1.indb 79 30/04/2019 4:46:36 PM


NMAT by GMAC™ Official Guide 2019

(A) 3.2
(B) 3.6
(C) 4.8
(D) 5.4
(E) 6.2
Solution
It is given that the average number of members in the two colonies together is 3.5. Therefore,
12 × 4 + 20 × Y = 32 × 3.5
48 + 20 Y = 112
20 Y = 64
Y = 3.2 members per house
The correct answer is A.

Simultaneous Equations
In linear equations, we were working with one variable, namely x. In simultaneous equations, we will be
working with two variables, namely x and y.
Let us look at this equation
3x + 4y = 24
From this equation, can you find the values of x and y? Obviously not!
As a rule, if you want to find the numerical value for N variables, you will need N different equations. In
linear equations, we are trying to find the value of one variable, so a single equation is sufficient. However,
in the above equation, we are trying to find the values of two variables x and y, so we need two different
equations that we will combine and solve simultaneously.

Important Learning:  To find the numerical value for N variables, we need N number of equations.

Let us say we have the following two equations given to us,


x + 6y = 9 (1)
3x + 4y = 24 (2)
We can isolate x in Eq. (1) as
x = 9 − 6y
and we can then substitute this value of x in Eq. (2)
3(9 - 6y) + 4y = 24 (3)
3
We can then solve Eq. (3) as a normal linear equation to get the value of y as . We can then substitute
this value of y in either Eq. (1) or Eq. (2) to find the value of x. 14
3
x+6× =9
14
54
⇒x=
7
80

Book 1.indb 80 30/04/2019 4:46:38 PM


2.0  Quantitative Skills Review  2.5  Algebra

Example 2

The price of two cups, seven pans and four saucers is Rs. 110 while of one cup and two saucers is
Rs. 20. Find the price of three pans.
(A) 30
(B) 40
(C) 60
(D) 70
(E) 80
Solution

As per the problem:


2x + 7y + 4z = 110
Also, 2x + 4z = 40
Therefore,
7y = 70 or y = 10
Price of three pans will be Rs. 30.
The correct answer is A.

Example 3

Two apples and five bananas cost Rs. 17, while three apples and four bananas cost Rs. 15. What is the
price of an apple?
(A) Rs. 1
(B) Rs. 1.50
(C) Rs. 2
(D) Rs. 2.50
(E) Rs. 3
Solution

Let the price of an apple be X.


And, the price of a banana be Y.
Now, as per the question, we have:
2 X + 5 Y = 17 (1)

And, 3 X + 4 Y = 15 (2)

Multiply Eq. (1) by 4 and Eq. (2) by 5, we get:


8 X + 20 Y = 68
15 X + 20 Y = 75
Solving, we get: X = Rs. 1
The correct answer is A.

81

Book 1.indb 81 30/04/2019 4:46:38 PM


NMAT by GMAC™ Official Guide 2019

Quadratic Equations
An equation of the form ax2 + bx + c = 0, where a, b and c are real and a ≠ 0, is called a quadratic equation.

How to solve a quadratic equation


The following two methods are used to solve a quadratic equation:
1. Method of factorisation: This is the most popular method of solving a quadratic equation. If ax2 +
bx + c = 0 is the equation, we divide b into two parts such that their sum is b and product is ac.
For example, consider the equation x2 + 5x + 6 = 0.
Now, 5 has to be divided into two numbers such that their product = 6 and their sum is 5.
Therefore,
x2 + 2x + 3x + 6 = 0
x(x + 2) + 3(x + 2) = 0
(x + 3)(x + 2) = 0
x = −3, −2

2. Shridharacharya’s method: This is the method through which we all learnt the solution of quadratic
equations for the first time. If ax2 + bx + c = 0 is the equation, then

- b ± ( b2 - 4ac)
x=
2a

where (b2 − 4ac) is known as D, the discriminant of the equation, which also decides the nature of the
roots of the quadratic equation, as follows:
• If D > 0, the roots are real and unequal.
• If D = 0, the roots are real and equal.
• If D < 0, the roots are imaginary.

Important Learning:  The solutions to a quadratic equation are technically called its roots.

Properties of quadratic equations


1. If ax2 + bx + c = 0 is the equation and the two roots are a and b, then
b
a + b = Sum of the roots = -
a
c
ab = Product of the roots =
a
2. If we know the roots, we can find the equation using
x2 - (sum of roots)x + product of roots = 0

Example 4

If the sum of the roots of an equation is 5 times the product of the roots, find the relation between
b and c. 3

82

Book 1.indb 82 30/04/2019 4:46:38 PM


2.0  Quantitative Skills Review  2.5  Algebra

(A) b = 5c
3

(B) b = 3c
5
(C) b = - 5c
3

(D) b = - 3c
5

(E) b = 2c
5

Solution

For a quadratic equation,


b c
Sum of the roots = - and product of the roots = .
a a
According to the problem,

b 5 c
- =  
a 3  a
-3ab = 5ac
5c
b=-
3

The correct answer is C.

Note: The NMAT by GMAC™ will not test any skills beyond quadratic equations.

2 Inequalities
Introduction
While equations tell us that two parts of an equation are equal, inequalities tell us that one part is bigger or
smaller than the other.
2x + 3 < 7
Inequalities basically give us an idea of the relative size of two values.

Solution of an inequality
The value(s) of the variable(s) which makes the inequality a true statement is called its solutions. The set of
all solutions of an inequality is called the solution set of the inequality. For example, x – 1 ≥ 0, has infinite
number of solutions as all real values greater than or equal to one make it a true statement. The inequality
x2 + 1 < 0 has no solution in R as no real value of x makes it a true statement.

How to solve an inequality


An inequality is solved, in the same way as you solve an equation, by isolating the variable on one side and
simplifying it. It is just that the sign used will not be the ‘=’ sign but the inequality sign, depending on the
relation between the two parts of the inequality.
83

Book 1.indb 83 30/04/2019 4:46:39 PM


NMAT by GMAC™ Official Guide 2019

1. > greater than


2. < less than
3. ≥ greater than or equal to
4. ≤ less than or equal to
So, the solution to the inequality 2x + 3 < 7 is x < 2.
However, there is one major difference between equations and inequalities that needs to be kept in mind—
if the inequality is multiplied or divided by a negative number, the sign of the inequality is reversed.
For example, if the inequality -5x > 3 is multiplied by -1, the resulting inequality is
5x < -3

Important Learning:  If an inequality is multiplied or divided by a negative number, the sign of the
inequality gets reversed.

Inequality Rules

Rule 1: Equal numbers may be added to (or subtracted from) both sides of an equation.
Rule 2: Both sides of an equation may be multiplied (or divided) by the same non-zero number.
Two Important Results

(a) If a, b ∈ R and b ≠ 0, then


a
(i) ab > 0 or > 0 ⇒ a and b are of the same sign
b
a
(ii) ab < 0 or < 0 ⇒ a and b are of the Opposite sign
b
(b) If a is any positive real number, i.e., a > 0, then
(i) |x| < a ⇔ – a < x < a
|x| ≤ a ⇔ – a ≤ x ≤ a
(ii) |x| > a ⇔ x < – a or x > a
|x| ≥ a ⇔ x ≤ – a or x ≥ a

Example 1

Which of the following describes all possible solutions to the inequality |a + 4| < 7?
(A) a < 3
(B) a > -11
(C) 3 > a > -11
(D) -11 > a > 3
(E) a > 11 or a < -11
Solution

Note the absolute value sign in the original inequality. This basically means that the solution could
lie on either side of the number line. So, you will have to solve this inequality in two ways to get the
entire range of solutions for a.
84

Book 1.indb 84 30/04/2019 4:46:39 PM


2.0  Quantitative Skills Review  2.5  Algebra

a - 4 < 7  or  a + 4 > -7


a < 3  or  a > -11
So,

The correct answer is C.

3 Logarithm
Definition: Every positive real number N can be expressed in exponential form as
N = ax …(1) e.g. 49 = 72
Where ‘a’ is also a positive real different than unity and is called the base and ‘x’ is called the exponent.
We can write the relation (1) in logarithmic form as
logaN = x …(2)
Hence the two relations
a x = Nù
ú
and log a N = x ú
û
are identical where N > 0, a > 0, a ≠ 1
Hence logarithm of a number to some base is the exponent by which the base must be raised in order to get
that number. Logarithm of zero does not exist and logarithm of (–) ve reals are not defined

Important Learning:  Unless stated otherwise, we assume the base to be 10.

Important Formula
1. Log (ab) = log a + log b
2. Log (a/b) = log a – log b
3. Log (a)n = n log a
4. Logab = log b/ log a (to any base)
The logarithm of 1

Recall that any number raised to the power zero is 1: a0 = 1.


The logarithmic form of this is
loga 1 = 0

1
Important Learning:  log b a = .
log a b

85

Book 1.indb 85 30/04/2019 4:46:40 PM


2.8  Geometry and Mensuration

Book 1.indb 86 30/04/2019 4:46:40 PM


2.0  Quantitative Skills Review  2.8  Geometry and Mensuration

2.9 What is Measured?


Geometry consists of topics such as Lines, Angles, Triangles, Polygons, Circles and some Three-
Dimensional Figures.
You will be tested on your ability to visualise different shapes and identify patterns such as a triangle within
a square or a square within a circle.
You will also be tested on the application of common geometric theorems such as Pythagoras theorem,
Base angle theorem, Longest side theorem, etc.

2.10 Overall Test Taking Strategies


• Geometry is all about shapes, so the best way to handle geometry problems is to draw the figure and
then visually analyse the problem.
• Do not assume anything about a given figure that is not explicitly written or mentioned. Thus, even if
an angle looks like a right angle, do not assume it is so, unless explicitly stated.
• Remember the properties of special triangles such as 45-45-90 and 30-60-90 triangles.
• While attempting questions on circles, always try to calculate the radius first. Everything else will
follow from there.
• While working with squares and rectangles, try to break them up into triangles by making the
diagonals.
The next few sections will provide you with in-depth strategies for approaching each topic.

87

Book 1.indb 87 30/04/2019 4:46:40 PM


NMAT by GMAC™ Official Guide 2019

1 Lines and Angles


Introduction
Geometry is a branch of mathematics that deals with measurement of various parametres of geometric
figures. The concept of geometry is based, to a large extent, on the understanding of different shapes in
terms of the length of their sides, their degree measures, etc.

Lines
A line is a one-dimensional figure—it only has length but no width or thickness. It can be extended
indefinitely in both directions; thus, a line is infinite. A finite line, that is, a line having two end points, is
called a line segment. It is named after its end points, such as line AB in the figure below:

A B

A line segment

Important Learning:  The shortest distance between two points is always a straight line.

Ray
A part of a line with one end point is called a ray.

Ray

Angles
An angle (∠) is formed when two lines meet at a point. This meeting point is called the vertex. Angles are
measured in degrees (°). An angle of x° can be seen in the figure below.

X∞

Angle x

An acute angle measures between 0° and 90°. A right angle measures exactly 90°. An obtuse angle
measures between 90° and 180°. An angle of 180° is a straight line.

88

Book 1.indb 88 30/04/2019 4:46:41 PM


2.0  Quantitative Skills Review  2.8  Geometry and Mensuration

These angles can be seen in the figure below.

Obtuse Acute

Right Straight

Types of angles

The sum of measures of the angles on a straight line is always 180° as shown in the figure below.

x∞
y∞

Sum of measures of angles on a straight line

In the above figure, ∠x + ∠y = 180°


Similarly, the sum of measures of an angle around a point is always 360°, as shown in the figure below.

A
B

D
C

A + B + C + D = 360°

Sum of measures of angles around a point

Two angles are congruent if their measures are equal.


Supplementary angles add up to 180°.
Complementary angles add up to 90°.

89

Book 1.indb 89 30/04/2019 4:46:41 PM


NMAT by GMAC™ Official Guide 2019

Example 1

In the figure given what is the value of a + b?

a° b°
A B

(A) 30°
(B) 60°
(C) 90°
(D) 100°
(E) 180°
Solution

Line AB is a straight line, so ∠a + ∠b + 90° = 180°


or ∠a + ∠b = 90°
The correct answer is C.

Intersecting lines
When two lines intersect each other, the opposite angles have the same measure called vertical angles.

D
C

Angles formed by intersecting lines

In the figure above,


∠A = ∠B
∠C = ∠D
A line bisects an angle if it divides the angle into two equal parts.

90

Book 1.indb 90 30/04/2019 4:46:42 PM


2.0  Quantitative Skills Review  2.8  Geometry and Mensuration

In the figure below, line BD bisects angle ABC.


Thus, ∠ABD = ∠CBD
A
D

B C

Angle bisector

Perpendicular lines
When two lines intersect at a 90° angle, they are called perpendicular.
M

A N B

AB ⊥ MN

Perpendicular lines

In the figure above, line segment AB is perpendicular to line segment MN.

Parallel lines
Parallel lines are two lines that lie in the same plane but never intersect one another.
l1
l2
Parallel lines

Lines l1 and l2, in the figure above, are said to be parallel and are denoted as l1 || l2.
If two parallel lines intersect a third line (called a transversal), each of the parallel lines will intersect the
third line at the same angle, that is, all acute angles will be equal, all obtuse angles will be equal and any
acute angle will be supplementary to any obtuse angle.

91

Book 1.indb 91 30/04/2019 4:46:42 PM


NMAT by GMAC™ Official Guide 2019

Line P

H I Line Q

K J

O L
Line R
N M

Transversal P intersecting parallel lines Q and R

Thus, in the figure above,


1. ∠H = ∠J = ∠O = ∠M
2. ∠I = ∠K = ∠L = ∠N
3. ∠H + ∠I = ∠K + ∠J = ∠O + ∠L = ∠N + ∠M = 180°

2 Triangles
Introduction
A triangle is a closed figure with three straight sides and three angles. It is denoted by the symbol ∆. The
sum of interior angles of a triangle is always 180°. Also, the measure of each exterior angle of a triangle is
equal to the sum of the interior opposite angles.
A

°c

°a °b °d

B C D

Sum of angles of a triangle

Thus, in the figure above,


∠a + ∠b + ∠c = 180°

∠a + ∠c = ∠d

Example 1

In the figure given, if XW bisects ∠X, and if ∠Y = 80° and ∠Z = 60°, what is the measure of
∠YXW?

92

Book 1.indb 92 30/04/2019 4:46:43 PM


2.0  Quantitative Skills Review  2.8  Geometry and Mensuration

X Z

(A) 20°
(B) 30°
(C) 35°
(D) 40°
(E) 60°
Solution

∠X + ∠Y + ∠Z = 180°
or, ∠X + 80 + 60 = 180
or, ∠X = 40°
1
∠YXW = ∠X = 20°
2
The correct answer is A.

Types of triangles
1. Scalene triangle: None of the three sides are equal.
2. Isosceles triangle: Two of the sides (and angles) are equal.
3. Equilateral triangle: All three sides (and angles) are equal.
4. Right triangle: One of the angles is 90°.
5. Acute triangle: All three angles are less than 90°.
6. Obtuse triangle: Any one angle is greater than 90°.

Equilateral Isosceles Scalene Acute Right Obtuse

Types of triangles

93

Book 1.indb 93 30/04/2019 4:46:43 PM


NMAT by GMAC™ Official Guide 2019

Length of the sides of a triangle


The length of any one side of a triangle is less than the sum of lengths of the other two sides and greater
than the difference between the lengths of the other two sides.

Pythagoras theorem
Pythagoras theorem applies only to right triangles and states that in a right triangle the square of the
hypotenuse (the longest side) is equal to the sum of squares of the other two sides, as illustrated in figure
below.

c
a

a2 + b2 = c2

Pythagoras theorem

There are two special types of right triangles commonly tested on the NMAT by GMAC™ whose sides
always have the same ratio.
These are illustrated in the figure below.
1. 45-45-90 or Isosceles right-angled triangle
2. 30-60-90 triangle

45°
30°

√2x x 2x √3x

45° 90° 60° 90°


x x

Special triangles

Important Learning:  In 45-45-90 and 30-60-90 triangles, if you know the length of any one side, you
can calculate the length of the other two sides.

94

Book 1.indb 94 30/04/2019 4:46:44 PM


2.0  Quantitative Skills Review  2.8  Geometry and Mensuration

Congruency of Triangles:
In congruent triangles corresponding parts are equal and we write in short ‘CPCT’ for corresponding parts of
congruent triangles.
Criteria for Congruence of Triangles:
• SAS congruency Rule: Two triangles are congruent if two sides and the included angle of one
triangle are equal to the two sides and the included angle of the other triangle.
• ASA congruency Rule: Two triangles are congruent if two angles and the included side of one
triangle are equal to two angles and the included side of the other triangle.
• SSS congruency Rule: If three sides of one triangle are equal to the three sides of another triangle,
then the two triangles are congruent.
• RHS Congruence criterion: If under a correspondence, the hypotenuse and one side of a right-
angled triangle are respectively equal to the hypotenuse and one side of another right-angled triangle,
then the triangles are congruent.
Similarity of Triangles
Two triangles are similiar, if
(i) their corresponding angles are equal and
(ii) their corresponding sides are in the same ratio (or proportion)

Theorems:

1. If a line is drawn parallel to one side of a triangle to intersect the other two sides in distinct points, the
other two sides are divided in the same ratio
2. If a line divides any two sides of a triangle in the same ratio, then the line is parallel to the third side
Criteria for Similarity of Triangles
Two triangles are similar, if
(i) their corresponding angles are equal and
(ii) their corresponding sides are in the same ratio (or proportion).
That is, in ∆ ABC and ∆ DEF, if

(i) ∠A = ∠D, ∠B = ∠E, ∠C = ∠F and

AB BC CA
(ii) = = , then the two triangles are similar (see Fig.)
DE EF FD

B C E F

95

Book 1.indb 95 30/04/2019 4:46:44 PM


NMAT by GMAC™ Official Guide 2019

• AAA criterion of similarity: If in two triangles, corresponding angles are equal, then their
corresponding sides are in the same ratio (or proportion) and hence the two triangles are similar.
This criterion is referred to as the AAA (Angle–Angle–Angle) criterion of similarity of two
triangles.
• SSS criterion of similarity: If in two triangles, sides of one triangle are proportional to (i.e., in the
same ratio of ) the sides of the other triangle, then their corresponding angles are equal and hence the
two triangles are similiar. This criterion is referred to as the SSS (Side–Side–Side) similarity criterion
for two triangles
• SAS criterion of similarity: If one angle of a triangle is equal to one angle of the other triangle and
the sides including these angles are proportional, then the two triangles are similar. This criterion is
referred to as the SAS (Side–Angle–Side) similarity

Areas of Similar Triangles


The ratio of the areas of two similar triangles is equal to the square of the ratio of their corresponding sides

P
A

B C Q R
M N

2 2 2
ar (ABC) æ AB ö æ BC ö æ CA ö
=ç =ç =ç
÷
ar (PQR) è PQ ø ÷
è QR ø è RP ÷ø

Properties of Triangles
• Angles opposite to equal sides of a triangle are equal,
• Sides opposite to equal angles of a triangle are equal,
• A point equidistant from two given points lies on the perpendicular bisector of the line-segment
joining the two points and its converse,
• A point equidistant from two intersecting lines lies on the bisectors of the angles formed by the two
lines,
• In a triangle
i. side opposite to the greater angle is longer
ii. angle opposite the longer side is greater
iii. the sum of any two sides is greater than the third side.

96

Book 1.indb 96 30/04/2019 4:46:44 PM


2.0  Quantitative Skills Review  2.8  Geometry and Mensuration

Example 2

In the triangle given which of the following must be true?


Y

(a − 30)° a°
X Z

(A) XY = YZ
(B) YZ = XZ
(C) ∠X > 50°
(D) ∠Y = 50°
(E) XY = XZ
Solution

Since ∠Y and ∠Z are equal, the sides opposite them must also be equal. Hence XY = XZ.
Since ∠X is the smallest angle in the triangle, the side opposite to it will also be the shortest, that is,
YZ is shorter than XY and XZ.
Sum of angles of a triangle = 180°
a + a + (a - 30) = 180
So,
⇒ 3a = 210
⇒ a = 70

Thus, ∠Y = ∠Z = 70° and ∠X = 70 - 30 = 40°
So, it is clear that option (E) is the only one that can be correct.
The correct answer is E.

Area and perimeter of a triangle


1
Area of a ∆ = × base × height
2
Perimeter of a ∆ = Sum of lengths of its sides

Important concepts related to triangles


1. The largest angle in a triangle lies opposite to the longest side, the second largest angle lies opposite to
the second longest side and the smallest angle lies opposite to the shortest side.
2. If two triangles are similar, their sides and angles are in the same proportion.
3. If two triangles are congruent, their corresponding angles have the same measure and corresponding
sides have the same length.

97

Book 1.indb 97 30/04/2019 4:46:44 PM


NMAT by GMAC™ Official Guide 2019

3 Quadrilaterals
Introduction
A quadrilateral is a figure enclosed by four sides. It has four angles and four vertices. The sum of interior
angles of a quadrilateral is always 360°.
Types of Quadrilaterals
The figure given below shows the different types of quadrilaterals:

Quadrilateral

Parallelogram Trapezoid
Kite

Rectangle Isosceles
Rhombus Trapezoid

Square

Types of Quadrilaterals

Parallelogram
A quadrilateral is a parallelogram if opposite sides are parallel.

Parallelogram

98

Book 1.indb 98 30/04/2019 4:46:45 PM


2.0  Quantitative Skills Review  2.8  Geometry and Mensuration

Properties of a Parallelogram
• A diagonal of a parallelogram divides it into two congruent triangles.
• In a parallelogram, pair of opposite sides is equal and parallel
• In a parallelogram, opposite angles are equal.
• The diagonals of a parallelogram bisect each other
• The area of a parallelogram is the product of the base and the height
• The area is twice the area of a triangle created by one of its diagonals.
• The perimeter of a parallelogram is 2(a + b) where a and b are the lengths of adjacent sides.
• Adjacent angles of a parallelogram are supplementary.
Rhombus
A quadrilateral is a rhombus if all four sides have the same length.
C
B

D B C A
I I

D
A

Rhombus

Properties of a Rhombus
• Since a rhombus is a special type of parallelogram, all properties of parallelograms apply to rhombus.
• Opposite sides are parallel.
• Opposites sides are congruent.
• All sides are congruent.
• Opposite angles are congruent. Any pair of consecutive angles are supplementary.
• Diagonals bisect each other.
• The diagonals of a rhombus are perpendicular bisectors of each other.
Trapezium
A quadrilateral is a trapezoid if it contains one pair of parallel sides.

Trapezium

99

Book 1.indb 99 30/04/2019 4:46:45 PM


NMAT by GMAC™ Official Guide 2019

Properties of a Trapezium
• The parallel sides are called bases.
• The other two non-parallel sides are called legs.
• If the two non-parallel sides are equal and form equal angles at one of the bases, the trapezium is an
isosceles trapezium.

Rectangle
A rectangle, as shown in the figure below, is a quadrilateral with all four angles equal to 90°.

Rectangle

Properties of a Rectangle
• The opposite sides have equal length.
• The diagonals also have equal length.
• Area of a rectangle = length × breadth
• Perimeter of a rectangle = 2 (length + breadth)
Example 1

A 4-inch wide gold frame is placed around a rectangular photograph with dimensions 6 inches by 8
inches. What is the cost of framing this photograph if gold framing costs Rs. 100 per square inch?
(A) 72
(B) 176
(C) 7,200
(D) 17,600
(E) 20,200
Solution

We first need to start by calculating the area of the gold frame, which is the total area of the frame +
photograph minus the area of the photograph.
To get the dimensions of the frame + photograph, we need to add the width of the frame to the
photograph. Since the width of the photograph is 6 in., the total width of the photograph and the
frame becomes 6 + 4 + 4 = 14 inches (note that you will require to add the width of the frame twice,
since this frame will be on both sides of the photograph). Similarly, the combined height of the frame
+ photograph becomes 8 + 4 + 4 = 16 inches.
So, Area of frame = Combined area - Area of photograph
= (14 × 16) - (6 × 8) = 224 - 48 = 176 inches2

100

Book 1.indb 100 30/04/2019 4:46:45 PM


2.0  Quantitative Skills Review  2.8  Geometry and Mensuration

At the rate of Rs. 100 per square inch, the total cost of making this gold frame comes to Rs. 100 × 176
= Rs. 17,600.
The correct answer is D.

Example 2

In the figure given PQRS is a rectangle. If the area of ∆STR is 20, what is the area of ∆SPR?
P Q

S R

(A) 20
(B) 25
(C) 30
(D) 35
(E) 40
Solution

Let us drop a perpendicular TU from T on the line SR.


P Q

S U R

1
Area of ∆STR = × SR × TU = 20
2
1
And area of ∆SPR = × SR × PS
2
So, the only difference between the two areas is the height TU and PS. If we can find some relation
between them, we can arrive at the answer.
Now, the point of intersection of the diagonals of a rectangle also happens to be the midpoint of both
its sides.
So TU is the half of PS or PS = 2TU

101

Book 1.indb 101 30/04/2019 4:46:46 PM


NMAT by GMAC™ Official Guide 2019

1
So, area of ∆SPR = × SR × (2 × TU)
2
1
= × SR × TU × 2 = 20 × 2 = 40
2
The correct answer is E.

Example 3

In the figure given, PS || QR. What is the perimeter of the quadrilateral PQRS?

150
P S

20 25

Q R

(A) 320 units


(B) 345 units
(C) 360 units
(D) 380 units
(E) 400 units

Solution

Make a right triangle inside the figure by dropping a perpendicular ST from S to QR as shown in the
figure given.

P S

20 20 25

Q 150 T R

Now, PQ = ST = 20 units
So, use Pythagoras’ theorem to find the length of TR
SR2 = ST2 + TR2
252 = 202 + TR2
TR2 = 625 - 400
TR = 225 = 15 units
So, the perimeter of the figure PQRS = 20 + 150 + 15 + 25 + 150 = 360 units
The correct answer is C.

102

Book 1.indb 102 30/04/2019 4:46:46 PM


2.0  Quantitative Skills Review  2.8  Geometry and Mensuration

Square
A square, as shown in the figure below, is basically a rectangle with all sides equal.

Square

Properties of a Square
• All the properties of a rhombus apply, that is, sides are parallel, diagonals are perpendicular bisectors
of each other, and diagonals bisect the angles).
• All the properties of a rectangle apply (the only one that matters here is diagonals are congruent).
• Area of a square = s2 (side squared)
• Perimeter of a square = 4s (4 times side)

Important Learning:  All squares are rectangles but all rectangles are not squares.

4 Circles
Introduction
Technically, a circle is the set of all points in a plane which are equidistant from a certain point, called the
centre of the circle.

Important concepts related to circles


Circum
fere
nc
e
Chord

Centre

ter Rad
me ius
Dia

Parts of a circle
103

Book 1.indb 103 30/04/2019 4:46:47 PM


NMAT by GMAC™ Official Guide 2019

1. Chord: A chord is a segment whose end points lie on the circle.


2. Diameter: The diameter, as shown in the figure on the previous page, is the line segment connecting
two points on the circle which passes through the centre of the circle. The diameter is the longest
chord in a circle.
3. Radius: The distance between the centre of a circle and any point on the circle is called the radius. It
is basically half of the diameter.
4. Circumference: The distance around the outer boundary of a circle is called its circumference. It is the
same as the perimeter of the circle.
Circumference, C = 2pr
5. Area of a circle: A = pr2
6. Tangent: A tangent is a line that touches the circle at any one point on its circumference. A line-
drawn tangent to a circle is perpendicular to the radius at the point of tangency.
7. Arc: An arc is a portion of the circumference of a circle. In the figure below, AB is an arc of the circle.
The shorter distance between A and B is called the minor arc and the longer distance (via D) is called
the major arc. An arc that is exactly half the length of the circumference is called a semicircle.

Arc of a circle

The ratio of the length of an arc to the circumference of a circle is the same as the ratio of the angle
subtended by the arc at the centre of the circle to 360°. This is also the same as the ratio of the area of the
sector to the area of the entire circle. So,
Length of arc AB ∠ACB Area of sector ACB
= =
Circumference 360 Area of circle
This is a commonly tested concept on the NMAT by GMAC™, so make sure you are clear on its
application.

Important Learning:  The longest chord that can be drawn inside a circle is the same as its diameter.

104

Book 1.indb 104 30/04/2019 4:46:47 PM


2.0  Quantitative Skills Review  2.8  Geometry and Mensuration

Example 1

In the figure below, the ratio of the length of minor arc PSR to the circumference of the circle Q is
1:9. What is the measure of ∠a?


R
Q

(A) 30°
(B) 40°
(C) 45°
(D) 50°
(E) 60°
Solution

Here we need to use the ratio


Length of arc Measure of arc's central angle
=
Circumference 360o
1 a
⇒ =
9 360
360
⇒a= = 40
9
The correct answer is B.

Cyclic Quadrilaterals
A quadrilateral ABCD is called cyclic if all the four vertices of it lie on a circle.
D

B C

105

Book 1.indb 105 30/04/2019 4:46:48 PM


NMAT by GMAC™ Official Guide 2019

Properties of a Cyclic Quadrilaterals


The sum of either pair of opposite angles of a cyclic quadrilateral is 180º.

ÐA + ÐC = 180°
ÐB + ÐD = 180°

Key points to remember


1. The perpendicular from the centre of the circle to a chord bisects the chord.
2. Equal chords of a circle are equidistant from the centre.
3. The angle subtended by an arc at the centre is double the angle subtended by it at any point on the
remaining part of the circle.
4. A triangle drawn inside a circle, whose one side is the diameter of the circle, is always a right triangle.
5. When a square is inscribed in a circle, the diagonal of the square is the same as the diameter of the
circle.
6. When a circle is inscribed in a square, the diameter of the circle is the same as the side of the square.

5 Mensuration
Introduction
Mensuration is a branch of Mathematics that deals with measurement of various parametres of geometric
figures. The concept of mensuration is based on the understanding of the formulae and their applications.
Therefore, it is very important to know the formulae for solving problems in mensuration.

Cuboid
A cuboid is a three-dimensional figure having different length, breadth and height (see the figure below).

A cuboid

Following are the important formulae related to a cuboid. The dimensions of a cuboid are the length,
breadth and height denoted by l, b and h, respectively.
1. Volume = l × b × h
2. Total surface area = 2(lb + bh + lh)
Length of the longest diagonal (body diagonal) of the cuboid = l + b + h
2 2 2
3.
4. Area of the four walls = 2(lh + bh)

106

Book 1.indb 106 30/04/2019 4:46:48 PM


2.0  Quantitative Skills Review  2.8  Geometry and Mensuration

Example 1

A pit of dimensions 20 cm × 15 cm × 9 cm is dug and the soil is taken to fill a land of dimensions 12
cm × 18 cm. By how much will the height be increased?
(A) 10.5 cm
(B) 11.5 cm
(C) 12.5 cm
(D) 13.5 cm
(E) 14.5 cm
Solution

In this problem, the total volume of soil dug out has to be equal to the landfill. Let the height be
increased by y cm. Therefore,
20 ×15 × 9 = 12 × 18 × y ⇒ y = 12.5 cm

The correct answer is C.

Important Learning:  In all the problems where any three-dimensional figure is melted or dug out and
cast into or poured into or filled into another three-dimensional figure, it is the total volume that has to
be made equal.

Cube
A cube is a three-dimensional figure having length, breadth and height all equal (see the figure below).

A cube

Following are the important formulae related to a cube. The dimension of a cube is the side of the cube
denoted by l.
1. Volume = (Side)3 or l 3, where l is the side of the cube
2. Total surface area = 6l 2
3. Length of the longest diagonal = l × 3
4. Length of the face diagonal = l × 2

107

Book 1.indb 107 30/04/2019 4:46:49 PM


NMAT by GMAC™ Official Guide 2019

Cylinder
A cylinder is a three-dimensional figure having a height or length denoted by h and the radius of the base
denoted by r (see the figure below).

A cylinder

Following are the important formulae related to a cylinder. The dimensions of a cylinder are the radius of
the base and its height denoted by r and h, respectively.
1. Volume = p r2h
2. Curved surface area = 2p rh
3. Total surface area = 2p r2 + 2p rh
Example 2

If the radius of the base of a right circular cylinder is r and its volume is  66  r 2 cm3, find the height
 7 
of the cylinder.
(A) 3 cm
(B) 4 cm
(C) 6 cm
(D) 7 cm
(E) 9 cm

Solution

Volume of a cylinder = p r 2 h
r 2 22 2 66 2
Þ 66 ´ = ´r ´h= r
7 7 7
Þ h = 3 cm
The correct answer is A.

108

Book 1.indb 108 30/04/2019 4:46:49 PM


2.0  Quantitative Skills Review  2.8  Geometry and Mensuration

Cone
A cone is a three-dimensional figure having a height denoted by h, a slant height denoted by l and a radius
of the base denoted by r (see the figure below).

h
l

A cone

Following are the important formulae related to a cone. The dimensions of a cone are the radius of the base
and its height denoted by r and h, respectively.
1. l 2 = r 2 + h2
1
2. Volume = p r 2 h
3
Curved surface area = p rl
3.
Total surface area = p r2 + p rl
4.

Sphere
Following are the important formulae related to a sphere. The dimension of a sphere is its radius denoted
by r.
4p r 3
1. Volume =
3
Surface area = 4p r2
2.

109

Book 1.indb 109 30/04/2019 4:46:50 PM


2.11  Modern Math

Book 1.indb 110 30/04/2019 4:46:50 PM


2.0  Quantitative Skills Review  2.11  Modern Math

2.12 What is Measured?


Modern math includes advanced topics such as Permutation and Combination, Probability and Coordinate
Geometry.
You will be tested on the fundamental rule of counting and when to use addition and when to use
multiplication within the same.
You will also be tested on the expression of formulae such as nPr , nCr , equation of a line and its slope, etc.

2.13 Overall Test Taking Strategies


• These are difficult questions, so attempt them after you have attempted everything else.
• Read the question carefully and decide whether you are looking at a permutation question or a
combination question.
• Try to put the factorial sign in the numerator and denominator in such a manner that you can cancel
out a lot of the numbers.
• While attempting coordinate geometry questions, draw the coordinate plane and mark the points
given in the question on it so that it becomes easier for you to visualise the problem.
The next few sections will provide you with in-depth strategies for approaching each topic.

111

Book 1.indb 111 30/04/2019 4:46:50 PM


NMAT by GMAC™ Official Guide 2019

1 Permutation and Combination


Introduction
Permutation and combination are two of the most logical topics in mathematics, and their applications can
be observed and verified in real-life situations.

Fundamental principle of counting


The concepts in permutation and combination are based on the fundamental principle of counting, which
in turn comprises the rule of multiplication and the rule of addition. These are listed as follows:
1. Rule of multiplication: If a certain task can be done in m ways and after having done it, another
can be done in n ways, then the total number of ways in which the two tasks can be done together is
m × n.
Thus, if there are three ways of going from A to B and four ways of going from B to C, then the
number of ways of going to C from A via B = 3 × 4 = 12 ways.
2. Rule of addition: If a certain task can be done in m ways and another can be done in n ways, then
either of the two tasks can be done in m + n ways.
Thus, when tasks A and B both have to be completed, we multiply, but when either A or B has to be
completed, we add.

Important Learning:  Whenever the concept in the question is the same as ‘and’ we will use
multiplication and whenever the concept is same as ‘or’ we will use addition.

Permutation
While a combination deals only with selection, permutation is selection and arrangement both, that is,
in permutation, we are not only interested in selecting the things, but we also take into consideration the
number of possible arrangements of the selected things.
The permutation of n different things taken r at a time is
n!
n
Pr =
(n - r)!

The concept of permutation is illustrated in the following example.


Example 1

Suppose there are four children in a group. Find the number of ways in which any two children out of
the four can be arranged for a photograph.
(A) 8 ways
(B) 10 ways
(C) 12 ways
(D) 14 ways
(E) 16 ways
Solution

We need to select two children out of the four, and then arrange them. So, we need to take the
permutation of four different things, taken two at a time.

112

Book 1.indb 112 30/04/2019 4:46:50 PM


2.0  Quantitative Skills Review  2.11  Modern Math

4 4!
P2 =
( 4 - 2)!
24
= = 12 ways
2
The correct answer is C.

Permutation of N different things taken all at a time


The permutation of n different things taken all at a time is
n! n!
n
Pn = = = n!
(n - n)! 0 !

This is the result of selecting n things out of n given things, and then arranging them. But, if n things are
being selected out of n things, then selection will be possible in only one way and so the result obtained,
that is, n! is entirely due to the arrangement of the selected things.
This leads to a very important result of permutation, that is, n different things can be arranged in a straight
line in n! ways.
For example, if four students are to be arranged in a straight line for a photograph, the total number of ways
of doing so is 4! = 4 × 3 × 2 × 1 = 24 ways.

Permutation of identical things


This refers to the number of permutations of n things taken all at a time. Here, p things are alike of one
kind, q of second kind, r of third kind and so on. The permutation is given by
n!
p! × q ! × r !

that is, divide by the factorial of the number of identical things.


The concept is illustrated by means of the following example.
Example 2

In how many different ways can the letters of the word ARRANGE be arranged?

(A) 1200 ways


(B) 1240 ways
(C) 1260 ways
(D) 1300 ways
(E) 1340 ways
Solution
7!
Total number of ways = = 1,260 ways
2! × 2!

The correct answer is C.

113

Book 1.indb 113 30/04/2019 4:46:51 PM


NMAT by GMAC™ Official Guide 2019

Circular permutation
The total number of ways in which n different things can be arranged in a circle = (n − 1)! ways.
For example, if three people are to be seated on a circular table for dinner, the number of ways of doing
so is (3 − 1)! = 2 ways.

Important Learning:  If there are n different things, they can be arranged in a circle in (n − 1)!
different ways.

Distribution

Case 1
Different things to be distributed to different groups.
Example 3

In how many ways can five different balls be distributed in three different boxes?
(A) 143 ways
(B) 243 ways
(C) 343 ways
(D) 443 ways
(E) 543 ways
Solution

The first ball can go into any of the three boxes, and therefore, the first ball can be distributed in three
ways. Having done this, the second ball can be distributed in three ways and so on.
Therefore, the five balls can be distributed in 3 × 3 × 3 × 3 × 3 = 35 ways or 243 ways.
The correct answer is B.

Case 2
Identical things to be distributed to different groups.
Example 4

In how many ways can five identical balls be distributed in three different boxes?
(A) 15 ways
(B) 18 ways
(C) 20 ways
(D) 21 ways
(E) 24 ways
Solution

Five identical balls have to be distributed in three different boxes. If space has to be divided into three
boxes, we can do so by using two partitions.

114

Book 1.indb 114 30/04/2019 4:46:51 PM


2.0  Quantitative Skills Review  2.11  Modern Math

Now, one of the possible arrangements will be (bbb, bb, 0), that is, three balls in the first box, two balls
in the second box and no balls in the third box.
Other possible arrangements can be:

1. (bb, bbb, 0)
2. (b, bbb, b)
3. (b, bb, bb)
4. (b, b, bbb) and so on
If you observe closely, we are arranging and rearranging seven things in a straight line, out of which
five (balls) are identical of one type and two (partitions) are identical of another type.
7!
Thus, required number of ways = = 21 ways
5! × 2!
The correct answer is D.
Example 5

In how many different ways can 10 students of a class with roll numbers from 1 to 10 be seated in a
straight line such that one of the extreme positions has a student with an odd roll number while the
other extreme position has a student with an even roll number?
(A) 48 × 6!
(B) 44 × 12!
(C) 50 × 8!
(D) 50 × 6!
(E) 52 × 8!
Solution

Let the first place have an odd number. This place can be filled in five ways (1, 3, 5, 7 and 9).
The other extreme position can be also filled in five ways (2, 4, 6, 8 and 10).
These two extreme positions can be interchanged also. Therefore, the two extreme positions can be
first filled in
5 ways × 5 ways × 2 ways = 50 ways

Now, the remaining eight people can occupy eight available positions in 8! ways.
Total ways = 50 × 8! ways
The correct answer is C.

115

Book 1.indb 115 30/04/2019 4:46:51 PM


NMAT by GMAC™ Official Guide 2019

Example 6

Eight students were to be seated along two rows such that four students will be seated in each of the
two rows called A and B. Two of the eight students definitely want to be seated in row A while one
of them definitely wants to be seated in row B. In how many different ways can the eight students be
seated?
(A) 5,760
(B) 5,960
(C) 6,500
(D) 6,760
(E) 7,160
Solution
The two students who want to be in row A can be seated in 4 × 3 = 12 ways
The student who wants to be in row B can be seated in four ways
The remaining five people can be seated in 5! ways = 120 ways
Therefore, total number of ways = 12 × 4 × 120 = 5,760 ways
The correct answer is A.

Combination
Combination means selection only, that is, in combination, we are only interested in the selection of things
and not in their arrangement.
In general, the number of combinations of n different things taken r at a time is given by nCr , where
n!
n
Cr =
r !(n - r)!

where n! (read as n factorial) = n × (n − 1) × (n − 2) × … × 4 × 3 × 2 × 1


Thus,
0! = 1 (by definition)
1! = 1
2! = 2 × 1
3! = 3 × 2 × 1 and so on
Let us try to understand the concept of combination by taking an example.
Example 7

There are four fruits, an apple, a mango, a banana and an orange, and we need to select any two fruits
out of these four fruits. In how many ways can we do this?
(A) 4 ways
(B) 5 ways
(C) 6 ways
(D) 7 ways
(E) 8 ways

116

Book 1.indb 116 30/04/2019 4:46:51 PM


2.0  Quantitative Skills Review  2.11  Modern Math

Solution

Number of ways of selecting two fruits from four fruits is 4C2, that is,
4! 24
= =6
2 ! × ( 4 - 2)! 4
that is, six different ways, which will be AM, AB, AO, MB, MO and BO.
The correct answer is C.

Rules of combination
n
1. C0 = 1 way (there is only one way to select 0 things out of n different things).
n
2. Cn = 1 way (there is only one way to select n things out of n different things).
n
3. C1 = n ways (there are n ways to select one thing out of n different things).
n
4. Cr = nCn-r (the number of ways of selecting r things out of n different things is the same as identifying
those (n-r) things that will not be selected).

Combination of N different things taken 0 or some or all at a time


The number of combinations of n different things taken 0 or some or all at a time is:
n
C0 + nC1 + nC2 + nC3 + … + nCn
The above expression is the binomial expansion of (1 + 1)n = 2n
Therefore,
n
C0 + nC1 + nC2 + nC3 + … + nCn = 2n

Example 8

Eleven players are to be selected for a match out of an available list of 14 players. In how many ways
can this be done such that the best two identified players are always selected?
(A) 14C11
(B) 12P9
(C) 12C9
(D) 12C9 × 2!
(E) 12C11
Solution

If two people have to be definitely selected, then nine people have to be selected of the remaining
12 people = 12C9.
The correct answer is C.

117

Book 1.indb 117 30/04/2019 4:46:52 PM


NMAT by GMAC™ Official Guide 2019

2 Probability
Introduction
Probability is defined as the chance of happening of an event and is a measure of the likelihood that an
event will occur. It is used to quantify an attitude of mind towards some proposition of whose truth we
are not certain. The certainty we adopt can be described in terms of a numerical measure and this number,
between 0 and 1 (where 0 indicates impossibility and 1 indicates certainty), is called probability.
Thus, the higher the probability of an event, the more certain we are that the event will occur. A simple
example would be the toss of a fair coin. As the two outcomes are deemed equiprobable, the probability
1
of ‘heads’ equals the probability of ‘tails’, and each probability is or equivalently a 50% chance of either
‘heads’ or ‘tails’. 2

In many ways, the concept of probability can be said to be an extension of the concepts of permutation and
combination.
In examples and problems where the event is described, the basis of solving the problem happens to be the
classical definition of probability, which says
Number of favourable outcomes
Probability of an event E, denoted by P(E) =
Total number of outcomes

where the favourable number of outcomes or cases is as defined in the problem under the given set of
conditions and the total number of outcomes or cases is the number of ways of doing the task without any
condition being applicable.
Example 1

What is the probability that the month of May will have five Tuesdays?

2
(A)
3

(B) 2
5

(C) 3
5

(D) 3
7
3
(E)
8
Solution

The month of May will have 31 days which would get converted into 28 days (4 weeks) and 3 odd
days.
Therefore, each of the 7 days will definitely appear 4 times. The remaining 3 days can be
• Mon, Tue, Wed
• Tue, Wed, Thu
• Wed, Thu, Fri

118

Book 1.indb 118 30/04/2019 4:46:52 PM


2.0  Quantitative Skills Review  2.11  Modern Math

• Thu, Fri, Sat


• Fri, Sat, Sun
• Sat, Sun, Mon
• Sun, Mon, Tue
Total number of cases = 7.
Number of favourable cases = 3 (There are the three cases in which Tuesday appears.)
Required probability = 3 .
7
The correct answer is D.

Elements of probability
1. Scope: Probability is always defined for the future.
2. Random experiment: A random experiment is an experiment, trial or observation that can be
repeated numerous times under the same conditions. The outcome of an individual random
experiment must be independent and identically distributed. It must in no way be affected by any
previous outcome and cannot be predicted with certainty.
3. Sample space: The total number of ways in which an event can happen is called the sample space of
the event.
• Sample space for toss of coins:
(a) Tossing a coin: two elements (H or T)
(b) Tossing two coins: two elements × two elements = four elements (HT, TH, TT, HH)
(d) Tossing three coins: two elements × two elements × two elements = eight elements

(HHH, HHT, HTH, THH, TTT, TTH, THT, HTT)

• Sample space for throwing one or more dice:


(a) Throw a dice: six elements (1, 2, 3, 4, 5, 6)
(b) Throw two dice: 36 elements [(1, 1), (1, 2), (1, 3)…(1, 6), (2, 1), (2, 2)…(2, 6) and so on till
(6, 1), (6, 2), (6, 3), (6, 4), (6, 5), (6, 6)]

4. Range of probability of any event: As the number of favourable outcomes cannot be greater than
the total number of outcomes, therefore 0 ≤ P(E) ≤ 1, that is, probability of any event will always lie
between 0 and 1.

Important Learning:  The range of probability of any event will always lie between 0 and 1, both
inclusive.

5. Any event will either happen or not happen: P(E) + P(~E) = 1, that is, sum of the probability of
happening of an event and the probability of its not happening = 1.
6. Odds for and against an event: It is not necessary that a problem will always provide probabilities.
There are examples where odds in favour of an event or odds against an event are provided.

119

Book 1.indb 119 30/04/2019 4:46:52 PM


NMAT by GMAC™ Official Guide 2019

Number of favourable outcomes


Odds in favour of an event = Number of unfavourable outcomees

Number of unfavourable outcomes


Odds against an event = Number of favourable outcomees

7. Mutually exclusive events: Any two events are said to be mutually exclusive events if they cannot
occur together.
For two mutually exclusive events A and B

P(A or B) = P(A) + P(B)


and if the events are not mutually exclusive, then

P(A or B) = P(A) + P(B) − P(A and B)


In these expressions, P (A or B) represents the probability of either event A happening or event B
happening.
8. Independent events: Any two events where the probability of happening of one event is not
dependent on the other are said to be independent events.
For two independent events A and B,

P(A and B) = P(A) × P(B)


The above result will be true for any number of independent events.
For three independent events A, B and C

P(A, B and C) = P(A) × P(B) × P(C)

9. Pack of cards: Problems related to a pack of cards are frequently asked as a part of questions on
probability. In case of a pack of cards, remember the following:

• A pack has 52 cards (26 black and 26 red), which get divided into 13 cards each of four different
types or variety.
• These 13 cards are (Ace, 2, 3, 4, 5, 6, 7, 8, 9, 10, J, Q and K) and the four types are Hearts,
Diamond, Spade and Club. Technically, these four types are called suits. So, in a normal pack of
cards, there will be four aces, four ones, four twos and so on, thus, making a total of 52 cards.
• King, Queen and Jack are also popularly known as face cards. A pack of cards will, therefore, have
12 face cards.

Example 2

What is the probability that a card drawn at random from a pack of cards is either black or a jack?

17
(A)
52
7
(B)
13
27
(C)
52

120

Book 1.indb 120 30/04/2019 4:46:53 PM


2.0  Quantitative Skills Review  2.11  Modern Math

3
(D)
13
5
(E)
9
Solution
There are a total of (13 + 13) = 26 black cards and 4 jacks in a pack of cards. But remember that two
of these jacks are also black cards, so we will have to account for this overlap.
Therefore, number of favourable cards = 13 + 13 + 4 - 2 = 28.
Total cards = 52.
28 7
Required probability = = .
52 13
The correct answer is B.

10. AND rule: Suppose the probability of event A is P(A) and that of event B is P(B). Now, if these two
events are to occur together, i.e. if we want to calculate the probability of occurrence of event A and
that of event B, we need to multiply the individual probabilities of A and B.
Therefore, the probability of occurrence of A and B will be P(A) × P(B).
11. OR rule: When the probabilities of two events are given and we need to calculate the probability that
at least one of them occurs, we need to add the individual probabilities of the two events. Therefore, if
the probability of event A is P(A) and that of event B is P(B), then the probability that at least one of
the two events occurs is P(A) + P(B) - P(A and B).
12. Conditional Probability: When A and B are dependent events, then P(A/B) denotes probability
of happening of event A when B has already occurred. Similarly, P(B/A) denotes probability of
happening of B when A has already occurred.
Example 3

Two students are selected from a class of 5 girls and 12 boys. Find the probability that a particular pair
of girl and boy is selected.

1
(A)
136
1
(B)
63
1
(C)
51
10
(D)
21

(E) 13
61
Solution

Total number of possibilities = 17C2


17 × 16
=
1× 2
= 17 × 8 = 136

121

Book 1.indb 121 30/04/2019 4:46:53 PM


NMAT by GMAC™ Official Guide 2019

Number of favourable cases = 1


1
Therefore, the required probability = .
136
The correct answer is A.

Example 4

A bag contains five red and nine black balls, while another has three red and five black balls. A ball
is first selected at random from bag 1 and transferred to bag 2, and then a ball is drawn from bag 2.
What is the probability that the ball drawn is red?

25
(A)
103
47
(B)
126

(C) 33
97

(D) 47
97
61
(E)
131

Solution

We need to consider both the cases here.


Red ball is transferred, and then red ball is drawn.
Black ball is transferred, and then red ball is drawn.
5 4 9 3
= × + ×
14 9 14 9
20 27 47
= + =
126 126 126

The correct answer is B.

Example 5

Three cards are drawn from a pack of cards at random. Find the probability that they consist of both
colours.

26
C1 × 26 C 2
(A) 52
C3

(B) 2 × 52C1 × 26 C 2
26

C3

26
C3
(C) 52
C3

122

Book 1.indb 122 30/04/2019 4:46:54 PM


2.0  Quantitative Skills Review  2.11  Modern Math

(D) 2 × 52C1 × 26 C 2
26

C3

(E) None of these

Solution

Number of favourable cases = 2 red and 1 black or 1 red and 2 black


= 26C2 × 26C1 + 26C1 × 26C2 = 2 × 26C2 × 26C1
Total cases = 52C3
2 × 26C2 × 26
C1
Therefore, required probability = 52
C3
The correct answer is B.

Bayes’ Theorem
Bayes’ theorem is a direct application of conditional probabilities. This theorem is used to find the
conditional probability of an event P(A | B), say, when the “reverse” conditional probability P(B | A) is
known.
Let A1, A2, ... , An be a set of mutually exclusive events that together form the sample space S. Let B be any
event from the same sample space, such that P(B) > 0. Then,

P( A k Ç B )
P ( A k |B) =
P ( A 1 ) P ( B| A 1 ) + P ( A 2 ) P (B| A 2 ) + ¼+ P ( A n ) P ( B| A n )

We know that P(Ak ∩ B ) = P(Ak )P(B | Ak ), so, Bayes’ theorem can also be expressed as

P ( A k ) P ( B| A k )
P ( A k |B) =
P ( A 1 ) P ( B| A 1 ) + P ( A 2 ) P (B| A 2 ) + …+ P ( A n ) P ( B| A n )

Example 6

Three companies P, Q and R supply 20%, 30% and 50% of the books to a college. Past experience
shows that 2%, 4% and 5% of the books produced by these companies are defective. If a book was
found to be defective, what is the probability that the book was supplied by company Q?
8
(A)
77
10
(B)
77
12
(C)
77
17
(D)
77
19
(E)
77

123

Book 1.indb 123 30/04/2019 4:46:55 PM


NMAT by GMAC™ Official Guide 2019

Solution

Let us consider that P, Q and R be the events that books are supplied by companies P, Q and R
respectively.
Suppose D be the event that books are defective.
According to the question,
P(P) = 0.2, P(Q) = 0.3 and P(R) = 0.5
Also, P(D|P) = 0.02, P(D|Q) = 0.04, P(D|R) = 0.05

P(Q )P(D / Q )
Required probability, P(Q / D) =
P(P)P(D / P) + P(Q )P(D / Q ) + P(R)P(D / R)

0.3 × 0.04 0.012 0.012 12


= = = =
0.2 × 0.02 + 0.3 × 0.04 + 0.5 × 0.05 0.04 + 0.012 + 0.025 0.077 77

The correct answer is C.

3 Progression
Introduction
A series in which a particular relation exists between the terms is called a progression. There are three types
of progressions: arithmetic progression (AP), geometric progression (GP) and harmonic progression (HP).
For the NMAT by GMAC™, you need to focus on AP and GP.

Arithmetic progression
Popularly known as AP, it is a series of terms in which the difference between a term and the next term is
constant. This difference is called the common difference of the AP and is denoted by d.
Some examples of AP are:
1. 2, 4, 6, 8, 10, 12, 14, …
2. −3, −6, −9, −12, −15, −18, …
3. 1, 2, 3, 4, 5, 6, 7, 8, …
1 3 5 7
4. , 1, , 2, , 3, , 4, 
2 2 2 2
Properties of AP
The first term is called a, the common difference is called d and the number of terms is denoted by n.
Therefore, an AP would be like a, a + d, a + 2d, a + 3d, a + 4d and so on. Some important properties of AP
are listed below:
1. The nth term of an AP is given by Tn = a + (n − 1)d, a relation between the nth term, the first term,
the common difference and the number of terms.
2. If the same quantity (positive or negative) is added to each term of an AP, the series will continue to
be an AP.

124

Book 1.indb 124 30/04/2019 4:46:56 PM


2.0  Quantitative Skills Review  2.11  Modern Math

3. If the same quantity (positive or negative) is multiplied with or divides each term of an AP, the series
will continue to be an AP.
4. In an AP, the sum of the terms equidistant from the beginning and end is a constant and is equal to
the sum of the first and last terms. Let us try to understand this with the help of an example:
3, 6, 9, 12, 15, 18, 21, 24 is an AP

• Sum of the first and last terms = 3 + 24 = 27


• Sum of the second and second last terms = 6 + 21 = 27
• Sum of the third and third last terms = 9 + 18 = 27
• Sum of the fourth and fourth last terms = 12 + 15 = 27

What if the number of terms is odd?


Let us try to understand this with the help of another example:
4, 7, 10, 13, 16, 19, 22 is an AP

• Sum of the first and last terms = 4 + 22 = 26


• Sum of the second and second last terms = 7 + 19 = 26
• Sum of the third and third last terms = 10 + 16 = 26
• The middle term in this case, which does not form a pair, will be half the sum of the first and last
terms.

5. Sum to n terms of an AP:


n
Sn = (a + l )
2

where l is the last term, in this case the nth term of the AP, and l = Tn = a + (n − 1)d
Substituting for l, we get
n
Sn = [2a + (n - 1)d]
2
One can use either of the above formulae to find the sum to n terms of an AP.

6. Three consecutive terms in an AP will be a − d, a and a + d.


7. Four consecutive terms in an AP will be a − 3d, a − d, a + d and a + 3d.
8. Five consecutive terms in an AP will be a − 2d, a − d, a, a + d and a + 2d.
n(n + 1)
9. Sum of first n natural numbers = .
2
n(n + 1)(2n + 1)
10. Sum of squares of first n natural numbers = .
6
2
 n(n + 1) 
11. Sum of cubes of first n natural numbers =  2  .

12. If a, b and c are in an AP, then b is the arithmetic mean of the three numbers.

125

Book 1.indb 125 30/04/2019 4:46:56 PM


NMAT by GMAC™ Official Guide 2019

Important Learning:  In an AP, the sum of the terms equidistant from the beginning and end is a
constant and is equal to the sum of the first and the last terms.

Example 1

Which term of the AP series 3, 8, 13, ... is the term 78?


(A) 14
(B) 15
(C) 16
(D) 17
(E) 18
Solution
In the given AP
an = a + (n - 1)d = 78
We have a = 3, d = 8 - 3 = 5. Therefore,
3 + (n - 1) × (5) = 78
(n - 1) × 5 =78 - 3 = 75
75
n - 1 = = 15
5
n = 15 + 1 = 16
The correct answer is C.

Example 2
A man pays a rent of Rs. 50 for the first day, Rs. 100 for the second day and so on, with the rent on
each day being Rs. 50 more than the rent on the previous day. What is the total rent paid for the first
10 days?
(A) 2,750
(B) 2,800
(C) 3,050
(D) 3,100
(E) 3,350
Solution
The series is an AP with a = 50, d = 50 and n = 10
10
Sn = [2 × 50 + (10 − 1) × 50]
2
S n = 5 (100 + 450)
S n = 5 (550)
S n = Rs. 2 , 750
The correct answer is A.

126

Book 1.indb 126 30/04/2019 4:46:56 PM


2.0  Quantitative Skills Review  2.11  Modern Math

Geometric progression
GP refers to a series in which the ratio of a term to its previous term is constant. This ratio is called the
common ratio of the GP and is denoted by r.
Some examples of GP are:
1. 2, 4, 8, 16, 32, 64, …
1 1 1 1
2. , , , ,
3 9 27 81
3. −2, 4, −8, 16, −32, 64, …

Properties of GP
The first term is called a, the common ratio is r and the number of terms is denoted by n. Therefore, a GP
would be like a, ar, ar2, ar3, ar4 and so on.
Some important properties of a GP are listed below:
1. The nth term of a GP denoted by T n is given by T n = ar(n−1).
2. If a constant term (positive or negative) is multiplied with or divides each term of a GP, the series
continues to be a GP.
3. Sum to n terms of a GP is given by
a(r n - 1)
Sn =
r -1
a
S∞ =
1- r

a
4. Three consecutive terms in GP will be , a and ar.
r

a a
5. Four consecutive terms in GP will be 3 , , ar and ar3.
r r
6. If a, b and c are in GP, then
b c
=
a b
b2 = ac
b = ac

Then, b is called the geometric mean (GM) of a and c. GM of three numbers a, b and c is the cube root of
the product of the three numbers.

Example 3

Find the sum to infinite terms of the series 1 , 1 , 1 , 


4 16 64

(A) 1
2

(B) 1
3

127

Book 1.indb 127 30/04/2019 4:46:57 PM


NMAT by GMAC™ Official Guide 2019

1
(C)
4
1
(D)
6
1
(E)
8

Solution

As per the problem,


1
a = and r = 1
4 4
1
a 4 1 4 1
Sum to infinite terms = = = × =
1-r  1 4 3 3
 1 - 
4

The correct answer is B.

Harmonic Progression
A series of terms is said to be in Harmonic Progression (HP) if the reciprocal of the terms are in AP. As an
1 1 1
example, if a, b and c are in HP, then , and c will be in AP. Therefore,
a b
1 1 1 1
- = -
b a c b
2 1 1 a+c
= + =
b a c ac
2ac
b=
(a + c )
where b is called the harmonic mean (HM) of a and c.
Example 4

If the second term of a harmonic progression is 5 and the 5th term of the same harmonic progression
is 11, then find the 56th term.
12
(A)
99
13
(B)
99
37
(C)
99
41
(D)
99
55
(E)
99

128

Book 1.indb 128 30/04/2019 4:46:58 PM


2.0  Quantitative Skills Review  2.11  Modern Math

Solution

The reciprocals of the HP form an arithmetic progression a, a + d, a + 2d …..


1 1
Then, a + d = and, a + 4d =
5 11
Solving above two equations, we get:
13 −2
a= ,d=
55 55
th 13 −2 99
Therefore, 56 term of AP= a + 55d = + 55 × =
55 55 55
55
Hence, 56th term of HP =
99
The correct answer is E.

Relationship Between the Means of AP, GP and HP


If AM, GM and HM be the arithmetic, geometric and harmonic means between a and b, then the
following results hold:
a+b
AM = … (i)
2

GM = ab … (ii)

2ab
HM = … (iii)
a+b

Therefore, we can write:

a + b 2ab
AM × HM = . = ab = GM2
2 a+b

Or GM2 = AM × HM ...(iv)

Also, we have:

AM – GM = a + b − ab = a + b + 2 ab
2 2
2
 a− b
=   …(v)
 2 

129

Book 1.indb 129 30/04/2019 4:46:59 PM


NMAT by GMAC™ Official Guide 2019

which is positive if a and b are positive; therefore, the AM of any two positive quantities is greater than
their GM.
Also, from equation (iv) we have,
GM2 = AM × HM
Clearly then, GM is a value that would fall between AM and HM and from equation (v) it is known that
AM > GM, therefore we can conclude that GM > HM.
In other words, we can say that the arithmetic, geometric and harmonic means between any two +ve
quantities are in descending order of magnitude.
Sum to n Terms of Special Series

(i) Sum of the first n natural numbers:


n (n + 1)
∑ n = 1 + 2 + 3 + ... + n = 2
(ii) Sum of the squares of first n natural numbers.
n ( n + 1 )(2n + 1)
∑n 2
= 12 + 2 2 + 3 2 + ... + n 2 =
6
(iii) Sum of cubes of first n natural numbers:

 n (n + 1) 
2

∑ n 3 = 13 + 23 + 33 + ... + n 3 =  2 
 
Example 5

Evaluate: 62 + 72 + 82 + 92 + 102 + 112


(A) 449
(B) 450
(C) 451
(D) 452
(E) 453
Solution

Required Sum = (12 + 22 + 32 +…112) – (12 + 22 + 32 +…52)

11(11 + 1)(2 × 11 + 1) 5(5 + 1)(2 × 5 + 1)


S= −
6 6
11 × 12 × 23 5 × 6 × 11
S= − = 451
6 6
The correct answer is C.

130

Book 1.indb 130 30/04/2019 4:46:59 PM


2.0  Quantitative Skills Review  2.11  Modern Math

4 Set Theory
Introduction
A set is defined as a group or collection of objects having similar properties. The objects are called elements
of the set and are represented by small alphabets while the set itself is represented by capital letters. Also,
the number of distinct elements of the set is called the cardinal number of the set.

Representation of a set
There are two basic ways to represent a set:
1. Tabular or Roster method: In this method, all the elements of the set are shown or represented
within a curly bracket and separated by a comma.
For example, {a, e, i, o, u} is the set of all vowels in the English language.
Similarly, {1, 3, 5, 7, 9} is the set of all odd natural numbers less than 10.
2. Set-builder method: In this method, the elements of the set are not shown. Instead, the rule or
criteria to form or build the set are provided.
For example, B = {b: b is a vowel of the English language}.

Venn diagrams
The idea of representing a set and solving set-based questions on the basis of a diagram was first suggested
by Swiss mathematician Euler and was later developed by British mathematician Venn. So, the diagram
representing a set is known as Euler–Venn diagram or simply Venn diagram. A Venn diagram uses a closed
curve, generally an oval or a circle, to represent a set.

Types of sets
1. Empty set or Null set: A set having zero element or no element is called a null set or empty set or
void set represented by { } or F.
2. Singleton set: A set having a single element is called a singleton set.
3. Universal set: A set which is the combination of all possible sets under consideration is called the
universal set.
4. Equivalent sets: Any two sets having the same number of elements, that is, the same cardinal number,
are called Equivalent sets.
For example, Set A is the set of all the vowels in the English language while Set B is the set of all the
odd numbers less than 10. Then
Set A = {a, e, i, o, u}
Set B = {1, 3, 5, 7, 9}

Set A and Set B are called equivalent sets.


5. Equal sets: Two sets A and B are said to be equal sets if every element of A is also an element of B and
vice versa.
For example, if Set A = {a, c, b} and Set B = {c, b, a}, A and B will be known as equal sets.

Subset of a set
Set A is said to be a subset of Set B if every element of A belongs to B. Also, If A is a subset of B, then B is
called the superset of A.

131

Book 1.indb 131 30/04/2019 4:46:59 PM


NMAT by GMAC™ Official Guide 2019

Subsets are of two types, listed as follows:


1. Proper subset: If every element of A belongs to B but there is at least one element in B which does
not belong to A, then A is called a proper subset of B. This is represented by A ⊂ B.
2. Improper subset: If A is a subset of B and A is also equal to B, then A and B are called improper
subsets of each other. This is represented by A ⊆ B .
Some important properties of subsets:
1. Every set is a subset of itself.
2. Null set is a subset of all sets.
3. Universal set is a superset of all sets.
4. If the cardinal number of a set is n, then the number of subsets = 2n. Out of these, one subset is an
improper subset while the remaining (2n - 1) subsets are proper subsets.

Operations on sets
The following operations have been defined on sets:
1. Union of sets: Denoted by A ∪ B , it is the set of all those elements that belong to Set A or to Set B.
For example, if Set A = {2, 3, 5, 7, 9} and Set B = {1, 3, 5, 6, 8}, then A ∪ B = {1, 2, 3, 5, 6, 7, 8, 9}
2. Intersection of sets: Denoted by A ∩ B , it is the set of all those elements that belong to set A and
also to set B.

A B

The shaded region in the above figure denotes the intersection of Sets A and B. In the example above,
A ∩ B = {3, 5}
If A ∩ B is a null set, then A and B are said to be disjoint sets.
3. Complement of a set: Denoted by ~A, it is the set of all those elements that belong to the universal
set (U ) but are not present in set A.
As an example, let Set A be the set of all the single digit natural numbers and let Set B = {2, 3, 5, 7},
then the complement of Set B denoted by ~B = {1, 4, 6, 8, 9}

4. Difference of two sets: Denoted by A - B, it is the set of all those elements that belong to A but not
to B.
Let Set A be defined as A = {2, 7, 9, 11, 13, 14, 17, 19} and Set B = {2, 5, 8, 11, 12, 15, 17}, then,
A - B = {7, 9, 13, 14, 19} and B - A = {5, 8, 12, 15}.

132

Book 1.indb 132 30/04/2019 4:47:00 PM


2.0  Quantitative Skills Review  2.11  Modern Math

Important results on set operations:

1. n(A ∪ B) = n(A) + n(B) if A and B are disjoint sets

2. n(A ∪ B) = n(A) + n(B) - n(A ∩ B)

3. n(A ∪ B ∪ C) = n(A) + n(B) + n(C) - n(A ∩ B) - n(B ∩ C) - n(A ∩ C) + n(A ∩ B ∩ C)

Example 1

If set P is the set of all the prime numbers less than 50, find the cardinal number of P.
(A) 11
(B) 12
(C) 13
(D) 14
(E) 15
Solution
Prime numbers less than 50 are 2, 3, 5, 7, 11, 13, 17, 19, 23, 29, 31, 37, 41, 43, 47.
The cardinal number of the set P is 15.
The correct answer is E.

Important Learning:  If the cardinal number of a set is n, the number of subsets is 2n. Out of this,
(2n – 1) subsets are proper subsets and the remaining one is an improper subset.

Example 2

In an exam where 175 students appeared, 140 passed in physics, 150 in biology while 10 failed in both
the subjects. Find the percentage of people who passed in both the subjects.
(A) 68.71%
(B) 71.42%
(C) 74.56 %
(D) 78.54%
(E) 81.76%
Solution

Let x be the number of students who passed in both the subjects.


As 10 students failed in both the subjects, the number of students who passed in at least one of the
two subjects = 175 – 10 = 165. Then
140 + 150 – x = 165
x = 290 – 165 = 125 students
Percentage of students who passed in both the subjects
125
× 100 = 71.42%
175
The correct answer is B.

133

Book 1.indb 133 30/04/2019 4:47:00 PM


NMAT by GMAC™ Official Guide 2019

5 Coordinate Geometry
Introduction
Coordinate geometry requires us to work with the rectangular coordinate system. In a plane, two
perpendicular number lines are drawn, as shown in the figure below. The horizontal line is called the x-axis
and the vertical one is called the y-axis. The point of intersection of the two axes is called the origin and is
denoted by O.
y

7
6
(−,+) (4,5)
5
Quadrant II
4
Quadrant I
3
(+,+)
2
(−2,2)
1
0
x −7 −6 −5 −4 −3 −2 −1 O −1 1 2 3 4 5 6 7 x

−2
Quadrant III
−3
(−,−) Quadrant IV
−4
(−3,−4) (+,−)
−5
−6

−7 (+5,−6)

The coordinate plane

Any point in the coordinate plane can be specified by the location of its x coordinate and y coordinate.
Thus, if a point is specified as (2, 3), this means that it lies at a distance of 2 units to the right of the y-axis
and 3 units above the x-axis.

Distance formula
The distance between any two points in the coordinate plane can be found using the distance formula

134

Book 1.indb 134 30/04/2019 4:47:01 PM


2.0  Quantitative Skills Review  2.11  Modern Math

Y
2 2
( X1,Y1)
(X2 – X1) + (Y2 – Y1)

X ( X2,Y2 ) X

d = ( x 2 - x 1 )2 + ( y 2 - y 1 )2

Section Formula
The section formula tells us the coordinates of the point which divides a given line segment into two parts
such that their lengths are in the ratio m:n.
The section formula is helpful in coordinate geometry; for instance, it can be used to find out the centroid,
incentre and excentres of a triangle.
Internal Divisions with Section Formula

A(x1, y1) P(x, y) B(x2, y2)

If point P(x, y) lies on line segment AB (between points A and B) and satisfies AP : PB = m : n, then we say
that P divides AB internally in the ratio m : n. The point of division has the coordinates
 mx 2 − nx1 my 2 − ny 1 
P=  , 
 m−n m−n 
As a special case of internal division, if P is the midpoint of AB , then it divides AB internally in the ratio
1 : 1. Hence applying the formula for internal division and substituting m = n = 1, we get:

 x + y1 x 2 + y 2 
P=  1 ,
 2 2 
External Divisions with Section Formula

A(x1, y1) B(x2, y2) P(x, y)



If P = (x, y) lies on the extention of line segment AB (not lying between points A and B) and satisfies AP :

PB = m : n, then we say that P divides AB externally in the ratio m : n. The point of division is

P =  mx 2 − nx 1 , my 2 − ny 1 
 
m−n m−n 

135

Book 1.indb 135 30/04/2019 4:47:02 PM


NMAT by GMAC™ Official Guide 2019

Mid-point formula
The mid-point of a line connecting two points [(x1, y1) and (x2, y2)] can be calculated using the formula
(x2 + y2)
x1 + x2 y1 + y2
,
2 2
(x1 + y1)

 x + x 2 y1 + y 2 
Mid point =  1 ,
 2 2 

Example 1

C is the midpoint of line segment AB. The coordinates of A are (−2, 6) and the coordinates of C are
(2, 0). What are the coordinates of B?
(A) (2, 6)
(B) (2, 0)
(C) (0, −6)
(D) (6, −6)
(E) (3, 0)
Solution


Let the coordinates of B be x and y.
Now, as per midpoint formula
(-2 + x ) (6 + y )
− = 2 and =0
2 2
-2 + x = 4 and 6 + y = 0
x = 6 and y = -6
So, the coordinates of B are (6, −6).
The correct answer is D.

Slope of a line
Slope is the inclination of a line with reference to the positive direction of x-axis. It is denoted by m and is
calculated as
y1 - y 2
m=
x1 - x 2

The angle θ between the two lines having slopes m1 and m2 is given by:

tan θ = ±
( m1 − m2 )
1 + m1m2
If we take the acute angle between two lines, then

tan θ =
( m1 − m2 )
1 + m1m2
136

Book 1.indb 136 30/04/2019 4:47:02 PM


2.0  Quantitative Skills Review  2.11  Modern Math

If the lines are parallel, then m1 = m2


If the lines are perpendicular, then m1 m2 = – 1
1. If a line is slanting up from left to right, then its slope is positive.
2.
If a line is slanting down from left to right, then its slope is negative.
3.
If a line is parallel to x-axis, then its slope is zero.
4.
If a line is parallel to y-axis, then its slope is undefined.
5.
Two parallel lines have the same slope.
6.
The slopes of two perpendicular lines are negative reciprocals of each other. So, if the slope of line A is
1
3, then the slope of line B, which is perpendicular to line A, will be − .
3

Important Learning:  Parallel lines have the same slope and the slopes of perpendicular lines are
negative reciprocals.

Area of the triangle


Area of the triangle whose vertices are (x1,y1), (x2, y2) and (x3,y3) is

1
|x1(y2 – y3)+ x2 (y3 –y1) + x3 (y1 – y2)|
2
Collinearity of three points
There are two methods to find if three points are collinear. One is slope formula method and the other is
area of triangle method.

Slope formula method to find that points are collinear.


If three points P (h, k), Q (x1, y1) and R (x2, y2 ) are such that slope of PQ = slope of QR,
y1 - k y 2 - y1
i.e., =
x1 - h x 2 - x1
or (h – x1) (y2 – y1) = (k – y1) (x2 – x1) then they are said to be collinear.
Area of triangle to find if three points are collinear
Three points are collinear if the value of area of triangle formed by the three points is zero.
1
x1 (y 2 - y 3 ) + x 2 (y 3 - y 1 ) + x 3 (y 1 - y 2 ) = 0
2
Or, x1 (y 2 − y 3 ) + x 2 (y 3 − y 1 ) + x 3 (y 1 − y 2 ) = 0

Intercepts
1. The x-intercept is the x coordinate of the point where a line intersects the x-axis.
2. The y-intercept is the y coordinate of the point where a line intersects the y-axis.

137

Chapter 2.indd 137 01/05/2019 4:22:47 PM


NMAT by GMAC™ Official Guide 2019

Various forms of the equation of a line


(i) If a line is at a distance a and parallel to x-axis, then the equation of the line is
y=±a
(ii) If a line is parallel to y-axis at a distance b from y-axis then its equation is
x=±b
(iii) Point-slope form: The equation of a line having slope m and passing through the
point (x0, y0) is given by y – y0 = m (x – x0)
(iv) Two-point-form: The equation of a line passing through two points (x1, y1) and
(x2, y2) is given by
y 2 − y1
y – y1 = (x – x1)
x 2 − x1
(v) Slope intercept form: The equation of the line making an intercept c on y-axis and
having slope m is given by
y = mx + c
Note that the value of c will be positive or negative as the intercept is made on
the positive or negative side of the y-axis, respectively.
(vi) Intercept form: The equation of the line making intercepts a and b on x– and y–axis respectively is
x y
given by + = 1
a b
Example 2

Find the slope of line AB that is perpendicular to the line EF whose equation is 4y + 12x = 48.
2
(A)
3
1
(B)
3
(C) 3
(D) -3
(E) - 1
3
Solution
First rewrite the equation for EF in the slope intercept form,
4y + 12x = 48
Dividing the entire equation by 4, we get:
y + 3x = 12
y = -3x + 12
Thus, the slope of EF = -3
Since slopes of perpendicular lines are negative reciprocals of each other,
1
Slope of AB =
3
The correct answer is B.

138

Book 1.indb 138 30/04/2019 4:47:03 PM


2.0  Quantitative Skills Review  2.11  Modern Math

Example 3

In the xy-coordinate plane, line L is perpendicular to the y-axis and passes through the point (8, -4).
Which of the following is an equation of the line L?
(A) x = -4
(B) x = 8
(C) y = -8
(D) y = -4
(E) y - 4 = x + 8
Solution

L is perpendicular to the y-axis, that is it is parallel to the x-axis, which means its slope is 0. Then its
y-intercept has to be -4 since L is perpendicular to the y-axis.
Equation of L
y = mx + c
y = 0(x) + (-4)
y = -4
The correct answer is D.

Distance of a point from a line


The perpendicular distance (or simply distance) d of a point P (x1, y1) from the line Ax + By + C = 0 is given
by

Ax1 + By1 + C
d=
A 2 + B2

Example 4

Find the perpendicular distance from the point (3, 4) to the line 4x + 3y +5 = 0.
(A) 12/5 units
(B) 17/5 units
(C) 19/5 units
(D) 23/5 units
(E) 29/5 units
Solution
|ax1 + by 1 + c| |4 × 3 + 3 × 4 + 5| 29
Required distance, d = = = units
a 2 + b2 32 + 42 5
The correct answer is E.

Distance between two parallel lines


The distance d between two parallel lines y = mx + c1 and y = mx + c2 is given by
c1 − c 2
d=
1 + m2

139

Book 1.indb 139 30/04/2019 4:47:03 PM


NMAT by GMAC™ Official Guide 2019

Example 5


Find the perpendicular distance between the two parallel lines y = 4x + 5 and y = 4x + 7.
1
(A) units
17
2
(B) units
17
3
(C) units
17
4
(D) units
17
5
(E) units
17
Solution

|C1 − C 2 | |5 − 7| 2
Required distance, d = = = units
1 + m2 1 + 42 17

The correct answer is B.


Nature of two given lines
Two lines a1x + b1y +c1 = 0 and a2x + b2y + c2 = 0 are

a1 b1
(i) intersecting if ≠
a 2 b2

a1 b1 c1
(ii) parallel and distinct if = ≠
a 2 b2 c 2

a1 b1 c1
(iii) coincident if = =
a 2 b2 c 2

Important Learning:  Two lines in the same plane are parallel if their slopes are same but y-intercepts
(C1 and C2) are different.

140

Book 1.indb 140 30/04/2019 4:47:04 PM


Book 1.indb 141 30/04/2019 4:47:04 PM
2.14  Data Interpretation

Book 1.indb 142 30/04/2019 4:47:04 PM


2.0  Quantitative Skills Review  2.14  Data Interpretation

2.15 What is Measured?


The Data Interpretation section consists of tables, charts, pie diagrams, etc., that you will need to analyse
and apply while answering the questions.
Your mental math skills get tested indirectly as you will need to make a lot of quick calculations especially
using concepts of percentages, ratios, etc.
Your estimation skills also get tested as, in a lot of the questions, you will be able to arrive at the answer just
by intelligent estimation and elimination of wrong answer choices.

2.16 Overall Test Taking Strategies


• Scan the data once quickly but do not pay too much attention to every small detail as you can always
do so later, based on the questions that you get.
• The figures are all drawn to scale so you can estimate the answer, especially if the answer choices are
very far apart.
• Do not confuse percentages with actual numbers.
The next few pages will provide you with in-depth strategies for approaching this topic.

143

Book 1.indb 143 30/04/2019 4:47:04 PM


NMAT by GMAC™ Official Guide 2019

1 Data Interpretation
Introduction
Data Interpretation is an important area in testing the aptitude of a candidate. The primary objective in
Data Interpretation (henceforth referred to as DI) is to:
1. Assess the capability of data assimilation, that is, to understand a data in its given form.
2. Carry out calculations based on the given data.
3. Take effective business decisions based on the calculations.
The data could be in the form of a caselet, bar graph, line graph, pie chart, histogram, frequency polygon,
etc.

Concepts in data interpretation


Growth and growth rate
Growth in essence is the change in the value of a variable from one point in time to another point in time.
Although it is generally associated with an increase in the value of a variable, mathematically, the value of
growth can be negative.
Since, growth and growth rate are associated also with the change in a variable with respect to time, both of
them are usually calculated on the immediate previous value until and unless mentioned otherwise. While
growth is the change in the absolute value of a variable, growth rate is the percentage change in the value of
a variable. In order to understand the concept of growth and growth rate, let us look at an example:
Example 1

The following table gives the value of the total sales of AR Associates across four years. The sales value
is in Rs. (’000).

Year 2012 2013 2014 2015


Sale 150 200 240 300

(A)  Find the growth in the sales of AR Associates in the year 2014.
(B)  Find the growth rate of sales of AR Associates in the year 2015.
(C)  Find the growth rate of sales of AR Associates in the period 2012–2015.
(D)  Find the growth rate of sales of AR Associates in the year 2012.
(E) If the growth rate of sales of AR Associates in the year 2012 is 25%, find the value of the sales
of AR Associates in the year 2011.
Solution

(A)  The growth in the sales of AR Associates in the year 2014


2, 40 , 000 - 2, 00 , 000 = Rs. 40 , 000

(B)  Growth rate of sales in the year 2015


(3 , 00 , 000 - 2, 40 , 000) 6
× 100 = × 100 = 25%
2, 40 , 000 24

144

Book 1.indb 144 30/04/2019 4:47:04 PM


2.0  Quantitative Skills Review  2.14  Data Interpretation

(C)  Growth rate of sales in the period 2012–2015


(3 , 00 , 000 - 1, 50 , 000)
× 100 = 100%
1, 50 , 000

(D) If we want to find out the growth rate of sales in the year 2012, we need to find the value of
sales in the immediately preceding year, that is, 2011.
Since this data is not provided, the answer to the question cannot be determined.
(E) Let y be the sales of AR Associates in the year 2011. Therefore,
125
y× = 1, 50 , 000
100
5
y× = 1, 50 , 000
44

y = Rs. 1, 20 , 000

Percentage change in growth rate


Many of us do not differentiate between the calculation of percentage change in growth and growth rate.
Actually, these are two different concepts.
Percentage change in growth rate gives the percentage change between two values that are expressed in
percentage terms, that is, percentage change in growth rate gives the percentage change in two values that
are growth rates.
To understand the concept better, consider Example 1 again.
Example 2


Find the percentage change in the growth rate of AR associates in the year 2014.
(A) Decrease of 20%
(B) Increase of 20%
(C) Decrease of 40%
(D) Increase of 40%
(E) Increase of 50%
Solution

In order to answer this question, we would need the value of the growth rate of sales of AR Associates
in the year 2013 and the year 2014.
Growth rate of sales of AR Associates in the year 2013
(2, 00 , 000 - 1, 50 , 000) 100
× 100 = = 33.33%
1, 50 , 000 3

Growth rate of sales of AR Associates in the year 2014


(2, 40 , 000 - 2, 00 , 000) 100
× 100 = = 20%
2, 00 , 000 5

145

Book 1.indb 145 30/04/2019 4:47:05 PM


NMAT by GMAC™ Official Guide 2019

Therefore, percentage change in growth rate


(20 - 33.33)
× 100 = -40%
33.33
That is, a decrease of 40%.

The correct answer is C.

Percentage points
Percentage point is the difference between two values that are expressed in percentage terms. Consider the
following example.
Example 3

A girl got 55% marks in the fifth semester and 57.5% marks in the sixth semester. By how many
percentage points are the marks in the sixth semester more than the marks obtained in the fifth
semester?
(A) 1.5 percentage points.
(B) 2.5 percentage points.
(C) 3.5 percentage points.
(D) 4.5 percentage points.
(E) 5.5 percentage points.
Solution

Required value = 57.5% - 55% = 2.5 percentage points.

The correct answer is B.

Market share
Market share is the total percentage of the market serviced by a company, product or brand. Market share
can be calculated either on volume basis (number of units sold) or value basis (revenue or sales or turnover).
Also average product price is equal to the total market value divided by the total market volume. Consider
the following example.
Example 4
In a market there are three products being sold. The price along with the total sale in units is given for
each of the three products.

Product Price/Unit Sale in Numbers Sale in Rs.


P Rs. 5 10,000 50,000
Q Rs. 4 15,000 60,000
R Rs. 4.5 20,000 90,000

(A)  Find the market share of product P on volume basis.


(B)  Find the market share of product Q on value basis.
(C)  Find the average market price.
146

Book 1.indb 146 30/04/2019 4:47:05 PM


2.0  Quantitative Skills Review  2.14  Data Interpretation

Solution

(A) Total market volume = 45,000 units.


Total sale of product P = 10,000 units.
10 , 000 200
Market share of P on volume basis = × 100 = = 22.22%
45, 000 9
(B) Total market value = Rs. 50,000 + Rs. 60,000 + Rs. 90,000 = Rs. 20,000
Sales of product Q = Rs. 60,000
60 , 000
Market share of product Q on value basis = × 100 = 30%
2, 00 , 000
(C) Average market price is given by
Total market value 2, 00 , 000 200 40
= = = = Rs. 4.44/unit
Total market volume 45, 000 45 9

These concepts in Data Interpretation are illustrated through the following solved examples.
Data for Examples 5–7:  The following table gives the sales of four companies in lakhs across four years
from 2004 to 2007. Read the data carefully and answer the questions that follow.

Company 2004 2005 2006 2007


A 120 130 145 160
B 150 166 160 190
C 170 185 190 200
D 110 125 140 150

Example 5


By what percentage are the sales of company B in the year 2005 more than that of company A in the
year 2004?
(A) 20%
(B) 33.33%
(C) 38.33%
(D) 40%
(E) 46%
Solution

Required value
 166 - 120  5 230
 120  × 100 = 46 × 6 = 6 = 38.33%

The correct answer is C.

147

Book 1.indb 147 30/04/2019 4:47:05 PM


NMAT by GMAC™ Official Guide 2019

Example 6


By how much value is the average sale of company C more than the average sale of company D during
the period?
(A) 35 lakhs
(B) 38 lakhs
(C)  45 lakhs
(D)  55 lakhs
(E)  None of these
Solution

Average sales of company C is


745
170 + 185 + 190 + 200 =
4
525
Average sales of company D is 110 + 125 + 140 + 150 =
4
745 525 220
Difference is - = = 55 lakh
4 4 4
The correct answer is D.

Example 7


If the four companies account for a 40% market share by value in 2007, what was the total sale of the
market?
(A) 14 crore
(B)  16 crore
(C)  17.5 crore
(D)  18 crore
(E)  20 crore
Solution

Total sales of the four companies in 2007 = 160 + 190 + 200 + 150 = Rs. 700 lakh
This is 40% of the total sales. Therefore,
Total sales = 700 × 2.5 lakh = Rs. 17.5 crores

The correct answer is C.

148

Book 1.indb 148 30/04/2019 4:47:05 PM


2.0  Quantitative Skills Review  2.14  Data Interpretation

Data for Examples 8–10:  The given pie chart shows the percentage market share by volume of five
companies P, Q, R, S and T in the television market in the year 2011. The total sale volume-wise in the
market in the year 2011 is 4,00,000 TV sets.

Example 8


If S is the others category and company X has a share of 30% in this category, find the number of TV
sets sold by company X in the year 2011.
(A) 10,000
(B) 12,000
(C) 15,000
(D) 18,000
(E) 20,000
Solution

Total sale of the others category = 10% of 4,00,000 = 40,000


Sales of company X = 30% of 40,000 = 12,000
The correct answer is B.

Example 9

If the total sale of two products P2 and P3 of company P is 30000, find the percentage contribution of
products P2 and P3 to the total sales unit-wise of company P in the year 2011.
(A) 37.5%
(B)  40%
(C) 50%
(D) 60%
(D) 75%
Solution

P2 and P3 have together sold 30000 units.


Company P has a total sale of 20% of 4,00,000 = 80,000
Therefore, contribution of P2 and P3
30 , 000 100
× 100 = 3 × = 3 × 12.5 = 37.5%
80 , 000 8

The correct answer is A.

149

Book 1.indb 149 30/04/2019 4:47:06 PM


NMAT by GMAC™ Official Guide 2019

Example 10


By what percentage is the market share of R more than the market share of P?

(A) 20%
(B) 25%
(C) 30%
(D) 35%
(D) 40%
Solution

The percentage by which the market share of R is more than that of P is


 25 - 20 
  × 100 = 25%
20 

The correct answer is B.


Data for Examples 11–13:  Two thousand students applied for admission to various programmes at
AU University. Out of the total applicants, 20% did not take the admission test. The following table gives
the cumulative frequency in percentage of the mark range received by those students who appeared for the
admission test.

Marks Cumulative Frequency (%)


≤ 10 marks 15
≤ 20 marks 25
≤ 30 marks 40
≤ 40 marks 60
≤ 50 marks 85
≤ 60 marks 100

Example 11


What is the number of students who received marks in the range of 21–30 in the admission test?
(A) 240
(B) 400
(C) 500
(D) 640
(E) None of these
Solution
As per the problem, 2,000 students applied out of which 20% did not appear for the test.
Number of students who appeared = 80% of 2,000 = 1,600
Percentage of students in the range of 21–30 marks = 40 - 25 = 15%
Therefore, 15% of 1,600 = 240 students

The correct answer is A.

150

Book 1.indb 150 30/04/2019 4:47:06 PM


2.0  Quantitative Skills Review  2.14  Data Interpretation

Example 12


If more than 40 marks are required to qualify for the next round, find the difference between the
number of students who qualified for the next round and those who failed to qualify for the next
round.
(A) 160
(B) 240
(C) 320
(D) 380
(E) 420
Solution

Percentage of students who qualified for the next round = 40%


Percentage of students who did not qualify for the next round = 60%
Difference between the two = 20% of 1600 = 320 students
The correct answer is C.


Example 13


By what percentage is the number of students in the range of 41–50 marks more than those in the
range of 0–10 marks?
(A) 10%
(B) 20%
(C) 33.33%
(D) 55.55%
(E) 66.66%
Solution

Percentage of students in the range of 41–50 marks = 25%


Percentage of students in the range of 0–10 marks = 15%
Therefore,
 25 - 15 
  × 100 = 66.66%
15 

The correct answer is E.

151

Book 1.indb 151 30/04/2019 4:47:06 PM


NMAT by GMAC™ Official Guide 2019

Data for Examples 14–16:  The given bar graph shows the total sales value (in Rs. lakh) and the profit
percentage for a company ABC Ltd. for 4 years from 2008 to 2011. Answer the questions based on the
following data.

10
2011 200
25
2010 175
Sales in Rs. Lakhs
15
2009 150 Profit%
20
2008 120

0 100 200

Example 14


If the total sales is equivalent to the selling price, what is the value of profit for ABC Ltd. for the year
2010?
(A)  Rs. 35 lakh
(B)  Rs. 38.75 lakh
(C)   Rs. 40 lakh
(D)  Rs. 45 lakh
(E) None of these
Solution
We need to find out the profit for the year 2010. Therefore,
125 5
CP × = 175 ⇒ CP × = 175 ⇒ CP = 4 × 35 = 140
100 4
Profit = 175 - 140 = Rs. 35 lakh

The correct answer is A.

Example 15


Which year has shown the highest percentage change in profit %?
(A) 2008
(B) 2009
(C) 2010
(D) 2011
(E)  Cannot be determined

152

Book 1.indb 152 30/04/2019 4:47:06 PM


2.0  Quantitative Skills Review  2.14  Data Interpretation

Solution

Percentage change in profits for 2008 cannot be calculated since we do not know the profits for 2007.
(20 - 15)
Percentage change in profits for 2009 = × 100 = 25%
20
(25 - 15)
Percentage change in profits for 2010 = × 100 = 66% approx
15
(25 - 10)
Percentage change in profits for 2011 = × 100 = 60%
25
Thus, the maximum change has been for the year 2010.

The correct answer is C.

Example 16

By what percentage is the sales in 2011 more or less than the cost in 2008?
(A) 50%
(B) 100%
(C)  200%
(D) 250%
(D) 300%
Solution

Cost in 2008 = Rs. 100 lakhs


Sales in 2011 = Rs. 200 lakhs
 200 - 100 
Percentage change =  × 100 = 100%
 100 
The correct answer is B.

Important Learning:  The knowledge of fractional equivalents of various percentages, such as 12.5% is
1
the same as , will be very important while attempting data interpretation questions.
8

153

Book 1.indb 153 30/04/2019 4:47:07 PM


2.17  Data Sufficiency

Book 1.indb 154 30/04/2019 4:47:07 PM


2.0  Quantitative Skills Review  2.17  Data Sufficiency

2.18 What is Measured?


Data sufficiency questions measure your ability to analyse data and decide whether it is enough to answer
the given question. The actual answer to the question is irrelevant for our purpose.
A lot of the data sufficiency questions will test you on arithmetic concepts such as prime numbers, odd and
even numbers, negative exponents and fractions, etc.

2.19 Overall Test Taking Strategies


• Do not try to solve every statement. You only need to determine whether the given statement is
sufficient to answer the given question.
• Consider each statement separately, especially when you are looking at the second statement.
• Read the question carefully. If the question asks you whether you can find the value of y and you
realise that you cannot do so, you still have a definite answer, that is, no.
• Remember the answer choices—(A), (B), (C), (D), (E)—and what each of them stands for.
The next few pages will provide you with in-depth strategies for approaching the topic.

155

Book 1.indb 155 30/04/2019 4:47:07 PM


NMAT by GMAC™ Official Guide 2019

1 Data Sufficiency
Introduction
The primary objective of data sufficiency questions is to find out whether the given data is sufficient to
answer the question asked. In order to understand data sufficiency, we will need to look at the following
aspects:
1. The structure of a data sufficiency problem.
2. The answer choices of a data sufficiency problem.
3. How to approach a data sufficiency problem.
4. The common errors in a data sufficiency problem.

Structure of a data sufficiency problem


In a data sufficiency problem, a question is asked which needs to be answered based on the given data.
The given data is usually in the form of two statements called (1) and (2). The question asked needs to be
answered based on the two given statements. For example:
Question: What is the speed of the train?
(1)  The train covers 50 km in 30 minutes.
(2)  The length of the train is 200 m.

Answer choices
The answer choices given in a data sufficiency problem are as follows:
   (A) Statement (1) ALONE is sufficient, but Statement (2) ALONE is not sufficient.
   (B) Statement (2) ALONE is sufficient, but Statement (1) ALONE is not sufficient.
   (C) BOTH statements (1) and (2) TOGETHER are sufficient, but NEITHER statement
ALONE is sufficient.
   (D) EACH statement ALONE is sufficient.
   (E) Statements (1) and (2) TOGETHER are NOT sufficient, and additional data is needed.

Note:  Although the options will be the same in most of the cases, it is not necessary that
they will always be the same. It is suggested that a student should always read the instruc-
tions about the options while answering the question.

Approach to a data sufficiency problem


The following steps are to be followed while solving a data sufficiency problem:
1. Check whether the question can be answered using statement 1 alone without using the data given in
statement 2.
2. Check whether the question can be answered using statement 2 alone without using the data given in
statement 1.
3. If neither of the two given statements can answer the question alone, combine the data given in the
two statements to answer the question asked. In this case, the answer option of the question will be
option (C).
4. If the question asked cannot be answered by using the given data, the answer option will be
option (E).

156

Book 1.indb 156 30/04/2019 4:47:07 PM


2.0  Quantitative Skills Review  2.17  Data Sufficiency

Common errors in data sufficiency problems


Some commonly made errors are listed as follows:
1. While checking whether the question asked can be answered by using any one of the two statements
alone, you pick up data from another statement by mistake.
2. You need to combine the two statements only when you are sure that either of the two statements
cannot answer the question alone.
3. If you get more than one answer, it is not acceptable in a data sufficiency problem. For example,
if x2 = 49, then x will have two values of +7 and -7. In a situation like this, we would say that the
question cannot be answered.
4. Even a definite ‘No’ is an answer to a data sufficiency problem. For example, if the question asked is:
‘Is 1 an example of a prime number?’, then it has a definite answer which is ‘No’.
These common errors in approach to data sufficiency problems are illustrated through the following
examples.
Example 1

What is the value of x?


(1) x2 - 5x + 6 = 0
Solution

If we solve for x, the quadratic equation will give the values as x = 2 or x = 3.


So the given problem has more than one answer. This is not acceptable in data sufficiency. We always
need a unique solution. It is here that data sufficiency is different from quantitative skills.
Directions for examples 2-10: Each data sufficiency problem consists of a question and two statements,
labelled (1) and (2), which contain certain data. Using these data and your knowledge of mathematics and
everyday facts (such as the number of days in July or the meaning of the word counterclockwise), decide
whether the data given are sufficient for answering the question and then indicate one of the following
answer choices:

(A) Statement (1) ALONE is sufficient, but statement (2) ALONE is not sufficient.

(B) Statement (2) ALONE is sufficient, but statement (1) ALONE is not sufficient.
(C) BOTH statements TOGETHER are sufficient, but NEITHER statement ALONE is sufficient.
(D) EACH statement ALONE is sufficient.

(E) Statements (1) and (2) TOGETHER are NOT sufficient.

Example 2

Can I fill up the tank of 100 L capacity completely? (Measurements are accurate and there should be
no overflow.)
(1) I have a bucket which can exactly measure 5 L.
(2) I have a mug which can exactly measure 3 L.
Solution

Many students make the mistake of thinking that only statement I alone can answer the question
asked because the important aspect for them is that the tank should be filled whereas the important
aspect is whether we can answer the question uniquely about the tank getting filled.
157

Book 1.indb 157 30/04/2019 4:47:07 PM


NMAT by GMAC™ Official Guide 2019

Therefore, in the above example, using statement I alone, the answer is yes, the tank can be filled
completely and so statement 1 alone is able to answer the question asked.
Similarly, using statement 2 alone, the answer is a definite no, the tank cannot be filled completely and
so statement 2 alone is also able to answer the question asked.
So, as per the standard answer options of data sufficiency, the correct answer to the question is
option (D).
The correct answer is D.

Let us now look at some more illustrative examples to understand the options better.
Example 3

What is the value of x?


(1) x3 = -27
(2) x2 = 9
Solution

If we use the first statement alone, we will get a unique value of x = -3. Therefore, the question can be
answered by using statement 1 alone.
If we use statement 2 alone, we will get two values of x as 3 and -3. As data sufficiency requires a
unique answer, the question cannot be answered by using statement 2 alone.
The correct answer is A.

Important Learning:  Even though one of the two answers to the question using statement 2 is -3
which is obtained from statement 1 also, the question cannot be answered by using statement 2 alone.

Example 4

On which day was Naveen born?


(1)
On 25th November, Naveen celebrated his ninth birthday.
(2)
Naveen was born on a Friday.
Solution

The question is asking about the day when Naveen was born and not the date. From statement 1 we
get the date but not the day.
Using statement 2 alone, we can conclude that Naveen was born on a Friday.
The question can be answered by using statement 2 alone but not by using statement 1 alone.
The correct answer is B.
Example 5

What is the speed of the train?


(1) In an hour, the train covers a distance of 50 km after stopping for 15 min.
(2) The train is 150-m long and crosses a man moving in the same direction at 5 km/h in 10 s.

158

Book 1.indb 158 30/04/2019 4:47:07 PM


2.0  Quantitative Skills Review  2.17  Data Sufficiency

Solution

We can find the speed of the train using statement 1 alone. Do remember, the question does not want
us to find the value of the speed of the train.
What is required to be known is that distance covered and time taken can be used to find out the
speed of a moving body.
Similarly, using statement 2 alone, the question can be answered.
The correct answer is D.

Example 6

What is the profit earned in rupees?


(1) The marked price is Rs. 400 and it is sold at a discount of 30%.
(2) The marked price is 50% more than the cost price.
Solution

In order to answer the question, we will require the values of the cost price and the selling price.
While statement 1 can give us the selling price, we do not know the cost price.
Also, statement 2 can give us the cost price but we do not know the selling price.
Therefore, either of the two statements is not sufficient to answer the question alone.
Both the statements together can answer the question asked.
The correct answer is C.

Example 7

What is the ratio of the prices of A and B?


(1) Three years ago, their prices were in the ratio 4:5.
(2) In the last 3 years, the price of both A and B has increased by Rs. 5000.
Solution

When we look at a question like this, many of us would be tempted to mark option (C) as the answer.
The question cannot be answered by using either of the two statements alone, and therefore, we will
try and answer the question by combining the two statements.
Let their prices be 4x and 5x, respectively.
Also, their prices have increased by Rs. 5000 each.
The ratio of their prices will be (4x + 5000):(5x + 5000), which will not give any definite value of their
prices.
The correct answer is E.
Example 8

In how much time will the work be completed if 15 men work for 8 h every day?
(1) 20 women working for 7 h can do the work in 12 days.
(2) 8 men working for 15 h can do the work in 18 days.
159

Book 1.indb 159 30/04/2019 4:47:08 PM


NMAT by GMAC™ Official Guide 2019

Solution

The question cannot be answered by using statement 1 alone because we do not know the efficiency
comparison of a man and a woman.
Using statement 2 alone and equating the total work to be done in the section Time and Work, we
have
8 × 15 × 18 = 15 × 8 × y

Solving, we get: y = 18 days


The correct answer is B.

Important Learning:  Do not make the mistake of picking up information from Statement 1 while
checking Statement 2 and vice versa.

Note:  In data sufficiency problems that ask for the value of a quantity, the data given in the statements are
sufficient only when it is possible to determine exactly one numerical value for quantity.

Example 9

Find the value of the number if it is less than 100.


(1) The number is a perfect square.
(2) It is a multiple of 16.
Solution

Using Statement 1 alone, the question cannot be answered as there can be a number of possibilities,
such as 1, 4, 9, 16, 25, 36 and so on.
Using Statement 2 alone, there will again be a number of possibilities, such as 16, 32, 48, 64, etc.
However if we combine both the statements, there is only one possibility = 64.
Thus, C is the answer.
The correct answer is C.

Example 10

Find the speed of the stream.


(1) A log of wood travels 4 km in 3 h.
(2) Speed of a boat while moving downstream is 7 km/h.
Solution

A log of wood does not have any power of its own so it moves at the same rate as the speed of the
stream. Thus, Statement 1 is sufficient to answer the question.
The speed of a boat does not tell us anything about the speed of the stream. So Statement 2 is not
sufficient.
The correct answer is A.

160

Book 1.indb 160 30/04/2019 4:47:08 PM


Book 1.indb 161 30/04/2019 4:47:08 PM
3.0  Quantitative Skills Practice

Book 1.indb 162 30/04/2019 4:47:08 PM


3.0  Quantitative Skills Practice

3.1 Practice Questions


Solve the problems and indicate the best of the answer choices given.
Numbers: All numbers used are real numbers.
Figures: A figure accompanying a problem-solving question is intended to provide useful
information for solving the problem. Figures are drawn as accurately as possible.
Exceptions will be clearly noted. Lines shown as straight are straight, and lines that
appear jagged are also straight. The positions of points, angles, regions, etc., exist in the
order shown, and angle measures are greater than zero. All figures lie in a plane unless
otherwise indicated.

1. Arithmetic pencils represented approximately what percentage


of the money Prakash paid for all the pencils?
(A) 7%
1. Which of the following is equal to (B) 13%
 12   60   45  (C) 67%
  4  2 ? (D) 83%
 5  2  3 
(E) 88%
(A) 1 4. Company H distributed Rs. 4,000 and 180 pens
12 evenly among its employees, with each employee
1 getting an equal integer number of Rupees and an
(B)
6 equal integer number of pens. What is the highest
1 number of employees that could work for Company H?
(C)
4 (A) 9
1 (B) 10
(D)
3 (C) 20
1 (D) 40
(E)
2 (E) 180

2. Express 7.583 as a fraction: 5. If t is divisible by 12, what is the least possible


t2
91 integer value of a for which a might not be an
(A) 2
12 integer?
44
(B) (A) 2
6 (B) 3
99 (C) 5
(C)
14 (D) 6
22 (E) 40
(D)
3
6. If 5k+ 1 = 2,000, what is 5k + 1?
148
(E) (A) 399
21
(B) 401
3. Prakash bought a bag of 15 magic pencils for Rs.60. (C) 1,996
One-third of the pencils cost Rs. 2 each and the rest (D) 2,000
cost Rs. 5 each. If there was a hole in the bag and
all of the more expensive pencils fell out, the lost (E) 2,001

163

Book 1.indb 163 30/04/2019 4:47:09 PM


NMAT by GMAC™ Official Guide 2019

7. Which of the following is equal to ( x )( x ) ?


2 3 13. What is the difference between the sum of all even
positive integers between 1 and 100 (inclusive) and
(A)
5
x the sum of all odd positive integers between
100 and 150?
(B)
6
x
(A) –575
(C)
3 2
x
(B) –475
(D)
5 6
x (C) 225
(E)
6 5
x (D) 475
(E) 575
8. If 12514488 is written as an integer, how many 14. Find the smallest 4-digit number which, when
consecutive zeroes will that integer have at the end? increased by 8, is divisible by 12, 18, 30 and 45.
(A) 22 (Real NMAT Question)
(B) 32 (A) 1,072
(C) 42 (B) 1,080
(D) 50 (C) 1,088
(E) 112 (D) 1,096
9. If n is the smallest of three consecutive positive (E) 1,120
integers, which of the following must be true?
15. When 5, 8 and 12 divide a multiple of 13, they leave
(A)
n is divisible by 3 remainders of 3, 6 and 10, respectively. What is the
(B)
n is even least such number? (Real NMAT Question)
(C)
n is odd (A) 169
(D) (n)(n + 2) is even (B) 478
(E)
n(n + 1)(n + 2) is divisible by 3 (C) 598
(D) 1,298
17
10. If is expressed as a terminating decimal, (E) 1,602
2 ´ 513
10

how many zeroes are located to the right of the


16. If (XY)2 = PQX, where each letter represents a distinct
decimal point before the first non-zero digit?
digit, then find P + Q. (Real NMAT Question)
(A) 10
(A) 5
(B) 12
(B) 6
(C) 13
(C) 7
(D) 15
(D) 8
(E) 17
(E) 9
11. If 25 4 = 10 + a, and x is an integer, what could be
5 6 x

the minimum positive value of a? 17. Which of the following is a perfect square? (Real
NMAT Question)
(A) 0
(B) 30,000 (A) 97,474
(C) 30,000,000 (B) 1,23,301
(D) 10,000,000,000 (C) 1,37,641
(E) 30,000,000,000 (D) 1,70,567
(E) 1,77,243
12. What is the unit digit of 786?
(A) 0 18. Four clocks were set right at the same time. One
clock gains ‘a’ minutes in a day, second clock loses
(B) 1
‘b’ minutes in a day, third clock gains ‘c’ minutes in
(C) 3 a day and the fourth clock gains ‘d’ minutes. After
(D) 7 how much time will all the clocks again show the
(E) 9 correct time simultaneously if a + b + c + d = 1 hour,

164

Book 1.indb 164 30/04/2019 4:47:10 PM


3.0  Quantitative Skills Practice

a : b = c : d = 2 : 1 and b : d = 3 : 1? (Real NMAT 23. When a two digit number is divided by the sum of its
Question) digits, the quotient is 4. If the digits are reversed, the
new number is 6 less than twice the original number.
(A) 3 hours
The number is
(B) 12 hours
(A) 12
(C) 24 hours
(B) 21
(D) 24 days
(C) 24
(E) 144 days
(D) 42
19. Using the digits 1, 2, 3 and 4, some two digit
(E) Both (C) and (D)
numbers can be formed. The sum of these numbers
is AA0. AA is: (Real NMAT Question) 24. A five-digit number is formed using the digits 1, 3,
(A) 11 5, 7 and 9 without repetition. What is the sum of all
such possible numbers?
(B) 22
(C) 33 (A) 6666600
(D) 44 (B) 6666660
(E) 55 (C) 6666666
(D) 6666000
20. The sum of the last digits of the numbers of the form (E) None of these
22n + 1, for n = 0, 1, 2, 3 and 4, when divided by 7
gives a remainder 25. A positive integer ‘A’ is a multiple of 180 and it has
40 factors. If ‘A’ is less than 3000, then the value of
(A) 1
A/40 is
(B) 2
(A) 54
(C) 3
(B) 60
(D) 4
(C) 240
(E) 5
(D) 270
21. If ‘a’ and ‘b’ are prime numbers, then what is the (E) Cannot be determined
H.C.F. of the numbers (a2 + b2), (a + b + 1) and
26. Given a sequence of 58 terms; such that each term
(a2 + b2 – 1)?
has the form P + n where P stands for the product
(A) 1 2 × 3 × 5 …× 61 of all prime numbers less than or
(B) 2 equal to 61, and ‘n’ takes, successively, the values 2,
3, 4, …, 59. If M is the number of primes appearing
(C) 3
in this sequence, then what is the value of M? (Real
(D) 4 NMAT Question)
(E) Cannot be determined
(A) 0
22. A three digit number is such that its hundredth digit (B) 16
is equal to the product of the other two digits which (C) 17
are prime numbers. Also, the difference between
(D) 57
the number and its reverse is 297. Then, what is the
ten's digit of the number? (E) 58

(A) 2 27. Five satellites monitor a certain region during


different times, as part of a study. Alpha1 orbits
(B) 3
the region every 6 days, Beta1 orbits every 4
(C) 5 days, Gamma1 orbits every 9 days, Delta1 orbits
(D) 6 every 12 days and Epsilon1 orbits every 18 days.
(E) 7 There are certain readings to be collected about

165

Book 1.indb 165 30/04/2019 4:47:10 PM


NMAT by GMAC™ Official Guide 2019

the region from the satellites when all the five 32. How many multiples of 19 are there between 100 to
monitor the region together. The readings have 500? (Real NMAT Question)
been collected for a 292 day period. If there has
(A) 19
to be a minimum of a dozen readings collected,
for how many more days will the satellites have (B) 20
to monitor the region together? (Real NMAT (C) 21
Question) (D) 22
(A) 36 days (E) 23
(B) 90 days 33. What is the digit in the unit’s place of (9843 × 12222)
(C) 140 days + 72959 + 2543? (Real NMAT Question)
(D) 204 days (A) 5
(E) 432 days (B) 6
28. The HCF of three numbers is 6 and their sum is (C) 7
120. How many such triplets exist? (Real NMAT (D) 8
Question) (E) 9
(A) 18 34. In how many ways can the number 243 be resolved
(B) 24 into two factors? (Real NMAT Question)
(C) 29 (A) 1
(D) 32 (B) 2
(E) 35 (C) 3
29. How many consecutive zeros will be there at the (D) 4
end of the product of all prime numbers falling (E) 5
in the first 100 natural numbers? (Real NMAT
Question) 35. For how many positive integral values of ‘a’ is
a2 + 60
(A) 1 an integer? (Real NMAT Question)
a+2
(B) 4
(A) 3
(C) 5
(B) 5
(D) 10
(C) 7
(E) 12
(D) 8
30. Which of the following exactly divides x, where x is
(E) 10
the sum of the first 15 prime numbers? (Real NMAT
Question) 36. Roshni celebrated her birthday on 3rd March 1896,
which was a Tuesday. In which year will her birthday
(A) 11
again be a Tuesday? (Real NMAT Question)
(B) 21
(A) 1900
(C) 31
(B) 1901
(D) 41
(C) 1902
(E) 51
(D) 1903
31. If I insure my life for Rs. 15,000, I must pay a
(E) 1907
premium of Rs. 28 per Rs. 1,000 every year. How
much do I have to pay annually? (Real NMAT 37. The difference of the squares of two consecutive
Question) numbers is 91. What is the sum of these numbers?
(Real NMAT Question)
(A) Rs. 350
(B) Rs. 380 (A) 0
(C) Rs. 420 (B) 46
(D) Rs. 450 (C) 54
(E) Rs. 480 (D) 91
(E) None of these
166

Book 1.indb 166 30/04/2019 4:47:10 PM


3.0  Quantitative Skills Practice

38. How many positive integers n are there between 10 (A) 8


and 99, such that (n – 1) (n – 2) ... 3 ∙ 2 ∙ 1 is not (B) 8.5
divisible by n? (Real NMAT Question)
(C) 9
(A) 15 (D) 9.5
(B) 18 (E) 10.5
(C) 21
44. The average monthly income of Anil and Hemant is
(D) 88 Rs. 4,025. The average monthly income of Hemant
(E) 89 and Chandan is Rs. 6,250. The average monthly
income of Anil and Chandan is Rs. 5,200. What is
39. Which is the first digit from the left in 223?
Hemant’s monthly income? (Real NMAT Question)
(Real NMAT Question)
(A) Rs. 4,590
(A) 2
(B Rs. 5,075
(B) 3
(C) Rs. 5,950
(C) 4
(D) Rs. 6,075
(D) 6
(E) Rs. 6,950
(E) 8
40. A and B when divided by 56 leave remainders of 48 45. Average weight of 37 students is 42 kg. When their
and 32 respectively. When divided by 44, both leave teacher joined them, their average weight increased
a remainder of 24. If the sum of A and B is divisible by 2 kg. What is the weight of the teacher?
by the sum of the divisors, then find the minimum (A) 112 kg
value of (A + B). (Real NMAT Question)
(B) 114 kg
(A) 1,200 (C) 116 kg
(B) 1,800 (D) 118 kg
(C) 2,600 (E) 120 kg
(D) 3,400
46. Two persons of average age 40 years leave a group
(E) 3,800
and hence the average age of the remaining group
41. If a and b are positive integers such that a2 + 2b increases from 50 to 52 years. Find the number of
= b2 + 2a + 5, find the value of b. (Real NMAT persons originally in the group.
Question)
(A) 5
(A) 0 (B) 7
(B) 1 (C) 8
(C) 2 (D) 10
(D) 3 (E) 12
(E) Cannot be determined
47. If the average weight of 8th, 9th and 10th class is in the
42. What is the LCM of nC1 and nC2, where n is odd? ratio of a:b:c and the number of students in class 8th,
(Real NMAT Question) 9th and 10th is in the ratio of x:y:z, then the average
(A) n weight of all the three classes considered together is
n(n - 1) a+b+c
(B) (A)
2 x+y+z
(n - 1) a b c
(C) (B) + +
2 x y z
n
(D) ax + by + cz
2 (C)
x+y+z
(E) n − 1
ax + by + cz
43. If the average of a, b, c, 5, and 6 is 6, what is the (D)
a+b+c
average of a, b, c, and 13?
(E) Cannot be determined

167

Book 1.indb 167 30/04/2019 4:47:11 PM


NMAT by GMAC™ Official Guide 2019

48. The average age of a husband and wife who were (A) 86%
married 5 years ago, was 25 years then. The (B) 85%
average age of the family including the husband,
(C) 16%
the wife and two children who were born during the
interval is 16 years now. How old are the children (D) 15%
now? [All of them have different ages with integral (E) 14%
values.]
53. Machines X and Y pack books continuously, each
(A) 1 year, 3 years working at a constant rate, but Machine Y works 50%
(B) 4 years, 1 year faster than Machine X. If Machine Y packs 48,000
more books in a 24-hour period than Machine X does,
(C) 2 years, 3 years
what is Machine X’s packing rate in books per hour?
(D) 2 years, 2 years
(A) 4,000
(E) None of these
(B) 6,000
49. Ramakant spent 29% of his salary and then lost 71% (C) 8,000
of the remainder. If he was left with Rs. 39,121 after
(D) 12,000
losing the money, what is Ramakant’s salary? (Real
NMAT Question) (E) 16,000

(A) Rs. 39,121 54. The production of rice increased by 75% from 1990 to
1995. From 1995 to 2000, there was a 100% increase.
(B) Rs. 45,000
What is the percentage increase in the production of
(C) Rs. 70,000 rice from 1990 to 2000? (Real NMAT Question)
(D) Rs. 1,79,482
(A) 250%
(E) Rs. 1,90,000
(B) 280%
50. The number that is 50% greater than 80 is what percent (C) 285%
less than the number that is 25% less than 200?
(D) 290%
(A) 5% (E) 295%
(B) 10%
55. Ankur bought 5 Pizzas, 7 Samosas and 4
(C) 15%
ice-creams. Sanjeev bought 6 Pizzas, 14 Samosas
(D) 20% and 8 Ice creams for an amount which was 50%
(E) 25% more than what Ankur paid. What percentage of
the total amount spent by Ankur was spent on the
51. Aakash spends 50% of his income on rent, utilities,
Pizzas?
and insurance, and 20% on food. If he spends 30%
of the remainder on video games and has no other (A) 37.5%
expenditure, what percent of his income is left after (B) 45%
all the expenditure?
(C) 50%
(A) 0% (D) 56.5%
(B) 9% (E) 62.5%
(C) 20%
56. By selling the burger at Rs. 260 per piece, Sameer
(D) 21%
gains 30%. Find the cost price of the burger per piece?
(E) 30%
(A) Rs. 150
52. In a class of 40 students, exactly 90% had lower marks
(B) Rs. 200
than Varun's marks. 60 new students join Varun’s
class. If Varun’s marks were higher than those of 80% (C) Rs. 250
of the new arrivals, what percent of the combined class (D) Rs. 300
now had higher marks than Varun's marks? (E) Rs. 350

168

Book 1.indb 168 30/04/2019 4:47:11 PM


3.0  Quantitative Skills Practice

57. Ghosh Babu, a trader, marked up his goods 30% the first day? (Assume that he has purchased apples
over the cost price and then he gave the discount of only on the first day.) (Real NMAT Question)
5%. What was the profit percentage of Ghosh Babu in
(A) 25,000
the whole transaction?
(B) 30,000
(A) 19.5%
(C) 35,000
(B) 21.5%
(D) 40,000
(C) 23.5%
(E) 45,000
(D) 25.5%
63. The price of a watch is reduced by 10%. What is
(E) None of these
the percentage by which this new price has to be
58. The value of (p % q + q% of p) is: increased so that the watch is restored to its former
value? (Real NMAT Question)
(A) p% of q
(A) 11.1%
(B) q% of p
(B) 11.2%
(C) 2% of pq
(C) 11.4%
(D) pq% of 3
(D) 11.5%
(E) None of these
(E) 12%
59. ‘p’ is five times as large as ‘q’. By what percent is q 64. Fahim and Farhan together earned Rs. 37,570
less than p? from a business venture. They divided it and each
2 deposited his share in a bank. At the end of 9 years
(A) 16 %
3 the amount Farhan had in the bank was the same
as the amount Fahim had at the end of 7 years.
(B) 37.5%
If the interest was compounded annually at 10%
(C) 60% per annum, what was Farhan’s share of the money
(D) 80% earned from the business venture? (Real NMAT
(E) 90% Question)
(A) Rs. 14,174
60. A 735 gm sample of a 16% (by weigh) solution of
iodine in alcohol is kept for three days. Some of the (B) Rs. 16,437
alcohol gets evaporated and the concentration of the (C) Rs. 17,000
solution becomes 20% (by weight). What amount of (D) Rs. 18,785
alcohol gets evaporated?
(E) Rs. 20,570
(A) 135 gm
65. The interest rate, compounded annually, that would
(B) 140 gm
bring a principal of Rs.1,200 to a final value of
(C) 147 gm Rs.1,650 in 2 years is approximately:
(D) 150 gm
(A) 17%
(E) 215 gm
(B) 18%
61. The price of a pair of shoes increases from Rs. 200 (C) 19%
to Rs. 600. What is the percentage increase in the (D) 20%
price? (Real NMAT Question)
(E) 21%
(A) 50%
66. An amount becomes 3 times in 6 years on a certain
(B) 100% rate of simple interest. In how many years it will
(C) 150% become 24 times?
(D) 200% (A) 48 Years
(E) 300% (B) 50 Years
62. An apple vendor sells 50% of his stock everyday. (C) 69 Years
10% of his stock gets spoiled overnight. After three (D) 70 Years
nights he calculates that a total of 3,305 apples have
(E) 72 Years
spoiled. How many apples did he start out with on

169

Book 1.indb 169 30/04/2019 4:47:11 PM


NMAT by GMAC™ Official Guide 2019

67. What is the value of the rate of interest if the 72. Parikshit invests Rs. 1546 in BNP bank at a certain
difference between the compound interests of the first rate of compound interest per annum. At the end of
and the second year is 4 times that of the principal? 8 years, he finds that his money has doubled. What
approximately is the rate of interest BNP bank paid him?
(A) 100%
(B) 300% (A) 9%
(C) 100% (B) 12%
(D) 200% (C) 15%
(E) 400% (D) 16%
(E) 18%
68. Some amount was divided into two equal parts. The
first part was invested at 10% per annum at simple 73. Rs. 16,820 is divided between Seeta and Geeta who
interest for 4 years. The second part was invested are 27 and 25 years old, respectively. Their money is
at 10% per annum at compound interest for 3 years. invested at 5% per annum compound interest in such
If the difference in the interests earned from the two a way that both receive equal money at the age of
investments is Rs. 1000, find the approximate value 40 years. Find the share of each out of Rs. 16,820.
of the total initial amount. (Real NMAT Question)
(A) Rs. 28,485 (A) Rs. 8,280 and Rs. 6,920
(B) Rs. 28,985 (B) Rs. 8,820 and Rs. 8,000
(C) Rs. 29,485 (C) Rs. 8,830.50 and Rs. 8,830.50
(D) Rs. 29,985 (D) Rs. 8,410 and Rs. 8,410
(E) Rs. 30,485 (E) Rs. 8,410 and Rs. 9,251

69. The value of a car depreciates at the rate of 10% per 74. If the interest on a sum of money is 15 paise per
annum. If its present value is Rs. 121500, then what rupee every quarter year, then what is the rate
was the value of the car two years ago? percent per annum?

(A) Rs. 100000 Note: 1 rupee = 100 paise (Real NMAT Question)
(B) Rs. 150000 (A) 15%
(C) Rs. 200000 (B) 30%
(D) Rs. 250000 (C) 45%
(E) Rs. 300000 (D) 60%
70. Ram lent Rs. 800 to a friend for 2 years and (E) 75%
one-fourth of this amount to another friend for 75. For what principal would the difference between the
3 years. He received Rs. 275 in total as simple compound interest and the simple interest at 11%
interest. What was the rate of interest? per annum for a period of 3 years be Rs. 37,631?
(A) 10.5% (Real NMAT Question)
(B) 12.5% (A) Rs. 4,13,941
(C) 15.5% (B) Rs. 1,36,730
(D) 17.5% (C) Rs. 5,00,000
(E) 19.5% (D) Rs. 10,00,000
71. At what interest rate per annum will a sum of money (E) Rs. 37,63,100
double itself in 8 years? 76. Sanya, Babli and Jhanvi started a new business.
1 Sanya’s capital was invested for a period which was
12
(A)
2 equal to four times Jhanvi’s period of investment
(B) 13% whereas Sanya and Babli invested for the same
(C) 15% period. Also, twice Sanya’s investment is equal to
Jhanvi’s investment, and Babli’s investment is equal
(D) 17%
1
(E) 19% to of Sanya’s investment. If the total year-end profit
2

170

Book 1.indb 170 30/04/2019 4:47:12 PM


3.0  Quantitative Skills Practice

from this business yielded Rs. 4,40,000, then what (A) 2.5
was the total share of Sanya and Babli in this profit? (B) 5
(Real NMAT Question)
(C) 10
(A) Rs. 2,40,000 (D) 15
(B) Rs. 2,50,000 (E) 20
(C) Rs. 2,55,000
(D) Rs. 2,60,000 81. A full glass of lemonade is a mixture of 20% lime
juice and 80% soda. The contents of the glass are
(E) Rs. 3,30,000
poured into a pitcher that is 200% bigger than the
77. Tina, Ishan, Abhishek and Fatima jointly started a glass. The remainder of the pitcher is filled with 16
business and invested a total of Rs. 80. If Tina’s litres of water. What was the original volume of lime
share increases by Rs. 3, Ishan’s share increases by juice in the mixture?
one-third of his share, Abhishek’s share decreases by
20% and Fatima’s share decreases by Rs. 4, all of (A) 1.6 litres
them would have equal amounts of money. What is (B) 3.2 litres
Fatima’s original share? (C) 4.8 litres
(A) Rs. 20.25 (D) 6.4 litres
(B) Rs. 23.50 (E) 8 litres
(C) Rs. 23.75 82. In a college dramatics team, the ratio of boys to girls
(D) Rs. 24.25 is 6 : 7. If there are 2 more girls than boys in the
(E) Rs. 24.75 team, how many boys are in the team?
78. Mukesh, Manish, Lalu and Jaggi bought a MOKIA (A) 12
mobile for £60. Mukesh paid one–half of the sum (B) 18
of the amounts paid by the other persons. Manish
(C) 24
paid one–third of the sum of the amounts paid by the
other persons. Lalu paid one–fourth of the sum of the (D) 30
amounts paid by the other persons. How much did (E) 36
Jaggi have to pay?
83. X cornflake is 55% fibre and Y cornflake is 70%
(A) £ 13 fibre. Sharad combines a certain amount of the two
(B) £ 15 cereals in a single bowl, creating a mixed cereal
(C) £ 17 that is 65% fibre. If the bowl contains 120 grams of
(D) £ 23 cereal, how much of the cereal, in grams, is X?
(E) None of these (A) 30
79. Yogesh and Mohan, two business partners, invest Rs. (B) 40
21000 and Rs. 17500 respectively in their garment (C) 60
business and at the end of the year both of them (D) 80
make a profit of Rs. 26400. Find their individual (E) 90
shares in the profit.
84. The ratio of the number of students in two
(A) Rs. 14400 and Rs. 12000 classrooms, C1 and C2, is 2 : 3. It is observed that
(B) Rs. 12000 and Rs. 14400 after shifting 10 students from C1 to C2, the ratio is
(C) Rs. 14000 and Rs. 12400 3 : 7. Further, how many students have to be shifted
(D) Rs. 14200 and Rs. 12200 from C2 to C1 for the new ratio to become 9 : 11?
(Real NMAT Question)
(E) none of these
(A) 10
80. A chemist is mixing a solution of ink and water. She (B) 15
currently has 30 litres of mixture solution, of which
10 litres are ink. How many litres of ink should the (C) 20
chemist add to her current mixture to attain a 50:50 (D) 25
mixture of ink and water if no additional water is (E) 30
added?

171

Book 1.indb 171 30/04/2019 4:47:12 PM


NMAT by GMAC™ Official Guide 2019

85. If a : b = b : c = c : d = 3, then find the value of was equal to the number of worksheets received by
d2 + c3 + b2c (Real NMAT Question) 3 students of Section C. How many worksheets did
cd + bc2 + abc each student of Section C receive?

1 Note: Assume that each student of a particular


(A) section received the same number of sheets. (Real
6
NMAT Question)
1
(B) (A) 1
3
(C) 3 (B) 2
(D) 6 (C) 3
(E) Cannot be determined (D) 4
86. In what ratio, solution X which contains 50% milk and (E) 5
solution Y which contains 30% milk be mixed so that 1
90. If the compound ratio of 8 : 5 and the inverse
the obtained solution contains 45% milk? 3
1 1
(A) 1 : 3 of 3 : 1 is 15 : x, then x is: (Real NMAT
5 3
(B) 2 : 3 Question)
(C) 3 : 2 (A) 20
(D) 3 : 5 (B) 40
(E) 3 : 1 (C) 60
87. A tank has a solution consisting of milk and water in (D) 80
equal proportion. This solution is transferred into a (E) None of the above
vessel having 100 ml pure water at the rate of 10 ml
per second. In how much time from the start of the 91. If a – b : b – c : c – d : d – e = 1 : 2 : 3 : 6, then
transfer, will the ratio of milk to water in the vessel a+e
what is the value of ? (Real NMAT Question)
be 1 : 3? d
(A) 5 seconds 2
(A)
(B) 10 seconds 7
(C) 15 seconds 2
(B)
5
(D) 20 seconds
3
(E) 25 seconds (C)
5
88. 20 tonnes of a 60:40 sand to cement mixture is 2
mixed with 30 tonnes of a 30:70 mixture. What is (D)
3
the total weight of cement in the final mixture? (Real
2
NMAT Question) (E)
1
(A) 20 tonnes
(B) 25 tonnes 92. Arvind sells clothes at a roadside market for which
he pays Rs. 150 per day to rent a table plus Rs. 10
(C) 27 tonnes
per hour to his salesman. He sells an average of Rs.
(D) 29 tonnes 78 worth of clothes per hour. Assuming no other
(E) 31 tonnes costs, which of the functions below best represents
profit per day P in terms of hours h that Arvind works
89. A teacher distributed 100 worksheets among the for?
students of three sections in a class. The worksheets
were distributed among 6 students of Section A, 12 (A) P(h) = 238 – 10h
students of Section B, and 17 students of Section C. (B)
P(h) = 72 – 10h
The number of worksheets received by 2 students
(C)
P(h) = 68h – 150
of Section A was equal to the number of worksheets
received by 5 students of Section C. The number (D)
P(h) = 78h – 160
of worksheets received by 2 students of Section B (E)
P(h) = –160h + 78

172

Book 1.indb 172 30/04/2019 4:47:12 PM


3.0  Quantitative Skills Practice

93. A batch of clips costs Rs. (p + 15) for a company to after 83 people have played the game? (Real NMAT
produce and each batch sells for Rs. p (9 –p). For Question)
which of the following values of p does the company
(A) Rs. 3,900
make a profit?
(B) Rs. 3,990
(A) 3
(C) Rs. 4,090
(B) 4
(D) Rs. 8,180
(C) 5
(E) Rs. 9,100
(D) 6
98. On selling 630 pens, a shopkeeper makes a
(E) 7
profit equal to the selling price of 90 pens. Find
94. A person purchased a smartphone for Rs. 8,000 the approximate profit percentage. (Real NMAT
and sold it at a profit of 25%. From that amount, he Question)
purchased another phone and sold it at a loss of
(A) 12.5%
20%. What is his overall profit or loss? (Real NMAT
Question) (B) 14.28%
(C) 16.67%
(A) profit of Rs. 2,000
(D) 20%
(B) profit of Rs. 1,000
(E) 22.22%
(C) loss of Rs. 2,000
(D) loss of Rs. 1,000 99. Two jars P and Q contain the same quantity of a
mixture of milk and water. The milk and water in P and
(E) neither profit nor loss
Q are in the ratio 5 : 2 and 4 : 1 respectively. What will
95. A shopkeeper claims a loss of 4% on his goods, but be the ratio in which these two mixtures have to be
uses weight equal to 840 gm instead of 1 kg. The blended to obtain a new mixture of milk and water in
shopkeeper actually makes a: the ratio of 3 : 1? (Real NMAT Question)

1 (A) 5 : 6
(A) 11 % gain
7 (B) 1 : 1
2 (C) 4 : 3
(B) 14 % gain
7 (D) 7 : 5
(C) 4% loss (E) 3 : 2
(D) 4% gain 100. A bus started driving towards the North from point
(E) 2% loss X, travelling at a constant rate of 40 km per hour.
An hour later, a truck started driving towards the
96. A product priced at Rs. 1000 would earn a North from the same point X at a constant rate of
shopkeeper a profit of 15%. Find the profit 30 km per hour. If each vehicle started with 8 litres
percentage earned by him if he decides to sell of diesel, that was consumed at a rate of 30 km
the product at a discount of Rs. 50 during the per litre, how many kilometres apart were the two
festivals. vehicles when the bus ran out of fuel?
(A) 3.34% (A) 30
(B) 9.15% (B) 60
(C) 9.25% (C) 90
(D) 9.30% (D) 120
(E) 9.50% (E) 150
97. Participation in a game show each time costs Rs. Directions for Question 101-104:  The following line chart
150. The 1st person wins Rs. 10, the second Rs. gives the distance covered, average speed and rest time
50, the 3rd Rs. 100 and the 4th Rs. 250. This cycle is for five different modes of transport used by Rajiv for five
repeated, with every 5th person winning Rs. 10, and different journeys. Here, the average speed is calculated
so on. How much profit has the game show made, without considering rest time. (Real NMAT Question)

173

Book 1.indb 173 30/04/2019 4:47:13 PM


NMAT by GMAC™ Official Guide 2019

(A) Bicycle and Car


Distance (km) Avg. speed (km/hr) Rest time (min)
(B) Bicycle and Bike
300 (C) Car and Bike
210 (D) Car and Train
250
195 (E) Train and Bike

105. Ramesh and Upendra start walking towards each other


60 45 90 40 75 from point A and point B respectively at the same
20
time. Before they meet each other, their speeds are
15 15 30 45 60 in the ratio 11 : 2. After meeting, Upendra increases
Car Bike Bicycle Van Train his speed so that his speed becomes ‘x’ times his
original speed and reaches point A ‘x’ hours earlier than
it takes for Ramesh to reach point B. Find the possible
101. Which of the following is the vehicle that took values of ‘x’. (Real NMAT Question)
the minimum travel time when considering the (A) 0 < x < 7.5
rest time?
(B) 7.5 < x < 15
(A) Bike (C) 15 < x < 22.5
(B) Bicycle (D) 22.5 < x < 30.25
(C) Car (E) 30.25 < x
(D) Train
106. Ram and Krishna moving towards each other met at
(E) Van
some point and reached the opposite ends at their
102. When travelling by bicycle, find the approximate destinations 10 minutes and 50 minutes respectively
percentage change in the time taken due after the time of meeting. What is the ratio of their
to rest. speeds?
(A) 11% (A) 5:1
(B) 12% (B) 1 : 25
(C) 13% (C) 25 : 1
(D) 14% (D) 1 : 5
(E) 15% (E) None of these
103. Which of the following is the approximate average 107. The speed of two persons P1 and P2 are related
speed across the given modes of transport, without
S12 + S22 a2 + b2
considering the rest time? as = , where a and b are unknown
S12 − S22 a2 − b2
(A) 36 km/hr
constants. Find the ratio of t1 and t2 where t1 and t2
(B) 40 km/hr are the time taken by P1 and P2 to cover the same
(C) 42 km/hr distance at speeds S1 and S2 respectively.
(D) 46 km/hr a+b
(A)
(E) 56 km/hr b
104. Which of the following combinations represents b
(B)
minimum travel time (without considering rest a
time)? 2a
(C)
Note: Consider half-way journey of each travel mode b
given in every option. For example, travel time a
(D)
for ‘Car and Bike’ can be calculated considering b
half the distances which have been given for car a+b
and bike. (E)
a−b

174

Book 1.indb 174 30/04/2019 4:47:15 PM


3.0  Quantitative Skills Practice

108. Two trains Ranipur Mail and Dhampur Mail start from 113. Train 1 travelled from Station 1 to Station 2 and Train
Ranipur to Dhampur and from Dhampur to Ranipur 2 travelled from Station 2 to Station 1. Both the trains
respectively. After passing each other, they take 4 started at the same time. After meeting each other,
hours 48 minutes and 3 hours 20 minutes to reach Train 1 took 4 hours to reach Station 2 and Train 2
Dhampur and Ranipur respectively. If Ranipur Mail is took 9 hours to reach Station 1. Train 1 travelled at
moving at 45 km/hr, the speed of Dhampur Mail is - a speed of 48 km/h. If both the trains travelled along
parallel tracks and the distance between Station 1 and
(A) 37.5 km/hr
Station 2 is 480 km, then after 5 hours of journey,
(B) 54 km/hr what will be the ratio of the distance of Train 1 from
(C) 58 km/hr its destination to the distance of Train 2 from its
(D) 60 km/hr destination? (Real NMAT Question)
(E) 64.8 km/hr (A) 1 : 3
109. The ratio of speeds of Amar and Jafar is 2:3 and (B) 1 : 2
therefore Amar takes 10 minutes more than Jafar to (C) 3 : 5
reach the destination. If Amar travelled at double the (D) 2 : 3
speed, Amar would have covered the distance in
(E) 3 : 4
(A) 15 minutes
114. After travelling a distance of 180 km, a train met with
(B) 20 minutes
3
(C) 25 minutes an accident and then travelled at of its original
4
(D) 27 minutes speed. It was 1 hour late in reaching its destination.
(E) 30 minutes If the accident had occurred a further 60 km on from
the place of the accident, the delay would have been
110. Patna and Ranchi are the two places on the same only 30 minutes. What is the total distance travelled
side of the bank of a uniformly flowing river. Ankur by the train? (Real NMAT Question)
takes 1 hour to row from Patna to Ranchi and 5
hours from Ranchi to Patna. Find the ratio of speeds (A) 200 km
of the boat in the still water to that of the river. (B) 300 km
(A) 3 : 2 (C) 400 km
(B) 5 : 2 (D) 500 km
(C) 4 : 3 (E) 600 km
(D) 2 : 1 115. The time it takes to construct a hut is inversely
(E) 5 : 4 proportional to the number of workers doing the
work. If it takes 40 workers giving 3 hours each to
111. In a 200 m linear race, if Sumit can beat Vinit do the job, how long will it take for 140 workers to
and Hari by 20 m and 30 m respectively. By what do the job, to the nearest minute?
distance can Vinit beat Hari in a 3600 m linear race?
(A) 51 minutes
(A) 160 m (B) 52 minutes
(B) 180 m (C) 53 minutes
(C) 200 m (D) 54 minutes
(D) 220 m (E) 55 minutes
(E) 250 m
116. A ski resort has enough wood to keep 20 rooms heated
112. Puja travels three different distances 10 km, 20 km, for 14 days. If the resort decides to save wood by
and 30 km at a speed of 1/2 km/hr, 1/3 km/hr and turning off the heat in 5 unoccupied rooms, and each
1 km/hr respectively. What is the ratio of the time room requires the same amount of wood to heat it, how
taken for the distances given? many extra FULL days will the wood supply last?
(A) 2:6:3 (A) 3
(B) 3:2:6 (B) 4
(C) 1:2:3 (C) 5
(D) 2:3:6 (D) 18
(E) 1:3:6 (E) 19
175

Book 1.indb 175 30/04/2019 4:47:15 PM


NMAT by GMAC™ Official Guide 2019

117. Working alone at their respective constant rates, Ajay 121. Two taps can separately fill a tank in 4 minutes and
can complete a certain job in 4 hours, while Firoz can 5 minutes respectively. Due to a small hole at the
do the same job in 3 hours. Ajay and Firoz worked bottom of the tank, the two taps together take 30
together on the job and completed it in 2 hours, but seconds more time to fill the tank. The hole can
while Ajay worked this entire time, Firoz worked for empty the completely filled tank in
some of the time and took 3 breaks of equal length.
980
How many minutes long were each of Firoz’ breaks? (A) minutes
71
(A) 5 minutes
980
(B) 10 minutes (B) minutes
81
(C) 15 minutes
980
(D) 20 minutes (C) minutes
91
(E) 25 minutes
980
118. A machine can manufacture 20 pens per hour, and (D) minutes
61
exactly 10 such pens fit into every box. Mahesh
packs pens in boxes at a constant rate of 3 boxes 981
(E) minutes
per hour. If the machine ran for 2 hours and was then 51
turned off before Mahesh started packing the pens 122. Three taps P, Q and R when filling together can fill a
in boxes, how many minutes would it take Mahesh to cistern in 3 hours. After 1 hour tap P is closed and
pack all the pens that the machine had made? the cistern is filled in 4 more hours. Find the time in
which tap P alone can fill the cistern?
(A) 40 minutes
(B) 45 minutes (A) 3 hours
(C) 80 minutes (B) 4 hours
(D) 160 minutes (C) 5 hours
(E) 800 minutes (D) 6 hours
(E) 7 hours
119. A tank can be filled by Pipe 1 in 7 hours and by
Pipe 2 in 5 hours. There is a waste pipe which is kept 123. Pipes P and Q can fill a tank in 12 minutes and 16
open when Pipe 2 is working; the tank then takes minutes respectively. Both are kept open for X minute(s)
8 hours 30 minutes to fill. What is the approximate and then Q is closed and P fills the rest of the tank in 5
time taken to fill the tank if all the three pipes are minutes. The time X after which Q was closed is –
working? (Real NMAT Question)
(A) 2 minutes
(A) 3 hours 20 minutes (B) 3 minutes
(B) 3 hours 50 minutes (C) 4 minutes
(C) 4 hours 15 minutes (D) 6 minutes
(D) 4 hours 50 minutes (E) 7 minutes
(E) 5 hours 30 minutes
124. Pipe X pours a mixture of acid and water, and pipe
120. Harsh alone can complete 2/3 of a coding project in
rd Y pours pure water into a bucket. After 1 hour, the
6 days. Sumit alone can complete 1/3rd of the same bucket got filled and the concentration of acid in the
project in 8 days and Mini can complete 3/4th of the bucket was noted to be 8%. If pipe Y was closed
same work in 12 days. All of them started coding after 30 minutes and pipe X continued to pour the
together. After 4 days, Harsh and Mini left the project. mixture, concentration of acid in the bucket after 1
How many more days will be required by Sumit to hour would have been 10%. What is the ratio of acid
complete the project? (Real NMAT Question) to the water in the mixture coming out of pipe X?

(A) 2.33 days (A) 13: 2


(B) 2.67 days (B) 2 : 15
(C) 3.33 days (C) 3 : 20
(D) 3.67 days (D) 1 : 5
(E) 4.33 days (E) 2 : 13

176

Book 1.indb 176 30/04/2019 4:47:15 PM


3.0  Quantitative Skills Practice

125. Two taps P and Q can fill a cistern in 12 minutes and 2


129. A and B can do a piece of work in 22 days, B and
18 minutes respectively. If both the taps are opened 9
together, how long it take to fill the cistern? 2
C can do it in 16 days, A and C can do the work in
3
5
(A) minutes 5
36 15 days. Arrange the three in decreasing order of
13
1
(B) 5 minutes productivity.
5
1 (A) C>A>B
(C) 6 minutes (B) B>C>A
5
(C) A>C>B
1
(D) 7 minutes (D) C>B>A
5
(E) B>A>C
(E)
None of these
130. A alone can complete a task in 10 days. B can
126. There are 12 workers who have been recruited to complete the same task in 15 days. If A and B work
dig a 20 km long tunnel. It takes one worker to dig together, how much time will it take to complete the
250 m of tunnel in a week. How many more workers same task? (Real NMAT Question)
are needed to complete the work in 2 weeks?
(A) 3 days
(A) 12 (B) 5 days
(B) 18 (C) 6 days
(C) 20 (D) 9 days
(D) 24 (E) 10 days
(E) 28
131. The daily wage of a skilled mason is double the daily
127. If 33 unskilled workers can do a work in 15 days of wage of an unskilled mason. If 45 unskilled masons
12 hours each, how many skilled workers can do get a wage of Rs. 15,525 in 48 days, how many
50% more work in 11 days of 9 hours each? (Assume skilled masons should work for 16 days to earn Rs.
that it takes 2 skilled workers to do the work of 5 5,750? (Real NMAT Question)
unskilled workers.) (A) 18
(A) 36 (B) 20
(B) 42 (C) 25
(C) 64 (D) 30
(D) 90
(E) 32
(E) 100
132. Harish is twice as productive as Girish and together
128. Ajay finishes a work in certain number of days. He they finish a project in 18 days. In how many days
3 will Harish alone finish the project? (Real NMAT
got two assistants who work as fast as him. If Question)
4
all three work together, then in what fraction of (A) 20 days
time would they finish the job as compared to Ajay (B) 21 days
working alone?
(C) 24 days
5 (D) 27 days
(A)
3
(E) 30 days
3
(B)
5 133. Nishit works twice as fast as Pradeep. Nishit and
Pradeep together can work three times faster than
2
(C) Bhuvan. If Nishit, Pradeep and Bhuvan together work
3 on a job, in what ratio should they share the earnings?
2
(D)
5 Note: The ratio of individual share of earnings for all
3 3 workers is the same as the ratio of their relative
(E) efficiencies. (Real NMAT Question)
2
177

Book 1.indb 177 30/04/2019 4:47:17 PM


NMAT by GMAC™ Official Guide 2019

(A) 2 : 1 : 1 138. How many times do the hands of a clock coincide in


(B) 2 : 3 : 1 a day?
(C) 4 : 2 : 1 (A) 20
(D) 4 : 2 : 3 (B) 21
(E) 4 : 3 : 2 (C) 22
134. A water tank had 3 taps. The first tap could fill the (D) 23
tank in 10 minutes and the second tap could fill the (E) 24
tank in 15 minutes. When all the 3 taps were opened
simultaneously, the tank was filled in 22 minutes. In 139. How many times are the hands of a clock at right
how many minutes did the third tap fill or empty the angles in a day?
tank? (Real NMAT Question) (A) 22
(A) 6.50 (B) 24
(B) 8.25 (C) 44
(C) 9.27 (D) 48
(D) 15.67 (E) None of these
(E) 18.50
140. At what time between 2 O’clock and 3 O’clock are
135. A group of 20 software engineers can finish a the two hands of a clock together?
development project in 30 days. They got a project
(A) 2:00 10/11 O’clock
with the deadline of 35 days. After how many days
should 5 engineers be released so that the project (B) 2:05 10/11 O’clock
completes on time? (Real NMAT Question) (C) 2:10 10/11 O’clock
(A) 5 (D) 2:15 10/11 O’clock
(B) 8 (E) 2:20 10/11 O’clock
(C) 10 141. If August 15, 1947 was a Friday, then, what was the
(D) 12 day on January 26, 1950?
(E) 15 (A) Thursday
136. What will be the angle (in degree) between the minute (B) Friday
hand and hour hand of an analog clock when the (C) Saturday
clock shows 5 hours and 25 minutes?
(D) Sunday
(A) 10° (E) Monday
(B) 12.5°
(C) 30° 142. What was the day on April 20, 1984?

(D) 42.5° (A) Thursday


(E) None of these (B) Friday
(C) Saturday
137. An analog clock which gains 5 seconds every 3
minutes was set right at 7 AM. What is the true time (D) Sunday
in the afternoon of the same day, when the clock (E) Monday
indicated quarter past 4 o’clock?
143. If in a certain year, the month of January had exactly
7 4 Wednesdays and 4 Sundays, then January 1 of that
(A) 59 minutes past 3
12 year was a -
7 (A) Saturday
(B) 58 minutes past 3
11
(B) Monday 
(C) 4:00 PM (C) Wednesday
(D) 4:30 PM (D) Friday
(E) None of these (E) Thursday

178

Book 1.indb 178 30/04/2019 4:47:18 PM


3.0  Quantitative Skills Practice

144. Puja born in 1900s realised that in 1980 his age was
the square root of the year of her birth. When was
Puja born?
(A) 1929
(B) 1936
(C) 1940
(D) 1946
(E) 1949

145. If 09.12.2001 happens to be a Sunday, then


09.12.1971 would have been a
(A) Saturday
(B) Monday 
(C) Wednesday
(D) Friday
(E) Thursday

179

Book 1.indb 179 30/04/2019 4:47:18 PM


NMAT by GMAC™ Official Guide 2019

2. Algebra

(A) 5a2
(B) 5a2 + 10
1. Manish has 60 marbles that he wants to divide a2 + 10
(C)
among himself and his 12 friends. The marbles don’t (D) 5a2 + 6a + 10
necessarily have to be divided equally. If Manish (E) 5a2 – 6a + 10
wants to have more marbles than any of his friends,
what is the least number of marbles he can have? 6. a is inversely proportional to b. Also, it is given that a
= 24 when b = 2. What is the value of b when a = 6?
(A) 5
(A) –2
(B) 6
(B) –1
(C) 7
(C) 2
(D) 8
(D) 4
(E) 12
(E) 8
2. If a - b = 0 and ab ≠ 0, , which of the following
2 2
7. Three people sit down to eat 14 pieces of cake. If two of
must be true? Indicate all such statements. the people eat the same number of pieces, and the third
1. a = b person eats two more pieces than each of the other
two, how many pieces are eaten by the third person?
2. a = b
(A) 3
a2 (B) 4
3. 2 = 1
b (C) 5
(A) 1 only (D) 6
(B) 2 only (E) 7
(C) 3 only 8. There are a number of beads of three different
(D) 1 and 2 only colours: red, blue and yellow, and each colour has a
(E) 2 and 3 only different value. If the value of a red bead plus a blue
bead is 4.25, the value of a blue bead plus a yellow
3. If ( x - y ) = 20 and ( x + y ) = 12 what is the value bead is 2.75, and the value of a red bead plus a blue
bead plus a yellow bead is 4.5, what is the value of a
of x2 – y2 ? red bead plus a yellow bead?

(A) (A) 0.25


2 15
(B) 2
(B)
4 15
(C) 2.25
(C)
3 20 (D) 2.75
(D)
6 12 (E) 3
(E)
2 21 9. National Cricket Academy offers two different pricing
packages for cricket coaching. Under the ‘Regular’
a 8 - b8 pricing plan, classes can be bought for a flat rate of
4. If ab ≠ 0, =
( )( )
a 4 + b 4 a 2 + b2 Rs. 80 per hour. Under the ‘Exclusive’ pricing plan,
after paying an initial fee of Rs. 495, classes can be
(A) 1 availed for a rate of Rs. 15 per hour. If Karan buys
(B)
a–b the ‘Exclusive’ pricing plan, how many classes does
he need to take in order to have spent exactly 40%
(C) (a + b) (a – b) less than he would have under the ‘Regular’ plan?
(D) (a2 + b2) (a2 – b2)
(A) 10
a -b (B) 12
(E) 2 2
a+b (C) 15
5. Which of the following is equal to (a – 2)2 + (a – 1)2 + (D) 18
a2 + (a + 1)2 + (a + 2)2 ? (E) 20

180

Book 1.indb 180 30/04/2019 4:47:19 PM


3.0  Quantitative Skills Practice

10. A student took a test in which 3 marks were given 1 1 1 1 1


for each correct answer and 0.5 marks were 14. If + = m and pq = , find 2 + 2
p q n p q
deducted for an incorrect answer. If the test had
25 questions and the student attempted all the 1 2
(A) −
questions and got 40 marks in total, what is the m2 n
difference between the number of correct and
incorrect answers? 2
(B) m2 –
n
(A) 5
(B) 10 (C) m2 – 2n
(C) 12 1
(D) – 2n
(D) 15 m2
(E) 18 1
(E) + 2n
11. A group of friends contributed to the cost of a m2
party where each person had to contribute the
same integer amount. Since three people did not 15. Let f(x + 2) + f(5x + 6) = 2x – 1 for all real x. Find
participate, the remaining people had to pay Rs. the value of f(1).
10 more. If the total amount contributed is the (A) –2
minimum value possible, what would be the per
(B) –1
person contribution had 10 people contributed to
the party? −5
(C)
(A) Rs. 2 2
(B) Rs. 5 −3
(D)
(C) Rs. 6 2
(D) Rs. 8 (E) None of these
(E) Rs. 10 16. For what value of K, the given set of equations would
12. If p + q + r = 0, where a ≠ b ≠ c, then have no solution?

p2 q2 r2 4x – Ky = –7 and 5x + 3y = 2
+ + =
2p2 + qr 2q2 + pr 2r2 + pq 12
(A)
5
(A) 0
(B) 0
(B) 1
−12
(C) – 1 (C)
5
(D) pqr
(E) p + q + r −6
(D)
5
13. Out of a group of swans, seven times half of the 6
square root of the number of swans were seen (E)
5
going away from a bank of a river and only one pair
remained in the water. How many swans were there Directions for Questions 17 and 18:  Answer the
in the group? questions based on the following.
(a) 9 The following operations are defined for real numbers.
(B) 16 A @ B = A if A is greater than B else A @ B = B
(C) 25 A % B = AB if A x B is positive else A % B = A
(D) 36 Note that all other mathematical symbols have their usual
(E) 49 meanings.

181

Book 1.indb 181 30/04/2019 4:47:21 PM


NMAT by GMAC™ Official Guide 2019

17. [( – 4)@( – 5)]%2 22. If p and q are roots of x2 + 7x + 12 = 0, then the


equation whose roots are (p + q)2 and (p – q)2 is
(A) – 8
(B) – 10 (A) x2 – 50x + 49 = 0
(C) – 5 (B) x2 + 50x – 49 = 0
(D) – 4 (C) x2 – 10x + 3 = 0
(E) – 7 (D) x2 – 10x + 4 = 0
(E) x2 – 50x – 49 = 0
1@ − 1
18. %K, K ≠ 0 23. Which of the following could be the quadratic
( ) @ ( −K )
− K
1
(A) K2 equation for which one root is 1 times the other
2
1 root and the difference between the roots is 1?
(B)
K
(A) x2 + 3x + 3 = 0
−1
(C) (B) x2 + 4x + 3 = 0
K
(C) x2 – 5x + 6 = 0
(D) 1
(D) x2 + x – 6 = 0
(E) Cannot be determined
(E) x2 – 3x – 3 = 0
19. If 3x 3 - 7 = 185 , what is x 2 - x ? 24. If x2 – 6x + 9 = 0, what is the value of x3? (Real
(A) –4 NMAT Question)
(B) 8 (A) – 27
(C) 12 (B) – 9
(D) 16 (C) 6
(E) 27 (D) 9
20. If the roots of the equation ax + bx + c = 0 are
2 (E) 27
reciprocal of the roots of the equation px2 + qx + r 25. If p and q are the roots of the equation ax2 + bx + c
= 0, then which of the following represents = 0, then what is the value of p4 – q4? (Real NMAT
relation(s) between a, b, c, p, q and r? (Real Question)
NMAT Question)
a 4 - b 4 - c4
(A)
1
a=
(A)
p a 4 + b 4 - c4
(B)
1 b
b=
(B)
q
± 4 b2 - ac
(C)
a
( )
b
c=
(C)
1
r
± 4 b2 - 2ac
(D)
a
( )
(D)
a = p, c = r and b = 1 b
(E)
a = r, c = p and b = q
± 4 b2 - 2ac
(E)
a
( ) b2 - 4ac

21. If a, b and c are the three positive integers in 26. If the sum of the roots of (a + 1)x2 + (2a + 3)x + (3a
geometric progression, then the roots of the + 4) = 0 is –1, then what is the product of the roots?
equation ax2 + 4bx + 2c = 0 are – (Real NMAT Question)

(A) Imaginary (A) 0


(B) Equal (B) 1
(C) Rational (C) 2
(D) Real (D) 3
(E) Irrational (E) 4

182

Book 1.indb 182 30/04/2019 4:47:23 PM


3.0  Quantitative Skills Practice

27. If 2(x – 1)3 + 3 ≤ 19, then the value of x must be: (A) 26
(A) greater than or equal to 3 (B) 27
(B) less than or equal to 3 (C) 28
(C) greater than or equal to –3 (D) 29
(D) less than or equal to –3 (E) 30
(E) less than –3 or greater than 3 32. What is the minimum value of
12
28. If 3a + 7 ³ 2a + 12 , then æ 2401 1 1 ö , where x > 0?
çx + 2401 + 2 ´ x 2401 ´ 2401 ÷
è x x ø
19
a£-
(A) (Real NMAT Question)
5
19 (A) 2
a³-
(B)
5 (B) 4
a ³5
(C) (C) 16
19 (D) 49
a£-
(D) or a ³ 5
5 (E) It cannot be determined.
19
-
(E) £a £5
5 33. If log102 = 0.3010, what is the value of log5256?
(Real NMAT Question)
x
29. If x is an integer and 192 < x 12 and < 12,
12 (A) 3.11
which of the following can be the value of x? (B) 3.26

(A) 2 (C) 3.44

(B) 3 (D) 3.67

(C) 4 (E) 3.82

(D) 5 34. Find the value of ‘x’ if


(E) 12
625log366 + 12log749 = 11logx169
a x
30. If and are reciprocals, then, which of the (A) 10
b y
(B) 11
following must be true?
(C) 13
(A) ab < 0 (D) 17
a x (E) 19
(B) < -1
b  y 
35. If log30 3 = x and log30 5 = y, then find the value of
a
(C) <1 log8 30.
b
a y (A) 3(1 – x – y)
(D) =-
b x
1
y a (B)
(E) > 3(1 - x - y)
x b
3
31. It costs a certain chair manufacturing unit (C)
Rs. 11,000 to operate for one month, plus Rs. 300 (1 - x - y)
for each chair produced during the month. Each of (1 - x - y)
the chairs sells for a retail price of Rs. 700. What is (D)
3
the minimum number of chairs that the manufacturing
unit must sell in one month to make a profit? (E) None of these

183

Book 1.indb 183 30/04/2019 4:47:25 PM


NMAT by GMAC™ Official Guide 2019

36. If ap = bq = cr = ds, then find the value of loga (bcd).


 1 1 1
p + + 
(A)
 q r s
(B) 1
1 1 1
(C) + +
q r s
p
(D)  1 1 1
 q + r + s 

 1 1 1
 + + 
(E)  q r s 
p
37. If logy x = 10, then find the value of logx3 y 6
1
(A)
6
1
(B)
5
(C) 5
(D) 6
1 1
(E) Both and
6 5
2 5
38. If P = , then find the
625log3 25 + 25log125 27 + 5log8 125
value of ‘P’.
(A) 90
(B) 92
(C) 122
(D) 136
(E) 154

184

Book 1.indb 184 30/04/2019 4:47:26 PM


3.0  Quantitative Skills Practice

3. Geometry 5. ABC is a triangle and P is a point inside it such that


∠BPC = ∠CPA = ∠APB. Then P is always

1. A motorcycle tyre has spokes that go from a centre (A) the centroid of the triangle
point in the hub to equally spaced points on the rim of (B) the incentre of the triangle
the wheel. If there are fewer than six spokes, what is (C) the circumcentre of the triangle
the smallest possible angle between any two spokes?
(D) the orthocentre of the triangle
(A) 18° (E) None of these
(B) 30°
6. A, B and C are the angles of a ∆ABC. If A = 4B, then
(C) 40°
(D) 60° (A) B is always less than 30°
(E) 72° (B) B is always less than 36°
(C) B is always less than 45°
2. What is the value of a + b + c + d?
(D) B is always less than 50°
(E) None of the above is necessarily correct

7. In a circle, chords ST and BC bisect each other
internally at P, where ST = 8 units. Find the area of
the isosceles ∆ABC, where A lies on the circle.

b° d° (A) 64 sq. units

(B) 16 3 sq. units

(C) 64 3 sq. units


a° 40 °
(D) 16 sq. units
(E) 48 sq. units
(A) 240°
(B) 320° 8. If the biggest possible right-angled triangle is cut out
(C) 360° from a circle of radius 10 cm, what will be its area
(in cm2)
(D) 500°
(E) 540° (A) 96
(B) 100
3. Square 1 is inscribed in circle 1 and then another
circle 2 is inscribed in square 1. Then another (C) 112
square 2 is inscribed in circle 2 and so on. What is (D) 141
the diameter of the circle 5 if the radius of circle 1 is (E) None of these
50 cm?
9. In ∆ABC, AB = AC and ∠ABC = 40°. If O is the
(A) 25 cm centre of the circle inscribed in ∆ABC, then find angle
(B) 25√2 cm OAB.
(C) 50 cm (A) 20°
(D) 50√2 cm (B) 50°
(E) None of these (C) 70°
4. For how many triplets (A, B and C) the identity (D) 80°
sin2A + sin2B = sin2C is possible, where A, B and C (E) 100°
are the three interior angles of ∆ABC.
10. The distance between two walls is 64 feet. A 65 feet
(A) 3 ladder is resting on wall A and the tip of the ladder
(B) 6 touches the wall at a point 60 feet above the ground.
(C) 12 When the ladder is moved without shifting its feet
and is made to rest on wall B, the tip of the ladder
(D) 15
touches wall B at a point ‘x’ feet above the ground.
(E) Infinite What is the value of ‘x’? (Real NMAT Question)
185

Book 1.indb 185 30/04/2019 4:47:27 PM


NMAT by GMAC™ Official Guide 2019

(A) 32 (A) 22.5π - 25


(B) 36 (B) 22.5π - 50
(C) 42 (C) 45π - 25
(D) 48 (D) 45π - 50
(E) 52 (E) 45π - 100
11. What is the area of a regular hexagon whose 15. In the figure below, the two small circles are identical.
perimeter is 12 inches? (Real NMAT Question) What is the radius (in cm) of each small circle, if the
216 3 in2
(A) side of the square is 2 cm? (Real NMAT Question)
(B)
36 3 in2

(C) 12 3 in2
(D)
6 3 in2
(E)
4 3 in2

12. Find the sum of the interior angles of a nine-sided


regular polygon. (Real NMAT Question)
(A) 20°
(B) 360°
(C) 810°
(D) 1260° (A) 2 -
2 - 2 1+ 2
(E) 1620°
2 + 2 + 2 1+
(B) 2
13. When the breadth of a rectangle is increased by (C)
2 - 2 + 2 1+ 2
100%, the perimeter increases by 10 units whereas
when the length of the rectangle is reduced by 50%, (D)
2 + 2 - 2 1+ 2
the perimeter of the rectangle is reduced by 10
units. What is the perimeter of the rectangle? 2+ 2 -2
(E) 2-1
(A) 20 units 16. In the given figure, the two circles are concentric at
(B) 30 units centre O. SQ, SU are tangents to the smaller circle
(C) 40 units and QP is a tangent to the bigger circle. Also, SQ =
SU. If ∠PQR = 80o , what is the measure of ∠RST ?
(D) 50 units
(Real NMAT Question)
(E) 60 units
P
14. In the given figure, O is centre of the larger circle of
Q
radius 10 cm while P is the centre of the smaller circle. R
If AB = 10√2 cm, find the area of segment ABC lying
outside the smaller circle. (Real NMAT Question) S O V

T
U

(A) 20°
O
(B) 40°

(C) 60°

A B (D) 80°
P
(E) 100°

186

Book 1.indb 186 30/04/2019 4:47:29 PM


3.0  Quantitative Skills Practice


17. Find the image of (3, 12) with respect to the line civilisation. If the ruler at that time had a head that
12x – 5y + 12 = 0. (Real NMAT Question) was 10-inches long, a fence that measured 600 feet
at that time would today have a length of how many
(A) (–3, –12)
feet? (Today, 12 inches = 1 foot)
(B) (12, 3)
(A) 300
(C) (12, –5)
(B) 450
 12 -5 
(D)
 12 , 12  (C) 500
(D) 750
 795 1908 
(E)
 169 , 169  (E) 800
21. A 12 inch × 16 rectangular picture is displayed on

18. If the area of triangle ABC is 1 cm2, what is the area an 18 inch × 30 inch rectangular frame. What is
of the quadrilateral AMGN where G is the centroid of the area of the part of the frame not covered by the
the triangle ABC? (Real NMAT Question) picture?
A (A) 150 square inches
(B) 244 square inches
(C) 264 square inches
N
M G (D) 348 square inches
(E) 384 square inches

22. A solid cube with side length of 2 feet is cut into 2


inch x 2 inch x 4 inch cuboids. What is the ratio of

B O
C the total surface area of all the resulting cuboids to
the surface area of the original cube? (1 foot = 12
1 inches)
(A) cm2
6 (A) 2 : 1
1 (B) 4 : 1
(B) cm2
5 (C) 5 : 1
1 (D) 8 : 1
(C) cm2
4
(E) 10 : 1
1
(D) cm2 23. A sector of a circle having radius of 4 units has an
3 area of 4 π square units. What is the arc length of
1 the sector?
(E) cm2
2 (A)
π units
19. An equilateral triangle has a side length of p units. (B) 2 π units
Another equilateral triangle is inscribed in the above
(C) 4 π units
triangle by joining the midpoints of the sides of the
triangle and this process is continued infinitely. Find (D) 6 π units
the value of p, if the sum of the areas of all such (E) 8 π units
triangles is 36 3 if units. (Real NMAT Question)
24. Let a right circular cylinder’s radius be r and height
(A) 6 be h. What will be the change in volume of this
r
(B)
6 2 cylinder if r becomes and h becomes 2h?
2
(C)
6 3
(A) 50% decrease
(D)
7 3
(B) No change
(E)
8 3 (C) 25% increase
20. In an ancient civilisation, the measure of a foot (D) 50% increase
was the size of the head of the current ruler of the (E) 100% increase

187

Book 1.indb 187 30/04/2019 4:47:31 PM


NMAT by GMAC™ Official Guide 2019

25. A conical cap just covers two spheres placed one 30. A cylinder, a hemisphere and a cone have identical
above the other on a table. If the radii of the spheres base and same height. The area of their curved
1 surfaces will be in the ratio -
are 1” and 2 ” respectively, what is the height of
4 (A) 1:1: 2
the cone? (Real NMAT Question)
(B)  2 : 2 :2
(A) 5.8”
(B) 6.2” (C) 2 : 2 :2
(C) 7.5”
(D) 8.1” (D)  2 : 2 :1
(E) 9.3”
(E) 2 : 1: 2
26. Find the surface area of the biggest cube that can 31. What is the total surface area of a cuboid whose
completely fit inside a sphere of radius 10 cm. length, breadth and height are 10 cm, 15 cm and 20
(A) 125 sq. cm cm respectively?
(B) 200 sq. cm (A) 1000 cm2
(C) 600 sq. cm (B) 1100 cm2
(D) 400 sq. cm (C) 1200 cm2
(E) 800 sq. cm (D) 1300 cm2
(E) 1400 cm2
27. Find the ratio of the volume of a sphere to the
volume of a cone if the radius of the sphere is the
32. How many parallelograms each with a base of
same as radius of the base of the cone and the 15 cm and heights of 5 cm, 10 cm, 15 cm and so
height of the cone is the same as the diameter of the on, will have a total area of 4,125 cm2? (Real NMAT
base. Question)
(A) 2 : 1 (A) 6
(B) 1 : 2 (B) 9
(C) 4 : 3 (C) 10
(D) 3 : 4 (D) 12
(E) 3 : 7 (E) 14
28. Find the number of spherical balls, each of diameters 33. The area of a semicircle of radius ‘r’ is 3 times more
1 mm that can be made from a solid sphere of radius than that of the area of a circle of radius ‘R’. What is
4 cm. R : r? (Real NMAT Question)
(A) 512 (A) 1 : 3
(B) 5120 (B) 3 : 1
(C) 25600 (C)
1: 6
(D) 51200
(D)
2 2 :1
(E) 512000
(E)
1: 2 2
29. What is the volume of the smallest cube that
is needed to enclose a cuboid of dimensions
10 cm × 5 cm × 2 cm, such that the cuboid can be
placed inside the cube in any possible orientation?
(A) 100 cm3
(B) (125)3/2 cm3
(C) (129)3/2 cm3
(D) (100)3/2 cm3
(E) (139)3/2 cm3

188

Book 1.indb 188 30/04/2019 4:47:32 PM


3.0  Quantitative Skills Practice

4. Modern Math

(C) 64
(D) 81
1. For a cricket match team selection, 2 batsmen, (E) None of these
3 bowlers, and 1 wicketkeeper are to be picked.
6. There are 6 equally spaced points A, B, C, D, E and
There are 23 players available to play as batsmen,
F marked on a circle whose radius is R. How many
21 other players available to play as bowlers, and
convex pentagons of distinctly different areas can be
9 other players available to play as wicketkeepers.
drawn using these points as vertices?
If the maximum possible number of complete sets
of 6 players are formed, how many of the available 6
(A) P5
players will not be on a team? 6
(B) C5
(A) 7 (C) 5
(B) 9 (D) 1
(C) 11 (E) 6
(D) 13
7. One of the management test papers comprises of
(E) 15 9 questions divided equally among three sections,
2. How many five-digit numbers can be formed namely section I, section II and section III. There are
using the digits 5, 6, 7, 2, 9, 0 if no digits can be fifteen different questions available such that there
repeated? are five questions for every section for designing the
test. If no two sections bear a common question,
(A) 64 then how many different tests can be designed?
(B) 120
(A) 480
(C) 240
(B) 640
(D) 600
(C) 800
(E) 720
(D) 880
3. Five friends, Akshita, Binod, Chetan, Dravid, and (E) 1000
Eshan are to be arranged in a line. How many such
arrangements are possible if Binod is not allowed to 8. A shop sells 5 different types of sweets. In how many
stand next to Dravid? different ways a total of 8 sweets can be purchased?
(A) 24 (A) 125
(B) 48 (B) 495
(C) 72 (C) 795
(D) 96 (D) 840
(E) 120 (E) 930
4. How many 5 digit numbers can be formed using the 9. A box contains 90 balls of different colours: 13
digits 1, 2, 3, 4, 5 and 6 (without repetition) that are yellow, 19 green, 27 red, 10 black, 7 brown and 14
divisible by 8? (Real NMAT Question) white. Find the smallest number V such that any V
(A) 56 balls drawn from the box will contain at least 14 balls
of the same colour.
(B) 64
(C) 72 (A) 69
(D) 84 (B) 70
(E) 96 (C) 72
(D) 76
5. Among three different boxes, 10 identical balls have
(E) 79
to be distributed. In how many ways can this be done
such that every box has at least 2 balls? 10. Salim has total 9 friends, 5 girls and 4 boys. In how
(A) 15 many ways can Salim invite them for his birthday
party, if there have to be exactly 3 girls in the
(B) 16
invitees list?

189

Book 1.indb 189 30/04/2019 4:47:32 PM


NMAT by GMAC™ Official Guide 2019

(A) 80 (C) 1 × 3 × 5 × ... × (2n + 1)(2n)


(B) 160 (D) 1 × 3 × 5 × ... × (2n + 1)(2n)(n!)
(C) 200
(E) 1 × 3 × 5 × ... × (2n – 1)(24)(n!)
(D) 240
(E) 320 16. Each factor of 210 is written on a piece of paper,
and all the pieces of paper are mixed up. If a piece of
11. In a library, there are 10 research scholars. In how paper is randomly picked up from this mix, what is the
many ways can 4 of them be selected? (Real NMAT probability that a multiple of 42 is written on the paper?
Question)
1
(A)
(A) 205 16
(B) 210 5
(B)
(C) 215 42
(D) 220 1
(C)
(E) 225 8
3
12. How many four-lettered words (both with and without (D)
meaning) can be formed using the letters of the word 16
‘BALLASTIC’? (Real NMAT Question) 1
(E)
4
(A) 1,016
(B) 1,126 17. As per a weather forecast, the probability of hail is
(C) 1,190 1 for any given day next week. What is the chance
(D) 1,206 6
that there will be hail on both Thursday and Friday?
(E) 1,356
1
13. The number of boys in a class is twice the number of (A)
girls. A team of 8 students is to be selected with 5 36
boys and 3 girls in the team. In how many ways can (B) 1
this be done if the number of students in the class is 12
15? (Real NMAT Question) 1
(C)
(A) 56 6
(B) 112 1
(D)
(C) 252 3
(D) 262 2
(E)
(E) 2,520 3

14. Of all the five digit numbers which can be formed 1


18. A classroom has 12 girls and 20 boys. of the girls
using digits 1, 2, 3, 4 and 5, how many are prime 4
numbers (repetition of digits is not allowed)? (Real in the class have cell phones. If a child is selected at
NMAT Question) random from the class, what is the probability that she
is a girl who does not have a cell phone?
(A) 0
(B) 1 3
(A)
(C) 4 32
(D) 8 (B) 9
32
(E) 12
(2n ) ! 3
15. Which of the following represents ? (Real (C)
n! 8
NMAT Question) 23
(D)
32
(A) 1 × 3 × 5 × ... × (2n – 1)(2n) 29
(E)
(B) 1 × 3 × 5 × ... × (2n + 1)(n!) 32

190

Book 1.indb 190 30/04/2019 4:47:33 PM


3.0  Quantitative Skills Practice

19. A cube has sides numbered 1 through 6. If the cube 1


is rolled three times, what is the probability that at (C)
6
least one of the rolls will result in a number higher
2
than 4? (D)
16
13 1
(A) (E)
19 4
11
(B) 23. Two apples and five bananas are defective out of 10
13 apples and 20 bananas contained in a fruit basket. If
14
(C) Sanjeev takes out two fruits at random, what is the
19 probability that either both are bananas, or both are
19 good?
(D)
27 119
(A)
12 435
(E)
31 338
(B)
20. There is an 80% chance that Deeksha will skip her 435
lunch and 25% chance that there will be a power 841
(C)
failure. If these events are independent, what is the 870
probability that Deeksha will skip her lunch OR that
217
there will be a power failure? (D)
870
(A) 20%
(E) None of these
(B) 80%
(C) 85% 24. If ‘M’ and ‘N’ are two independent events and
(D) 95% P(M) = 0.5 and P(N) = 0.4, find P(M/N).

(E) 105% (A) 0.4

21. Bag A contains 3 white and 3 red beads. Bag B (B) 0.5
contains 6 white and 3 red beads. One of the two (C) 0.6
bags will be chosen at random, and then two beads (D) 0.74
will be drawn from that bag at random without (E) 0.88
replacement. What is the probability that the two
beads drawn will be of the same colour? 25. The roll numbers of students in the class are in
the range from 100 to 199 (both inclusive). If the
7
(A) teacher selects one student at random, what is the
20 probability that his/her roll number is divisible by 3?
9
(B) 1
10 (A)
5
9
(C) 32
20 (B)
99
11
(D) 33
20 (C)
100
13
(E) 2
20 (D)
3
22. Two different unbiased dice are rolled together. What
(E) None of these
is the probability of getting a sum of more than or
equal to 10 after adding the numbers shown on the 26. An integer x is chosen at random from the numbers
tops of both the dice?
336
1 1 to 50. Find the probability that x + ≤ 50.
(A) X
12
7
1 (A)
(B) 10
9

191

Book 1.indb 191 30/04/2019 4:47:35 PM


NMAT by GMAC™ Official Guide 2019

17 (C) Rs. 42000


(B)
25 (D) Rs. 63000
19 (E) Rs. 81000
(C)
50 30. If the collection of a movie is Rs. 100,000 for
the first day, Rs. 120,000 for the second day,
13
(D) Rs. 140,000 for the third day and so on, that is, the
50 collection increases by Rs. 20,000 every day, then
3 find the total collection for the first 10 days.
(E)
10 (A) Rs. 1200,000
27. Find the probability that in a random arrangement of (B) Rs. 1400,000
the letters of the word ‘MANAGEMNT’ two A’s always (C) Rs. 1600,000
come together. (Real NMAT Question)
(D) Rs. 1700,000
1 (E) Rs. 1900,000
(A)
9
31. The 5th and 9th terms of a harmonic progression are
2
(B) 2 2
9 and respectively. What is the first term of the
1 3 9
(C) progression? (Real NMAT Question)
3
4 (A)
−3.5
(D)
9 (B)
−1.5
8 (C)
−0.67
(E)
9 (D) 0
28. There are 5 King cards, 6 Queen cards and 7 Jack (E) 1.5
cards in a box. Rohit draws 3 cards from it, one after
another. What is the probability of the second card 32. On January 1, Ajit put Re 1 in his piggy bank. Every
drawn being a Jack, if the first card drawn is not day, he put in Rs. 2 more than the total amount
replaced? (Real NMAT Question) of money already in the piggy bank. Which of the
following expressions gives the total amount of
56 money in Ajit’s piggy bank at the end of January?
(A)
153 (Real NMAT Question)
7
(B) (A) 230
18
(B) 231
60
(C) (C) 3(230) - 2
153
65 (D) 3(231) – 2
(D)
153 (E) 3(230)
137 33. A person saves Rs. 200 more each year than in the
(E)
306 previous year. If he started with Rs. 400 in the first
year, how many years would he take to save Rs.
29. Manoj plans to work at a coffee shop during his 18,000 (excluding interest)?
summer holidays. He will be paid as per the following
schedule: at the end of the first week, he will receive (A) 10 years
Rs. 1000. At the end of each subsequent week, he (B) 12 years
will receive Rs.1000, plus an additional amount equal (C) 15 years
to the sum of all payments he has received in the
previous weeks. How much money will Manoj be paid (D) 18 years
in total if he works for 6 weeks at this coffee shop? (E) None of these

(A) Rs. 18000 34. If the second term of a geometric progression is 6


(B) Rs. 20000 and the fifth term is 48, then what is its tenth term?

192

Book 1.indb 192 30/04/2019 4:47:36 PM


3.0  Quantitative Skills Practice

(A) 2236 4
(A)
(B) 2146 11
(C) 1536 4
(B)
(D) 1246 13
(E) 1146 11
(C)
4
35. p, q, r and s are any four positive real numbers, the
13
p q r s (D)
minimum value of + + + is 4
q r s p
15
(A) 0 (E)
4
(B) 1
(C) 2 40. How many terms are common in the two arithmetic
progressions given below?
(D) 2 2
AP1: 1, 4, 7, ..., t45
(E) 4
AP2: 1, 3, 5, 7, ..., t55 (Real NMAT Question)
36. If a, b, c and d are in GP, then (a3 + b3)–1, (b3 + c3) –1,
and (c3 + d3) –1 are in ____ . (A) 15
(B) 16
(A) AP
(C) 18
(B) GP
(D) 19
(C) HP
(E) 22
(D) AP or GP
(E) None of these 3 1
41. If of all the chocolates have nuts and of all
4 3
37. In one day, what is the sum of the numbers on which
the chocolates have both nuts and fruits, then what
the hour hand of a clock points each time the minute
fraction of all the chocolates has nuts but no fruits?
hand is on 12? (Real NMAT Question)
1
(A) 12 (A)
4
(B) 78
5
(C) 156 (B)
12
(D) 160
1
(E) 178 (C)
2
38. Three numbers are in geometric progression such 7
(D)
that the product of them is 27 and the sum of 12
the products taken in pairs is 91. What is the
5
(E)
third number in the progression? (Real NMAT
Question) 6

1 42. In a shop, there are 500 sweet products. 210 are


(A) biscuit-based, 160 are milk-based and 160 are
3
coconut-based. 80 have both milk and coconut but
(B) 3 no biscuit, 70 have milk and biscuit but no coconut,
(C) 9 60 have biscuit and coconut but no milk. How many
(D) 27 items have all three ingredients if 200 have none of
these ingredients? (Real NMAT Question)
1
(E) or 27 (A) 10
3
(B) 30
39. The 10th term of harmonic progression
(C) 50
1 4 2 4 (D) 130
, , , , ... is (Real NMAT Question)
5 19 9 17
(E) Cannot be determined

193

Book 1.indb 193 30/04/2019 4:47:37 PM


NMAT by GMAC™ Official Guide 2019

43. In a locality of 40 houses, each house is subscribed 47. There are certain number of students in BRT
to at least one newspaper from amongst, The Sun, University. Of all, 25 students study Quant, 12 study
Daily Mail and Daily Mirror. 24 houses subscribe to English, and 18 study Reasoning. Of these, 10
Daily Mirror. 13 houses subscribe to both The Sun students study both Quant and English, 9 study both
and Daily Mirror. 26 houses subscribe to Daily Mail English and Reasoning and 14 study both Reasoning
out of which 16 also subscribe to The Sun. There and Quant. If 8 students study all the three subjects,
is no house which subscribes to The Sun alone. how many students are there in the class?
7 houses subscribe to all three newspapers. How
(A) 28
many houses subscribe to only two newspapers?
(Real NMAT Question) (B) 29
(C) 30
(A) 12
(D) 31
(B) 14
(E) Cannot be determined
(C) 16
(D) 18 48. Out of 200 food items, 80 are sweet, 60 are spicy,
(E) 20 60 are tangy. Among these, there are 40 sweet and
spicy items, there are 50 spicy and tangy items, and
44. In a class, the ratio of the number of boys having
there are 50 tangy and sweet items. What is the ratio
mobile phones to the number of girls having mobile
of the maximum possible food items of all 3 types
phones is 5 : 2. There are a total of 100 students
and the minimum possible food items of all 3 types?
in the class and only 35 students of them have one
(Real NMAT Question)
mobile phone each in their possession. If the boys
form only the 3/5th part of the class, then what (A) 5:3
percentages of the girls in the class have mobile (B) 4:3
phones?
(C) 3:2
(A) 20% (D) 2:3
(B) 24% (E) 1:1
(C) 25%
49. (0, 4 3 - 3) and (-4, -3) are vertices of an equilateral
(D) 40% triangle. Which of the following could be the co-ordinates
(E) 50% of the third vertex? (Real NMAT Question)
45. In a class of 100 students, 60 students like Physics (A) (-4, 3)
whereas 45 students like Chemistry. Five students
(B) (-4, -4 3 )
don’t like either Physics or Chemistry. How many
students like both Chemistry and Physics? (C) (4, 3)
(D) (4, -3)
(A) 0
(E)
(4, 4 3 )
(B) 5
(C) 10
50. Find ‘a’, if (0, a), (11, 7) and (9, 11) are collinear.
(D) 15 (Real NMAT Question)
(E) 20
(A) −15
46. A survey of 500 students of DMP University, (B) −11
produced the information that 285 students study (C) 15
MBBS, 195 study BHMS, 115 study BAMS, 45 study
(D) 22
MBBS and BAMS, 70 study MBBS and BHMS, 50
study BHMS and BAMS, and 50 study none of the (E) 29
three subjects. How many students study all the 51. Distance between two points A and B whose coordinates
three subjects? are (a, 2) and (4, 6) is 5 units. What is the value of ‘a’?
(A) 10 (A) 0
(B) 15 (B) 1
(C) 18 (C) 7
(D) 20 (D) 1 or 7
(E) 25 (E) Cannot be determined
194

Book 1.indb 194 30/04/2019 4:47:38 PM


3.0  Quantitative Skills Practice

52. If the coordinates (5, 11), (a, 10) and (15, 9) are the
three points on a straight line, then the value of ‘a’ will
be:
(A) 5
(B) 8
(C) 10
(D) 12
(E) Cannot be determined

195

Book 1.indb 195 30/04/2019 4:47:38 PM


NMAT by GMAC™ Official Guide 2019

5. Data Interpretation

(C) 106
(D) 108
Directions for Questions 1–4:  The table below shows the (E) 110
information about number of laptops (figures in 1000s) of Directions for Questions 5-8: Refer to the following table
different models produced and rejected by a company over and answer the questions that follow:
six years.
Number of trousers produced by 5 factories over 5 months
1. In case of Type Q laptop, in which year was the ratio of 2016.
of rejection to production the highest among the
given years? Month Prisma Shelby Kooper Wendy Caret
(A) 2010 Jan 900 850 350 1000 850
(B) 2011 Feb 800 700 1050 1100 850
(C) 2012 Mar 1050 800 1000 1100 950
(D) 2014 Apr 800 850 850 1100 850
(E) 2015 May 950 900 1050 1150 850
2. In which year was the ratio of rejection to production Total 4500 4100 4300 5450 4350
the lowest among the given years for type T laptop?
(A) 2010 5. For which factory was the number of trousers
(B) 2012 manufactured in March the highest percentage of the
total number of trousers produced by that factory
(C) 2013 during the five-month period?
(D) 2014
(A) Prisma
(E) 2015
(B) Shelby
3. What was the difference in Type R laptops rejected (C) Kooper
between 2011 and 2012?
(D) Wendy
(A) 150 (E) Caret
(B) 200
6. The number of trousers manufactured by Wendy in
(C) 250 April is what percentage of the number of trousers
(D) 2000 manufactured by Wendy in January?
(E) 2400 (A) 10%
4. The acceptable (not rejected) Type T laptops in 2012 (B) 91%
were what percentage of those in 2011? (C) 110%
(A) 8 (D) 115%
(B) 14 (E) 125%

Number of laptops of different models produced and rejected by a company over the years (figures in 1000s)

Laptop P Q R S T
model
Year Produced Rejected Produced Rejected Produced Rejected Produced Rejected Produced Rejected
2010 20 2 50 3 15 0.5 80 5 60 4
2011 35 3 45 2 20 0.55 75 4 58 4
2012 15 0.5 40 2.5 17 0.7 58 2 62 3.5
2013 25 0.25 42 2.3 25 1.5 65 3 40 1.5
2014 30 1.5 48 2.5 30 2 68 3 45 2
2015 27 1.5 41 2.1 26 1.75 72 3.5 50 2.3

196

Book 1.indb 196 30/04/2019 4:47:38 PM


3.0  Quantitative Skills Practice

7. Which of the five factories has the highest ratio of the 10. What was the median GPA among the 3,000 students
number of trousers manufactured in April to number in 1980?
of trousers manufactured in February?
(A) 3.7
(A) Prisma (B) 3.3
(B) Shelby (C) 3.0
(C) Kooper (D) 2.7
(D) Wendy (E) 2.3
(E) Caret
11. Approximately what percentage of the students in
8. For which factory was the number of trousers 2010 earned at least a 3.0 GPA?
manufactured in February and March together the
(A) 25%
lowest among the five factories?
(B) 50%
(A) Caret
(C) 67%
(B) Wendy
(D) 80%
(C) Kooper
(E) 97.5%
(D) Shelby
12. Approximately what percentage of the students in
(E) Prisma
1980 earned a GPA less than 3.0?
Directions for Questions 9–12:  Refer to the following
(A) 33%
graph and answer the questions.
(B) 37.5%
(C) 50%
(D) 62.5%
(E) 75%
Directions for Questions 13–16:  Go through the given
graph and solve the questions based on it.

 omparison of GPAs of 3000 students in 1980 and in


C
2010
9. What was the mode for the GPA among the 3,000
13. What is the ratio of male players to female players on
students in 2010?
the Athletics team?
(A) 3.7
(A) 37 : 61
(B) 3.3
(B) 9 : 17
(C) 3.0
(C) 16 : 23
(D) 2.7
(D) 14 : 19
(E) 2.3
(E) 61 : 37

197

Book 1.indb 197 30/04/2019 4:47:39 PM


NMAT by GMAC™ Official Guide 2019

14. All players, except those in Athletics and Cricket 17. In 2006, the populations of Town A and Town B
teams, are a part of only one team. If there are a each increased by 10% as compared to 2005. If
total of 76 male players in different university sports the population of Town A in 2005 was 5000 and
teams, how many male players are in both Athletics the percentage of the population living below the
team and Cricket team? poverty line for all seven towns in 2006 remains
the same as in 2005, which of the following is
(A) 11
the approximate population of Town B below the
(B) 17 poverty line in 2006?
(C) 37
(A) 2500
(D) 54
(B) 3000
(E) 76
(C) 3500
15. In which of the following university sports team(s) do
(D) 4000
male players outnumber female players?
(E) 4500
(A) Athletics, Tennis and Football
18. In 2007, the population of Town D increased by 10%
(B) Cricket
as compared to 2005 and the population of Town
(C) Football and Cricket G reduced by 5% as compared to 2005. If the
(D) Football population of Town G in 2005 was 9000, what is
(E) Tennis and Athletics the total population of Towns D and G in 2007?

16. What is the ratio of female tennis players to male (A) 19200
basketball players on the university sports teams? (B) 19770

(A) 5 : 14 (C) 19870

(B) 9 : 14 (D) 19970

(C) 7 : 18 (E) None of these

(D) 14 : 9 19. If in 2005 the total population of the seven towns


(E) 18 : 7 together was approximately 55,000, what will be the
approximate population of Town F in that year below
Directions for Questions 17–20:  Study the following chart the poverty line.
to answer the question given below:
(A) 2500
Town % of the population below the poverty line (B) 3000
A 45 (C) 3500
B 52 (D) 4000
C 38 (E) 4500
D 58
E 46 20. The population of Town C is 2000 in 2005. What will
F 49 be the ratio of the population of Town C below the
poverty line to that of Town E below the poverty line
G 51
in that year?
Percentage distribution of the population of seven towns of (A) 207 : 76
the state in 2005. (B) 76 : 207
(C) 152 : 207
G A (D) 76 : 307
15% 13% (E) 87 : 207
F B
13% 16% Directions for Questions 21–24:  Go through the given
table and solve the questions based on it.
E
C  (Real NMAT Question)
D 8%
18%
17%

198

Book 1.indb 198 30/04/2019 4:47:39 PM


3.0  Quantitative Skills Practice

Production of Major Crops (tonnes) Directions for Questions 25–28:  Go through the given
information and solve the question based on it.
Y1 Y2 Y3 Y4 Y5
Foodgrain (Kharif) 69 78 99 102 111 Stock 1, Stock 2 and Stock 3 are stocks of small cap
companies and Stock 4 is the stock of a mid cap company.
Foodgrain (Total) 108 130 176 197 217 For each investor, mid cap stocks are given greater
Cereals (Kharif) 65 74 94 98 106 weightage than small cap stocks and all the small cap
Cereals (Total) 97 119 162 186 203 stocks have the same weightage. The weightage is different
for each investor. (Real NMAT Question)
Pulses (Kharif) 4 4 5 4 5
Pulses (Total) 12 11 14 11 14
Stocks
Rice (Kharif) 40 50 66 73 80
Stock Stock Stock Stock
Rice (Total) 42 54 74 85 93 Investor 1 2 3 4 Total
Oilseeds (Kharif) 7 5 10 12 14
Paul 6,500 7,500 7,000 5,000 36,000
Oilseeds (Total) 10 9 19 18 24
Bindu 7,300 6,700 6,300
Total foodgrain production = Production of rabi foodgrain Rohan 6,600 7,300 6,900
+ Production of kharif foodgrain
Sheela 4,800 8,900 39,500
21. In which year did the production of rabi foodgrains Thomas 5,600 3,200 1,200 8,400
exhibit the highest percentage increase over the
preceding year?
25. Which of the following can be the ratio of the
(A) Y1 weightage of Stock 4 to the weightage of Stock 2
(B) Y2 that Paul holds?
(C) Y3 (A) 1 : 4
(D) Y4 (B) 1 : 3
(E) Y5 (C) 1 : 2
22. From Y2 to Y5, for which crop was the percentage (D) 3 : 1
increase in the rabi crop production the highest? (E) 4 : 1
(A) Rice
(B) Pulses 26. If the ratio of the weightage of small cap stocks
to that of mid cap stocks is 1 : N (where N is any
(C) Cereals integer), what can be the maximum value of Stock 3
(D) Oilseeds that Sheela holds?
(E) Foodgrain (A) Rs. 8,000
23. What was the average production of rabi cereals (B) Rs. 8,450
from Y1 to Y5? (C) Rs. 16,900
(A) 20 tonnes (D) Rs. 19,000
(B) 32 tonnes (E) Rs. 27,600
(C) 45 tonnes
27. If the sum of the values of Stock 1 and Stock 3 that
(D) 54 tonnes Bindu holds is the same as the value of Stock 4 that
(E) 66 tonnes she holds, what is the ratio of the weightage of her
small cap stocks to the weightage of her mid cap
24. From Y3 to Y4, what was the percentage increase in stocks?
the total rabi production of the five major crops?
(A) 1 : 7
(A) 21.64%
(B) 2 : 9
(B) 42.45%
(C) 4 : 15
(C) 48.82%
(D) 7 : 20
(D) 52.56%
(E) 9 : 20
(E) 58.34%

199

Book 1.indb 199 30/04/2019 4:47:39 PM


NMAT by GMAC™ Official Guide 2019

28. In Thomas’s portfolio, the value of Stock 3 is one- 31. A bike costs $1,000. Aman has INR 45,500, Euro
eighth of the value of Stock 4. What is the ratio of 600 and 77,000 NPR with him. He would be able to
the weightage of a small cap stock to the weightage buy the bike on 23rd June using (assume that he can
of a mid cap stock that Thomas holds? use only one type of currency for the transaction):
(A) 3 : 4 (A) EUR 600
(B) 7 : 9 (B) INR 45,500
(C) 7 : 8 (C) NPR 75,000
(D) 8 : 9 (D) NPR 80,000
(E) 9 : 8 (E) None of the above

Directions for Questions 29–32:  Use the graph, which 32. On 18th June, the INR/Dollar ratio was 43.5. On that
refers to the currency ratio of 20th June to 27th June 2008, day INR 6,000 would be equal to:
to answer the following questions. (Real NMAT Question) (A) 4,285 Euro
1.8 Dollar/Euro 82 NPR/Dollar (B) 4,582 Euro
1.7 81
(C) 4,825 Euro



(D) 5,205 Euro
1.6  80  
(E) Cannot be determined
1.5  79
 Directions for Questions 33–36:  Go through the given
1.4  78  graph and solve the questions based on it. This chart
indicates the sales of Herbal Beauty Products Company
20 22 23 24 27 20 22 23 24 27 during a period of time. (Real NMAT Question)
Sales (in packs) of five different products of a Herbal beauty
INR/Dollar products company during 2000 and 2005
2000 2005
47
2,410,000

46  1,890,000
  1,460,000

45  1,010,000
748,000 611,000 510,000
246,000 344,000 250,000
44
Herbal Hair Herbal Hair Herbal Face Herbal Face Herbal
Wash Vitaliser Cream Wash Moisturiser
20 22 23 24 27

29. On 22nd June, how much NPR would be equal to the 33. What is the ratio of percentage growth in sales of
value of INR 4,000? Herbal Hair Wash and Herbal Hair Vitaliser during
2000–2005 (rounded off to the nearest integer)?
(A) 6,596
(B) 6,956 (A) 1 : 4
(C) 9,656 (B) 1 : 5
(D) 9,665 (C) 5 : 4
(E) None of these (D) 4 : 1
(E) 7 : 8
30. If the NPR/Dollar ratio follows the same trend from
27th to 30th as from 24th to 27th, what will be the 34. What is the percentage growth in overall sales (in
value of 150 Dollars on 30th June? terms of packs) of Herbal Beauty products in
2005 compared to 2000?
(A) 11,250
(B) 11,520 (A) 104.3%
(C) 12,150 (B) 138.6%
(D) 12,510 (C) 164.8%
(E) None of these (D) 171.3%
(E) 183.6%

200

Book 1.indb 200 30/04/2019 4:47:40 PM


3.0  Quantitative Skills Practice

35. For which product did the sales increase by almost (C) VTS
78% from 2000–2005? (D) SCHAPE
(A) Herbal Moisturiser (E) BAJAZ
(B) Herbal Face Wash
38. What is the difference between the total numbers of
(C) Herbal Face Cream two-wheelers on the roads of the country from the
(D) Herbal Hair Wash year 2007 to year 2009?
(E) Herbal Hair Vitaliser (A) 4900
36. What is the approximate ratio of the percentage (B) 4500
growth in sales of Herbal Hair Vitaliser to the (C) 5100
percentage growth of the rest of the products in the (D) 5200
period 2000–05?
(E) 5400
(A) 5 : 3
39. What is the approximate percentage of RHONDA two-
(B) 5 : 2
wheelers out of the total number of two-wheelers on
(C) 7 : 2 the roads of the country in the year 2008?
(D) 6 : 1
(A) 30%
(E) 9 : 1
(B) 33.33%
Directions for Questions 37–40:  Answer the questions on (C) 35%
the basis of the information given below. (D) 38%
The table given below shows the number of two-wheelers (E) 45%
(motorised) running on the roads of a country XYZ during
the period 2006 – 2010. These are the only type of two- 40. What is the average number of two-wheelers running
wheelers on the roads of the country. on the roads of the country in the year 2008?
37. The second highest annual growth over the entire (A) 118756
period has been experienced by which of the (B) 118765
following two-wheelers:
(C) 119576
(A) RHONDA (D) 181756
(B) SICTOR (E) 191756

2006 2007 2008 2009 2010

VTS 1120 1300 1800 1900 2100

SULPAR 194830 249200 266000 325000 438200

SCHAPE 35600 42300 43300 49200 57000

RHONDA 417500 479200 403400 416700 471000

MAHA 143600 172600 150400 162400 193800

SICTOR 1100 1600 2300 3000 2400

MUZUKI 114500 137600 121700 125700 158000

BAJAZ 58800 65000 64900 68000 80400

SLOGAN 12400 14400 15000 16500 21000

201

Book 1.indb 201 30/04/2019 4:47:40 PM


NMAT by GMAC™ Official Guide 2019

Directions for Questions 41–43:  Read the information Directions for Questions 44–46:  Answer the questions on
given below and answer the questions that follow. the basis of the information given below.
The table given at the bottom of this page depicts the marks The line graph below depicts the number of employees
obtained by 1000 students in English and Computer Science who left the company ABC Pvt. Ltd. and the number of new
in an entrance exam conducted by JET (Junior Entrance Test) joinees in that year. Also, it is known that the number of
employees in the year 2012 was 2000.
41. What is the difference between the percentage of
students who secured more than 60% marks in 600
aggregate and those who secured more than 40%
500
marks in aggregate?
400
(A) 0%
(B) 27% 300 New Joinees
Employees that left
(C) 46% 200

(D) 54% 100


(E) 73%
0
2013 2014 2015 2016 2017
42. What is the total number of students securing more
than 20 marks in English and 40 marks in Computer
Science? 44. What was the percentage change in number of
employees in the company from the year 2012 to
(A) 40 year 2013?
(B) 70
(A) 2%
(C) 260
(B) 3%
(D) 840
(C) 4%
(E) Cannot be determined
(D) 5%
43. The percentage of the number of students securing (E) Data insufficient
more than 60% marks in Computer Science is
approximately what percent of those getting more 45. In which of the following years was the number of
than 40% marks in aggregate? employees in ABC Pvt. Ltd. the maximum?
(A) 20% (A) 2014
(B) 29% (B) 2015
(C) 31% (C) 2016
(D) 36% (D) 2017
(E) 42% (E) None of these

            Marks out of 50 >40 >30 >20 >10 >0


Subject

English 90 320 800 920 1000

Computer Science 40 210 660 810 1000

Average marks per subject 70 270 730 870 1000

202

Book 1.indb 202 30/04/2019 4:47:40 PM


3.0  Quantitative Skills Practice

46. In which of the following two years was the number (C) 3
of employees in ABC Pvt. Ltd. the same? (D) 4
(A) 2013 and 2017 (E) 5
(B) 2013 and 2015 Directions for Questions 50–53:  Maize, Javar and Barley
(C) 2016 and 2017 are the three components of a grain product. These grains
(D) 2014 and 2017 are mixed in the ratio 1 : 2 : 3 to form the product. The bar
graph below gives the prices (in Rs) of 500 kg of Maize,
(E) None of these
1000 kg of Javar and the average cost of 500 kg each of
Directions for Questions 47–49:  Answer the questions on Maize, Javar and Barley. (Real NMAT Question)
the basis of the information given below. Go through the given graph and solve the questions based
on it.
450
405

380

370
370

6000
365

400
340

5000 5000
325

320
320

350 5000
4000 4000
270

265

300
4000 3500
250 3000 3000 3000 3000
Volume of cars sold 3000 2500 2500
170

160

200 2000
150

Sales per unit volume 2000


150
1000
100

50 0
1996 1997 1998 1999
0
Maize Average of three grains Jawar
2011 2012 2013 2014 2015 2016 2017

The bar chart represents the volume of cars sold in a 50. What was the ratio of the price of a kilogram of
particular year and the sales per unit volume of a particular Barley in 1996 to that in 1997?
year.
(A) 3 : 1
47. By what percentage is the total volume of sales in the (B) 4 : 3
year 2015 greater/smaller than that in 2011? (C) 8 : 7
(A) 45.6% (D) 13 : 14
(B) 50.3% (E) 8 : 9
(C) 54.2% 51. The price of Barley increased from 1997 to 1998.
(D) 61.4% What is the percentage increase in the price?
(E) 66.67% (A) 50%
48. What is the difference between the average of the (B) 67%
volume of cars sold and that of the sales per unit (C) 68%
volume for the whole period?
(D) 100%
(A) 40.00 (E) 124%
(B) 41.57
52. Rahul bought 10 kg each of Maize, Javar and Barley
(C) 43.21 in the given years. In which year(s) did he pay the
(D) 45.12 highest amount of money?
(E) 50.73 (A) 1996
49. In how many of the years, the trend is such that when (B) 1997
there is an increase in volume sold over the previous (C) 1998
year, then there is a decrease in the sales per unit
volume over the previous year and vice versa? (D) 1996 and 1997
(E) 1997 and 1999
(A) 1
(B) 2

203

Book 1.indb 203 30/04/2019 4:47:41 PM


NMAT by GMAC™ Official Guide 2019

53. Assuming that the three grains are the only 56. Which of the following month reported the maximum
ingredients used in making the product, what was the percentage increase in the price of share D in
cost of 6,000 kg of the grain product in 1997? comparison to the previous month?
(A) Rs. 17,250 (A) February
(B) Rs. 20,750 (B) May
(C) Rs. 25,500 (C) June
(D) Rs. 35,850 (D) July
(E) Rs. 36,450 (E) September
Directions for Questions 54–57:  The table below shows 57. For which company’s shares was the absolute simple
the price per share of five companies in the year 2012. monthly change rate the maximum?
Assume that the price per share of each company remains
Annual change rate
the same throughout the month. (Real NMAT Question)
Price per share in December – Price per share in January
Go through the given table and solve the questions = × 100
based on it. Price per share in January

54. Mr. X bought 60 shares of Company C and 40 shares Annual change rate
Absolute simple monthly change rate =
of Company D in the month of April and sold these 12
100 shares in the month of December. What was his (A) Company A
approximate profit percentage in this transaction?
(B) Company B
(A) 7.8% (C) Company C
(B) 8.4% (D) Company D
(C) 9.2% (E) Company E
(D) 9.7%
Directions for Questions 58–61:  The bar chart shows
(E) 11.7% the standings of 16 drivers after six races in the season.
55. What was the average price of share E as a There are a total of 9 races this season. There are 8 teams
percentage (approximate) of the average price of participating in the racing championship and each team has
share C in 2012? fielded 2 drivers. In each race the winner gets 10 points, 1st
runner up gets 8, 3rd place driver gets 6, 4th place driver
(A) 15.21% gets 5, 5th place driver gets 4, 6th place driver gets 3, 7th
(B) 30.24% place driver gets 2 points and 8th place driver gets 1 point.
(C) 68.56% The rest of the drivers do not get any points. The driver with
the highest number of points in the championship will be
(D) 105.87% declared the winner of the championship. Also the team with
(E) 156.87% the highest aggregate of both the drivers taken together will
be declared the champion team at the end of the season.
(Real NMAT Question)

Jan Feb Mar Apr May June July Aug Sept Oct Nov Dec

A 1226 1186 986 1008 1108 1217 1198 1237 1081 1901 1826 1736

B 286 307 296 317 412 383 365 392 267 412 320 281

C 2208 2107 2317 2424 2200 1987 1921 1876 2612 2516 2712 2916

D 1512 1628 1427 1397 1556 1297 1397 1297 1456 1395 1296 1150

E 782 857 687 789 852 684 586 750 621 586 684 528

204

Book 1.indb 204 30/04/2019 4:47:41 PM


3.0  Quantitative Skills Practice

54
(C) 2
(D) 3
(E) 4
39
61. Which of the following cannot be true?
29
27 (A) Team 2 was declared the champion team at the
22 22 21
end of the season.
(B) D4 won the last race of the season and Team 3
10
was the champion team.
4
(C) D1 got top 3 finishes in all the 6 races and he
3
2 1 0 0 0 0 was not the winner of the 6th race.
D1 D2 D3 D4 D5 D6 D7 D8 D9 D10 D11 D12 D13 D14 D15 D16
(D) D8 was among the top 3 finishers in the 3rd race
but he did not score any points in the 1st race.
The table below lists the drivers for each team. (E) All of the above statements can be true.
Team Drivers Directions for Questions 62–65:  Go through the
information below and answer the questions based on it.
Team 1 D1 and D2
An experiment was conducted to study the effect of acid rain
Team 2 D3and D6
on five species of aquatic animals that were released in a lake.
Team 3 D4 and D10 The original pH of the lake was 6.5. (Real NMAT Question)
Team 4 D5 and D9 Effect of Acid Rain on Aquatic Species
Team 5 D7 and D12 Aquatic Number pH Tolerance of Different Aquatic Species
Team 6 D8 and D11 Species Released
pH pH pH pH pH pH
at pH 6.5
Team 7 D13 and D15 6.5 6.0 5.5 5.0 4.5 4.0
Trout 24     X X
Team 8 D14 and D16
Bass 82    X X X
58. If D1 got a top 3 finish in each of the first 6 races, Perch 40      X
then what is the minimum number of races that were Frog 73      
won by him in the first 6 races?
Clam 12   X X X X
(A) 1
 Indicates pH levels which can be tolerated
(B) 2
X Indicates pH levels which con-not be tolerated
(C) 3
(D) 4 62. If the pH of the lake decreases from 6.5 to 4.5
(E) It cannot be determined. because of acid rain, what percentage of the total
number of aquatic animals that were released into
59. D1 finished in the top 3 in the 7th race but did not the lake at pH 6.5 are expected to survive?
finish the last 2 races due to some unavoidable
circumstances. D2 left D1 behind and took the (A) 17.32%
driver’s championship. Which of the following can be (B) 23.10%
the lowest position of D2 in the remaining 3 races? (C) 31.60%
(D) 48.92%
(A) 4th
(E) 52.64%
(B) 5th
(C) 6th 63. Assuming that the original pH level of the lake was
6.5, at what pH level would approximately 59.3%
(D) 7th of the original number of aquatic animals that were
(E) 8th released in the lake most likely survive?
60. If D2 did not win any points in one of the 6 races (A) pH 4.0
that have been completed so far, then which of the (B) pH 5.5
following cannot be the number of points scored by
him in any of the 6 races? (C) pH 5.0
(D) pH 5.5
(A) 0
(B) 1 (E) pH 6.0

205

Book 1.indb 205 30/04/2019 4:47:41 PM


NMAT by GMAC™ Official Guide 2019

64. The pH of the lake first decreased because of acid (A) 39%
rain from 6.5 to 4.5, and then the pH was increased (B) 52%
artificially from 4.5 to 5.5. If 12 aquatic animals of
(C) 47%
each species were then introduced into the lake at
pH 5.5, how many species would most likely record (D) 58%
greater than 60% change in the number of animals at (E) None of these
pH 5.5 over pH 6.5?
67. What is the approximate average number of rooms
(A) 1 built per million dollars spent by different states
(B) 2 during the given period of time?
(C) 3 (A) 1.25
(D) 4 (B) 1.72
(E) 5 (C) 0.69
65. Approximately how many times greater is the total (D) 1.43
number of aquatic animals that will most likely not (E) None of these
survive when the pH changes from 6.5 to 4 than the
68. If the value of investment increases at a simple
total number of aquatic animals that will most likely
interest rate of 10% per annum, what is the
not survive when the pH changes from 6.5 to 4.5?
approximate average value of investment per hotel
(A) 0.5 times room constructed in the year 2010?
(B) 1.3 times
Note: Here, the term ‘investment’ refers to cost of
(C) 2.5 times
constructing hotel rooms.
(D) 4.1 times
(A) 0.62 million dollars
(E) 7.5 times
(B) 0.69 million dollars
Directions for Questions 66–69:  Go through the data
(C) 0.80 million dollars
given below and solve the questions based on it.
(D) 0.84 million dollars
ountry X hosted the Commonwealth Games in 2010. One
C
of the main concerns of the organizing committee for the (E) None of these
Commonwealth Games was to arrange hotel rooms for 69. What was the approximate combined value of all the
the guests. For that purpose, the governments in seven investments, in 2007, if the interest rate was 10%
states of the country decided to construct new hotels. The compounded annually?
following table gives information about hotels constructed in
different states. (Real NMAT Question) Note: Here, the term ‘investment’ refers to cost of
constructing hotels.
Number Cost (million Year of
Hotel State
of Rooms dollars) completion (A) 1800 million dollars
Hotel A 600 275 2008 State M (B) 2125 million dollars
Hotel B 320 210 2009 State N (C) 1670 million dollars
Hotel C 250 250 2009 State O (D) 2445 million dollars
Hotel D 400 430 2008 State P (E) None of these
Hotel E 520 310 2010 State Q Directions for Questions 70–73:  Use the table to solve
Hotel F 450 400 2008 State R the questions.
Hotel G 500 250 2010 State S he following table gives the sales of various sections of
T
three departmental stores that began operations in 2005.
66. What is the cost of hotels completed in 2008 as a All values are in million dollars. (Real NMAT Question)
percentage of the cost of hotels to be completed in
all three years?

206

Book 1.indb 206 30/04/2019 4:47:42 PM


3.0  Quantitative Skills Practice

2005 2006 2007 2008 2009 73. Which one of the following statements is correct?
Topper's Top (A) The ‘Others’ section has contributed to more
Men 13.5 12.5 9.8 11.5 12.5
than 50% of sales for Topper’s Top across all
years.
Women 14.5 8.8 12.2 16.2 18.7
(B) Between 2006 and 2009, sales of the women’s
Others 21.2 16.2 25.6 21.7 25.2 section of Topper’s Top declined steadily whereas
Olivestyle that of the ‘Others’ section steadily increased.
Men 21.5 18.5 17.2 18.5 15.4 (C) The men’s section sales at Oliveside are always
Women 8.2 9.5 10.8 11.5 12.7 the lowest among all three stores.
Others 16.5 28.5 21.5 22.7 21.5 (D) The women’s section sales at Eastside have
always been higher than the average sales of all
Eastside
three sections at Eastside.
Men 4.3 5.4 8.6 11.2 9.5
(E) The ‘Others’ section sales of Eastside are always
Women 6.5 7.5 9.2 10.8 11.8 the highest among all three stores.
Others 6.5 7.2 6.8 6.5 7.5
Directions for Questions 74–77:  The given graph shows
Total Sales 112.7 114.1 121.7 130.6 134.8
the distribution of a net property tax of 16.6 million dollars
levied by a district government. Go through the given graph
70. For which year did the sales increase the most, and solve the questions based on it. (Real NMAT Question)
relative to the previous year, for the men’s section of
Olivestyle?
Distribution of Property Tax
(A) 2005 A
(B) 2006

its
Libra
(C) 2007

r Un
(D) 2008

ries

Othe
(E) 2009
Counties
71. Which of the following represents the years during
Schools
which the total sales of men’s sections of the three 46.6%
O
departmental stores lay between 30% and 40% of To
wn
the total sales of all the sections for the three stores? sh
ips
Cities
(A) 2005 and 2006 and
Towns
(B) 2005 and 2007
(C) 2005, 2006 and 2007
B
(D) 2005, 2006 and 2008
(E) 2006, 2008 and 2009
74. If points A, O, and B can be joined to form a straight
72. The highest percentage growth in the sales of all line, what was the property tax levied by the
three stores together, relative to the previous year, government on Libraries?
was achieved in:
(A) 3,40,000 dollars
(A) 2005 (B) 4,88,235 dollars
(B) 2006 (C) 5,64,400 dollars
(C) 2007 (D) 16,60,000 dollars
(D) 2008 (E) 18,30,000 dollars
(E) 2009

207

Book 1.indb 207 30/04/2019 4:47:42 PM


NMAT by GMAC™ Official Guide 2019

75. The tax levied on Schools, Libraries, and Counties 78. Which of the following statements regarding the
was 75% of the total tax levied. The total tax levied profits earned by the two brands in the first three
on Townships and Cities and Towns was 3.652 years (2000 to 2002) is true?
million dollars. What was the tax levied on Other
(A) Trots earned 42% more profits than Britz.
Units?
(B) Trots earned a 5% increase in profits during this
(A) 4,98,000 dollars time frame.
(B) 5,15,000 dollars (C) Britz earned close to 1/42 times the profit
(C) 8,30,000 dollars recorded by Trots.
(D) 12,45,000 dollars (D) Trots earned close to 1.6 times the profits
(E) 16,60,000 dollars recorded by Britz.
(E) The average profits earned by Trots was 25
76. The combined tax levied on Townships, Cities and
million dollars more than that earned by Britz.
Towns, and Other Units was equal to the tax levied
on Counties. The tax levied on Counties was half the 79. What is the difference in the mean profits earned by
combined tax levied on Schools and Libraries. How the two brands in the last 3 years (2003 to 2005)?
much tax was levied on Counties?
(A) $ 0
(A) 25,00,000 dollars (B) $ 10
(B) 27,50,000 dollars (C) $ 15
(C) 32,60,000 dollars (D) $ 48
(D) 41,50,000 dollars (E) $ 145
(E) 83,00,000 dollars
80. The ratio of profits earned by Trots in 2005 to the
77. The tax levied on Cities and Towns was 6 times the profits it earned in 2004 is the same as the ratio of
tax levied on Other Units and 4.5 times the tax levied profits earned by the brand in 2006 to the profits it
on Townships. If the tax levied on Other Units was earned in 2005. The ratio of profits earned by Britz
4,98,000 dollars, what per cent of the total taxes in 2004 to the profits it earned in 2003 is the same
was levied on Townships? as the ratio of profits earned by the brand in 2006 to
the profits it earned in 2005. What is the difference
(A) 3%
in profits earned by the two brands in 2006?
(B) 4%
(A) 30 million dollars
(C) 13.5%
(B) 45 million dollars
(D) 18%
(C) 105 million dollars
(E) 20%
(D) 152 million dollars
Directions for Questions 78–81:  The graph shows the
(E) 252 million dollars
profit trends of two different brands of pet food; Britz and
Trots. Go through the given graph and solve the questions 81. If the profits earned by Trots increased by 15% in
based on it. (Real NMAT Question) 2006 from 2005, the mean profits earned between
Britz 2000 and 2006 by Trots is 8 million dollars more
Trots than the mean profits earned by Britz in the same
Profit Trends period. What profits did Britz earn in 2006?
90
80 (A) 42 million dollars
80
70
(B) 50 million dollars
(million dollars)

60 70
60
50 (C) 78 million dollars
Profit

50
40 30 48
35 (D) 92 million dollars
30
20 28
(E) 96 million dollars
23
10 20 15
Directions for Questions 82–85:  Go through the given
0
graphs and solve the questions based on them. (Real
2000 2001 2002 2003 2004 2005
NMAT Question)
Year

208

Book 1.indb 208 30/04/2019 4:47:42 PM


3.0  Quantitative Skills Practice

Disability Support Pension Recipients decrease in the number of recipients in the 30–34
by Age and Gender in 2005 age group from 2005 to 2006?
100
96
92
(A) 6%
88
84
(B) 7%
80
76
(C) 18%
72
(D) 44%
Number of Recipients

68
64
(E) 80%
(in thousands)

60
56
52 84. What is the approximate percentage decrease in the
48
44
total number of recipients aged above 49 years and
40 below 65 years from 2005 to 2006?
36
32 (A) 1%
28
24 (B) 4.2%
20
16 (C) 5.1%
12
8 (D) 7.8%
4
0 (E) 100%
35-39 40-44 45-49 50-54 55-59 60-64
Age 85. The number of male recipients between the age of
Male 35 and 69 decreased by 5% from 2005 to 2006. If
Female
Disability Support Pension Recipients there were 20,000 male recipients in the age group
by Age and Gender in 2006 of 65–69 in the year 2006, how many male recipients
100 were in the same age group in the year 2005?
96
92
88 (A) 8,000
84
80 (B) 10,000
76
72 (C) 18,000
Number of Recipients

68
64 (D) 20,000
(in thousands)

60
56
(E) 44,000
52
48 Directions for Questions 86–89:  Go through the table
44
40
given and solve the questions based on it.
36 The table gives the cost structure for packaged food
32
28 products—Potato Chips, Jam and Packaged Juice. All the data
24 pertains to the N grams packet of each of the three products.
20
16
MRP = maximum retail price = the price at which the retailer
12 sells the article to the customer. (Real NMAT Question)
8
4
0 Cost structure for select packaged food products
35-39 40-44 45-49 50-54 55-59 60-64
Potato Chips Jam Packaged Juice
Age
INR INR INR
82. What is the approximate percentage increase in MRP 10 50 68
the number of recipients in the 60–64 age group
between 2005 and 2006? Retailer’s Margin + 0.83 7.20 11.80
discount
(A) 5.5% Distributor’s Margin 0.44 2.18 3.50
(B) 10.2% Sales Tax 0.94 5.37 8.20
(C) 11.4% Excise Duty 1.04 0.00 0.00
(D) 16% Distribution 0.41 2.81 2.70
(E) 45% Returns 0.27 1.06 0.68
83. In each year, there was a total of 1,50,000 recipients Manufacturing Cost 1.10 0.87 5.10
in the three age groups that are immediately before Packaging Cost 1.50 6.18 12.60
45–49. What was the approximate percentage Raw Material Cost 1.25 9.52 5.80

209

Book 1.indb 209 30/04/2019 4:47:43 PM


NMAT by GMAC™ Official Guide 2019

86. For which of the following items is the raw material Note: A person day of employment is the product of
cost, as a percentage of MRP, the maximum? the number of people employed and the number of
days each person worked.
(A) Jam
(B) Potato chips 90. Between Y1 and Y4, what was the percentage
(C) Packaged juice increase in the number of people employed?
(D) Potato chips and jam (A) 23.42%
(E) None of these (B) 24.29%
87. A distributor sells 5 packets of potato chips, 12 packets (C) 68.39%
of jam and 8 packets of packaged juice (N grams of (D) 72.63%
each). What is his total margin for these items (INR)? (E) 79.36%
(A) 28.52 91. In Y1, the total number of rural households was
(B) 38.24 3.2 crores. Between Y1 and Y4, there was a 20%
(C) 48.27 increase in the total number of rural households.
What is the ratio of the number of households which
(D) 56.36
were not provided employment in Y1 to the number
(E) 64.12 of households which were not provided employment
88. If there is no sales tax, then the price of which of the in Y4?
following products will come down over MRP by the (A) 11 : 4
maximum percentage?
(B) 17 : 4
(A) Jam (C) 33 : 2
(B) Potato chips (D) 55 : 2
(C) Packaged juice (E) 32 : 1
(D) Potato chips and jam
92. From Y1 to Y3, what was the average number of
(E) It will come down equally in all three cases people employed per year under the employment
89. What percentage of the MRP is the retailer’s margin scheme?
in the case of packaged juice? (A) 0.32 crore
(A) 8.3% (B) 0.39 crore
(B) 10.2% (C) 0.47 crore
(C) 12.8% (D) 0.49 crore
(D) 16.2% (E) 0.63 crore
(E) Cannot be determined 93. Which year recorded the highest growth rate in the
Directions for Questions 90–93:  Go through the given table number of person days per household?
and solve the questions based on it. (Real NMAT Question) (A) Y1–Y2
Rural Employment Scheme
(B) Y2–Y3
Year Y1 Y2 Y3 Y4 Y5 (C) Y3–Y4
Households Provided 2.1 2.3 3.4 3.8 4.5
(D) Y4–Y5
Employment (crore) (E) Y5–Y6
Number of Person Days of 90 108 144 156 200
Directions for Questions 94–97:  The following chart
Employment Generated (crore)
represents the age-wise distribution of personal income of a
Number of Person 17 20 36 40 46 state in the year 2007. Use it to answer the given questions.
Days Per Household
(Real NMAT Question)
Number of Working Days 230 230 230 231 230

210

Book 1.indb 210 30/04/2019 4:47:43 PM


3.0  Quantitative Skills Practice

User Votes
26 to 35 years Votes Percentage Rating
21 to 25 years 130°
50° 9,539 10% 10
8,871 9.3% 9
40°
18 to 20 years 20,032 21.0% 8
140° 27,759 29.1% 7
Above 35 years
15,739 16.5% 6
6,582 6.9% 5
Total income = Rs 5,400 crore 2,957 3.1% 4
1,526 1.6% 3
94. What is the personal income of the 21 to 25 years 2
954 1%
age group (Rs. in crore)?
1,431 1.5% 1
(A) 632
(B) 750 98. How many users voted?
(C) 785 (A) 100
(D) 810 (B) 709
(E) 840 (C) 85,851
95. What is the percentage of the personal income of the (D) 93,959
above 35 years age group to that of the 21 to 25 (E) 95,390
years age group?
99. Exactly 12% of the number of voters who gave a
(A) 180% rating of 5 or below were below 18 years of age.
(B) 280% How many users who gave a rating of 5 or below
(C) 320% were below 18 years of age?
(D) 440% (A) 824
(E) 520% (B) 1,614
96. What is the ratio of personal income of the 26 to 35 (C) 6,868
years age group to that of the above 35 years age (D) 13,448
group? (E) 13,450
(A) 7 : 13 100. What was the rating given by almost half the users?
(B) 7 : 15
(A) 1 or 2
(C) 13 : 14
(B) 4 or 5
(D) 14 : 13
(C) 6
(E) Cannot be determined
(D) 7 or 8
97. If the total income of the state increases by 12% (E) 10
every year, what will be the total income in the year
2009 (Rs in crores)? 101. What is the average (arithmetic mean) rating given by
the users?
(A) 6,774
(A) 6.196
(B) 7,667
(B) 7.000
(C) 7,676
(C) 7.096
(D) 7,776
(D) 12.900
(E) 8,676
(E) 29.100
Directions for Questions 98–101:  The given graph shows
Directions for Questions 102–105:  A group of 25,000
how a particular movie was rated by users of an online
families were asked about the usage of technology by all
movie review site. Each user could vote only once. Go
children aged 3 to 18 years in their family. There were
through the given graph and solve the questions based on it.
45,000 children in this age group. The children who used
(Real NMAT Question)

211

Book 1.indb 211 30/04/2019 4:47:43 PM


NMAT by GMAC™ Official Guide 2019

technology were asked how often they used each device. Go 103. What is the difference between the number of
through the given graphs and solve the questions based on children aged 3 to 18 years who use the smartphone
them. (Real NMAT Question) weekly and the number of children aged 3 to 18
years who use the e-reader weekly?
Family Technology Ownership & Child Use
80 78
(A) 2,889
77
(B) 4,500
70
(C) 5,612
60
(D) 6,423
52
50 46 (E) 8,334
43
% 40
34 36 104. Which device is used daily by exactly 11,934 children
30 27 who are aged 3 to 18 years?
24
20 (A) Laptop
10
11 (B) Smartphone
(C) Tablet
0
Laptop Smartphone Tablet iPod E-reader (D) iPod
Child Use Frequency Key (E) E-reader
6% 6% Family owns 105. What is the ratio of the total number of children who
Child uses use the laptop less often to the total number of
children who use the e-reader less often?
43% 51% 29% 65% (A) 6 : 13
Key (B) 7 : 12
Laptop Smartphone Daily (C) 12 : 7
7% 5%
13% Weekly (D) 30 : 17

Less often
(E) 24 : 11
32%
47% 42% 53%
46%
55%

Tablet iPod E-reader

102. How many children aged 3 to 18 years use the


device which is owned by exactly 11,500 families?
(A) 3,910
(B) 8,500
(C) 10,140
(D) 10,750
(E) 15,300

212

Book 1.indb 212 30/04/2019 4:47:44 PM


3.0  Quantitative Skills Practice

6. Data Sufficiency

(1) The amount of water initially in the pool is 500 litres.
(2) Water is pumped into the pool at a rate of 50
litres per minute and out of the pool at a
Directions for Questions 1–42: A question is
rate of 20 litres every 4 minutes.
followed by two statements, numbered (1) and
(2). Using the information provided and general 6. What is the price of tea?
knowledge, decide whether the information given is
(1) The price of coffee is Rs. 5 more than that of tea.
sufficient to solve the problem.
(2)
The price of coffee was Rs. 5 less than the price of
(A) Statement (1) ALONE is sufficient, but a cold drink, which cost three times the price of tea.
statement (2) ALONE is not sufficient.
(B) Statement (2) ALONE is sufficient, but 7. What is the cube root of y?
statement (1) ALONE is not sufficient. (1) The 7th root of y is 12.
(C) BOTH statements TOGETHER are sufficient, but (2) The 22nd root of y is 2.
NEITHER statement ALONE is sufficient.
8. In triangle ABC, if AB = x units, BC = x + 4 units,
(D) EACH statement ALONE is sufficient.
and AC = y units, which of the three vertex angles of
(E) Statements (1) and (2) TOGETHER are NOT triangle ABC has the greatest degree measure?
sufficient.
(1) y = x + 8
1. What percentage of a group of people are men with (2) x = 4
MBA degrees?
9. What percentage of Debating club members enrolled
(1) Of all the men in the group, 25% have MBA at a certain school are from India?
degrees.
(1) Of the Indian students enrolled at the school,
(2) Of all the women in the group, 30% have MBA 20% are members of the Debating club.
degrees.
(2) Of the non-Indian students enrolled at the school,
2. In a classroom, one student is to be selected at 45% are members of the Debating club.
random to solve a question. What is the probability 10. If the sequence S has 150 terms, what is the 121st
that a girl will be selected? term of S ?
(1) Two-fifths of the students in the classroom are (1) 
The first term of S is 32.
boys.
 he 138th term of S is 1248, and each term of S
(2) T
(2) 15 of the students in the classroom are boys. after the first is 18 more than the preceding term.

3. If a is an integer, is a + 1 even? 11. What is the selling price of the mixture if the ratio of
the two qualities of tea mixed is 3:4? (Real NMAT
(1) a + 2 is an even integer.
Question)
(2) a – 1 is an odd integer.
(1) Cost price of the first quality of tea is Rs. 180
4. If Udit saved Rs. 1200 of his earnings last month, per kg.
how much did Udit earn last month?
(2) Cost price of the second quality of tea is
1 Rs 225 per kg.
(1) Udit spent of his earnings last month on
2
12. The radius of circle O below is 8 cm. What is the
1
household expenses and saved of the area of the minor segment? (Real NMAT Question)
4
remainder.
(2) Of his earnings last month, Udit paid twice as
much in rent as he saved.
5. Water is pumped into a partially filled swimming pool .O
at a constant rate through an inlet pipe. At the
same time, water is pumped out of the pool at a
A B
constant rate through an outlet pipe. At what rate, in
litres per minute, is the amount of water in the pool
increasing?

213

Book 1.indb 213 30/04/2019 4:47:44 PM


NMAT by GMAC™ Official Guide 2019

(1) The chord AB subtends an angle greater than 20. What is the value of X, if X is an integer?
90° at the centre.
(2)
AB = 8 3 cm (1) X = 9

13. If ab is such that a and b both are integers, then is b (2) X2 – 1< 0
a multiple of 6? (Real NMAT Question)
21. Is parallelogram ABCD a rhombus?
ab is a perfect square.
(1)
(1) The four triangle enclosed by the diagonals and
ab is a perfect cube.
(2) the sides have equal areas.
14. What is the area of the quadrilateral? (2) A circle can be inscribed in ABCD touching all
 (Real NMAT Question) the four sides.
(1) The co-ordinates of the vertices are (1, 5), (2, 3), 22. Is a = b = c = 1?
(5, 4) and (4, 7).
(1) a2 + b2 + c2 = ab + bc + ca
(2) The intersection of its diagonals is at the origin.
(2) a2 + b2 = 2c2
15. A circular park has an iron railing surrounding it. The
length of the fence that surrounds circular garden M 23. Pipe A can fill a tank in ‘a’ hours and pipe B can
is 1/3 the length of the fence that surrounds circular fill the same tank in ‘b’ hours. If both the pipes are
garden N. What is the area of circular garden N? opened together for 2 hours, then what is the volume
(Assume that the fence has negligible width.) (Real (in cc) of water in the tank after 2 hours?
NMAT Question)
(1) a = 6 and b = 8
(1) The area of M is 64p square metres. (2) Volume of the tank is 100 cc
(2) The diameter of M is 1/3 that of N.
24. Four lectures Arithmetic, Biology, Chemistry and
16. The two brothers bought 4 items of antique artefacts Dermatology were scheduled, one on each day on
to decorate their showroom. However, on persistent four consecutive days, but not necessarily in that
requests from customers, both brothers each sold order. On which day was Chemistry scheduled?
two pieces. Whose transaction resulted in a better
profit? (Real NMAT Question) (1) The first lecture was scheduled on Monday, 14th
January 2016 and was followed by Dermatology.
(1) Ramu sold one item at p% profit and the other at
(2) Arithmetic was not scheduled on 16th January
p% loss though he had bought both items at the
2016 and there was a gap of one day between
same price.
Arithmetic and biology.
(2) Somu made q% profit on one item and on the
other q% loss though he sold both items at the 25. Find the cost of flooring a square room if the cost of
same price. flooring the room per square metre is Rs. 25. (Real
NMAT Question)
17. The absolute difference between a two digit number
and the number formed by reversing the digits of that (1) The square room has a veranda of uniform width
number is D. What is the number? of 3 metres around it.
(1) D = 36 (2) The area of the veranda is 432 square metres.
(2) The sum of the digits of the number is 12. 26. What is the depreciated value of a car on 1st January
1987 that was bought on 1st January 1983?
18. Who is the shortest among the five friends A, B, D, E
(Real NMAT Question)
and F?
(1) The purchase value was Rs. 60,000.
(1) D is taller than F but shorter than A and F is not
the shortest. (2) Each year, the value of the car depreciates at
the rate of 15% of its value on 1st January in
(2) E is shorter than B.
that year.
19. If a, b and c are digits, is (a + b + c) a multiple of 9? 27. If A and B are single digit non-zero numbers, what is
(A digit can be one of the integers 0, 1, 2, 3, 4, 5, the product of A and B? (Real NMAT Question)
6,7, 8 or 9.)
(1) When the two-digit number AB is added to BA,
(1) The three digit number abc is a multiple of 9. the digits at units, tens and hundreds place in
(2) [(a × b) + c] is a multiple of 9. the sum are L, 8 and 1 respectively.

214

Book 1.indb 214 30/04/2019 4:47:44 PM


3.0  Quantitative Skills Practice


(2) When the two-digit number AB is added to BA, 36. What is Mr Kashyap’s salary? (Real NMAT Question)
the digits at units place and tens place in the
(1) Mr Kashyap bought 50 kg rice with one-fourth of
sum differ by 1.
his salary.
28. If the height of a rectangular tank is 5 feet and its (2) Had the rice been 20% cheaper than its current
width is 2 feet, how many litres of water are required price, Mr Kashyap could have bought 50 kg rice
to fill the tank completely? (Real NMAT Question) for $ 100 more than one-tenth of his salary.
(1) The longest diagonal inside the tank is 10 feet 37. What is the sum of 3 successive prime numbers?
long. (Real NMAT Question)
(2) 28.317 litres of water occupy 1 cubic foot of
(1) The 3 prime numbers are in arithmetic
space.
progression.
29. Samir’s income is Rs. 9,360. Is Rahul’s income equal (2) The first of them is 5.
to Rs. 7,200? (Real NMAT Question)
38. Is z > 1? (Real NMAT Question)
(1) The difference between the incomes of the two
is Rs. 2,160. logz x > logz x 2 [x is an integer]
(1)
(2) Samir’s income is 3 per cent more than Rahul’s z2 < z
(2)
income.
x3
30. A box contains 62 balls. How many white balls are 39. Find the value of 2
. (Real NMAT Question)
y +1
there in the box? (Real NMAT Question)
x2 1
(1) The number of black balls in the box is 37. (1) =
y2 4
(2) The number of blue balls in the box is 25.
x 1
31. Which day is it on the 19th of a given month? (Real (2) =
y 2
NMAT Question)
(1) The last day of the month is Sunday. 40. Find the value of ‘n’. (Real NMAT Question)
(2) The last Tuesday of the month is 25th. (1) The number of factors of ‘n’ is only two.
32. Is x greater than y? (Real NMAT Question) (2) ‘n’ is a natural number between 5,743 and
5,779.
(1) x + y = 10
(2) x and y are prime numbers. 41. What is the speed of the boat in still water? (Real
NMAT Question)
33. If 5b = 20, what is the value of a2 + 2b? (Real NMAT
Question) (1) The boat covers a distance of 140 km in 7
hours upstream.
(1) a2 + b = 28 (2) The boat covers the same distance in 5 hours
(2) 2a2 = 12b downstream.
34. In a group of 150 students, find the number of girls. 42. If Kriti paid a total of Rs. 1,350 for two dresses and
(Real NMAT Question) sold one at 6% loss and the other at 7.5% profit,
what was the price she paid for each dress? (Real
(1) Each girl was given 50 paisa, while each boy NMAT Question)
was given 25 paisa to purchase goods totalling
Rs. 49. (1) Kriti made neither a profit nor a loss from the
transaction.
(2) Girls and boys were given 30 paisa each to buy (2) Kriti sold one dress for Rs. 705.
goods totaling Rs. 45.

35. Find the value of x. (Real NMAT Question)


(1)
x = 4096
x 64 = 8128
(2)

215

Book 1.indb 215 30/04/2019 4:47:45 PM


NMAT by GMAC™ Official Guide 2019

3.2 Answer Explanations


The following discussion is intended to familiarise you with the most efficient and effective
approaches to the kinds of problems common to quantitative skill questions. The particular
questions in this chapter are generally representative of the kinds of problem-solving questions
you will encounter on the NMAT exam. Remember that it is the problem-solving strategy that is
important, not the specific answers or details of a particular question.

4. The answer is the greatest common factor (GCF) of


1. 12 = 2 3
the two numbers.
60 = 2 15
4,000 = 2 × 2 × 2 × 2 × 2 × 5 × 5 ×5
45 = 3 5 = 25 × 53
Thus, we get 180 = 2 × 2 × 3 × 3 × 5 = 22 × 32 ×5
These numbers have 2 × 2 × 5 in common, so 20 is
2 3 2 15 3 5 3 15 5 the GCF.
× × 2 = × 2 × 1
5 2 4
3 5 2 3
The correct answer is C.
3 15 5 3 3 5 5
= × 2 × 1 = × ×
5 2 3 5 22
3 5. If t is divisible by 12, then t2 must be divisible by
 3×5  1 144 or 2 × 2 × 2 × 2 × 3 × 3. Therefore, t2 can be
= 2 =
 2 × 3 × 5  4 divided evenly by 2 at least four times, so a must be
at least 5.

The correct answer is C. The correct answer is C.

6. The key to solving this problem is realising that


1
2. 0.3 = 0.3333 = 5k+ 1 = 5k51
3
Now, 5k+1 = 2,000
1 1 1 5k51 = 2,000
Thus, 0.003 = × =
3 100 300
Now divide both sides by 5:
Thus, 7.583 = 7.58 + 0.003 5k = 400
758 1 So, 5k + 1 = 401
= +
100 300
The correct answer is B.
2275 91
= =
300 12
1
The correct answer is A. 7. Since a square root is the same as a exponent
2
1
1 and a cube root is the same as a exponent.
3. of all the pencils is 5 pencils. These 5 pencils 3
3
Therefore, converting the given expression into
each cost Rs. 2, for a total of Rs. 10.
fractional exponents, we get:
The remaining 10 pencils cost Rs. 5 each, for a total 1 1 1 1 3 2 5
of Rs. 50. +  + 6 
x 2x 3 = x 2 3
= x 6 = x 6 = 6 x5
If all of these more expensive pencils are lost, then
50 The correct answer is E.
the lost pencils represent of all the money paid.
60
8. In order to answer this question, you have to
50 understand what creates zeroes at the end of a
Thus, required % = x 100 = approximately 83%
60 number. You will notice that zeroes are created by
The correct answer is D. 10’s, each of which is created by one 2 and one 5.

216

Book 1.indb 216 30/04/2019 4:47:46 PM


3.0  Quantitative Skills Practice

So to answer this question, you simply need to work 40,000,000,000 = [a power of 10] + a
out how many pairs of 2’s and 5’s are in the given To minimise 'a' while keeping it positive, maximise
expression the power of 10 while keeping it less than 25546.
12514488 = (53)14 × (24 × 3)8 = 542 × 232 × 38 The greatest power of 10 that is less than
Even though there are 42 powers of 5, there are only 40,000,000,000 is 10,000,000,000, or 1010.
32 powers of 2, so you can only make 32 pairs of Thus:
one 5 and one 2. 40,000,000,000 = 10,000,000,000 + a
Therefore, the number of consecutive zeroes at the 30,000,000,000 = a
end = 32
The correct answer is E.
The correct answer is B.
12. The units digits of 7 to positive integers create a
9. For three consecutive integers, the possibilities are repeating pattern (this works for digits other than
[odd, even, odd] or [even, odd, even]. 7 also). By multiplying 7 by itself repeatedly in the
Since n could be an odd or an even, option (B) and calculator, you can generate the pattern:
(C) are eliminated. 71 = 7
Option (D) is true only if n is even, but not if n is odd, 72 = 49
so option (D) is also eliminated.
73 = 343
In any set of three consecutive integers, one of the
74 = 2,401
integers must be divisible by 3, but not necessarily n,
so option (A) is also eliminated. 75 = 16,807
For the same reason, (E) must be true, as n (n + 1)(n 76 = 117,649
+ 2) can be thought of as “the product of any three 77 = 823,543
consecutive integers.” Since one of these integers 78 = 5,764,801
must be divisible by 3, the product of those three
Pattern: 7, 9, 3, 1
numbers must also be divisible by 3.
To find the 86th item in a pattern of 4 repeating
The correct answer is E. items, find the remainder when 86 is divided by 4,
that is, 2. The second element in the pattern is 9.
10. Decimal placement can be determined by how The correct answer is E.
many times a number is multiplied or divided by 10.
Multiplying moves the decimal point to the right, and 13. Remember that when dealing with evenly spaced
dividing moves the decimal point to the left. Look for integers, the average is the middle value.
powers of 10 in the given fraction, remembering that
10 = 2 × 5. The sum of the even integers between 1 and 100
= 51 × 50 = 2550
17 17 17 17 17 / 125 0.136 The sum of the odd integers between 100 and 150
= = = = =
210513 21051053 (2 × 5)10 53 101053 1010 1010 = 125 × 25 =3125
   
The required difference = 2550 – 3125 = –575
There is no zero to the right of the decimal point
before the first non-zero digit in 0.136. However, The correct answer is A.
dividing by 1010 will move the decimal to the left 10
places, resulting in 10 zeros between the decimal 14. The LCM of 12, 18, 30 and 45 is 180. Thus, the
and the ‘136’ part of the number. answer has to be 8 less than a multiple of 180.
The correct answer is A. 1080 - 8 = 1072 is the only number that fits and
should be the correct answer.
11. Note that 25 x 4 = 100, and the other side of the
equation involves a power of 10. Separating out the The correct answer is A.
‘pairs’ of 25 and 4 on the left, we have:
15. It’s fastest if you try all the given options. The divisibility
25546 = 10x + a by 5 criteria will bring you down to B, C and D. B and
41(45255) = 10x + a D do not leave a remainder of 10 when divided by 12.
41(100)5 = 10x + a Only C does so and should be the correct answer.
4(1010) = 10x + a The correct answer is C.

217

Book 1.indb 217 30/04/2019 4:47:46 PM


NMAT by GMAC™ Official Guide 2019

16. It will help if you remember your squares here. Since So,
the result is a 3 digit number, the maximum value for all the clocks will show the correct time after a time
XY can be 31 since 312 = 961 and 322 = 1024. which is LCM of 24, 48, 72 and 144. The LCM is
Moreover, X cannot be 2, because its square will also 144 days.
end with 2, which is not possible. So, we need to
The correct answer is E.
check for numbers from 12 to 19 and 31 only. The
only such number is 19 as 192 = 361
19. The numbers are 11, 12, 13, 14, 21, 22, 23, 24,
Thus, P = 3; Q = 6 and X = 1 31, 32, 33, 34, 41, 42, 43, 44
And P + Q = 3 + 6 = 9
Sum of numbers = 440
The correct answer is E. Thus AA = 44

17. We need to find the square root of all the given The correct answer is D.
numbers.
20. For n = 0, 22n + 1 = 20 + 1 = 21 = 2
Only option (C) has an integral square root as 371,
that is, √1,37,641 = 371. For the other option we For n = 1, 22n + 1 = 22 + 1 = 23 = 8
won’t get an integer as square root. For n = 2, 22n + 1 = 24 + 1 = 25 = 32
The correct answer is C. For n = 3, 22n + 1 = 26 + 1 = 27 = 128
18. a : b = c : d = 2 For n = 4, 22n + 1 = 28 + 1 = 29 = 512
a = 2b Therefore, the sum of the last digits of the above
numbers = 2 + 8 + 2 + 8 + 2 = 22
c = 2d
When 22 is divided by 7, it gives a remainder of 1.
now a + b + c + d = 1 hour
The correct answer is A.
2b + b + 2d + d = 1 hour
b + d = 20 minutes 21. If a and b are odd prime numbers, then
now b : d = 3
a2 + b2 = even number
b = 3d
a + b + 1 = odd number
b + d = 20 minutes
a2 + b2 – 1 = odd number
3d + d = 20
Therefore, the H.C.F. of these numbers will be 1.
d=5
Similarly for the case a = b = 2, the H.C.F. will be 1.
Now C = 2d = 2(5) = 10
When only one of them is equal 2, it will again give
a + b + 10 + 5 = 1 hour the H.C.F. as 1.
a + b = 45 minutes
2b + b = 45 Alternatively,

a = 15 a2 + b2 – 1 and a2 + b2 are two consecutive positive


a = 2(15) = 30 integers, whose H.C.F. will always be equal to 1.
When a clock gains 12 hours or loses 12 hours then The correct answer is A.
it shows the correct time
First clock gains 30 minutes in a day means 22. The two digits at unit's place and ten's place can
12 hours gain in 24 days be only 2 or 3. So, the hundredth digit must be 6.
So, the number is either 632 or 623. The difference
Second clock loses 15 minutes in a day means
between 623 and 326 is 297.
12 hours loss in 48 days
Third clock gains 10 minutes in a day means The correct answer is A.
12 hours gain in 72 days
Fourth clock loses 5 minutes in a day means 23. Such questions can be solved easily on working with
12 hours losses in 144 days the answer choices.

218

Book 1.indb 218 30/04/2019 4:47:46 PM


3.0  Quantitative Skills Practice

Checking option (C), we get: As readings have already been collected for 292
24 days, so the number of days (more) for which the
= 4 and 24 × 2 = 48 satellites have to monitor the region together
6
= 432 – 292 = 140 days
Digits when reversed, number is 42.
The correct answer is C.
So, the difference is (48 – 42) = 6
The correct answer is C. 28. Let the numbers be 6a, 6b and 6c where a, b, and c
are co-primes.
24. If we assume that any digit out of five digits is at So, 6a + 6b + 6c = 120
fixed position, then the remaining four digits can be
⇒ a + b + c = 20
arranged in 4! (24) ways. So, each of the five digits
will appear at each of the five places 4! (24) times. Triplets for (a, b, c) = (1, 1, 18) (1, 2, 17) (1, 3, 16)
(1, 4, 15) (1, 5, 14) (1, 6, 13) (1, 7, 12) (1, 8, 11)
So, the sum of the digits at each position is 24 (1, 9, 10) (2, 3, 15) (2, 5, 13) (2, 7, 11) (2, 9, 9) (3,
(1 + 3 + 5 + 7 + 9) = 600 and hence the sum of all 3, 14) (3, 4, 13) (3, 5, 12) (3, 6, 11) (3, 7, 10) (3, 8,
such numbers will be 9) (4, 5, 11) (4, 7, 9) (5, 6, 9) (5, 7, 8) (6, 7, 7).
= 600(1 + 10 + 100 + 1000 + 10000) = 6666600 There are 24 such triplets.
The correct answer is A. The correct answer is B.

25. 180 = 22 × 32 × 51 29. There is only one 5 and one 2 that can give 0 at the
p q
We know that for a number N = a × b × c , the r end in the set of 25 prime numbers between 1 and
number of factors of N is calculates as: 100. Hence, there would be only one zero at the end
of the resultant product.
n = (p + 1) × (q + 1) × (r + 1)
Now, we should try to break-up 40 as a product of 3 The correct answer is A.
factors. The only possible value of A satisfying all the
given conditions is as below: 30. First 15 prime number are: 2,3,5,7,11,13,17,19,23,
A = 24 × 33 × 51 (As 5 × 4 × 2 = 40) 29,31,37,41,43,47
Therefore, the required value Sum = 328 which is exactly divisible only by 41 (out
A 2 ´3 ´5 4 3 1 of given options).
= = 54
40 40 The correct answer is D.

The correct answer is A. 31. Annual premium = Rs. 15,000


26. P + 2 = 2.3.5.7...61 + 2 = 2[(3.5.7,....61) + 1] Rate of premium per year = Rs. 28 for Rs. 1000
divisible by 2. 28
P + 3 = 2.3.5.7...61 + 3 = 3[(2.5.7....61) + 1] Required amount = 15, 000 × = Rs. 420
1000
divisible by 3 and so on
The correct answer is C.
All those numbers P + n of the sequence P + 2,
P + 3, ....P + 59 for which ‘n’ is prime are divisible by
32. The first multiple of 19 just after 100 = 114
‘n’ because ‘n’ already occurs as a factor in P. In all
remaining numbers, ‘n’ is composite and hence can be Largest multiple of 19 just before 500 = 494
factored into primes that are smaller than 59. Hence So, the multiples of 19 between 100 and 500 are
all terms are divisible by ‘n’ and all the numbers of the
sequence are composite. Therefore, there is no prime 114, 133, 152, ………….., 494
number in the series. Hence, the value of M is 0. This series is an arithmetic progression (AP).
The correct answer is A. First term, a = 114
Common difference,
27. LCM of 6, 4, 9,12 and 18 = 36 d = t2 – t1 = 133 – 114 = 19
Five satellites are together every 36 days. To have a Let the number of terms be n.
minimum of 12 such readings, readings have to be Last term of AP,
collected for 36 × 12 = 432 days. tn = a + (n – 1)d

219

Book 1.indb 219 30/04/2019 4:47:47 PM


NMAT by GMAC™ Official Guide 2019

⇒ 494 = 114 + (n – 1)19 while in a leap year, the number of odd days is 2.
⇒ 494 – 114 = (n – 1)19 Therefore,
380 3rd March 1897 = Tuesday + 1 day = Wednesday
⇒ (n - 1) =
19 3rd March 1898 = Wednesday + 1 = Thursday
⇒ n = 20 + 1 = 21 3rd March 1899 = Thursday + 1 = Friday
The correct answer is C. 1900 is not a leap year because it is a century year
(in case of century years, the year is a leap year if it
33. We know the cyclicity of 8 is 4, that is, the unit place is multiple of 400)
of 84n + k is 8k. 3rd March 1900 = Friday + 1 = Saturday
Digit in unit’s place of 843 = 8( 4 ×10 ) + 3 = 83 = 2 3rd March 1901 = Saturday + 1 = Sunday
Digit in unit’s place of 222 = 2( 4 × 5) + 2 = 22 = 4 3rd March 1902 = Sunday + 1 = Monday
3rd March 1903 = Monday + 1 = Tuesday
Digit in unit’s place of 956 = 9( 4 ×14 ) + 3 = 93 = 9
Hence, in year 1903 her birthday is on Tuesday
Digit in unit’s place of 2543 = 2( 4 ×135) + 3 = 23 = 8
The correct answer is D.
Therefore, digit in unit’s place of (9843 × 12222) +
72959 + 2543
37. Let the numbers be n and n +1.
= (2 × 4) + 9 + 8 = 8 + 9 + 8 = 2 5
According to the question,
Hence, the digit in unit’s place is 5.
(n + 1)2 - n2 = 91
The correct answer is A.
⇒ n2 + 2n + 1 – n2 = 91
34. 243 can be written as following: ⇒ 2n + 1 = 91
⇒ 2n = 90 ⇒ n = 45
243 = 1 × 243
243 = 3 × 81 Required sum = n + n + 1 = 45 + 46 = 91
243 = 9 × 27 Alternatively,
Hence, there are three ways of resolving the number If numbers are a and b, then
243 into two factors.
a2 – b2 = 91⇒ (a – b)(a + b) = 91
Alternatively,
Since a and b are consecutive numbers, a – b = 1.
243 = 35, and so the number of factors is 6 (5 + 1).
So, a + b = 91
Thus, the number of ways in which 243 can be
expressed as product of two factors The correct answer is D.

1 1 38. Here, we have to find the number of prime numbers


= × Number of factors = × 6 = 3
2 2 between 10 and 99. Therefore, Prime numbers
between 10 to 99
The correct answer is C.
= 11, 13, 17, 19, 23, 29, 31, 37, 41, 43, 47, 53,
a2 + 60 a2 + 4a + 4 - 4a - 8 + 4 + 60 59, 61, 67, 71, 73, 79, 83, 89 and 97
35. =
a+2 (a + 2) Hence, there are 21 such positive integers.
(a + 2)2 - 4(a + 2) + 64 Alternatively,
   =
a+2 Note that we need not count all the prime numbers.
64 We know that there are 25 prime numbers from 1 to
   = (a + 2) - 4 +
a+2 100, of which 2, 3, 5 and 7 are less than 10, and the
rest are more than 10.
Possible value of ‘a’ for the given expression to be an
So, required number of prime numbers = 25 – 4 = 21
integer = 2, 6, 14, 30 and 62
Hence, 5 positive integral values are possible. The correct answer is C.

The correct answer is B. 39. The left most digit of 220 is 1. When we multiply this
with 23, we will get the left most digit to be 8.
36. 3rd March, 1896 was Tuesday. We know that
in an ordinary year, the number of odd day is 1 The correct answer is E.

220

Book 1.indb 220 30/04/2019 4:47:49 PM


3.0  Quantitative Skills Practice

40. Given that A and B when divided by 56 leave 42. Writing the expressions for n C1 and n C2
remainders of 48 and 32 respectively, that is,
A = 56x + 48 and B = 56y + 32 n n! n
C1 = =
(n - 1) ! 1
Also, A and B when divided by 44, both leave a
n! n(n - 1)
remainder of 24, that is, n
C2 = =
(n - 2) ! 2
A = 44a + 24 and B = 44b + 24.
Finally, the sum of A and B is divisible by the sum of n(n - 1)
LCM of above two numbers =
the divisors, that is, 2
A + B = (56 + 44)k = 100k The correct answer is B.
Since A = 56x + 48, we can write this as A = 44x +
a +b +c +6
12x + 48 = 44x + 12x + 24 + 24. 43. 6= or, a + b + c = 19
5
So, 12x + 24 should be divisible by 44, which is
possible when x = 9. It is not necessary, or possible, to determine the
values of a, b, and c individually. The second average
Therefore, the smallest value of A = 56 × 9 + 48 includes all three variables, so the values will be
= 552 summed again anyway.
General value of A = LCM(56, 44) k + 552 =
616k + 552  a + b + c + 13  32
Required average =  = =8
Similarly, B = 56y + 32 = 44y + 12y + 24 + 8,  4  4
which means 12y + 8 should be divisible by 44.
This is possible when y = 3. The correct answer is A.
So, the smallest value of B = 56y + 32 = 56 × 3 + 32
44. Anil + Hemant = 8050
= 200
General value of B = 616m + 200 Therefore, Hemant = 8050 - Anil
Now, Hemant + Chandan = 12500
A + B = 616k + 552 + 616m + 200 Therefore, Hemant = 12500 - Chandan
= 616(k + m) + 752, 8050 - Anil = 12500 – Chandan
which will be divisible by 100, when k + m = 3 Or,   Chandan - Anil = 12500 - 8050 = 4450..........i
Thus, the smallest value of A + B = 616 × 3 + 752 And, Chandan + Anil = 10400.................................ii
= 2600 Adding equations i and ii, we get:
The correct answer is C. 2 Chandan = 14850
Therefore, Chandan = 7425
41. The given equation is, Hemant = 12500 - Chandan
2 2
a + 2b = b + 2a + 5    = 12500 - 7425
2 2
a + 2b – b – 2a = 5    = 5075
⇒ (a – b) (a + b) – 2(a – b) = 5 The correct answer is B.
⇒ (a – b)(a +b – 2) = 5
45. Weight of the teacher
⇒ a – b)(a +b – 2) = 1 × 5
= Average weight + Weight contributed by increase
⇒ (a – b)(a +b – 2) = (–1) × (–5)
in average
⇒ Both the factors on the left-hand side are integers,
and the only integer factors of 5 are ±5 and ±1. = 42 + 38 × 2 = 118 kg
So, a – b = ±5 or ±1. The correct answer is D.
This gives us four possibilities, but the only solution 46. Let number of men be n.
for which both a and b are positive is a – b = 1 and a
+ b – 2 = 5, gives us a = 4 and b = 3. Total weight of n men initially = 50 × n = 50n.....(i)
The correct answer is D.

221

Book 1.indb 221 30/04/2019 4:47:50 PM


NMAT by GMAC™ Official Guide 2019

After 2 persons leave, then resultant weight 51. The expenditure on rent, utilities, and insurance = 50%
= 52(n – 2).....(ii)
The expenditure on food = 20%
Eq. (i) and (ii) can be written as
The total expenditure on rent, utilities, insurance and
50n = 52(n – 2) + 80 food = 50% + 20% = 70%.
  n = 12 The income left = 100% – 30% = 30%
The correct answer is E. The expenditure on video games = 30% of 30%
= 0.30 × 0.30 = 0.09, or 9%.
47. Let average weight of class 8th, 9th and 10th students Therefore, the percentage of the income left after all
be ka, kb and kc. of the expenditure = 30% – 9% = 21%
Let number of students in class 8th, 9th and 10th be The correct answer is D.
jx, jy and jz.
Total weight of all students 52. 90% of 40 students or 0.9(40) = 36 students had
Then average weight = lower marks than Varun.
Total number of students
Of the 60 new students, 80% or 0.80(60) = 48
=
(ka × jx ) + (kb × jy ) + (kc × jz) students had lower marks than Varun.

jx + jy + jz
Thus, 36 + 48 = 84 students in the new, larger class
jk(ax + by + cz)
= have marks lower than Varun.
j(x + y + z) The new class has 100 students, 84 of whom have
k(ax + by + cz) lower marks than Varun.
=
x + y + z There are 16 students remaining, one of which is
Varun. Since Varun has the lowest marks of this
Here we do not have the value of k and hence it group of 16 students, there are 15 students above
cannot be determined. him.

The correct answer is E. Since the class has exactly 100 students, the
15
48. The average age of husband and wife now is 30 required percentage = × 100 = 15%.
100
years and the average age of husband, wife and the
children now is 16 years. The correct answer is D.

Total ages of children = (16 × 4) – (30 × 2) 53. Machine Y’s work in a 24-hour period exceeds
  = 4 years Machine X’s work by 48,000.
As all of them have different ages with integral Let Machine X’s packing rate in books per hour be a
values, the only possibility is (1, 3) years.
That is to say:
The correct answer is A. 36a – 24a = 48,000
12a = 48,000
49. Let Ramakant's total salary be Rs. S.
a = 4,000
Amount left with him = 0.29 × 0.71 × S = 39121 Machine X packs 4,000 books per hour.
So, S = Rs. 1,90,000 The correct answer is A.

The correct answer is E. 54. Let the production of rice in 1990 be 100 tonnes.

50. The number that is 50% greater than 80 = 1.5(80) Then, the production of rice in 1995 = 100 × 1.75
= 120. = 175 tonnes
Similarly, the number that is 25% less than 200 And the production of rice in 2000 = 175 × 2
= 0.75(200) = 150. = 350 tonnes
30 Thus the percentage increase in production from
Thus, the required percentage = × 100 = 20% 1990 to 2000 = (350 - 100)/100 × 100 = 250%
150
The correct answer is D. The correct answer is A.

222

Book 1.indb 222 30/04/2019 4:47:51 PM


3.0  Quantitative Skills Practice

55. Let 5 pizzas, 7 samosas and 4 ice-creams cost 60. Amount of iodine in the mixture = 16% of 735 gm
Rs. x. That is,
Now, the amount of iodine becomes 20% of the
5 pizzas + 7 samosas + 4 ice-creams = x ... (i) mixture.
Then, 6 pizzas, 14 samosas and 8 ice-creams will If amount of mixture after evaporation is x, then
cost Rs. 1.5x. That is, 20% of x = 16% of 735 or x = 0.8 × 735
6 pizzas + 14 samosas + 8 ice creams = 1.5x ...(ii) Amount evaporated = 735 – 0.8 × 735 = 147 gm
In the first case, if Ankur has decided to buy all the
The correct answer is C.
items double, it would cost him Rs. 2x. That is,
10 pizzas + 14 samosas + 8 ice-creams = 2x ...(iii) 61. Required percentage increase
Subtracting Eq. (ii) from (iii), we get:
600 - 200
4 Pizzas cost Rs. 0.5x. = ´ 100 = 200%
200
So, 5 Pizzas will cost Rs. 0.625x.
This is the amount that Ankur have spent on Pizzas. The correct answer is D.
Hence, fraction of the total amount paid
62. Suppose the number of apples on the first day = Y
= 0.625 = 62.5%
Apples sold on the first day = 0.5Y
The correct answer is E.
Apples gone rotten on the first night = 0.05Y
100 Apples remaining on the second day = 0.45Y
56. Cost price = 260 × = Rs. 200
100 + 30 Apples sold on the second day = 0.225Y
The correct answer is B. Apples gone rotten on the second night = 0.0225Y
Apples remaining on the third day = 0.2025Y
57. Let cost price be Rs. 100.
Apples sold on the third day = 0.10125Y
Then, marked price = Rs. 130 Apples gone rotten on the third night = 0.010125Y
On this a discount of 5% is given. Hence, the total number of rotten apples
So, discount = 5% of 130 = Rs. 6.5 = 0.05Y + 0.2025Y + 0.010125Y = 0.082625Y
Hence, the selling price of goods As per the question, we have:
= Rs. (130 – 6.5) = Rs. 123.5 0.082625Y = 3305
So, the profit percentage = 23.5% So, Y = 40000 apples
The correct answer is C. The correct answer is D.

58. (p % q + q% of p) 63. Suppose the original price of the watch is Rs. 100.
pq qp 2pq
= + = So, it becomes 90 after reduction. In order to bring
100 100 100 its value back to 100, we need to increase the value
= 2% of pq by 10.
The correct answer is C. So, the required percentage increase

59. Given that, p = 5q 10


= × 100 = 11.11%
90
Percentage by which q is less than p
p-q The correct answer is A.
= ´ 100
p
64. Let the share of Farhan be x, then share of Fahim
5q - q must be (37, 570 – x)
= ´ 100
5q Now, both invested money at the same compound
4q rate at an interest of 10%
= ´ 100 = 80% According to given condition, x(1 + 10%)9
5q
= (37, 570 – x)(1 + 10%)7
The correct answer is D.

223

Book 1.indb 223 30/04/2019 4:47:52 PM


NMAT by GMAC™ Official Guide 2019

So, x(1 + 10%)2 = 37,570 – x According to the question,


2
So, x(1.1) = 37,570 – x PR2
4P =
1.21x = 37,570 – x 10000
Solving it we get x = Rs.17,000 ⇒ R = 200%
The correct answer is C. The correct answer is D.

t
Alternatively,
  r 
65. A = P 1 +   We know that difference between SI and CI for the
  100  
PR2
first two years is . It is given that difference of
(where A = amount; P = principle; r = rate of 1002
interest; t = time intervals) CI in the second year and that in the first year is 4
2 times the principal. Since CI for first year is same as
 
or, 1650 = 1200 1 +  r   SI for each of the years, the above difference is also
  100   the difference between CI of the first and the second
year.
2
or, 1650 = 1 +  r   So,
PR2
= 4P or r = 200%
1200   100   1002
 r 
or, 1.375 = 1 +  68. Let the amount be 2x.
 100 
Interest earned on amount deposited at
r 
or, 1.172 = 1 + 
 100  ( 4x )(10)
simple interest = = 0.4x
Thus, r = approximately 17%. 100

The correct answer is A. Interest earned on amount deposited at compound


éæ 10 ö
3
ù
66. Given that an amount ‘A’ becomes three times in 6 interest = x êç 1 + ÷ - 1ú = 0.331x
êëè 100 ø ûú
years which means increment of ‘2A’ occurs in 6
years. That is, increment of ‘A’ occurs in 3 years. Difference = 0.069x = 1000
For the amount to be 24 times, increment of ‘23A’ is
⇒ x = 1000
required. Therefore, it will take 69 years to become 0.069
24 times.
Therefore, total initial amount
The correct answer is C.
2000
= 2x = = Rs. 28,985
67. Let us consider the compound interests for the first 0.069
and the second year be CI1 and CI2 respectively. The correct answer is B.
PR
CI for the first year, CI1 = 69. According to the question,
100
2 2
æ r ö æ 10 ö 2
é æ R ö
2
ù 121500 = P ç 1 - ÷ = P ç1 - ÷ = P(0.9)
CI for the second year, CI2 = êP ç 1 + ÷ - P ú - CI1 è 100 ø è 100 ø
êë è 100 ø úû
121500
é æ 2
ù PR 2 ⇒P= = 150000
R ö æ R ö PR 0.92
= êP ç 1 + ÷ - Pú - = Pç ÷ +
êë è 100 ø úû 100 è 100 ø 100
The correct answer is B.
Therefore, the difference between the compound 70. Let the rate of interest be r.
interests for the first year and the second year is
given by: According to the question,
2
PR 800 ´ 2 ´ r 200 ´ 3 ´ r
CI2 – CI1 = + = 275
1002 100 100

224

Book 1.indb 224 30/04/2019 4:47:54 PM


3.0  Quantitative Skills Practice

Solving the above equation, we get: 2


x  5 
r = 12.5% ⇒ = 1+
16820 - x  100 
The correct answer is B. 2
x  21
⇒ =
PRT 16820 - x  20 
71. Simple interest =
100 ⇒ 400x = 441(16820 – x)
P´R´8 ⇒ 841x = 16820 × 441
P =
100
16820 × 441
R = 12.5% ⇒x=
841
The correct answer is A. ⇒ x = Rs. (20 × 441) = Rs. 8820

Alternatively, Seeta’s share = Rs. 8820


In case of simple interest, an amount doubles if the Geeta’s share = Rs. (16820 – 8820) = Rs. 8000
rate of interest becomes 100%. The correct answer is B.
Let us consider the rate of interest is r.
Then, the time period ‘td’ in which total interest 74. Rate of interest = 15 paise per rupee for 3 months
becomes 100% is given by as below: Rate of interest per annum
100 100 100 15 paise per rupee
td = or r = = = 12.5% = × 12
r td 8 3
= 60 paise per rupee
72. An amount doubles if the rate of interest becomes 60
100%. = = 0.6
100
Let us consider the rate of interest compounded
= (0.6 × 100) = 60% per annum
annually is r.
Then, we know that the time period ‘td’ in which total The correct answer is D.
interest becomes 100% is given by as below:
75. Let the principal be Rs. x.
72
td = Time = 3 years
r
Rate = 11% per annum
72 72
Or, r = = = 9% Simple interest,
td 8
Principal × Time × Rate
The correct answer is A. SI =
100
73. Let Seeta’s share be Rs. x. x × 3 × 11 33x
SI = = Rs.
100 100
Then, Geeta’s share = Rs. (16280 – x)
Compound interest
Rate = 5% per annum
Time  Rate 
Time

 Rate  CI = Principal  1 + - 1
Amount = Principal  1 +
 100  
 100  
 
3
According to the question 11 
CI = x  1 +  - 1
40 - 27 40 - 25  100  
 5   5 
x 1 + = (16820 - x )  1 +
100  100   111 
3
 
CI = x   - 1
15 
 100  
 5 
x  1 + 100  1367631 - 1000000
⇒ = CI = x ×
16820 - x  5 
13
1000000
 1 + 100  367631x
CI = Rs.
1000000

225

Book 1.indb 225 30/04/2019 4:47:55 PM


NMAT by GMAC™ Official Guide 2019

According to the question, 78. Mukesh paid 1/2 of what others paid.

367631x 33x ⇒ Mukesh paid 1/3rd of the total amount = £ 20


⇒ - = 37631
1000000 100 Similarly,
367631x - 330000x Manish paid 1/4th of the total amount = £ 15 and
⇒ = 37631 Lalu paid 1/5th of the total amount = £ 12.
1000000
37631x Remaining amount of £ 60 – £ 20 – £ 15 – £ 12
⇒ = 37631
1000000 = £ 13 is paid by Jaggi.
⇒ x = 1000000 The correct answer is A.
Thus, principal = Rs. 10,00,000
79. Since their investments in the business are for the
The correct answer is D. same duration i.e. 1 year, profits will be shared in the
76. Let’s assume Sanya and Babli invested for 12 months ratio of their investments i.e. 21000 : 17500
= 6 : 5.
Therefore, Jhanvi invested for 12/4 = 3 months
6
Let Sanya’s investment be 100 So, Yogesh’s share = × 26400 = Rs. 14400
11
Therefore, Jhanvi’s investment = 100 × 2 = 200
5
And, Babli’s investment = 0.5 × 100 = 50 Mohan’s share = × 26400 = Rs. 12000
11
Therefore, their total contribution is:
The correct answer is A.
Sanya = 100 × 12 = 1200
Jhanvi = 200 × 3 = 600 80. The chemist now has 10 litres of ink in a 30-litre
Babli = 50 × 12 = 600 mixture, so she must have 20 litres of water. You
Thus, the ratio of Sanya : Jhanvi : Babli want to know the amount of ink you must add in
order to make this mixture a 50% solution. Since no
= 1200 : 600 : 600
additional water is added, the solution must finish
=2:1:1 with 20 litres of water. Therefore, she also needs a
Therefore, the share of Sanya and Babli in profit total of 20 litres of ink, or 10 more litres than the
3 mixture currently contains.
= × 440000
4 The correct answer is C.
= 330000
The correct answer is E. 81. 200% bigger means ‘three times as big as’ the
original. If the pitcher is three times as big as the
glass, then pouring the contents of the glass into the
77. Let Tina’s share be T, Ishan’s share be I, Abhishek’s
share be A and Fatima’s share be F. 1
pitcher will make the pitcher full. If adding another
I 80A 3
Given that T + 3 = I + = =F–4 16 litres fills up the pitcher, the 16 litres must be
3 100
We get: 2
equal to the remaining of the pitcher’s capacity.
T = F – 7................(i) 3
3 1 16
I = (F – 4)................(ii) Then, of the pitcher’s capacity = = 8 litres.
4 3 2
5 So, the quantity of the juice mixture = 8 litres.
A = (F – 4) ................(iii)
4 Therefore, the volume of lime juice in the mixture =
Also given that, T + I + F + A = 80................(iv) 20% of 8 litres.
Substituting the values from Eq. (i), (ii) and (iii) in Eq. = (8)(0.2) = 1.6 litres
(iv), we get: The correct answer is A.
é 3F 5F ù
ê F + 4 + 4 + F ú – 7 – 3 – 5 = 80 82. The ratio of boys to girls is 6 : 7. If you introduce
ë û
the unknown multiplier x, the number of boys is 6x,
F = Rs. 23.75 and the number of girls is 7x, where x is a positive
The correct answer is C. integer.

226

Book 1.indb 226 30/04/2019 4:47:56 PM


3.0  Quantitative Skills Practice

According to the question, After the second shift, percentage of students in C1


7x – 6x = 2 is 45%, which means 15 students must have shifted.
or, x = 2
a abc
Finally, substitute the value of x into the expression 85. = =3
b b2c
for the number of boys: 6x = 6(2) = 12. There are 12
boys on the team. b bc2
= =3
c c2
The correct answer is A. c cd
= =3
d d2
83. Use the weighted average formula to get the ratio of
X to Y: b2c + c2 + d2 b2c + c2 + d2 1
= =
2 2
(
abc + bc + cd 3 b c + c + d
2 2
3 )
 0.55x + 0.70 y  , where x is the amount of
 x+y  = 0.65 The correct answer is B.

X and y is the amount of Y. 86. Given that solution X contains 50% milk and solution
0.55x + 0.70y = 0.65(x + y) Y contains 30% milk. Using the Alligation method,
we can determine the ratio of volume of the two
0.55x + 0.70y = 0.65x + 0.65y mixtures:
55x + 70y = 65x + 65y
50 30
55x + 5y = 65x
5y = 10x
y 2
Thus, = 45
x 1
2
Since y and x are in a 2 to 1 ratio, of the total is y
3
1
and of the total is x. 15 5
3
1 3 : 1
Since the total is 120 grams, X accounts for x
3 Therefore, the required ratio = 3:1
120 = 40 grams of the mixed cereal. The correct answer is E.
The correct answer is B.
87. Let the required time be ‘x’ seconds.
84. Let the number of students in the two classes C1 and Volume of solution transferred = 10x ml
C2 be 2x and 3x respectively.
Volume of milk transferred = 5x ml
Given (2x – 10) : (3x + 10) = 3 : 7.
Volume of water transferred = 5x ml
Solving, we get x = 20. So, number of students in
Ratio of volume of milk to the volume of water in the
C1 and C2 is 30 and 70 respectively. solution = 5x : 5x + 100 = 1 : 3
Let y be the number of students to be shifted from ⇒ x = 10 seconds
C2 to C1.
So, (30 + y) : (70 – y) = 9 : 11 The correct answer is B.

Solving, we get: y = 15 88. Type I Mixture:


The correct answer is B. Total weight of mixture = 20 tonnes
Alternatively, Ratio of sand to cement = 60 : 40
Initially, we had 40% students in C1. Even after the Weight of cement
shift, the total number of students remains the same. 40
The percentage of students in C1 (after the shift) = 20 × = 8 tonnes
60 + 40
= 30%, which means 10% of total students shifted.
Since we are given that 10 students shifted, the Type II Mixture:
number of students earlier in C1 and C2 are 40 and Total weight of mixture = 30 tonnes
60, and after the shift, it becomes 30 and 70. Ratio of sand to cement = 30 : 70

227

Book 1.indb 227 30/04/2019 4:47:57 PM


NMAT by GMAC™ Official Guide 2019

Weight of cement 92. Arvind’s per hour sale = Rs. 78


70 Amount paid to the salesman per hour = Rs. 10
= 30 × = 21 tonnes
30 + 70 Therefore, the profit per hour = Rs. 68
Weight of cement in final mixture Also, the amount paid for the roadside market per
= 21 tonnes + 8 tonnes = 29 tonnes day = Rs. 150
So, the formula for his daily profit will be
The correct answer is D.
Profit = Revenue – Expenses
89. Let the number of worksheets received by each P(h) = 68h – 150
student of section A be 5x. The correct answer is C.
Then, the number of worksheets received by 2
students of section A = 10x 93. We know that profit equals revenue minus cost.
So, the number of worksheets received by each Therefore, the company’s profit is:
students of section C p (9 – p) – (p + 15) = 9p – p2 – p – 15
10x = –p2 + 8p – 15
= = 2x
5 = –(p2 – 8p + 15)
= –(p – 5)(p – 3)
And, the number of worksheets received by each
student of section B When p = 5 or p = 3, the profit becomes 0), so
eliminate (A) and (C). For p > 5, the profit becomes
2x negative (that is, the company loses money). The
=3× = 3x
2 profit is only positive if (p – 5) and (p – 3) have
opposite signs, which occurs when 3 < p < 5.
According to the question,
The correct answer is B.
(5x × 6) + (3x × 12) + (2x × 17) = 100
⇒ 30x + 36x + 34x = 100 94. Selling price of smartphone = 8000 × 1.25
⇒ 100x = 100
    
= 10000
⇒ x=1
CP of 2nd phone = 10000
Number of worksheets received by student of section
SP of 2nd phone = 10000 × 0.8 = 8000
C = 2x = 2 × 1 = 2
Hence, overall the person made no profit or loss.
The correct answer is B.
The correct answer is E.
16 4 16
90. Given ratios are 8 : and : .
3 3 5 95. If CP is Rs. 100 for 1 kg, actual CP = Rs. 84
Given that the compound ratio is 15 : x. So, And, SP = Rs. 96
4 16 16 12 2
8× : × = 15 : x So, profit = × 100 = 14 % gain
3 3 5 84 7
32 256 15
⇒ : = The correct answer is B.
3 15 x
On solving, we get x = 24 96. Profit = 15%, Discount = 5%

The correct answer is E. 15(−5)


So, net profit% = 15 – 5 +
100
91. a-b+b-c+c–d= d–e
   = 10 – 0.75 = 9.25%.
⇒ a + e = 2d
The correct answer is C.
a+e
⇒ =2
d 97. The company saves Rs. 140 from the first person,
The correct answer is E. Rs. 100 from the second and Rs. 50 from the third
person and loses Rs. 100 from the fourth player. So,

228

Book 1.indb 228 30/04/2019 4:47:58 PM


3.0  Quantitative Skills Practice

Net profit = 140 + 100 + 50 – 100 = 190 30 km per litre × 8 litres = 240 km
Therefore, profit per cycle = Rs. 190 So the bus will end up 240 km to the north of its
83 people are playing means 20 complete cycles 240
starting point, which happens = 6 hours after it
and 3 persons more. 40
started.
Total profit = (20 × 190) + 140 + 100 + 50
The truck started an hour later and thus travelled (30
= 3800 + 290 = Rs. 4,090 km per hour)(6 hours – 1 hour) = 150 km by that
The correct answer is C. time.
Therefore the two vehicles were 240 – 150 = 90 km
98. Let selling price of 1 pen be Re. 1. apart when the bus ran out of fuel.

Then, selling price of 630 pens = Rs. 630 The correct answer is C.
And, profit = Selling price of 90 pens = Rs. 90 General Solution for Questions 79–82
Therefore, cost price of 630 pens
= Rs. (630 – 90) = Rs. 540 Av speed Distance
Time (actual
Rest Total time
Vehicle journey in
Thus, required profit percentage (km/h) (km)
hours)
time(hrs) (hrs)

90 Car 60 210 3.5 0.25 3.75


= × 100 = 16.67%
540 Bike 45 195 4.33 0.25 4.58

The correct answer is C. Bicycle 20 90 4.5 0.5 5


Van 40 250 6.25 0.75 7
99. Use alligation Train 75 300 4 1 5
5 4
Fraction of milk in P and Q are and
7 5 101. From the above table it is clear that car took the
least travel time.

respectively. The resultant mixture has milk fraction The correct answer is C.

102. Time taken by bicycle without rest = 4.5 hours


3
. Refer to the following diagram: Change in time due to rest = 0.5 hours
4
Hence, percentage change in time
5/7 4/5 0.5
× 100 = 11.11% = 11% approximately.
4.5
3/4
The correct answer is A.

1/20 : 1/28 103. Average Speed = Total distance


Total time taken
Total distance
So, total time taken =
4 3 16 − 15 1 Average speed
− = =
5 4 20 20
3 5 21 − 20 1 For car, total time taken = 210 = 3.5 hours
Similarly, − = = 60
4 7 28 28 Similarly,
195
1 1 For bike, total time taken = 45 = 4.33 hours
Therefore, required ratio = :
20 28
90
= 28 : 20 = 7 : 5 For bicycle, total time taken = = 4.5 hours
20
The correct answer is D. 250
For van, total time taken = = 6.25 hours
40
100. Since the limiting factor in this case is the bus’ fuel
supply, we must calculate how far the bus is able to 300
For train, total time taken = = 4 hours
drive before running out of diesel. That is, 75

229

Book 1.indb 229 30/04/2019 4:47:59 PM


NMAT by GMAC™ Official Guide 2019

Adding them all, we get: Using the above formula, we get:

Total time = 22.58 hours Ratio of speeds = 50 : 10 = 5 :1


Total distance covered by all modes of travel The correct answer is A.
= 1045 km
Alternatively,
Total time taken for the actual journey = 22.58 hours
Let speeds of P and Q be x and y respectively.
Average speed = 46 kmph
Then, after crossing each other, the distance
We need not find the times taken, as they have travelled by P = 10x and that by Q = 50y.
already been found out in the table.
Also, when they started their journey and met at
The correct answer is D. some point M, then ratio of their speeds will be equal
to the ratio of the distances covered by them.
104. Calculating the minimum travel time for the following:
So, x : y = 50y : 10x
Bicycle + Car = 1.75 + 2.25 = 4 hours
Bicycle + Bike = 2.25 + 2.165 = 4.415 hours ⇒ x2 : y2 = 5 : 1 ⇒ x : y = 5 :1.
Car + Bike = 1.75 + 2.165 = 3.915 hours
Car + Train = 1.75 + 2 = 3.75 hours S12 + S22 a2 + b2
107. Given that, =
S12 − S22 a2 − b2
Train + Bike = 2.165 + 2 = 4.165 hours
Applying componendo and dividendo, we get:
The correct answer is D.
S 2 a2
Alternatively, 12 = 2
S2 b
The least time taking vehicles are car and train, so
the sum of the half of their times is also going to be S1 a t2
the least. So, = =
S2 b t1
105. Let speeds of Ramesh and Upendra be 11 km/hr and
2 km/hr respectively. When Ramesh travels 11 km, t1 b
⇒ =
Upendra travels 2 km. Currently speed of Ramesh is t2 a
11 The correct answer is B.
= 5.5 times that of Upendra. After meeting each
2
other, Upendra has to travel 11 km, and Ramesh 108. Ranipur mail's speed
has to travel 2 km. If they both want to reach the Dhampur mail's speed
respective ends at the same time, then ratio of Time taken by Dhampur mail to reach Ranipur
=
speeds of Upendra to Ramesh = 11 : 2 = 5.5 : 1 Time taken by Ranipur mail to reach Dhampur
= 11 x 5.5 : 11 = 60.5 : 11. But Upendra’s speed
45 200 5
60.5 = =
right now is 2 km/hr. So, x should be = 30.25 Dhampur Mail's Speed 288 6
2
so that they both reach together. Since Upendra Dhampur Mail’s speed = 54 km/hr
The correct answer is B.
should reach earlier, his speed should be more and
so x > 30.25.
109. Given that the ratio of time taken is 3:2.
The correct answer is E. If difference is 1 minute, Amar takes 3 minutes.
If difference is 10 minutes, Amar takes 30 minutes.
106. If two objects P and Q start at the same time in Therefore, at double speed, Amar takes 15 minutes.
opposite directions from points A and B respectively
The correct answer is A.
and after passing each other, P reaches B in p
seconds and Q reaches A in q seconds. 110. Let the speed of the boat in still water be u and the
speed of the flowing river be v.
Then, speed of P : speed of Q = q: p

230

Book 1.indb 230 30/04/2019 4:47:59 PM


3.0  Quantitative Skills Practice

Therefore, upstream speed = u – v 3


And, downstream speed = u + v 114. After accident, the speed of the train becomes of
4
So, 5 : 1 = (u + v) : (u – v)
4
⇒ u + v = 5u – 5v its original speed, so the time taken would be of
⇒ 4u = 6v 3
So, the required ratio = 6 : 4 = 3 : 2. the original time. So, due to accident the train had to
The correct answer is A. 1
travel for an extra duration of of the original time.
3
111. Ratio of speed of Sumit : Vinit : Hari Now, as the train was 1 hour late, it means the actual
= 200 : 180 : 170 = 20 : 18 : 17 time required for the remaining journey was 3 hours.
In the time Vinit covers 3600 m, Hari can cover It is also given that if the accident had occurred a
further 60 km on from the place of the accident, the
3600 × 17
= m delay would have been only 30 minutes. It means
18 then the train would have taken an extra time of
= 200 x 17 = 3400 m 30 minutes in addition with the actual time of 90
So, Vinit can beat Hari by 200 m. minutes. It means in 90 minutes the train can travel
60 km when it is travelling with the original speed.
The correct answer is C.
So, original speed is 40 km/h.
112. Ratio of distances = 10:20:30 = 1:2:3 Distance travelled in 3 hours = 120 km
1 1 Thus, total distance travelled by the train
Ratio of speed = : : 1 = 3:2:6
2 3 = (180 + 120) km = 300 km
1 1
So, ratio of time = :1: = 2:6:3 The correct answer is B.
3 2
The correct answer is A. 115. As the product of time taken to construct a hut and
the number of workers doing the work is always
113. Speed of Train 1 = 48 Km/h constant, we have:
Time taken by Train 1 to reach Station 2 after (3 hours) x (40 workers) = (t hours) x (140 workers)
meeting = 4 hours 120 6
or, 
t= = hours
Time taken by Train 2 to reach Station 1 after 140 7
meeting = 9 hours 6
= × 60
We know that, 7
= approximately 51 minutes.
Speed of Train 1
= The correct answer is A.
Speed of Train 2
Time taken by Train 2 to reach Station 1 116. The resort has 20(14) = 280 of wood.

Time taken by Train 1 to reach Station 2 If the resort only needs to heat 15 rooms instead
48 9 of 20, divide 280 by 15 to get 18.666…. You are
⇒ = asked for extra FULL days,
Speed of Train 2 4
So extra FULL day will be 4.
48 × 2
Speed of Train 2 = = 32 km/h The correct answer is B.
3

Distance of Train 1 from destination after 5 hours 117. Ajay and Firoz’ combined rate = 1 + 1 = 7
4 3 12
= 480 – 5(48) = 480 – 240 = 240
14
Distance of Train 2 from destination after 5 hours So, in 2 hours, they should have completed
12
= 480 – 5(32) = 480 – 160 = 320 7
= of the job.
6
Required ratio = 240 : 320 = 3 : 4 7 1
Therefore, Firoz’ breaks cost them - 1 = job
The correct answer is E. worth of productivity. 6 6

231

Book 1.indb 231 30/04/2019 4:48:01 PM


NMAT by GMAC™ Official Guide 2019

Thus, Firoz’ break time = the amount of time it would Therefore, work remaining = 1 - 31/36
1 = 5/36
have taken him to do of the job = 30 minutes
6 Time taken by Sumit to complete 5/36 of the work
Therefore, each of his 3 breaks was = 5/36 × 24 = 3.33 days
30 minutes The correct answer is C.
= 10 minutes long.
3
The correct answer is B. 121. Without the hole, the two taps will fill the tank in
118. First, figure out how many boxes worth of pens the 1 20
=
machine produced in the 2 hours that it was on. 1 1 9 minutes
+
20 pens per hour is the rate, and 2 hours is the time: 4 5
Work = (20 pens per hour) × (2 hours) = 40 pens. Let the hole empties the tank in X minutes, then
Now, since there are 10 pens per box, compute the 1 20 1
number of boxes: = +
1 1 1 9 2
 1 box  + −
The number of boxes = 40 pens ×  4 5 x
 10 pens 
= 4 boxes 1 81
⇒ =
So Mahesh must pack 4 whole boxes to X 980
accommodate all the pens that the machine had
980
made. ⇒x= minutes
81
Mahesh’s rate is 3 boxes per hour, while the total
work is 4 boxes. Rearrange and plug in: The correct answer is B.
æ4ö
Time = ç ÷ hours 122. If x, y and z are the time taken by P, Q and R
è3 ø respectively to fill the cistern alone, then
4
= ´ 60 = 80 minutes 1 1 1 1
3 + + = ................(i)
x y z 3
The correct answer is C.
Also, P works for 1 hour, whereas Q and R work for
119. Pipe 1 can fill 1/7 of tank in 1 hour 5 hours in total. Then,

Pipe 2 can fill 1/5 of tank in 1 hour 1 1 1


+ 5 × + 5 × = 1................(ii)
Pipe 2 and the waste pipe together fill 1/8.5 of tank x y z
in 1 hour Multiplying Eq. (i) by 5 and subtracting Eq. (ii) from it,
So, the fraction of tank which waste pipe can empty we get:
in 1 hour = (1/5 - 1/8.5) of tank = 0.7/8.5 of tank 1 5 2
If all the pipes are working together, in 1 hour the 4 × = −1 =
x 3 3
tank filled is 1/7 + 1/5 - (0.7/8.5) = 1/3.84 of tank.
So, the tank will be filled in 3.84 hours = 3 hours 50 x = 6 hours
minutes. So, P alone can fill the cistern in 6 hours.

The correct answer is B. The correct answer is D.

120. Harsh can complete the entire project in 6 × 3/2 123. P works for X + 5 minutes whereas Q works for X
= 9 days minutes only. So,
Sumit can complete the entire project in 8 × 3/1 1 1
(X + 5) × +X× =1
= 24 days 12 16
Mini can complete the entire project in 12 × 4/3 Or X = 4 minutes
= 16 days The correct answer is C.
Total work done by all 3 in one day = 1/9 + 1/24
+ 1/16 = 31/144 124. In one hour,
Therefore, total work done by all 3 in 4 days
Suppose the quantity of water from pipe Y = w1
= 31/144 × 4 = 31/36

232

Book 1.indb 232 30/04/2019 4:48:02 PM


3.0  Quantitative Skills Practice

The quantity of water from pipe X = w2, 127.


And, acid from pipe X = a Skilled workers Days Hours Work
In first case, when the bucket is full,
2 15 12 1
a 8 x 33
= ................(i) 5
w1 + w 2 + a 100
y 11 9 1.5
w + w2 + a w w
1 = 12.5 or 1 + 2 = 11.5
a a a  2  15   12   3 
Thus, y =   × 33 ×   ×  ×   = 36
 5  11   9   2 
In second case, the bucket is not full
a 10
= The correct answer is A.
w1 100 ................(ii)
+ w2 + a
2 Alternatively,
w W2 M1D1H1 M2D2H2
1 + =9 Use the formula: = .
2a a W1 W2
2 
W2 × 33  × 15 × 12
⇒ = 18 – 11.5 = 6.5 = 13 : 2 So,  5  y × 11 × 9 or y = 36
a =
1 1.5
The ratio of acid to water in pipe X = 2 : 13
128. Since each of the two assistants work ¾ as fast as
The correct answer is E.
Ajay, all the three put together will work
1
125. Part of the cistern filled by tap P in 1 minute =
12 3 3 5
1 + + = as fast as ajay alone can do it.
4 4 2
1
Part of the cistern filled by tap Q in 1 minute =
18 Hence, if all three work together, they can finish the
Part of the cistern filled by both the taps in 1 minute 2
work in of the time taken by Ajay alone to do the
5
1 1 5
= + = work.
12 18 36
36 1 The correct answer is D.
So, the cistern can be filled in = 7 min
5 5 Alternatively,
The correct answer is D. Let Ajay does 4 units per day. The job is 4 units of
work. So assistants would do 3 units each per day.
126. Given that one worker can dig 250 m of tunnel in a
week. It means one worker in two weeks can dig 500 4 2 2
Hence, it takes them = days which is
m of tunnel. 4+3+3 5 5
of what Ajay takes working alone.
So, for digging 20,000 m of tunnel in two weeks, the
number of workers required is given by:
129. The minimum time is taken by C as he is not present
20,000
n = = 40 workers 2
500 in the maximum time of 22 days.
9
The number of existing workers = 12
Similarly, the maximum time is taken by B as he is
So, the number of extra workers required
= 40 – 12 = 28 5
not present in the minimum time of 15 days.
13
The correct answer is E. The correct answer is A.

233

Book 1.indb 233 30/04/2019 4:48:02 PM


NMAT by GMAC™ Official Guide 2019

130. Fraction of work done by both in 1 day Ratio of share of earning = Ratio of their relative
1 1 1 efficiency
= + =
10 15 6 Ratio of share of Nishit, Pradeep and Bhuvan
= 2x : x : x = 2 : 1 : 1
Required number of days = 6 days
The correct answer is A.
The correct answer is C.
134. Since the overall time taken is 22 minutes, which is
15525 115 more than the time taken by individual taps, the
31. Daily wage of an unskilled mason =
1 =
45 × 48 16 third tap will empty the tank.

115 115 Let the third tap empties the tank in x minutes.
Daily wage of a skilled mason = 2 × = Then,
16 8
1 1 1 1
Wages of a skilled mason in 16 days + - =
10 15 x 22
115 1 1 1 1
= × 16 = 230 = + -
8 x 10 15 22
Number of skilled masons who earn Rs. 5,750 in 16 1 33 + 22 - 15
⇒ =
5750 x 330
days = = 25 1 40
230 ⇒ =
x 330
The correct answer is C. 330 33
⇒x= = = 8.25 minutes
40 4
132. Let Girish’s efficiency be x work/day.
The correct answer is B.
Then, Harish’s efficiency = (x × 2) work/day
Total efficiency = (x + 2x) work/day = 3x work/day 135. A group of 20 software engineers can finish a
Total work development project in 30 days. So,
= Efficiency × Number of days Total work = 30 × 20 = 600 man days
= (3x × 18) = 54x Work done by 15 Software engineers in 35 days
Required number of days = 15 × 35 = 525 man days
Total work So, remaining work = 600 – 525 = 75
=
Harish’s Efficiency This remaining work is done by 5 engineers during
54x initial days.
= = 27 days
2x So, required number of days
Alternatively, 75
= = 15 days
If Harish takes x days to complete the work, then 5
Girish would take 2x days. So, The correct answer is E.
1 1 1
+ =
x 2x 18 136. At 5 o’clock, the hour hand was at 5. At 5:25, the
3 1 minute hand is at 5 and hour hand has moved some
⇒ = ⇒ x = 27 days degrees. Speed of hour hand is 0.5°/min. So, in 25
2x 18
minutes, it would have moved 0.5 × 25 = 12.5°.
The correct answer is D.
So, the angle between the two hands is 12.5°.
133. Let Pradeep’s efficiency be x work/day. The correct answer is B.
Then, Nishit’s efficiency = 2x work/day 15
Bhuvan’s efficiency 137. From 7 AM to 4:15 PM, total hours are 9 hours
60
x + 2x 37
= = x work/day = hours
3 4

234

Book 1.indb 234 30/04/2019 4:48:04 PM


3.0  Quantitative Skills Practice

Since the clock is gaining 5 seconds in every 3 1 + 2 = 3 odd days


5 So, August 15, 1949 would be a Friday + 3 days,
minutes, therefore when the clock shows 3 hours
60 that is, Monday.
37
= hours, true time is 3 hours. Number of days from August 15, 1949 to January
12
37 26, 1950 = 16 + 30 + 31 + 30 + 31 + 26 = 164
So, if the clock is showing hours, true time is
4
= 3 × 3 = 9 hours Therefore, number of odd days = 3
Thus the true clock will show 4 PM. Hence, January 26, 1950 day would be Monday + 3
The correct answer is C days, that is, Thursday.
The correct answer is A.
138. Hands will coincide once between 1 and 2 o’clock;
once between 2 and 3 o’clock, and so on. Between 142. In such questions, always start from today. If today
11 and 1 o’clock, the two hands coincide at 12 is 8 March, 2018, and it is Thursday, we can work
o’clock. So, overall the two hands coincide 11 times backwards to April 20, 1984.
in 12 hours and so 22 times in 24 hours.
From 8 March, 1984 to 8 March, 2018, there are 34
The correct answer is C years, of which there are 8 leap years and 26 normal
years.
139. Hands will be at right angle twice between 1 and 2 So, there would be 8 × 2 + 26 × 1 = 42 odd days,
o’clock, twice between 2 and 3 o’clock, and so on. which means 0 odd days.
The only exceptions are between 2 to 4 o’clock and So, 8 March, 1984 was also Thursday.
between 8 to 10 o’clock, when the two hands are at
right angles only once each. Thus the two hands will Now, from 8 March, 1984 to 20 April, 1984, number
be at right angle 22 times in 12 hours or 44 times in of days = 23 + 20 = 43, which means 1 odd day.
24 hours. So, April 20, 1984 would be Friday.

The correct answer is B.


The correct answer is C
143. Since there are exactly 4 Wednesdays and 4
140. To coincide with each other, the angle between them
Sundays in the month of January having 31 days,
should be 0°.
the 1st of January that year has to be on Thursday
Time of meeting because only then the month will end on a Saturday,
with the month having 5 Thursdays, 5 Fridays and 5
Init ial gap between hands at 2o'clock
= Saturdays.
Relative speed of hands of clock
The correct answer is E.
60 60 120 10
= = = = 10 minutes
6 − 0.5 5.5 11 11 144. If her age is x in the year 1980, then she was born in
Hence the two hands of the clock will coincide at the year 1980 – x.
2:10 10/11 O’clock. As per the question,
The correct answer is C. x = 2
1980 − x or x + x – 1980 = 0
Alternatively,
−1 + 1 + 4 × 1980
The two hands will coincide when both the hands are Or x = = 44
2
between 2 and 3 only. The only option is C.
So, she was born in 1936.
141. In an ordinary year, there are 52 weeks and one
The correct answer is B.
extra day and in a leap year there are 52 weeks and
2 extra days. Alternatively,
From August 15, 1947 to August 14, 1949, there is The year of birth has to be a perfect square. The
one ordinary year and one leap year, that is, only option is 1936.

235

Book 1.indb 235 30/04/2019 4:48:04 PM


NMAT by GMAC™ Official Guide 2019

145. Total difference of years = 30 years


Leap years in these 30 years = 8
Normal years in these 30 years = 22
So, total number of odd days = 8 × 2 + 22 = 38

38
So, final odd number of days = = 3 days
7

On 09.12.2001 is Sunday, so on 09.12.1971 there


would have been Thursday.
The correct answer is E.

236

Book 1.indb 236 30/04/2019 4:48:04 PM


3.0  Quantitative Skills Practice

2. Algebra a8 – b8
=
a4 + b4 ( ) (a
4
) a
- b4
=
4
- b4

1. The simplest way to approach this question is to


( )(
a4 + b4 a2 + b2 a4 + b4 ) ( ) (a
2
+b ) a
2 2
+ b2

divide the marbles evenly among all 13 people and Now, factor a4 – b4 according to the same pattern:
then adjust this number so that Manish ends up with
the most marbles. a 4 – b4
=
( )(
a 2 + b2 a 2 - b2
= a 2 – b2
)
60
= 4.61
a 2 + b2 (
a 2 + b2 )
13
Let Manish have 5 marbles. Then each of his friends = (a + b) (a – b)
can have at the most 4 marbles, giving a total of The correct answer is C.
(12 × 4) + 5 = 53 marbles. We still have 7 marbles
left, so Manish may not necessarily end up with the 5. We know that (a – b)2 = a2 – 2ab + b2 and
most number of marbles.
Then, let Manish have 6 marbles. Now each of his (a + b)2 = a2 + 2ab + b2
friends can have at the most 5 marbles, giving a Then,
total of (12 × 5) + 6 = 66 marbles. But we only have (a – 2)2 + (a – 1)2 + a2 + (a + 1)2 + (a + 2)2 = (a2
a total of 60 marbles, so if Manish keeps 6, there is – 4a + 4) + (a2 – 2a + 1) + (a2) + (a2 + 2a + 1) + (a2
no way any of his friends can end up with a greater + 4a + 4)
number of marbles. Thus, the answer is 6.
= (a2 + 4) + (a2 + 1) + (a2) + (a2 + 1) + (a2 + 4)
The correct answer is B. = 5a2 + 10

2. a2 – b2 = 0 can be rewritten as a2 = b2. The correct answer is B.

From here, it might look as though a = b, but this is 6. The phrase “inversely proportional” means that
not necessarily the case. For example, a could be a = k/b or ab = k, where k is a constant.
3 and b could be -3. Algebraically, when you square
root both sides of a2= b2, you do NOT get a = b, but As the product of a and b is always constant, we
rather |a| = |b|. Thus, statement 1 is not necessarily have:
true and statement 2 is true. 24 × 2 = 6 × b,
Statement 3 is also true: or b = 8.
2 2
a –b =0 The correct answer is E.
or, a2 = b2
7. Let the number of pieces of cake eaten by each
a2
or, =1 of the two people who eat the same quantity be P
b2 and the number of pieces of cake eaten by the third
The correct answer is E. person be T.
According to the question:
3. The algebraic identity x 2 - y 2 = ( x + y ) ( x - y ) P + P + (P + 2) = 14
⇒ 3P + 2 = 14
Then, x 2 - y 2 = 12 × 20
⇒ 3P = 12
= 240 ⇒P=4
= 16 × 15 You can use the value of P to solve for T:
= 4 15 Therefore, the number of pieces eaten by the third
person = 4 + 2 = 6
The correct answer is B.
The correct answer is D.

4. We know that x2 – y2 = (x + y) (x – y) 8. Suppose the value of the red beads, the blue beads
a8 can be written as (a4)2 and the yellow beads are r, b and y respectively.
a4 can be written as (a2)2 From the question,
The given expression can now be solved as follows: r + b = 4.25

237

Book 1.indb 237 30/04/2019 4:48:05 PM


NMAT by GMAC™ Official Guide 2019

b + y = 2.75 Also, the total contribution


r + b + y = 4.5 = (x – 3)(y + 10) = xy + 10x – 3y – 30
You need to find out ‘r+y’ Since the contribution is the same, therefore:
If r + b = 4.25, then r + b + y = 4.5 could be xy = xy + 10x – 3y – 30
rewritten as:
⇒ 10x – 3y = 30
4.25 + y = 4.5
⇒ 3y = 10x – 30
y = 0.25
If x = 3, y = 0; this is not possible as the total
Since b + y = 2.75 and y = 0.25: contribution will be 0.
b + 0.25 = 2.75 (Note that when x = 3 or 4, y will not be an integer
b = 2.5 value).
Since r + b = 4.25 and b = 2.5: If x = 6, y = 10; this condition is possible, so the
minimum total contribution possible is Rs. 60.
r + 2.5 = 4.25
If 10 people had joined for the party, contribution per
r = 1.75
60
Therefore, r + y = 1.75 + 0.25 = 2. person = = Rs. 6
10
The correct answer is B. The correct answer is C.

9. Let us consider, 12. Assume some values of p, q and r such that p + q


E = price under the 'Exclusive' plan + r = 0 and find the value of the expression that is
given, so suppose p = 1, q = – 1 and r = 0.
R = price under the 'Regular' plan
x = the number of classes Karan takes We find that,
Then, p2 q2 r2 1 1
2 + 2 + 2 = + +0 =1
E = 495 + 15x 2p + qr 2q + pr 2r + pq 2 2
R = 80x
The correct answer is B.
0.6R = E
Now, solving the above equations by substitution: 13. Let the number of swans be x.
0.6R = 495 + 15x
1
0.6(80x) = 495 + 15x So, we get: 7 × × x +2 = x
2
48x = 495 + 15x
Now using options, we realise that x = 16 satisfies
33x = 495 the equation.
x = 15 So, number of swans is 16.
The correct answer is C. The correct answer is B.

10. Let the number of correct answers be x; so the 2


1 1  1 1 1
number of incorrect answers will be 25 – x. 14. − 2 =  +  − 1× = m2 – 2n
 p q pq
2
p q
According to the problem,
3x – 0.5(25 – x) = 40 The correct answer is C.

⇒ 3x – 12.5 + 0.5x = 40 15. Given f(x + 2) + f(5x + 6) = 2x – 1


⇒ x = 15
When (x + 2) = 1, x = – 1, at which the value of (5x
The number of incorrect answers = 25 – 15 = 10 + 6) is also 1.
The required difference = 15 – 10 = 5 So, putting x = –1 in the given equation, we get:
The correct answer is A. f(1) + f(1) = – 3

11. Let the number of persons be x and contribution per −3


f(1) =
person be Rs. y. 2

Therefore, the total contribution = Rs. xy The correct answer is D.

238

Book 1.indb 238 30/04/2019 4:48:05 PM


3.0  Quantitative Skills Practice

16. For no solution of the equations Hence, roots are real and distinct.
4x – Ky = –7 and 5x + 3y = 2 The correct answer is D.

4 − K −7 22. The roots are – 4 and – 3.


= ≠
5 3 2
Then, (p + q)2 = ( – 7)2 = 49;
−12
K = (p – q)2 = ( – 4 + 3)2 = 1
5
The correct answer is C. So, required equation is
x2 – (49 + 1)x + (49)(1) = 0
17. – 4 > – 5, therefore, [( – 4)@( – 5)] = ( – 4) Or, x2 – 50x + 49 = 0
Then, ( – 4)%2 = – 4 (Since A × B is negative) The correct answer is A.
The correct answer is D.
23. Let p and q be the roots of the quadratic equation.
1@ − 1 Given that, p = 1.5q and p – q = 1 or q – p = 1
18.
( −K ) @ ( −K ) % K, K >0 From these equations, we get:

1 p = 3 and q = 2 (or) p = – 3 and q = – 2


= %K Since the quadratic equation with roots p and q is
-K
given by (x – p) (x – q), it follows that equations can
1 be x2 – 5x + 6 = 0 (or) x2 + 5x + 6 = 0
=
-K
The correct answer is C.
The correct answer is C.
24. x2 – 6x + 9 = 0
19. 3x3 – 7 = 185 (x – 3)2 = 0
or, 3x 3 = 192 ⇒x=3
So, x3 = 27
192
or, x = 3
= 64
3 The correct answer is E.
Thus x = 3 64 = 4
25. Equation given is
Thus, x2 – x = 16 – 4 = 12
ax2 + bx + c = 0
The correct answer is C.
-b ± b2 - 4ac
Roots of equation =
20. Let the roots of equation ax2 + bx + c = 0 be m 2a
and n. -b + b2 - 4ac
p=
Then roots of equation, px2 + qx + r 2a
= 0 are 1/m and 1/n -b - b2 - 4ac
q=
Now, m + n = -b/a and mn = c/a 2a
1/m + 1/n = -q/p and 1/mn = r/p p4 – q4 = (p2 – q2) (p2 + q2)
Also, 1/m + 1/n = (m + n)/mn.  2
 2
 -b + b2 - 4ac   -b - b2 - 4ac  
So, -b/c = -q/p and also c/a = p/r   -  
 2a   2a  
By these relations we get a = r, c = p and b = q
 2
 -b - b2 - 4ac  
2
The correct answer is E.  -b + b2 - 4ac 
  +   
 2a   2a  
21. Discriminant, D = (4b)2 – 4a(2c) 

( 
) ( ) 
2
Since a, b and c are in GP, b2 = ac 4 -b × b2 - 4ac  ( -b)2 + b2 - 4ac
Hence, discriminant, D = 16b2 – 8b2 = 8b2 >0 = 2 ×
4a2  4a2 
Note that 8b2 cannot be zero as a, b and c are  
positive integers.

239

Book 1.indb 239 30/04/2019 4:48:06 PM


NMAT by GMAC™ Official Guide 2019

-b × b2 - 4ac  2b2 - 4ac  29. 192 < x 12,


=  2a2 
a2
192
-b or, <x
= 4 (b2 - 2ac) b2 - 4ac 12
a
or, 16 < x
If values of p and q are reversed, then the value of or, 4 < x
b 2 Similarly,
p4 – q4 will be (b - 2ac) b2 - 4ac .
a4 x
< 12
Therefore, the value of 12
b 2 or, x < 12x 12
p4 – q4 = ± (b - 2ac) b2 - 4ac
a4 or, x < 12
The correct answer is E. The correct answer is D.

26. Standard form of quadratic equation is


ax 2 + bx + c = 0. a   x 
30. If     < 0 , then the two fractions must have
We know for the above quadratic equation, b   y 
b a   x 
opposite signs, that is,   must < be
0 the negative
Sum of the roots = -
a  b   y 
c x a  y
And, product of the roots = inverse of or =–  .
a y b  x
Now, given quadratic equation is Thus, D is the answer. The other choices may or may
(a + 1)x2 + (2a + 3)x + (3a + 4) = 0 not be true.
So, we have The correct answer is D.
2a + 3
- = -1
a +1 31. Let b equal the number of chairs sold. Each chair sells
for Rs. 700, so the total revenue is Rs. 700b. The
⇒ -2a - 3 = -a - 1
cost is equal to 11,000 plus 300 for every chair sold.
⇒ -a = 2 ⇒ a = -2
(700b) – (11,000 + 300b) > 0
-6 + 4
Product of roots = =2 700b – 11,000 – 300b > 0
-2 + 1
400b – 11,000 > 0
The correct answer is C. 400b > 11,000

27. 2(x – 1)3 + 3 ≤19 b > 27.5


If b must be greater than 27.5, then the
2(x – 1)3 ≤ 16 manufacturing unit needs to sell at least 28 chairs to
(x – 1)3 ≤ 8 make a profit.
x–1≤2 The correct answer is C.
x≤3
The correct answer is B. 32. Let x2401 be a.
We have to minimise (a + 1/a + 2 × a × 1/a)1/2
28.
(3a + 7) ³ 2a + 12 or – ( 3a + 7 ) ³ 2a + 12 = (a + 1/a + 2)1/2
a + 7 ³ 12 or –3a – 7 ³ 2a + 12 For a and 1/a we have A.M. (arithmetic mean)
a ³5 or –7 ³ 5a + 12 = (a + 1/a)/2
19 G.M. (geometric mean) = (a × 1/a)1/2 = 1
or – ³a
5 As, we know A.M. ≥ G.M.
The correct answer is D. So, (a + 1/a)/2 ≥ (a × 1/a)1/2

240

Book 1.indb 240 30/04/2019 4:48:08 PM


3.0  Quantitative Skills Practice

So, (a + 1/a) ≥ 2 36. Let ap = bq = cr = ds = k


Its minimum possible value is 2 (for A.M. = G.M.) Or, a = k1/p, b = k1/q, c = k1/r and d = k1/s
So, minimum possible value of (a + 1/a + 2 × a ×
loga (bcd) = logk1/p (k1/q × k1/r × k1/s)
1/a)1/2 = (2 + 2)1/2 = 2
The correct answer is A. = logk1/p (k1/q + 1/r + 1/s)

33. log102 = 0.3010 and also log1010 =1 æ 1 1 1ö


ç q + r + s÷
è ø = p æ 1 + 1 + 1ö
So, log10(2 × 5) = 1 = ç q r s÷
1 è ø
log102 + log105 = 1 and so log105 = 0.6990 p
Now, log5256 = log10256/log105 = log1028/log105
The correct answer is A.
= 8log102/log105 = 8 × 0.3010/0.6990 = 3.44
The correct answer is C. 1
37. logy x = 10 ⇒ logx y =
10
34. Given 625log36 6 + 12log7 49 = 11logx 169
6 2 1
1 So, logx3 y 6 = log y = =
⇒ 6252 log6 6 + 122 log7 7 = 11logx 169 3 x 10 5


1
⇒ 6252 + 122 = 11logx 169 The correct answer is B.
2 5
⇒ 25 + 144 = 11logx 169 38. Given P =
625log3 25 + 25log125 27 + 5log8 125
5
⇒ 169 = 11logx 169 3 3
log 3 log2 5
⇒ x = 11. = 6252 log25 3 + 25 3 5
+5 3

The correct answer is B. 2


log5 3
= 6252 + 52 log5 3 + 55 log5 2
35. log30 3 + log30 5 + log30 2 = 1 = 54 log5 3 + 5log5 3 + 5log5 2
2 5

[log a + log b + log c = log abc]


= 34 + 32 + 25 = 122
⇒ x + y + log30 2 = 1
⇒ log30 2 = 1 – x – y The correct answer is C.
⇒ 3 log30 2 = 3(1 – x – y)
⇒ log30 8 = 3(1 – x – y)

1
⇒ log8 30 =
3(1 - x - y)

The correct answer is B.

241

Book 1.indb 241 30/04/2019 4:48:08 PM


NMAT by GMAC™ Official Guide 2019

3. Geometry This is Pythagoras Theorem.


So, we get infinite number of triplets.
1. If there are n spokes, there will be n angles between The correct answer is E.
them. Thus, the measure of the angle between
360 5. None of the conditions holds in any case.
spokes is
n
The correct answer is E.
Since n < 6, we can rewrite this expression as
360 6. We know that, the sum of the angles of a triange is
, which means the answer has to be 180o. That is,
(less than 6)
A + B + C = 180o
greater than 60°.
Given that A = 4B
The only answer choice that matches this
requirement is E. ⇒ 5B + C = 180o

The correct answer is E. 180° - C C


⇒ B = = 36o –
5 5
2. The interior figure shown is a pentagon, although an So, B has to be less than 36o.
irregular one. The sum of the interior angles of any
The correct answer is B.
polygon can be determined using the formula (n – 2)
(180), where n is the number of sides:
7. If two chords bisect each other, then chords are
(5 – 2)(180) = (3)(180) = 540 necessarily the diameters.
Using the rule that angles forming a straight line As given ST = BC = 8 = diameter
sum up to 180, the interior angles of the pentagon
(starting at the top and going clockwise) are 180 – c, Also, DABC is an isosceles right triangle because BC
180 – d, 140, 180 – a, and 180 – b. The sum of is diameter and angle in a semi-circle is a right
these angles can be set equal to 540. angle.

540 = (180 – c) + (180 – d) + 140 + (180 – a) + Therefore, AB = AC = 4 2


(180 – b) 1
So, area = × 4 2 × 4 2 = 16 sq. units
540 = 140 + 4(180) – a – b – c – d 2
The correct answer is D.
540 – 140 – 720 = – (a + b + c + d)
– 320 = – (a + b + c + d) 8. Hypotenuse of the largest right-angled triangle is
So, a + b + c + d = 320°. equal to the diameter of the circle. Also, this triangle
will be an isosceles triangle with sides forming the
The correct answer is B.
right angle.
3. Diameter of circle 1 = Diagonal of square 1 Hence, area of this triangle
= 100 cm 1
100 50 = ´ 20 ´ 10 = 100 sq.cm
Side of square 1 = = cm = Diameter of 2
circle 2 2 2 Note that in case of isosceles triangle, area is
Similarly, the diameter of circle 5 = 25 cm maximum.]

The correct answer is A. The correct answer is B.

4. By sine rule, we get: 9. O is the incentre of the triangle. Note that angle
bisectors of a triangle meet at incentre, that is, a line
a b c joining the incentre and vertex of a triangle bisects
= = =K
sin A sinB sinC the angle at the vertex.
a b c ∠ABC = ∠ACB = 40°
⇒ = sin A, = sin B, = sin C
K K K ∠CAB = 100°
Putting these values in the given expression, we get: So, ∠OAB = 50°
a2 + b2 = c2 The correct answer is B.

242

Book 1.indb 242 30/04/2019 4:48:09 PM


3.0  Quantitative Skills Practice

10. ∆ CEA is a right angled triangle, Now in triangle ODB, OD = DB and similarly OD = AD.
⇒ CE2 = CA2 + AE2 So, we can say ÐB = ÐDOB = 45°
⇒ 652 = 602 + AE2 Similarly ÐA = ÐDOC = 45°
2
⇒ AE = 625 ⇒ AE = 25 feet So, ÐAOB = 90°
⇒ BE = AB – AE = 64 – 25 = 39 feet So, area of sector AOB = 1/4 π (10)2 = 25π
∆ DBE is a right angled triangle, Area of segment ABC = Area of sector AOB - area of
2 2 2 triangle OAB
⇒ ED = BD + BE
= 25π - 1/2 × 10 × 10
⇒ 652 = 392 + AE2
= 25π - 50
⇒ AE = 52 feet
Area of segment ABC lying outside smaller circle
The correct answer is E.
= area of segment ABC - area of smaller circle
11. Side of the hexagon = 12/6 = 2 = a Now for smaller circle diameter is DC and we have
OC = OD + DC
Area of a hexagon = (3√3/2)a2
DC = 10 - 5√2
= (3√3/2) 22
So, radius of smaller circle is (10 - 5√2)/2
= 6√3 in2
= (5 - 5/√2)
The correct answer is D. So, the required area = (25π - 50) - π(5 - 5/√2)2
= 25π - 50 - π (25 + 25/2 - 25√2)
12. Sum of interior angles of a polygon = 180 (n - 2)
= (25√2 - 12.5) π - 50
= 180 (9 - 2) = 22.5π - 50
= 180 × 7
The correct answer is B.
= 1260°
The correct answer is D. 15.

13. Let the length and the breadth of the rectangle be ‘l’
and ‘b’ respectively.
According to the question, 2 cm
2(l + b) + 10 = 2(l + 2b)
⇒ 2b = 10
⇒ b = 5 units
O
Again, from the question, we have:

  1  A B
2    + b  + 10 = 2(l + b)
 2  Let the radius of the quadrant be R and the radius of
⇒ l = 10 units the circle is r.
Hence, the perimeter of the rectangle
2R = 2 2 or R = 2
= 2(10 + 5) = 30 units
Now, OB = OA + AB2
2 2
The correct answer is B.
2 2 2
⇒ (r + 2 ) = r + (2 – r)
14. Let OC meet AB at D. As we have right triangle ODB, ⇒ r2 + 2 + 2 2 r = r2 + 4 + r2 – 4r
2 2 2
OD + DB = OB . ⇒ r2 – (4 + 2 2 )r + 2 = 0
OB = 10,
(4 + 2 2 )
2
DB = 10√2/2 = 5√2. 4+2 2 ± -8
So, OD = 5√2
Radius =
2

243

Book 1.indb 243 30/04/2019 4:48:10 PM


NMAT by GMAC™ Official Guide 2019

The given line and the line joining the given point and
4 + 2 2 – 16 + 8 + 16 2 – 8 its image are perpendicular, so the product of their
=
2 slopes = –1

4 + 2 2 – 4 1+ 2 12 - b 12
= ⇒ × = -1
2 3-a 5
= 2 + 2 − 2 1+ 2 ⇒ 144 – 12b = 5a – 15
⇒ 5a + 12b = 159
The correct answer is D.
5
⇒5× b + 12b = 159
16. 12
P ⇒ 169b = 159 × 12
80° Q
R 159 × 12 1908
⇒b= =
169 169
S V
O 5 5 1908 795
a= b = × =
12 12 169 169
T
U
The correct answer is E.

Draw a line from O to Q and O to S. 18. We know that a centroid divides a triangle into 6
∠PQO = 90° (Radius of circle is perpendicular to the smaller triangles of equal area. Quadrilateral AMGN
tangent) contains two such small triangles. Therefore,
∠PQR + ∠RQO = ∠PQO 2 1
Area of the quadrilateral AMGN = cm2 = cm2
⇒ 80° + ∠RQO = 90° 6 3
⇒ ∠RQO = 10° The correct answer is D.
OQ = OS (radius of bigger circle)
In ΔOSQ 19. Given that the side of the first equilateral triangle = p
∠RQO = ∠RSO = 10° (opposite angle to opposite 3 2
side) So, area = p
4
∠RST = 2 ∠RSO
Area of the triangle formed by joining the midpoints
∠RST = (2 × 10°) = 20° of the first triangle will be 1/4th of the area of the first
The correct answer is A. 1  3 2
triangle, that is,  p .
4 4 
17. Let the image of of (3, 12) with respect to the line
12x – 5y + 12 = 0 be at (a, b). This process will go on.
So, midpoint of (3, 12) and (a, b) will lie on the line So, sum of the areas of all such triangles
12x – 5y + 12 = 0.
 3 + a 12 + b  3 2 1  3 2 1  3 2
Midpoint of (3, 12) and (a, b) =  , = p +  p +  p  + ...
 2 2  4 4  4  16  4 
Since this point is on the line 12x – 5y + 12 = 0, so
3 2
p
 3 + a  12 + b  4 = 36 3
12  - 5 + 12 = 0
 2   2  1 -
1
4
⇒ 36 + 12a – 60 – 5b + 24 = 0
3 2
⇒ 12a – 5b = 0 ⇒ p = 36 3
3
⇒ 12a = 5b
⇒p=6 3
5
⇒a= b
12 The correct answer is C.

244

Book 1.indb 244 30/04/2019 4:48:12 PM


3.0  Quantitative Skills Practice

20. Total number of inches in the fence of the circle is 4 units, so the circumference of the
= 600 × 10 = 6000
whole circle is 2 π (4) = 8 π units. Since the sector is
Thus, in modern times 6000 inches 1
of the circle, the arc length is
6000 4
= foot = 500 feet
12 1
( )(8 π ) = 2 π units.
The correct answer is C. 4

21. The area of the picture is 12 × 16 = 192 square The correct answer is B.
inches. The area of the frame is 18 × 30 = 540
square inches. 24. According to the formula for the volume of a right
The required area can be calculated by subtracting circular cylinder, the original volume is V = πr2h. As
the area of the picture from the area of the frame. per the question:
Therefore, the required area = 540 – 192 = 348 r 2πr 2h πr 2h
square inches. Y = π   (2h ) = =

 2 22 2
The correct answer is D.
Thus, the volume, which was once πr2h, is now
22. The surface area of the original cube is 6 × (24)2 æ p r 2h ö
ç ÷ . This is a 50% decrease.
= 3,456 square inches. è 2 ø
(As 1 foot = 12 inches) The correct answer is A.
To cut the large cube into 2 inch × 2 inch × 4 inch
cuboids, two dimensions (say length and width) 25.
will be sliced every 2 inches, while one dimension A
(say height) will be sliced every 4 inches. Thus,
 24   24   24  E
 2  ×  2  ×  4  = 864 cuboids can be cut from B

the large cube.


F
The equation for the surface area of a cuboid is:
C
2(lb + bh + hl) = In this case, that is
2(2 × 2) + 2(2 × 4) + 2(2 × 4) = 8 + 16 + 16 = 40
square inches per cuboid.
D
Since there are 864 cuboids, the total surface area
Let AB = h; ∆ABE ∼ ∆ACF;
is:
Therefore,
40 × 864 = 34,560 square inches.
1
Finally, the ratio of the total surface area of all the AB AC h h + 1 + 2 4
cuboids to the surface area of the original cube is = ; =
BE CF 1 1
2
4
34, 560
= = 10 : 1 5 13
3, 456 h=
4 4

The correct answer is E. 13
⇒h=
5
23. The radius of the circle is 4 units, so the area of the
13 9 9
circle is π (4)2 = 16 π . The area of the sector is Total height = +1+ + = 8.1 inches
5 4 4
 4π  1 The correct answer is D.
4 π units, or  = of the circle. The radius
 16π  4

245

Book 1.indb 245 30/04/2019 4:48:13 PM


NMAT by GMAC™ Official Guide 2019

26. The diagonal of the biggest cube that can completely Hence, the desired ratio will be
fit inside the sphere must be equal to the diameter of 2πr2 : 2πr2 : 2 πr
2
that sphere.
⇒ 2:2: 2
Let us consider if ‘s’ is the length of the sides of the
cube and ‘r’ is the length of the radius of the sphere, ⇒ 2 : 2 :1
then we have:
The correct answer is D.
3 s = 2r = 20 cm
Therefore, total surface area of the cube 31. Total surface area = 2(lb + bh + hl)
2400 = 2(10 × 15 + 15 × 20 + 20 × 10)
= 6s2 = = 800 sq. cm
3 = 2(150 + 300 + 200) = 1300 cm2
The correct answer is E. The correct answer is E.
4
27. Volume of sphere = πr3 32. Area of parallelogram = Base × Height
3
Case 1:
1 2 2 3
Volume of cone = πr (2r) = πr Base = 15 cm and height = 5 cm
3 3
Area = (15 × 5) cm2 = 75 cm2
Ratio of the volumes = 2 : 1
Case 2:
The correct answer is A. Base = 15 cm and height = 10 cm
4 Area = (15 × 10) cm2 = 150 cm2
28. Volume of the solid sphere = π (4)3 cm3
3 Case 3:
Base = 15 cm and height = 15 cm
4
Volume of one small ball = π (0.05)3 Area = (15 × 15) cm2 = 225 cm2
3
3 and so, on
So, number of balls =  4  = 512000 We notice that 75 cm2, 150 cm2, 225 cm2 ….. are in
 0.05 
arithmetic progression. So,
The correct answer is E. a = 75 cm2
d = (150 – 75) = 75 cm2
29. The largest distance from any point to any other Sum of arithmetic progression
point on the cube is the distance between the
n
diagonally opposite vertices. Sn = 2a + (n - 1)d
2
The distance = 102 + 52 + 22 = 129 Let the number of terms be n. Then,

Hence, the side of the cube must be equal to n


4125 = [2 × 75 + (n - 175
) ]
129 cm. 2
Hence, volume = (129)3/2 cm3 n
⇒ 4125 = × 75[2 + n - 1]
The correct answer is C. 2
4125 × 2
⇒ = n + n2
30. Assume the radius is ‘r’ and height is ‘h’. 75
Thus, area of the curved surfaces of ⇒ n2 + n – 110 = 0
(A) Cylinder – 2πrh = 2πr2 (As h = r) ⇒ n2 + 11n – 10n – 110 = 0
(B) Hemisphere = 1/2 (4πr2) = 2πr2 ⇒ n(n + 11) – 10(n + 11) = 0
⇒ (n – 10) (n + 11) = 0
(C) Cone = πrl = πr r2 + h2 = πr r2 + r2 = 2πr2

246

Book 1.indb 246 30/04/2019 4:48:14 PM


3.0  Quantitative Skills Practice

As n cannot be negative so, n = 10.


So, number of parallelograms = 10
The correct answer is C.

πr2
33. Area of semicircle =
2
Area of the second circle = πR2
According to the question,
πr2
= 4πR2
2
R2 1
⇒ =
r2 8
⇒R:r=1:2 2
The correct answer is E.

247

Book 1.indb 247 30/04/2019 4:48:15 PM


NMAT by GMAC™ Official Guide 2019

4. Modern Math ways in which the five friends can be lined up with
Dravid and Binod standing together.
Therefore, there are 120 – 48 = 72 arrangements
1. To figure out the ‘limiting factor’, take the number
where Binod will be separated from Dravid.
of players available for each position and figure out
how many sets could be formed in each case, if The correct answer is C.
there were more than enough players in all the other
positions. 4. Any number is divisible by 8 if its last 3 digits are
divisible by 8.
Batsmen: 23 players available ÷ 2 players needed
per set = 11.5 sets = 11 complete sets. So, we have to make total possible 5 digit numbers
Bowlers: 21 players available ÷ 3 bowlers needed per so that last 3 digits are divisible by 8 using the digits
set = 7 complete sets. 1, 2, 3, 4, 5 and 6 and without repetition.
Wicketkeepers: 9 players available ÷ 1 wicketkeeper So, we must first fix the last 3 digits and only then
needed per set = 9 complete sets. can the other digits be fixed.
Thus, only 7 complete sets can be formed, using Case 1 3rd digit is 1
all of the available bowlers and some of the other (a) If 4th digit is 2 then we have only 120 and 128
players. A total of 7 × 2 = 14 batsmen are required, divisible by 8 but, these can’t be used.
leaving 23 – 14 = 9 unused batsmen. Likewise, 7 ×
1 = 7 wicketkeepers are required, leaving 9 – 7 = 2 (b) If 4th digit is 3 then there is only 1 possibility of
unused wicketkeepers. In all, there are 9 + 2 = 11 136—these are such 6 possible numbers (as we
unused players, who will not be on any team. have 3 × 2 × 1 × 1 × 1 such possible numbers).
(c) If 4th digit is 4—we can’t find the number which is
The correct answer is C. divisible by 8.
(d) If 4th digit is 5 then 152 is only possible number
2. For the first digit, there are only five options (5, 6, 7,
divisible by 8—we again have such 6 numbers
8, and 9) because a five-digit number must start with
(again 3 × 2 × 1 × 1 × 1).
a non-zero integer. For the second digit, there are 5
choices again, because now zero can be used but (e) If 4th digit is 6—we can’t have such a number
one of the other numbers has already been used, and which is divisible by 8.
numbers cannot be repeated. For the third number, Case 2 3rd digit is 2
there are 4 choices, for the fourth there are 3 choices,
and for the fifth number there are 2 choices. Thus, the (a) If 4th digit is 1 then we have only 216 and so
total number of choices is (5)(5)(4)(3)(2) = 600. such 6 numbers are possible.
(b) If 4th digit is 3 then there is no such number.
The correct answer is D. (c) If 4th digit is 4—we can’t find the number which is
divisible by 8.
3. The number of ways in which the friends can be
(d) If 4th digit is 5 then 256 is the only possible
arranged with Binod and Dravid separated is equal
number divisible by 8—we again have such 6
to the total number of ways in which the friends can
numbers (again 3 × 2 × 1 × 1 × 1).
be arranged minus the number of ways they can be
arranged with Binod and Dravid together. (e) If 4th digit is 6—we have 264 divisible by 8 and
again such 6 numbers are possible.
The total number of ways to arrange 5 people in a line
is 5! = 120. Case 3 3rd digit is 3
To compute the number of ways to arrange the 5 (a) If the 4th digit is 1 then we have only 312 and so
friends such that Binod and Dravid are standing such 6 numbers are possible.
together, group Binod and Dravid as one person, (b) If the 4th digit is 2 then there is no such number.
since they must be lined up together. Then the (c) If the 4th digit is 4—we can’t find the number
problem becomes one of lining up 4 students, which which is divisible by 8.
gives 4! possibilities.
(d) If 4th digit is 5 then 352 is only possible number
However, remember that there are actually two divisible by 8—we again have such 6 numbers
options for Binod and Dravid coming together: Binod (again 3 × 2 × 1 × 1 × 1).
first and then Dravid or Dravid first and then Binod.
Therefore, there are (4!)(2) = (4)(3)(2)(1)(2) = 48 total (e) If the 4th digit is 6—no such number is possible.

248

Book 1.indb 248 30/04/2019 4:48:15 PM


3.0  Quantitative Skills Practice

Case 4 3rd digit is 4 624 and 632.


th
(a) If the 4 digit is 1 then we have only 416 and so The leftmost two digits in each of the above cases
such 6 numbers are possible. can be filled in 3 × 2 = 6 ways
(b) If the 4th digit is 2 then no such number is So, total such numbers are 6 × 14 = 84
possible.
5. First distribute 2 balls in each of the boxes. So, we
(c) If the 4th digit is 3 then we have 432 and such 6 are left with 4 identical balls to be distributed in 3
numbers are possible. boxes.
(d) If the 4th digit is 5 then 456 is the only possible
We know that the number of ways of distributing 'n'
number divisible by 8—we again have such 6
identical balls among 'k' different boxes = n+k–1Ck–1
numbers (again 3 × 2 × 1 × 1 × 1).
6!
(e) If the 4th digit is 6—no such number is possible. The number of ways = 4+3–1C3–1 = 6C2 = = 15
4! × 2!
Case 5 3rd digit is 5
(a) If the 4th digit is 1 then we have only 512 and so The correct answer is A.
such 6 numbers are possible.
6. Since the hexagon formed by joining those 6 points
(b) If the 4th digit is 2 then there is no such number.
is regular, any pentagon formed using 5 vertices will
(c) If the 4th digit is 3—536 and so such 6 numbers be congruent.
are possible.
Hence, only one pentagon with distinctly different
(d) If the 4th digit is 4—no such number is possible.
area can be formed.
(e) If the 4th digit is 6—no such number is possible.
The correct answer is D.
Case 6 3rd digit is 6
(a) If the 4th digit is 1 then no such number is 7. In each section 3 questions are to be selected from
possible. the five designated questions. This can be done in
(b) If the 4th digit is 2 then 624 is possible and so 6 5C3 ways.
numbers can be formed.
Hence, the total number of different tests
(c) If the 4th digit is 3 then we have 632 and such
6 numbers are possible. = 5C3 x 5C3 x 5C3 = 1000
(d) If the 4th digit is 4 then no such number can be The correct answer is E
formed.
(e) If the 4th digit is 5—no such number is possible. 8. If five different types of sweets A, B, C, D and E then
the required answer will be non-negative integral
Total such possible numbers are 12 + 18 + 12 + 18
solution of the equation
+ 12 + 12 = 84
A + B + C + D + E = 8
The correct answer is D.
Hence required answer
Alternatively,
= 8+5–1C5–1 = 12C4 = 495
The last three digits of the number can be from 121
to 165, or 213 to 265, …, or 612 to 654. The correct answer is B
In the list 121 to 165, the numbers which are
multiples of 8 without repetition of digits and which 9. We have 13 Y (yellow) balls, 19 G (green) balls, 27 R
use available digits only, are 136 and 152. (red) balls, 10 B (black) balls, 7 b (brown) balls and
14 W (white) balls in all.
Similarly, in the list 213 to 265, the numbers are
216, 256 and 264. Let us take out maximum 13 balls of different
In the list 312 to 365, the numbers are colours, then we can
312 and 352. take out (13Y) + (13G) + (13R) + (10B) + (7b) +
(13W) balls = 69 balls.
In the list 412 to 465, the numbers are
If we take one more ball out now, we will certainly get
416, 432 and 456.
14 balls of either green, red or white colour. That is,
In the list 512 to 546, the numbers are if we take out 70 balls from the bag, we will get 14
512 and 536. balls (at least) of same colour.
In the list 612 to 654, the numbers are The correct answer is B.

249

Book 1.indb 249 30/04/2019 4:48:15 PM


NMAT by GMAC™ Official Guide 2019

10. Out of 5 girls, 3 girls can be invited in 5C3 ways. [(2n)(2n − 2)(2n − 4)(2n − 6)...(4)(2)][(2n − 1)(2n − 3)(2n − 5)...(3)(1)]
Nothing is mentioned about the number of boys that n!
Salim has to invite.
[2n (n)(n − 1)(n − 2)(n − 3)...(2)(1)][(2n − 1)(2n − 3)(2n − 5)...(3)(1)]
Salim can invite 1, 2, 3, 4, or even no boy. n!

Out of 4 boys, Salim can invite them in the said [2n (n !)][(2n − 1)(2n − 3)(2n − 5)...(3)(1)]
manner in = 5C3 x (2)4 = 10 x 16 = 160
n!
The correct answer is B.
= 2n(2n – 1)(2n – 3)(2n – 5)…(3)(1).
11. Out of 10 scholars, we can select 4 of them in 10C4 The correct answer is A.
ways.
Now, 10C4 = 210 16. The factors of 210 are as follows:
The correct answer is B. 1 and 210
2 and 105
12. The given word is ‘BALLASTIC’.
3 and 70
These are seven different alphabets. Four-lettered 5 and 42
words that can be formed when all the letters are
6 and 35
different = 7 × 6 × 5 × 4 = 840
7 and 30
When two letters are different and other two letters
are A’s 10 and 21
4! 6 × 5 × 4! 4 × 3 × 2! 14 and 15
= 6C2 × = × = 180 Out of the list of 16 factors, there are two multiples
2! 4! × 2! 2! × 2!
of 42 (42 and 210).
When two letters are different and other two are L’s
2 1
Thus, the answer is or .
4! 6 × 5 × 4! 4 × 3 × 2! 16 8
= 6C2 × = × = 180
2! 4! × 2! 2! × 2! The correct answer is C.
When two letters are A’s and two letters are L’s
17. In this case, there are two independent events: hail
4! on Thursday and hail on Friday. The question asks
= =6
2! × 2! about the probability that there will be hail on both
the days. Hence, we simply need to multiply the
Total number of words individual probabilities together as follows to arrive at
= 840 + 180 + 180 + 6 = 1206 the answer:

The correct answer is D. 1 1 1


= ´ =
6 6 36
13. Number of boys = 10 The correct answer is A.
Number of girls = 5
10
18. There are 12 girls and 20 boys in the classroom.
Required number of ways = C5 × 5C3 = 2520
1
The correct answer is E. If of the girls have cell phones, then there are
4
1
14. The sum of the digits is 15 and the number will 12 ´ = 3 girls with cell phones. Therefore, there
4
always be a multiple of 3, so it can never be a prime are 12 – 3 = 9 girls who do NOT have cell phones.
number.
Therefore, the probability of choosing a girl who does
The correct answer is A. not have a cell phone is the number of girls without
cell phones divided by the total number of children,
15. Given expression = 9
which is
(2n !) (2n)(2n − 1)(2n − 2)(2n − 3)...(4)(3)(2)(1) 32
=
n! n! The correct answer is B.

250

Book 1.indb 250 30/04/2019 4:48:16 PM


3.0  Quantitative Skills Practice

19. The probability that at least one roll results in chosen, you can obtain a match by either choosing a
a number higher than 4 is equal to 1 minus the
probability that all three of the rolls result in numbers pair of white OR a pair of red, so you must add their
4 or lower. For one roll, there are 6 possible probabilities to get the total chance of a pair. This gives
outcomes and 4 ways in which the outcome can be 1 1 2
P (Bag A Pair) = + + .
4 2 5 5 5
4 or lower, so the probability is = . Thus, the
6 3
Similarly, if Bag B is chosen, the probability of
probability that all three rolls result in numbers 4 or
æ 6 öæ 5 ö 5
æ 2 öæ 2 öæ 2 ö 8 a pair of white beads is ç ÷ ç ÷ = and the
lower is ç ÷ ç ÷ ç ÷ = . This is the result that è 9 ø è 8 ø 12
è 3 ø è 3 ø è 3 ø 27  3  2 1
probability of a pair of red beads is     = .
you do NOT want; subtract this from 1 to get the  9  8 12
probability that you do want: Therefore, the probability of a pair is P (Bag B pair) =
æ 8 ö 19 5 1 6 1
Therefore, the required probability = 1 - ç ÷ = + = =
è 27 ø 27 12 12 12 2 . The probability of choosing Bag
The correct answer is D. A AND a pair from Bag A is the product of the two
æ 1öæ 2 ö 1
events, ç ÷ ç ÷ = .
20. This question is an OR question, so you may be è 2 øè 5 ø 5
tempted to simply add the two probabilities. However,
this will give you a number greater than 100%, which is Similarly, the probability of choosing Bag B AND a
NEVER possible: 0.8 + 0.25 = 1.05. This is because æ 1öæ 1ö 1
this figure double-counts the cases where Deeksha pair from Bag B is ç ÷ ç ÷ = . The total probability
è 2 øè 2 ø 4
skips her lunch and there is a power failure. Subtract
out these cases in order to find the desired value. of choosing a pair will be the probability of choosing
Bag A and a pair from Bag A OR choosing Bag B and
In order to calculate the probability that Deeksha will a pair from Bag B, meaning you must sum up these
skip her lunch AND that there will be a power failure, two events. This gives: P (pair)
multiply the individual probabilities together:
0.8 × 0.25 = 0.2 1 1 4 5 9
= + = + + .
Then, subtract to find the required probability: 5 4 20 20 20
1.05 – 0.2 = 0.85, or 85% The correct answer is C.
The correct answer is C.
22. Total possible results on rolling two dice,

21. The probability of choosing Bag A, P(A), and the n(S) = 62 = 36


probability of choosing Bag B, P(B), must be the Let A be the event that a sum of 10 or more occurs.
1 A = {(4,6), (5,5) (5,6), (6,4), (6,5), (6,6)}
same, that is, P(A) = P(B) = .
2 n (A) = 6
1
If Bag A is chosen, what is the probability of a matched Required probability =
6
pair? First, compute the probability of two whites. The
The correct answer is C.
3
probability of the first white is and the probability
6
2 23. Let A be the event of getting two bananas and B be
of the second white is , , so the probability of a first the event of getting two good fruits.
5
æ 3 öæ 2 ö 1 Therefore, (A ∩ B) will be the event of getting two
AND second white is ç ÷ ç ÷ = . Similarly, the
è 6 øè 5 ø 5 good bananas.
20
3  2 1 1 2 C2
probability of two reds is ( ) =+ .+If Bag A is P(A) = P(Get 2 bananas) =
6  5  5 5 5
30
C2

251

Book 1.indb 251 30/04/2019 4:48:18 PM


NMAT by GMAC™ Official Guide 2019

23
C2 28. Number of King cards = 5
P(B) = P(Get 2 good fruits) = 30
C2 Number of Queen cards = 6
15
C Number of Jack cards = 7
P(A ∩ B) = 30 2
C2 There are two cases of drawing second card: First
338 card is Jack or first card is not Jack.
P(A ∪ B) = = P(A) + P(B) – P(A ∩ B) =
435 Required probability
The correct answer is B.
= Probability (first card is jack and second card is
24. Since the events are independent, the outcome of also jack) + (first card is not jack and second card is
one will not affect the other. jack)
Required probability
Given that P(M) = 0.5 and P(N) = 0.4
P(M/N) means the probability of the event M given 7 6 11 7 7
= × + × =
that the event N has already occurred. 18 17 18 11 18
So, the P(M/N) = P(M) = 0.5 The correct answer is B.

The correct answer is B.


29. The given information can be tabulated as follows:
25. Total number of roll numbers = 100
Week Payment for the week Cumulative payment
The number of rolls numbers which are multiples of 3
no.
= 33
33 1 1000 1000
So, the required probability =
100 2 1000 + 1000 = 2000 1000 + 2000 = 3000
The correct answer is C.
3 1000 + 3000 = 4000 3000 + 4000 = 7000
26. Let the number be x. Then, 4 1000 + 7000 = 8000 7000 + 8000 = 15000
5 1000 + 15000 = 16000 15000 + 16000 = 31000
336
x + ≤ 50 or (x – 8)(x – 42) ≤0 6 1000 + 31000 = 32000 31000 + 32000 = 63000
x
Or, 8 ≤ x ≤ 42
The correct answer is D.
So, x can be anything from 8 to 42, that is, 35
numbers. 30. The collection figures are in AP, with the first term as
35 7 100,000 and the common difference is 20,000.
Required probability = =
50 10 Then,
The correct answer is A.
10
S10 = [2a + (n–1) x d]
27. Given word is ‘MANAGEMNT’. We see that there are 2
two M, N and A each. Therefore, = 5[200,000 + (9 x 20,000)
= 5 (380,000)
Number of ways of arranging letters of
= Rs. 1900,000
9!
‘MANAGEMNT’ =
2! × 2! × 2! The correct answer is E.
Cases where both A’s are together, can be
considered as a single unit. Therefore, 31. As, the terms are in H.P. so, their reciprocal must be
in A.P. (Arithmetic progression)
8!
Favourable outcomes =
2! × 2! So, 3/2 and 9/2 are the 5th and 9th terms of the A.P.
8! and we know if a and d are 1st term and common
2! × 2! 2 difference of A.P.,
Required probability = =
9! 9 Then, 9th term – 5th term = 4d = 9/2 - 3/2
2! × 2! × 2!
So, d = 3/4
The correct answer is B.

252

Book 1.indb 252 30/04/2019 4:48:19 PM


3.0  Quantitative Skills Practice

Now, the 1st term of A.P. = 5th term - 4d p q r s


= 3/2 - 4(3/4) = -3/2 35. , , , are all positive real numbers and for
q r s p
Hence, the 1st term of H.P. must be the reciprocal of positive numbers.
1st term of A.P. and so, the 1st term of H.P. is
1/(-3/2) = -2/3 = -0.67 A.M. ≥ G.M.
The correct answer is C. p q r s
+ + +
So, q r s p 4p q r s
³ ´ ´ ´
32. Ajit puts Re 1 on January 1, Rs. 3 on January 2, Rs. 4 q r s p
6 on January 3, Rs. 12 on January 4 and so on. p q r s
or, + + + ³4
So, the series is 1 + 3 + 6 + 12 + 24 + … q r s p
The 2nd to 31st term are in G.P. with 1st term 3 The correct answer is E.
and common ratio = 2
36. Suppose b = ar, c = ar2 and d = ar3. Then
So, the total money in Ajit’s piggy bank at the end of
January is sum of series 1 + 3 + 6 + 12 + 24… 1 1
3 3
= 3
= 1 + sum of 30 terms of G.P. with 1st term and a +b a (1 + r3 )
common ratio 3 and 2 respectively 1 1
3 3
= 3 3 and
n
= 1 + a(r - 1)/(r - 1) (where a = 3; r = 2 and n = 30) b +c a r (1 + r3 )
So, amount at the end of January is 1 1
3 =
= 1 + 3(2 30
- 1)/(2 - 1) c + d3 a3 r6 (1 + r3 )
30
= 1 + 3(2 - 1) Clearly, (a3 + b3)–1, (b3 + c3) –1, and (c3 + d3) –1 are
30
= 1 + 3(2 ) - 3 1
also in GP with common ratio 3
= 3(230) - 2 r

The correct answer is C. The correct answer is B.

33. Here, the person saves Rs. 400 in the first year, Rs. 600 37. In each hour, the hour hand points at different
in the second year, Rs. 800 in the third year and so on. numbers. So, the numbers pointed at are 1, 2, 3, 4,
5, 6, 7, 8, 9, 10, 11 and 12.
Hence, this forms an AP with the first term, a = 400
and the common difference, d = 200. Sum of natural numbers from 1 to 12
According to the question, n(n + 1) 12(12 + 1)
n = = = 6 × 13 = 78
Sn = [2a + (n – 1)d] 2 2
2
n Since the hour hand points at these numbers twice in
⇒ 18000 = [800 + (n – 1)200]
2 a day of 24 hours, the required answer = 2 × 78 =
n 156
⇒ 18000 = [600 + 200n] = 300n + 100n2
2 The correct answer is C.
We have n2 + 3n – 180 = 0
a
⇒ n = 12 38. Let the first term of geometric progression be
r
The correct answer is B. and the common ratio be r.
a
34. Given that, T2 = ar = 6 and T5 = ar4 = 48 First term =
r
Hence, r = 2 and a = 3 a
Second term = ×r = a
So, T10 = 3 × 29 = 1536 r
The correct answer is C. Third term = a × r = ar

253

Chapter 3.indd 253 01/05/2019 4:28:17 PM


NMAT by GMAC™ Official Guide 2019

According to the question, The correct answer is A.


a
× a × ar = 27 40. In AP1, tn = 3n – 1 and so t45 = 135 – 1 = 134
r
In AP2, tn = 2n – 1 and so t55 = 110 – 1 = 109
⇒ a3 = 27 ⇒ a = 3
LCM of common difference of the two series = LCM
Again, as per the question,
(3, 2) = 6, which is the common difference of the AP
a   a series of the terms common to the two APs.
 r × a + (a × ar ) +  ar × r  = 91 Since the first common term is 1, next common
terms will be 7, then 13, and so on.
3   3
⇒  × 3 + (3 × 3r ) +  3r ×  = 91 tn = 6n – 5, which should be less than or equal to 109.
r   r
So, 6n – 5 ⇒ 109
9
⇒ + 9r + 9 = 91 114
r ⇒n≤ ⇒ n = 19
6
9r2 + 9r + 9
⇒ = 91
r The correct answer is D.
⇒ 9r2 – 82r + 9 = 0 3 1
2
41. Since of the chocolates have nuts and of the
⇒ 9r – 81r – r + 9 = 0 4 3
⇒ 9r(r – 9) – 1(r – 9) = 0 chocolates have both nuts and fruits, we can simply
⇒ (9r – 1) (r – 9) = 0 3 1
subtract – to get all the chocolates with nuts
⇒ 9r – 1 = 0 Or, r – 9 = 0 4 3
but no fruit.
1
⇒r= Or, r = 9
9
3 1 5
Therefore, the third number – =
4 3 12
1 1 The correct answer is B.
= 9r = 3 × 9 = 27 or 9r = 3 × =
9 3
42. Total = 500
The correct answer is E.
Biscuit based = 210
1 4 2 4
39. Given harmonic proportion is , , , , ... Milk based = 160
5 19 9 17
Coconut based = 160
Corresponding arithmetic proportion will be
Milk and coconut = 80
19 9 17
5, , , , ... Milk and biscuit = 70
4 2 4
Biscuit and coconut = 60
First term, a = 5
All three = x
19 1
Common difference, d = -5= - None = 200
4 4
t10 = a + 9d Now, the formula is
Total - None = A + B + C - (AB + BC + CA) - 2(ABC)
 1  11
= 5 + 9 × -  = Or 500 - 200 = 210 + 160 + 160 - (80 + 70 + 60)
 4 4
- 2x
So, the 10th term of the given harmonic progression
Or, 300 = 530 - 210 - 2x
4
= Or, 2x = 320 - 300
11

254

Book 1.indb 254 30/04/2019 4:48:22 PM


3.0  Quantitative Skills Practice

Or, x = 20/2 = 10 46. Students studying all the three subjects


The correct answer is A. = 500 – (285 + 195 + 115 – 45 – 70 – 50) – 50 = 20
The correct answer is D.
43. The given information can be put into Venn diagram
form as drawn below:
47. It is not given that all of them study at least one
subject.
The Sun
Daily Mail The correct answer is E.
9 26
0 10-x 48. Let’s assume that the number of possible food items
of all 3 types is zero. Then, we get the following
7 figure (first figure).
6 x
Sweet Sweet
11-x Spicy
80 60 80
–10 40 30 0
–30 10
24 Daily Mirror
0 40
50 50 10 10
Let x be the number of persons who have subscribed
to only Daily Mail and Daily Mirror. Then, 10 – x –40 0
subscribed to only Daily Mail and 11 – x to Daily
Mirror. The sum of all is 40. So,
60 Tangy 60 Tangy
0 + 9 + 7 + 6 + 10 – x + x + 11 – x = 40 or x = 3.

Therefore, number of persons who have subscribed
to only two = 9 + 6 + 3 = 18. Sweet Sweet
Spicy Spicy
The correct answer is D.
80 60 80 60
–10 40 30 0
44. Number of girls with mobile phones –30 10

 2  0 40
=   × 35 = 10 50 50 10 10
 5+2
3 –40
Total number of boys = × 100 = 60 0
5
So, total number of girls = 100 – 60 = 40 60 Tangy 60 Tangy

Therefore, the required percentage
This assumption leads to negative values in some
10 regions. The greatest negative term is – 40, and so
= × 100 = 25%
40 the number of possible food items of all 3 types is
The correct answer is C. taken to be 40 now, and redraw the figure (second
one above). So, the minimum possible food items of
45. If x students like both the subjects, then number of all 3 types is 40.
students who like only Chemistry is (45 – x) and The maximum possible food items of all 3 types
the number of students who like only Physics is cannot be more than the least of the pair-wise
(60 – x). numbers given. So, the maximum possible value
cannot exceed 40. And in case of 40, we already
Hence, (45 – x) + (60 – x) + x + 5 = 100 know that rest of the data is consistent. So, in this
⇒ 110 – x = 100 case, minima and maxima are same.
⇒ x = 10 Therefore, required ratio is 1 : 1.
The correct answer is C. The correct answer is E.

255

Book 1.indb 255 30/04/2019 4:48:22 PM


NMAT by GMAC™ Official Guide 2019

49. Note that the point (0, 4√3 - 3) lies on the y axis
and the point (-4, -3) lies in the third quadrant. The
(a − 4)2 + ((2 − 6)2 = 5
easiest way to form an equilateral triangle would be ⇒ a2 − 8a + 16 + 16 = 5
to take the mirror image of either of these two points
in the opposite quadrant, that is, (0, - 4√3 - 3) or ⇒ a2 − 8a + 32 = 25
(4, -3). Option D gives us one of these two options ⇒ a2 − 8a + 7 = 0
and so should be the correct answer.
⇒ a2 − 7a − a + 7 = 0
The correct answer is D. ⇒ a(a − 7) − 1(a − 7) = 0
50. If the points (x1, y1), (x2, y2) and (x3, y3) are ⇒ (a − 1)(a − 7) = 0
collinear (lie on same line) then their slopes are ⇒ a = 1, 7

equal. So,
7 − a 11 − 7 The correct answer is D.
=
11 − 0 9 − 11
52. Let us assume that A, B and C are the three points
On solving, we get: a = 29 on a straight line.
The correct answer is E. For these three points A, B and C to be on a straight
line, we have the following result:
51. Given that the distance between the points A(a, 2)
and B(4, 6) is 5 units. Slope of AB = Slope of BC
10 − 11 9 − 10
So, using the distance formula, the distance AB is ⇒ =
a−5 15 − a
given by: ⇒ −1(15 − a) = −1(a − 5)
⇒ 2a = 20 ⇒ a = 10

Therefore, the value of a = 10
The correct answer is C.

256

Book 1.indb 256 30/04/2019 4:48:23 PM


3.0  Quantitative Skills Practice

5. Data Interpretation The required percentage for Wendy


1100
3 = ´ 100 = approximately 20 %
1. Ratio in 2010 = = 0.06 5450
50
The required percentage for Caret
2
Ratio in 2011 = = 0.044
45 950
= ´ 100 = approximately 22 %
2.5 4350
Ratio in 2012 = = 0.0625
40 The correct answer is C.
2.5 1100
Ratio in 2014 = = 0.052 6. The required percentage = ´ 100 = 110%
48 1000
2.1
Ratio in 2015 = = 0.051 The correct answer is C.
41
Thus, the ratio was highest in 2012.
7. While you can try calculating the required ratio for all
The correct answer is C. the companies to get to the answer, the faster way
to calculate this question is to look at the figures for
the companies for April and February. Only in the
4 case of Shelby is the April figure greater than the
2. Ratio in 2010 = = 0.066
60 February figure. Thus, the highest ratio has to be that
3.5 of Shelby.
Ratio in 2012 = = 0.056
62 The correct answer is B.
1.5
Ratio in 2013 = = 0.037
40 8. The number of trousers manufactured in February
2 and March by Caret = 850 + 950 = 1800
Ratio in 2014 = = 0.044
45 The number of trousers manufactured in February
2.3 and March by Wendy = 1150 + 1100 = 2200
Ratio in 2015 = = 0.046
50 The number of trousers manufactured in February
Thus, the ratio was lowest in 2013 and March by Kooper = 1050 + 1050 = 2050

The correct answer is C. The number of trousers manufactured in February


and March Shelby = 700 + 800 = 1500
3. The required difference = 700 – 550 = 150 The number of trousers manufactured in February
and March by Prisma= 1050 + 800 = 1850
The correct answer is A.
The correct answer is D.
æ 62 - 3.5 ö
4. The required percentage = ç ÷ ´ 100
è 58 - 4 ø 9. The mode of a list of numbers is the number that
occurs most frequently in the list. In the bar graph for
= approximately 108%
GPA, dark gray bars represent the students in 2010,
The correct answer is D. and the mode of that data set is indicated by the
tallest dark gray bar. This is at grade point average
5. The required percentage for Prisma of 3.3. There were 625 students with a grade point
average of 3.3 in the year 2010, more students than
1050 had any other grade point average.
= ´ 100 = approximately 23.33 %
4500
The required percentage for Shelby The correct answer is B.

800 10. The median is the ‘middle value’ of an ordered list


= ´ 100 = approximately 19 %
4100 of numbers, dividing the list into roughly two equal
The required percentage for Kooper parts. For the 3,000 students in 1980, the median
GPA is the average of the 1,500th and the 1,501st
1000 highest GPA. The students in 1980 are represented
= × 100 = approximately 23.25 %
4300 by the light gray bars.

257

Book 1.indb 257 30/04/2019 4:48:24 PM


NMAT by GMAC™ Official Guide 2019

150 students had a 4.0 GPA. 14. Note that there are 5 vertical grid lines for every 10
225 students had a 3.7 GPA. (Total with this GPA and players, so each vertical grid line accounts for 2
higher = 150 + 225 = 375) players.
300 students had a 3.3 GPA. (Total with this GPA and Male athletes are represented by the light gray bars
higher = 375 + 300 = 675) for each sport. Sum the male athletes on each of the
450 students had a 3.0 GPA. (Total with this GPA and separate teams.
higher = 675 + 450 = 1,125) Males on Volleyball team: 0
475 students had a 2.7 GPA. (Total with this GPA and Males on Athletics team: 37
higher = 1,125 + 475 = 1,600) Males on Tennis team: 9
Males on Football team: 10
The 1,500th and 1,501st students fall between the
1,125th and 1,600th students. Thus, the 1,500th and Males on Cricket team: 17
1,501st highest grade point averages are both 2.7. Males on Basketball team: 14
There are 0 + 37 + 9 + 10 + 17 + 14 = 87 male
The correct answer is D.
players in all of the teams combined, but there are
only a total of 76 male players. Thus, there must be
11. In 2010,
87 – 76 = 11 male players who are counted twice by
350 students had a 4.0 GPA. being on both the Athletics and Cricket teams.
525 students had a 3.7 GPA. The correct answer is A.
625 students had a 3.3 GPA.
15. A sport in which male players outnumber female
500 students had a 3.0 GPA.
players will have a shorter dark gray bar than a light
Thus, there were 350 + 525 + 625 + 500 = 2000 gray bar. This is only the case for Football, where
students who earned at least a 3.0 GPA in the year there are 10 male players and 7 female players.
2
2010, out of a total of 3000 students. This is of
3 The correct answer is D.
the students, or about 67% of the students.
The correct answer is C. 16. There are 9 female tennis players and 14 male
basketball players.
12. In 1980,
Thus, the required ratio = 9 : 14
150 students had a 4.0 GPA.
The correct answer is B.
225 students had a 3.7 GPA.
16
300 students had a 3.3 GPA. 17. The population of B in 2005 = 5000 x
13
450 students had a 3.0 GPA. = approximately 6150
In 1980, 150 + 225 + 300 + 450 = 1,125 students 16
had a GPA of 3.0 or higher. The population of B in 2006 = 6150 x
13
Thus, 3,000 – 1,125 =1,875 students earned a GPA = approximately 6750
less than 3.0. As a percent of the class, this was
The population below poverty line = 52% of 6750
 1875  = approximately 3500
 3000  × 100 = 62.5%
The correct answer is C.
The correct answer is D.
17
18. The population of Town D in 2005 = 9000 x
13. Note that there are 5 vertical grid lines for every 15
= 10200
10 players, so each vertical grid line accounts for
110
2 players. On the Athletics team, there are between The population of Town D in 2007 = 10200 x
36 and 38 men (so it must be 37) represented 100
= 11220
by the light gray bar. On the Athletics team, there 95
The population of Town G in 2007 = 9000 x
are between 60 and 62 women (so it must be 61) 100
= 8550
represented by the dark gray bar. In fractional form,
37 Therefore, the required total = 11220 + 8550
the required ratio =
61 = 19770
The correct answer is A. The correct answer is B.

258

Book 1.indb 258 30/04/2019 4:48:25 PM


3.0  Quantitative Skills Practice

19. The population of Town F below the poverty line = (208 - 171)/171 × 100 = 21.64%
55000 13 49 = 3500 The correct answer is A.
= × ×
100 100 100
The correct answer is C. 25. Let the ratio of mid cap to small cap for Paul be
x : y.
20. The population of Town C below the poverty line Now all small cap stocks have same weightage,
38 So, the value of Paul’s portfolio must be x(stock 4)
= 2000 x = 760
100 + y(stock 1 + stock 2 + stock 3)
The population of Town E below the poverty line So, 36,000 = x(5,000) + y(6,500 + 7,500 + 7,000)
2000 46 36,000 = x(5,000) + y(21,000)
= × 18 × = 2070
8 100 Now, x > y (as weightage of mid cap is more than
Therefore, the required ratio = 760:2070 small cap stocks)
= 76:207 If we check values, then only option D and E have x >
y. So, we check only D and E
The correct answer is B.
Using option D, x = 3 and y = 1 gives the correct
Alternatively, value of portfolio
We do not need to know the population of any town The correct answer is D.
to solve this question.
Required ratio = (8 × 38) : (18 × 46) = 76 : 207. 26. Let value of stock 3 which Sheela holds be x
21. Let’s calculate the total rabi production in each year So, value of her portfolio is 1(4,800 + x) + N(8,900)
using the table we have made earlier: The value of her portfolio is 39,500
Total rabi production in Y1 = 84 So, we have 1(4,800 + x) + N(8,900) = 39,500
Total rabi production in Y2 = 112 For, x to be maximum, N must be minimum and we
Total rabi production in Y3 = 171 know weightage of mid cap stock must be more than
that of small cap.
Total rabi production in Y4 = 208
So, N > 1.
Total rabi production in Y5 = 235
Also, given N must be an integer. The smallest
While you can calculate the actual percentage
integer > 1 is 2 and hence for x to be maximum,
increase for each year, by observation this will clearly
N=2
be in Y3. Calculating the actual percentages will take
too much time. This should be avoided. So, we have 1(4,800 + x) + 2(8,900) = 39,500
Solving it we get, x = Rs.16,900
The correct answer is C.
The correct answer is C.
22. Again, do not try to actually calculate. For foodgrains
and cereals, the production has approximately doubled 27. Let the weightage of small to mid cap stocks for
from Y2 to Y5. For pulses it is much less, so pulses Bindu be x : y where y > x
are out of consideration. For oilseeds it’s 2.5 times,
So, sum of value of stock 1 and 3 is x(7,300 +
whereas for rice it is more than 3 times. Thus, the
6,700)
percentage increase has to be the highest for rice.
Value of mid cap stock 4 is y(6,300)
The correct answer is A. As, value of small cap and mid cap stocks is same

23. Required average = (32 + 45 + 68 + 88 + 97)/5 Hence, x(7,300 + 6,700) = y(6,300)


14,000 x = 6,300y
= 330/5 = 66
Therefore, x : y = 6,300 : 14,000 = 63 : 140 =
The correct answer is E. 9 : 20

24. Total rabi production in Y3 = 171 The correct answer is E.

Total rabi production in Y4 = 208 28. Let the ratio of small cap to mid cap for Thomas be
Therefore, percentage increase x : y (y > x)

259

Book 1.indb 259 30/04/2019 4:48:25 PM


NMAT by GMAC™ Official Guide 2019

Total value of stock 3 = x(1200) Percentage growth in sales of herbal hair vitaliser
Total value of stock 4 = y(8,400) = (1890000 – 246000)/246000 × 100
It is given, x(1200) = 1/8 y(8,400) = 668.2%
So, x : y = 8,400/(8 × 1200) = 7 : 8 Therefore, required ratio = 138/668 = 1 : 5 approx
The correct answer is C. The correct answer is B.

29. On 22nd June INR 46 = 1 dollar 34. Total sales of Herbal Beauty product in 2000

Therefore, INR 4000 = 4000/46 = 87 dollar approx. = 1010000 + 246000 + 748000 + 344000 +
nd 250000
Now, on 22 June, 1 dollar = 80 NPR
= 2598000
Therefore, 87 dollar = 87 × 80 = 6960 NPR
Total sales of Herbal Beauty product in 2005
Option B comes closest to this and should be the
= 2410000 + 1890000 + 1460000 + 611000
correct answer.
+ 510000
The correct answer is B. = 6881000
Alternatively, Therefore, percentage growth
NPR/INR = (NPR/Dollar) ÷ (INR/Dollar) = (6881000 - 2598000)/2598000 × 100
nd
= 80/46 (for 22 June) = 164.8%
80
So, the number of NPRs for INR 4000 = × 4000 The correct answer is C.
46
= NPR 6956
30. If the NPR/dollar ratio follows the same trend, on 35. By observation, we can eliminate Hair wash, Hair
30th July 1 dollar = 82 NPR (the actual number will Vitaliser and Moisturiser because in all three the
be slightly lower than 82 but let’s take 82 as it would sales have more than doubled from 2000 to 2005,
make calculations easier) that is, an increase of more than 100%.

Therefore, 150 dollars = 150 × 82 = 12300 NPR For face cream and face wash, we can calculate the
percentage increase:
Since the actual number should have been slightly
lower than 82, we should choose the option slightly Face cream = (1460000 - 748000)/748000 × 100
lower than 12300. = 95.1%
The correct answer is C. Since face cream is incorrect, the answer has to
be face wash, so you don’t necessarily have to
31. For 23rd June 1$ = INR 45.5 and so 1000$ calculate it.
= INR 45,500. So, Aman can buy the bike using However, if we do calculate, % increase for face
INR 45,500 wash = (611000 - 344000)/344000 × 100
For 23rd June 1 Euro = $1.6 and so 600 Euro = = 77.6% = 78% approx.
$(600 × 1.6) = $960. It is $40 less than the cost of
the bike So, Aman can’t buy the bike using Euro 600 The correct answer is B.
For 23rd June 1$ = NPR 78 and so 77,000 NPR =
$(77,000/78) which is again less than $1,000. So, 36. Percentage growth in sales of Herbal Hair Vitaliser
Aman can’t buy the bike using 77,000 NPR = 668.2% (already calculated in Q72)
The correct answer is B. Percentage growth in sales of the rest of the
products = (4991000 – 2352000)/2352000 × 100
32. We cannot arrive at the answer since all the answer = 112.2 %
choices are given in Euro and we do not know the
INR/Euro or Dollar/Euro ratio on June 18th. Thus, required ratio = 668/112
= 6 : 1 approx
The correct answer is E.
The correct answer is D.
33. Percentage growth in sales of herbal hair wash
= (2410000 - 1010000)/1010000 × 100 37. Only Sulpar and Sictor have more than doubled over
the entire period.
= 138.6%

260

Book 1.indb 260 30/04/2019 4:48:25 PM


3.0  Quantitative Skills Practice

Overall growth of Sulpar 41. Percentage of students securing more than 60%
270
 438200  marks in aggregate = × 100 = 27%
=  − 1 × 100 = 125% 1000
 194830 
And percentage of students securing more than 40%
Overall growth of Sictor
730
marks is = × 100= 73%
 2400  1000
=  − 1 × 100 = 218.18%
 1100  Hence, the difference = 73% – 27% = 46%

Therefore, Sictor experienced the second highest The correct answer is C.


annual growth rate.
42. The number of students securing more than 20
The correct answer is B. marks in English = 800

38. Total number of two-wheelers on roads of the country The number of students securing 40 marks in
in year 2007 Computer Science cannot be determined from the
given information.
= 1300 + 249200 + 42300 + 479200 + 172600 +
1600 + 137600 + 65000 + 14400 The correct answer is E.
= 1163200
43. Number of students securing more than 60% in
Total number of two-wheelers on roads of the country computers is 210 and number of students securing
in year 2009 more than 40% in aggregate is 730.
= 1900 + 325000 + 49200 + 416700 + 162400 +
3000 + 125700 + 68000 + 16500 210
So, the required percentage = × 100
= 1168400 730
Required difference = 1168400 – 1163200 = 5200 = 28.67% = 29%

The correct answer is D. The correct answer is B.

39. Total number of RHONDA two-wheelers in the year 44. Number of employees in 2012 = 2000
2008 = 403400 Change in number of employees in 2013

Total number of two-wheelers in the year 2008 = 350 – 250 = 100

= 1800 + 266000 + 43300 + 403400 + 150400 + So, the required percentage change
2300 + 121700 + 64900 + 15000 = 1068800 2100 − 2000
= × 100 = 5%
2000
So, the required percentage
403400 The correct answer is D.
= × 100 = 37.74% ≈ 38%
1068800 General Solution for Questions 45–46
The correct answer is D. Number of employees in 2013
= 2000 – 250 + 350 = 2100
40. From the explanation of the previous question, we
Number of employees in 2014
have:
= 2100 -– 450 + 300 = 1950
Total number of two-wheelers in the year 2008
Number of employees in 2015
= 1068800
= 1950 – 300 + 400 = 2050
Therefore, average number of two-wheelers in the
year 2008 Number of employees in 2016
1068800 = 2050 – 250 + 500 = 2300
= = 118756
9 Number of employees in 2017
The correct answer is A. = 2300 – 400 + 350 = 2250

261

Book 1.indb 261 30/04/2019 4:48:26 PM


NMAT by GMAC™ Official Guide 2019

45. Looking at the above calculation, it is clear that 51. If price of barley in 1998 is x,
the number of employees in ABC Pvt. Ltd. was the
3000 + 2500 + x
maximum in the year 2016. then = 3000 or x = 3500.
3
The correct answer is C.
The price of barley in 1997 is Rs. 1750 and the price
46. Looking at the above calculation, it is clear that none of barley in 1998 is Rs. 3500.
of the given two years has the number of employees So, the increase is 100%.
in ABC Pvt. Ltd. the same.
The correct answer is D.
The correct answer is E.
52.
47. Total volume of sales in the year 2011
= 405 × 150 = 60750 Maize Java Barley
Total volume of sales in the year 2015 1996: 3000 2500 2000
= 60 = 50 = 40
= 370 × 265 = 98050 50 50 50
Hence, the required percentage 1997 4000 1750 1750
= 80 = 35 = 35
50 50 50
98050 − 60750 3730
= × 100 = × 100 = 61.4%
60750 6075 1998 3000 2500 3500
= 60 = 50 = 70
50 50 50
The correct answer is D.
1999: 4000 1500 500
48. Average of the volume of cars sold = 80 = 30 = 10
50 50 50
405 + 325 + 370 + 380 + 265 + 340 + 160
=
7 From the above calculation we see that in the year
1998 he pays the highest amount of money.
2245
= = 320.71 Alternatively,
7
Average of the sales per unit volume Since he buys equal amount of each, and the graph
gives average price of the three cereals in which equal
150 + 170 + 270 + 320 + 370 + 365 + 320 amounts were taken, the highest price would be paid in
=
7 the case where the average price is highest. Obviously,
1965 the average price is highest in the year 1998.
= = 280.71
7 The correct answer is C.
Therefore, the required difference
53. Maize: 1000 kg
= 320.71 – 280.71 40
Javar: 2000 kg
The correct answer is A. Barley: 3000 kg
In 1997
49. This happens only in the duration 2011 – 2012, Price of maize = 8000
2014 – 15 and 2015 – 2016. In rest of the duration
the trend in both are same. Price of javar = 7000
Price of barley =10500
The correct answer is C.
So, total Price
50. 1996: Let the price of barley be x = 8000 + 7000 + 10500 = 25500
3000 + 2500 + x The correct answer is C.
Then, = 2500, which gives
3
x = 2000 54. Cost of purchase of shares
Similarly, the price of barley in 1997 is Rs. 1750
= 60 × 2424 + 40 × 1397 = 201320
Therefore the ratio is 8 : 7
Selling price of shares
The correct answer is C.

262

Book 1.indb 262 30/04/2019 4:48:26 PM


3.0  Quantitative Skills Practice

= 60 × 2916 + 40 × 1150 July and September. The amount of increase in Feb


= 220960 is less than that in case of May, whereas base value
220960 − 201320 in Jan is more than that in April. So, percentage
Profit percentage = × 100 increase in May will be higher than in Feb. Also,
201320
percentage increase in July is less than 10% whereas
= 9.75%
that in the month of May is more than 10%. So now,
we need to compare May with Sept only.
The correct answer is D.
In both the cases, the amount of increase is same,
that is, 159. But figure in Aug is less than in April.
55. Average price of share E So, percentage increase in September is more than
Sum of price per share of all the months for E 8406 in May.
=
12 12
= 700.5 57. Annual change rate

Average price of share C Price per share in December – Price per share in January
= × 100
Price per share in January
Sum of price per share of all the months for C 27796
=
12 12 Annual change rate
Monthly change rate =
= 2316.33 (approx.) 12

Required percentage Annual Change Rate Monthly Change Rate


700.5 × 100 A 41.60% 3.47%
= = 30.24% (approx.).
2316.33 B – 1.75% – 0.15%
Note that we need not divide the summation by 12
C 32.07% 2.67%
for C and E.
D – 23.94% – 2.00%
The correct answer is B.
E – 32.48% – 2.71%
56. Percentage change in the price of share D in
comparison to the previous month across the months The correct answer is A.
is as shown below:
Alternatively,
Months Share Percentage change Company B, D and E have reported decrease. Also,
Jan 1512 the company having highest annual rate will also have
Feb 1628 7.67%
highest monthly rate. So, annual rate for A is roughly

Mar 1427 – 12.35% 5


× 100 which is approximately 40%, whereas that
Apr 1397 – 2.10% 12
May 1556 11.38% 7
for C is roughly × 100, which is approximately
June 1297 – 16.65% 22
33%.
July 1397 7.71%
Aug 1297 – 7.16% 58. He has scored 54 points in 6 races, i.e., an average
Sept 1456 12.26% of 9 points per race. Let us say he won only 1 race,
Oct 1395 – 4.19%
so he has to score the remaining 44 points in 5
races at an average of 8.8 points per race which will
Nov 1296 – 7.10% not be possible (because scores in those 5 races is
Dec 1150 – 11.27% either 8 or 6). If he wins only 2 races then he has to
score remaining 34 points in 4 races at an average
The correct answer is E. of 8.5 points per race which again is not possible
(because scores in those 4 races is either 8 or 6).
Alternatively, If he wins 3 races then he has to score 24 points in
Check only those months which are given in the remaining 3 races at an average of 8 points. Hence,
options. Also note that June reports percentage the minimum number of victories for D1 will be 3.
decrease. So, we need to only check for Feb, May, The correct answer is C.

263

Book 1.indb 263 30/04/2019 4:48:26 PM


NMAT by GMAC™ Official Guide 2019

59. For finding the lowest position of D2, let us assume 62. Total number of aquatic animals released into the
that D1 finished at a lowest possible position i.e. 3rd. lake at pH 6.5 = 24 + 82 + 40 + 73 + 12 = 231
So, D1 will have a total 60 points at the end of the
Total number of aquatic animals expected to survive
season. Now to minimise the position let us assume
when the pH of the lake changes from 6.5 to 4.5
D2 got highest possible points in 2 of the races and
hence he gets 20 points from 2 races. Now to win he = 40 + 73 = 113
must secure atleast 2 points which can be obtained 113
Required percentage = × 100 ≈ 48.92%
by finishing in 7th place. 231
The correct answer is D.
The correct answer is D.
60. He has not scored a point in one of the races and
63. Original number of aquatic animals released in the
hence he has to score 39 points in remaining 5
lake at pH 6.5 = 24 + 82 + 40 + 73 + 12 = 231
races. 0 points he has already scored so option (A) is
ruled out. If we see for he can score 10, 10, 10, 8, Number of aquatic animals that would survive when
1, 0 points and get a total of 39 points. He can also the pH changes from 6.5 to 6
score 10, 10, 10, 3, 6, 0 or 10, 10, 10, 5, 4, 0. So, = 24 + 82 + 40 + 73 + 12 = 231
only 2 points cannot be scored in any scenario.
Percentage of original number of aquatic animals that
The correct answer is C. would survive when the pH changes from 6.5 to 6
231
= × 100 = 100%
61. After 6 races, we have the following points for 231
different teams:
Number of aquatic animals that would survive when
Team Points the pH changes from 6.5 to 5.5
Team 1 93 = 24 + 82 + 40 + 73 = 219
Team 2 51 Percentage of original number of aquatic animals that
Team 3 30 would survive when the pH changes from 6.5 to 5.5
Team 4 26 219
= × 100 = 94.8%
Team 5 22 231
Team 6 12 Number of aquatic animals that would survive when
Team 7 0 the pH changes from 6.5 to 5 = 24 + 40 + 73 =
Team 8 0 137
Statement (A): Team 2 can have both of its drivers Percentage of original number of aquatic animals that
finishing 1st and 2nd in the last 3 races and hence would survive when the pH changes from 6.5 to 5
54 points can be obtained taking their total to 105. = (137 ÷ 231)100 = 59.3%.
Team 1 may or may not be able to overhaul the
The correct answer is C.
score, so statement (A) can be true.
Statement (B): Team 3 is currently at 30 points, Alternatively,
even if both the drivers of the team get 1st and 2nd The number of aquatic animals to survive
position in the remaining 3 races, they will have a
= 59.3% of 231 = 136.983 = 137.
total of 84 points which is lower than the current
score of Team 1 and hence Team 3 can never win So, number of animals that can’t survive
the race. So, (B) can never be true. = 231 – 137 = 94.
Statement (C): There are two possibilities for D1: From the table, it is clearly visible that this happens
10, 10, 10, 8, 8, 8 and 10, 10, 10, 10, 8, 6. Both at pH level 5.
the cases are only of top 3 finishes and no other
64. Number of trout at pH 6.5 = 24
case is possible. Also, he may not finish first in the
6th race. So, Statement (C) can also be true. Number of trout at pH 4.5 = 0
Statement (D): There are many possibilities. For
Number of trout at pH 5.5 = 0 + 12 = 12
example, 0, 0, 10, 0, 0, 0 and 0, 2, 8, 0, 0, 0.
Percentage change in the number of trout at pH 5.5
The correct answer is B.
over pH 6.5 = (24−12)100 ÷ 24 = 50%

264

Book 1.indb 264 30/04/2019 4:48:27 PM


3.0  Quantitative Skills Practice

Number of bass at pH 6.5 = 82 Number of perches that will not survive when the pH
changes from 6.5 to 4 = 40
Number of bass at pH 4.5 = 0
Number of frogs that will not survive when the pH
Number of bass at pH 5.5 = 0 + 12 = 12 changes from 6.5 to 4 = 0
Percentage change in the number of bass at pH 5.5 Number of clams that will not survive when the pH
over pH 6.5 = (82−12)100 ÷ 82 ≈ 85.4% changes from 6.5 to 4 = 12
Number of perch at pH 6.5 = 40 Total number of animals that will not survive when the
pH changes from 6.5 to 4
Number of perch at pH 4.5 = 40
= 24 + 82 + 40 + 0 + 12 = 158
Number of perch at pH 5.5 = 40 + 12 = 52
Number of trout that will not survive when the pH
Percentage change in the number of perch at pH 5.5 changes from 6.5 to 4.5 = 24
over pH 6.5 = (52−40)100 ÷ 40 = 30%
Number of bass that will not survive when the pH
Number of frogs at pH 6.5 = 73 changes from 6.5 to 4.5 = 82
Number of frogs at pH 4.5 = 73 Number of perches that will not survive when the pH
changes from 6.5 to 4.5 = 0
Number of frogs at pH 5.5 = 73 + 12 = 85
Number of frogs that will not survive when the pH
Percentage change in the number of frogs at pH 5.5
changes from 6.5 to 4.5 = 0
over pH 6.5 = (85−73)100 ÷ 73 = 16.4%
Number of clams that will not survive when the pH
Number of clams at pH 6.5 = 12 changes from 6.5 to 4.5 = 12
Number of clams at pH 4.5 = 0 Total number of animals that will not survive when the
Number of clams at pH 5.5 = 0 + 12 = 12 pH changes from 6.5 to 4.5

Percentage change in the number of clams at pH 5.5 = 24 + 82 + 0 + 0 + 12 = 118


over pH 6.5 = (12 −12)100 ÷ 12 = 0% Required number of times = 158 ÷ 118 ≈ 1.3 times
Therefore, only for bass, the percentage change in The correct answer is B.
the number of animals at pH 5.5 over pH 6.5 will be
greater than 60%. Alternatively,
The correct answer is A. At pH 4, all but frogs will die.
So, number of deaths
Alternatively,
= 231 – 73 = 158.
All trouts will die at pH of 4.5 and then we will have
12 trout when pH is at 5.5. The percentage change Number of deaths at pH 4.5
= 50% decrease. = 231 – (40 + 73) = 118
None of perch or frog will die at pH 4.5 and so 158
Required number of times = = 1.34 times
percentage change in perch from 40 to 40 + 12 will 118
be less than 50%. Similarly, percentage change in
frog will also be less than 50%. 66. Total cost of hotels completed in 2008 (in million
dollars)
All bass would die at pH 4.5 and then we will have 12
bass at pH 5.5. = 275 + 430 + 400 = 1105
Total cost of hotels to be completed in all three years
82 − 12 (in million dollars)
Percentage change = × 100 = 85%.
82
= 275 + 210 + 250 + 430 + 310 + 400 + 250
In case of clams, all will die and later we will have 12
= 2125
clams. So, percentage change = 0%.
Required percentage
65. Number of trout that will not survive when the pH
changes from 6.5 to 4 = 24 1105
= × 100 = 52%
2125
Number of bass that will not survive when the pH
changes from 6.5 to 4 = 82 The correct answer is B.

265

Book 1.indb 265 30/04/2019 4:48:27 PM


NMAT by GMAC™ Official Guide 2019

67. Total cost 71. In 2005, total sales in men’s section in all the
departmental store
= 275 + 210 + 250 + 430 + 310 + 400 + 250
= 13.5 + 21.5 + 4.3 = 39.3
= 2125
Required percentage
Number of rooms
= 600 + 320 + 250 + 400 + 520 + 450 + 500 39.3
= × 100 = 34.87%
112.7
= 3040
Required number of rooms In, 2006, total sales in men’s section in a
departmental store
3040
= = 1.4305 ≈ 1.43 = 12.5 + 18.5 + 5.4 = 36.4
2125
Required percentage
The correct answer is D.
36.4
= × 100 = 31.90%
68. Investment increases at a simple interest of 10% per 114.1
annum. So, the hotels whose constructions had In 2007, total sales in men’s section in all
been completed earlier, their investment values will departmental store
increase accordingly.
= 9.8 + 17.2 + 8.6 = 35.6
Total investment Required percentage
= (275 × 1.2) + (210 × 1.1) + (250 × 1.1) + (430 ×
35.6
1.2) + 310 + (400 × 1.2) + 250 = × 100 = 29.25%
121.7
= 330 + 231 + 275 + 516 + 310 + 480 + 250
= 2392 million dollars In 2008, total sales in men’s section in all
departmental store
Total number of Rooms = 3040
= 11.5 + 18.5 + 11.2 = 41.2
Average value of investment per hotel room in year
2010 Required percentage
2392 41.2
= = 0.786 million dollars ≈ 0.8 million dollars = × 100 = 31.55%
3040 130.6
The correct answer is C. In 2009, total sales in men’s section in all
departmental store
69. Investment here increases at compound interest of
= 12.5 + 15.4 + 9.5 = 37.4
10% per annum compounded annually from the
year 2007. Required percentage

So, the actual investment made in different hotels will 37.4


= × 100 = 27.74%
be compounded depending upon their completion 134.8
times. So, the hotels completed in the year 2008 will
Hence, in the year 2005, 2006 and 2008 total
be 1.1 times their actual investment in the year 2007.
sales of men’s section of all departmental stores lay
Similarly, hotels completed in the year 2009 and between 30% and 40% of the total sales.
2010 will be 1.21 and 1.331 times respectively their
The correct answer is D.
actual investments.
Actual investment made in the year 2007 72. Percentage growth of sales in 2006
275 + 430 + 400 210 + 250 310 + 250 114.1 - 112.7
= + + = × 100
1.1 1.12 1.13 112.7
= 1004.54 + 380.16 + 420.74 1.4
= × 100 = 1.24%
= 1805.44 million dollar 112.7
The correct answer is A. Percentage growth of sales in 2007
121.7 - 114.1
70. As per data given in the table, for the men’s section = × 100
114.1
of Olivestyle, the increase was only in the year 2008.
7.6
= × 100 = 6.66%
The correct answer is D. 114.1

266

Book 1.indb 266 30/04/2019 4:48:28 PM


3.0  Quantitative Skills Practice

Percentage growth of sales in 2008 Therefore, required tax


130.6 - 121.7 25
= × 100 = 16.6 × = 4.15 million dollars
121.7 100
8.9 = 41,50,000 dollars
= × 100 = 7.31%
121.7 The correct answer is D.
Percentage growth of sales in 2009
77. Given that the tax levied on cities and towns
134.8 - 130.6
= × 100
130.6 = 6 × Tax levied on other units
4.2 = 6 × 498000 dollars = 2988000 dollars
= × 100 = 3.21%
130.6 According to the question,
4.5 × Tax levied on township = 2988000
Hence, the highest percentage growth in the sales
of all three stores together, relative to the previous So, tax levied on township
year, was achieved in year 2008. 2988000
= = 664000
The correct answer is D. 4.5
Thus, required tax on township
73. From the given table, we can conclude that the
women’s section sales at Eastside have always been 664000 × 100
= = 4%
higher than the average sales of all three sections at 16.6 × 10, 00000
Eastside.
The correct answer is B.
The correct answer is D.
78. Profit earned by Trots during 2000 to 2002
74. The property tax levied by the government on = 113 million dollars
libraries
Profit earned by Britz during 2000 to 2002
= (50 – 46.6)% = 3.4% of 16.6 million dollars = 71 million dollars
3.4 Therefore, ratio of profits earned by Trots to that by
= × 16.6 million dollars
100 Britz
= 0.5644 million dollars = 5,64,400 dollars = 113 : 71 ≈ 1.6 : 1.
Hence, Trots earned close to 1.6 times the profit
The correct answer is C.
earned by Britz.
75. Tax levied on school, libraries and counties The correct answer is D.
75
= × 16.6 million dollars 79. Profit earned by Trots during 2003 to 2005
100
= 15 + 50 + 80 = 145 million dollars
= 12.45 million dollars
Profit earned by Britz during 2003 to 2005
Tax levied on towns and cities was
= 15 + 60 + 70 = 145 million dollars
= 3.652 million dollars
Required difference
Therefore, tax levied on other units
= 145 – 145 = 0 dollar
= 16.6 – 12.45 – 3.652
= 0.498 × 1000000 = 4,98,000 dollars The correct answer is A.

The correct answer is A. 80. Ratio of profits earned by Trots in 2005 to the profits
it earned in 2004 = 80 : 50 = 8 : 5 = Ratio of profits
76. If the tax levied on counties is x%, then the tax levied earned by Trots in 2006 to the profits it earned in
on Townships, Cities and Towns, and other Units 2005.
together is also x%.
Therefore, profit earned by Trots in 2006
Also, tax levied on Schools and Libraries is 2x%.
8
So, x% = 25%. = × 80 = 128 million dollars
5

267

Book 1.indb 267 30/04/2019 4:48:29 PM


NMAT by GMAC™ Official Guide 2019

For Britz: Total number of recipients in the 35–44 age group in


Profit (2006) : Profit (2005) = Profit (2004) : 2006
Profit (2003) = 60000 + 80000 = 1,40,000
= 60 : 15 = 4 : 1. Number of recipients aged below 35 years in 2005
So, profit in 2006 = 1,50,000 – 1,32,000 = 18,000
= 4 × 70 = 280 million dollars Number of recipients aged below 35 in 2006
Thus, required profit = 1,50,000 – 1,40,000 = 10,000
= 280 – 128 = 152 million dollars Percentage decrease
The correct answer is D. 18000 – 10000
= × 100 = 44.44%
18000
81. Profit earned by Trots in 2005
The correct answer is D.
= 80 million dollars
Profit earned by Trots in 2006 84. Total number of recipients aged above 49 years and
below 65 years in 2005
115
= 80 × = 92 million dollars
100 = (104 + 144 + 140) × 1000
Total profit earned by Trots between 2000 to 2006 = 388 × 1000
= 350 million dollars Total number of recipients aged above 49 years and
below 65 years in 2006
Mean profits earned by Britz between 2000 to 2006
= (96 + 132 + 156) × 1000
350
= = 50 million dollars = 384 × 1000
7
Therefore, required percentage
According to the question,
388 - 384
Mean profit earned by Trots – Mean profit earned by = × 100 = 1.03%
388
Britz = 8 million dollars
⇒ 50 – Mean profit earned by Britz = 8 million dollars The correct answer is A.
Mean profit earned by Britz = 42 million dollars
85. Total number of male recipients between the age of
So, total new profit earned by Britz in 2006 35 to 64 in 2005
= 42 × 7 = 294 million dollars
= 356 × 1000
Thus, required profit earned by Britz in 2006
Total number of male recipients between the age of
= 294 – 216 = 78 million dollars 35 to 69 in 2006
The correct answer is C. = (360 + 20) × 1000 = 380 × 1000
Let the number of male recipients in the age group of
82. Number of recipients in the 60–64 age group in 65 to 69 in 2005 be x thousand.
2005 According to the question
= 100 + 40 = 140
95
Number of recipients in the 60–64 age group in (356 + x ) × = 380
100
2006
= 96 + 60 = 156 380 × 100
⇒ 356 + x =
Thus, required percentage 95
⇒ x = 400 – 356
156 - 140
= × 100 = 11.4% (approx.) ⇒ x = 44 × 1000 = 44,000
140
The correct answer is C. The correct answer is E.

83. Total number of recipients in the 35–44 age group in 86. Raw material cost for jam as a percentage of MRP
2005 9.52
= × 100 = 19.04%
= 56000 + 76000 = 1,32,000 50

268

Book 1.indb 268 30/04/2019 4:48:30 PM


3.0  Quantitative Skills Practice

Raw material cost for potato chips as a percentage 90. Person days of employment = Number of people
of MRP employed × Number of days each person worked
Number of people employed in Y1× 230 = 90
1.25
= × 100 = 12.5% Number of people employed in Y1
10
Raw material cost for packaged juice as a 90
= = 0.391 crore
percentage of MRP 230
58 Number of people employed in Y4 × 231 = 156
= × 100 = 8.53%
68 Number of people employed in Y4
Raw material cost for potato chips and jam as a
156
percentage of MRP = = 0.675 crore
231
10.77
= × 100 = 17.95% Required percentage increase
60
Hence, Jam has the maximum percentage raw 0.675 - 0.391
= × 100 = 72.63%
material cost per MRP. 0.391

The correct answer is A. Alternatively,


Since number of working days is almost same, the
87. Packets of potato chips sold = 5 required percentage is same as percentage increase
in person days of employment generated.
Packets of jams sold = 12
So, required percentage increase
Packets of packaged juice sold = 8
So, required margin 156 - 90
= × 100 ≈ 73%.
= 5 × 0.44 + 12 × 2.18 + 8 × 3.50 90
= 2.2 + 26.16 + 28 = 56.36 The closest option is D.

The correct answer is D. The correct answer is D.

88. The percentage of Jam over MRP 91. The total number of rural households in Y1 = 3.2
crores
5.37
= × 100 = 1.74% The total number of rural households which did not
50
get an employment in Y1
The percentage of potato chips
= 3.2 – 2.1 = 1.1 crores
0.94
= × 100 = 9.4%
10 The total number of rural households in Y4
The percentage of packed juice 120
= 3.2 × = 3.84 crores
8.2 100
= × 100 = 12.06%
68 The total number of rural households which did not
The percentage of potato chips and jam get an employment in Y4
0.94 + 5.37 = 3.84 – 3.80 = 0.04 crores
= × 100 = 10.52%
60 Required ratio = 1.1 : 0.04 = 110 : 4 = 55 : 2
So, the price of packed juice products will come The correct answer is D.
down over MRP by the maximum percentage.
The correct answer is C. 92. Person days of employment = Number of people
employed × Number of days each person worked
89. In the above question the information about the Number of people employed in Y1
retailer’s margin and discount combined only has = 0.39 crores (from previous question)
been given. So, we cannot determine what percent is
retailer’s margin of MRP of packaged juice. Number of people employed in Y2 × 230 = 108

The correct answer is E. Number of people employed in Y2

269

Book 1.indb 269 30/04/2019 4:48:31 PM


NMAT by GMAC™ Official Guide 2019

108 96. Reflex angle for the personal income of the 26 to 35


= = 0.47 crores age group = 130°
230
Reflex angle for the personal income of the above 35
Number of people employed in Y3
years age group = 140°
144 Required ratio = 130 : 140 = 13 : 14
= = 0.63 crores
230
The correct answer is C.
Required average
97. Total income in the year 2007 = Rs. 5400 crore
0.39 + 0.47 + 0.63
= = 0.49 crore
3 Rate of increment = 12% per year
Thus, total income in the year 2009
The correct answer is D.
= 1.12 × 1.12 × 5400 ≈ 6774 crores
93. Growth rate in the different years: The correct answer is A.
Growth rate in the year Y1–Y2
98. The total nos. of users voted
20 - 17
= × 100 = 17.65% = 9539 + 8871 + 20032 + 27759 + 15739 +6582
17
+ 2957 + 1526 + 954 + 1431
Growth rate in the year Y2–Y3
= 95390
36 - 20
= × 100 = 80% The correct answer is E.
20
Growth rate in the year Y3–Y4 99. Total number of users who gave rating 5 or below
40 - 36 = 6,582 + 2,957 + 1,526 + 954 + 1,431
= × 100 = 11.11%
36 = 13,450
Growth rate in the year Y4–Y5 Therefore, required number of users
= 12% of 13,450
46 - 40
= × 100 = 15% 12
40 = × 13450 = 1,614
100
In the above question there is no data given for Y6 so we
cannot determine the growth rate for year Y5 and Y6. The correct answer is B.

The correct answer is B. 100. 29.1% users gave rating 7 and 21% users gave
rating 8, so the percentage of total users who
94. Total income = Rs. 5400 crore gave rating 7 and 8 = 29.1 + 21 = 50.1% = 50 %
Reflex angle for the personal income of the 21 to 25 (approx.)
years age group = 50° Thus, the rating given by almost half of the user is 7
Thus, personal income of age group 21 to 25 years or 8.
50 The correct answer is D.
= × 5400 = 750 crores
360
101. The total number of ratings given
The correct answer is B.
=10 × 10 + 9.3 × 9 + 21 × 8 + 29.1 × 7 + 16.5 ×
95. Reflex angle for the personal income of the above 35 6 + 6.9 × 5 + 3.1 × 4 + 1.6 × 3 + 1 × 2 + 1.5 × 1
years age group = 140° = 100 + 83.7 + 168 + 203.7 + 99 + 34.5 + 12.4
Reflex angle for the personal income of the 21 to 25 + 4.8 + 2 + 1.5
years age group = 50° = 709.6
Required percentage Thus, required average rating
140 709.6
= × 100 = 280% = = 7.096
50 100
The correct answer is B. The correct answer is C.

270

Book 1.indb 270 30/04/2019 4:48:33 PM


3.0  Quantitative Skills Practice

102. 11,500 represents in percentage terms 105. Total number of children who use the laptop = 0.52
11500 × 45,000
× 100 = 46%, which refers to the families
25000 Total number of children who use the laptop less
owning tablets. often
Number of children aged 3 to 18 years who use the 6 52
tablet = 45, 000 × ×
100 100
34 Total number of children who use the e-reader = 0.11
= 45, 000 × = 15,300
100 × 45,000
The correct answer is E. Total number of children who use the e-reader less
often
103. Number of children who use the smartphone weekly 11 12
= 45, 000 × ×
43 29 100 100
= 45, 000 × × = 5,611.5
100 100 Required ratio

Number of children who use the e-reader weekly 6 52 11 12


= 45, 000 × × : 45, 000 × ×
100 100 100 100
11 55
= 45, 000 × × = 2,722.5 = 6 × 52 : 11 × 13
100 100
= 6 × 4 : 11 = 24 : 11
Required difference
Alternatively,
= 5,611.5 − 2,722.5 = 2,889 Required ratio
The correct answer is A.
= 0.52 × 0.06 : 0.11 × 0.13 = 24 : 11
104. From option A, total number of children who use the The correct answer is E.
laptop daily

52 51
= 45, 000 × × = 11,934
100 100
We need not check other options.
The correct answer is A.

271

Book 1.indb 271 30/04/2019 4:48:33 PM


NMAT by GMAC™ Official Guide 2019

6. Data Sufficiency 6. Using statement 1 alone, we can say that the price
of coffee = x + 5, where x is the price of tea.

1. Since we don’t have the total number of people given Using statement 2 alone, we know that the price of
to us, or the ratio of men to women, we cannot coffee = 3x − 5, where x is the price of tea.
arrive at the answer even by combining both the Combining both the statements, we have
statements.
x + 5 = 3x - 5
The correct answer is E. 2x = 10 ⇒ x = 5

2 3 The correct answer is C.


2. If of the students are boys, this means that of
5 5
the students are girls. So the required probability is 7. You don’t need to actually calculate the 7th and
22nd roots of y; doing so will only waste your time.
3
. Statement 1 is sufficient. However, the important thing to note is that you will
5 get a unique value for y from either of the two given
Statement 2 does not help since we don’t know the statements. What that value is does not matter to
total number of students in the class. you. Thus, the answer is D.
The correct answer is A. The correct answer is D.

3. Statement 1: Since a + 2 is even, a is an even 8. You know that in a triangle, the largest angle lies
integer, and therefore a + 1 would be an odd integer; opposite to the longest side. So all we need to figure
SUFFICIENT out is which of the three sides is the longest.
Statement 2: Since a–1is an odd integer, a is an From Statement 1, AC is the longest side, so it is
even integer. Therefore a + 1 would be an odd Sufficient.
integer; SUFFICIENT.
From Statement 2, we still don’t have a value for y.
The correct answer is D. So it is NOT Sufficient.

4. From statement 1, Let Udit’s earning last month be x The correct answer is A.
1 1 9. You can’t do anything with either of the two
Then × x = 1200
4 2 statements because the total number of students is
By solving the above equation, we will get Udit’s not given to you, neither is the ratio of Indian to non-
earnings for the last month; SUFFICIENT. (Remember, Indian students given.
you do not need to try to solve this equation, it will
only waste your time). The correct answer is E.
Statement 2 just tells us the relation between Udit’s
savings and his rent, which does not tell us anything 10. Statement 1 is not sufficient since it does not provide
about his overall earnings. So, the statement 2 is us with the Common difference.
INSUFFICIENT. Statement 2 is Sufficient because it provides us
The correct answer is A. with the common difference (18) and also with one
specific term.
5. From Statement 1, the original amount of water in The correct answer is B.
the pool is irrelevant, since we only have to figure
out the rate of increase. So, Statement 1 is Not 11. The two statements simply give the cost price of the
Sufficient two types of tea. However, from the cost price we
From Statement 2, water is being pumped in at 50 cannot get any idea of the selling price. Thus, both
20 the statements together are NOT SUFFICIENT to
litres per minute and pumped out at = 5 litres answer the question asked.
4
per minute. Thus, the net rate of addition = 50–5 The correct answer is E.
= 45 litres per minute. So Statement 2 ALONE is
sufficient. 12. Statement 1 does not provide us any valuable
The correct answer is B. information and is NOT SUFFICIENT.

272

Book 1.indb 272 30/04/2019 4:48:34 PM


3.0  Quantitative Skills Practice

Statement 2, however, does help us, if we make Note that statement 2 still does not tell us anything
the triangle OAB as shown below and draw a about Ramu. Hence, NOT SUFFICIENT
perpendicular OP onto AB. However, by combining the two statements, we
From statement 2, we can find AOB and hence area can conclude that Ramu’s deal was better. Hence,
of segment. Thus statement 2 is SUFFICIENT. SUFFICIENT

The correct answer is B. The correct answer is C.

13. Individually the statements don’t give us any 17. Let the number be xy.
information as to whether b is a multiple of 6 so NOT Then, the number can be represented as 10x + y
SUFFICIENT. and the number formed by reversing the digits can
Combining the two statements, for any number to be be represented by 10y + x.
a perfect square and a perfect cube, the power has to From statement 1:
be 6 or some multiple of 6. For example, 26 = 64 and D = |(10x + y) – (10y + x)| = 36
64 is a perfect square (82) and a perfect cube (43).
⇒ |9(x – y)| = 36
Thus, both the statements together are SUFFICIENT.
⇒ |x – y| = 4
The correct answer is C.
⇒ x – y = ±4
14. Using statement 1 alone—as coordinates of all the Hence, the question cannot be answered using
vertices are given, so each side and both diagonals statement 1 alone.
can be obtained and using them, area of quadrilateral From statement 2:
can be obtained (by considering it as 2 triangles x + y = 12
and for each triangle 2 sides are adjacent sides
Hence, the question cannot be answered using
of quadrilateral and 3rd side for both is the same
statement 2 alone.
diagonal). Therefore, statement 1 alone is sufficient.
Combining statements 1 and 2:
Using statement 2 alone—the given information is
Clearly, the number could be 48 or 84 and hence, a
inadequate to find the area of the quadrilateral
unique number cannot be determined by combining
The correct answer is A. the statements 1 and 2.
So, both the statements are not sufficient.
15. From statement 1, we can calculate the radius
of M. From this we can get the circumference of The correct answer is E.
M, which will give us the circumference of N. The
circumference of N will give us the radius of N, which 18. From Statement 1:
in turn will give us the area of N. So, SUFFICIENT Using the first statement one can deduce that
Statement 2 does not help because we haven’t been A > D > F such that F is taller than at least one
given an actual figure for the diameter to work with. person, either B or E.
So, NOT SUFFICIENT Hence, the question cannot be answered using
The correct answer is A. statement 1 alone.
From Statement 2:
16. Statement 1 tells us that Ramu sold his items for no Using the second statement one can deduce
profit no loss. But, we do not know anything about that B > E.
Somu. So, NOT SUFFICIENT
Hence, the question cannot be answered using
Statement 2 tells us that Somu sold his items for a statement 2 alone.
slight loss. To understand this, tick any number as Combining statements 1 and 2:
the selling price for each of Somu’s items. Let’s say,
Combining both the statements we can deduce that
Somu sold each item for Rs. 100 and let q = 10%
F is greater than both B and E and, hence, E is the
Then, Somu’s total SP = 200 smallest.
CP for 1 item sold at 10% profit = 91 approx So, the question can be answered using both the
CP for 2nd item sold at 10% loss = 111 approx statements.
Thus total CP = 91 + 111 = 202 The correct answer is C.
Thus loss = 202 - 200 = 2

273

Book 1.indb 273 30/04/2019 4:48:34 PM


NMAT by GMAC™ Official Guide 2019

19. Divisibility rule of 9 says that a number is divisible Combining both the statements also we cannot find
by 9 if sum of digits is divisible by 9. So, as per the definite answer.
statement 1, the number abc is a multiple of 9 and
The correct answer is E.
so sum of digits is also divisible by 9.
So, a + b + c is divisible by 9. 23. Using statement 1
Hence, statement 1 alone is sufficient. Let the total capacity of the tank be V cm3.
Statement 2 is not sufficient since choosing Fraction of the total volume of the tank filled in 2
a = 0 = b and c = 9 makes it valid and (a + b + c) hours when both the pipes are opened together
is 9 but choosing a = 4 = b and c = 2 also makes
it valid with (a + b + c) equal to 10, which is not  1 1
divisible by 9. = 2  + 
 6 8
 1 1
The correct answer is A. So, volume of water in the tank = 2  +  V
 6 8
20. Each statement independently gives the answer. Since, the value of ‘V’ is not known, hence statement
1 alone is not sufficient to answer the question.
9 gives + 3 and not –3. By convention we always
take a positive root only. Using statement 2
The second statement gives the value of X as 0. Obviously, this statement alone is not sufficient to
answer the question.
The correct answer is C.
Combining both the statement together, we get:
21. Statement 1 is true for parallelogram, rectangle as
 1 1
well as a rhombus. Volume of water in the tank = 2  +  × 100
 6 8
So, statement 1 alone is not sufficient. The correct answer is C.
From statement 2, sum of AB + CD is equal to sum
of AD + BC. 24. Using statement 1
Since AB = CD and AD = BC (in a parallelogram), Nothing can be said about the schedule of Chemistry.
we can say that AB = BC = CD = DA, and so the Using statement 2
parallelogram is a rhombus.
Nothing can be said about the schedule of Chemistry.
Hence, statement 2 alone is sufficient to determine
Combining both the statement together, we get:
the nature of ABCD.
The correct answer is B. 14th 15th 16th 17th

22. Using statement 1 Monday Tuesday Wednesday Thursday

a2 + b2 + c2 = ab + bc + ca Arithmetic Dermatology Biology Chemistry


or a2 + b2 + c2 – ab – bc – ca = 0
Hence, by using both the statements together, we
1
or [(a – b)2 + (b – c)2 + (c – a)2] = 0 can see that Chemistry was scheduled on Thursday.
2
⇒a=b=c The correct answer is C.
Therefore, we cannot say that a = b = c = 1
3m 3m
Hence, statement 1 alone is not sufficient to answer
the question. 3m
a
Using statement 2
From equation a2 + b2 = 2c2, we cannot say that a, 25.
b and c are definitely equal to 1. They could be equal a a
to 2, 3, 4 and so on.
Hence statement 2 alone is also not sufficient to a
answer the question. 3m

274

Book 1.indb 274 30/04/2019 4:48:34 PM


3.0  Quantitative Skills Practice

Total cost of flooring the square room = Area of using Statement 1, and we can convert the units to
square room x Cost of flooring per square metre get the answer in litres.
Let side of the square be ‘a’ metre. Then, Using Statement 2 alone, we cannot find the volume
Area of square room = a m 2 2 of water.
Cost of flooring per square metre = Rs. 25 The correct answer is A.
2
Total cost of flooring the square room = Rs. 25 a
29. Samir’s income = Rs. 9360
Statement (1) is INSUFFICIENT.
Statement (2) is INSUFFICIENT. From statement 1, Rahul’s income can be Rs. 7200
From Statements (1) and (2) together, we get: or Rs. 11520. It is INSUFFICIENT.
2 2
a = (a + 6) – Area of Verandah From statement 2, Sameer’s income
⇒ a2 = (a + 6)2 – 432 103
⇒ a = 33 m = × Rahul’s income
100
Now, total cost of flooring the room can be
100
calculated. Rahul’s income = 9360 × = Rs. 9087.38
103
The correct answer is C. (approx.). It is SUFFICIENT.

26. Statement (1) alone is not sufficient, as we do not The correct answer is B.
know the depreciation rate.
30. Total number of balls in box = 62
Statement (2) alone is not sufficient, since we don’t
From statement (1), number of black balls = 37.
know the purchase value.
We don’t know the number of balls of other colours.
Combining both the statements (1) and (2) together,
we have It is INSUFFICIENT.
4 From statement (2), number of blue balls = 25.
 15 
Depreciated value =  1 −  × 60000 We don’t know the number of balls of other colours.
 100 
It is INSUFFICIENT.
We need not find the value of the above expression. Combining statements (1) and (2) together, total
number of black and blue balls in the box
The correct answer is C. = 37 + 25 = 62.

27. From statement (1),we get: It means number of white balls in the box = 0.

AB + BA = 18L The correct answer is C.

So, LHS = (10A + B) + (10B + A) = 11(A + B), which 31. From Statement (1), it is not clear which date is
is a multiple of 11. Therefore, 18L should also be a Sunday of the month. So, we cannot determine which
multiple of 11. So, L must be 7. day is it on the 19th of the given month.
Now, 11(A + B) = 187 or A + B = 17. The only
possibilities of (A, B) are: (8, 9) or (9, 8). In any case, From Statement (2), we have
the product of A and B will be 72. Last Tuesday of the month is 25th.
Therefore, statement (1) alone is sufficient to answer Tuesday = 25th
the above question. Monday = 24th
Statement (2) alone is not sufficient to answer the Sunday = 23rd
question.
Saturday = 22nd
The correct answer is A. Friday = 21st

Thursday = 20th
28. Using Statement 1: The length of the tank can be
found by applying the below formula: Wednesday = 19th
Thus, Statement (2) ALONE is sufficient to determine
length of diagonal = l2 + b2 + h2 .The volume of the the day is it on the 19th of a given month.
tank, in cubic feet, can therefore be calculated by The correct answer is B.

275

Book 1.indb 275 30/04/2019 4:48:35 PM


NMAT by GMAC™ Official Guide 2019

32. From Statement (1), we have From Statement (2), we have


x + y = 10 x 64 = 8128
From the above linear equation, it cannot be
( )
64
⇒ x 64 = 82
determined which variable is greater.
⇒ x = 64
So, Statement (1) ALONE is not sufficient.
Similarly, Statement (2) ALONE also is not sufficient. So, Statement (2) ALONE is sufficient to answer the
Also, we cannot get the unique answer on combining question.
Statements (1) and (2) The correct answer is D.
The correct answer is E.
36. From Statement (1), we have
33. Given 5b = 20 ⇒ b = 4 Mr Kashyap’s salary = Price of (50 × 4) kg of rice =
From Statement (1), we have Price of 200 kg of rice
⇒ a2 + b = 28 So, Statement (1) ALONE is not sufficient.
2
⇒ a + 4 = 28 Statement (2) ALONE is also not sufficient as there
2 are two unknown variables – cost of the rice and Mr
⇒ a = 24
Kashyap’s salary.
So, we can find the value of a2 + 2b.
Combining both the Statements, we have
Thus, Statement (1) ALONE is sufficient.
Let the current price of rice be $ x/kg.
From Statement (2), we have
Mr Kashyap’s salary = 200 × $ x = $ 200x
2a2 = 12b
Reduced price of rice
⇒ a2 = 6b
20 4x
⇒ a2 = 6 × 4 =$x-$x× =$ /kg
100 5
So, we can find the value of a2 + 2b.
According to the question,
Thus, Statement (2) ALONE is sufficient.
4x $200x
The correct answer is D. 50 × $ = + $100
5 10
34. We are given that x + y = 150, where x and y are the We can find the value of x from the above equation
number of girls and boys respectively. and hence, Mr Kashyap’s salary can be determined.
From Statement (1), we have So, both statements together are sufficient but
neither statement alone is sufficient.
0.5x + 0.25y = 49
From the above two equations, we can get the The correct answer is C.
number of girls.
37. From Statement (1), we can determine 3 successive
So, Statement (1) ALONE is sufficient to answer the
prime number which are in arithmetic progression
question.
as 3, 5 and 7. So, their sum can be calculated.
From Statement (2), we have
Thus, Statement (1) ALONE is sufficient.
0.3x + 0.3y = 45, which is not independent equation
From statement (2), the three successive prime
to the given equation.
numbers starting from 5 are 5, 7 and 11. So, their
So, Statement (2) ALONE is not sufficient. sum can be calculated.
The correct answer is A. Thus, Statement (2) ALONE is sufficient.
The correct answer is D.
35. From Statement (1), we have
x = 4096 = 64 38. From Statement (1), we have
So, Statement (1) ALONE is sufficient to answer the
logz x > logz x 2
question.

276

Book 1.indb 276 30/04/2019 4:48:36 PM


3.0  Quantitative Skills Practice

Now, log of a greater number is less than that of a From Statement (1), we have
smaller number only when the base lies between 0 Speed of boat in upstream,
and 1. Now x is given as an integer which means that
x2 is greater than x. 140
Sb – S w = = 20
Thus, Statement (1) ALONE is sufficient. 7
From Statement (2), we have We cannot find the speed of the boat in still water
from the above equation.
z2 < z
Thus, Statement (1) ALONE is not sufficient.
⇒ z2 – z < 0
From Statement (2), we have
⇒ z(z – 1) < 0
Speed of boat in downstream,
⇒0< z <1
140
Thus, Statement (2) ALONE is sufficient. Sb + S w = = 28
5
The correct answer is D. We cannot find the speed of the boat in still water
from the above equation.
39. From Statement (1) or (2) ALONE or after combining
Thus, Statement (2) ALONE is not sufficient.
both the statements also we cannot get original value
of x and y and hence we cannot get any answer for Combining Statements (1) and (2), we can get the
the given question. value of Sb and hence the value of Sw.
Thus, both statements together are sufficient, but
Thus, Statements (1) and (2) together are not
neither statement ALONE is sufficient.
sufficient.
The correct answer is C.
The correct answer is E.
42. Given that the cost price of two dresses is Rs. 1350.
40. From Statement (1),
From Statement (1), we have
The number of factors of ‘n’ is two, gives that ‘n’ is a
prime number but there are plenty of prime numbers. Price of first dress 7.5% 5
= =
Thus, Statement (1) ALONE is not sufficient to answer Price of second dress 6% 4
the question. Therefore, the cost price of first dress
From Statement (2),
5
That ‘n’ is a natural number between 5743 and 5779, = 1350 × = Rs. 750
9
gives a list of 35 numbers.
Thus, Statement (2) ALONE is also not sufficient to So, the price of second dress =1350 – 750 = Rs. 600
answer the question. Thus, Statement (1) ALONE is sufficient.
Using both statements we have n = 5749, which is
From Statement (2),
the only prime number between 5743 and 5779.
It is not clear which dress is being talked about, first
The correct answer is C. one or second one.
Thus, Statement (2) ALONE is not sufficient.
41. Let Sb be the speed of boat in still water and Sw be the
speed of stream. The correct answer is A.

277

Book 1.indb 277 30/04/2019 4:48:36 PM


4.0  Language Skills Review

Book 1.indb 278 30/04/2019 4:48:36 PM


4.0  Language Skills Review 4.0 Language Skills Review

4.0 Language Skills Review


This section provides a comprehensive review of the key language concepts assessed in the NMAT by
GMACTM exam. These concepts are elucidated with the help of several examples. It is recommended that
you review the concepts and practice the different examples to build proficiencies.
The Language Skills section of the NMAT by GMAC™ exam will test you on the following three broad
areas:
1. Vocabulary
2. Grammar
3. Comprehension
The question types that you can expect to see within each of these areas are:

Vocabulary
1.
Synonyms
2.
Antonyms
3.
Analogies
4.
Fill in the Blanks
5.
Cloze Test

Grammar
1. Identify the Error
2. Choose the Correct Preposition

Comprehension
1. Reading Comprehension
2. Parajumbles
The next few sections will explain some of the important strategies to approach each of these question types
and include some practice questions at the end.

279

Book 1.indb 279 30/04/2019 4:48:37 PM


NMAT by GMAC™ Official Guide 2019

4.1 Top Tips to Prepare for Language Skills


1. You will get 32 questions in the Language Skills section on the NMAT by GMAC™ exam, that you
will have to attempt in 22 minutes.
2. This section will test you on all three areas of English—vocabulary, grammar and comprehension.
3. Manage your time carefully. You have an average of 40 seconds per question, but you will need some
more time while attempting Reading Comprehension questions. So, your target should be 30 seconds
each for the rest of the questions, which will give you around 10–11 minutes to attempt the 7–8
Reading Comprehension questions that will appear on the test.
4. Who does it make perfect?—ensure that you practice enough test questions to get your timing correct.
This is particularly important for grammar questions as you will need to remember the important
grammar rules.
5. More importantly, maintain an error log of your practice questions and go through it from time to
time to identify and strengthen your weak areas.
6. To improve both your comprehension and vocabulary, start reading well-written books, or a good
newspaper, every day. Pay special attention to the Editorial section of newspapers, which has the most
enriching vocabulary.
7. Maintain a vocabulary diary. Write down any new word you come across while reading the newspaper,
or attempting practice questions in this diary, and go through it every day.
8. Try to learn words and not just memorise them. Simply put, you should be aware of the usage of a
word and not just its dictionary meaning.
9. Prefer quality over quantity. Instead of memorising 100 words haphazardly, learn the meaning of 20
words properly. Similarly, instead of practicing a huge number of grammar questions, focus on a small
number of questions and try to understand why you are making the mistakes that you are making and
how to correct them in subsequent practice sessions.
10. Pay close attention to the strategies given in the next few sections to help you make intelligent guesses
about meaning and usage of words. With these strategies, you do not need to know the meaning of
every word in the dictionary.
11. Make no assumptions and do not use any real-world knowledge, especially on Reading
Comprehension questions.

280

Book 1.indb 280 30/04/2019 4:48:37 PM


Book 1.indb 281 30/04/2019 4:48:37 PM
4.2 Vocabulary

Book 1.indb 282 30/04/2019 4:48:37 PM


4.0  Language Skills Review  4.2  Vocabulary

4.3 What is Measured?


The vocabulary section will test you on your understanding and awareness of word meanings, both within
and without context.
Vocabulary-in-context questions will also indirectly test your comprehension skills because you will be able
to correctly answer the question only if you have understood its intended meaning.
You will also be tested on your knowledge of common word roots, deceptive words, etc.

4.4 Overall Test Taking Strategies


• Do not blindly focus on the meaning of words; instead, pay attention to the overall meaning being
conveyed by the sentence.
• Try to use your knowledge of common word roots and word charge to eliminate options.
• Try to make a prediction, especially on fill in the blank type questions, before you look at the options.
Use keywords and connectors to do so.

The next few sections will provide you with in-depth strategies for approaching each topic.

283

Book 1.indb 283 30/04/2019 4:48:37 PM


NMAT by GMAC™ Official Guide 2019

1 Introduction
Vocabulary means the words used in a language. For the purpose of the NMAT by GMAC™ it is words
used in the English language that we will be concerned with. You can expect around 10-15 vocabulary
based questions on the NMAT by GMAC™. Vocabulary will be tested at two levels – simple vocabulary
and vocabulary within context. Simple vocabulary questions are your typical synonym and antonym
questions in which you are asked for the meanings of random words with no context provided for
that word.
As against this, vocabulary-in-context questions will ask you for the meaning of words in a particular
context, such as when they are used in a Reading Comprehension passage or in the form of fill in the blank
questions. For these questions, just knowing the meaning of the word is not enough. You also must be able
to understand the meaning of the sentence or passage in which these words have been used to arrive at the
answer correctly.

Vocabulary Concepts
In the subsequent pages, we will review the fundamentals of the various vocabulary concepts. While we
have made an attempt to provide detailed lists and explanations, please note that they are not exhaustive.
Their purpose is to serve as guideposts in your preparation and provide a framework for it. Kindly use
the information provided in the following pages to build a baseline and then work on enhancing your
knowledge.

Word roots
A root word is the most basic part of a word. It is what remains after the various affixes have been removed.
Given below is a list of some commonly used word roots. You are advised to go through the list and use it
to build your vocabulary.

Root, Prefix, or suffix Meaning Examples


a- without amoral, amorphous, asexual
ambi- on both sides ambidextrous, ambivalent
ante- before or in front antecedent, antedate
anti- against antipathy, antisocial
aqu/aqua- water aquatic, aqueous, aquarium, aqueduct
bene- good benefit, benediction, benevolent
bi- two bifurcate, biannual, bisect
bio- life biology, biography, biome
cede/ceed go or yield precede, exceed, recede
circum- around circumscribe, circumnavigate, circumvent
contra- against or opposite contradiction, contraception, controversy
cycl circle bicycle, cyclical
de- reduce or remove deescalate, defenestrate, decelerate
di-, dis- apart or away digress, disappear, diverge
dict speak or say edict, dictation, dictator, prediction, contradiction

284

Book 1.indb 284 30/04/2019 4:48:37 PM


4.0  Language Skills Review  4.2  Vocabulary

Root, Prefix, or suffix Meaning Examples


dox belief orthodox, paradox, heterodox
du-, duo- two dual, duology, duochrome
em-, en- into, in embrace, enclose, encircle
esce becoming coalesce, adolescence, obsolescent, tumescent
ex- out or way exit, exhale, extirpate, exile
extra-, extro- beyond or outside extraordinary, extraterrestrial
fid/e faith bonafide, fidelity, confide
fore before, previously, earlier forestall, before, forebear, forebode, forecast
gram writing, letters diagram, grammar, epigram, telegram
graph writing, recording stenography, autograph, graphics
hetero- different heterosexual, heterozygous, heterogeneous, heterodox
homo- same homogenous, homosexual, homologous
hyper- excessive hyperactive, hyperbole
hypo- under, below hypothermia, hypocrite, hypoglycemic
inter- between intercede, interlude
intra-/intro- inside, within introvert, intramural, intravenous
junct joining juncture, conjunction, disjunction
-less without listless, aimless, heartless
-logy the study of biology, geology, psychology
mal, male bad, evil malediction, malice
mis- bad or incorrect misprint, misbehave, misstep
-ness state of being likeness, greatness
non- not, without nonfiction, nonresident
ob- against or before obdurate, obfuscate
omni- all, everything omnipotent, omniscient, omnivorous
pedi, pede foot pedestrian, pedicure
phil love or affinity bibliophile, philanthropy
pre- before or earlier pretest, preamble
pro- before or forward proceed, prologue
re- again, backwards reaction, rebound, reuse
sub- under or lower submarine, subprime
temp time temporal, contemporary, temporarily
tort twist tortuous, contortion
trans- across or beyond transnational, transit
un- not or opposite unimpressive, unwanted, unwarranted

285

Book 1.indb 285 30/04/2019 4:48:37 PM


NMAT by GMAC™ Official Guide 2019

Taking words apart

Word Prefix Root Suffix Meaning


Concurrence con: with curr: run ence: act of Act of existing or occurring at the same time
Exaggerate ex: out ag: to do, act — To overstate or overemphasize
ir: not vers: turning ible: able to Irrevocable, that which cannot be undone
Irreversible
re: again
Malediction male: bad dict: to say — Damnation, curse or evil words
Precursor pre: before curs: to run or: one who Forerunner, antecedent
Unity uni: one — ty: state of State of being united
un: not ten: holding able: able to A point of view that cannot be argued or
Untenable
defended

Prefixes and Suffixes


Prefixes
A prefix refers to an alphabet or a group of alphabets that comes at the start of a word and assists in
indicating or modifying its meaning. For example: the word 'prefix' starts with the prefix pre-, which means
'before'.
An understanding of the different prefixes can prove quite beneficial in order to understand the meaning of
words and build associations, However, at times a prefix can indicate multiple meanings. For example, the
prefix im- can mean ‘not’ or ‘into’.

Suffixes
A suffix refers to an alphabet or a group of alphabets that comes at the end of a word to form a new word,
or alter its grammatical function.
Suffixes also assist in delineating the meaning of a word.

Suffix Meaning Example


VERB SUFFIXES
-ate become mediate, collaborate, create
-en become sharpen, strengthen, loosen
-ify, -fy make or become justify, simplify, magnify, satisfy
-ise, -ize become publicise, synthesise, hypnotise
ADJECTIVE SUFFIXES
-able, -ible capable of being edible, fallible, incredible, audible
-al having the form or character of fiscal, thermal, herbal, colonial
-esque in a manner of or resembling picturesque, burlesque, grotesque
-ful notable for handful, playful, hopeful, skilful
-ic, -ical having the form or character of psychological, hypocritical, methodical,
nonsensical, musical
-ious, -ous characterised by pious, jealous, religious, ridiculous

286

Book 1.indb 286 30/04/2019 4:48:37 PM


4.0  Language Skills Review  4.2  Vocabulary

Suffix Meaning Example


-ish having the quality of squeamish, sheepish, childish
-ive having the nature of inquisitive, informative, attentive
-less without meaningless, hopeless, homeless
NOUN SUFFIXES
-acy state or quality democracy, accuracy, lunacy
-al the action or process of remedial, denial, trial, criminal
-ance, -ence state or quality of nuisance, ambience, tolerance
-dom place or state of being freedom, stardom, boredom
-er, -or person or object that does a specified reader, creator, interpreter, inventor,
action collaborator, teacher
-ism doctrine, belief Judaism, scepticism, escapism
-ist person or object that does a specified Geologist, protagonist, sexist, scientist,
action theorist, communist
-ity, -ty quality of extremity, validity, enormity
-ment condition enchantment, argument
-ness state of being heaviness, highness, sickness
-ship position held friendship, hardship, internship
ADVERB SUFFIXES
-ly related to or quality softly, slowly, happily, crazily, madly
-ward, -wards direction towards, afterwards, backwards, inward
-wise in relation to otherwise, likewise, clockwise

These affixes can have a dramatic impact on the meanings of words. Knowledge of various prefixes and
suffixes can help you build understanding of words, their meanings and usages.
Keep in mind that even though affixes are required for forming words, they cannot stand alone in a
sentence as they are not words in their own rights.

Types of Questions
1. Synonyms and Antonyms
2. Contextual Usage
3. Inappropriate Usage
4. Fill in the Blanks
5. Cloze Test
6. Analogies
7. Idioms and Phrasal Verbs

Vocabulary Building Techniques


Given below are a few tips to help improve your vocabulary. Go through them and use the ones that prove
beneficial for you. They will not only help you identify the meanings of new words but also aid in enhancing
their retention.

287

Book 1.indb 287 30/04/2019 4:48:37 PM


NMAT by GMAC™ Official Guide 2019

Tips for Improving Vocabulary


Context
One can often identify at least part of a word’s meaning from the way it is used in the sentence. So, when
you come across an unfamiliar word while reading, rather than first looking it up in the dictionary, try to
figure out its meaning from the words around it. The author often provides clues to help you unlock the
general meaning.
Use the Dictionary
In order to find all the meanings of a particular word and understand its evolution, you should look it up in
the dictionary. The dictionary is a reliable source of definitions and correct spellings, pronunciation, parts of
speech, and derivations.
Structure of the word
Knowledge of the parts of words—prefixes, roots, suffixes—will help you understand and unlock the
meanings of whole groups of words. As explained previously, a prefix is the part that is attached to the
beginning of a word. The root is the basic part of a word; most of our root words come from Latin and
Greek. A suffix is the part that is attached to the end of a word.
Read across subject areas
There is no substitute for extensive reading. The more you expose yourself to new words, the more words
you will learn.
Make up as many associations and connections as possible. Say the word aloud to activate your auditory
memory. Relate the word to words you already know. Start with a simple word—example Happy—and
associate it with progressively difficult words—Cheery, Merry, Jovial, Jocular, Content, Radiant, Blithe,
Exuberant, Ebullient, Propitious, etc.
Play with words. Play Scrabble, Boggle, and do crossword puzzles. These and other word games are
available on the computer, so you are not dependent on a partner to play.
Use vocabulary lists or vocabulary cue cards. Flash cards are a great way to improve your vocabulary as
they organise and structuralise material in a convenient and easy to learn format.
Take vocabulary tests. This book consists of comprehensive practice tests that will help you assess and
enhance your verbal ability. Practice these diligently and keep adding to your skills.

2 Synonyms and Antonyms


The root ‘onym’ means name and the root ‘syn’ means same; so, synonym literally means ‘same name’.
Similarly, the root ‘anti’ means opposite; so, antonym means ‘opposite name’.
In synonym questions you need to select a word from the given options that is closest in meaning to the
word given in the question stem, and in antonym questions you need to select a word from the given
options that is opposite in meaning to the word given in the question stem.

Synonyms
As stated above, synonyms are words that have the same or nearly the same meanings. Questions on
synonyms can be easily solved if one’s vocabulary is strong. The elimination method may also come handy,
where inappropriate options can be deleted to reach the answer.

288

Book 1.indb 288 30/04/2019 4:48:37 PM


4.0  Language Skills Review  4.2  Vocabulary

Antonyms
On the other hand, antonyms are words that have opposite meanings. Similar to questions on synonyms,
these questions can also be solved using elimination method.

Different Types of Antonyms


Antonyms fall in three categories, namely, Relational Antonyms, Graded Antonyms, and Complementary
Antonyms.
Relational Antonyms: These are the sets of word pairs which are responsible for showing the relationship
between two opposites such as there can’t be a child without a parent or it’s either all or nothing. Relational
Antonyms wouldn’t exist without the other. Let’s understand this with the help of the following examples,
• Front: Back
• Uncle: Aunt
• Hello: Goodbye
Graded Antonyms: These are the sets of word pairs which are responsible for showing variation between
the two opposites. For example, little and big are antonyms but you will observe a lot of changes before you
get to the opposite meaning. Let’s understand this with the help of following examples, Huge, Big, little,
bulky, full-size, slight, petite etc., Some other examples are as follows,
• Sad: Happy
• Healthy: Sick
• Smart: Stupid
Complimentary Antonyms: These are the sets of word pairs that have no degree of relation. There is only
the possibility of two complete opposites. For example,
• Dead: Alive
• Male- female
• On: Off
Thus, while using antonyms; always try to keep in mind the context behind the word for which you are
looking to use antonyms.

Tips for Solving Questions On Synonyms And Antonyms


Use your knowledge of Word ‘charge’ to narrow down your choices
Words can have a positive charge (a positive meaning), a negative charge (a negative meaning) or a neutral
charge (neutral meaning). If you can identify whether your desired answer should have a negative or a
positive charge, you can narrow down your choices.
For example, let’s say the question asks you to select the synonym of UNRULY from the following options:
(A) Faithful
(B) Ethical
(C) Perseverant
(D) Disobedient
(E) Depressing
Now, you may not be aware of the exact meaning of UNRULY but you may have heard of it being used
somewhere in the negative sense, such as in newspapers. For example, you may remember this headline

289

Book 1.indb 289 30/04/2019 4:48:37 PM


NMAT by GMAC™ Official Guide 2019

from a newspaper that you had read sometime in the past ‘The police used tear gas to control the unruly
mob’. So, then you know that ‘unruly’ is a negative word, which means you also know that the synonym of
‘unruly’ will also be a negative word. With this knowledge, you can immediately eliminate options A, B, and
C because they are all positive words. So, you now have a 50% chance of getting the answer correct because
you have managed to come down to two options.
The correct answer, by the way, is (D) because ‘Unruly’ means ‘disobedient’ or ‘difficult to control’.
Let us take another example:
Select the synonym of FRACTIOUS from the following options:
(A) Obedient
(B) Loyal
(C) Perseverant
(D) Unruly
(E) Virulent
Now, you may not be aware of the exact meaning of FRACTIOUS, but you may have heard of it being
used somewhere in the negative sense. Then, you know that the synonym will also be a negative word, in
which case you can immediately eliminate options (A), (B) and (C) because they are all positive words. So,
you now have a 50% chance of getting the answer correct because you have managed to come down to two
options.
The correct answer here is (D) because fractious means unruly.

Important Learning:  In order to become good at identifying word charge, start reading the newspaper
and some current affairs magazines. Even if you won’t remember all the words you come across, your
subconscious mind will remember whether they carry a positive, negative, or neutral connotation.

Use your knowledge of Word Roots to eliminate options


You would have seen earlier that we broke up the words synonym and antonym into their respective roots
to explain their meaning. If you are aware of some common roots, this can at least help you eliminate some
of the options. For example, let us say the question asks you to select the antonym of BENIGN from the
following options:
(A) Syncretism
(B) Favourable
(C) Malevolent
(D) Acrid
(E) Verbose
Now, even if you do not know the meaning of ‘benign’ but are aware that its root ‘bene’ means good
(think beneficial, benevolent, etc.), then you immediately know that the antonym will be a negative word
signifying harmful or something along those lines.
If you are aware that the root ‘mal ’ means something negative or harmful (think malignant, malnutrition,
etc.), then you can immediately select (C) as the correct answer, even if you do not know the meaning of
malevolent. At least, you can definitely eliminate option (B), which is more of a synonym of benign. You
can also eliminate option (A) because ‘syn’ means same, and it is extremely unlikely that something with

290

Book 1.indb 290 30/04/2019 4:48:39 PM


4.0  Language Skills Review  4.2  Vocabulary

same will mean harmful. So, the knowledge of root words can help you eliminate options or even take you
to the correct answer.
Let us consider another example: Let’s say that in a particular question you have made the prediction
that your answer should be a positive word and you are stuck between the two words – Malediction and
Benediction – both of whose meanings you do not know.
How do you decide which word to go with then?
Word roots can come to your rescue here because you may have heard of words such as malnutrition or
malnourishment, which should tell you that ‘mal’ has a negative connotation. Similarly, you may have heard
of words such as beneficial or benevolent, which should tell you that ‘bene’ has a positive connotation. Thus,
if you need to pick the positive word from amongst the two, you should pick ‘Benediction’ (which means
‘blessing’).

Identify a relation between some of the options


Let us say the question asks you to select the antonym of REFULGENT from the following options:
(A) Radiant
(B) Distant
(C) Dull
(D) Glowing
(E) Noisy

Now, most likely you would not know the meaning of REFULGENT. However, you will notice that two
of the options—radiant and glowing—are in fact synonyms. Then, there is no way that these two can be
your answer because they mean the same thing and you obviously cannot have two answers to the question.
Thus, even without knowing the meaning or charge of the word in question, you have managed to eliminate
two options.
The correct answer, is (C) because refulgent means shiny or glowing, so dull is the antonym of refulgent.
So, the lesson for you is that you can get to the correct answer even if you don’t know the meaning of every
given word. However, it definitely helps if you have a good vocabulary so go through the word list given at
the end of this section and try to remember as many of those words as you can.
Let us consider a few examples:
SYNONYMS
Directions for examples 1-2: Select the word or phrase that is nearest in meaning to the word given in the
question stem.
Example 1


SERENDIPITY
(A) Pity
(B) Sympathy
(C) Good luck
(D) Sarcasm
(E) Reclusive

291

Book 1.indb 291 30/04/2019 4:48:39 PM


NMAT by GMAC™ Official Guide 2019

Solution
Serendipity means good luck or good fortune. Thus, (C) is the best answer.
Sarcasm—irony; taunt
Reclusive—someone who stays away from people or society
The correct answer is C.

Example 2


PHILANTHROPY
(A) Charity
(B) Philander
(C) Flounder
(D) Misanthrope
(E) Anathema

Solution
Philanthropy means charity. Thus, (A) is the best answer.
Philander—to flirt
Flounder—to struggle
Misanthrope—someone who hates humans
Anathema—a curse
The correct answer is A.

ANTONYMS
Directions for examples 1-2: Select the word or phrase that is nearly the opposite in meaning to the word
given in the question stem.
Example 1

RESTIVE
(A) Craven
(B) Anxious
(C) Calm
(D) Vigorous
(E) Agile

Solution
Restive sounds like rested, but it actually means anxious or uneasy. The best answer, therefore, is calm.
Craven—cowardly
Agile—flexible
The correct answer is C.

292

Book 1.indb 292 30/04/2019 4:48:39 PM


4.0  Language Skills Review  4.2  Vocabulary

Example 2


PARTIAL
(A) Disinterested
(B) Biased
(C) Apprehensive
(D) Naive
(E) Recondite

Solution
Partial means biased. Disinterested means unbiased and should be the best answer.
Apprehensive—cautious
Naive—innocent
Recondite—difficult to understand
The correct answer is A.

The English language is a dynamic and evolving language. Hence, it is imperative for any student of the
language to not only understand the literal meaning of a word, but also to form associations and understand
contextual implications. A question on identifying synonyms and antonyms helps the testing authorities
do just that; therefore, it is recommended, that you work with the list provided below as an exercise sheet-
understand the meanings provided and then add your own words to the list. This shall help you enhance
vocabulary as well as increase retention of new words.

Synonyms and Antonyms that Appear Frequently on NMAT by GMAC™


S.No Word Synonym Antonym
1. Abandon Discard, vacate, renounce, relinquish, Retain, maintain, uphold, stay
abnegate, forsake
2. Accord Agree, grant Withhold, remove, disagree
3. Adversity Difficulty, misfortune Fortune, luck, serendipity
4. Affluent Plentiful, rich Poor, impoverished
5. Aggravate Annoy, infuriate Alleviate, appease, mollify
6. Amenable Agreeable, favourable Uncooperative, resistant
7. Anguish Distress, sorrow Happiness, contentment
8. Apathetic Dispirited, lifeless Enthusiastic, eager, passionate
9. Arrogant Disdainful, imperious Modest, unpretentious
10. Astonish Confound, overwhelm Explain, describe
11. Atrocious Appalling, detestable Admirable, superb
12. Augment Add, enlarge Decrease, hinder
13. Awkward Graceless, inept Graceful, amenable
14. Baffle Confuse, deceive Enlighten, comprehend
15. Banal Common, plain Original, indigenous

293

Book 1.indb 293 30/04/2019 4:48:39 PM


NMAT by GMAC™ Official Guide 2019

S.No Word Synonym Antonym


16. Barren Desolate, sterile Fertile, fruitful, stimulating
17. Berate Criticise, disapprove Praise, applaud
18. Betray Deceive, fool Loyal, conceal
19. Bias Inclination, predisposition Impartial, fair, unbiased
20. Bitter Acrid, sour Warm, balmy, sweet, amicable
21. Bliss Happiness, joy Misery, affliction, anguish
22. Bluff Boast, feign Diplomatic, evasive
23. Brief Concise, short Lengthy, extensive
24. Brisk Fast, swift Sluggish, slow
25. Candid Honest, truthful Secretive, guarded
26. Caricature Cartoon, imitation Authentic, unadulterated
27. Casual Informal, natural Formal, thorough
28. Cease Desist, stop Begin, continue
29. Chaotic Disordered, messy Tidy, well-ordered
30. Cherish Esteem, love Neglect, abandon
31. Circumvent Avoid, go around Embrace, confront
32. Commemorate Celebrate, honour Disgrace, dishonour
33. Compensate Balance, recompense Penalise, deprive
34. Competent Able, capable Inadequate, unfit
35. Conceive Design, plan Misconstrue, impromptu
36. Contradict Deny, oppose Confirm, corroborate
37. Courteous Polite, well-mannered Rude, Offensive
38. Craving Desire, longing Animus, abhorrence, antipathy
39. Credulous Confident, trustful Worldly, suspicious
40. Decent Honourable, pure Improper, indecent
41. Designate Name, select Untitled, incognito
42. Detain Hold, keep Release, liberate
43. Disclose Announce, reveal Conceal, hide
44. Dogma Belief, view Open-mindedness, liberal
45. Durable Constant, lasting Perishable, frail
46. Dwindle Abate, diminish Increase, flourish
47. Eager Earnest, keen Uninterested, apathetic
48. Eccentric Abnormal, idiosyncratic Ordinary, conventional
49. Emanate Arise, radiate Withdraw, absorb
50. Embezzle Purloin, steal Compensate, return

294

Book 1.indb 294 30/04/2019 4:48:39 PM


4.0  Language Skills Review  4.2  Vocabulary

S.No Word Synonym Antonym


51. Eminent Distinguished, prominent Unimportant, unknown
52. Endure Last, persist Fade, short-lived
53. Exhaust Deplete, empty Invigorate, refresh
54. Exhilarated Cheerful, zestful Depressing, boring
55. Explicit Definite, specific Vague, implicit
56. Fastidious Exacting, particular Easy-going, sloppy
57. Federation Alliance, band Authoritarianism, despotism
58. Feeble Helpless, infirm Strong, brave
59. Fervour Intensity, passion Apathy, unconcern
60. Feud Argument, dispute Accord, comradeship
61. Filth Dirt, squalor Clean, fresh
62. Flatter Compliment, praise Offend, insult
63. Fleet Nimble, swift Dilatory, sluggish, tardy
64. Frivolous Inconsequential, trivial Sensible, serious, important
65. Frugal Prudent, saving Extravagant, lavish
66. Genuine Actual, real Fake, bogus
67. Goad Provoke, badger Dissuade, deter, dissuade
68. Greed Avarice, longing Generous, temperance, asceticism
69. Guile Cunning, deceit Candor, frank, direct
70. Gullible Credulous, unsuspicious Cynical, suspicious
71. Habitual Accustomed, regular Unaccustomed, occasional
72. Handicap Disability, disadvantage Benefit, advantage
73. Harass Annoy, disturb Assist, aid, soothe
74. Harmless Innocuous, inoffensive Dangerous, objectionable
75. Harsh Hard, coarse Refined, smooth
76. Hasty Abrupt, hurried Considered, slow
77. Haughty Arrogant, pretentious Modest, humble
78. Humiliate Humble, shame Aggrandise, glorify
79. Hygiene Cleanliness, sanitation Uncleanliness, dirty
80. Hypocrisy Duplicity, falseness Honesty, sincerity
81. Ideal Goal, perfection Attainable, real, concrete
82. Idle Lazy, unoccupied Industrious, sedulous, diligent
83. Ignorant Stupid, unintelligent Discerning, learned
84. Illogical Incongruent, rambling Logical, rational
85. Illustrious Eminent, famous Unknown, obscure

295

Book 1.indb 295 30/04/2019 4:48:40 PM


NMAT by GMAC™ Official Guide 2019

S.No Word Synonym Antonym


86. Imitate Copy, reflect Original, veritable, authentic, innovative
87. Immense Huge, mammoth, gargantuan, Diminutive, small, bantam
Brobdingnagian
88. Impartial Candid, impersonal, disinterested Biased, partisan
89. Implicate Accuse, insinuate Absolve, exonerate, acquit
90. Importune Beg, solicit Command, aid, benefaction
91. Inadvertent Accidental, unintentional Deliberate, intentional
92. Indifferent Apathetic, disinterested Watchful, prudent, observant
93. Isolate Detach, quarantine Amalgamate, integrate, unify
94. Jargon Argot, slang Standard, profoundness
95. Jovial Genial, merry Miserable, gloomy
96. Justification Excuse, reason Indictment, question
97. Juvenile Adolescent, immature Mature, adult
98. Keen Clever, observant Reluctant, unenthusiastic
99. Labour Toil, work Leisure, idleness
100. Liberal Lenient, open-minded Conservative, bigoted
101. Limitation Boundary, constraint Extension, strength
102. Lucid Clear, understandable Equivocal, ambivalent, ambiguous
103. Lucky Auspicious, fortunate Unfortunate, hapless, ill-fated
104. Manipulate Control, shape Transparent, expound
105. Marginal Borderline, limited Core, vast
106. Maze Complexity, labyrinth Order, harmony, cohesion
107. Merge Blend, fuse Separate, split
108. Narrow Confined, restricted Wide, broad
109. Necessary Mandatory, requisite Dispensable, expendable, non- essential
110. Negate Contradict, refute Confirm, validate, ratify
111. Negligent Careless, remiss Conscientious, careful
112. Nice Affable, benign Impolite, unfriendly, discourteous,
haughty
113. Noble Aristocratic, distinguished Ignoble, dishonourable
114. Novice Beginner, nonprofessional, neophyte, tyro Adroit, maestro, expert, veteran
115. Nuisance Annoyance, offence Benediction, gratification, help
116. Obedient Faithful, loyal Disobedient, rebellious, unruly
117. Objection Disapproval, protest Acquiescence, approval
118. Obligatory Compulsory, required Voluntary, optional
119. Opportune Advantageous, auspicious Disadvantageous, ill- timed

296

Book 1.indb 296 30/04/2019 4:48:40 PM


4.0  Language Skills Review  4.2  Vocabulary

S.No Word Synonym Antonym


120. Pacify Appease, placate Agitate, incite, provoke
121. Paramount Chief, leading Subordinate, inconsequential, marginal
122. Partisan Biased, dogmatic Impartial, fair, unbiased
123. Passive Inactive, lethargic Active, assertive
124. Permeate Diffuse, disseminate, percolate, imbue Block, sparse, barrage
125. Perpetuate Endure, preserve Halt, eradicate, expunge, annihilate
126. Perplex Astonish, baffle Elucidate, facilitate, enlighten
127. Persecute Afflict, harass Succor, protection
128. Radiate Effuse, emanate Concentrate, converge, focus
129. Realise Accomplish, fulfil Forfeit, relinquish
130. Receptacle Container, repository
131. Reconcile Atone, conciliate Estrange, alienate
132. Regret Deplore, grieve Applaud, welcome
133. Reliable Dependable, trustworthy Unreliable, untrustworthy, dodgy
134. Sanction Approval, permit Prohibit, ban
135. Shallow Superficial, trivial Profound, serious
136. Shrewd Careful, calculating Gullible, candid, naïve
137. Slight Delicate, slender, insult Sturdy, burly, compliment
138. Spontaneous Impromptu, unplanned Planned, forced, calculated
139. Stabilise Balance, steady Destabilise, weaken, wobble
140. Tame Domesticate, subdue Wild, independent, fierce
141. Tangle Intertwine, twist Disentangle, unravel
142. Thrift Conservation, prudence Profligacy, extravagance
143. Tumult Agitation, commotion Silence, tranquility
144. Turbulent Disordered, violent Peaceful, calm, quiet
145. Urbane Suave, sophisticated, debonair, elegant, Uncouth, boorish
cultivated
146. Vain Boastful, inflated Modest, humble, unpretentious
147. Valid Authorised, legitimate Invalid, illegal, void
148. Variety Assortment, diversity Uniformity, agreement
149. Verify Authenticate, substantiate Invalidate, refute, disprove
150. Yearling Youngling, col, filly, whelp, puppy Elder, doyen, veteran
151. Yearn Desire, pine for, longing Detest, hate, abominate, loathe
152. Yielding Pliant, tractable, compliant, submissive Stubborn, obstinate, obdurate, recalcitrant

297

Book 1.indb 297 30/04/2019 4:48:40 PM


NMAT by GMAC™ Official Guide 2019

3 Analogies
An analogy literally means ‘Drawing a comparison in order to show a similarity in some respect’. An
analogy basically uses a relationship between two (or more) elements to show similar relationship among
another set of elements. So, these questions aim to test overall logical understanding of the candidates
and how coherently they understand the different kinds of relationships among various elements.

Types of Analogies
There are various types of relationships which are used in analogy-based questions. Below is one such list
which shows the various relationships with one example each:

Type Example Type Example Type Example


Country & Japan: Yen Animal/thing Crow: Caw; Unit & Part Pen: Nib; Blade:
Currency & Sound Rain: Patter Fan; Book: Chapter
Instrument & Ammeter: Unit & Class Cup: Crockery Universal Pair Chair: Table;
Measurement Current Door: Window
Quantity & Unit Power: Watt Unit & Home Cow: Byre Study & Topic Cardiology: Heart
Unit & Group Fish: Shoal Game & Skating: Rink; Word & Extreme Quarrel: War;
Venue Cricket: Pitch Speak: Shout
Animal & Baby Deer: Fawn Worker & Tailor: Needle; Word & Synonym Solicit: Request
Tool Sculptor:
Chisel
Male & Female Wizard: Tool & Pen: Action Word & Antonym Chaos: Peace
Witch Action
Animal & Eagle: Swoop Product & Oil: Seed; Worker & Actor: Stage;
Movement raw material Metal; Ore Working place Mechanic: Garage

Let’s explore the various types of questions based on Analogy that are asked and the right way to solve
them:

Types of Questions Asked:


I. Completing analogous pair: Such questions give the relationship between a pair; along with this, the
first element of second pair is given and we have to find the second element based on the relationship
similar to the one given by first pair.
II. Simple Analogy: In such questions a simple statement is given where a relationship is given and we’re
asked the second element for the term given in question.
III. Choosing the analogous pair: In such questions, a pair is given in the question and we’ve to find a
suitable pair from the options given that resembles the pair given in the question:
IV. Multiple word analogy: In these questions three elements are given in a pair instead of two and we
have to select the suitable option.

Techniques of Solving Analogy Questions


In analogy questions, as tested on the NMAT by GMAC™, you will be given a pair of words in the
question stem that will have some relation between them. You need to select a pair of words from the
given choices that express the same relation as between the words in the question stem.

298

Book 1.indb 298 30/04/2019 4:48:40 PM


4.0  Language Skills Review  4.2  Vocabulary

1.
Trickle : Gush
(A) Run : Walk
(B) Rise : Collapse
(C) Puppy : Dog
(D) Sip : Gulp
(E) Room : Window

Keep in mind that in an analogy question, there will always be some connection between the words given to
you in the question stem. The first step is to identify that link or connection; let’s call this making a bridge.
Once you have made this bridge, plug your answer choices into this bridge and identify the answer choice
for which this bridge holds true. That is your answer.
So, in the above question, trickle means to fall slowly, whereas gush means to fall rapidly. Thus, the bridge
between the two words can be to gush is to trickle quickly.
Now, let’s try to plug in our answer choices in this bridge.
(A) To walk is to run quickly? No. In fact, the opposite is true. Remember that since you have made your
original bridge starting with the second word (gush), you should do the same while plugging the answer
choices into this bridge. So, you need to start with the second word ‘walk’ and not with the first word ‘run’.
(B) To collapse is to rise quickly? Absolutely not. These are, in fact, antonyms.
(C) To dog is to puppy quickly? Makes no sense.
(D) To gulp is to sip quickly? Yes! This option matches our original bridge perfectly and should be the
correct answer.
(E) To window is to room quickly? Makes no sense.
Hence the correct answer is (D).

Important Learning:  Make sure you plug into the answer choices in the same order as in the original
bridge. So, if you have started with the second word in the original bridge, start with the second word
while plugging in the answer choices and vice versa.

Tips for Solving Analogy Questions


Thus, the key in any analogy question in NMAT by GMAC™ is to make the original bridge correctly.
Some common bridges that you may encounter include cases in which the two words are antonyms of one
another (rise: collapse) or in which one word is a higher degree of the other (joy: euphoria).
Though there are numerous analogous relations, there are certain underlying associations that can help a
student ascertain the particular pair. These include:
Cause and effect - This analogy establishes a causal relationship between the two terms – one is the effect
of the other. For example- thirst – water, accident – injury, fire – burn and so on.
Opposite words – Words that convey diametrically opposite ideas or states are also used to establish
analogy, like Life – death , happiness – sadness, brave – timid and so on.

299

Book 1.indb 299 30/04/2019 4:48:42 PM


NMAT by GMAC™ Official Guide 2019

Group analogies – Essentially, these analogies include collective nouns. For example – a tower of giraffes, a
waddle of penguins, a pride of peacocks, a murder of crows and so on.
Classification analogies – These analogies establish a relation between an object and the group it belongs
to. One thing that should be kept in mind is that one object can belong to different categories. For example:
knife is used in the kitchen to cut vegetables and also as a weapon to cause violence.
Related objects – Such analogies establish a relationship between objects. These relations can be based on
utility, process or end result. For example: driver and car, doctor and patient, writer and book.
Analogy related to function – Such analogies are based on objects/people and their functions. For
example: Cap: Head :: Gloves: Hands.
Efforts to result – These analogies are established between the effort made towards an endeavour and the
end product. For example: read-newspaper, write-book, paint-painting.
Problem and solution – These analogies present a solution to a problem. For example: dirty-cleaning, tired-
rest and so on.
Example 1


Expand : Area
(A) Bend : Flexibility
(B) Noxious : Escape
(C) Drone : Monotonous
(D) Proliferate : Number
(E) Length : Increase

Solution
Here is the bridge between the words in the question stem—to expand is to increase the area.
Plugging this bridge in the answer choices, we get:
(A) To bend is to increase the flexibility—Incorrect
(B) To noxious is to increase the escape—Incorrect
(C) To drone is to increase the monotonous—Incorrect
(D) To proliferate is to increase the number—Correct
(E) To length is to increase the increase—Incorrect
The correct answer is D.

Example 2


Critic : Fault
(A) Artist : Praise
(B) Amateur : Persevere
(C) Athlete : Practice
(D) Arbitrator : Mediate
(E) Thief : Arrest

300

Book 1.indb 300 30/04/2019 4:48:42 PM


4.0  Language Skills Review  4.2  Vocabulary

Solution
Here is the bridge between the words in the question stem—the work of a critic is to (find) fault.
Plugging this bridge in the answer choices, we get:
(A) The work of an artist is to praise—Incorrect
(B) The work of an amateur is to persevere—Incorrect
(C) The work of an athlete is to practice—Incorrect (This is not the ‘work’ of an athlete. The work of
an athlete would be to compete.)
(D) The work of an arbitrator is to mediate—Correct
(E) The work of a thief is to arrest—Incorrect
The correct answer is D.

4 Fill in the Blanks


Fill in the Blanks questions are staples of aptitude tests. Most tests have them in some version or the other.
You may have had Fill in the Blanks type questions even in school exams, so the format is probably familiar.
Basically, it’s exactly what the name says: a sentence (or paragraph) is given with one or more words
missing, indicated by a blank. You have to fill in the correct word/s from the options given, going by the
context of the rest of the sentence (or paragraph). In short, a key skill that you need to Fill in the Blanks is a
bit of common sense!
Of course, the questions are not always as simple. Fill in the Blanks can test your vocabulary, grammar skills,
usage skills and/or reasoning skills. If you are weak in any of these areas, you may falter in these questions.
How can one type of question test so many different skills? That’s because the missing word can be any
part of speech or any kind of word (i.e. a short, common one or a long, obscure one), and may fit into the
sentence in any way.
As the name suggests, these questions will consist of a sentence or a group of sentences containing one or
two blanks. There will be some options given to you for each blank and you will need to select the correct
word from those options that can be put into that blank.
The key thing to keep in mind is that the word that goes into the blank(s) cannot be a figment of your
imagination; rather, it has to make sense with the meaning conveyed by the rest of the sentence. This
is where the comprehension part comes in. By giving these fill in the blank questions on the test, the
NMAT by GMAC™ is not just testing you on vocabulary but also on comprehension.

Solving Fill in the Blank Questions


Always try to predict the answer
In these questions always try to predict the answer before you take a look at the answer choices given to
you. This will prevent you from getting confused between similar looking answer choices. Your prediction
doesn’t have to be very accurate – even something broad like ‘the word should be a positive word’ or a
‘negative word’ can be good enough to eliminate some wrong answers.

301

Book 1.indb 301 30/04/2019 4:48:42 PM


NMAT by GMAC™ Official Guide 2019

Let’s say you get a fill in the blank question on the test such as this one,
Since Indian cricket team is in great form, it will ________ the upcoming cricket World Cup.
(A) lose
(B) surrender
(C) win
(D) abandon
(E) wreck

The first thing that you should do is just read the above sentence and not look at the options. After reading
the sentence, make a prediction in your head as to what kind of word you think should go in the blank.
The word can be anything, it can even be in Hindi. In the above sentence, the logical word has to be ‘win’
because if the team is in great form, the only logical thing is for it to emerge victorious. Once you have
made this prediction, look at the answer choices and go with the one that best matches your prediction, that
is (C).
This approach will also help you identify whether your weak area is vocabulary or comprehension. For
example, if you notice that you are making the wrong predictions for what you think should be the answer,
then you are not understanding the meaning of the sentence correctly. In that case, even if you memorise
the entire dictionary, it won’t help because your problem is not vocabulary in the first place. Similarly, if you
notice that you are making the correct predictions but not getting the answer right, then you need to work
at first strengthening your vocabulary base.
Use Keywords and Connectors to make predictions
You have just learnt above that you should always try to predict the correct answer for vocabulary in context
questions. In order to be able to make these predictions correctly, try to look for two kinds of clues:
1) The Keywords
2) The Connectors
Keywords are words that tell you the meaning of the word that should go in the blank.
For example consider this sentence:
Known for their bravery, horses are used as symbols of _________ in several cultures.
(A) arrogance
(B) courage
(C) loyalty
(D) speed
(E) stamina

As most of you might have guessed, the correct answer should be (B), courage. But why can’t the answer be
(C), loyalty? Because the sentence talks about horses being known for their ‘bravery’ i.e. courage, so ‘bravery’
becomes your keyword in this sentence. Hence, even though horses are also known for loyalty, speed, and
stamina, the answer still has to be courage because it is connected to the keyword in the sentence.

Important Learning:  Remember that the Keyword does not necessarily have to be a word; it can also
be a phrase or a clause.

302

Book 1.indb 302 30/04/2019 4:48:43 PM


4.0  Language Skills Review  4.2  Vocabulary

If ‘bravery’ were to be replaced with ‘devotion’ in the original sentence, then what should be the answer?
Known for their devotion, horses are used as symbols of _________ in several cultures.
The answer will then change to ‘loyalty’, because the Keyword now becomes ‘devotion’. This is how
Keywords can help you decide which word to go with in the blank, so you must consciously look for the
keyword in every sentence that you see.
However, sometimes the Keyword, on its own, may not be enough to convey the entire meaning of the
sentence. For example, consider a variation of the above sentence:
Although horses are known for their devotion, in some cultures they are used as symbols of _________.
(A) arrogance
(B) courage
(C) loyalty
(D) speed
(E) treachery

The Keyword is still ‘devotion’ but the meaning of the sentence has reversed because of the use of
‘Although’. We call such words Connectors because they help you determine the connection between two
parts of a sentence – whether they are connected in the same manner or in a contrasting manner.
In the above example the word that goes into the blank has to contrast with the Keyword ‘devotion’, so the
answer should actually be ‘treachery’.
Here is a list of some common ‘contrasting’ and some ‘same-direction’ Connectors:
Same Direction Connectors
• Because
• Since
• And
• Hence
• As a result of
• Also
• Due to
• Thus
• Likewise
• Moreover
• Consequently
• Additionally

Important Learning:  The ; (semi colon) is also a same direction connector.

Contrasting Connectors
• Despite
• Yet
• But

303

Book 1.indb 303 30/04/2019 4:48:44 PM


NMAT by GMAC™ Official Guide 2019

• However
• Nonetheless
• Nevertheless
• While
• Although
• Ironically
• Rather
• Contrastingly
Note that every sentence may not necessarily have a Connector. In such sentences the meaning, obviously,
always goes in the same direction.

Connector Type Examples


Similar likewise, in the same way, also, and
Concession / Contrasting although, but, however, nonetheless, nevertheless
Exemplify for instance, for example, particularly
Conclusion therefore, hence, consequently, as a result of, thus
Explanation so ... that, since, because, due to
Emphasis in particular, indeed
Contrasting despite, in spite of, conversely, surprisingly, in contrast, yet, while, rather
More Information moreover, additionally, furthermore

Types of Questions Asked


Fill in the Blanks can be classified on the basis of how many blanks the sentence has, or whether they
consist of a sentence or a paragraph. There can also be experimental types of Fill in the Blanks, in which the
blank is done away with altogether!
Let’s see these varieties one by one.

Single Blanks
Single blank Fill in the Blanks are the most basic type of Fill in the Blanks, and the other varieties are
variants of them.

Double Blanks
There is not much need to explain what these entail—this type of Fill in the Blanks consists of sentences
with two blanks instead of one.
Double blank Fill in the Blanks can be simply a double version of single blank Fill in the Blanks, i.e.
sentences with two independent missing words instead of one, or they can be slightly trickier, with two
missing words that depend on each other based on their meaning. In the latter case, each of the options
has to be first evaluated to see if the two words do indeed have the necessary connection, and then checked
against the sentence to see if they fit the overall meaning.
As Fill in the Blanks are not very reading-intensive, they can help you score more marks in very little time.
So make sure you give Fill in the Blanks their due attention!
One big advantage of solving Fill in the Blanks questions is that it takes less time per question to solve
them. So be sure you have your basic strategy for this question type in place.
304

Book 1.indb 304 30/04/2019 4:48:44 PM


4.0  Language Skills Review  4.2  Vocabulary

Let us consider a few examples:


Directions for examples 1-2: Each of the sentences below consists of one blank or two blanks. Choose the
word or set of words for each blank that best fits the meaning of the sentence as a whole.
Example 1


One requirement of a good book is that it deepens and extend our knowledge, not that it merely
__________what we already know.
(A) enhance
(B) confirm
(C) modify
(D) reduce
(E) vilify

Solution

Keyword—deepen and extend our knowledge


Connector—not (contrast)
The word that goes in the blank has to contrast with the keyword
Prediction—reiterate, restate
Confirm comes closest to our prediction and should be the correct answer.
The correct answer is B.

Example 2


There are many things to be said against American newspapers, but much of the __________ is
__________ when one considers that every now and then they develop a great writer like Don
Marquis.
(A) blandishment; kindled
(B) somnolence; underscored
(C) indictment; quashed
(D) criticism; upheld
(E) applause; negated

Solution

Keyword—many things to be said against


Connector—but (contrast)
Prediction for Blank 1—criticism, complaint
Prediction for Blank 2—acceptable, worthwhile
Indictment and quashed come closest to our prediction and should be the correct answer.
The correct answer is C.

305

Book 1.indb 305 30/04/2019 4:48:44 PM


NMAT by GMAC™ Official Guide 2019

5 Paragraph-Based Fill in The Blanks (Cloze Test)


You can probably tell what Paragraph-based Fill in the Blanks entail: they involve whole paragraphs instead
of single sentences. These are also called cloze test questions. The method of solving these is essentially the
same for the other two types of Fill in the Blanks we have seen so far.
Let us revisit the key concepts once more. As you will have seen by now, these questions are fairly simple to
tackle, as long as your vocabulary—and to a slightly lesser extent, you grammar and logic skills—are in good
shape. A little bit of practice will help you gain the speed to spot the right option and/or eliminate incorrect
ones fairly quickly.
Start by forming shadow words—these are likely words that could fill the blanks in the sentence—when
reading the sentences for the first time.
Follow this up by reading the options—if one of them is the shadow word you have already thought of, or
a close synonym, then you have your answer right there! Even if none of the options are the exact word you
first thought of, the shadow-word technique should help you eliminate at least one or two options.
Ensure that your answer matches the tone and style of the sentence.
Connector Words are very helpful so ensure that you do not miss them in a sentence. These connector
words tell you whether the word should be negative or positive.
Remember, some Fill in the Blanks may require you to fill the connector.
Know your prepositions and idioms well as these will help you in eliminating options.
Let us consider a few examples:
Directions for examples 1–4: The questions below consist of short passages from which several words
have been deleted and replaced with blanks. For each blank, select from the choices the word that best fits
that blank, taking into account the overall meaning conveyed by the passage.
Lady Hawkins had earned the___(1)___ of her extensive family by marrying below her station and
moving into her poor husband’s___(2)___, shabby house. Her father refused to talk to her and her
brothers had___(3)___all roads to reconciliation by publicly declaring that they never had a sister. The
family was in shock that a genteel lady, strictly brought up in___(4)___to rules that governed society,
would actually take such a step.
Example 1

(A) ire
(B) veneration
(C) love
(D) sympathy
(E) guilt

Solution
If you read the entire passage, it becomes clear that Lady Hawkins' family is upset with her. So the
correct word should be upset or angry. Ire comes closest to this and should be the correct answer.
Veneration—high degree of respect
The correct answer is A.

306

Book 1.indb 306 30/04/2019 4:48:45 PM


4.0  Language Skills Review  4.2  Vocabulary

Example 2

(A) uncouth
(B) malignant
(C) lavish
(D) ornate
(E) dilapidated

Solution
The keyword here is shabby, which tells us that the word that goes in the blank has to be a negative
word. This eliminates (C) and (D). Uncouth and malignant, while both negative terms, are not used for
a house. This eliminates (A) and (B) also. Dilapidated is the perfect answer, as it means in a state of
disrepair and is typically used for buildings.
Uncouth—ill mannered
Malignant—harmful
Ornate—beautiful or embellished
The correct answer is E.

Example 3

(A) reverberated
(B) decimated
(C) obfuscated
(D) revealed
(E) ridiculed

Solution
The sentence is saying that her brothers had broken or blocked all roads to reconciliation, that is, they
did not want a reconciliation. The best answer then has to be decimated which means to destroy.
Reverberate—to echo or vibrate
Obfuscate—to confuse
The correct answer is B.

Example 4

(A) denouncement
(B) dissonance
(C) absence
(D) deference
(E) scorn

Solution
The sentence is saying that Lady Hawkins had been brought up in accordance with the said rules.
Thus, the best answer should be deference, which means respect. She was taught to respect the rules
that governed society.
Denouncement—to condemn openly
Dissonance—discord or disagreement
Scorn—contempt or disdain
The correct answer is D.
307

Book 1.indb 307 30/04/2019 4:48:45 PM


NMAT by GMAC™ Official Guide 2019

6 NMAT by GMAC™ Word List


As you can see, vocabulary will comprise a large chunk of the questions you will see on NMAT by
GMAC™ verbal section.
While there is no end to the number of words you can learn, to start with here is a list of 1000 words
commonly tested on the NMAT by GMAC™. Of course, this is not a foolproof list and you will, almost
certainly, see words from outside this list on the test. But, as a first step, it is a good idea to start with this
list. Once you have learnt these words you can easily learn more words beyond this list.
Remember that consistency is the most important thing when it comes to expanding your vocabulary. We
suggest that you start with a small number of words every day—say 20 words or so—and then gradually
increase this number. If you progress at the rate of 20 words a day you will have completed all 1000 words
in just 50 days!

Important Learning:  While learning words, aim for quality and not quantity. If you study words
consistently, you will be able to learn a lot of words in relatively less time.

High Frequency Words:

A
Abase lower; degrade; humiliate
Abash make (someone) feel embarrassed, disconcerted or ashamed
Abate to reduce in amount, degree or intensity; lessen
Abdicate to relinquish (power or responsibility) formally
Abet assist, usually in doing something wrong; encourage
Abhorrent one that is hated; disgusting, loathsome or repellent
Abjure to renounce, repudiate, retract or give up, usually under oath
Abnegation renunciation; self-sacrifice
Abominable detestable; extremely unpleasant
Abortive unsuccessful; fruitless
Abridge condense or shorten
Absolve pardon (an offence)
Abstemious exercising moderation and self-restraint in appetite and behaviour
Abstruse difficult to understand; recondite
Abut to touch at one end or side; lie adjacent
Abyss an immeasurably deep chasm, depth or void
Accolade an expression of approval; praise
Acerbic sour or bitter tasting; acidic
Acme the highest point, as of achievement or development
Acquiesce assent; agree passively; comply without protest
Acumen shrewdness shown by keen insight
Acquit free from a charge or accusation
Acrimony bitter ill-natured animosity in speech or behaviour

308

Book 1.indb 308 30/04/2019 4:48:46 PM


4.0  Language Skills Review  4.2  Vocabulary

Adage wise saying; proverb


Admonish take to task; to criticise for a fault
Advocate support or push for something
Aegis protection; endorsement; guidance
Aesthetic concerning or characterised by an appreciation of beauty or good taste
Affectation a deliberate pretense or exaggerated display
Aggress take the initiative and go on the offensive
Alacrity liveliness and eagerness
Alleviate to lessen the pain; to make something better
Altruism the quality of unselfish concern for the welfare of others
Ambiguous open to two or more interpretations; of uncertain nature or significance
Ambrosial worthy of the gods; highly pleasing to the senses—especially that of taste
Ameliorate to make better
Amenable disposed or willing to comply
Amnesty a general pardon granted by a government, especially for political offences
Anachronism from an incorrect time period
Anarchy a state of lawlessness and disorder
Anathema a damnation or a curse
Animadversion harsh criticism or disapproval
Antediluvian old; ancient
Antipathy deep-seated hatred; extreme hostility and dislike
Apathy lack of interest or concern; indifference
Aphorism a short pithy instructive saying; an adage
Apocalyptic prophetic of devastation or ultimate doom
Apogee final climactic stage; the highest/farthest point
Appall filled with apprehension or alarm; cause to be unpleasantly surprised
Appellation a name, title or designation
Apposite something of striking appropriateness and pertinence
Approbation official recognition or approval
Arduous difficult to accomplish; demanding considerable mental effort and skill
Arrogate seize and take control without authority and possibly with force
Articulate able to express oneself easily in clear and effective language
Artless having or displaying no guile, cunning or deceit; innocent
Assiduous perseverance in carrying out an action; diligent
Assuage to satisfy or appease; to calm or to pacify
Attenuate to make slender, fine or small; to lessen the density of; rarefy
Audacious fearlessly, often recklessly, daring; bold
Austere severe or stern in disposition or appearance; bare
Autonomous independent in mind or judgment; self-directed
Avarice immoderate desire for wealth; cupidity
Aver to assert formally as a fact; to justify or prove

309

Book 1.indb 309 30/04/2019 4:48:46 PM


NMAT by GMAC™ Official Guide 2019

B
Badger annoy persistently
Baffle frustrate; perplex
Bait food or other lure used to catch fish or trap animals
Balk hesitate; recoil
Balmy mild and pleasant; soothing
Banal repeated too often; familiar through overuse; boring
Bane something causing misery or death
Baleful deadly or sinister
Base contemptible; morally bad; inferior in value or quality
Bask derive or receive pleasure from; get enjoyment from
Bawl cry loudly
Bedlam a state of extreme confusion and disorder
Bedraggle make wet and dirty, as from rain
Befuddle confuse thoroughly
Begrudge envy; give or allow unwillingly
Beguile attract; cause to be enamoured
Behemoth huge creature; something of monstrous size or power
Belittle lessen the authority, dignity or reputation of; express a negative opinion
Bellicose having or showing a ready disposition to fight
Belie represent falsely
Belligerent someone who fights or is aggressive
Bellow shout loudly and without restraint
Bemused confused; lost in thought; preoccupied
Benediction the act of praying for divine protection
Benevolent showing kindness; generous
Benign kindly; favourable; not malignant
Bequeath leave or give by will after one’s death
Berate censure severely or angrily
Besmirch charge falsely; attack the good name and reputation of someone
Blandish praise somewhat dishonestly
Blasphemy the act of depriving something of its sacred character
Boisterous noisy and lacking in restraint or discipline
Bombastic ostentatiously lofty in style
Boorish ill-mannered and coarse; contemptible in behaviour or appearance
Brackish slightly salty
Braggart a very boastful and talkative person
Brusque abrupt and curt in manner or speech
Bucolic descriptive of rural or pastoral life
Burgeon grow and flourish
Buttress a support, usually of stone or brick

310

Book 1.indb 310 30/04/2019 4:48:46 PM


4.0  Language Skills Review  4.2  Vocabulary

C
Cache hiding place, a collection of similar items stored in a hidden or inaccessible place
Cacophony a loud harsh or strident noise
Cajole persuade by praise or false promise; coax; wheedle
Callous hardened; without sympathy for the sufferings of others
Camaraderie goodwill and light-hearted rapport between or among friends
Canard unfounded false rumour; exaggerated false report
Candid free from prejudice; impartial; frank
Capitulate to surrender under specified conditions; come to terms
Capricious characterised by or subject to whims; impulsive and unpredictable
Cardinal of foremost importance; paramount
Caricature a representation of a person that is exaggerated for comic effect
Carnage the savage and excessive killing of many people
Castigation punishment; chastisement; reproof
Cataclysm an event resulting in great loss and misfortune; a great flood
Catholic relating to the Church; comprehensive or universal
Cavalier casual and offhand; arrogant
Cede surrender formally
Celerity speed; rapidity
Censure harsh criticism or disapproval
Certitude certainty
Charlatan a person who makes fraudulent, and often voluble, claims to skill or knowledge
Chasm a deep opening in the earth’s surface; a difference of ideas, beliefs or opinions
Chagrin strong feelings of embarrassment
Chicanery deception by trickery or sophistry
Choleric characterised by anger
Circumlocution an indirect way of expressing something
Citadel a stronghold into which people could go for shelter during a battle; fortress
Clairvoyant a person who can look into the future
Coercion using force to cause something to occur
Cogent powerfully persuasive
Cognizant marked by comprehension and perception: fully informed and aware
Collusion secret agreement or conspiracy
Colossus a person of exceptional importance and reputation
Comeliness the quality of being good looking and attractive
Commensurate corresponding in size, degree or extent; proportional
Commiserate to feel or express sympathy or compassion
Compendium a concise but comprehensive summary of a larger work
Complacent contented to a fault; self-satisfied and unconcerned
Complaisant showing a cheerful willingness to do favours for others
Concurrent occurring or operating at the same time

311

Book 1.indb 311 30/04/2019 4:48:46 PM


NMAT by GMAC™ Official Guide 2019

Condone excuse, overlook or make allowances for; be lenient with


Congeal to thicken or to solidify
Connoisseur an expert in some field, especially in the fine arts
Consecrate render holy by means of religious rites
Consequential having great significance, following as a result or effect
Contentious argumentative; quarrelsome; controversial
Conundrum a difficult problem; a puzzling situation
Convene call together
Convivial fun loving; fond of good company
Convoluted having numerous overlapping coils or folds
Copious affording an abundant supply
Cornucopia the property of being extremely abundant
Corporal of or relating to the body
Corpulent excessively fat
Covert covered over; sheltered; secret
Cower show submission or fear
Craven an abject coward
Credulous believe too readily; gullible
Crestfallen brought low in spirit; dejected
Cryptic secret; obscure in meaning
Culmination a concluding action
Culpable deserving blame or censure as being wrong or evil
Cursory hasty and without attention to detail; not thorough
Cynicism feeling of distrust

D
Dabble work in an amateurish manner
Dainty delicate; delicately beautiful
Dandy a man who is much concerned with his dress and appearance
Dapper neat in appearance and quick in movements
Dauntless having or showing courage
Dawdle loiter; hang around; waste time doing nothing
Deadpan impassive; with no show of feeling; with an expressionless face
Dearth a scarce supply; a lack
Debacle a complete failure
Debase degrade; reduce in quality or value; degenerate
Debauchery extreme indulgence in sensual pleasures; immoral self-indulgence
Debilitate weaken (through heat, hunger, illness); enfeeble
Decadence the state of being degenerate in mental or moral qualities
Decapitate to cut off the head; behead
Decipher decode
Decorum appropriate behaviour; good manners

312

Book 1.indb 312 30/04/2019 4:48:47 PM


4.0  Language Skills Review  4.2  Vocabulary

Decrepit weak and in bad condition (from old age)


Decry to condemn openly
Defection withdrawing support or help despite allegiance
Deference high degree of respect or courtesy
Defunct no longer in use, force or operation
Delectable greatly pleasing, normally associated with food; delicious
Deleterious having a harmful effect; injurious
Deluge a great flood or a heavy downpour
Demur to voice opposition; object
Demure shy
Denigrate to defame or belittle
Depravity moral corruption or degradation
Derelict deserted by an owner or keeper; abandoned; run-down; dilapidated
Derision the act of deriding or treating with contempt
Descry to discover by careful observation or scrutiny; detect
Despondent the condition of being depressed
Detrimental causing damage or harm; injurious
Diatribe a bitter, abusive denunciation
Dictum an authoritative statement
Diffident lacking self-confidence
Digress turn aside, especially from the main subject of attention
Dilapidation a state of deterioration due to old age or long use
Dilatory wasting time
Dilemma a confusing situation; a difficult choice
Dilettante lacking the required professional skill
Dirge a funeral hymn or lament
Disabuse free somebody (from an erroneous belief )
Discern detect with the senses
Disconsolate sad beyond comforting; incapable of being consoled
Disgruntled in a state of sulky dissatisfaction
Disparage to speak of in a slighting or disrespectful way; belittle
Disquietude feelings of anxiety that make you tense and irritable
Dissemble to disguise or conceal behind a false appearance
Disseminate to scatter widely, as in sowing seed
Dissidence disagreement, especially with the government
Dissuade discourage someone from a particular course of action
Divulge reveal a secret
Dogmatic orthodox; conventional
Dolorous showing sorrow
Dregs the sediment in a liquid; the basest or least desirable portion; residue
Droll arousing laughter
Dulcet pleasing to the ear

313

Book 1.indb 313 30/04/2019 4:48:47 PM


NMAT by GMAC™ Official Guide 2019

E
Ebb recede; lessen; diminish
Ebullience zestful enthusiasm
Eclectic combining elements from a variety of sources
Ecstasy rapture; very strong feeling of joy and happiness
Edifice building (of imposing size)
Effeminate having feminine traits or qualities; characterised by weakness and excessive refinement
Effervescence enthusiasm; vivacity; the process of bubbling as gas escapes
Effete exhausted, infertile or no longer effective; no longer possessing a unique quality
Efficacy power or capacity to produce a desired effect; effectiveness
Effulgence brilliant radiance; bright and sending out rays of light
Egregious bad or offensive; strong and offensive in odour or flavour
Egress a path or opening for going out; an exit
Elated filled with excited joy and pride; overjoyed
Elegy a poem or song composed especially as a lament for a deceased person
Elicit to bring or draw out
Eloquent persuasive, powerful discourse
Elucidate explain; make clear; clarify; enlighten
Elusive evasive; not frank; baffling; hard to grasp, catch or understand
Emaciation extreme thinness and wasting, caused by disease or undernutrition
Emanate issue forth; come out
Emancipate action or process of setting free, especially from legal, social or political restrictions
Embroil involve in dispute; complicate
Eminent rising above others; high; lofty; distinguished
Emolument salary; payment for an office; compensation
Emulate imitate; rival; try to equal or excel
Encomium warm, glowing praise
Endemic prevalent in or peculiar to a particular locality, region or people
Endorse approve; support
Enervate to weaken or destroy the strength or vitality
Engender to procreate; propagate; give rise to
Engross occupy fully; absorb
Enigmatic difficult to explain or understand
Enjoin to give orders to
Ennui the feeling of being bored by something tedious
Enthrall hold spellbound
Entice lure; persuade to do (something wrong); attract; tempt
Entrench fix firmly or securely
Epiphany a revelatory manifestation of a divine being
Equanimity the quality of being calm and even-tempered; maintaining composure
Equivocal deliberately ambiguous or vague

314

Book 1.indb 314 30/04/2019 4:48:47 PM


4.0  Language Skills Review  4.2  Vocabulary

Erratic lacking consistency, regularity or uniformity


Erudite extremely learned
Eschew to keep away from or to avoid
Esoteric known by a restricted number of people; understood by few
Eulogy high praise or commendation
Euphemism substituting a mild, indirect or vague term for one considered harsh, blunt or offensive
Euphoria a feeling of great happiness or well-being
Exacerbate to increase the severity; to aggravate further
Exasperate to provoke or annoy to an extreme degree
Exhume to remove from a grave; to dig out of the earth something that has been buried
Exigent requiring immediate action or remedy; demanding; exacting
Exodus a departure of a large number of people
Exonerate to free from blame
Expatriate to give up residence in one’s homeland; to send into exile
Expiate to make amends; atone
Expurgate remove parts considered harmful or improper for publication
Extirpate to destroy totally; exterminate
Extol to pay tribute or homage to; to honour
Exuberant full of unrestrained enthusiasm or joy

F
Fabrication a deliberately false or improbable account
Façade the face or front of a building or a showy misrepresentation intended to conceal
something unpleasant
Facile superficial; not deep
Factitious not produced by natural forces
Fallacy a misconception resulting from incorrect reasoning
Falter hesitate; weaken in purpose or action; walk or move unsteadily with weakness
Farce broad comedy; mockery; humorous play full of silly things happening
Fastidious giving careful attention to detail; hard to please; excessively concerned with cleanliness
Fathom comprehend
Fatuous devoid of intelligence
Fawn try to gain favour by cringing or flattering
Feign pretend
Felicity pleasing and appropriate manner or style; contentment; joy
Feral not domestic; wild
Fervent characterised by intense emotion; extremely hot
Fetid having a foul smell
Fickle changeable (in affections or friendship); faithless
Fidelity the quality of being faithful
Finesse delicate skill
Flabbergasted as if struck dumb with astonishment and surprise

315

Book 1.indb 315 30/04/2019 4:48:47 PM


NMAT by GMAC™ Official Guide 2019

Flag become less intense; lessen


Flagrant conspicuously and outrageously bad or reprehensible
Fledgling any new participant in some activity; young and inexperienced
Flout treat with contemptuous disregard
Fluke a stroke of luck
Flustered thrown into a state of agitated confusion
Forage the act of searching for food and provisions
Formidable inspiring fear or extremely impressive in strength or excellence
Fortuitous occurring by chance (positive) rather than intentional
Fractious likely to be troublesome or easily irritated or annoyed
Fructify make productive or fruitful
Frugality prudence in avoiding waste
Frustrate hinder or prevent (the efforts, plans or desires of )
Furtive secretive; sly; done with caution and stealth

G
Gainsay to deny, dispute or contradict; to speak or act against
Gait a person’s manner of walking
Galvanise to stimulate (someone) into taking action
Gambol gay or light-hearted recreational activity for diversion or amusement
Gamut entire range
Gape open the mouth wide; stare wonderingly with the mouth open
Garbled not orderly or coherent; lacking continuity
Gargantuan of great mass; huge and bulky
Garish overbright in colour; unpleasantly bright; gaudy
Garrulous full of trivial conversation; talkative
Gauche clumsy (in social behaviour); coarse and uncouth
Gaunt lean and angular; thin and bony; emaciated
Germane relevant and appropriate
Gesticulate motion; gesture
Ghastly shockingly repellent; inspiring horror
Gibe mock; make jeering remarks
Giddy dizzy; causing dizziness
Gingerly very carefully
Gist essence; main point; substance
Gloat express evil satisfaction; look at or think about with evil satisfaction
Gluttonous given to excess in consumption of especially food or drink
Grandiloquence high-flown style; excessive use of verbal ornamentation
Gregarious instinctively or temperamentally seeking and enjoying the company of others
Grouse complain
Grovel show submission or fear
Gullible easily tricked because of being too trusting

316

Book 1.indb 316 30/04/2019 4:48:47 PM


4.0  Language Skills Review  4.2  Vocabulary

H
Hackneyed repeated too often; overfamiliar through overuse
Hail frozen rain
Hale healthy
Hallowed blessed; consecrated
Harangue a loud bombastic declamation expressed with strong emotion
Harbinger something that precedes and indicates the approach of something or someone
Harrowing agonising; distressing; traumatic
Haughty proud and arrogant
Headstrong willful; stubborn; unyielding
Heckle verbally harass, as with gibes
Heed pay attention to
Herald messenger; sign of something to come; announce; proclaim
Heterodox characterised by departure from accepted beliefs or standards
Hiatus an interruption in the intensity or amount of something
Hone to sharpen; make perfect or complete
Hoodwink conceal one’s true motives by pretending to have good intentions so as to gain an end
Hubris overbearing pride or presumption
Husband use cautiously and frugally

I
Iconoclast someone who attacks cherished ideas or traditional institutions
Idiosyncrasy a characteristic, habit, mannerism or the like that is peculiar to an individual
Ignominy a state of dishonour
Illicit illegal
Illusory illusive; deceptive; not real
Imminent close in time; about to occur
Immutable unchanging
Impair make worse or less effective or imperfect
Impassioned filled with passion; fervent
Impassive having or revealing little emotion or sensibility; not easily aroused or excited
Impeccable faultless; perfect
Impecunious not having enough money to pay for necessities
Impede block or obstruct
Impending nearing; approaching; about to happen
Impenitent not penitent or remorseful
Imperious having or showing arrogant superiority to and disdain of those one views as unworthy
Impertinence the trait of being rude and inappropriate; inclined to take liberties
Impervious not admitting of passage or capable of being affected
Impetuous characterised by undue haste and lack of thought or deliberation
Impetus incentive; stimulus; momentum

317

Book 1.indb 317 30/04/2019 4:48:47 PM


NMAT by GMAC™ Official Guide 2019

Impiety without respect for God or religious values


Implacable incapable of being consoled/calmed
Implausible highly imaginative but unlikely
Implicate incriminate; involve incriminatingly; show to be involved (in a crime)
Implicit understood but not stated; implied
Implore ask or beg earnestly; beseech
Imponderable difficult or impossible to evaluate with precision
Impoverish make poor
Impuissance powerlessness revealed by an inability to act
Impunity exemption from punishment or loss
Inadvertent happening by chance or unexpectedly or unintentionally
Inane silly; senseless
Incapacitate permanently injure or in any way made unable to perform an action
Incarcerate imprison
Incessant uninterrupted; unceasing
Incinerate reduced to ashes
Incontrovertible impossible to deny or disprove
Incorrigible impossible to correct or reform
Incumbent currently holding an office or a position of authority
Indiscreet lacking good judgement; thoughtless
Indolent disinclined to work or exertion; lazy
Inebriate become drunk or drink excessively
Inexorable not to be moved by persuasion; unyielding
Infallible incapable of failure or error
Infringe advance beyond the usual limit
Ingenious showing inventiveness and skill
Ingenuous inability to mask your feelings, lacking in sophistication or worldliness
Inherent in the nature of something though not readily apparent
Inimical not friendly
Innocuous not injurious to physical or mental health; incapable of causing harm
Inordinate beyond normal limits
Insidious working or spreading in a hidden and usually injurious way
Insipid lacking interest or significance or impact; without flavour or taste
Insular narrowly restricted in outlook or scope; suggestive of the isolated life of an island
Intangible incapable of being perceived by the senses, especially the sense of touch
Interment the ritual placing of a corpse in a grave
Interminable tiresomely long; seemingly without end
Intractable difficult to manage or mould or change
Intransigent impervious to pleas, persuasion, requests or reason
Intrepid without fear or cannot be intimidated
Intuition a keen and quick insight; the ability to perceive the truth in something

318

Book 1.indb 318 30/04/2019 4:48:47 PM


4.0  Language Skills Review  4.2  Vocabulary

Inundate fill or cover completely or beyond normal capacity


Inured made tough and immune by habitual exposure
Invective abusive or venomous language used to express blame or bitter deep-seated ill will
Irascible quickly aroused to anger
Irrepressible impossible to control or suppress
Irresolute uncertain how to act or proceed
Itinerary a proposed route of travel

J
Jabber chatter rapidly or unintelligibly
Jargon a characteristic language of a particular group
Jeer showing your contempt by derision
Jeopardise pose a threat to; present a danger to
Jest activity characterised by good humour
Jibe an insulting remark to someone
Jocose/Jocular given to (having a tendency of ) joking
Jubilant joyful and proud, especially because of triumph or success
Juxtapose place side by side

K
Kernel central or vital part; core
Kindle call forth (emotions, feelings and responses) or cause to start burning
Kinship a close connection marked by community of interests or similarity in nature or
character
Kleptomaniac someone with an irrational urge to steal in the absence of an economic motive
Knack special talent
Knave a deceitful and unreliable scoundrel
Knell tolling of a bell, especially to indicate a funeral, disaster and so on
Knoll little round hill; hillock
Knotty intricate; difficult; tangled
Kudos an expression of approval and commendation

L
Labyrinth complex system of paths or tunnels in which it is easy to get lost
Lachrymose showing sorrow
Lackadaisical idle or indolent, especially in a dreamy way; lacking spirit or liveliness
Lacklustre lacking lustre (shine, gloss); dull
Laconic brief and to the point
Laggard someone who lags behind
Lament grieve; express sorrow
Languid lacking spirit or liveliness
Languish lose vigour, health or flesh, as through grief; become feeble

319

Book 1.indb 319 30/04/2019 4:48:47 PM


NMAT by GMAC™ Official Guide 2019

Largess liberality in bestowing gifts; extremely liberal and generous of spirit


Lassitude weariness; listlessness
Laud praise, glorify or honour
Lax careless; negligent; not paying enough attention
Lethargic deficient in alertness or activity
Levity a manner lacking seriousness
Libertine a dissolute person; usually a man who is morally unrestrained
Limpid crystal clear
Linger be slow in leaving; delay going
Linguistic consisting of or related to language
Lionise treat (a person) as a celebrity
Lissome moving and bending with ease
Listless lacking in spirit or energy; languid
Livid extremely angry
Loath reluctant; unwilling; disinclined
Loathe find repugnant
Loquacious full of trivial conversation
Loutish ill-mannered and coarse and contemptible in behaviour or appearance
Lucid transparently clear; easily understandable, transmitting light; able to be seen through
with clarity
Lucrative producing a sizeable profit
Lugubrious excessively mournful
Luminous softly bright or radiant
Lurid glaringly vivid and graphic; marked by sensationalism

M
Magnanimity liberality in bestowing gifts; extremely liberal and generous of spirit
Maim mutilate; injure lastingly; disable
Maladroit not skillful
Malady illness
Malapropism the unintentional misuse of a word by confusion with one that sounds similar
Malediction the act of calling down a curse that invokes evil (and usually serves as an insult)
Malevolent having or exerting a malignant influence
Malfeasance wrongful conduct by a public official
Malinger to pretend illness, especially in order to shirk one’s duty, avoid work and so on
Malleable adaptable; tractable; yielding
Malodorous having an unpleasant smell
Manifest evident; visible; obvious
Manipulate control or play upon (people, forces, etc.) artfully; maneuver
Mar spoil the appearance of
Martyr one who suffers for the sake of principle
Masquerade wear a mask or disguise; pretend

320

Book 1.indb 320 30/04/2019 4:48:47 PM


4.0  Language Skills Review  4.2  Vocabulary

Masticate chew (food); to bite and grind with the teeth


Maul handle roughly; batter; injure by beating
Maverick someone who exhibits great independence in thought and action
Maxim proverb; truth pithily stated
Mayhem violent disorder
Meagre scanty; inadequate
Meander wind or turn in its course; follow a winding or turning course; move aimlessly and idly
Meddlesome intrusive; interfering
Medley mixture
Meek submissive; patient and long-suffering
Melancholy gloomy; morose
Melee a noisy riotous fight
Mellifluous sounds that are pleasing to the ear
Menace something that is a source of danger
Mendacity the tendency to be untruthful
Mendicant a pauper who lives by begging
Mesmerise hypnotise
Metamorphosis change of form
Meticulous marked by extreme care in treatment of details
Mettle the courage to carry on
Misanthrope someone who dislikes people in general
Misconstrue interpret in the wrong way
Misdemeanor misbehaviour; misdeed; a crime less serious than a felony
Misnomer an incorrect or unsuitable name
Misogynist a misanthrope who dislikes women in particular
Mitigate make less severe or harsh
Mollify make less rigid or softer; make more temperate, acceptable or suitable
Mollycoddle treat with excessive indulgence
Morose showing a brooding ill humour
Mundane not ideal or heavenly; found in the ordinary course of events
Munificent very generous
Myopic unable to see distant objects clearly; lacking foresight or scope

N
Naive marked by or showing unaffected simplicity and lack of guile or worldly experience
Nascent being born or beginning
Natty marked by up-to-dateness in dress and manners
Nausea feeling of sickness and desire to vomit
Nebulous lacking definition or definite content
Necromancy conjuring up the dead, especially for prophesying
Nefarious extremely wicked
Nemesis something that brings an end to something; causing misery or death

321

Book 1.indb 321 30/04/2019 4:48:47 PM


NMAT by GMAC™ Official Guide 2019

Neophyte any new participant in some activity


Nepotism favouritism (to a relative)
Nettle cause annoyance in; disturb
Nimble quick in movement; agile; quick in understanding
Noisome foul smelling; causing or able to cause nausea
Nonchalant marked by complete lack of concern
Nonplussed filled with bewilderment
Nostalgia longing for the past
Notoriety disrepute; ill fame
Notoriety the state of being known for some unfavourable act or quality
Novice someone new to a field or activity
Noxious injurious to physical or mental health
Nuance a subtle difference in meaning or opinion or attitude
Nugatory of no real value
Numismatics the collection and study of money (and coins in particular)

O
Obdurate stubborn; resistant
Obeisance the act of obeying; dutiful or submissive behaviour
Obese excessively fat
Obfuscate make obscure or unclear
Objurgating to reproach or denounce vehemently; upbraid harshly; berate sharply
Obliterate destroy completely; wipe out
Oblivious inattentive or unmindful; unaware; wholly absorbed
Obnoxious causes disapproval or harm to something
Obscure dark; vague; unclear; not well known
Obsequious attentive in an ingratiating or servile manner
Obsolete outmoded; no longer used
Obstinate persist stubbornly
Obstreperous boisterously and noisily aggressive or defiant
Obtuse slow to learn or understand; lacking intellect
Obviate prevent the occurrence of; prevent from happening
Occlude block passage through
Odious sincerely hated and despised
Olfactory concerning the sense of smell
Ominous threatening; of an evil omen
Omnipotent having unlimited power
Omnipresent universally present; ubiquitous
Onerous burdensome, tiring, heavy load that makes one weary
Onus an onerous or difficult concern
Opprobrium a state of extreme dishonour and disgrace
Opulence wealth as exhibited by sumptuous living

322

Book 1.indb 322 30/04/2019 4:48:48 PM


4.0  Language Skills Review  4.2  Vocabulary

Ordain order by virtue of superior authority; decree


Ordeal severe trial or affliction; difficult experience
Orthodox traditional; (of someone) conservative in belief; adhering to an established doctrine
Ossified set in a rigidly conventional pattern of behaviour, habits or beliefs
Ostentatious intended to attract notice and impress others
Ostracise avoid speaking to or dealing with; expel
Overbearing having or showing arrogant superiority to and disdain of those one views as unworthy
Overt open and observable; not secret or hidden

P
Pacify soothe; make calm or quiet; subdue
Paean a formal expression of praise
Painstaking taking pains; showing hard work; taking great care
Palatable agreeable; pleasing to the taste
Palate roof of the mouth
Palette board on which painter mixes pigments
Palindrome a word or phrase that reads the same backward as forward
Pallid pale; wan
Palpable easily perceptible; obvious
Paltry meagre
Pan criticise harshly
Panacea hypothetical remedy for all ills or diseases
Panache flair; flamboyance
Pandemic widespread; affecting the majority of people
Pandemonium wild noisy disorder
Panegyric a formal expression of praise
Paradox a statement that contradicts itself
Paragon a perfect embodiment of a concept
Paramount foremost in importance; supreme
Paraphernalia equipment; odds and ends used in a particular activity
Paraphrase restatement of text in one’s own words
Parched extremely dry; very thirsty
Pare cut away the outer covering or skin of (with a knife); trim
Parochial narrowly restricted in outlook or scope
Parry dodge; circumvent
Parsimonious excessively unwilling to spend
Partisan one sided; prejudiced
Passive inactive
Patent obvious; easily seen; open for the public to read
Pathogenic able to cause disease
Pathos tender sorrow; pity
Patronise be a regular customer or client of

323

Book 1.indb 323 30/04/2019 4:48:48 PM


NMAT by GMAC™ Official Guide 2019

Paucity scarcity; dearth


Pauper very poor person
Peccadillo slight offence or fault
Pecuniary pertaining to money
Pedagogue someone who educates young people
Pedant a person who pays more attention to formal rules and book learning than they merit
Pedestrian lacking wit or imagination
Pejorative having a disparaging, derogatory or belittling effect or force
Pellucid transparently clear; easily understandable
Penitent feeling or expressing remorse for misdeeds
Penurious excessively unwilling to spend
Peremptory not allowing contradiction or refusal
Perennial recurring again and again
Perfidy an act of deliberate betrayal
Perfunctory hasty and without attention to detail; as a formality only
Pernicious working or spreading in a hidden and usually injurious way
Peroration the concluding section of an oration
Perspicacious having keen mental perception and understanding; acutely insightful and wise
Peruse examine or consider with attention and in detail
Pervasive spreading or spread throughout
Philanthropy donations to charity
Phlegmatic showing little emotion
Pillage the act of stealing valuable things from a place
Pillory to expose to public derision, ridicule or abuse
Pine have a desire for something or someone
Pique to arouse an emotion or provoke to action
Pith the choicest or most essential or most vital part of some idea or experience
Pithy concise and full of meaning
Pittance an inadequate payment
Placate to appease or pacify, especially by concessions or conciliatory gestures
Plagiarise take without referencing from someone else’s writing or speech
Plebiscite a vote by the electorate determining public opinion on a question of national
importance
Plethora extreme or excess
Pluck courage or resolution in the face of difficulties
Plumb examine thoroughly and in great depth; exactly
Plummet drop sharply
Polemic a controversial argument, as one against some opinion, doctrine and so on
Potion a medicinal or magical or poisonous beverage
Pragmatic concerned with practical matters
Prattle idle or foolish and irrelevant talk
Precursor something that precedes and indicates the approach of something or someone

324

Book 1.indb 324 30/04/2019 4:48:48 PM


4.0  Language Skills Review  4.2  Vocabulary

Predilection a predisposition in favour of something


Preen to be exultant or proud
Preponderant having superior power and influence
Prescience the power to foresee the future
Presumptuous unwarrantedly or impertinently bold
Prevaricate be deliberately ambiguous or unclear in order to mislead or withhold information
Pristine immaculately clean and unused
Privation a state of extreme poverty
Probity having strong moral principles
Proclivity a natural inclination
Prodigal wastefully or recklessly extravagant
Prodigious so great in size or force or extent as to elicit awe
Profligate shameless; dissolute; extravagant
Profound showing intellectual penetration or emotional depth; pervasive or intense; thorough
Profuse produced or growing in extreme abundance
Proletariat a social class comprising those who do manual labour or work for wages
Proliferate cause to grow or increase rapidly
Prolific productive
Prolix tediously prolonged or tending to speak or write at great length
Promulgate put a law into effect by formal declaration; promote an idea or cause
Propound put forward, as of an idea
Propriety correct or appropriate behaviour
Prosaic lacking wit or imagination
Proscribe command against; prohibit
Proselytise convert to another faith or religion
Prudence discretion in practical affairs
Puerile displaying or suggesting a lack of maturity
Pugilist someone who fights with his fists for sport
Punctilious marked by precise accordance with details
Pungent strong and sharp
Pusillanimous lacking in courage and manly strength
Putrefy become putrid; decay with an offensive smell

Q
Quack medically unqualified
Quaff to swallow hurriedly or greedily
Quagmire a soft wet area of low-lying land that sinks underfoot
Qualms a sudden feeling of apprehensive uneasiness
Quandary state of uncertainty or perplexity, especially as requiring a choice between equally
unfavourable options
Quarantine isolation to prevent the spread of infectious disease
Quarry animal hunted or caught for food

325

Book 1.indb 325 30/04/2019 4:48:48 PM


NMAT by GMAC™ Official Guide 2019

Quash put down by force or intimidation


Queasy causing or fraught with or showing anxiety
Quench suppress or crush completely; satisfy one’s thirst
Querulous habitually complaining
Quibble argue over petty things
Quiddity the quality that makes a thing what it is
Quiescent being at rest; quiet; still; inactive or motionless
Quirk a strange attitude or habit
Quisling a person who betrays his or her own country by aiding an invading enemy
Quiver a shaky motion
Quixotic not sensible about practical matters; idealistic and unrealistic
Quorum a gathering of the minimal number of members of an organisation to conduct business
Quotidian found in the ordinary course of events; usual or customary

R
Rabble mob; noisy crowd
Rabid marked by excessive enthusiasm for and intense devotion to a cause or idea
Racketeer a person who has dishonest and fraudulent dealings
Raconteur a person skilled in telling anecdotes
Raffish marked by a carefree unconventionality or disreputableness
Raffle lottery
Rail criticise severely
Rake immoral or dissolute person
Rally come or bring together; call up or summon
Ramification one of the results following from an action or decision
Rampant growing or spreading uncontrollably; growing in profusion
Ramshackle in deplorable condition
Rancid smelling of fermentation or staleness
Rancorous showing deep-seated resentment
Rank offensive in odour or flavour
Rankle irritate; fester; annoy
Ransack search thoroughly; pillage
Rant speak violently or excitedly; rave
Rapacious devouring or craving food in great quantities
Rapport close relationship; emotional closeness; harmony
Rapt engrossed; absorbed; enchanted
Rapture great joy and delight; ecstasy
Rarefy make more subtle or refined; become thin
Rave an extravagantly enthusiastic review
Recalcitrant marked by stubborn resistance to authority
Recant to reject or disavow a formerly held belief or opinion
Recondite difficult to penetrate; incomprehensible to one of ordinary understanding or knowledge

326

Book 1.indb 326 30/04/2019 4:48:48 PM


4.0  Language Skills Review  4.2  Vocabulary

Recuperate get over an illness or shock


Redoubtable worthy of respect or honour
Referendum a legislative act is referred for final approval to a popular vote by the electorate
Relegate assign to a lower position; reduce in rank
Remiss failing in what duty requires
Remonstrate censure severely or angrily
Renascence a second or new birth
Rendezvous a meeting planned at a certain time and place
Renege fail to fulfill a promise or obligation
Repertoire the entire range of skills or aptitudes or devices used in a particular field or occupation
Reprehensible bringing or deserving severe rebuke or censure
Reprisal a retaliatory action against an enemy in wartime
Repudiate eject as untrue, unfounded or unjust
Requiem a song or hymn of mourning composed or performed as a memorial to a dead person
Rescind cancel officially
Resilience an occurrence of rebounding or springing back
Restive being in a tense state
Reticence hesitation; shyness
Reverent feeling or showing profound respect or veneration
Rhetoric study of the technique and rules for using language effectively
Ribald someone who uses vulgar and offensive language
Risqué suggestive of sexual impropriety
Robust sturdy and strong in form, constitution or construction
Rupture burst

S
Sacerdotal associated with the priesthood or priests
Sacrilege blasphemous behaviour
Sacrosanct must be kept sacred
Sagacious acutely insightful and wise
Salubrious promoting health; healthful
Salutary synonym of salubrious
Salvage rescue (goods or property) from loss
Sanctimonious excessively or hypocritically pious
Sanction the act of final authorisation; restrictions or limitations
Sanguinary marked by eagerness to resort to violence and bloodshed
Sanguine a blood red colour; confidently optimistic and cheerful
Sap deplete
Sapid full of flavour
Sardonic disdainfully or ironically humorous; scornful and mocking
Satiate fill to satisfaction
Satire witty language used to convey insults or scorn

327

Book 1.indb 327 30/04/2019 4:48:48 PM


NMAT by GMAC™ Official Guide 2019

Saturnine sluggish in temperament; gloomy; taciturn


Saunter a leisurely walk
Savour enjoy; have a distinctive flavour, smell or quality
Scale climb up; ascend
Scanty meagre
Scapegoat someone who is punished for the errors of others
Schism division of a group into opposing factions
Scintillate sparkle; flash; be animated; be full of life
Scion a descendent or heir
Scoff laugh (at); mock; ridicule
Scorch a discolouration caused by heat, sear, burn
Scowl frown angrily
Scrupulous arising from a sense of right and wrong; principled
Scrutinise examine closely and critically
Scurrilous grossly or obscenely abusive
Seasoned experienced
Secede withdraw from an organisation or communion
Seclusion isolation; solitude
Sedate cause to be calm or quiet as by administering a sedative to
Sedentary requiring sitting or little activity
Sedition incitement of discontent or rebellion against a government
Sedulous marked by care and persistent effort
Seedy rundown; decrepit; disreputable
Seminal very important; containing seeds of later development
Senescent growing old
Sententious given to excessive moralising
Sepulcher a chamber that is used as a grave
Serendipity good luck in making unexpected and fortunate discoveries
Servitude state of subjection to an owner or master or forced labour imposed as punishment
Sever cut off from a whole
Shard a broken piece of a brittle artefact
Silhouette a drawing of the outline of an object
Simper to smile in a silly, self-conscious way
Simulate create a representation or model of
Sinewy consisting of tendons or resembling a tendon; possessing physical strength and weight;
rugged and powerful
Sinister threatening or foreshadowing evil or tragic developments
Skirmish a minor short-term fight
Sloth a disinclination to work or exert yourself
Solicitous anxious or concerned; eager
Somatic affecting or characteristic of the body as opposed to the mind or spirit

328

Book 1.indb 328 30/04/2019 4:48:48 PM


4.0  Language Skills Review  4.2  Vocabulary

Sophistry a false or deceptive argument


Sophomore a second year undergraduate
Soporific sleep inducing
Sordid meanly selfish; dirty; filthy
Specious plausible but false
Spendthrift someone who spends money prodigally
Sporadic recurring in scattered and irregular or unpredictable intervals
Spurious intended to deceive; fake
Squander spend extravagantly; waste
Static showing little if any change; angry criticism
Steep let sit in a liquid to extract a flavour or to cleanse
Stentorian very loud or powerful in sound
Stickler someone who insists on something
Stoic someone who is seemingly indifferent to emotions
Stolid having or revealing little emotion or sensibility; not easily aroused or excited
Strut a proud stiff pompous gait
Stultify deprive of strength or efficiency; make useless or worthless
Stupefy make senseless or dizzy by or as if by a blow
Stymie hinder or prevent the progress
Sublime lofty or grand
Succinct expressed in few words; concise
Succulent full of juice
Suffuse cause to spread or flush or flood through
Sully to soil, stain or tarnish
Supercilious expressive of contempt
Superfluous more than is needed, desired or required
Supplant take the place or move into the position of
Supplicate ask humbly (for something)
Surreptitious conducted with or marked by hidden aims or methods
Swelter suffer from intense heat
Sycophant a person who tries to please someone in order to gain a personal advantage

T
Tacit implied by or inferred from actions or statements
Taciturn habitually reserved and uncommunicative
Tawdry cheap and shoddy
Tedium dullness owing to length or slowness
Teetotaller one who abstains from drinking
Temerity fearless daring
Tempestuous characterised by violent emotions or behaviour
Tenacity persistent determination
Tendentious having or showing a definite tendency, bias or purpose

329

Book 1.indb 329 30/04/2019 4:48:48 PM


NMAT by GMAC™ Official Guide 2019

Tenet a religious doctrine that is proclaimed as true without proof


Tenuous lacking substance or significance; thin or slender in form
Tepid moderately warm; feeling or showing little interest or enthusiasm
Terse brief and to the point
Tether tie with a tether
Thrall the state of being under the control of another person
Throes violent pangs of suffering
Thwart hinder or prevent
Timorous timid by nature or revealing timidity
Tirade a speech of violent denunciation
Titan a person of exceptional importance and reputation
Toady a person who tries to please someone in order to gain a personal advantage
Topography precise detailed study of the surface features of a region
Torpid slow and apathetic
Torpor inactivity resulting from lethargy and lack of vigour or energy
Torque a twisting force
Tortuous not straightforward
Tousled in disarray; extremely disorderly
Tractable easily managed or controlled
Traduce speak unfavourably about
Transgression the action of going beyond or overstepping some boundary or limit
Transient one who stays for only a short time
Translucent allowing light to pass through diffusely
Transmute change in outward structure or looks
Travesty any grotesque or debased likeness or imitation
Trenchant incisive or keen; vigorous; clear-cut
Truculent defiantly aggressive
Truism an obvious truth
Truncate make shorter as if by cutting off
Tryst a secret rendezvous; a date
Tumefy expand abnormally
Turbid cloudy; murky
Turpitude a corrupt or depraved or degenerate act or practice
Tutelage teaching pupils individually
Tyro someone new to a field or activity

U
Ubiquitous being present everywhere at once
Ulterior being beyond what is seen or avowed; intentionally kept concealed
Umbrage a feeling of anger caused by being offended
Unabashed not embarrassed

330

Book 1.indb 330 30/04/2019 4:48:48 PM


4.0  Language Skills Review  4.2  Vocabulary

Unconscionable lacking a conscience


Unctuous characterised by excessive piousness or moralistic fervour
Undermine to attack by indirect, secret or underhand means
Underscore give extra weight to
Undulate move in a wavy pattern or with a rising and falling motion
Unfathomable impossible to understand
Unfeigned not pretended; sincerely felt or expressed
Unflagging unceasing
Unfledged young and inexperienced
Unfrock to deprive (a monk, priest, minister, etc.) of ecclesiastical rank, authority and function
Ungainly lacking grace in movement or posture
Unimpeachable free of guilt; not subject to blame
Unkempt not properly maintained or cared for
Unprecedented having no precedent
Unremitting not slackening or abating; incessant
Unsavoury morally offensive
Unseemly not in keeping with accepted standards of what is right or proper in polite society
Unstinting very generous
Unsullied free from blemishes
Untenable incapable of being defended or justified
Untoward contrary to your interests or welfare
Unwieldy difficult to use or handle or manage because of size or weight or shape
Unwitting not aware or knowing
Upbraid express criticism towards
Upshot the final issue, the conclusion or the result
Urbane sophisticated; polished; refined in manner
Usurp seize and take control without authority and possibly with force
Usury the act of lending money at an exorbitant rate of interest
Utilitarian having a useful function
Utopia an imaginary place considered to be perfect or ideal

V
Vacillate be undecided about something
Vacuous devoid of matter
Vagary an unpredictable or erratic action, occurrence, course or instance
Vainglorious feeling self-important
Valediction the act of saying farewell
Vanguard the leading position in any movement or field
Vantage the quality of having a superior or more favourable position
Vapid lacking significance, liveliness, spirit or taste
Variegated having a variety of colours
Venal capable of being corrupted

331

Book 1.indb 331 30/04/2019 4:48:49 PM


NMAT by GMAC™ Official Guide 2019

Vendetta any prolonged and bitter feud or rivalry


Venerate regard with feelings of respect and reverence
Veracity conformity to truth or fact; accuracy
Verbatim using exactly the same words
Verbose using or containing too many words
Verisimilitude the appearance of truth; the quality of seeming to be true
Vertiginous having or causing a whirling sensation, liable to falling
Vestige an indication that something has been present; trace of something that is disappearing
Vex to irritate; annoy; provoke
Viable capable of being done
Vicarious suffered or done by one person as a substitute for another
Vicissitude a change or variation occurring in the course of something
Vie compete for something
Vigilant carefully observant or attentive
Vignette a brief literary description
Vilify spread negative information about
Virtuoso having or revealing supreme mastery or skill
Virulent infectious; having the ability to cause disease
Viscuous thick
Vitreous relating to or resembling or derived from or containing glass
Vitriol abusive or venomous language
Vituperative marked by harshly abusive criticism
Vociferous conspicuously and offensively loud
Volition the act of making a choice
Voluble marked by a ready flow of speech
Voluminous large in number or quantity
Voracious devouring or craving food in great quantities
Vulnerable exposed to the possibility of being wounded or hurt

W
Waft be driven or carried along, as by the air
Wag move from side to side
Waive forego; dispense with
Wallow an indolent or clumsy rolling about; delight greatly in
Wanderlust very strong or irresistible impulse to travel
Wane a gradual decline (in size, strength, power or number)
Wastrel someone who dissipates resources self-indulgently
Waver the act of moving back and forth
Welter be immersed in; a confused multitude of things
Wheedle influence or urge by gentle urging, caressing or flattering
Whet make keen or more acute; stimulate
Whimsical determined by chance or impulse or whim rather than by necessity or reason

332

Book 1.indb 332 30/04/2019 4:48:49 PM


4.0  Language Skills Review  4.2  Vocabulary

Whittle cut small bits or pare shavings from


Wilful done by design; intentional
Wily marked by skill in deception
Winnow blow away or off with a current of air
Winsome charming in a childlike or naive way
Wizened lean and wrinkled by shrinkage as from age or illness
Wont an established custom
Wraith a visible spirit
Wreck a serious accident; smash or break forcefully
Writ a legal document issued by a court or judicial officer
Wry humorously sarcastic or mocking

X
Xenophobia a fear of foreigners or strangers
Xenophylic an attraction to foreign peoples, cultures or customs
Xerothermic characterised by heat and dryness
Xylophone a musical instrument

Y
Yearn have a desire for something
Yeoman farmer who owns and works his land
Yield bear, produce or provide
Yoke a connection, usually between cows on a farm
Yokel simple-minded country person; bumpkin
Yore time long past

Z
Zany ludicrous; foolish
Zealot a fervent and even militant proponent of something
Zenith highest point; apex
Zephyr a slight wind
Zest great enjoyment or excitement; gusto
Zoology study of animals

IDIOMS and PHRASAL VERBS


The English language is a living language having more than 1,000,000 words. It is one of the most
flexible languages in the world. Just like any other language, understanding the English language demands
knowledge beyond the literal meaning of words. It requires its speakers to understand the connotation of a
word, its tone and figurative usage. Idioms fall into this final category.
Idioms are a group of words/ expression that have a figurative meaning- a meaning separate from the literal
meaning conveyed by the constituent words.

333

Book 1.indb 333 30/04/2019 4:48:49 PM


NMAT by GMAC™ Official Guide 2019

Therefore, a comprehensive knowledge of idioms/phrasal verbs and their usage shall be of great assistance,
not only for the examination, but also for enhancing your understanding and comprehension of the
language.
As stated above, there are numerous idioms in the English language and the list is continuously evolving.
One of the most effective ways to increase one’s skill set in this section is to group idioms based on
common areas or themes. Let us look at a few examples:

Idioms based on Animals

Idiom Meaning Idiom Meaning


Paper Tiger Pretending to be tougher Bee in one’s bonnet An idea that constantly
or more dangerous than occupies one’s thought.
one really is.
Dropping like flies Dying/giving up quickly Catch a weasel asleep To surprise someone who
is normally alert and on
his/her guard.
Eager beaver A person who is excited A fine kettle of fish A situation that is not
about doing certain work satisfactory; a mess.
Straight from the horse’s Directly from the original Nest egg Money saved for the
mouth source. future.
Elephant in the room A problem or controversial Red Herring A red herring is a
issue everyone is aware misleading piece of
of but tries to avoid information aimed at
talking about because it is providing a distraction
embarrassing or will cause from the real facts of a
conflict. situation.

Idioms based on Body Parts

Idiom Meaning Idiom Meaning


To get someone’s back up To really annoy someone. Take it on the chin To be brave and not
complain when bad things
happen to you.
To have your back to the To be in a difficult or To raise eyebrows To shock people.
wall desperate situation.
Water off a duck’s back Criticisms of or warnings To drag your feet To be deliberately slow
to a particular person that
have no effect.
Achilles heel A small fault or weakness Cost and arm and a leg To be very expensive.
in a person or system that
can result in its failure.
Break a leg Used to wish someone By the skin of ones teeth. To barely just manage to
good luck. do something

334

Book 1.indb 334 30/04/2019 4:48:49 PM


4.0  Language Skills Review  4.2  Vocabulary

Idioms based on Colours

Idiom Meaning Idiom Meaning


White Elephant An expensive item that is In the black Meaning successful or
costly to maintain. profitable.
Blue pencil To censor or limit Blue Stocking Intelligent/Scholarly
information. woman.
Chase rainbows When someone tries to A golden handshake A large sum of money
get or achieve something that is paid to a retiring
that is difficult or manager or director, or to
impossible. a redundant worker.
To look through When someone is overly Red in tooth and claw Involving opposition or
rose-coloured/ tinted optimistic about things in competition that is violent
spectacles/ glasses life. and without pity.
Scream blue murder Is to scream loudly and Nail your colours to the To say publicly and firmly
for a long time, especially mast what you believe or who
in order to protest about you support.
something.

Idioms based on Food

Idiom Meaning Idiom Meaning


The best thing since sliced An excellent person or The apple doesn’t fall from A child usually behaves in
bread thing. the tree a similar way to his or her
parent(s).
The apple of somebody’s Used to indicate the Half a loaf is better than Something is better than
eye person whom someone no bread nothing.
loves most and is very
proud of.
The icing on the cake Extra and not essential Still waters run deep Means a person who
element that is added to seems to be quiet or
an already good situation shy may surprise you by
or experience and that knowing a lot or having
makes it even better. deep feelings.
A good egg A person who you can rely A fish out of water A person who feels
on to behave well. uncomfortable or awkward
because he or she is in
surroundings that are not
familiar.
A rotten apple Someone who does bad A curate’s egg Something with both good
things and influences and bad parts or qualities.
other people so that they
do bad things too.

There are several other themes that can be used to group idioms and their usage. These could be –
• Time
• Clothing
• Emotions

335

Book 1.indb 335 30/04/2019 4:48:49 PM


NMAT by GMAC™ Official Guide 2019

• Technology
• Humour
• Money
• Names
• Nature…………….

NOTE: Do not confuse idioms with idiomatic usage. Here is a list of some common usages that are
tested.

Words Usage Examples


ability to Your ability to lie has been self-evident at times.
access to Sam has access to the academic resources available at the research centre.
act as These E-Retailers act as revenue generators for the postal department.
allows for Futuristic brain probe allows for wireless control of neurons.
allows to Children are now allowed to watch almost all TV shows.
as...as Railways are as crowded as buses.
associate with I don’t like to associate with people such as Tom, Dick or Harry.
a responsibility to Everyone believes that teachers have a responsibility to inspire.
a result of Lowered crime rates are a direct result of harsher sentences to criminals.
agree to After she made a lot of promises, he agreed to go for a movie.
agree with I may not agree with what you have to say, but I’ll defend to the death your right to
say it.
among Should be used instead of ‘between’ when discussing more than two items.
She was the best athlete among all the six former students.
attend to X The doctor attended to the inebriated victim.
attribute X to Y In India, we attribute rainfall to the Gods.
attributed to X The extinction of the dinosaurs has been attributed to many reasons: the Ice Age,
an asteroid collision, etc.
based on Almost all inferences are based on statistical surveys conducted in the recent past.
begin to Sam will begin to understand the value of time when it is too late.
believe that In order to succeed, we must first believe that we can.
believe in An idea isn’t responsible for the people who believe in it.
believe X to be Y I believe education to be a major cause for poverty.
between ... and Only used to discuss two things.
Every morning he wakes up and decides between tea and coffee; as if there is any
choice!
both ... and Both fever and cold can be symptoms of other larger problems.
care about Cars and cameras are important. I don’t care about other stuff.
care of A true friend is the greatest of all blessings, and that which we take the least care of
all to acquire.

336

Chapter 4.indd 336 01/05/2019 4:35:49 PM


4.0  Language Skills Review  4.2  Vocabulary

Words Usage Examples


centers on The new agenda is too centered on the promotion of business and wealth creation
at the expense of everything else.
choose to Why is he unhappy now? This is the life he chose to live.
comprises Does not take any preposition.
This story comprises many colourful characters.
consistent with His aspirations are not consistent with the effort he puts in to achieve them.
consists of This story consists of many colourful characters.
contend that I contend that not only can you laugh at adversity, but it is also essential to do so if
you are to deal with setbacks without defeat.
consider Does not take any preposition.
This story is considered too violent for children.
contrast X with Y Contrast chocolates with any other food on the face of the earth today and ask
yourself whether chocolates are not the best thing to be found.
convert to You may convert your brains to slush by watching a lot of TV.
compare X to Y Used to highlight similarities.
It’s extremely dangerous and may I add pointless to compare anyone else to
Shakespeare.
compare X with Y Used to highlight differences.
Playing sports is a better way to spend your time compared with playing video
games.
count on She is counting on the administrative genius of her friend to win the elections.
concerned with ... than It is ironical that they are more concerned with schooling than education.
conform to In every generation the youth is supposed to conform to the prejudices and the
opinions of the old guard.
decide to Takes a verb.
They decided to continue fighting.
decide on Takes a noun.
They also decided on the plans for infiltration of the enemy camp.
depend on The result of the sports competition now depends on the athletics team.
different from Number is different from quantity.
difficult to Success is difficult to define.
distinguish between X and Y Innovation distinguishes between a leader and a follower.
distinguish X from Y Every person’s life ends the same way. It is only the details of how she lived and
how she died that distinguish one person from another.
either ... or For all meals, it is either rice or roti—how banal!
essential to They who can give up essential liberty to obtain a little temporary safety deserve
neither liberty nor safety.
except for An Englishman never enjoys himself, except for when it is for a noble purpose.
expect to You can expect to do well if you put in the hard work.
grow from All mighty trunks grow from small seeds.

337

Chapter 4.indd 337 01/05/2019 4:35:50 PM


NMAT by GMAC™ Official Guide 2019

Words Usage Examples


grow out of So many people try to grow out of childhood too fast, and it’s not fun! You should
stay a kid as long as possible!
in order to I don’t build in order to have clients. I have clients in order to build.
Read in order to live.
indicate that I don’t understand why the accent you speak in has to indicate what level of
intellect you have.
invest in He has been investing in the real estate market for the past few years.
identical with Faith is never identical with piety.
in contrast to A wonderful thing about a book, in contrast to a computer screen, is that you can
take it to bed with you.
independent from I truly believe that women should be financially independent from their men.
independent to I am not telling men to step away from speaking for women’s rights; rather, I am
focusing on women becoming independent enough to fight for themselves.
indifferent towards People don’t care enough. They don’t get worked up enough. They don’t get angry
enough. They don’t get passionate enough. I’d rather somebody hate what I do than
be indifferent towards it.
leads to Prosperity often leads to pride, which leads to sin.
leave behind It is not the honor that you take with you, but the heritage you leave behind.
leave with If you can come to the Olympic Games and leave with a medal then that is alright.
Like Used only for direct comparison, to mention a list use ‘such as’:
Always remember that you are absolutely unique. Just like everyone else.
Likely If you have no critics you’ll likely have no success.
localised in As you emphasize your life, you must localise and define it... you cannot do
everything.
mean by I’ve said many times, ‘You learn to win through not liking to lose.’ And that’s what I
mean by learning how to win.
mean for When I say something untrue on the air, I mean for it to be transparently untrue.
mistook ... for Like all dieters, I mistook eating less for eating well.
modeled after I went to a business school in Mumbai that is conceptual and interdisciplinary and
modeled after Harvard.
more than ever Investors demand dividends now more than ever.
native to The alphonso is native to the Konkan region of Maharashtra.
native of I have been particularly struck with the overwhelming evidence which is given as to
the fitness of the natives of India for various offices and employments.
need to I think we all have a need to know what we do not need to know.
necessary to It is not necessary to change. Survival is not mandatory.
neither...nor Neither a borrower nor a lender be.
not ... but I describe not men, but manners; not an individual, but a species.
not only...but also China not only does not support hacking but also opposes it.

338

Chapter 4.indd 338 01/05/2019 4:35:50 PM


4.0  Language Skills Review  4.2  Vocabulary

Words Usage Examples


prefer to An industrious sinner I much prefer to a lazy saint.
proceed against There was enough evidence with the state to proceed against the criminal.
proceed with In the arts, may you proceed with abandon. In life, may you proceed with balance
and stealth.
prohibit from I was prohibited from working and that was a very interesting period of my life.
potential to Her remarks had the potential to create a big argument in her family.
range from X to Y Evidence from mobile devices has provided critical help in solving crimes ranging
from homicides to drug trafficking.
refer to As far as the laws of mathematics refer to reality, they are not certain, and as far as
they are certain, they do not refer to reality.
regard as Exercise should be regarded as a tribute to your body.
require ... to Teenagers who are never required to vacuum are living in one.
result in War is a series of catastrophes which result in victory.
rivalry between X and Y There is no rivalry between films and theatres.
responsible for You can’t be responsible for the way people treat each other.
save for The less money you owe, the less income you’ll need and the less you’ll have to save
for tomorrow.
save from Many are saved from sin by being so inept at it.
seem that It seems that fighting is a game where everybody is the loser.
seem to Clock watchers never seem to be having a good time.
so that I’m whispering so that the media doesn’t hear me.
subscribe to Etiquette does not render you defenceless. If it did, even I wouldn’t subscribe to it.
But rudeness in retaliation for rudeness just doubles the amount of rudeness in the
world.
such as Use ‘such as’ to for a list; do not use ‘like’.
I cannot understate the ability to handle classical texts such as Shakespeare,
Faust, etc.
tie to We are more tied to our faults than to our virtues.
transmit to I would give no thought of what the world might say of me, if I could only transmit
to posterity the reputation of an honest man.

Phrasal Verbs
Phrasal verb Meaning
Abide by To accept a decision, a law or a rule
Account for To provide an explanation
Add up To amount to, make sense
Advise against To counsel someone not to do something
Agree with To have the same point of view as another

339

Chapter 4.indd 339 01/05/2019 4:35:50 PM


NMAT by GMAC™ Official Guide 2019

Phrasal verb Meaning


Allow for To take into account, to consider
Appeal to To beg or plead
To be attractive or interesting
Apply for To request for something in a formal way (job, permit, loan etc.)
Back away To retreat, out of fear or dislike
Back down To admit defeat
Back up To support or encourage
To make a copy of (file, programme, etc.)
Bank on To place one’s hopes on something / someone
Black out To faint, lose consciousness
Block off To separate using a barrier.
Blow up To explode
To get angry
Boil down to as the crux of a matter, the key point/s
Boot up To start a computer by loading an operating system or programme
Break away To separate from a crowd
Break down To stop operating, functioning
To lose control of one’s emotions
Break into To enter by force
Break out To start suddenly
Break out of To escape from a place by force
Break up To come to an end (marriage, relationship)
Bring up To raise (a child)
Brush up on To improve, refresh one’s knowledge of something
Bump into To meet by chance or unexpectedly
Burn out exhaustion caused due to extreme work
Call back To return a phone call
Call off To cancel
Calm down To relax, lose anger or stress
Carry on To continue
Carry out To do something as specified (a plan, an order, a threat)
To perform or conduct (test, experiment)
Check in To register at a hotel or airport

340

Book 1.indb 340 30/04/2019 4:48:50 PM


4.0  Language Skills Review  4.2  Vocabulary

Phrasal verb Meaning


Check out To settle one’s bill and leave (a hotel)
To investigate
Clam up To stop speaking
Clamp down on To act strictly to prevent something
Come across To find by luck
To appear, seem, make an impression
Come forward To present oneself
Count on To rely or depend on (for help)
Cut down on To reduce in number or size
Cut out To remove using scissors
To stop doing something
Deal with To handle, take care of (problem, situation)
Die down To calm down, become less strong
Do without To operate without
Drag on To last unnecessarily longer than required or expected
Draw up To create (contract, agreement, document)
Dress up To put on elegant clothes
Drop in To visit someone unexpectedly
Drop off To deliver someone or something
To fall asleep
Drop out To discontinue one’s education
Ease off To reduce, become less severe or slow down (pain, traffic, work)
End in To finish in a certain way; result in
End up To finally reach a state, place or action
Fall through Something that fails to materialise
Figure out To comprehend something
Fill out To fill-up (a form/an application)
Find out To discover or obtain information
Focus on To concentrate on something
Get along (with) To be friendly or comfortable with
Get at To insinuate
Get away To run away, escape
Get by To subsist or survive
Get in To enter

341

Book 1.indb 341 30/04/2019 4:48:50 PM


NMAT by GMAC™ Official Guide 2019

Phrasal verb Meaning


Get off To get down from a bus, train or airplane
To remove
Get on To board a bus, train or plane
Get on with (something) To continue to do; make progress
Get on (well) with To have a good relationship with
(somebody)
Get out To leave
Get out of To avoid doing something
Get over To recover from (illness, disappointment)
Get over To recover from (illness, disappointment)
Get rid of To eliminate
Get together To meet each other
Get up To rise, leave bed
Give in To cease opposition; yield
To hand in; submit
Give up To stop doing something
Go through To experience
Grow up To spend one’s childhood; develop; become an adult
Hand in To submit (report, homework)
Hand out To distribute
Hang out To spend time in a particular place or with a group of friends
Hang up To end a phone conversation
Hold on To wait
To grip tightly
Hurry up To be quick, act speedily
Iron out To resolve by discussion, eliminate differences
Join in To participate
Join up To engage in, become a member of
To meet and unite with
Keep on To continue doing something
Keep up with To stay at the same level as someone or something
Kick off To begin, start
Leave out To omit, not mention
Let down To disappoint

342

Book 1.indb 342 30/04/2019 4:48:50 PM


4.0  Language Skills Review  4.2  Vocabulary

Phrasal verb Meaning


Look after To take care of
Look down on To consider as inferior
Look on To be a spectator at an event
Look for To try to find something
Look forward to To await or anticipate with pleasure
Look up to To admire
Make fun of To laugh at/ make jokes about
Make up To invent (excuse, story)
Mix up To mistake one thing or person for another
Move in To arrive in a new home or office
Move out To leave your home/office for another one.
Nod off To fall asleep
Own up To admit or confess something
Pass away To die
Pass out To faint
Pay back To reimburse
Put off To postpone, arrange a later date
Put on To turn on, switch on
Put out To extinguish
Put up To accommodate, give somebody a bed
Pick up To collect somebody
Point out To indicate/direct attention to something
Rely on To count on, depend on, trust
Rule out To eliminate
Run away To escape from a place or suddenly leave
Run into To meet by accident or unexpectedly (also: bump into)
Run out of To have no more of something.
Set off To start a journey;
Set up To begin a business venture
Shop around To compare prices of similar things or services
Show off To indulge in behavior that is designed to gain attention/impress
Show up To arrive

343

Book 1.indb 343 30/04/2019 4:48:50 PM


NMAT by GMAC™ Official Guide 2019

Phrasal verb Meaning


Shut up (impolite) To ask someone to stop talking
Sit down To take a seat
Stand up To rise from a sitting position
Stick up for To defend someone
Take after To be similar in appearance or character
Take care of To look after
Take off To leave
Take on To recruit/take someone on-board
Take out To take off; sweep away
Tell off To scold/reprimand in strong terms
Think over To reflect on an idea before taking a decision.
Try on To put on clolthing in order to check its fitment
Turn down To decline
Use up To completely finish a product (so that there’s none left)
Watch out To warn someone asking them to be careful
Wear out To become unworthy of being used
Extremely tired
Work out Performing physical exercise
To find a solution or calculate something
Wipe off To clear away a table or clean a board

344

Book 1.indb 344 30/04/2019 4:48:50 PM


Book 1.indb 345 30/04/2019 4:48:50 PM
4.5 Grammar

Book 1.indb 346 30/04/2019 4:48:50 PM


4.0  Language Skills Review  4.5  Grammar

4.6 What is Measured?


The grammar section will test your ability to apply common grammar rules such as agreement of the subject
with the verb and with the pronoun, tenses, modifiers, etc.
You will also be tested on the correct usage of idiomatic expressions such as forbid to (and not from),
between.....and (not or), etc.
Your ability to understand the meaning of sentences will also be tested as you will need to identify the
option that conveys the correct meaning, using the least number of words.

4.7 Overall Test Taking Strategies


• Read the entire sentence carefully and make sure you have understood its meaning.
• Avoid using the ear (or what sounds correct) to answer questions. Try to apply the grammar rules
instead.
• Always narrow down to two choices and read both the choices back into the original sentence before
selecting your final answer.
The next few sections will provide you with in-depth strategies for approaching each topic.

1 Introduction
Grammar is perhaps the most important non-reading-based area of study for aptitude tests. This is not only
because grammar-related questions are found in some form or the other in almost all aptitude tests, but also
because grammar permeates the very fabric of the language, so any test which tests your English language
skills ultimately tests your English grammar skills as well. Indeed, it is no exaggeration that grammar skills
are second in importance only to reading skills as far as aptitude tests are concerned.
However, this book is not an in-depth guide to grammar—for two reasons. Firstly, learning the grammar
of a language to an advanced level is too vast an undertaking. Secondly, knowing a lot of grammar rules in
minute detail may confuse you more than it will help.
It makes more sense to focus on the small number of key rules and possible grammatical errors that
most questions are based on. Hence, the time taken to solve the grammar questions will decrease and the
accuracy will also improve.
For the purpose of this book, we assume that you already have a working knowledge of English grammar, in
that you can read, write, speak and understand it moderately well. We will also assume that you are familiar
with basic terms of grammar, such as nouns, verbs, phrases, clauses, subject, object, etc. (A basic definition
is given for each in the next section). But, what we will essentially focus on are the finer points of grammar,
which are explicitly tested in aptitude tests such as the NMAT by GMAC™.

2 Parts of Speech
Grammar-based questions can be of a number of types; we will learn some of these types later in this
section. But first, we need to review the more theoretical aspects of the kind of grammar that you are likely
to be tested on. Since aptitude tests are either multiple-choice tests or would require you to type in the
correct response in some cases, grammar questions focus mostly on identifying or correcting grammatical
errors in a given sentence or paragraph.

347

Book 1.indb 347 30/04/2019 4:48:50 PM


NMAT by GMAC™ Official Guide 2019

But before we look at errors and rules, let us begin by understanding the Parts of Speech.

Part of Speech Definition Sentence Usage

Verb Expresses action or state Wow! Seema and her talented sister have
participated successfully in several competitions.

Noun Name of a person, place, thing or Wow! Seema and her talented sister have
even idea participated successfully in several competitions.

Adjective Describes a noun Wow! Seema and her talented sister have
participated successfully in several competitions.

Adverb Describes a verb, adjective or adverb Wow! Seema and her talented sister have
participated successfully in several competitions.

Pronoun Substitutes a noun Wow! Seema and her talented sister have
participated successfully in several competitions.

Preposition Links a noun or pronoun to another Wow! Seema and her talented sister have
part of a sentence. participated successfully in several competitions.

Conjunction Connects clauses or sentences or Wow! Seema and her talented sister have
words participated successfully in several competitions.

Interjection A short exclamation which Wow! Seema and her talented sister have
expresses emotion. participated successfully in several competitions.

Cases of Nouns/Pronouns

Nominative Case Accusative Case Genitive Case

Also called the subjective case. Also called the objective case. Also called the possessive case.

It refers to the subject of the It is called the objective case because it It refers to the possession of
verb. refers to the direct object of a verb. On another noun.
the other hand, the ‘dative case’ refers to
the indirect object of the verb.

A noun or pronoun is in the A noun or pronoun is in the objective A noun or pronoun is in the
subjective case when it is used as case when it is used as a direct object, possessive case when it is used to
the subject of the sentence. an indirect object, or an object of the show ownership of an object.
preposition.

Examples: Examples: Examples:


Nancy participated in the state Neha gave me money. Where did you find my books?
championship. The teacher gave the notes to the This pen is mine.
I love dancing. student.

348

Chapter 4.indd 348 01/05/2019 4:36:31 PM


4.0  Language Skills Review  4.5  Grammar

Kindly refer the following table in case of any doubts –

First Person Second Person Third Person Impersonal


Subjective Singular I You She, He It
Plural We You They They
Objective Singular Me You Her, him It
Plural Us You Them Them
Possessive Singular My Your Her, His Its
Subjective Plural Our Your Their Theirs
Possessive Singular Mine Yours Hers, His Its
Objective Plural Ours Your Theirs Theirs

I/we/you/he/she/it/they are pronouns that represent the subject in a sentence while me/us/you/him/her/
it/them are pronouns that represent the object.
I gave the packet to him.
She gave a rose to me.
We are faster than they are fast, gets shortened to:
We are faster that they.
We are faster than them. (Incorrect)

Use of I v/s ME
Both I and me are 1st person singular pronouns, which means that they are used by one person to refer to
himself or herself.
I is the subject pronoun, and is used for the person “doing” the verb. Consider the following examples:
I am waiting for my friends (I is the subject for am waiting)
I don’t think he is as willing to consider a new idea. (I is the subject for think)

Me is the object pronoun, used as the object (or receiver) of the action of the verb, as in these examples:
My father took me to the doctor. (Me is the object of took)
She motivated me to participate in the competition. (Me is the object of motivated)

Note: If you are having trouble deciding the pronoun to be used, try completing the sentence. That should
help you decide the right pronoun.

Use of Who, Which, and That


Who, which and that are relative pronouns (that is, they are used to refer back to a person or thing
previously mentioned). While we use ‘who’ to refer to people; ‘which’ and ‘that’ are used largely to refer to
things.
However, when introducing a restrictive relative clause, we use ‘that’ or ‘who’ or ‘which’ without a comma.
Consider the following examples:
My car that is big consumes a lot of petrol (‘That’ is used for the purpose of identification and definition. In
this sentence, ‘that is big’ defines the ‘car’ being referred to. It indicates the presence of more than one car
and restricts the information being provided to a particular car. )

349

Book 1.indb 349 30/04/2019 4:48:50 PM


NMAT by GMAC™ Official Guide 2019

My car, which is big, consumes a lot of petrol (in this sentence, ‘which’ is introducing additional information
that does not impact the sentence as a whole; I can simply say – “my car consumes a lot of petrol”)
We don’t use ‘that’ to introduce a non- restrictive additional clause –
My bag, that is blue, is quite old (Incorrect)
My bag, which is blue, is quite old (Correct)

After two antecedents, one of which is the name of a person and the other, the name of some animal or
thing, use ‘that’ in place of ‘who’ or ‘which’.
The lady and her pet dog that came yesterday have come again today.

After words such as all, any, none, only, alone, nothing, use ‘that’ in place of ‘which’ or ‘who’.
Man is the only animal that can think.
All that glitters is not gold.
In case of any doubts, refer to the following table:

Relative Relative clause referring to Relative clause Restrictive relative clause Non-restrictive
pronoun person or people referring to things relative clause
that √ √ √ X
which X [used in traditional texts] √ √ √
who √ X √ √

Use of Who and Whom


The pronoun ‘who’ is used in place of a noun/pronoun in the nominative case while the pronoun ‘whom’ is
used in place of a noun/pronoun in the accusative case.
In case of any doubts, follow this simple rule; if you can replace the word with ‘he’ or ‘she’; use ‘who’.
However, if you find yourself using ‘her’ or ‘him’ to replace the word, then use ‘whom’. The same principle
applies to the use of ‘whoever’ and ‘whomever’.
Consider the following sentences:
Fakir is the man (who/whom) has been chosen (The correct answer is ‘who’; as ‘he’ is the man who has been chosen)
Fakir is the man (who/whom) we have chosen ( The correct pronoun is ‘whom’ as ‘We- subject’ have chosen ‘him’)

Use of ‘each other’ and ‘one another’


Each other refers to two items while one another refers to more than two items.
At the campus I came across my colleague and we complimented each other.
The guests at the party knew one another.
(The sentence implies that each guest knew the rest.)
The scientists at the conference were exchanging ideas with each other.
(This sentence implies that the exchange was happening between two scientists at a time)

350

Book 1.indb 350 30/04/2019 4:48:50 PM


4.0  Language Skills Review  4.5  Grammar

Countable and uncountable nouns


Nouns are ‘naming words’- words that are used to name persons, things, animals, places, ideas, or events
are nouns. There are different types of nouns but more often than not, the errors come from countable and
uncountable nouns.
Countable – Countable nouns refer to anything that can be assigned a count, and has a singular and plural
form. Examples: Bottle, Computer, Shirt
Mass (Non-countable) –Non-countable nouns are those nouns that can’t be counted as it is not possible
to break them into separate elements. They need determiners and quantifiers in order to be put in units and
counted. Examples: Milk, Oil, Sand, Oxygen
Please note: Uncountable nouns are always in singular number. They don’t have any plural form!
Note: All these nouns are uncountable as per the English language: accommodation, advice, behavior,
information, news, progress, traffic, travel, trouble, weather, work, tea, water, air, knowledge, beauty, anger,
fear, love, money, research, safety, evidence. Also, there are certain nouns that, though countable by definition,
are always treated as uncountable in the English language. These include: baggage, bread, furniture, luggage,
sugar, rice and so on.
The noun ‘hair’ is considered uncountable in English; hence it does not have a plural version. It can be countable
only when referring to individual hairs. Consider the examples given below:
She has long blond hair.
I washed my hair yesterday.
My father is getting a few grey hairs now. (Refers to individual stands of hair)
I found a hair in my soup! (Refers to a single strand of hair)
Note: There are certain non-count nouns that convey a different meaning when used in the plural form.
Let us consider a few examples:
Water – The non-count ‘water’ is used to indicate any quantity of water, whether a small amount (get me a
glass of water) or large amount (the water is too cold)
Waters - The plural ‘waters’, on the other hand, is used especially for an area of seawater, and tends to appear
in more formal contexts (“shipping lanes in international waters”) and in literary contexts (“the azure skies
and clear waters of the Pacific”).
Wood - Wood is (uncountable) the substance making up the central part of the trunk and branches of a tree
used as a material for construction, to manufacture various items, and so on or as fuel.
Woods - Woods is (uncountable) a dense collection of trees covering a relatively small area; smaller than a
forest.
Certain other words that convey different meanings in their singular and plural form are: work-works,
paper-papers, money-monies, good- goods, and so on.

351

Book 1.indb 351 30/04/2019 4:48:50 PM


NMAT by GMAC™ Official Guide 2019

Examples of quantifiers
With uncountable nouns With countable nouns With Both
much many all
a little/little/very little a few/few/very few enough
a bit (of ) a number (of ) more/most
a great deal of several less/least
a large amount of a large number of no/none
a large quantity of a great number of not any
a majority of some
any
a lot of
lots of
plenty of

Many is used before plural countable nouns while much is used before uncountable nouns:
Anuj didn’t have much experience.
On the way she made many mistakes.
Much time was wasted on unproductive tasks. (Considerable amount of time)
Before countable nouns, we use “few” and before uncountable nouns, we use “less” or “little”.
Few and A few have different meanings: Few is equivalent to “something negligible” or “hardly any” while,
a few is equivalent to “some”. Example:
Few persons can keep a promise.
A few persons are convinced about the new boss.
Same is the case with ‘little’ and ‘a little’.
Collective Nouns– Collective nouns refer to a collection of persons or things that is considered as a whole.
Example: class, pride, fleet, pack, deck, flock.
Use of ‘Among’ and ‘Between’
‘Between’ is used to refer to relationship involving only two things while ‘among’ represents relationship
involving several persons or things.
Among the forty students, she was the brightest.
An agreement was reached between Goodland and Badland to maintain peace.

Use of ‘As’ and ‘like’


As is used both as a conjunction and as a preposition:
The crowd rejoiced as the Sun came up. (conjunction)
India is as big as Brazil. (conjunction)
Modi has been very successful as a chief minister. (preposition)
‘Like’ is generally used as a preposition and is therefore followed by a noun, pronoun or a gerund.
Personalities like Nehru still inspire people.
There is nothing like having a hot cup of tea on a cold morning.

352

Book 1.indb 352 30/04/2019 4:48:51 PM


4.0  Language Skills Review  4.5  Grammar

Note: Use ‘like’ to compare nouns and ‘as’ to compare clauses.


• Sanjay is a good student, like his brother.
• You should behave with others as you would want them to behave with you.

Using the correct Adverb


Adverbs are words that are used to modify or describe verbs. They typically end with an ‘ly’ construction.
For example, in the sentence ‘Jackie runs quickly’ the adverb quickly describes or modifies the verb runs.
However, adverbs can also modify adjectives and even other adverbs.
This is a seemingly difficult question (the adverb seemingly modifies the adjective difficult)
Kumud eats very slowly (the adverb very modifies the adverb slowly, which in turn modifies the verb eats)
Here is a list of some important adverbs from the exam perspective:
i. Since – Since is used to denote when an activity started. For example, I have been waiting here since
yesterday.
ii. Too – Too is used before adjectives. The use of too much before adjectives is wrong.
You are too much nice to him. INCORRECT
You are too nice to him. CORRECT
iii. Enough – When enough is used to modify adjectives or adverbs, it is always put after the adjective or
adverb.
Is the house enough large? INCORRECT
Is the house large enough? CORRECT
iv. Fairly/rather – Use fairly to modify positive adjectives and rather to modify negative adjective.
Akash is fairly confident of securing 90% or above in this years’ examination. CORRECT
Akash is rather confident of securing 90% or above in this years’ examination. INCORRECT
Akash is rather unsure about how he will fare in the examination this year. CORRECT
Akash is fairly unsure about how he will fare in the examination this year. INCORRECT
v. Still – Use still to show that something started in the past and continues into the present.
He still hasn’t given me the answer to my question.

Using the correct Article


There are three articles in the English language – a, an and the.
The articles a and an are used before singular nouns. In choosing between a and an, you need to focus on
the pronunciation of the word that comes immediately after the article. If that word starts with a vowel
pronunciation, go with an and if that word starts with a consonant pronunciation, go with a.
For example,
This is a pen
This is an apple.
The is called a definite article and is used when you are trying to refer to a specific noun.
For example,
This is the pen that I bought yesterday (you are talking about a particular pen and not any pen in general)
This is the apple that fell on Newton. (you are talking about a specific apple and not about apples in general)

353

Book 1.indb 353 30/04/2019 4:48:51 PM


NMAT by GMAC™ Official Guide 2019

Using the correct Prepositions


Prepositions are words that provide some additional information to whoever is reading the sentence. This
information can include where something takes place (such as ‘at’ the mall), when or why something takes
place (such as ‘after’ lunch), or general descriptive information (such as the man ‘with’ the golden gun).
Remember that prepositions are always followed by a noun; they are never followed by a verb.
Prepositions for place – above, across, along, at, behind, below, beside, between
Prepositions for position – down, by, from, in, in front of, into, inside, near, off
Prepositions for direction – on, opposite, outside, over, around, through, to, towards, up
Prepositions for time – at, after, before, by, during, for
Other important prepositions – as, like, about, with, without, for
Here are a few important points to be remembered with respect to prepositions:
A preposition is usually placed before its object, but sometimes it is placed after it. Examples:
Who are you talking to (Incorrect: To whom are you talking?)
What are you looking for (Incorrect: For what are you looking?)
Which of these chairs did you sit on? (Incorrect: You sat on which of these chairs?)
A word may be used as an adverb or a preposition. One decides its role on the basis of the position and
function of the word in that particular sentence.
Example:
She walked up the stairs and entered the chamber. (Preposition)
Rahul took the up escalator and reached the top of the apartments. (Adjective)
The businessman upped his offer by a million rupees and bought the house. (Verb)
The teacher agreed to take up the matter with the higher authorities. Particle (‘take up’ = phrasal verb)
His business venture is on the up. (Noun)
Getting her to agree to anything is an uphill task.

3 Common Errors
Subject-Verb Agreement Errors
The verb of a sentence must agree in terms of person and number with its subject.
For example, if the subject is in the third-person singular form, the verb should also be in the third-person
singular; if the subject is in the first-person plural, the verb should also be in the first-person plural; and so
on.
Verb forms
What exactly do we mean by the terms ‘first-person’, ‘third-person’ and so on? And how is a verb expressed
in these forms? The following table, which shows the conjugation of the verb ‘to be’ (in the simple present
tense) should make it clear.

354

Book 1.indb 354 30/04/2019 4:48:51 PM


4.0  Language Skills Review  4.5  Grammar

First-person Second-person Third-person


Singular I am You are He/She/It is
Plural We are You are They are

Unlike in some languages such as French, English verbs do not have to worry about gender agreement. So,
English verbs mostly change with respect to number i.e. one or many.
So why is subject-verb agreement such a big deal?
First, it is very easy to make the subject a long winded one and hence to lose the original subject itself. For
example: ‘The recent epidemic of contagious diseases has caused a lot of concern to the government’. In this
sentence a naive reader may just assume the subject to be diseases instead of epidemic.
Second, some common subjects themselves are prone to confusion. For instance: ‘each’ is singular, ‘the
number’ is singular whereas ‘a number’ is plural, etc.
Let’s see some examples where subject-verb agreement can pose a problem.
Example 1

These shoes fits me perfectly.


‘Shoes’ is plural, so it should take the plural form of the verb ‘to fit’. Like all verbs except ‘to be’, the plural
form of ‘to fit’ is the same as the infinitive, i.e. ‘fit’. Don’t mistake the ‘s’ at the end of the singular form of a
verb for the ‘s’ in the plural form of most nouns! The two have nothing to do with each other.
These shoes fit me perfectly.
Subject-verb agreement errors often occur in cases where the subject and verb of a sentence are far apart.
Example 2

This box of books need to be put away.


Since the plural word ‘books’ comes right before the verb ‘need’, it’s tempting to think that it is the subject,
but in fact the correct subject is the singular word ‘box’. It is, in fact, the box that needs to be put away. So
the verb should be in the singular form:
This box of books needs to be put away.
Words like ‘no one/nobody’, ‘someone/somebody’, ‘everyone/everybody’, ‘anyone/anybody’, ‘either’, ‘neither’, ‘each’,
‘another’, etc. are always considered singular, and therefore take singular verbs.
Example 3

Neither of my sisters are coming to the party.


‘Neither’ is the subject, and it is singular, so the verb should also be singular.
Neither of my sisters is coming to the party.
Collective nouns—i.e. nouns which denote a group of objects, people, etc. can cause some confusion in case
of subject-verb agreement. For example, which of the following sentences is correct?
Example 4

The audience do not seem to like the play.


The audience does not seem to like the play.

355

Chapter 4.indd 355 01/05/2019 4:36:31 PM


NMAT by GMAC™ Official Guide 2019

The collective noun ‘audience’ is used with a plural verb in the first sentence and with a singular verb in the
second. Neither is incorrect in this context, as generally speaking, collective nouns can be considered either
singular or plural.
In most cases, collective nouns take a singular verb.
However, there are some cases when they have to take a plural verb.
Example 5

The family next door is fighting amongst themselves again.


‘Family’ is a collective noun, and mostly takes a singular verb. But in this context, a plural verb would make
more sense, as it is the individual members of the family who are fighting (with each other), not the family
as a single unit. So the correct form should be:
The family next door are fighting amongst themselves again.
Some More Examples:
Here are some more examples of subject-verb disagreement. The changed parts are underlined in the
correct versions.
Incorrect—This pair of shoes feel uncomfortable.
Correct—This pair of shoes feels uncomfortable.
Incorrect—One of my neighbour’s kids have broken my window.
Correct—One of my neighbour’s kids has broken my window.
Incorrect—As the principal walks in, the class sits down in their seats.
Correct—As the principal walks in, the class sit down in their seats.

Pronoun Antecedent Agreement Errors


As you must already know, a pronoun is a word that stands for a noun or group of nouns. So instead of
referring to a particular person or thing by his/her/its name throughout a discussion, we can refer to him/
her/it by the pronouns ‘he/she/it’.
Example 6

I have lost my pen. Have you seen it?


I watched this film with my brother—he did not like it but I surely did.
The noun that a pronoun stands in for is called its antecedent. It may or may not be explicitly stated in the
given sentence.
Pronouns can be used for nouns that are the subjects or the objects in a sentence, but most change their
forms according to their position in the sentence.
Ravi arrived late this morning­—He arrived late this morning.
I gave the letter to Ravi—I gave the letter to him.
At first glance, pronouns may seem quite simple and straightforward. But they can often be a source of
confusion in long, complicated sentences, with multiple possible antecedents.
A pronoun should clearly refer to only one antecedent.
If there is ambiguity as to which antecedent it refers to, the sentence can be considered erroneous.

356

Book 1.indb 356 30/04/2019 4:48:51 PM


4.0  Language Skills Review  4.5  Grammar

Example 7

I bought a new dress and handbag, but I don’t like it.


What does ‘it’ refer to here—the dress or the handbag? It’s not clear. It’s best to either state which one is
meant, or use words like ‘former’ or ‘latter’ in order to be specific.
I bought a new dress and handbag, but I don’t like the handbag/the latter.
A pronoun must unambiguously refer to an antecedent and agree with its antecedent in terms of person and
number.
Example 8

When a person goes for an interview, you should always look your best.
Assuming from the context that ‘person’ is the antecedent of the pronouns ‘you’ and ‘your’, the correct
pronouns should be in the third-person singular, just like the word ‘person’.
When a person goes for an interview, he/she should always look his/her best.
Sometimes, a pronoun in certain expressions has no antecedents, especially ‘it’, when used as the subject in
sentences stating the time or weather, and occasionally in other situations as well:
It is 10 o’clock right now.
It started raining heavily.
It was obvious whom he was talking about.
Some More Examples
Incorrect—Me and my friend went to a concert last night.
Correct—My friend and I went to a concert last night.
Incorrect—He took the DVD out of the DVD player and examined it carefully.
Correct—He took the DVD out of the DVD player and examined the DVD/the DVD player carefully.
Incorrect—The workers were happy because we were getting a raise.
Correct—The workers were happy because they were getting a raise.

Tense Consistency Errors


The tense of a verb is the time the action it refers to takes place in. In English, there are three main tenses:
• Present
• Past
• Future
This may seem intuitive enough—but there is more to tense than just that. Each tense can further be
expressed in one of three aspects:
Simple: The Simple aspect is just what its name implies: the simplest form of any tense; it merely shows
that the given action takes place in the particular time indicated by the tense.
Progressive/Continuous: The Progressive aspect indicates that the action is in progress at the particular
time indicated by the tense.
Perfect: The Perfect aspect indicates that the action is complete at the particular time indicated by the
tense.

357

Book 1.indb 357 30/04/2019 4:48:51 PM


NMAT by GMAC™ Official Guide 2019

The following tables show the verbs ‘to be’ and ‘to close’(as an example of most other verbs) in all three
tenses with all three aspects.
To be:

Present Past Future


Simple am/are/is was/were will be
Progressive/Continuous am/are/is being was/were being will be being
Perfect have/has been had been will have been

To close:

Present Past Future


Simple close closed will close
Progressive/Continuous am/are/is closing was/were closing will be closing
Perfect have/has closed had closed will have closed

Tense consistency errors occur when the tense of a sentence does not match the overall context, or when
it changes abruptly in a sentence (in cases when there is more than one verb in a sentence), without any
external reason for the change.
A correct sentence would ensure that the verbs used in the sentence are in the correct tense and are constructed
in parallel form.
Example 9

I see this film yesterday.


If the event occurred ‘yesterday’, i.e. in the past, then the verb should be in the past not present tense. The
correct form should be:
I saw this film yesterday.
Example 10

I ate my dinner and have dessert.


Since the first verb ‘ate’ is in the simple past, then the second verb ‘have’ should also be in the past, as there
is no reason to assume that the two events happen at different times. The correct version can be:
I ate my dinner and had dessert.
Not all tense changes in a sentence are incorrect, however.
If the time-frame of an action changes, then the tense can change accordingly.
Example 11

I am eating my dinner right now, and later I will have dessert.


In the above sentence, two actions are described, both in different tenses (present progressive and simple
future), but the words ‘right now’ and ‘later’ make it clear that they occur at different times. So, there is no
tense error.
The aspect of a verb is also a factor that is prone to errors.
The simple aspect shows a habitual action, while the progressive is restricted to actions that are in progress at a
particular point of time.

358

Chapter 4.indd 358 01/05/2019 4:37:00 PM


4.0  Language Skills Review  4.5  Grammar

Example 12

He was playing cricket in his college days.


The above sentence is incorrect, as he cannot possibly have played cricket constantly throughout his college
days—rather, it was something he did habitually. So the verb ‘play’ has to be in the simple past:
He played cricket in his college days.
On the other hand:

Example 13

He played cricket when the accident occurred.


The above sentence is incorrect for the following reason.
When a specific, one-time event is stated to happen at the same time as another action, that action should be in the
progressive aspect.
So the correct form is:
He was playing cricket when the accident occurred.
Similarly, the perfect aspect is used to show that an action has been completed at a particular point in time or
before another action occurred.
Example 14

We already bought the tickets, when she said she couldn’t come.
The context makes it clear that buying the tickets occurred and was completed before she said she couldn’t
come. So the verb ‘buy’ has to be in the past perfect tense:
We had already bought the tickets when she said she couldn’t come.
Universal facts are always stated in the simple present tense, no matter what tense the rest of the sentence is in.
Example 15

We learnt in Geography class that the South Pole was in Antarctica.


The South Pole is always in Antarctica, i.e. it is a fact that does not change with time. So it should be in the
simple present tense here:
We learnt in Geography class that the South Pole is in Antarctica.
There are many more ways in which tense errors can crop up, and the above are only a few common ones. It
is not possible to list all the situations. But hopefully, you now know enough to be on the lookout for tense
errors.
Some More Examples:
Incorrect—You are telling the same story at every party.
Correct—You tell the same story at every party.
Incorrect—She will leave for London by this time tomorrow.
Correct—She will have left for London by this time tomorrow.
Incorrect—Until just a few centuries ago, people did not know that the earth orbited the sun.
Correct—Until just a few centuries ago, people did not know that the earth orbits the sun.
359

Book 1.indb 359 30/04/2019 4:48:51 PM


NMAT by GMAC™ Official Guide 2019

Modifier Errors
A modifier is a word, phrase or clause which modifies (i.e. says something more about) another word or
phrase in that sentence. Ideally, a modifier must be placed next to the word or phrase it modifies. When
it isn’t placed in this position, especially such that it results in confusion or a bizarre idea, we get a modifier
error.
Example 16

She wore a gold bracelet on her wrist, which cost a lot of money.
The modifying clause ‘which cost a lot of money’ obviously refers to the bracelet not the wrist, so it should
be next to that word. The sentence needs to be rephrased in one of the following ways in order to correct
this error:
She wore a gold bracelet, which cost a lot of money, on her wrist.
She wore on her wrist a gold bracelet, which cost a lot of money.
On her wrist, she wore a gold bracelet which cost a lot of money.
Modifier errors can be broadly classified into two types:
• Misplaced Modifier
• Dangling Modifier

Misplaced Modifier
The above example is of the former type, i.e. one in which the modifier has been placed incorrectly in the
sentence, and simply shifting it into the correct position can correct the error.
Here’s another example of misplaced modifier, in which the modifier is a single word.
Example 17

Once I moved to the front of the classroom, I could hear the professor speaking clearly.
In the above sentence, what does the adverb ‘clearly’ modify? The verb ‘hear’ or ‘speaking’? It most likely
modifies ‘hear’, but the sentence is still ambiguous. Moving the modifier next to the verb it modifies makes
the sentence much better:
Once I moved to the front of the classroom, I could clearly hear the professor speaking.

Dangling modifier
Dangling modifier errors are more complicated, as in those cases, the modifier does not apply to any word
directly stated in the sentence, as that word is only implicit in the context.
Example 18

Thinking about his life so far, there seemed to be so many mistakes.


Who is thinking about his life in this sentence? We can assume there is a ‘he’ involved, but it is not stated
in the sentence. The sentence needs to be rephrased thus to include the name of the person doing the
thinking, or a suitable pronoun:
Thinking about his life so far, he seemed to have made so many mistakes.
Please note that correcting this error also involves rephrasing the second half of the sentence substantially.

360

Book 1.indb 360 30/04/2019 4:48:51 PM


4.0  Language Skills Review  4.5  Grammar

Some More Examples


Let’s see a few more examples of each of these types of errors and how to correct them.

Misplaced modifiers:
Incorrect—I saw a gift that would be perfect for my cousin in the shop window.
Correct—In the shop window, I saw a gift that would be perfect for my cousin.
Incorrect—We go to the beach to watch the waves crash on the rocks every Saturday.
Correct—Every Saturday, we go to the beach to watch the waves crash on the rocks.
Incorrect—He accidentally hit his brother with a bat in the head.
Correct—He accidentally hit his brother in the head with a bat.
Dangling modifiers:
Incorrect—While serving dinner, the plate fell from her hands and broke.
Correct—While she was serving dinner, the plate fell from her hands and broke.
Incorrect—After waiting for an hour at the bus stop, there was still no sign of the bus.
Correct—After waiting for an hour at the bus stop, I/he/she/we/they could still see no sign of the bus.
Incorrect—After three months in the desert, this wet weather is quite a refreshing change.
Correct—After three months in the desert, I/he/she/we/they find/s this wet weather to be quite a
refreshing change.
Note: The categorisation of modifiers into these two types is only in order to better acquaint you with the
range of modifier errors. Please note that you will never need to differentiate between the two in grammar-
related questions in aptitude tests. The differentiation is for your own knowledge only.

Parallel Construction Errors


Parallel Construction refers to the structure of a sentence in which certain words, phrases or clauses that
have a similar logical/grammatical status are expressed in a similar manner. When these words, phrases or
clauses—i.e. the parallel elements—are not expressed in the same grammatical form, you get a parallel
construction error.
Example 19

In my free time, I like to read, watch TV and playing video games.


The sentence involves three activities of the same status—i.e. that are parallel. So they all should be in the
same grammatical form. The first two parallel verbs—‘read’ and ‘watch’—are in the infinitive form. But
the third one—‘playing’—is in the gerund/present participle form. It should also be in the infinitive, i.e.
‘play’. This is how the sentence should be correctly framed:
In my free time, I like to read, watch TV and play video games.
In grammar, it is essential that all parts of speech, i.e. nouns, verbs, adjectives, adverbs, etc., as well as larger parts,
like phrases and clauses be constructed in a parallel form in the sentence.
For example, here’s one with nouns:

361

Book 1.indb 361 30/04/2019 4:48:51 PM


NMAT by GMAC™ Official Guide 2019

Example 20

Please remember to bring your textbooks, pens and your notepads.


In this case, ‘your’ already applies to all the three items, i.e. ‘textbooks’, ‘pens’ and ‘notepads’, so there is no
need to repeat it before the last one.
Please remember to bring your textbooks, pens and notepads.
Here’s another example involving clauses:
Example 21

Either you should exercise more or eat less if you want to lose weight.
In case of correlative conjunctions (i.e. conjunctions that appear only in pairs) like ‘either ... or’, ‘neither ... nor’,
‘not only ... but also’, etc., the parts of the sentence after each one should be parallel.
So in this case, ‘either’ should come before ‘exercise’, as it is parallel to ‘eat less’, which comes after ‘or’:
You should either exercise more or eat less if you want to lose weight.
Not all sentences which include lists involve parallel construction, however. Beware of over-generalizing!
For example, see the following sentence. Is it correct?
Example 22

The maid washed the dishes, the clothes, and dusted the furniture.
This sentence is incorrect, because the three activities are not parallel. ‘The dishes’ and ‘the clothes’ are
parallel, as they are the objects of the same verb ‘washed’; but ‘dusted the furniture ’involves a separate verb
and is therefore a separate activity. This is the correct form of the sentence:
The maid washed the dishes and the clothes, and dusted the furniture.
Parallel construction is also important when comparing two or more things. The compared items must be
parallel, so it is vital to ask exactly what is being compared to what.
Example 23
The population of my city is much greater than the capital city.
In the above sentence, the population of one city is being compared not to the population of another, but to
the city itself ! Obviously this is incorrect, and the correct form should be:
The population of my city is much greater than that of the capital city.
Some More Examples
Let’s see some more examples of parallel construction errors and how they can be corrected.
Incorrect—We are going for a hike and rock climbing.
Correct—We are going for hiking and rock climbing.
Incorrect—This meeting is not about finding fault, but finding a solution.
Correct—This meeting is not about finding fault, but about finding a solution.
Incorrect—This flat is nice, but the view is not as pretty as the flat we saw yesterday.
Correct—This flat is nice, but the view is not as pretty as that of the flat we saw yesterday.

362

Book 1.indb 362 30/04/2019 4:48:52 PM


4.0  Language Skills Review  4.5  Grammar

Idiomatic Errors
Certain nouns or verbs can be used only in a set idiomatic pattern with certain prepositions or adverbs in
order to express new meanings or even completely different meanings. Hence, it is important to learn the
use of such set of word. Prepositions in conjunction with words are particularly notorious in this regard, as
their usage is often not in keeping with their more common meanings.
These are quite a few errors you need to remember.
For example, are the following sentences correct?
Example 24

I am interested into languages.


He is turning in a tyrant.
The prepositions in both sentences—i.e. ‘into’ and ‘in’—have similar meanings in themselves, and so may
seem equally valid in either sentence. But as a matter of fact, ‘into’ cannot be used after ‘interested’ and ‘in’
cannot be used after ‘turn’ (in this context, at least); the other way around, however, is completely correct:
I am interested in languages.
He is turning into a tyrant.
Certain verbs can form an idiomatic phrase with certain prepositions or adverbs in order to express new
meanings or even completely different meanings. Such verbs are called phrasal verbs, and they are a very
common feature of English. Many verbs can take a multitude of different prepositions, and express a
different meaning with each. For example, does the verb ‘stand’ have anything to do with standing in the
following sentences?
Example 25

His strange new hair style makes him stand out from the crowd.
She promised to stand by her husband, no matter what difficulties he faced.
‘Stand out’ means to be conspicuous. ‘Stand by’ means to support. Neither meaning has anything to do with
standing, but in combination with ‘out’ and ‘by’ respectively, the verb ‘stand’ takes on whole new meanings.
These meanings are idiomatic, in the sense that they cannot be predicted from the constituent words.
Idioms can also be certain set phrases that have a figurative, not literal, meaning. For example, in the
following sentence, does the phrase ‘by heart’ have anything to do with the heart?
Example 26

She has read that book so often that she has almost learnt it by heart.
To know or learn something ‘by heart’ means to memorise it thoroughly. This meaning, clearly, has nothing
to do with the literal meanings of ‘by’ or ‘heart’. Similarly, cats and dogs have nothing to do with rain, yet
the idiomatic phrase ‘cats and dogs’ is used to signify heavy rain, as in the following example:
It’s just a light drizzle right now, but soon it will be raining cats and dogs.
Some More Examples
Incorrect—Do you think my new hairstyle is very different than my old one?
Correct—Do you think my new hairstyle is very different from my old one?

363

Book 1.indb 363 30/04/2019 4:48:52 PM


NMAT by GMAC™ Official Guide 2019

Incorrect—He prefers rock music above classical music.


Correct—He prefers rock music to classical music.
Incorrect—She was glad enough to come first in her class, but winning a scholarship as well was the iced
cake.
Correct—She was glad enough to come first in her class, but winning a scholarship as well was the icing on
the cake.

Quick Tips and Strategies


These are some guidelines that can be helpful when solving grammar-related questions:
• While reading the original sentence, be careful of verbs, prepositions, pronouns and modifiers.
• Whenever you see a verb, find its subject, see whether it agrees in number and in person.
• Whenever you see a pronoun, find whether it has a clear antecedent and if it does whether it agrees
in gender and number. Also check the case of the pronoun - it should be in correct form as per its
case.
• Modifiers and parallel construction sentences can be spotted by the use of commas. Be careful about
these errors in sentences with commas.
• When you have to identify correct or incorrect sentences, it is always safer to focus on finding errors.

Types of Questions
Grammar-related multiple-choice questions on the NMAT by GMAC™ can be broadly categorised into
two main types:
Type 1: Where you have to simply identify which part of a sentence is grammatically incorrect;
Type 2: Where you identify the correct preposition that will complete the sentence.

Identifying the Error


The basic format is: 4/5 sections of a sentence are underlined, some of which may have grammatical errors.
You have to identify which of the numbered parts is/are grammatically incorrect.
These questions test not only grammar and usage, but also logical consistency. Please note that ‘logical
consistency’ only means that the sentence/s should make logical sense and not have internal contradictions.
It does not mean that you need to analyse the logic in any detail as you would in a critical reasoning
question.
Let’s see some examples now.
Directions for examples 1-3: Each sentence below has four underlined words or phrases. Identify the one
underlined word or phrase that must be changed in order to make the sentence correct. Mark E for no
error.
Example 1


Cristina starting the test later than the rest of the students but was still able to complete it in the
allotted time.
(A) starting the
(B) later than
(C) but

364

Book 1.indb 364 30/04/2019 4:48:52 PM


4.0  Language Skills Review  4.5  Grammar

(D) it
(E) No error

Solution

The first clause of the sentence—Cristina starting the test later than the rest of the students—does
not contain a verb whereas a verb is needed here. Starting, which is a participle, needs to be change to
started the, the verb.
The correct answer is A.

Example 2


Priscilla and I was punished by the teacher for not completing the assignment on time.
(A) Priscilla and I
(B) was punished
(C) not completing
(D) on time
(E) No error

Solution

The plural subject Priscilla and I does not agree with the singular verb was. Thus, B is the error—the
singular was needs to be changed to the plural were.
The correct answer is B.
Example 3


While some may doubt the feasibility of the proposal, it is based on empirical evidence, unlike policies
that result from either fanciful suppositions or from political whims.
(A) doubt the feasibility
(B) it is based
(C) unlike policies that
(D) from political whims
(E) No error

Solution

The sentence has an error of parallel structure. Whatever comes after either, the same construction has
to be repeated after or. Since either is followed by a noun phrase, or should also do the same. Instead
or is followed by the preposition from.
The correct answer is D.

Choose The Correct Preposition


In this question type, you will be given three separate sentences with one blank in each. Below these
sentences, you will be given 6 prepositions labelled (a) to (f ). Finally, you will be given five answer choices
with some possible combinations of these prepositions. You need to identify the combination that can
correctly go into each of the three blanks.

365

Book 1.indb 365 30/04/2019 4:48:52 PM


NMAT by GMAC™ Official Guide 2019

Let us take a look at an example:


1. We walked ______ the river bank until we found our clothes.
2. Jaggu was accused ______ cheating in the exam.
3. The MD apologised ______ the CEOs poor behaviour.
(a) in
(b) along
(c) of
(d) to
(e) for
(f ) by

(A) bce
(B) cbe
(C) ecd
(D) dce
(E) fce
Solution

You walk along the river bank.


You are accused of something.
You apologise for something.
The correct answer is A.

As you can see, these questions are not testing you on any specific grammar rule as such; rather, they are
testing you on the usage of words and phrases. It again helps if you have been in the habit of reading as
you may then be able to recall the correct use of these prepositions. If you are not in the habit of reading, it
makes sense to start reading a good newspaper or magazine.
In terms of the concept knowledge required, grammar-based questions are possibly the most difficult type
of questions in aptitude tests. But don’t let that put you off. Getting good at grammar is worth the effort, as
it not only helps you out in grammar-based questions—which are a regular feature in all aptitude tests—
but also helps you gain a firmer grasp on the English language itself, which in turn will help you with
reading-based questions, as well as with your spoken English.

366

Book 1.indb 366 30/04/2019 4:48:52 PM


Book 1.indb 367 30/04/2019 4:48:52 PM
4.8 Comprehension

Book 1.indb 368 30/04/2019 4:48:52 PM


4.0  Language Skills Review  4.8  Comprehension

4.9 What is Measured?


The comprehension section will test your ability to read, comprehend and interpret unfamiliar content and
to answer questions about the same.
You will be tested on your understanding of the English language, your word power and your ability to
comprehend jumbled up text.

4.10 Overall Test Taking Strategies


• While reading passages, focus on the comprehension aspect and not on the speed aspect.
• Do not panic if the passage looks lengthy or difficult to comprehend. To ensure you are able to
make best use of time, make use of the strategies provided in the following section.
• Try to make a passage map for every passage and predict the topic and purpose before you attempt
the questions.
• Do not use your knowledge or common sense to answer questions. Only answer on the basis of
what is given in the passage.
• W  hile attempting parajumble questions, try to create a link between two options and eliminate
wrong answer choices.

The next few sections will provide you with in-depth strategies for approaching each topic.

369

Book 1.indb 369 30/04/2019 4:48:52 PM


NMAT by GMAC™ Official Guide 2019

1 Introduction
Comprehension or the ability to make sense of some randomly assigned block of text will primarily be
tested on Reading Comprehension question type on the NMAT by GMAC™. Reading Comprehension
(RC) questions will test you on your understanding or comprehension of unfamiliar texts from long
passages. You will likely see one to two RC passage in the verbal section of the NMAT by GMAC™ with
5-10 questions asked from it. The passages are typically 400-600 words in length though they can at times
be longer or shorter.
In Reading Comprehension, students have to read passage(s) and answer questions that follow the passages.
This area tests the ability of the student to quickly grasp what is being said in the passage before answering
the questions. Some of the questions test your ability to recognise implications and draw inferences. Others
test your ability to understand and critique the ideas and information provided in the passage; while some
might require an application of the logic used in the passage.
The difficulty level of the questions in the section can be easy/medium or difficult, with several questions
being direct lifts from the passage. The trick lies in enhancing both the speed and range of your reading in
order to be equipped with the skills required to attempt this paper.
Let us begin with a quick revision of the basics.

2 Reading Comprehension
Speed Reading
What is speed reading?
Speed reading is essentially a method of reading rapidly by taking in several words or phrases at a glance or
by skimming. Speed reading increases the reading rate and furthers comprehension of the text.

Skimming and Scanning


Skimming and scanning are two very different strategies for speed reading. They are each used for different
purposes, and they are not meant to be used all the time.
Skimming refers to looking only for the general or main ideas, and works best with non-fiction (or factual)
material. With skimming, your overall understanding is reduced because you don’t read everything. You
read only what is important to your purpose.
Unlike skimming, when scanning, you look only for a specific fact or piece of information without reading
everything. You scan when you look for your friend’s phone number in the telephone list, and for the
sports scores in the newspaper. For scanning to be successful, you need to understand how your material
is structured as well as comprehend what you read so you can locate the specific information you need.
Scanning also allows you to find details and other information in a hurry.
How do you raise your reading speed comfort level? First, you need to learn how to be uncomfortable! You
need to enter your discomfort zone.
You know you’re in your reading discomfort zone when you get an uneasy feeling when you’re trying
something new. Most new speed readers feel it the first few times they try to read fast and realise their
comprehension isn’t what it should be. This uneasiness is expected, necessary for the learning process, yet
temporary. This is due to the fact that the human brain needs a little time to adapt to these increasing
speeds of comprehension and reading. Using this technique can help a person increase their comprehension
and reading speed anywhere from 20 percent to a 100 percent.

370

Book 1.indb 370 30/04/2019 4:48:52 PM


4.0  Language Skills Review  4.8  Comprehension

Speed reading is about using reading strategies and also about having a speed reading mind-set. It
means believing you can read faster and you will read faster. It means not being overly concerned about
comprehension at first, but knowing that it will follow when your eyes become adept at picking up
information in a new way.
If you find yourself in your discomfort zone and want to re-enter the comfort zone with faster speeds under
your belt, here are a few ideas:

Skip Small Words


Each word requires a different amount of time to process. In fact, some words are so intuitive; they can be
skipped without losing the meaning of the sentence. Most native English readers do this instinctively to
some degree, but this technique can be improved through practice.

The Underlining Hand Motion /Pointer Method


This is a great beginner technique to force your eyes to adapt to a faster reading speed. In this technique we
use a pen/pencil to trace the words we are reading, at a slightly faster pace. The pen/pencil acts as a guide
and forces your eyes to follow the words at an enhanced speed.

Eliminate Vocalisation/Sub-Vocalisation
Vocalisation/Sub-Vocalisation is pronouncing the words while you read, either audibly or inaudibly. This
habit is a problem because it invariably slows down your speed for a simple reason – your speed is now
a function of how fast you read out the words, which will not exceed 200 words per minute; an average
reading speed.
A simple exercise that an individual can try to stop subvocalizing is to use their inner voice by humming or
counting either in their head or out loud while reading at their normal rate. People will immediately notice
that they can comprehend the text much faster even though their ability to subvocalise has been blocked.

Regression
The tendency to go back to the parts of the passage or re-read the text is called regression. Regression
unnecessarily slows you down and also affects your comprehension as result of frequent jumps in your
understanding while reading.
Readers should focus on reading a passage completely even though they feel that their concentration has
temporarily wandered or they have missed something. Remember that the brain is very good at filling in
learning gaps. Staying focused on the line of words that is currently being read before continuing can also
increase their comprehension of the text and its meaning. Additionally, important concepts are frequently
repeated in a text all the time. Reading and comprehension speed is cut down by at least 30 percent if a
person stops or regresses more than two times in every row.

Vocabulary Increase
Reading speeds and comprehension will increase if the reader takes time to understand the words that
they are reading. If a person cannot understand the meaning of the words in a text, their reading speed will
decline. This is because a reader will need to pause in order to figure out what they are reading. Additionally,
learning words often requires a quick look in the dictionary.

Practice
A person cannot improve their comprehension and reading skills without constant practice. This is always
important for a reader to continue developing their ability to comprehend more and read faster. The most
effective way of achieving sufficient training in processing information and reading at a faster rate is to

371

Book 1.indb 371 30/04/2019 4:48:52 PM


NMAT by GMAC™ Official Guide 2019

practice. One way to make reading practice more interesting is to find a text that a person likes to read. This
step is also vital for training the reader’s eyes to skim faster across a page. Finally, always bear in mind that
there are no limits to how much a person can improve their reading comprehension and reading speed. All
it takes is practice.

Eclectic Reading
You must increase your scope of reading. Reading across areas is vital for competitive examinations.
Familiarity with different topics would help you comprehend the passage easily and would provide you a
cutting edge in solving Reading Comprehension questions.

Active Reading
To make the most of time you spend studying, you need to read actively.
Active reading simply means reading something with a determination to understand and evaluate it
for its relevance to your needs. It is different from passive reading. It is an important skill that aids your
comprehension. An active reader applies certain questions and theories that help enhance clarity. It involves
applying logical and rhetorical skills.
If you just read passively, then you are dealing only with the superficial aspect of the text. This allows you
to answer certain types of questions. But typical RC passages represent a wide variety of questions and in
order to tackle all of them you need the ability to read critically.
Active reading enables a reader to comprehend the underlying structure – tone, inference and organisation
that allows him/her to answer all types of questions.
Try these techniques to make your reading active:

Keywords
Make note of key words and phrases as you read. This will assist you in identifying the ideas and developing
an ability to understand the link between them. Some ways to identify these are –
Correlating sets of words
Correlating words refer to the use of synonyms, antonyms, nouns, adverbs, verbs and adjectives to describe
ideas and/or arguments in the passage. Identifying the related words helps you come closer to the main idea
of the passage. Be selective - too much highlighting won’t help.
Connecting words and their function
Certain words function as a link between various ideas expressed by the author. These words are of high
importance as they bring in the contrast /link between the ideas, opinions and thoughts expressed in the
passage. Link words are many and can be categorised in terms of the function they serve in the overall
progression of the thought process.
Cause – Because, Thereby
Consequence – Thus, Therefore
Time Sequence – While, When
Concession – Nevertheless, However
Opposition – But, Against
Ideas That Recur
Just like similar words are used to re-emphasize an idea, ideas reappearing in a passage reiterate a point.
Authors use this technique to build the main idea or concept. This skill, to realise when an idea is being

372

Book 1.indb 372 30/04/2019 4:48:52 PM


4.0  Language Skills Review  4.8  Comprehension

reiterated through examples, analogies and highlighting different aspects of the same phenomenon etc…
is critical to getting close to the main idea of the passage. Look for "guiding words/phrases" that help you
understand the text - phrases like ‘most importantly’, ‘in contrast’, ‘on the other hand’.
Analyse the Argument
While reading, use the margins to make notes - you may do so by paraphrasing the key points, question the
author's claims, jot down data , examples or important information and so on. Since this process requires a
more in-depth interaction with the passage, your retention and understanding will be enhanced.
Critically analyse the text by asking questions such as - When and by whom was the text written? Who
was the target audience and so on.
Keeping Pace with Inferences
Inferences use logical thought to arrive at a point. Every piece of text progresses in a logical manner and
these areas in a passage are fertile ground for questions, especially, inferential questions, which judge
the readers’ ability to comprehend the reasoning of the author. Some common ways in which an author
proceeds to build inferences are – contrast, cause, consequence, purpose and explanation.
Pay Attention to the Opening Paragraph
The manner in which the author introduces the text reveals a lot about the writers’ attitude, aim, viewpoint
or approach. Writers use different ways to introduce texts. For example, a text can be introduced with the
help of analogies, anecdotes, examples, contrasts etc. An analogy is a comparison in which an idea or a
thing is compared to another thing that is quite different from it. It aims at explaining that idea or thing
by comparing it to something that is familiar. An anecdote is a short account of an amusing or interesting
incident which helps enliven the text.

Nature of Passages
Usually, the passages can be from any subject area. For example, you can get a passage discussing the l
aunch of a new Indian satellite or a passage detailing some new development in the Indian banking sector
and so on.
Keep in mind that the passages will not always be interesting or fun to read; as a matter of fact some of
them will be downright boring and difficult to understand. The language of the passages will be similar to
what you are likely to see in publications such as The Hindu or The Economist.
The problem most students face on RC is that they have to go through text from areas they aren’t
conversant with and answer questions based on this. The moment you see a passage from an unfamiliar
area you immediately start telling yourself that you will do badly on this passage because you have no
idea about the subject area. If you start with this negative thought process, things will obviously only go
downhill for you.
Please keep in mind that you are not expected to have any prior knowledge of the topic in the first place.
All the information that you need to answer the questions is given to you in the passage. You just need to
comprehend the passage and select the correct answer from the options provided.

Make a Passage Map


A good way of approaching a passage, especially a long one, is to make a map of the passage. A passage map
is nothing but one or two lines for every paragraph in the passage, highlighting why the author has written
that paragraph. Note that the passage map does not have to highlight what the author has written but
rather why he has written, what he has written, which means that the points in your passage map should

373

Book 1.indb 373 30/04/2019 4:48:52 PM


NMAT by GMAC™ Official Guide 2019

always start with verbs such as describe, explain, praise, criticise, condemn, etc. Most of the time you will
find this information in either the first or the last sentence of each paragraph.

Identify Topic and Purpose


Apart from the passage map, there are two more things you need to be absolutely clear about before you
look at the first question – The Topic and the Purpose of the passage.
The Topic of the passage is nothing but a word or a phrase that captures the essence of the passage. The
topic tells you what subject matter the entire passage revolves around. You may even get a question asking
you to identify the topic of the passage from the given options.
The Purpose is the most important part of the passage and will answer the question – why did the author
write the entire passage? When thinking about the purpose, think on three lines – is the author positive i.e.
is he trying to praise or support something, is the author negative i.e. is he trying to criticise something, or
is the author simply neutral i.e. is he just describing or explaining something. Obviously this will also clarify
the tone of the passage for you. If you are clear about the purpose of the passage, you will not have to keep
referring back to the passage to check each option; rather you will be able to eliminate a lot of the options
just by looking at them because they contradict the author’s primary purpose.

Important Learning:  It is very important to keep in mind whether the overall tone of the passage is
positive, negative or neutral. Else you will get some easy questions wrong.

Engage with the Passage – Do not read Passively


While reading, try to engage with the passage. This will also help prevent your concentration from
wandering. The best way to get yourself involved with the passage is to try to predict what will come next in
the passage. When you do this, you are essentially putting yourself in the author’s shoes and thinking like
him, which will help you get a great understanding of the passage.
You can use the last sentence of a paragraph to predict what will come in the next paragraph. For example,
if the paragraph ends by stating that scientists have proposed a solution for a problem, the next paragraph
will most probably provide you with the details of this solution. Once you see that most of your predictions
are turning out to be correct, you will find it more fun to read the passage and you’ll also notice an increase
in your confidence levels as you go about tackling the passage.

Make Use of Transition words


While making a passage map or generally reading a passage, try to make use of transition words to
understand the overall structure of the passage and also to predict what will happen next in the passage. For
example, if the author starts a paragraph with the words Similarly or Likewise, then you immediately know
that whatever he has described in the earlier paragraph, the same thought process will continue in this
paragraph as well.
Contrastingly, if the author is praising something in a paragraph, and the next paragraph starts with the
words However or Despite, then you immediately know that the author will now talk about some negative
or contrasting aspect of that thing. Transition words will make it very easy for you to understand the broad
structure of a passage; these will mostly be found in the first and the last sentence of a paragraph, so pay
attention to these sentences.

Important Learning:  Always pay extra attention to the first and last sentence of every paragraph as
these contain the most important points.

374

Book 1.indb 374 30/04/2019 4:48:55 PM


4.0  Language Skills Review  4.8  Comprehension

Types of Reading Comprehension Passages


One should pay attention to the organisation of the passage, the structure and logical flow of ideas. Along
with this, an understanding of the different types of Reading Comprehension passages—whether they are
argumentative, discursive, descriptive or analogous in their construction shall also help a candidate develop
effective strategies to approach these passages. Certain key features of the passage structures are:
Argumentative Passages
These passages present the arguments made by the author with respect to a particular subject. In such
passages the author presents arguments and counter arguments with the assistance of arguments, facts
and information. These passages are logic driven in their construction and elucidation. Questions in such
passages demand an understanding of the logical structure of the passage and inferences based on the same.
Discursive Passages
In such passages, the author presents his/her ideas and opinions and reinforces the same with the use of
logic and data.
Descriptive Passages
Descriptive passages are generally narrative in nature. They include a description of an event/activity/
phenomenon. In these passages, techniques of skimming and scanning might be of assistance as the
questions will be more factual in nature and can be located easily.
Analogy based Passages
These passages draw comparisons between two unrelated objects or points of reference. These passages
demand an in-depth understanding of the analysis and comparisons being drawn. Hence, you are required
to analyse the structure and draw inferences.

3 NMAT by GMAC™ Reading Comprehension Question


Types
Global questions
Global questions are questions that cannot be answered without reading the passage in totality, that is, the
answer is not explicitly written in the passage. The main purpose/primary concern type questions will fall
under this question type. Here are a couple of examples:
• What is the main purpose of the author in writing the passage?
• What is the primary concern of the third paragraph in the passage?
Note that a passage will never tell you what its main purpose is. However, if you have made a good passage
map, then you should not have much difficulty in answering this question. Also remember that, since the
answer is not given in the passage, you do not really need to go back to the passage to answer a Global
question. This will also save you time.
A trick to answering Global questions in NMAT by GMAC™ is to make use of the fact that each
of the options in such questions will start with a verb, which will have a positive, negative or neutral
connotation. Thus, if you are clear on the author’s tone, you should be able to eliminate two or three
options immediately just by looking at the first word of every option.

375

Book 1.indb 375 30/04/2019 4:48:55 PM


NMAT by GMAC™ Official Guide 2019

For example, if you know that the author’s tone is neutral, then options that start with words, such as
arguing, praising, criticising and so on, will never be correct because these have either a positive or a negative
connotation. The correct answer in this case would start with neutral words such as describe, explain, analyse
and so on.

Important Learning:  The answer to a Global question is never given in the passage, so do not waste
your time searching for the answer in the passage.

Detail questions
As the name suggests, Detail questions will ask you questions related to what is explicitly mentioned in
the passage. In that sense, they are the opposite of Global questions, as the answer to these questions will
always be stated in the passage. So, make sure that you go back and read the answer from the passage before
selecting an option. This is where a good passage map comes in handy while tackling long passages because
you do not have to waste time trying to find the answer in the entire passage. Here are two examples of
Detail questions:
• Which of the following is provided by the author as an example of reverse osmosis?
• Each of the following is mentioned in the passage as a side effect of medicine EXCEPT:

Use of EXCEPT on Detail questions


Since the answer to a Detail question is always written in the passage, it might seem that these questions
would be relatively easier to answer. However, the test maker has a way of making these questions confusing
and lengthy by the use of words such as ‘EXCEPT’.
For example, a question may state that According to the passage, each of the following is true of a steam turbine
EXCEPT.
In this case, four of the options will be mentioned in the passage and you will need to identify the fifth
option that is not mentioned in the passage. Thus, you need to check each of the options against the
information in the passage, which makes the entire exercise take longer than would a regular question.
Also, the wording of the options will be confusing, in the sense that there will not be that one option which
contains words that have never been mentioned in the passage (thereby, making it easy for you to identify
this as the correct answer). Rather, all the options will contain keywords from the passage; it is just that one
of them will convey some incorrect information about the keyword. Thus, make sure that you read every
option with a critical eye.

Vocabulary-in-context questions
These questions will ask you to identify the meaning of a word or a phrase as used in the passage. The
keyword here is in context, that is, the answer always has to be with reference to the passage. The dictionary
meaning of the word will rarely be the correct answer; in fact, this is one of the most common wrong
answer traps.
To answer these questions correctly, go back and read the sentence which contains this word or phrase.
Then, read one or two sentences before and after this sentence to get an idea of the context. Now, look at
each option and eliminate. Here are a couple of examples of vocabulary-in-context questions:
• What is the meaning of the word ‘explosion’ as used in the passage?
• The word ‘cynosure’, as used in the passage, is closest in meaning too.

376

Book 1.indb 376 30/04/2019 4:48:56 PM


4.0  Language Skills Review  4.8  Comprehension

Important Learning:  The answer to a vocabulary-in-context question will almost never be the literal or
dictionary meaning of the word.

Function questions
Function questions will ask you to identify the function of a word, a sentence, a paragraph, a punctuation
mark and so on in the context of the overall passage. These are essentially Why questions, that is, they
will ask you why the author uses a particular word or sentence in the passage. To answer these questions
correctly, you will need to put yourself in the author’s shoes and think like the author. Obviously, the
understanding of the main purpose of the passage becomes crucial while answering these questions.
Please remember that the question is not asking you for your opinion on something, as this is what students
end up providing most of the time. It is asking you for the author’s rationale behind writing something in
the passage.
Function questions will either start with the interrogative Why or they will end with the phrase in order to.
Here are a couple of examples of Function questions:
• Why does the author provide the example of the atomic clock in the passage?
• The author provides the example of the atomic clock in the passage in order to.

Inference questions
The dictionary meaning of the term ‘Inference’ is to derive by reasoning, and this is exactly what you will
be required to do on Inference questions—arrive at an answer that is not explicitly stated in the passage,
but that can be definitely concluded given the information in the passage. So, Inference questions will
require you to arrive at an answer that can be concluded or stated, based on the information provided in the
passage. The answer to these questions will never be directly stated in the passage.
Inference is an important question type tested on NMAT by GMAC™ Reading Comprehension, and
is also the question type on which students make the most mistakes. This is because students often tend
to read too much between the lines, that is, they end up over-inferring from the passage. Hence, make it
a point to avoid strongly worded or extreme-sounding options—options containing words such as must be
true, always be the case, never be the case, cannot be determined and so on—and go with more open ended and
vague options—options containing words such as usually, sometimes, possibly, might be true and so on. Here
are a couple of examples of Inference questions:

• Which of the following is implied by the author in the fourth paragraph?


• Which of the following options would the author of the passage most likely agree with?

Important Learning:  While attempting Inference questions, always avoid extreme or strongly worded
options.

377

Book 1.indb 377 30/04/2019 4:48:58 PM


NMAT by GMAC™ Official Guide 2019

3 Tips for attempting Reading Comprehension passages


i. Keep a track of timelines/statistics – If you notice that the passage is talking about different
time periods, then make sure you keep a track of this chronology as you will almost invariably be
questioned on this. Similarly if you see a lot of statistics or numbers mentioned in the passage, you
will most likely be questioned on these.

ii. Don’t use outside knowledge, your opinion, or common sense – All the information you require to
answer any RC questions is already given to you in the passage. You should NEVER make use of any
outside information or common sense while answering questions. Even if you disagree with what the
passage states, don’t let this cloud your judgment. Approach the passages very objectively.

iii. Read the questions carefully – After reading the passage, when you start attempting the questions,
make sure that you read every question carefully. This is specially the case for TRUE/NOT TRUE
type questions in which you can easily mistake one for the other and get your answer totally wrong.

iv. Read all the answer choices – Even if you like one particular answer choice, make sure you read
all the answer choices after it as well because it is possible that there is another answer choice that
answers the question even better than the one that you liked initially.

v. Try to eliminate some options – In case you aren’t able to arrive at the answer to a question, at least
try to narrow down your choices by eliminating some options which you know are definitely incorrect.
Its very unlikely that you will be confused amongst 4 or 5 options. Usually, you will easily be able
to eliminate 2 or 3 options after which, even if you make a blind guess, you have a 50% chance of
selecting the correct answer.

4 Para Jumbles
Para jumbles are jumbled paragraphs. Basically, you are given a paragraph - but the sentences are not in the
right order. It is now up to you to rearrange these sentences to form a coherent, logical paragraph.
This type of question is quite common in management entrance tests.
Para jumbles are not necessarily a test of your language skills. Questions on parajumbles are designed to
assess your reasoning rather than reading comprehension. In parajumbles, we are essentially given some
statements which have to be put in a coherent sequence. Hence, fundamentally, we are trying to solve a
verbal puzzle using clues in the form of mandatory pairs.
This means that even if you are not conversant with the idea being discussed or familiar with the topic on
which the paragraph is based, you can still easily resolve the jumble with high accuracy levels.

Para Jumble Concepts


Here are a few things NOT to be done while attempting these questions –
1. Reading the sentences and trying to resolve the jumble on the basis of the flow of ideas. While this
approach makes sense intuitively, the paragraph might be a tricky one wherein the flow of ideas might
not be easily discernible. Most people lose focus by reading the statements given over and over again.
Sometimes, it is next to impossible to make out which sentence follows which one.
2. Not attempting a para jumble because the topic on which the paragraph is based is from a genre that
you are not conversant with.

378

Book 1.indb 378 30/04/2019 4:48:58 PM


4.0  Language Skills Review  4.8  Comprehension

3. Reading the sentences in the order of all the combinations provided. Generally, every jumbled
paragraph has four options – each option suggests a different sequence for resolving the jumble. While
it is tempting to read the sentences in all the possible combinations, doing this will not only confuse
you but, after some time, every combination will start appearing correct! Also, solving all options will
waste your time tremendously.
The best way to solve a parajumble is to try and identify Mandatory Pairs. A mandatory pair is a sequence
that you know cannot exist in any other order. In other words, 2 or more statements that have to be
grouped together, if we are following the rules of the language, constitute a mandatory pair.
There are many ways of identifying mandatory pairs. But there is only one basic approach to identify them
- look at parajumbles like a detective and search for clues that the thief or in this case, the papersetter has
left for you (Keywords). Then, just like a detective, use these clues to form a sequence or connection and
complete the chain!

IDENTIFICATION OF MANDATORY PAIRS


Names, Proper Nouns, Pronouns
Look at the statements given below:
A. Mr. Kumar checked the quality of food and asked flood victims about the help from the government.
B. He also instructed officials to focus more on the quality of food, and sanitation for women, at the
camps.
C. Chief Minister Nitish Kumar and Minister for Water Resources Rajiv Ranjan Singh and other
officials inspected several relief camps in Patna district, from Maner to Athamalgola.
What, according to you, should be the sequence of the statements given? Obviously, the answer is CAB.
Let us understand why. Statement c contains the full name of the person – Nitish Kumar. Statement a
contains the surname – Mr. Kumar, while statement b introduces the personal pronoun – he. Thus, the
correct sequence is CAB.
Sometimes, we can identify mandatory pairs or a longer sequence with the help of the names, proper
nouns and pronouns used. Also, keep in mind that English demands the presence of an antecedent for a
pronoun, that is, if a pronoun has been used; it needs to refer to a noun or another pronoun. Identifying the
antecedent can also help identify the mandatory pair.
The use of personal pronouns (I, me, you, us, he, they, it etc…) and demonstrative pronouns (this, that,
these, those) is extremely helpful in resolving parajumbles.

Cause and Effect


There are instances where a cause and effect relationship can be identified. Such a relationship may exist in
the form of a mandatory pair or run through the paragraph. There are some cause-effect indicators (refer
the table). The presence of any of these words acts as a clue to identify a cause and its effect, which shall
help us form a mandatory pair.

SHOWING CAUSE For, As a result of…, Because …, Due to (the fact that), In order to …, Resulting
from ….., Since…
SHOWING EFFECT So, Accordingly, As a result, Consequently, Hence, Therefore, Thus, (If ) … then, …
resulting in, So …, that…, then
SHOWING CONDITION Even if, If … then, In case, Provided that, Providing, Unless, Whether or not

379

Book 1.indb 379 30/04/2019 4:48:58 PM


NMAT by GMAC™ Official Guide 2019

Look at the statements given below:


A. Otherwise the BJP would not have opposed FDI in retail today.
B. It is evident that there is no agreement on economic reform
C. Nor would allies of ruling Congress oppose disinvestment.
D. All this will have a detrimental impact on India's journey towards becoming a world leader.
A. ABCD
B. ABDC
C. BACD
D. BCAD
E. DABC
In the above set of statements, it can easily be ascertained that statement A is describing the consequence
of something, the cause of which has been explained in statement B. Thus, BA becomes a mandatory pair.
Hence, option C becomes the correct answer.

Chronology
At times you can see a logical chain of events in the parajumble. It could also be in the form of a set of
instructions to be followed in a certain order. If you look for keywords associated with the sequence, you
can easily figure out the right order.
Also, some questions have a statement that refers to a point in time. The reference may be in the past,
present or future. Accordingly you can decide its place in the sequence. In such situations – the past will
always come first, followed by the present and then the future.
A. An ishango bone was found in the Congo with two identical markings of sixty scratches each and
equally numbered groups on the back.
B. Presently, the earliest known archaeological evidence of any form of writing or counting are scratch
marks on a bone from 150,000 years ago.
C. These markings are a certain indication of counting and they mark a defining moment in western
civilisation.
D. But the first really solid evidence of counting, in the form of the number one, is from a mere twenty-
thousand years ago.
A. DCBA
B. BDAC
C. BACD
D. ABCD
E. DABC
In the above set of statements, statement B opens the paragraph by stating what is available with us at
present, while statement D sets the chronology by giving indication of the first piece of data made available.
Thus, BD becomes a mandatory pair making option B the right answer.

Transition Words
Transition words indicate a shift in ideas. They ensure the logical organisation and connection of sentences.
These may be of two types –

380

Book 1.indb 380 30/04/2019 4:48:58 PM


4.0  Language Skills Review  4.8  Comprehension

Extending words (also, again, as well as, furthermore, in addition, likewise, moreover, similarly, consequently,
hence, subsequently, therefore, thus…)
Contrasting words (yet, but, however, still, nevertheless…)
These words provide clues with respect to the sequence of sentences (sentences that shall precede or follow
a particular sentence.). A list is provided for your benefit (please note that the words provided herein are
not exhaustive, merely indicative)

SHOWING TIME (Beginning, And, Before, As, As long as, At the same time (as), Every time, Since, So
During, Ending) long as, The first/second/next/last time, When, Whenever, While, By the
time, Till, Until, Afterward(s), At first, Initially, Meanwhile, Meantime,
Simultaneously, Eventually, Finally….

SHOWING A SEQUENCE And, After, As soon as, Ever, since, Following, Later, Once, Next,
Subsequently, Then, Thereafter, First, Firstly, Initially, To begin, Next, Then,
Finally, Last, Lastly

SHOWING CONCLUSION So, In all, In summation, In conclusion, To conclude

SHOWING CONTRAST But, Either … or, Yet, Although, Despite, Even though, In spite of, Instead
of, Though, Whereas, While, By comparison, By contrast, Conversely,
However, In contrast, Instead, Nevertheless, Nonetheless,,On the contrary,
On the other hand, Otherwise

SHOWING SIMILARITY Neither … nor, Either … or, In other words, Likewise, Similarly

ADDING INFORMATION And, Additionally, Also, Besides, Further, Furthermore, In addition,


Moreover

ADDING EXAMPLE For example, For instance, In particular

A. Hence, more and more administrators are becoming aware of the critical need to keep parents
apprised of the newer methods used in schools.
B. Therefore, the great influence of parents cannot be ignored or discounted by the teacher.
C. However important we may regard school life to be, there is no gain saying the fact that children
spend more time at home than in the classroom.
D. They can become strong allies of the school personnel or they can consciously or unconsciously hinder
and thwart curricular objects.
A. BADC
B. CDBA
C. CBDA
D. CDAB
E. DABC
In the above mentioned parajumble, we can identify two transition words – Hence and Therefore. Both
these keywords are used to explain the consequence of something. Upon perusal of the text, it is apparent
that statement B is the explanation for statement C while statement a concludes the passage, Thus, CB
become a mandatory pair making option C the correct response.

381

Book 1.indb 381 30/04/2019 4:48:58 PM


NMAT by GMAC™ Official Guide 2019

General to Specific
In case one is stuck between two statements that appear to form a mandatory pair but one can’t decide the
sequence of the statements that is, which one should come first, we follow the principle of general to specific.
Let us elucidate the concept with an example:
A. He made an interesting comment about our store’s pricing policy.
B. He said that we could offer discounts and incentives to encourage people to buy in lesser quantities
but more frequently.
C. Discounts could be given ranging from 15 – 20 percent on every purchase made.
D. The lesser the quantity, the more frequently the customers will have to visit the store. The more
frequent the visits, the more incentives they can earn.
A. ABCD
B. ABDC
C. BDCA
D. ABCD
E. DABC
In the jumble given above, there can be some confusion between BCD and BDC. In such a scenario, we
follow the general to specific rule. Statement C is a specific example of the proposal outlined in statement
D. Hence the correct sequence is ABDC, making answer option B the correct response.
Some other ways in which mandatory pairs can be identified are through the use of –
Obvious Openers: You may sometimes come across statements that are obvious openers that is, it is clear
that the paragraph begins with them. They could either be introducing the idea or contain all the relevant
terms/definitions. These statements can be definitions, universal truths or philosophical statements.
Obvious Conclusions: Sometimes, you can easily figure out the concluding line of the paragraph.
Keywords might also be available to help narrow your choices.
Use of Acronyms: If both full form as well as short form is present in different sentences, then the sentence
containing full form will come before the sentence containing short form.
Definition and Example: If any sentence is working as an example, place it after the sentence it is
explaining. It might not necessarily form a mandatory pair but the example has to follow the idea/
hypothesis/theory it is elucidating.
Use of Articles: By their very definition, when the author uses ‘a / an’ - he wants to make a general
statement but when he uses ‘the’, he wants to refer back to some previously discussed noun. Hence, the use
of 'the' is quite unlikely in the opening sentence. Also, if both ‘a/an’ and ‘the’ are used for the same noun,
then the sentence containing ‘the’ will come after the sentence containing, a/an.

Logical Sequencing of Paragraph


While attempting questions on para jumbles, the key is to ensure that there is a logical flow in the
sentences of the paragraph. In order to ascertain the logical flow of sentences, the following steps should be
adopted:
• Read the question and the answer choices
• Examine the answer choices
• Eliminate incorrect choices
• Review the flow of events

382

Book 1.indb 382 30/04/2019 4:48:58 PM


4.0  Language Skills Review  4.8  Comprehension

Read through the paragraph and break down each sentence in your own words. Sometimes, you can
eliminate a few sequences by just going through the answer choices. You can take the help of the pronouns
used in the answer choices. Some sentences may contain pronouns such as these, those, they, it, or the. They
indicate that the given idea has been already discussed in the paragraph. Some sentences may contain
words such as ‘first’, ‘next’, ‘before’, ‘since’, ‘although’, or ‘finally’. These will help you identify the correct
sequence of the sentences in the paragraph. Once you pick your answer, make sure you read through the
given paragraph or the passage. This will ensure your answer fits into the flow. It will also help you check
whether you have chosen the correct answer. You will then be able to tell right away, if something is missing
or feels wrong.
Let us now consider a few examples:
Directions for examples 1-2: Choose the most logical order of sentences from among the given choices to
construct a coherent paragraph
Example 1

(a) A
 tal Bihari Vajpayee too was a fan of mangoes and his childhood friend Anna Kiwalkar would
send him two crates every week during the season.
(b) The exchange is termed as ‘mango diplomacy’ and is seen as winning over foes.
(c) Former PM Indira Gandhi’s love for mangoes was well known within her circle and close friends
visiting her would always get mangoes for her.
(d) This exchange is based on the intense national pride across South Asia about whose mangoes are
the most delicious.
(e) Prime Ministers of India and Pakistan frequently send the best varieties of mangoes from their
respective countries to each other.
(A) caedb
(B) bdeac
(C) deacb
(D) baedc
(E) cdeab
Solution

this exchange and the exchange are the clues to the right sequence—edb. Also sentence (a) has to come
immediately after sentence (c) because of the use of too in sentence (a).
The correct answer is A.

Example 2

(a) The app was initially designed for transferring money over the mobile since carrying cash was
risky.
(b) Vodafone brought m-pesa to India and launched it as a pilot in Rajasthan, rolling it out fully in
2013.
(c) But it soon changed into a financial service and became a big hit.
(d) M-pesa, a mobile based application, started in Kenya in 2007 as a CSR pilot project by Safaricom,
a Vodafone subsidiary.

383

Book 1.indb 383 30/04/2019 4:48:59 PM


NMAT by GMAC™ Official Guide 2019

(A) bcda
(B) dacb
(C) dcab
(D) bcda
(E) dbca
Solution

Sentences da form a logical pair since the ‘the app’ in sentence (a) refers to ‘application’ in sentence (d).
Also the entire paragraph is about M-pesa, so it should start with sentence (d). Thus, option (B) is the
correct answer.
The correct answer is B.

384

Book 1.indb 384 30/04/2019 4:48:59 PM


Book 1.indb 385 30/04/2019 4:48:59 PM
5.0  Language Skills Practice

Book 1.indb 386 30/04/2019 4:48:59 PM


5.0   Language Skills Practice

1 Antonyms
Select the word or phrase that is nearly the opposite in meaning to the word given in the question
stem.

1. CONTENTIOUS 7. VILIFY
(A) Arguable (A)
Disparage
(B) Agreeable (B)
Renege
(C) Similar (C)
Repudiate
(D) Controversial (D)
Exalt
(E) Truculent (E)
Affront

2. PEJORATIVE 8. TYRO
(A) Derogatory (A)
Renegade
(B) Nugatory (B)
Virtuoso
(C) Complimentary (C)
Miscreant
(D) Simple (D)
Hypochondriac
(E) Oratorical (E)
Iconoclast

3. STOIC 9. ALACRITY
(A) Indomitable (A)
Diffidence
(B) Agitated (B)
Apathy
(C) Sycophant (C)
Empathy
(D) Serene (D)
Finesse
(E) Impassive (E)
Intrepidity

4. MYOPIC 10. PLACATE


(A) Hypochondriac (A)
Appease
(B) Vindictive (B)
Quell
(C) Bigoted (C)
Exasperate
(D) Self-centered (D)
Renounce
(E) Far-sighted (E)
Reproach

5. INSULAR 11. INNOCUOUS


(A) Unbiased (A)
Insolent
(B) Circumscribed (B)
Irreverent
(C) Isolated (C)
Officious
(D) Dispassionate (D)
Prodigious
(E) Insouciant (E)
Detrimental

6. SUBTERFUGE 12. PROSAIC


(A)
Servitude (A)
Palpable
(B)
Guile (B)
Regressive
(C)
Deceit (C)
Uncanny
(D)
Castigation (D)
Imaginative
(E)
Guilelessness (E)
Humdrum

387

Book 1.indb 387 30/04/2019 4:48:59 PM


NMAT by GMAC™ Official Guide 2019

13. COUNTERFEIT 19. Select the word or phrase that is nearly the
opposite in meaning to the given word: (Real NMAT
(A)
Covert
Question)
(B)
Authentic
DENOUEMENT
(C)
Reprehensible
(D)
Laconic (A) Proposition
(E)
Spurious (B) Conspiracy
(C) Preamble
14. TENTATIVE
(D) Intrigue
(A)
Compendious
(E) Climax
(B)
Sinuous
(C)
Certain 20. Select the word or phrase that is nearly the
opposite in meaning to the given word: (Real NMAT
(D)
Tenuous
Question)
(E)
Tenacious
BETOKEN
15. DAUNT
(A) Belie
(A)
Exemplify
(B) Auspicate
(B)
Protract
(C) Foretoken
(C)
Parley
(D) Postpone
(D)
Disconcert
(E) Postpose
(E)
Hearten
2
1. Select the word that is nearly the opposite in
16. PHLEGMATIC
meaning to the given word: (Real NMAT Question)
(A)
Salubrious
(B)
Impassive PARSIMONIOUS
(C)
Ignominious (A) Greedy
(D)
Compassionate (B) Parisian
(E)
Clandestine (C) Penurious
17. SUCCINCT (D) Tight-fisted
(E) Philanthropic
(A)
Laconic
(B)
Churlish 22. Select the word or phrase that is nearly the
(C)
Garrulous opposite in meaning to the given word: (Real NMAT
Question)
(D)
Piquant
(E)
Whimsical IRASCIBLE

18. EFFACE (A) Outgoing


(A)
Obliterate (B) Agreeable
(B)
Propitiate (C) Friendly
(C)
Create (D) Benign
(D)
Jeer (E) Patient
(E)
Exult

388

Book 1.indb 388 30/04/2019 4:48:59 PM


5.0   Language Skills Practice

23. Select the word or phrase that is nearly the 27. STUPEFY (Real NMAT Question)
opposite in meaning to the given word: (Real NMAT
(A) Dumbfound
Question)
(B) Flabbergast
SIBYLLINE
(C) Sensitise
(A) Unprophetic
(D) Suffocate
(B) Perceptive
(E) Arouse
(C) Eloquent
28. IMPERTURBABLE (Real NMAT Question)
(D) Palpable
(E) Overt (A) Militant
(B) Cynical
24. Select the word or phrase that is nearly the
(C) Comical
opposite in meaning to the given word: (Real NMAT
Question) (D) Agitated
(E) Angry
DEMEAN
29. CLAMP (Real NMAT Question)
(A) incite
(A) Clinch
(B) praise
(B) Secure
(C) humble
(C) Release
(D) disgrace
(D) Entwine
(E) suppress
(E) Encompass
25. VIOLATION (Real NMAT Question)
30. VOLUPTUARY (Real NMAT Question)
(A) Offence
(A) Austereness
(B) Rejection
(B) Spartan
(C) Obedience
(C) Restrained
(D) Persuasion
(D) Disciplined
(E) Devastation
(E) Abstemious
26. PROFESS (Real NMAT Question)
(A) Affirm
(B) Divulge
(C) Confirm
(D) Conceal
(E) Asseverate

389

Book 1.indb 389 30/04/2019 4:48:59 PM


NMAT by GMAC™ Official Guide 2019

2 Analogies (C)
Clarinet: Musical Instrument
(D)
Bolero: Dress
Directions for Questions 1– 5:  Choose the word or pair (E)
Cha Cha: Dance
of words that best completes the relationship to the given
pair. 7. Scrawny: Slim

1. Crime: Police (A)


Nitpicking: Meticulous
(A) Watchman: Theft (B)
Shocking: Surprising
(B) Food: Eat (C)
Miserly: Economical
(C) Weight: Exercise (D)
Nosey: Inquisitive
(D) Flood: Dam (E)
Ludicrous: Absurd
(E) Play: Football
8. Tiara: Hair
2. Cleaver: Butcher
(A)
Muffler: Neck
(A) Screwdriver: Mechanic (B)
Anklet: Ankle
(B) Treadmill: Runner
(C)
Cravat: Neck
(C) Pen: Writer
(D)
Girdle: Wrist
(D) Scalpel: Surgeon
(E)
Sash: Waist
(E) Brush: Painter
9. Paleontology: Fossils
3. Captain: Ship
(A)
Entomology: Insects
(A) Teacher: School
(B)
Seismology: Earthquakes
(B) Manager: Office
(C)
Petrology: Rocks
(C) Guide: Tourist
(D)
Anthropology: Mankind
(D) Doctor: Hospital
(E)
Ornithology: Fish
(E) Hotel: Concierge
4. Dislike: Loathe 10. Canto: Poem

(A) Pain: Discomfort (A)


Island: Archipelago
(B) Coward: Foolhardy (B)
Piston: Engine
(C) Disquiet: Anxious (C)
Canopy: Rain
(D) Fear: Stress (D)
Mast: Ship
(E) Joy: Ecstasy (E)
Staircase: Building

5. Altruistic: Selfishness 11. Tepid: Boiling


(A) Enlightened: Wisdom (A)
Careful: Punctilious
(B) Befuddled: Clarity (B)
Drizzle: Downpour
(C) Flippant: Calm (C)
Like: Dote
(D) Assiduous: Diligence (D)
Avaricious: Rapacious
(E) Depressed: Sorrow (E)
Vain: Vainglorious

Directions for Questions 6-18:  Choose the pair of 12. Denigrate: Belittle
words that does not share the same relationship as the
(A)
Dearth: Scarcity
given pair.
(B)
Ephemeral: Transitory
6. Metal: Music (C)
Ineptitude: Competence
(A)
Expressionism: Painting (D)
Indolent: Lazy
(B)
Risotto: Dish (E)
Facile: Superficial

390

Book 1.indb 390 30/04/2019 4:49:00 PM


5.0   Language Skills Practice

13. Mozzarella: Cheese (C) Orange: Citrus


(A) Spaghetti: Pasta (D) Rose: Rosa
(B) Moccasin: Dress (E) Chocolate: Edible
(C) Macchiato: Coffee 20. Hedonism: Pleasure
(D) Viper: Snake (A) Asceticism: Self-indulgence
(E) Chrysanthemum: Flower (B) Altruism: Selfishness
14. Quiver: Cobras (C) Sexism: Equality
(A)
School: Fish (D) Eidolism: Ghosts
(B)
Congregation: Pigeons (E) Nihilism: Absolute power
(C)
Herd: Antelopes 21. Basil: Herb
(D)
Army: Caterpillars (A)
Ragdoll: Cat
(E)
Float: Crocodiles (B)
Frigate: Aircraft
15. Fit: Fiddle (C)
Alaskan Husky: Fox
(D)
Marmot: Squirrel
(A)
Fresh: Daisy
(E)
Equine: Horse
(B)
Slippery: Eel
(C)
Straight: Arrow 22. Disheveled: Tidy
(D)
Mad: Ox (A) Disdain: Contempt
(E)
Quiet: Church mouse (B) Discrepancy: Affront
16. Numerophobia: Numbers (C) Kindle: Ignite
(D) Dispel: Dissipate
(A)
Photophobia: Light
(E) Exasperate: Please
(B)
Ailurophobia: Dogs
(C) Ophidiophobia: Snakes 23. Trickle: Gush
(D)
Agoraphobia: Open Spaces (A)
Plunder: Pillage
(E)
Acrophobia: Heights (B)
Quash: Revoke
(C)
Breeze: Gale
17. Brush: Bristle
(D)
Acorn: Oak tree
(A)
Flower: Stamen
(E)
Tide: Ebb
(B)
Fruit: Pericarp
(C)
Aircraft: Fuselage 24. Trepidation: Trembling
(D)
Billboard: Awning (A)
Earthquake: Tsunami
(E)
Symphony Orchestra: Woodwinds (B)
Snowflakes: Deluge
(C)
Acquittal: Incarceration
18. Address: Location
(D)
Cold: Scald
(A) Annul: Abrogate
(E)
Sickness: Affliction
(B) Bark: Snap
(C) Mean: Generous 25. Mariticide: Husband
(D) Die: Expire (A) Uxoricide: Wife
(E) Current: Present (B) Ambicide: Uncle
Directions for Questions 19-27:  Find out the pair of (C) Sororicide: Brother
words that has the same relationship as the original pair. (D) Fratricide: Sister
19. Mollusc: Shell (E) Filicide: Friend
(A) Lemon: Rind
(B) Marshmallow: Confection

391

Book 1.indb 391 30/04/2019 4:49:00 PM


NMAT by GMAC™ Official Guide 2019

26. Endorse: Support:: ________: ________ 29. Bread: Grain (Real NMAT Question)
(Real NMAT Question)
(A) Many: Berries
(A) cavil: nitpick (B) Drink: Alcohol
(B) abrogate: renew (C) Hot: Beverage
(C) desiccate: destroy (D) Fruit: Grapes
(D) fulminate: fumigate (E) Wine: Fruit
(E) hypothecate: guess
30. Radius: Radii (Real NMAT Question)
27. Pre-empt: Anticipation: ________: ________ (A) Horse: Mare
(Real NMAT Question)
(B) Bees: Swarm
(A) plan: debriefing (C) Square: Side
(B) recruit: position (D) Author: Authors
(C) study: opportunity (E) Mathematics: Equation
(D) inoculate: vaccination
31. Diffident: Shy (Real NMAT Question)
(E) quarantine: prognosis
(A) Arrogant: Valiant
Directions for Questions 28–31: Choose the word or (B) Laconic: Sarcastic
pair of words that best completes the relationship to the
(C) Garrulous: Talkative
given pair.
(D) Onerous: Portentous
28. Kind: Benevolent (Real NMAT Question) (E) Incoherent: Stammering
(A) Empty: Full
(B) Blemish: Loyalty
(C) Absurd: Rational
(D) Stubborn: Obstinate
(E) Capricious: Unchanging

392

Book 1.indb 392 30/04/2019 4:49:00 PM


5.0   Language Skills Practice

3 Synonyms
Select the word or phrase that is nearest in meaning to the word given in the question stem.

1. EXPENDABLE 7. QUIBBLE
(A) Gushing (A)
Proscribe
(B) Anomaly (B)
Ingest
(C) Dejected (C) Hoodwink
(D) Superfluous (D)
Elucidate
(E) Parsimonious (E)
Carp

2. DISTRAUGHT 8. ENCUMBRANCE
(A) Subservient (A)
Machination
(B) Composed (B)
Obstacle
(C) Authoritative (C)
Skepticism
(D) Dogmatic (D)
Ruse
(E) Agitated (E)
Stoicism

3. RELENTLESS 9. OBSEQUIOUS
(A) Lenient (A)
Gawky
(B) Dogged (B)
Inert
(C) Painstaking (C)
Ingratiating
(D) Intermittent (D)
Diabolic
(E) Ephemeral (E)
Cantankerous

4. TRITE 10. ODIOUS


(A) Innovative (A)
Salubrious
(B) Exiguous (B)
Assiduous
(C) Hackneyed (C)
Laconic
(D) Opportune (D)
Restive
(E) Pertinent (E)
Execrable

5. DWINDLE 11. SERENDIPITOUS


(A) Enhance (A)
Extraneous
(B) Burgeon (B)
Sagacious
(C) Voracious (C)
Accidental
(D) Diminish (D)
Scathing
(E) Surge
(E)
Turbid
6. LOQUACIOUS
12. OBVIATE
(A)
Taciturn
(A)
Relinquish
(B)
Verbose
(B)
Vacillate
(C)
Mercurial
(C)
Tamper
(D)
Placid
(D)
Trounce
(E)
Unscrupulous
(E)
Forestall

393

Book 1.indb 393 30/04/2019 4:49:00 PM


NMAT by GMAC™ Official Guide 2019

13. VITUPERATION (C) Guarantee


(A)
Adulation (D) Apprise
(E) Negotiate
(B)
Opprobrium
(C)
Exaltation 20. ENTHRALL
(D)
Ostentation (A) Abridge
(E)
Recrimination (B) Admonish
(C) Cajole
14. SPOOKY
(D) Elucidate
(A)
Noxious (E) Enchant
(B)
Hapless 21. CRAVEN
(C)
Eerie
(A)
Candid
(D)
Apprehensive
(B)
Defiant
(E)
Lamentable
(C)
Cowardly
15. BASHFUL (D)
Juvenile
(A)
Mordant (E)
Illicit
(B)
Didactic 22. Select the word or phrase that is nearest in meaning
(C)
Erudite to the given word: (Real NMAT Question)
(D)
Demure
MAWKISH
(E)
Fallacious
(A) overly sentimental
16. VANDALIZE (B) melodramatic
(A) Venerate (C) nostalgic
(B)
Defile (D) tearful
(C)
Disparage (E) cynical
(D)
Quaver
23. Select the word that is nearest in meaning to the
(E)
Fetter given word: (Real NMAT Question)
17. INSCRUTABLE PRESCRIBE
(A)
Tractable (A) bundle
(B)
Perfidious (B) betray
(C)
Buoyant (C) deceive
(D)
Enigmatic (D) stipulate
(E)
Vociferous (E) admonish
18. INSOUCIANT 24. Select the word or phrase that is nearest in meaning
(A)
Staid to the given word: (Real NMAT Question)
(B)
Verdant AFFRONT
(C)
Oblique
(A) abstruse
(D)
Nonchalant
(B) examine
(E)
Squalid
(C) forbear
19. CORROBORATE (D) scorn
(A) Confirm (E) yield
(B) Invalidate

394

Book 1.indb 394 30/04/2019 4:49:00 PM


5.0   Language Skills Practice

25. Select the word or phrase that is nearest in meaning 28. OSTENSIBLY (Real NMAT Question)
to the given word: (Real NMAT Question)
(A) Calmly
ELUCIDATE (B) Quietly
(A) clarify (C) Actually
(B) confuse (D) Apparently
(C) befuddle (E) Conspicuously
(D) convince 29. CALIGINOUS (Real NMAT Question)
(E) contradict (A) Lambent
26. PORTRAY (Real NMAT Question) (B) Recondite
(C) Cretinous
(A) Feign
(D) Tenebrous
(B) Depict
(E) Sophomoric
(C) Invoke
(D) Conjure
(E) Simulate
27. POGROM (Real NMAT Question)
(A) Rescue operation
(B) Cartoon character
(C) Attempt to eradicate
(D) New version of memory
(E) Organised local massacre

395

Book 1.indb 395 30/04/2019 4:49:00 PM


NMAT by GMAC™ Official Guide 2019

4 Fill in the Blanks


Each of the sentences below consists of one blank or two blanks. Choose the word or set of words for each
blank that best fits the meaning of the sentence as a whole.

1. While there is no doubt that Manchester City is (C) a subjective……….surprisingly


________ football team, whether they are able to (D) a contentious…….nonetheless
perform to their ________ in the upcoming Premier
(E) a differentiating…………however
League season remains to be seen.
5. Many items of clothing are worn purely for
(A) a boring………talent
traditional or ceremonial purposes—because the
(B) a good…….detriment occasion demands it. For example, the tie has no
(C) an average………..capability function that anyone can explain,
(D) an excellent…….potential yet most people working in a corporate environment
would rarely be seen without one.
(E) an upcoming……….spectators
(A) accepted
2. Allergy to gluten has become surprisingly common in
recent times. Even the products made by companies (B) rational
that do not use any gluten in their manufacturing (C) critical
process ________ be considered completely (D) illogical
________ since there is no guarantee that gluten
(E) peculiar
will not be present in the raw materials that these
companies use to make their products. 6. Laughing during the funeral was a most __________
act _________ by him
(A) cannot………benign
(B) should…….unsafe (A)
egregious, perpetrated
(C) will not……….vulnerable (B)
inconspicuous, committed
(D) claim to…….harmless (C)
gross, inflicted
(E) are not to……….inconspicuous (D)
flagrant, executed
(E)
resplendent, wreaked
3. The findings of a recent study provide an interesting
example of the process of natural selection—certain 7. The vegetation here is quite _________ but the
light coloured animals living in environments that vegetation in the adjoining state is ___________
have been soot and other forms of making it a complete contrast.
pollution, eventually take on a darker colouration.
(A)
luxuriant, sparse
(A) painted with
(B)
copious, profuse
(B) coloured with
(C)
slender, sporadic
(C) satiated with
(D)
scanty, exiguous
(D) exposed to
(E)
bountiful, munificent
(E) whetted in
8. The old man wasn’t aware that his son, of whose
4. According to one point of view, it is not correct to
___________ he was completely ___________ was
use compassion as a ________ principle because
the talk of the town because of his flirtatious ways.
it is based on the assumption that just because an
animal is like me in certain aspects, it will be like (A)
turpitude, satisfied
me in some other aspects as well. ________, this (B)
depravity, certain
is an erroneous means of proving identity because
it ignores the various historical, cultural and other (C)
rectitude, convinced
references that might have crept in. Just because an (D)
righteousness, coaxed
animal looks like us does not mean that it is actually (E)
morality, unassertive
like us.
9. Budget travellers and birdwatchers are content to
(A) a universal………moreover plan their trips during the rainy season, when prices
(B) a discriminating………..furthermore _______.

396

Book 1.indb 396 30/04/2019 4:49:00 PM


5.0   Language Skills Practice

(A)
escalate 15. The new evidence will, in all likelihood, __________
(B)
plummet the ___________ of the murder charge.
(C)
descend (A) exonerate, gullible
(D)
abate (B) vindicate, defendant
(E)
diminish (C) castigate, criminal
(D) censure, wrongdoer
10. It is rather___________ of him to feel that he knows (E) acquit, reprobate
more about the topic than his teacher.
16. Completely_______ to the fact that the students were
(A)
discourteous bored, the Professor continued his _________ for
(B)
uncouth three hours.
(C)
presumptuous (A)
oblivious, harangue
(D)
peremptory (B)
unheeding, rant
(E)
arbitrary (C)
indifferent, onslaught
11. The police officer cornered the _________ man to (D)
impervious, polemic
see if he was __________. (E)
impassive, panegyric
(A)
startled,restrained 17. It’s only when the underwater clown turns to face
(B)
staggering, inebriated you that you understand why it’s the most feared
(C)
wobbly, despondent animal on Earth. From the front its head is no longer
(D)
rickety, intoxicated soft and jowly but _________ to an arrow that draws
(E)
fragile, sober its black eyes into a __________ looking V. The
________ smile is gone, and all you see are rows of
12. The ____________ “pretty ugly” implies that a person two-inch teeth capable of crunching down with almost
can be both attractive and unattractive at the same two tons of force.
time.
(A)
diminishes, joyous, dazed
(A)
sarcasm (B)
contracts, quivering, cheeky
(B)
euphemism (C)
stiffens, menacing, intimidating
(C) oxymoron (D)
tapers, sinister, bemused
(D)
antithesis (E)
subsides, threatening, indignant
(E)
rhetoric
18. The southern tip of Africa is more _______ than
13. The teacher doubted the _________ of the student many give it credit for: a spectacular blend of game-
when he submitted a paper written by someone else. spotting safaris, ________ of ancient cultures and
_________ landscapes ranging from widescreen
(A)
veracity dusty plains to thundering waterfalls.
(B)
candour
(A) distinct, traces, multitude
(C)
fidelity
(B) disparate, vestige, profuse
(D)
mendacity
(C) diverse, echoes, myriad
(E)
congruity
(D) distinguishable, memento, excessive
14. The road is quite ______________ and one does feel (E) different, reminder, riot
a little ____________ when driving through it.
19. In telling the story of one of history’s most celebrated
(A)
meandering, bizarre ________, the sinking of the White Star Line’s R.M.S.
(B)
discursive, giddy Titanic on her maiden voyage from Southampton on
(C)
serpentine, light headed April 15, 1912, Cameron was clearly inspired by the
challenge of reproducing the event with a physical
(D)
sinuous, surreal __________ and impact _________ in the numerous
(E)
digressive, nauseous previous film and TV versions of the event.
(A)
tragedies, authenticity, conceivable
(B)
calamities, plausibility, ineffable

397

Book 1.indb 397 30/04/2019 4:49:00 PM


NMAT by GMAC™ Official Guide 2019

(C)
afflictions, chicanery, improbable 23. The robber _________ the location of the stolen
(D)
disasters, verisimilitude, inconceivable money after being questioned for several hours.
(Real NMAT Question)
(E)
tribulations, artfulness, unimaginable
(A) emerged
20. The harsh teacher scolded the students in order
(B) expected
to establish her ________ over them. (Real NMAT
Question) (C) concealed
(D) disclosed
(A) humanity
(E) threatened
(B) authority
(C) generosity 24. It was rather difficult for Ashwin to bear such
(D) selflessness ________ comments from his seniors after having
been one of the company’s most diligent workers.
(E) worthlessness
(Real NMAT Question)
21. The Transport Commissioner’s _______ idea of (A) derogatory
making attendance at a road safety awareness
(B) erroneous
session mandatory for registration of a new vehicle
has failed to take off in the district. MVD sources (C) illaudable
said it faced many _______, both infrastructurally (D) measly
and practically, in such a small district. (Real NMAT (E) tedious
Question)
25. Having been selected to represent the Association
(A) lame … … jinx of Indian Managers at the International Convention,
(B) appreciable … … chaos _________. (Real NMAT Question)
(C) presuming … … obstacles (A) he gave a short acceptance speech
(D) overconfident … … hurdle (B) a speech had to be given by him
(E) ambitious … … impediments (C) the members congratulated him
22. We need to ________ in a few stops to stretch our (D) the members applauded him
legs when we determine how long it will take to drive (E) member congratulates him
to Aunt Suzy’s house. (Real NMAT Question)
(A) issue
(B) factor
(C) manage
(D) acquire
(E) intimidate

398

Book 1.indb 398 30/04/2019 4:49:01 PM


5.0   Language Skills Practice

5 Cloze Test
Questions 1–32 below consist of short passages from which several words have been deleted and replaced
with blanks. For each blank, select from the choices the word that best fits that blank, taking into account
the overall meaning conveyed by the passage.

Jane Sequeira was born in 1856. Her father was so great The “stout” discoveries mark what may be the oldest
a (1) , that the place of her birth is uncertain. beer-making facilities in Greece and upend the (7)
However, she generally assumed it to be Scotland, since she that the region’s ancient go-to drink was only wine,
had spent the first five years of her life there. While still very the researchers said.
young, Jane (2) traces of exquisite sensibility,
“It is an unexpected (8) for Greece,
soundness of understanding, and decision of character, but
because until now all evidence pointed to wine,” study
owing to the domineering and conservative nature of her
researcher Tania Valamoti, an associate professor of
father, she could not develop these personality traits any
archaeology at Aristotle University of Thessaloniki, in
further. When she turned eighteen, Jane became acquainted
Greece, told Live Science.
with Miss Beatrix Potter, who, (3) refined tastes
and considerable knowledge of the fine arts, seems to have
5. (A) liberally
given the first impulse to the formation of Jane’s character.
Buoyed by her new-found independence, Jane left her (B)
munificently
parents’ house, and (4) with a local community (C)
indulgently
worker—Mrs. Dinshaw—for four years, when she had to (D)
reasonably
return to her parental home to tend to her ailing mother.
(E)
judiciously
1. (A) lunatic
6. (A) archives
(B) historian
(B) describes
(C) wanderer
(C)
personates
(D) merchant
(D)
portrays
(E) warrior
(E)
renders
2. (A) repudiated
7. (A) genesis
(B) proclaimed
(C) created (B)
proposal
(D) surfaced (C)
assertion

(E) exhibited (D)


notion
(E)
allegation
3. (A) possessing
8. (A) detection
(B) surrendering
(B)
realisation
(C) utilising
(C)
find
(D) rejecting
(D)
apprehension
(E) affording
(E)
cognizance
4. (A) struggled India’s first Madame Tussauds wax museum opened its
(B) adhered ___(9)_____ here on Thursday with 50 life-like figures ___
(C) loathed (10)_____ history, sports, music, films, and politics, for the
public at the iconic Regal building in central Delhi.
(D) resided
“This is truly an exhilarating and emotional feeling to finally
(E) labored see Madame Tussauds in Delhi. Guests will be encouraged to
The ancient Greeks may have (5) indulged interact, perform and even reflect with our figures in unique
in wine, but that’s not the only alcoholic beverage they and immersive settings within the ___(11)______,” Anshul
imbibed, according to a new study that (6) Jain, General Manager and Director, Merlin Entertainment Pvt
the discovery of two potential Bronze Age breweries. Ltd, said here.

399

Book 1.indb 399 30/04/2019 4:49:01 PM


NMAT by GMAC™ Official Guide 2019

Madame Tussauds has been a successful tourist attraction Singapore is very ___(15)____ — a lot of places just add
in places like London, Las Vegas, New York, Orlando, San Chinese five spices and put it in the roaster.”
Francisco and Hong Kong. Its ____(12)____ facility here
Instead, the brothers drew upon Alvin’s 30 years of cooking
has wax statues of Indian personalities like Prime Minister
in Chinese restaurants in Birmingham, experimenting with
Narendra Modi, cinematic icon Amitabh Bachchan and Father
various Western techniques and in some cases introducing
of the Nation Mahatma Gandhi, along with many Hollywood
slightly offbeat spices, like thyme and oregano.
celebrities.The Delhi facility is the 23rd edition of Madame
Tussauds. The menu goes beyond roasts, too, offering an eclectic mix
ranging from Sichuan hot and sour soup to Singaporean
9. (A) door
comfort food dishes like steamed pork with salted egg. Both
(B)
gates versions of fried rice I tried had just the right amount of “wok
(C)
doors hei,” or char, a key element in the dish.
(D)
entry Anthony said he wanted to create a laid-back place where
(E)
entrance he could serve his favorite foods and British beers, chosen
to complement the roasts. (There are also ciders; “apples
10. (A) ranging go very well with roast pork,” he noted.) The setting is
(B)
stretching appropriately ____(16)_____, with wooden chairs and
(C)
incorporating marble-topped tables reminiscent of old-school Singaporean
coffee shops.
(D)
spanning
(E)
traversing 13. (A) rampant
(B)
widespread
11. (A) entertainment
(C)
ubiquitous
(B)
distraction
(D)
prevalent
(C)
attraction
(E) pervasive
(D)
extravaganza
14. (A) analogy
(E)
amusement
(B)
juxtaposition
12. (A) initial (C)
similarity
(B)
maiden (D)
discrepancy
(C)
introductory (E)
incongruity
(D)
opening
15. (A) exceptional
(E)
inaugural
(B)
delectable
Along a sleepy stretch not far from the dark alleys of the
(C)
mediocre
often seedy Geylang neighborhood, two brothers are quietly
trying to revolutionise Singapore’s roast meat scene. Char, a (D)
appetizing
cozy 70-seater, opened last January with Chinese-style roast (E)
substantial
meats being the stars of the menu — specifically traditional
meats like char siew (roast pork) and siew yoke (crispy roast 16. (A) ostentatious
pork belly), but done in somewhat untraditional ways. (B)
rustic
Instead of the ____(13)____ thinly sliced white and vermilion (C)
brash
Chinese roast pork you’ll find all over the island, Char’s (D)
grandiose
version of char siew appeared on our table as a glistening (E)
unimpressive
mound of thick, meaty chunks slathered in a gooey sauce
the colour of dark chocolate. The first bite hushed us — the I wanted _____(17)_____ to the coffee and doughnuts that
meat was so soft and packed with flavor I could almost the management got at work on Fridays. I began to _____
feel it melting on my tongue. The roast pork belly had a (18)_____ a way to get the goodies by being especially
lovely tenderness, too, a striking ____ (14)_____ with the nice to the supervisor’s secretary. I didn’t want to make my
crackling crispness of the skin. scheme _____(19)_____ so I didn’t stop in to see her every
day. I hoped she didn’t _____(20)_____ what I really wanted
“I love roast meats,” said Anthony Ung, who owns the
when I told her how much I loved jelly-filled doughnuts. I
restaurant with his chef brother Alvin. Mr. Ung, who was born
wanted to retain my image as a friendly co-worker and
in Vietnam, grew up in Birmingham, England, and has lived
still get free food. It worked. She started letting me in the
in Singapore for 20 years. Often, he said, “the char siew in

400

Book 1.indb 400 30/04/2019 4:49:01 PM


5.0   Language Skills Practice

conference room after the morning meeting to help myself (D) forgiven
to the leftover doughnuts. (Real NMAT Question) (E) omitted
17. (A) taste
24. (A) enlighten
(B) need
(B) brighten
(C) desire
(C) lighten
(D) excess
(D) signify
(E) access
(E) burden
18. (A) report In _____(25)_____, I should have told Veronica tonight how
(B) forecast I feel. I didn’t want to _____(26)_____ our friendship, so I
(C) evaluate didn’t tell her that I like her. Now I _____(27)_____ not letting
her know. We have developed such a close _____(28)_____
(D) memorise
that I feel like I have known her for years instead of a few
(E) manoeuvre months. At our first meeting, I was able to empathise with
her nervousness about being away from home for the first
19. (A) show
time. I felt the same way last year. Next week I will disclose
(B) appeal my feelings to her. (Real NMAT Question)
(C) evident
25. (A) totality
(D) oriented
(B) addition
(E) highlight
(C) retrospect
20. (A) veil (D) memorising
(B) reveal (E) recollection
(C) surpass
26. (A) sad
(D) clarified
(B) burst
(E) surmise
(C) effect
___(21)____, these modern tools of communication can (D) annoy
also be tools of ____(22)____, making us to feel so out
(E) jeopardise
of touch, so much more in need of real human contact.
If a computer message came addressed as simply “Hi”, 27. (A) guilt
I’d feel forgotten and ____(23)____. If however, a more (B) repel
personalised message or call were to come, it would
(C) offend
___(24)___my day and make me feel like I was part of their
family unit after all. (Real NMAT Question) (D) lament
(E) revenge
21. (A) Ironically
28. (A) rapport
(B) Naturally
(B) bondage
(C) Fastidiously
(C) sympathy
(D) Suspiciously
(D) compassion
(E) Contemptuously
(E) resemblance
22. (A) elevation
Journalists, on the whole, are _____(29)_____ investigators
(B) alienation and reporters with a sense of mission. They want to
(C) allegation discover and _____ (30) _____ the truth, to share what
(D) alleviation is of importance happening in the world today while
still respecting people’s rights and balancing the _____
(E) mollification
(31)_____ demand for news. By doing so they bring about
23. (A) discernible a positive change in society. This is why people choose to
become professional journalists, aiming to _____(32)_____
(B) neglected the best examples of how journalism can help society. (Real
(C) negligent NMAT Question)

401

Book 1.indb 401 30/04/2019 4:49:01 PM


NMAT by GMAC™ Official Guide 2019

29. (A) glued


(B) diligently
(C) fascinated
(D) persevered
(E) conscientious
30. (A) move
(B) reassign
(C) relocate
(D) transmit
(E) transport
31. (A) greedy
(B) devoted
(C) voracious
(D) insatiable
(E) gluttonous
32. (A) vie
(B) chase
(C) contend
(D) emulate
(E) challenge

402

Book 1.indb 402 30/04/2019 4:49:01 PM


5.0   Language Skills Practice

6 Identify the Error


Questions 1–35 each has a sentence with four underlined words or phrases. Identify the one underlined
word or phrase that must be changed in order to make the sentence correct. Mark E for no error.

1. In order to lose weight, you need to both eat in small 6. Although Rajesh studied hard for the test, yet he
quantities as well as exercise regularly. could only manage a second division.
(A) to lose (A) studied
(B) need to (B) for the test
(C) as well as (C) yet
(D) regularly (D) only manage
(E) No error (E) No error
2. The artist’s most impressive works have been 7. The large and barren expanse of the Sahara desert
produced at his home in New York, he moved there is at once forbiddingly empty, climatically harsh, and
from his native place, India. the beauty of it is haunting.
(A) artist’s most impressive (A) large and barren
(B) have been produced (B) is at once
(C) New York, he moved there (C) climatically harsh
(D) native India (D) the beauty of them is haunting
(E) No error (E) No error
3. Most Indian students pursue their MBAs immediately 8. The dormant volcano started spewing lava last year
after completing college, while a few working for two and it has been erupting sporadically ever since.
or three years before seeking admission.
(A) started spewing
(A) pursue their MBAs (B) and
(B) completing college, while (C) has been erupting
(C) working for (D) ever since
(D) before seeking (E) No error
(E) No error
9. In this store, we sell items sourced from not only Asia
4. The client that is coming for a meeting tomorrow is but also from the Central and Latin Americas.
one of our oldest and most important clients.
(A) items sourced
(A) that is (B) not only Asia
(B) for (C) from
(C) is one of (D) Central and Latin Americas
(D) most important clients (E) No error
(E) No error
10. The items served in my restaurant are far more
5. Hoping to receive a permanent position, the intern diverse and delicious than your restaurant.
put in 16 hours of work everyday.
(A) served in
(A) Hoping (B) are far more
(B) a permanent position (C) and delicious
(C) put in (D) your restaurant
(D) everyday (E) No error
(E) No error

403

Book 1.indb 403 30/04/2019 4:49:01 PM


NMAT by GMAC™ Official Guide 2019

11. In this museum, there is a large number of artefacts 17. The first inning of the match was more interesting
from every historical period, going as far back as the than the second one.
time of the ancient Greek.
(A)
first inning
(A) there is (B)
the match
(B) of artefacts from (C)
more interesting
(C) period, going (D)
second one
(D) as (E)
No error
(E) No error
18. Safety glass, a toughened glass sheet, is six times
12. When enquired as to which course he would prefer stronger than untreating glass. (Real NMAT Question)
to take, Laksh replied that either of the two courses
was fine with him. (A) Safety glass
(B) toughened
(A) as to which
(C) times stronger
(B) would prefer
(D) untreating
(C) replied that
(E) No error
(D) was fine
(E) No error 19. If I had the opportunity, I would have had someone pick
up the parcel from your house. (Real NMAT Question)
13. Each of the eyewitnesses have a different story to
narrate about the crime. (A) If I had
(A) Each of the (B) have had
(B) have a (C) pick up
(C) narrate (D) from your
(D) about the crime (E) No error
(E) No error 20. A major contribution of Maratha sculptors during the
14. Suppose if you were marooned on a deserted island, medieval age were the creation of the Buddha’s images
what would you do? in different human forms. (Real NMAT Question)

(A) Suppose if (A) A major

(B) marooned (B) Maratha sculptors


(C) what would (C) were the creation

(D) do (D) the Buddha’s

(E) No error (E) No error

15. The child loves both dancing and singing and refuses 21. One of the ancient traditions in India commit the guru as
to conform by the wishes of his strict parents. well as his disciples to celibacy. (Real NMAT Question)

(A) both dancing and singing (A) One of the


(B) refuses (B) commit the guru
(C) conform by (C) as well as
(D) wishes of (D) to celibacy
(E) No error (E) No error

16. When he heard the news of his father’s accident, he 22. Are you the one with who Ravi went home last night?
immediately returned back to his village. (Real NMAT Question)

(A)
he heard (A) one

(B)
father’s accident (B) who
(C)
returned back (C) went

(D)
to his village (D) last

(E)
No error (E) No error

404

Book 1.indb 404 30/04/2019 4:49:01 PM


5.0   Language Skills Practice

23. I wonder if you can give me an information about a 28. The Titanica, which plies from Singapore to Hong
certain Mr. Roy who used to live here. (Real NMAT Kong, is the largest cruise liner of the world. (Real
Question) NMAT Question)
(A) if (A) which plies
(B) an (B) to Hong Kong
(C) a (C) is the largest
(D) to (D) of
(E) No error (E) No error

24. Only about a hundred out of an estimating 3,000 29. Either of these stuffed toys are suitable for your little
known mineral species have been found at least four-year-old. (Real NMAT Question)
reasonably suitable for use as gems. (Real NMAT (A) these
Question)
(B) stuffed
(A) estimating (C) are
(B) been found (D) little
(C) reasonably (E) No error
(D) for use
30. After his years of dedicated service to the country,
(E) No error the politician and statesman were honoured
at a public felicitation ceremony. (Real NMAT
25. Shivering from the cold and trembling with fatigue,
Question)
the old man begged permission to lay down awhile
before moving on. (Real NMAT Question) (A) his years
(A) with fatigue (B) service to
(B) begged permission (C) and statesman
(C) lay down (D) were honoured
(D) awhile (E) No error
(E) No error 31. A representative of the Reserve Bank will provide the
students with an insight into the economic future of
26. Retiring C.E.O. Hira Byebye asked his managers to the country. (Real NMAT Question)
take interest in the management after his retirement.
(Real NMAT Question) (A) A representative
(B) provide the students
(A) Retiring C.E.O.
(C) insight into
(B) asked his
(D) of the country
(C) interest
(E) No error
(D) after his retirement.
(E) No error 32. Generally, plums has a greyish or whitish coating
that we can easily wipe off. (Real NMAT
27. The novice surpassed everybody’s expectations, Question)
providing vital leads, clinching three major deals in
(A) has
a week and reduced turnaround time to long-term
clients by nearly 40% to bag the coveted ‘Salesman (B) greyish
of the Month’ title. (Real NMAT Question) (C) whitish
(A) everybody’s (D) wipe off
(B) clinching (E) No error
(C) reduced 33. MTV, I believe, is an acronym for Music Television,
(D) coveted and this was a MTV presentation. (Real NMAT
Question)
(E) No error

405

Book 1.indb 405 30/04/2019 4:49:02 PM


NMAT by GMAC™ Official Guide 2019

(A) an
(B) for
(C) this
(D) a
(E) No error
34. You told me it’s name when I spoke about it last time,
but I have already forgotten. (Real NMAT Question)
(A) told me
(B) it’s
(C) spoke
(D) I have
(E) No error
35. When studying an assignment, the student read
quickly over the pages first, and missed the main
points and the outline. (Real NMAT Question)
(A) an assignment
(B) read quickly over
(C) missed
(D) and
(E) No error

406

Book 1.indb 406 30/04/2019 4:49:02 PM


5.0   Language Skills Practice

7 Choose the Correct Preposition


In questions 1–30, choose the set of prepositions whose meaning and sequence best fits the three given
sentences.

1. 1. Krishna said that he would arrive 3. 1.  The boy was hit _______ a stone.
2 and 3 pm. 2. He is writing _______ ink.
2. In the newly constructed shopping mall the 3. We discussed the matter _________ tea.
shops stay open 11 am to
(a) with
11 pm.
(b) by
3. The formalities are all over and the rocket is
ready for launch 30 minutes. (c) over
(a) at (d) in
(b) from (e) on
(c) on (f) into
(d) in (A) bde
(e) between (B) adf
(f) for (C) adc
(A) bde (D) def
(B) cdb (E) bac
(C) ebd
4. 1.  He was sitting ___________ his sister.
(D) eca
2. He distributed the toffees ______________ us.
(E) dbc
3. Butter is made _____________ milk.
2. 1.   The administration’s new proposals have been
(a) beside
met a lot of opposition from
the citizens of the town. (b) besides
(c) between
2.   The children are so excited to eat out that
they are unable to choose between a pizza (d) amongst
a burger. (e) from
3.   When asked to come to the party, Ramesh (f) of
declined saying that he had decided to stay
home and study instead. (A)
acf

(a) with (B)


ade

(b) or (C)
ace

(c) and (D)


bdf

(d) for (E) bce

(e) in 5. 1.  You must abstain __________ alcohol.


(f) at 2. There is no exception _________ the rule.
(A) acf 3. Your views don’t accord ________ mine.
(B) cbe (a) to
(C) abf (b) from
(D) dbe (c) with
(E) abe (d) of
(e) into
(f) for

407

Book 1.indb 407 30/04/2019 4:49:02 PM


NMAT by GMAC™ Official Guide 2019

(A) abc (A)


fce
(B) bac (B)
bcf
(C) dac (C)
eca
(D) dba (D)
bae
(E) bcf (E)
fce
6. 1.  Get ____ this bus. It goes to our destination. 9. 1.  T
 he man was entrusted ______ the care of his
2. She is standing ____the car. brother’s kids.
3. This material is different _____ that one. 2. He was convicted _____ murder and sent to
prison.
(a) by
3. He persists _____ forcing his wife to quit her
(b) on
job.
(c) from
(a) to
(d) onto
(b) with
(e) to
(c) of
(f) in
(d) on
(A)
dbc
(e) off
(B)
fac
(f) in
(C)
bac
(A)
acf
(D)
bfe
(B)
bea
(E)
cde
(C)
dfc
7. 1.  I knocked him _____ accidently.
(D)
fed
2. He goes to office ____ foot.
(E)
bcf
3. He is the boy who speaks ____ an accent.
(a) on 10. 1.  T
 he official refused to comply ______ my
request.
(b) with
(c) by 2. He could not go for a holiday for want _____
money.
(d) over
3. He managed to gain insight ______ the
(e) in working of the company barely within days
(f) from of joining.
(A)
acb (a) to
(B)
abc (b) of
(C)
dab (c) in
(D)
dcb (d) into
(E)
caf (e) with

8. 1.  He is _______ work. (f) off

2. Eighteen and _______ can watch this movie. (A) dbf


3. He made her feel special and cared ______. (B) ebd

(a) above (C) adb

(b) out of (D) fad

(c) after (E) efb

(d) on 11. 1.  Our army triumphed _______ the enemy.


(e) for 2. She always sides ________ him whenever he
(f) in and I argue.

408

Book 1.indb 408 30/04/2019 4:49:02 PM


5.0   Language Skills Practice

3. Her comments were a slur ______ the staff at (A)


abe
the hotel. (B)
bac
(a) to (C)
dac
(b) of (D)
dca
(c) in (E)
bcf
(d) over
14. 1.  I want to buy that dress but I don’t have my wallet
(e) with ______ me.
(f) on
2. He managed to get off ________ a light
(A)
dbf punishment.
(B)
ebd 3. There were at least five hundred people
(C)
adb ________ the luxury yacht.
(D)
def (a) to
(E)
efb (b) aboard
(c) with
12. 1.  We flew down to Mumbai _______ the weekend.
(d) of
2. ______ weekends, I like to watch football.
(e) into
3. One ____ five will be called for an interview.
(f) on
(a) on
(A)
fad
(b) with
(B)
bac
(c) by
(C)
dab
(d) over
(D)
dea
(e) in
(E)
fcb
(f) from
15. 1.  She splashed water ________ her face.
(A)
acb
2. He ate two pizzas and she ate one, so
(B)
abc
_________ them they ate three pizzas.
(C)
dae
3. I have to get these dollars changed _______
(D)
dcb rupees.
(E)
caf
(a) on
13. 1.  T
 here is a large proportion of people who are not (b) into
persuaded ______ his weight loss programme. (c) after
2. ______ the terms of the lease, you have no right (d) over
to make any changes to the apartment. (e) between
3. I can’t imagine what has come ______ him these (f) in
days.
(A)
fce
(a) over
(B)
aeb
(b) from
(C)
eca
(c) under
(D)
dae
(d) of
(E)
def
(e) into
(f) for

409

Book 1.indb 409 30/04/2019 4:49:02 PM


NMAT by GMAC™ Official Guide 2019

16. 1.  S
 he was adamant that she would work (a) into
__________ the night. (b) through
2. I prefer walking ___ driving. (c) behind
3. She won’t get the lead role as her age is (d) up
_________ her.
(e) out
(a) to (f) across
(b) in (A) adb
(c) beneath (B) aeb
(d) behind (C) bdb
(e) against (D) beb
(f) over (E) feb
(A)
dbf
19. 1.  H
 e just wanted to be left ___________ peace but
(B)
cfe his wife continued to nag him all evening.
(C)
bae 2. She frowns ___________ people who gossip in
(D)
fad the office during working hours.
(E)
efb 3. The principal disapproves ___________ teachers
punishing the students for minor mistakes. (Real
17. 1.  W
 ithering of rocks ______ long periods of time NMAT Question)
leads to the formation of clay minerals.
(a) of
2. Large bowls such as salad bowls are often
intended ______serve many people at the same (b) with
time. (c) over
3. The chemical compounds that make up perfume (d) in
can either be manufactured synthetically in labs (e) to
or extracted ______ the flowers or plants. (Real
(f) on
NMAT Question)
(A) bea
(a) to
(B) dfe
(b) off
(C) dfa
(c) over
(D) efa
(d) from
(E) bbc
(e) since
(f) despite 20. 1. A problem exists when we become aware of the
difference________ the reality and our desires.
(A) bac
(B) cba 2. Individuals differ ________ one another in their
ability to understand complex ideas.
(C) acb
(D) cad 3. IQ tests were mainly designed ________
identifying mentally challenged children. (Real
(E) cae
NMAT Question)
18. 1.  H
 e is so transparent, it is possible to see ______ (a) for
every ploy.
(b) from
2. Paintings should be done in bright shades
so that they may stand ______ more (c) besides
vividly.
(d) through
3. She ran ______ all of the main points of her
presentation in a few hours. (Real NMAT (e) towards
Question) (f) between

410

Book 1.indb 410 30/04/2019 4:49:02 PM


5.0   Language Skills Practice

(A) ceb 2. Rituparno is _____ the hospital.


(B) afd 3. He is mad _____ cleanliness and hygiene.
(C) fba  (Real NMAT Question)
(D) dac (a) in
(E) fac (b) on
21. 1. Good management comprises an array ______ (c) at
practices and experience.
(d) into
2. You must focus ______ your objectives.
(e) about
3. This is a serious matter and should not be
pushed ______ the carpet. (Real NMAT (f) over
Question) (A) abf
(a) at (B) acf

(b) of (C) adb


(D) cde
(c) in
(E) cae
(d) on
24. 1. Following the registration of the two complaints,
(e) under police placed Saeed ______ house arrest on
(f) although Sunday.

(A) adb 2. The news channel reported that he had been


selected ______ the seat.
(B) abd
(C) bde 3. The jail authorities stopped serving non-
vegetarian food ______ inmates.
(D) cde
(E) acb  (Real NMAT Question)
(a) to
22. 1. The manager’s eye lingered ______ the new
recruit. (b) for
2. He was looking ______ Tom for instructions. (c) after
3. She shivered ______ fear. (d) under
(Real NMAT Question) (e) inside
(a) on (f) besides
(b) under (A) bfc
(c) to (B) fbc
(C) dba
(d) at
(D) abc
(e) in
(E) dce
(f) by
25. 1. Access will require a sustained and expanded
(A) ace commitment _____ the international community.
(B) aec
2. _____ proof of Washington’s commitment to its
(C) cba key regional ally, the Senate voted on Thursday
(D) dae to triple non-military aid to Pakistan.
(E) fed
3. Micheal Chae, who migrated to Ireland, often fell
23. 1.  ­Rituparno is waiting for his friend _____ the ill, especially _____ consuming a certain species
hospital. of fish. (Real NMAT Question)

411

Book 1.indb 411 30/04/2019 4:49:02 PM


NMAT by GMAC™ Official Guide 2019

(a) as (A) adc


(b) by (B) adb
(c) from (C) bdc
(d) after (D) fcd
(e) besides (E) cda
(f) between
28. 1. It was a tough victory to pull _______ but they
(A) aef did it with aplomb.
(B) cad
2. It was disheartening to watch as they proceeded
(C) abf to quarrel and fall _______ with one another.
(D) adf
3. It is doubtful that after such a setback they will
(E) deb
ever manage to put _______ their differences
26. 1. People protested about the factory as its high and play together on the stage again.
emissions led ________ widespread pollution. (Real NMAT Question)
(a) out
2. She procured a stay order ________ the court.
(b) about
3. A campaign against female foeticide was (c) up
launched ________ some well-meaning activists.
(d) off
(Real NMAT Question)
(e) aside
(a) by
(f) since
(b) to
(A) cae
(c) off
(B) caf
(d) from
(C) dbe
(e) unto
(D) dae
(f) with
(E) baf
(A) faf
(B) acf 29. 1. He is in a hurry as he is leaving ______ Japan on
the evening flight.
(C) bda
(D) cad 2. The road to perdition is paved _______ good
(E) daf intentions.

27. 1. The first credible signs ______ the presence 3. I have been working on this novel _________
of the snow leopard have been found in many years now. (Real NMAT Question)
Uttarakhand. (a) through
2. The role of two senior bureaucrats came (b) for
________ the scanner. (c) to
(d) with
3. Subsequent _______ the Court’s orders, hearing
on the issue has been initiated. (Real NMAT (e) by
Question) (f) since
(a) of (A) bbc
(b) to (B) bdb
(c) off (C) bad
(d) under (D) bbe
(e) towards (E) bbf
(f) beneath

412

Book 1.indb 412 30/04/2019 4:49:03 PM


5.0   Language Skills Practice

30. 1. I live _____ Mumbai.

2. I stay _____ Opera House Street.

3. I am going _____ watch the play tonight.


(Real NMAT Question)
(a) on
(b) at
(c) in
(d) onto
(e) to
(f) for
(A) ace
(B) abd
(C) cad
(D) cbe
(E) cdf

413

Book 1.indb 413 30/04/2019 4:49:03 PM


NMAT by GMAC™ Official Guide 2019

8 Reading Comprehension
Each of the reading comprehension questions is based on the content of a passage. After reading the
passage, answer all questions pertaining to it on the basis of what is stated or implied in the passage.
For each question, select the best answer from the given choices.

Passage 1 (C) describe how parents and schools are making


women weak by encouraging them to focus only
If we look at the world around us and all that is happening on superficial beauty.
therein, we would be tempted to believe that either nature
has made great difference between men and women (D) argue that women are meant to and should be
or that past civilisations have been extremely partial encouraged to serve a higher purpose in society
towards men. In every sphere of life, women are routinely than just look attractive.
discriminated against and treated as inferior to men. While (E) propose a restructuring of the roles of women in
men are spoken about for their intellect and leadership, society by shifting the focus from attractiveness
women are only spoken about for their beauty. What to more noble ambitions.
makes matters worse is that it is women themselves who
are partly to be blamed for this state of affairs because 2. According to the passage, who among the following
they pay too much attention to their external appearance is responsible for the current state of women in the
and too little attention to what lies within their heads. The society?
myriad books authored by men, which portray women i. The parents of women
as beguiling mistresses rather than as devoted wives,
ii. The women themselves
have only fuelled this perception. Thanks to the specious
impression created by these books, women (with the iii. Books by some male writers
exception of a select few) become intoxicated by all the (A) i only
attention being showered on them and end up believing (B) ii only
that their purpose in life is to win the love and approval
(C) ii and iii only
of men, when they should actually be devoting their lives
to more noble pursuits in order to earn the respect of the (D) i and iii only
society. (E) i, ii and iii only
While I don’t intend to vigorously fight the people who view
3. What is the meaning of ‘specious’, as used in the
women as inferior to them, I definitely want to make my
passage?
stand on the issue clear. There is no denying the fact that
nature has made women inferior to men, at least physically. (A) large
Men have always been the protectors whereas women (B) deceptive
have been simply expected to yield, in no small measure
(C) accommodating
due to them being physical weaker. This is the way nature
made things and I am fine with it. What I have a problem (D) unconvincing
with is that men have taken this physical inferiority much (E) malicious
further and ordained women as the overall weaker sex who
are merely objects of allure. This is simply not correct as 4. Which of the following assertions would the author of
women can often be mentally much stronger than men. the passage most likely agree with?
Women also have a much greater capacity for bearing pain,
   i. Men are physically superior to women.
as any woman who has been through labour will testify. I
only request my fellow women to not get taken in by all the      ii. Women enjoy the attention lavished on them by men.
attention men shower on them for their beauty and try to rise
above such superficial concerns and make their mark on this iii. Women should rebel against the injustices meted
world. out to them by men.

1. The primary purpose of the passage is to: (A) i only


(B) iii only
(A) question male writers for not focusing on the
issues of women in their literary works. (C) ii and iii only
(B) reconcile two contrasting opinions about the role (D) i and ii only
of women in our society. (E) i, ii and iii only

414

Book 1.indb 414 30/04/2019 4:49:03 PM


5.0   Language Skills Practice

Passage 2 2. According to the passage, which of the following


could be a reason we are attracted towards the
In these modern times of fast food lunches, take-away dinners dishes made by our mothers?
and ready-to-eat breakfasts, we often reminisce about the
dishes that our mothers made when we were young. The (A) They were made using ingredients that are not
aromas and flavours of those dishes are still fresh in our available today.
minds, though several years have passed since we last had (B) They satisfied our hunger much better than
these. Is it the flavour that actually makes us remember these modern dishes do.
dishes? Or is it that these dishes are connected to other (C) They did not involve the use of artificial
pleasant memories of those times and of our younger selves, flavours.
which have left an indelible impression on our minds?
(D) They serve as reminders of our younger
If we think about how our mother’s garden used to be, selves.
there was nothing remarkable about it, at least at first (E) They were made from home grown
glance. What did stand out was that everything—flowers, vegetables.
fruits and vegetables—was neatly laid out and pleasing to
the eye. Widely contrasting fruits and vegetables such as 3. According to the passage, which of the following was
strawberries and cucumber, poppies and potatoes, turnips a unique feature of mother’s garden?
and tomatoes grew in perfect harmony in mother’s garden.
(A) The availability of rare herbs and vegetables.
One group of plants that one particularly remembers among (B) The use of sweet herbs.
this eclectic mix is the “sweet” herbs placed just outside
(C) The addition of mother’s love.
the kitchen. These are items that we never see on the table
and that never play leading roles in the dramatic act that (D) The effort that went into preparing the dishes.
is mother’s cooking. Nonetheless, these are absolutely (E) The neat and organised layout.
essential ingredients of mother’s food, just like members
4. From the information in the passage, which of the
of the cast who play small but important roles and without
following is not true of the herbs discussed in the last
whom the act would lose a lot of its colour and charm. It is
paragraph of the passage?
these performances that one remembers long after one has
forgotten the names of the leading cast members. The herbs (A) They tasted sweet.
in mother’s garden are no different. (B) Their memory lingered long after the food was
1. The author compares herbs in mother’s garden with eaten.
members of ‘the cast’ primarily in order to: (C) They were usually not the main ingredient of a
dish.
(A) describe the supporting role played by these
herbs in preparing a dish. (D) They added flavour to a dish.
(B) identify a similarity between these herbs and (E) They were not laid out on the dining table.
other fruits and vegetables.
(C) analyse the relationship between herbs and
cooking.
(D) explain how these herbs play a vital role in
providing flavour to a dish.
(E) point out the key differentiating ingredients in
mother’s dishes.

415

Book 1.indb 415 30/04/2019 4:49:03 PM


NMAT by GMAC™ Official Guide 2019

Passage 3 iii. If a person is taking a medication that prevents


the release of adrenaline in the human body.
A clutch of new studies in the field of the psychology of
emotion offer opportunities for humans to have better (A) ii and iii only
control over their emotions. Unfortunately, some other (B) i and ii only
research has proven conclusively that certain emotions (C) iii only
that are triggered spontaneously, most notable among
them being anger, are often accompanied by the release of (D) ii only
various types of chemicals and hormones. These chemicals (E) i, ii and iii
and hormones, interestingly, help to further sustain the
2. According to the passage, a rational person differs
emotional outburst. To take an example, anger causes the
from a highly impulsive person in that:
release of adrenaline, and this adrenaline further helps to
sustain, and even magnify, that aroused state of anger. (A) a rational person’s body produces a controllable
To make matters worse, the rate of dissipation of this amount of hormones.
adrenaline is not within the control of the conscious human (B) a rational person is able to better control his
mind. Hence, when such emotions get triggered, they tend judgment of events.
to circumvent our conscious mind, a fact that has several
(C) a rational person does not let his hormones
vital implications for emotional impulse control.
affect his emotions.
The solution then is to somehow be able to avoid setting off (D) a rational person does not allow emotions to
such impulses that are beyond the control of our conscious bypass his conscious mind.
mind, something that is easier said than done. One way of (E) a rational person has no less a tendency to jump
doing so is to get a better understanding of how we perceive to conclusions.
different events around us. The idea is to view everything with
an objective mindset and avoid the ‘jumping to conclusions’ 3. What is the meaning of the word ‘misconstrue’ as
syndrome, which can trigger these unwanted impulses. To used in the passage (paragraph two)?
illustrate, it is possible for two people to view the same event (A) Mislead
but reach entirely different conclusions about its connotation.
(B) Misdemeanor
Let’s say if we see a group of people pointing in our direction
and laughing, many of us will misconstrue such an act, (C) Misguide
thinking that those people are laughing at us and become (D) Misinterpret
irritated or angry. Such a reaction could then trigger the (E) Misrepresent
adrenaline-fuelled anger, and we could even end up getting
into a fight with those people. However, a smarter, and less 4. Why does the author use the term 'unfortunately' in
impulsive, approach could be to simply view the event for the second sentence of the passage?
what it is—a group of people pointing at something and
laughing. For all we know, they could be pointing at something (A) To underscore the inherent sorrow that a
behind us. Such people will not jump to conclusions; rather, person who is unable to contain his emotions,
they will wait for more clarity on the actions of those people faces.
before coming up with a reaction. Remember that even (B) To provide a contrast with the previous sentence
though we cannot control the action of those people, we can by stating that in some cases a person may not
always control our reaction to their actions, by addressing be able to voluntarily control his emotions in the
them calmly or even simply ignoring them. first place.
(C) To assert that a person not being able to
1. According to the passage, in which of the following contain his emotions is not the desired state
hypothetical cases would a person be able to of affairs.
consciously control his or her emotions?
(D) To conclude that all the efforts of researchers
   i. If a person becomes angry at the slightest of have gone in vain because there is actually a
provocations but no hormone is released. connection between hormones and emotions.
(E) To arrive at a conclusion about the relationship
     ii. If a person consumes a recently developed drug between hormones and emotions later in the
that greatly increases the rate of dissipation of passage.
hormones in the human body.

416

Book 1.indb 416 30/04/2019 4:49:03 PM


5.0   Language Skills Practice

Passage 4 1. What is the primary purpose of the passage?

Once a charitable foundation bestows a grant of money to (A) To describe why it is imperative for charitable
a chosen recipient, or “beneficiary,” the foundation’s main foundations to keep a track of the activities of
defence against misuse/waste of its grant is the individual the beneficiaries they have funded.
contractual agreement between foundation and beneficiary. (B) To argue for the creation of more standardised
These contracts tend to be idiosyncratic, varying with the contracts between charitable foundations and
specific purposes of the foundation and the grant; thus, they their beneficiaries.
have not been the focus of much academic study. However, (C) To explain and support the proactive role certain
scholars have identified informal non-contractual control charitable organisations play in the activities of
mechanisms by which foundations guard against misuse of their beneficiaries.
their investment. Such mechanisms arise at three points: (D) To discuss the different ways in which charitable
in the initial screening of projects, in the decision of how foundations can control the misuse of their funds
much funding to allocate, and in the contract between the by the beneficiaries.
foundation and the beneficiary.
(E) To provide an explanation for the seemingly
Needless to say, a foundation’s power over its beneficiaries’ overbearing behaviour of charitable
activities is greatest before it has committed money to organisations towards their beneficiaries.
them. Foundations can best protect against unsatisfactory
use of their grants by awarding grants only to beneficiaries 2. According to the passage, each of the following is a
which intend to use the money for projects which the method used by charitable foundations to control the
foundation encourages. Screening may be done in two ways. use of their funds EXCEPT:
First, foundations can issue public statements on types of (A) making the grant of funds dependent on the
programmes they will—or will not—fund. This will prevent fulfillment of certain obligations and deliverables
mistaken misuse by recipients, though not fraudulent abuse. on the part of the beneficiary.
Foundations may issue absolute (negative) prohibitions;
(B) explicitly detailing the types of expenditure that
for example, a clause that no money will be given for
they will reimburse or not reimburse.
personal charity or religious education. Alternately, they may
affirmatively announce types of projects that they will fund. (C) prohibiting certain types of beneficiaries from
applying for their funds.
Second, foundations can take a proactive role, by working (D) acting as mentors to their beneficiaries and
with promising applicants to propose new projects or define guiding them in making appropriate use of the
the goals/parameters of existing ones. A foundation exercises funds.
considerable power through its grasp of the purse strings and
(E) giving out the total amount of the grant in a
the manner in which it doles out its grant. Most fundamentally,
staggered manner.
a foundation can specify what type of expenses it will, or
will not, reimburse. Furthermore, foundations can grant 3. According to the various criteria mentioned in the
themselves more discretion, and retain more of their passage, which of the following entities will most
bargaining power with beneficiaries, by disbursing only part of probably not be funded by a charitable organisation?
the total projected cost at regular intervals. This allows them
to monitor a project to ensure it has not strayed from the    i. An entity that requires the entire amount of the
desired parameters, and it also encourages beneficiaries to grant to be paid upfront.
meet deadlines and disclose results.
     ii. An entity that plans to undertake an activity
Once the foundation has disbursed some or all of its funding prohibited by the charitable organisation.
and the beneficiary’s project is underway, the foundation
may want to continue monitoring progress to prevent misuse iii. An entity that refuses to follow the subsequent
of funds or unwanted deviations from the originally-planned instructions of the charitable organisation arising
project. This supervision can be accomplished through over the course of its operations.
several methods. The foundation can specify in its agreement (A) ii only
with the beneficiary that its grant is a conditional grant,
(B) iii only
contingent on specified uses. An alternate approach is for the
foundation to appoint monitors to work with the beneficiary (C) ii and iii only
throughout the project. This is a common practice for venture (D) i, ii and iii
capital “foundations” in monitoring the start-up companies (E) i and ii only
which are the beneficiaries of their seed capital.

417

Book 1.indb 417 30/04/2019 4:49:03 PM


NMAT by GMAC™ Official Guide 2019

4. What is the meaning of the term ‘discretion’ as used


in the 3rd paragraph of the passage?
(A) Secrecy
(B) Responsibility
(C) Authority
(D) Credibility
(E) Gullibility

418

Book 1.indb 418 30/04/2019 4:49:03 PM


5.0   Language Skills Practice

Passage 5 War, the Guinness World Records website offers a telling


glimpse into the future of fact finding and record recording.
The biggest house of cards, the longest tongue and, of
course, the tallest man—these are among the thousands of 1. What is the main purpose of the author in writing the
records logged in the famous Guinness Book of Records. passage?
Created in 1955 after a debate concerning Europe’s fastest (A) To discuss the origins of the Guinness Book and
game bird, what began as a marketing tool sold to pub its evolution over the years.
landlords to promote Guinness, an Irish drink, became
the bestselling copyright title of all time (a category that (B) To criticise the fact that the Guinness Book, in
excludes books such as the Bible and the Koran). In its current form, stresses more on entertainment
time, the book would sell 120 million copies in over 100 than on education.
countries—quite a leap from its humble beginnings. (C) To explain the origin of the Guinness Book.
(D) To analyse what makes the Guinness Book the
In its early years, the book set its sights on satisfying man’s largest selling book in the world.
innate curiosity about the natural world around him. Its two
principal fact finders, twins Norris and Ross McWhirter, (E) To discuss how the Guinness Book originated
scoured the globe to collect empirical facts. It was their task and to advocate against the commercialisation
to find and document aspects of life that can be sensed or of the book in its current form.
observed, things that can be quantified or measured—but
2. Which of the following is mentioned in the passage
not just any things. They were only interested in superlatives:
as a record contained in the Guinness Book?
the biggest and the best.
(A) The world’s largest combustion engine.
In its latest incarnation, the book has found a new home on
(B) The world’s longest train.
the Internet. No longer restricted to the confines of physical
paper, the Guinness World Records website contains (C) The world’s heaviest train.
seemingly innumerable facts concerning such topics as the (D) The world’s widest mouth.
most powerful combustion engine or the world’s longest (E) The world’s strongest beard.
train. What is striking, however, is that such facts are found
sharing a page with the record of the heaviest train to be 3. According to the author, how is the current version of
pulled with a beard. While there is no denying that each of the Guinness Book different from its older versions in
these facts has its own individual allure, the latter represents terms of content?
a significant deviation from the education-oriented facts of
(A) The book is now available on the Internet.
earlier editions. Perhaps, there is useful knowledge to be
gleaned regarding the tensile strength of a beard, but this (B) The book now contains entertainment-oriented
seems to cater to an audience more interested in seeking facts along with educational ones.
entertainment than education. (C) The book now offers insight into the full
spectrum of life.
Originating as a simple bar book, the Guinness Book of
(D) The focus of the book has shifted from providing
Records has evolved over decades to provide insight into
education to providing entertainment.
the full spectrum of modern life. And although one may be
more likely now to learn about the widest human mouth than (E) The Guinness Book offers a telling glimpse into
the highest number of casualties in a single battle of the Civil the future of fact finding and record recording.

419

Book 1.indb 419 30/04/2019 4:49:03 PM


NMAT by GMAC™ Official Guide 2019

Passage 6 atrocities that he did commit, Agathocles only gained power


but no glory. Thus, he failed to become part of the clique of
There is no doubt that Machiavelli’s The Prince was very famous men who had earned glory in Machiavelli’s eyes.
different from other political texts of the Classical era in
that it veered more towards the shrewd and pragmatic 1. Which of the following can be inferred from the passage
modern form of politics. However, there is sufficient proof about most political theory in the Classical era?
that Machiavelli was not the creator of this political system. (A) It was not favoured by several important men of
In fact this system, called ‘realpolitik’ or politics without that time.
principles, was actually in practice long before Machiavelli’s
time. Machiavelli just made this system more popular and (B) It was considered mild and less aggressive in
mainstream. comparison to other political theories of that time.
(C) It was rooted more in practicality than in
One example of the use of realpolitik before Machiavelli’s principles.
time is of the evil Spanish imperial advisor Ortega Sorolla, (D) It was eventually abandoned in favour of
who openly stated that the ruling emperor was to be realpolitik countries.
supported at all costs and who imprisoned or executed
anyone who posed a challenge to the emperor. In fact, (E) At least some of its tenets were not rooted in
several historians believe that Sorella actually got several practicality.
exceptionally talented men secretly assassinated as he
2. Why does the author mention Chapter XII in the
was afraid that one day they would pose a threat to the
passage?
emperor’s throne.
(A) To explain that Machiavelli never actually
Supporters of Machiavelli believe that power was a tangible described what were the characteristics of a
entity, which could be reflected or ascertained by the size good law.
of the army that a ruler commanded. The ruler’s objective
(B) To show that Machiavelli considered arms
was to instill in the minds of his followers the fact that
superior to laws.
they should always think about war and how they would
be victorious in one, as neglecting this could often lead to (C) To highlight the low level of importance
losing one’s kingdom. For Machiavelli, the truly powerful Machiavelli assigned to the presence of good
kingdoms were those that could use their money and clout laws in a state.
to command large armies and use these armies to subjugate (D) To prove that good governance never formed a
other kingdoms. These conquered kingdoms then deservedly part of Machiavelli’s teachings.
became objects of ridicule and contempt. (E) To explain how Machiavelli was only concerned
with the presence of laws, not with their quality.
Machiavelli’s lack of focus on the quality of governance
within a state is clear from the fact that it is only as late 3. Which of the following assertions would Machiavelli
as in chapter XII that he talks about the importance of most likely agree with?
having good laws within a state. This too seems to be an
afterthought since Machiavelli pays no particular attention i. There is nothing wrong in showing contempt
to the quality of these laws. He simply makes open ended towards those who have lost in battle.
statements and never really goes down to describing in
details what attributes make up a ‘good’ law. ii. The king should perform only two functions—fight
good wars and make good laws.
According to Machiavelli, whatever actions a prince
undertakes are with the single mined intention of gaining a iii. A ruler who committed atrocities would still gain
good reputation in history books, even if that reputation is glory as long as he won wars.
earned after the death of the prince. Machiavelli narrates the (A) i only
example of Agathocles, who rose from poverty to become
(B) iii only
the king of Syracuse by killing all those who stood in his
path. This made Agathocles truly powerful in Machiavelli’s (C) i and iii only
eyes. However, Machiavelli still doesn’t consider Agathocles’ (D) ii and iii only
achievements as perfect because, by committing the (E) i and ii only

420

Book 1.indb 420 30/04/2019 4:49:03 PM


5.0   Language Skills Practice

4. Which of the following is a point of similarity between


Sorolla and Machiavelli?
(A) Both of them believed in the supreme position of
the emperor.
(B) Both of them believed that victory in a war was
paramount, no matter at what cost it came.
(C) Both of them believed that maintaining a strong
and able army was more important than making
good laws.
(D) Both of them believed that it was acceptable to
go to any lengths in the single minded pursuit of
one’s goal.
(E) Both of them were a creation of the initial
monopoly of the Classical political theory.

421

Book 1.indb 421 30/04/2019 4:49:03 PM


NMAT by GMAC™ Official Guide 2019

Passage 7 1. The author of the passage is most likely to agree


with which of the following?
It is not uncommon for close synonyms to be understood
to share the same meaning. The difference between (A) Two words can never be complete synonyms.
words like ‘hard’ and ‘difficult’, for example, goes tragically (B) The dictionary does not contain all the meanings
unnoticed. One may employ one or the other with complete of a word.
indifference, postulating no discrepancy between them. (C) The English language is inherently superfluous.
In general, this is well and good; most people lack the (D) Two words can never be used interchangeably.
scrupulous pedanticalness to quibble over such trifles.
Nevertheless, for those of us with ample compulsiveness (E) It is pedantic to quibble over the difference in
(and time), it is of significant value to comprehend such meaning between two close synonyms.
nuances.
2. What is the primary purpose of the passage?
Take for example the following sentences: (a) The test was (A) To explain the limitations of the dictionary.
hard. (b) The test was difficult. Is the difference between
(B) To highlight the importance of users of the
these synonyms readily apparent? Is there a noticeable
English language in conveying the feelings and
difference between them at all? Indeed, these questions are
moods attached to a word.
valid and warrant answer. For, what would be the point to
having multiple words with the exact same meaning? No, (C) To prove why two words can never be exact
and that would be superfluous with the English language synonyms.
being far too economical. While many close synonyms (D) To discuss the importance of subtle feelings
share similar, if not the same, dictionary definitions, the and moods attached to words having similar
feeling, or mood, they convey is utterly singular. Although meanings.
a dictionary can provide information about word meanings, (E) To criticise those who believe that synonyms can
pronunciations, etymologies, inflected forms, derived forms always be used interchangeably.
and so on, it cannot communicate how it feels to use a word.
3. According to the passage, each of the following is
Granted, the notion that close synonyms can be used true of a dictionary, EXCEPT:
interchangeably is prevalent among English speakers. And
alas, the dictionary—the text purported to be responsible (A) It can help pronounce words correctly.
for clarifying such issues—is of little assistance. In the end, (B) It provides information about the origin of words.
it is left to us, the speakers of the language—those actively (C) It does not communicate the feeling or mood
responsible for maintaining its sustenance and generation— attached to a word.
to understand how these words make us feel and what mood
(D) It provides all the meanings of a word.
we are inclined to attach to them. Using the examples and
insights described earlier, one may come to recognise these (E) It helps choose which word to use from amongst
subtle, yet crucial, differences. closely related synonyms.

422

Book 1.indb 422 30/04/2019 4:49:03 PM


5.0   Language Skills Practice

Passage 8 (A) To explain how fingerprints can have


humanitarian uses such as helping to identify
Criminal identification by means of fingerprints is one of victims of natural disasters.
the most potent factors in obtaining the apprehension (B) To illustrate the importance of fingerprints in
of fugitives who might otherwise escape arrest and different fields.
continue their criminal activities indefinitely. This type
of identification also makes possible an accurate (C) To state that the Civil File of the FBI has more
determination of the number of previous arrests and fingerprints than does the Criminal File.
convictions, which of course results in the imposition of (D) To point out that fingerprints have another more
more equitable sentences by the judiciary, in as much important use that is different from the one that
as the individual who repeatedly violates the law finds is commonly perceived by the people.
it impossible to pose successfully as a first or minor (E) To conclude that fingerprints play an important
offender. In addition, this system of identification enables secondary role in the civil society as well.
the prosecutor to present his case in the light of the
offender’s previous record. It also provides the probation 2. From the information in the passage, which of the
officers, parole board and the governor with definite following can be most properly inferred?
information upon which to base their judgement in dealing
(A) Fingerprint impressions obtained with the use
with criminals in their jurisdictions.
of ink are better than those obtained using
From earliest times, fingerprinting, because of its peculiar substances such as dirt or grease.
adaptability to the field, has been associated in the lay (B) Fingerprints have many more uses in criminal
mind with criminal identification to the detriment of the investigation than they do in civil cases.
other useful phases of the science. However, the Civil (C) The matching of fingerprints is the most foolproof
File of the Identification Division of the Federal Bureau of way of apprehending hardened criminals.
Investigation contains three times as many fingerprints as
(D) The use of fingerprint records helps the justice
the Criminal File. These civil fingerprints are an invaluable
system deliver more equitable sentences.
aid in identifying amnesia victims, missing persons and
unknown deceased. In the latter category, the victims of (E) The presence of saline substances on the ridges
major disasters may be quickly and positively identified if of fingertips makes it difficult to get a definite
their fingerprints are on file, thus providing a humanitarian fingerprint impression.
benefit not usually associated with fingerprint records.
3. What is the main purpose of the author in writing the
The use of fingerprints for identification purposes is based passage?
upon distinctive ridge outlines which appear on the bulbs on (A) To explain the need for collecting fingerprints.
the inside of the end joints of the fingers and thumbs. These
(B) To advocate the study of civil fingerprints to
ridges have definite contours and appear in several general
assist the judiciary.
pattern types, each with general and specific variations of
the pattern, dependent on the shape and relationship of the (C) To analyse the implications of maintaining a
ridges. The outlines of the ridges appear most clearly when database of fingerprints.
inked impressions are taken upon paper, so that the ridges (D) To praise the advocates for bringing to justice
are black against a white background. This result is achieved the repeat offenders.
by the ink adhering to the friction ridges. Impressions may (E) To discuss the history, usefulness and mode of
be made with blood, dirt, grease or any other foreign matter collecting fingerprints.
present on the ridges, or the saline substance emitted by
the glands through the ducts or pores which constitute their 4. According to the information in the passage, the
outlets. The background or medium may be paper, glass, impressions of the fingerprints can be obtained from
porcelain, wood, cloth, wax, putty, silverware or any smooth, all of the following, EXCEPT:
non-porous material.
(A) Saline substances
1. In the second paragraph, what is the function of the (B) Ink
sentence However, the Civil File of the Identification (C) Grease
Division of the Federal Bureau of Investigation
(D) Wax
contains three times as many fingerprints as the
Criminal File? (E) Perspiration

423

Book 1.indb 423 30/04/2019 4:49:04 PM


NMAT by GMAC™ Official Guide 2019

Passage 9 1. According to the passage, integration of images


from two ‘eyes’ is termed as:
The Cyclopses, according to mythology, were a race of bad-
tempered and rather stupid one-eyed giants. Not, perhaps, (A) Computer processing
a great portend for a new generation of robots. But Andrew (B) SLAM
Davison, a computer scientist at Imperial College, London, (C) Stereoscopic vision
thinks one eye is enough for a robot, provided its brain (D) Sensors
can think fast enough. For a robot to work autonomously,
it has to understand its environment. Stereoscopic vision, (E) Autonomous angles
integrating the images from two ‘eyes’ looking at the same
2. From the passage, each of these can be inferred,
thing from different angles, is one approach to achieve this,
EXCEPT:
but it involves a lot of complicated computer processing.
The preferred method these days, therefore, is Simultaneous (A) Digital cameras are cheaper than range
Localisation and Mapping (SLAM), which uses sensors such finders.
as laser-based range finders that ‘see’ by bouncing beams of (B) Range finders allow robots to see with one eye.
light off their surroundings and timing the return.
(C) The Cyclops is a mythical creature.
Dr. Davison, however, wants to replace the range finders, which (D) To work independently, a robot must be able to
are expensive and fiddly, with a digital camera, which is small, understand its surroundings.
cheap and well-understood. With this in mind, he is developing (E) Range finders have the ability to create 3D
ways to use a single, moving video camera to create continually maps.
updated 3D maps that can guide even the most hyperactive of
robots on its explorations. His technique involves collecting and 3. According to the passage, why is a digital camera
integrating images taken from different angles as the camera preferred over range finders?
goes on its travels. The trick is to manage to do this in real
(A) Development of images is better.
time, at frame rates of 100–1,000 per second.
(B) It is small and economical and well-understood.
The shape of the world pops out easily from laser data (C) It is more fiddly.
because it represents a direct contour map of the surrounding
(D) It can continuously update images.
area. A camera captures this geometry indirectly, and so
needs more (and smarter) computation if it is to generate (E) It can upload 3D maps.
something good enough for a self-directing robot. The
4. What is the main purpose of the author in writing the
answer is a form of triangulation, tracking features, such as
passage?
points and edges, from one frame to the next. With enough
measurements of the same set of features from different (A) To explain why SLAM is better than stereoscopic
viewpoints, it is possible, if you have a fast enough computer vision.
programme, to estimate their positions and thus, by inference, (B) To advocate the use of digital cameras in place
the location of the moving camera. of range finders.
However, developing such a programme is no mean feat. (C) To analyse emerging techniques in computers.
In the milliseconds between successive frames, relevant (D) To praise a scientist for his groundbreaking
information from each fresh image must be extracted and work.
fused with the current map to produce an updated version. (E) To discuss techniques for use in self-guided
The higher the frame rate, the less time there is to do this
robots.
work.

424

Book 1.indb 424 30/04/2019 4:49:04 PM


5.0   Language Skills Practice

Passage 10 1. Which of the following cannot be inferred from the


information in the passage?
A science fiction writer coined the useful term ‘cyberspace’ in 1982,
but the territory in question, the electronic frontier, is about 130 (A) The term ‘cyberspace’ has been in use for over
years old. Cyberspace is the ‘place’ where a telephone conversation a century.
appears to occur. Not inside your actual phone, but the plastic (B) People have used cyberspace to make profits.
device on your desk. Not inside the other person’s phone, but in (C) Cyberspace is not really a physical place.
some other city. The place between the phones. The indefinite place (D) Regulations have been made governing
out there, where the two of you, two human beings, actually meet cyberspace.
and communicate. Although it is not exactly ‘real’, ‘cyberspace’
is a genuine place. Things happen there that have very genuine (E) Some people have used cyberspace to commit
consequences. This ‘place’ is not ‘real’, but it is serious, it is earnest. crimes.
Tens of thousands of people have dedicated their lives to it, to the
2. What is the main purpose of the third paragraph in
public service of public communication by wire and electronics.
the passage?
People have worked on this ‘frontier’ for generations now. (A) To underline the importance of cyberspace.
Some people became rich and famous from their efforts, while
(B) To delineate the underlying threat of cyberspace
some just played in it, as hobbyists. Others soberly pondered
to normal man.
it, and wrote about it, and regulated it, and negotiated over it in
international forums and sued one another about it, in gigantic, (C) To explain how cyberspace means different
epic court battles that lasted for years. And, almost since the things to different people.
beginning, some people have committed crimes in this place. (D) To discuss the changes that have taken place in
cyberspace in the last two decades.
But in the past 20 years, this electrical ‘space’, which was once thin
(E) To discuss the political and social impact of
and dark and one-dimensional—little more than a narrow speaking
cyberspace.
tube, stretching from phone to phone—has flung itself open like a
gigantic jack-in-the-box. Light has flooded upon it, the eerie light of 3. Which of the following would the author of the
the glowing computer screen. This dark electric netherworld has passage not agree with?
become a vast flowering electronic landscape. Since the 1960s,
the world of the telephone has crossbred itself with computers and (A) Cyberspace includes email, social media
television, and though there is still no substance to cyberspace, websites, e-commerce and so on.
nothing you can handle, it has a strange kind of physicality now. It (B) It would be incorrect to consider cyberspace an
makes good sense today to talk of cyberspace as a place all its individual entity.
own because people live in it now. Not just a few people, not just (C) Books have been written about cyberspace.
a few technicians and eccentrics, but thousands of people, quite
(D) Modern cyberspace is an amalgam of
normal people—and not just for a little while either, but for hours
telephones and computers.
straight, over weeks, and months and years. Cyberspace today
is a ‘Net’, a ‘Matrix’, international in scope and growing swiftly and (E) Thousands of people are involved with
steadily. It is growing in size, wealth and political importance. cyberspace in some way or the other.

425

Book 1.indb 425 30/04/2019 4:49:04 PM


NMAT by GMAC™ Official Guide 2019

Passage 11 2. The author predicts that India:

The old civilisation of India was a concrete unity of many-sided (A) will rule the world sometime in the future.
developments in art, architecture, literature, religion, morals (B) will resurface as a philosophical power.
and science, so far as it was understood in those days. But the (C) will actively contribute to the progress of
most important achievement of Indian thought was philosophy. mankind.
It was regarded as the goal of all the highest practical and (D) will contribute again to spirituality.
theoretical activities, and it indicated the point of unity amidst
all the apparent diversities which the complex growth of culture (E) will progress due to its political machinery.
over a vast area inhabited by different peoples produced.
3. What is the primary purpose of the author in writing
It is not in the history of foreign invasions, in the rise of the passage?
independent kingdoms at different times, in the empires of this (A) To compare Indian philosophy with that of other
or that great monarch that the unity of India is to be sought. It is civilisations.
essentially one of spiritual aspirations and obedience to the law of
(B) To analyse the connection between Indian
the spirit, which were regarded as superior to everything else, and
civilisation and philosophy.
it has outlived all the political changes through which India passed.
(C) To trace the impact of political invasions on
The Greeks, the Huns, the Scythians, the Pathans and the Indian philosophy.
Mughals, who occupied the land and controlled the political (D) To discuss the points of unity in India.
machinery, never ruled the minds of the people. These political
(E) To discuss the rise of spiritual aspirations in
events were like hurricanes or the changes of season, mere
India.
phenomena of a natural or physical order which never affected
the spiritual integrity of Hindu culture. If, after a passivity of some 4. From the passage, which of the following cannot be
centuries, India is again going to become creative, it is mainly on concluded about Indian philosophy?
account of this fundamental unity of her progress and civilisation
and not for anything that she may borrow from other countries. (A) It has a goal of bringing together different
people.
1. Each of the following can be inferred from the (B) It brought together practical and theoretical
information in the passage EXCEPT: activities.
(A) India has faced various foreign invasions in the (C) It was immune to political upheavals.
past. (D) Spirituality is an important part of it.
(B) Philosophy was held in high regard in older (E) It is not subjugated by time.
civilisations.
(C) There has been complacency in Indian
philosophical thought.
(D) Unity of India was affected due to its vast
geographical area.
(E) Political invasion has not had any impact on
India’s spiritual integrity.

426

Book 1.indb 426 30/04/2019 4:49:04 PM


5.0   Language Skills Practice

Passage 12 (Real NMAT Question) Over the next couple of years, Dickens published two
Christmas stories including the classic A Christmas Carol.
As an author of short stories, plays and novels, Charles From 1849 to 1850, Dickens worked passionately on his
Dickens became known the world over for endearing favourite novel David Copperfield, a sort of autobiography
characterisation, vivid narration of ordinary lives, and and probably the first work of its kind.
depiction of the social and moral values of his time. For
years, Dickens thrilled the readers with his simple stories The death of his daughter and father and separation from
about simple people forced into real situations. his wife in the 1850s cast a dark shadow on Dickens’ writing
during this period. He returned to his original style with A
Although Dickens wanted to make a lot of money from Tale of Two Cities in 1859, a historical novel, followed by
his writings, his works invariably aimed to influence the Great Expectations in 1861, widely considered his greatest
consciousness of his readers even though it seemed like he literary feat.
knew what his readers wanted.
On June 9, 1870, Dickens suffered a stroke and died at his
To some critics, Dickens was an entertainer and his country home in Kent, England.
novels lacked intellectual challenge, but the London Times
described the British author as the greatest instructor of 1. The author most likely agrees with all of the following
the nineteenth century in his obituary. The unparalleled statements EXCEPT:
sentimentality, rage, plight and bitterness in his novels (A) Dickens blamed his parents for enduring a harsh
established Dickens as a spokesman for the downtrodden. childhood.
Dickens was born on February 7, 1812 into a poor family (B) Dickens was not impressed by the materialism
in Portsmouth on the southern coast of England. He was of the American society.
the second of eight children. In 1822, the Dickens family (C) Dickens’ novels portrayed the attitude of the
moved to Camden Town, a poor neighbourhood in London. society towards the downtrodden.
The extravagance of his father often brought financial (D) To some of his critics Dickens was an
embarrassment to the family and eventually imprisonment entertainer and his writing was not intellectually
for him in 1824. This brought an abrupt end to Dickens’ challenging.
childhood. He had to discontinue his schooling and support
his family by doing manual work at a factory. Young Dickens (E) 
David Copperfield, a biographical account of
was rudely introduced to long hours of harsh working Dickens’ life, is often cited as his best literary
conditions and poor pay. He felt orphaned and betrayed by achievement.
his parents who he believed should have taken care of him
2. The passage is primarily concerned with:
in his childhood. These feelings later became a recurring
theme of many of his novels. Characters like Oliver Twist and (A) Dickens’ novels and his characters.
David Copperfield developed from the harsh experiences of (B) Dickens’ success as a writer.
Dickens’ early life.
(C) Dickens’ harsh childhood.
Dickens’ literary journey started in 1833 when he submitted (D) Dickens’ life story.
sketches to various magazines and newspapers under the
(E) Dickens’ writing.
nickname “Boz”. In 1836, he published The Posthumous
Papers of the Pickwick Club, which became widely popular 3. Which of the following statements is best taken to be
with magazine readers. Soon, his first novel, Oliver Twist, true in the context of the passage?
depicting the life of an orphan and inspired by Dickens’ own
(A) Dickens played the role of an instructor through
experience as an impoverished child, hit the stands. The
his writings.
overwhelming success of Oliver Twist turned Dickens into a
celebrity and over the next few years he had to struggle to (B) Dickens was the best British author of the
match the literary and commercial standard the book had nineteenth century.
set. (C) No other Dickens’ novel could ever match the
standard set by Oliver Twist.
In 1842, Dickens and his wife, Kate, went on a tour of the
United States, where people went crazy listening to him. (D) Dickens’ writing became sarcastic after the
Upon his return, Dickens wrote American Notes for General death of his daughter and father.
Circulation, a sarcastic travelogue criticising American (E) All of Dickens’ novels were based on the
culture and materialism. personal experiences of his childhood.

427

Book 1.indb 427 30/04/2019 4:49:04 PM


NMAT by GMAC™ Official Guide 2019

4. Which of the following statements, if true, would


most weaken the author’s argument?
(A) Dickens’ parents were responsible for his harsh
childhood.
(B) Dickens sympathised with his characters who
often survived a life of struggle.
(C) Dickens’ writing emphasised how simple people
got forced into real situations.
(D) Characterisation of David Copperfield and Oliver
Twist are suggestive of Dickens’ harsh life.
(E) Dickens wrote his classics during the period of
shock that he experienced after the deaths of
his daughter and father.

428

Book 1.indb 428 30/04/2019 4:49:04 PM


5.0   Language Skills Practice

Passage 13 (Real NMAT Question) 1. What is the primary purpose of this passage?

A team of scientists needs to understand the impact of a (A) Refute the importance of computer simulation.
nuclear explosion on a concrete bunker. Another team of (B) Discuss simulation, their uses and advantages.
engineers is trying to design an airplane that can withstand (C) Outline the developmental history of simulations.
high wind shear while flying. It is not always possible to (D) Debate the pros and cons of computer
carry out direct experimentation or trial to test many real-life simulation.
situations. Herein, lies the need for developing a modelling
system that can incorporate all variables and data to produce (E) Describe one particular application of the
a result that can help study the phenomenon at hand. This simulation.
modelling system is what we call computer simulations.
2. Based on the passage, which of the following best
They were first designed as a tool to study metrological
describes the author’s tone when writing about
phenomenon and nuclear physics. The list has steadily grown
computer simulations?
to incorporate many other areas of modern society like
medical sciences, construction, economics, astrophysics etc. (A) Wry humour
A computer simulation is a computer programme or a model (B) Bitter sarcasm
that tries to recreate a model of a particular system in a (C) Open optimism
virtual world. For example, if you want to see the impact (D) Veiled cynicism
of a comet striking Earth, you do not want to wait for a
(E) Forthright scepticism
real event, but you also do not want to be caught napping
before such an event actually occurs. So, what you do is 3. The author mentions all of the following EXCEPT:
create a virtual system which resembles a real-world system
with objects similar to Earth and the comet, along with (A) simulations have reduced the need for direct
parameters like speed, atmosphere, mass etc., as close experimentation.
to reality as possible. This may seem like some grandiose (B) simulations are able to account for changing
plan of science fiction, but even medical science benefits variables in a system.
by using simulations to see the impact of new medicines, (C) simulations have a limited capacity to evaluate
thereby helping to cut down risks associated with clinical hypothetical scenarios.
trials. Architects use the same modelling approach before
(D) studying the effects of a new drug being
designing a new bridge or a skyscraper.
developed can be aided by simulations.
There are different types of simulation models. First is the (E) the amount of traffic a new bridge in the city can
discrete model in which the changes to the system occur at handle can be gauged with simulations.
specific times. Second is the continuous model in which the
state of the system changes continuously over time. Third 4. Based on the given information, what can be inferred
is the mixed model in which both discrete and continuous about the author’s opinion of the use of computer
elements are contained. The type of data needed to build simulations in medicine?
a computer simulation includes: the overall process flow,
what is being produced or acted on (entities), frequencies at (A) The author is sceptical.
which the entities arrive in the programme, the length of time (B) The author is circumspect.
individual steps in the process take, probability distribution that (C) The author remains neutral.
characterise real-life uncertainties and variations in the process. (D) The author supports its use.
Computer simulations are also helpful in analysing “what if” (E) The author is against its use.
scenarios. The advantages of computer simulations include
gaining a greater understanding of the process; identifying
any potential obstructions or flaws; being able to effectively
evaluate processes which have changing variables and even
help mitigate possible problems. Computer simulations have
indeed provided an enormous impetus to how we do our
research and planning.

429

Book 1.indb 429 30/04/2019 4:49:04 PM


NMAT by GMAC™ Official Guide 2019

Passage 14 (Real NMAT Question) 1. What is the central idea of the passage?
Cancer, a word that elicits dread in nearly everyone, (A) evolution of cancer
continues to puzzle scientists. Despite attempts made by (B) role of genes in cancer
science to decipher this terrible disease, scientists still
struggle to find an answer to questions such as, ‘why does (C) early warning signs of cancer
cancer strike some and not others?’, ‘Are its ‘seeds’ part of (D) procedures for detecting cancer
our genetic makeup?’ (E) how an abnormal cell mass can cause cancer
Different terms are used when discussing cancer. Neoplasm
is an abnormal cell mass that develops when the controls 2. According to the passage, cancer is an ‘intimate
of the cell cycle and cell division malfunction. However, all enemy’ because:
neoplasms are not cancerous. Benign neoplasms do not (A) of the kind of treatment available for cancer.
spread, and are always seen to be local affairs. They are
(B) most of us know someone who has had cancer.
usually surrounded by a capsule and grow slowly, seldom
killing their hosts if they are removed before they affect (C) it is closely related to the body’s immune
vital organs. Malignant neoplasms, on the other hand, are system.
non-encapsulated, grow more relentlessly and can even kill. (D) it brings people affected by cancer closer
They resemble immature cells and invade their surroundings together.
rather than push them aside. Malignant cells are also (E) the ‘seeds’ of cancer appear to be in our own
capable of metastasis—i.e., they tend to spread via blood to genes.
distant parts of the body and form new masses.
But what causes the transformation? In other words, what 3. According to the author, what can be concluded from
converts a normal cell to a cancerous one? It is well known the passage?
that cancer-causing elements or carcinogens can be found (A) Occurrence of cancer depends on our genetic
in radiation, mechanical trauma, certain viral infections and make-up.
many chemicals (tobacco tars, saccharine). All of these have
(B) Abnormal masses in the body are the sole
one common factor—all of them cause mutations, which are
reason for cancer.
changes in DNA that alter the expressions of certain genes.
Usually carcinogens are eliminated by the immune system (C) Cancer has now become a very common
or certain enzymes. It must also be stated here that it takes disease and can be treated easily.
much more than just one mutation to change a normal cell (D) There are several factors that cause cancer and
into a full-fledged cancer cell. genes play a significant role.
It was with the discovery of oncogenes (cancer causing (E) Though cancer is a dreadful disease, there are
genes) followed by proto-oncogenes that the role of genes several promising new treatments.
started to gain prominence. Although proto-oncogenes code
for proteins that are needed for normal cell division and 4. According to the passage, the author most likely
growth, many of them have fragile areas that break when agrees with all of the following statements EXCEPT:
exposed to carcinogens, converting them to oncogenes. As (A) Surgical removal is a treatment for benign
a result, problems such as, ‘switching on’ of dormant genes neoplasm.
that allows cells to become invasive, arise. Oncogenes
have been discovered in only 15 to 20 percent of human (B) It takes only one genetic mutation to make a
cancers. Therefore, it came as no surprise when the tumour normal cell cancerous.
suppressor genes were discovered recently – these, as (C) Most carcinogens are eliminated by enzymes or
their names suggest, suppress or prevent cancer. The the immune system.
tumour suppressor genes not only put the ‘brakes’ on cell (D) Carcinogens cause changes in DNA that alter
division but they help with DNA repair and help to deactivate the expression of genes.
carcinogens, thereby enhancing the immune system’s ability
(E) Chemicals such as tobacco tar and saccharine
to destroy cancer cells. It is when the tumour suppressor
can act as carcinogens.
genes are damaged or changed in some way, that the
oncogenes are free to ‘do their thing’. Whatever the precise
genetic factor at work, the seeds of cancer do appear to be
in our own genes. Cancer is indeed intimate!

430

Book 1.indb 430 30/04/2019 4:49:04 PM


5.0   Language Skills Practice

Passage 15 (Real NMAT Question) 1. The passage is primarily concerned with:

According to Wimsatt and Beardsley, even though the (A) an evaluator’s paradigm for evaluation.
influence of the author’s ‘intention’ upon the critic’s opinion (B) the worth of the objective at the source.
has been disputed in a number of discussions, it is doubtful
whether most of its ‘romantic corollaries’ are widely (C) public knowledge regarding a literary text.
subject to questioning. The author duo finds the design or
(D) the significance of Wimsatt and Beardsley’s ideas.
intention of the author neither accessible nor desirable as a
yardstick for judging the accomplishment of a literary text. (E) the question regarding the revision of literary texts.
They define ‘intention’ as “design or plan in the author’s
mind. Intention has obvious affinities for the author’s 2. The author mentions all of the following EXCEPT:
attitude toward his work, the way he felt, what made him (A) the work of the author belongs to the public.
write.”
(B) language gives life to a piece of literary work.
Even though a text comes into existence only through (C) critics consign meaning to books and poems.
the medium of an author, it is erroneous to assign the
(D) no one has ever questioned the intention of the
author’s design to the status of a standard by which
poet.
critics are to evaluate the text. Besides, the question
arises as to how a critic is to find out the ‘intention’ behind (E) practical passages require an understanding of
a text. Wimsatt and Beardsley believe that if the poet was the author’s intention.
successful in bringing out his intention through the words
3. According to the passage, the author most likely
he had written, it must be evident in the poem itself. And
agrees with all of the following statements EXCEPT:
if the poet was not successful in doing so, the critic must
move outside the text to search for the intention. Unlike (A) the intention of the author is redundant.
practical messages, which are successful only if the (B) a text is never for the author to call his own.
readers correctly infer the intention of the author, poems
should just be, not mean. (C) the author’s intention must always be preserved.
(D) the author’s attitude towards his work is set in
Another argument that the duo puts forward is regarding stone.
the process of revision. Authors often revise their work,
thereby creating multiple versions of the same text. This (E) authors often revise their work creating multiple
raises the question of which intention is to be considered versions of the text.
by the critics. By extension, it raises the question of 4. What is meant by the statement that “The poem
whether the author’s “former concrete intention was not belongs to the public”?
his intention”.
(A) The author writes only for the public.
Wimsatt and Beardsley conclude that a text is neither the
critic’s nor the author’s. It is detached from the author at (B) Critics can never influence the public’s ideas.
birth and goes about the world beyond his power of intention (C) The public assigns meaning to a work of
or ability to control it. The poem belongs to the public. literature.
It comes to life through language, which is the peculiar (D) Literature can be possessed only by the public.
possession of the public and it is about the human being,
(E) Authors and critics are outside the public
which is an object of public knowledge.
domain.

431

Book 1.indb 431 30/04/2019 4:49:04 PM


NMAT by GMAC™ Official Guide 2019

Passage 16 (Real NMAT Question) 1. What is the central idea of the passage?

Advertising communicates the firm’s employment needs (A) That jobs are primarily advertised through
to the public through media such as newspapers, radio, newspapers and employment journals.
television and industry publications. The internet is the (B) The various aspects of the job that should be
newest and fastest growing external recruitment method. kept in mind both as the advertiser and as the
Regardless of the advertising method utilised in determining respondent to the advertisement.
the content of an advertising message, a firm must decide (C) That there is a difference between trade
on the corporate image it wants to project. Obviously, the journals and newspapers regarding employment
firm should give prospective employees an accurate picture opportunities.
of the job and the organisation. (D) That job advertisements occupy an important
The firm’s previous experience with various media should place in newspapers and employment journals of
suggest the most effective approach for specific types companies.
of jobs. A common form of advertising that provides (E) That using a variety of advertising platforms
broad coverage at a relatively low cost is the newspaper like newspapers and journals to communicate
advertisement. Such advertisements generate a vast employment opportunities can determine a
number of candidates most of whom aren’t qualified, and better outcome.
these inquiries are costly to process. This situation increases 2. Although newspaper ads are a reliable source of
the likelihood of poor selection decisions. At the same job recruitments, there are risks involved. This is
time, the firm should attempt to appeal to the self-interest because:
of prospective employees, emphasising the job’s unique
qualities. The ad must tell potential employees why they (A) these days, there are other sources for finding
should be interested in that particular job and organisation. job vacancies as well.
The message should also indicate how an applicant is to (B) the self-interest of the employees is not catered
respond—apply in person, apply by telephone or submit a to in a proper manner.
resume by fax or e-mail.
(C) at times, the image of the company that is
Although few individuals base their decisions to change jobs recruiting new candidates is not projected well.
on advertising, an ad creates awareness, generates interest (D) though the ads are low cost, the coverage
and encourages a prospect to seek more information is broader so the chances of making a good
about the firm and the job opportunities that it provides. selection are rare.
Examination of the Sunday edition of any major newspaper (E) there are enhanced chances of poor selection as
reveals the extensive use of newspaper advertising in the vast majority that apply are unsuitable for the
recruiting. vacant seat.
Advertising in Professional and Trade Journals: Certain 3. What is the tone of the author in the passage?
media attract an audience that is more homogeneous in
(A) Critical
terms of employment skills, education and orientation.
Advertisements placed in such publications as the Wall (B) Laudatory
Street Journal relate primarily to managerial, professional (C) Analytical
and technical positions. The readers of these publications (D) Argumentative
are generally individuals qualified for many of the positions
(E) Cynical
advertised. Focusing on a specific labour market minimises
the likelihood of receiving marginally qualified or even 4. According to the passage, the author most likely
totally unqualified applicants. agrees with all of the following statements EXCEPT:

Virtually every professional group publishes a journal that (A) the use of journals for recruitments has its own
is widely read by its members. Advertising for a marketing limitations.
executive position in Marketing Forum, for example, would (B) media offers a wide variety of recruitment
hit the target market because marketing professionals are services these days.
virtually the exclusive readers. Trade journals are also widely (C) newspaper ads are no longer in demand for job
utilised. However, using journals does present problems. recruitments.
For example, they lack scheduling flexibility; their publishing
(D) newspaper ads often generate a large number of
deadlines may be weeks prior to the issue. Because firms
unsuitable candidates.
cannot always anticipate staffing far in advance, journals
have obvious limitations for recruitment. (E) advertising is rarely the basis for an employee’s
decision to change jobs.
432

Book 1.indb 432 30/04/2019 4:49:04 PM


5.0   Language Skills Practice

Passage 17 (Real NMAT Question) detrimental economic effects on the host countries
concerned.
Bribery is deep-rooted in the cultures of many nations. It distorts
market mechanisms and creates unfair competitive advantages 1. What is the central idea of the passage?
for certain firms, and hence, leads to an inefficient allocation of (A) Bribery as a conventional practice in developed
national resources. Although bribery is officially frowned upon countries.
throughout the world (including those countries in which it is
extensively practised), it is not illegal in a number of countries. (B) Considering acceptance of bribes as a wrong
Inducements paid to public servants are subject to particular practice.
criticism, as in this case it is the innocent taxpayer (rather (C) Acceptance of bribes across different cultures.
than private shareholders) who ultimately foots the bill. Across (D) Bribes as an accepted norm in the business
various nations, acceptance of bribes by government officials world for government officials.
results from the historical tradition of ‘prebendalism’ i.e., the
(E) Merits and demerits of deep-rooted bribery and
charging of fees by holders of public positions who receive only
how the adoption of a moderate approach is
a nominal stipend. Governments overlook petty corruption on
debatable.
the part of junior officials because otherwise they would have to
increase the latter’s wages to levels sufficient to induce them to 2. At times, even the concerned authorities tend to
remain in their jobs. Nations with poorly developed tax systems overlook minor cases of corruption. This is because:
do not have the wherewithal to finance an extensive civil service
(A) bribes help keep the officials engaged in the job.
and thus, implicitly rely on public officials supplementing their
meagre salaries by taking unofficial gratuities. (B) bribes are now accepted as normal in the
business world.
Large-scale bribery in advanced industrial countries results, (C) bribes are sometimes the sole means of earning
perhaps, not from pernicious intent on the part of the a good income.
bribers, but rather from the fact that so many Western
(D) bribes are accepted as a personal fee equivalent
markets are dominated by a small handful of firms in
to a tip given to a waiter or a chauffer.
uncompetitive environments, i.e., it is lack of genuine
competition within a sector that is the root cause of (E) bribes act as a substitute for an incentive for
corruption. Bribery in this situation is little more than a junior officials who would otherwise find the jobs
natural business response to a particular situation. In many low-paying.
of the less developed nations, bribery is accepted as a 3. From the passage, it can be inferred that:
normal part of business life. Bribes are not considered to
be ‘corrupt’ payments, but rather fees for commissioning (A) bribery is more widely accepted in developed
a service—just like any other service fee or personal nations.
commission. Equally, low-value payments might be seen as (B) bribery is vital for the survival of many big and
the equivalent of the ‘tips’ conventionally given to waiters, small firms.
waitresses or taxi drivers in developed countries. (C) there is little difference between a ‘bribe’ and a
‘gift’.
The case for adopting a liberal approach towards bribery
can be based on the following arguments: (D) accepting bribes has become a part and parcel
of the world today.
(a) Distinguishing between a ‘bribe’ and a ‘gift’ is (E) inducements being paid to the people are
problematic. Note how the giving of small gifts is very drawing greater criticism worldwide.
common in Western countries. But what constitutes
a ‘small’ gift and where should the line be drawn 4. The author agrees with all the following statements
between low- and high-value items? An expensive gift EXCEPT:
to a managing director with a million pound annual (A) Bribery is just a normal business response.
salary might be considered reasonable, yet the same
(B) Bribery is frowned upon only in developed
gift to a lower-rank employee could be deemed as
countries.
corruption!
(C) Concluding deals without bribery is becoming
(b) There is, in reality, no reliable means for ensuring difficult these days.
that all gifts are reported, so why bother attempting (D) Not being engaged in bribery can lead many
to control the practice? firms to economic losses.
(c) Firms not engaging in bribery in some countries (E) Bribery in industrial countries results when some
stand to lose money, resulting in job losses and other firms function in an uncompetitive environment.

433

Book 1.indb 433 30/04/2019 4:49:05 PM


NMAT by GMAC™ Official Guide 2019

Passage 18 (Real NMAT Question) externalised (that is, passed along to the consumer as higher
prices), they would not be much of a deterrent. Moreover,
Almost every regulatory policy is created to achieve some it would take a large bureaucracy to monitor carefully the
desirable social goal. When more than 10,000 people are level of pollution discharged, and it would require a complex
killed annually in industrial accidents, who would disagree calculation to determine the level of tax necessary to
with the goal of a safer workplace? Who would dissent encourage businesses not to pollute.
from greater highway safety, when more than 50,000
perish each year in automobile accidents? Who would 1. What is the central idea of the passage?
disagree with policies to promote equality in hiring, when (A) Regulatory activities of government exist at
the history of opportunities for women and minorities is different levels.
one of discrimination? Who would disagree with policies to
reduce industrial pollution, when pollution threatens health (B) The pros and cons of using the incentive system
and lives? However, there may be more than one way to were proposed by Schultze.
achieve these—and many other—desirable social goals. (C) The old policies framed for workplaces could not
be successful for several reasons.
Charles L. Schultze, chair of former President Carter’s (D) An overview of the regulatory policies for
Council of Economic Advisors, is a critic of the current achieving social goals and how the incentive
state of federal regulation. Schultze reviewed the regulatory system can be helpful.
activities of the Environmental Protection Agency (EPA) and
the Occupational Safety and Health Administration (OSHA). (E) How different regulatory policies that encourage
Neither agency’s policies, he concluded, had worked very specific social behaviours in different set-ups
well. He described the existing system as command and have failed miserably.
control policy. The government tells business how to reach 2. In Charles Schultze’s argument, the claim is that:
certain goals, checks that these commands are followed, and
punishes offenders. (A) the use of the incentive policy is limited to just a
few situations.
Schultze advocates an incentive system. He argues that (B) the EPA and OSHA should not tell businesses how
instead of telling construction businesses how their ladders to regulate themselves.
must be constructed, measuring the ladders, and charging
(C) regulatory policies should be created to achieve
a small fine for violators, it would be more efficient and
certain desirable social goals.
effective to levy a high tax on firms with excessive worker
injuries. Instead of trying to develop standards for 62,000 (D) the government should minimise pollution and
pollution sources, as the EPA now does, it would be easier workplace accidents before they become too
and more effective to levy a high tax on those who cause severe.
pollution. The government could even provide incentives (E) an incentive system would be more effective
in the form of rewards for such socially valuable behaviour and efficient than the command-and-control
as developing technology to reduce pollution. Incentives, regulation.
Schultze argues, use market-like strategies to regulate
3. According to the author, what can be concluded from
industry. They are, he claims, more effective and efficient
the passage?
than command-and-control regulation.
(A) Goals leading to safer workplaces were not achieved.
Not everyone is as keen on the use of incentives as
(B) If incentives are provided, social goals can be
Schultze. Defenders of the command-and-control system
achieved.
of regulation compare the present system to preventive
medicine—it is designed to minimise pollution or workplace (C) Socially desirable behaviours need to be
accidents before they become too severe. Defenders of the achieved to fulfill social goals.
system argue, too, that penalties for excessive pollution or (D) There are several ways in which socially desirable
excessive workplace accidents would be imposed only after goals can be achieved.
substantial damage had been done. They also add that if (E) Schultze’s incentive system has received
taxes on pollution or unsafe work environments were merely acceptance as well as criticism from people.

434

Book 1.indb 434 30/04/2019 4:49:05 PM


5.0   Language Skills Practice

4. According to the passage, the author most likely


agrees with all of the following statements EXCEPT:
(A) Incentives, in the form of rewards, could be
given for socially valuable behaviours.
(B) An incentive system would reward businesses
that develop technology to reduce pollution.
(C) An incentive system would involve developing
standards for thousands of pollution sources.
(D) Regulatory policies have been created to
address workplace and highway safety,
discrimination and pollution.
(E) The command-and-control system punishes
businesses that do not reach certain goals set
by the government.

435

Book 1.indb 435 30/04/2019 4:49:05 PM


NMAT by GMAC™ Official Guide 2019

Passage 19 (Real NMAT Question) Your self-concept is not a fixed construct. It was not
implanted at birth to remain constant for the rest of your
Your self-esteem is closely related to your self-concept. life. Things change. Individuals with low self-esteem may be
Through your self-concept, you describe who you are. locking on to events and experiences that happened years
Through your self-esteem or self-worth, you evaluate who ago and tenaciously refusing to let go of them.
you are. People derive their sense of self-worth from
comparing themselves to others. A belief that you cannot Some of your self-image problems may be so ingrained that
cook like a chef may not, in itself, lower your self-esteem. you need professional help. A trained therapist can help
But if there are several things you can’t do well, or many you sort through them. Because you have spent your whole
important tasks that you cannot seem to master, these lifetime developing your self-esteem, it is not easy to make
shortcomings may begin to colour your overall sense of big changes. But as you have seen, talking through problems
worth. can make a difference.

Psychologist Eric Berne developed the concept of a life 1. What is the central idea of the passage?
position to describe people’s overall sense of their own (A) Comparing and contrasting intrapersonal and
worth and that of others. He identified four positions: (1) ‘I’m interpersonal communication.
OK, you’re OK,’ or positive regard for self and others; (2) ‘I’m
OK, you’re not OK,’ or positive regard for self and low regard (B) Discussing the differences between self-esteem
for others; (3) ‘I’m not OK, you’re OK,’ or low self-regard and and self-concept.
positive regard for others and (4) ‘I’m not OK, you’re not OK,’ (C) Discussing the stages that people go through
or low regard for both self and others. Your life position is a when they are facing change.
driving force in your relationships with others. People in the (D) Encouraging the use of therapy and counselling
‘I’m OK, you’re OK,’ position have the best chance of healthy to deal with stress.
relationships because they have discovered their own
(E) Explaining the concept of self-esteem and
talents and recognise that others have talents different
describing ways to improve self-image.
from their own.
2. According to the passage, visualisation can prove
Intrapersonal communication is communication within helpful in enhancing self-worth. This is clear from the
yourself—self-talk. Realistic positive self-talk can have fact that:
a reassuring effect on your level of self-worth and your
interactions with others. Conversely, repeating negative (A) a person gets a chance to enhance self-worth
messages about your lack of skill and ability can keep you when he compares himself to others.
from trying and achieving. (B) an individual’s self-concept can change with time
as he visualises things.
Visualisation takes the notion of self-talk one step further.
(C) it helps in reframing events that can change
Besides, just telling yourself that you can achieve your goal,
one’s perspective towards life.
you can actually try to ‘see’ yourself conversing effectively
with others, performing well on a project or exhibiting (D) it takes self-talk one step ahead and helps in
some other desirable behaviour. Research suggests that an visualising things positively which yields positive
apprehensive public speaker can manage fear not only by results.
developing skill in public speaking, but also by visualising (E) looking to future events and experiences can
positive results when speaking to an audience. give a boost to your self-concept.

Reframing is the process of redefining events and 3. According to the author, what can be concluded from
experiences from a different point of view. Just as reframing the passage?
a work of art can give the picture a whole new look, (A) Communication, when taking the shape of
reframing events can change your perspective. visualisation, reframing and self-talk, can foster a
positive self-image.
Having at least one other person who can help you honestly
reflect on your virtues and vices can be extremely beneficial (B) People can improve the overall sense of their
in fostering a healthy, positive self-image. The more credible own worth and that of others by several ways.
the source of information, the more likely you are to believe (C) Self-worth and self-concept are the most
it. Having a trusted friend, colleague or counsellor who can significant assets of human life.
listen without judging you and give you the straight scoop (D) Self-concept can be changed at any stage of life.
about yourself can help you avoid ‘pity parties’.
(E) There are a variety of ways in which healthy self
image can be fostered.

436

Book 1.indb 436 30/04/2019 4:49:05 PM


5.0   Language Skills Practice

4. According to the passage, the author agrees with all


of the following statements, EXCEPT:
(A) developing honest relationships is important to
creating a healthy self-image.
(B) speech communication empowers each of us.
(C) people can also help an individual reflect upon
his virtues and vices.
(D) people and circumstances change.
(E) your self-concept is formed before you are born
and remains the same.

437

Book 1.indb 437 30/04/2019 4:49:05 PM


NMAT by GMAC™ Official Guide 2019

9 Parajumbles
Choose the most logical order of sentences from among the given choices to construct a coherent
paragraph.

1. (a) Only then can any virus make use of its single reading books, writing emails, knitting, taking a
talent, which is to take control of a host’s cellular catnap, and any number of other diversions.
machinery and use it to churn out thousands of
copies of itself (b)  P
 eople who are unable to drive themselves could
experience a new kind of independence.
(b)  In this way, one infected cell soon becomes
billions. (c)  M
 oreover, self-driving cars could re-contextualise
land use on a massive scale.
(c)  These viruses then move from one cell to the
next, transforming each new host into a factory (d)  In this imagined mobility utopia, drone trucks
that makes even more virus. would haul packages across the country and no
human would have to circle a city block in search
(d)  A
 virus is nothing more than a few strands of a parking spot.
of genetic material wrapped in a package of
(A) adbc
protein—a parasite, unable to function on its
own. (B) acbd
(C) cbad
(e)  In order to survive, it must find a cell to infect.
(D) abcd
(A) dceab (E) bacd
(B) bcead
4. (a) The reward system was about pleasure and
(C) deacb somehow learning what yields it, and little more.
(D) decab
(b)  If you had opened a textbook on brain rewards
(E) bdcea
in the late 1980s, it would have told you that the
2. (a) If this is so, then public spaces function like a dopamine and opioids that swished and flickered
stage in the same way that our own homes and around the reward pathway were the blissful
living rooms do. brain chemicals responsible for pleasure.
(b)  S
 everal scholars over the years have drawn (c)  It wasn’t until the early 1990s, after rigorous
analogies between life and theatre. research, that he felt bold enough to go public
(c)  For instance, the eminent sociologist Erving with his new thesis.
Goffman suggested that life is a series of
(d)  S
 o when Berridge, a dedicated young scientist
performances, in which we are all continually
who was more David than Goliath, stumbled upon
managing the impression we give other people.
evidence in 1986 that dopamine did not produce
(d)  A
 rchitecture, landscaping, the dimensions of the pleasure, but in fact desire, he kept quiet.
stage, and the other actors around us all offer
(A) acbd
cues about how we should perform and how we
should treat one another. (B) badc
(A) acdb (C) bdac
(B) badc (D) cbad
(C) cdeb (E) cdba
(D) edcb 5. (a) Those who do remember him tend to use him as
(E) bcad a cautionary tale.

3. (a) Self-driving cars promise to create a new kind of (b)  H


 is charismatic personality, combined with his
leisure, offering passengers additional time for controversial scientific accomplishments, landed

438

Book 1.indb 438 30/04/2019 4:49:05 PM


5.0   Language Skills Practice

him on TV more than once, notably on The Steve (C)


cbda
Allen Show (D)
dcba
(c)  Instead of dazzling audiences with complicated (E)
dcab
science, McConnell captivated them with awe-
8. (a) Point-of-sale advantages include stores offering
inspiring concepts
no shipping charges and free ship-to-store
(d)  M
 cConnell’s experiments have largely faded from charges.
scientific memory (b) The internet can be a great resource for
shoppers looking to expand their choices of
(e)  B
 ut at the end of the ‘50s, McConnell was a big products to buy and an invaluable way for saving
deal money.
(A) cdbae (c) Price-comparison websites make deal hunting
(B) bcdae easier and also help guide shoppers to online
stores with the best reputations by posting
(C) daebc
reviews submitted by other shoppers.
(D) edcba
(d) Online stores are highly competitive not only

(E) aedbc
 with other online stores, but also with brick-and-mortar
6. (a) The band stretches from the Tropic of Cancer competitors.
to the Tropic of Capricorn, though there are (A)
dbac
occasional reefs at higher latitudes—near
(B)
bdca
Bermuda, for instance.
(C)
bdac
(b)  R
 eefs can be hundreds of feet tall and thousands (D)
acbd
of acres in area.
(E)
dbca
(c)  T
 he world’s largest reef, or really reef system, is
9. (a) Nearly 24 hours after the snafu, Oscars host
the Great Barrier Reef, along the east coast of
Jimmy Kimmel broke his silence on the shocking
Australia.
moment with a few words on his show, Jimmy
(d)  C
 oral reefs are found in a band that circles the Kimmel Live.
globe like a cummerbund. (b) “I don’t know if you know this but I hosted the
Oscars last night,” Kimmel quipped, opening his
(e)  U
 nlike the Great Wall of China, the Great Barrier ABC show.
Reef, which extends more than fourteen hundred (c) La La Land was incorrectly named the Best
miles, is actually visible from the space. Picture when Moonlight was really the winner.
(A) acdeb (d) In case you’ve been living under a rock, the
(B) dacbe Oscars were rocked by an epic mistake on
(C) aedbc Sunday night.

(D) dbcea (A)


abdc
(E) cdaeb (B)
bdca

7. (a)  L
 ogging operations, which provide the world’s (C)
bdac
wood and paper products, also cut countless (D)
abcd
trees each year. (E)
dcab
(b) Often, small farmers will clear a few acres 10. (a) Thus, they are more at risk for adult health
by cutting down trees and burning them problems such as heart disease, type 2
in a process known as slash and burn diabetes, stroke, several types of cancer, and
agriculture. osteoarthritis.
(c) The biggest driver of deforestation is agriculture. (b) According to an article in The New York Times
(d) Farmers cut forests to provide more room for all of these health effects are contributing to a
planting crops or grazing livestock. shorter lifespan of five years for these obese
children.
(A)
cdab
(B)
cdba

439

Book 1.indb 439 30/04/2019 4:49:05 PM


NMAT by GMAC™ Official Guide 2019

(c) One study showed that children who became times contradictory criticism of the bill from their
obese as early as age 2 were more likely to be own caucus, with zero chance of support from
obese as adults. Democrats.
(d) Children who are obese are likely to be obese as (b) Senate Republican leaders breathed a collective
adults. sigh of relief on Wednesday as all 52 GOP
senators voted to begin debate on the tax
(A)
cdba
reform bill.
(B)
cbda
(c) For Republicans, time is of the essence.
(C)
dcab
(d) If they do not pass the tax reform bill this
(D)
dcba week, it will be sidelined by next week’s
(E)
dbac legislative struggle to secure funding to avoid a
government shutdown.
11. (a) Not a bad legacy for a man who, after his first
studio went belly up, placed his career in the (A)
abdc
hands of a cartoon mouse. (B)
bdca
(b) While he was at it, he built fantastical (C)
bdac
amusement parks, developed a brand (D)
bacd
recognised by children and adults all over the
(E)
dcab
world, and created an educational foundation for
future entertainment innovators. 14. (a) Too many Catholics, Americans in particular, still
(c) The Walt Disney Company, which reported more see the situation of Chinese Catholics through
than $38 billion in revenue in 2010, was started the lenses of the Cold War.
by a high-school dropout who loved to draw and (b) News of the Holy See’s possible rapprochement
had a passion for learning. with China’s Communist government on the
(d) Walt Disney founded an empire on fantasy and appointment of bishops has aroused charges
risk taking. of a betrayal of the “underground church” and
fears of the abandonment of Catholics who for
(A)
cdab
decades have suffered for the sake of their
(B)
cdba fidelity to Rome.
(C)
cbda (c) Most foreigners are ignorant of the changes
(D)
dcba that have affected Chinese Catholics in recent
(e) dcab decades.
(d) Western journalists have always been too easily
12. (a) The Land of Morning Calm is rapidly becoming swayed by misleading accounts circulated by
one of Asia’s most popular destinations. those opposed to an entente between Rome and
(b) Since then an Olympics and a World Cup have Beijing.
kick-started a vibrant modern economy.
(A)
cbda
(c) South Korea’s rise has been nothing short of
(B)
cdba
meteoric after breaking from military dictatorship
in the 80s. (C)
bdac
(d) Almost every year South Korea or Seoul features (D)
bdca
on top ten lists of the best places to visit, and (E)
adcb
with good reason.
15. (a) Great players end their careers with anywhere
(A)
cbda from 25 to 50 such scores.
(B)
cdba (b) Every sport has record breakers, but of his
(C)
bdac contemporaries, only Michael Jordan, Lance
(D)
bdca Armstrong and prescandal Tiger Woods come
anywhere close to matching him in redefining the
(E)
adcb
realms of possibility.
13. (a) But even as the vote pushed the bill past its (c) In cricket, a batsman who hits a century, or a
latest test, it set up a critical few hours in which “ton”--another term for a 100-plus-run innings--
Republicans will have to address serious and at

440

Book 1.indb 440 30/04/2019 4:49:05 PM


5.0   Language Skills Practice

displays the most consistent measure of batting (c) Why has Holmes continued to captivate
prowess. generation after generation when other fictional
(d) Tendulkar’s ton of tons is beyond great. detectives of the Victorian period are forgotten?
(d) But first it will be useful to summarise the life of
(A)
dbac
Holmes’s creator.
(B)
cadb
(A)
cdba
(C)
cbda
(B)
cbda
(D)
bdca
(C)
acdb
(E)
dbca
(D)
acbd
16. (a) In recent years, with the master crippled by (E)
dbac
rheumatoid arthritis, Swinston or another
veteran often led students and company 19. (a) Furthermore, kids are also a viable market
dancers through the steps. because they can easily influence lifestyle, and
(b) A little before 6 p.m., some 40 current and provide a view of overall trends in society; how it
former dancers, in warm-up clothes or street is now and what it is likely to become.
garb, drifted onto the wooden floor. (b) The advertising industry views teenagers in
(c) As a pianist picked out strains of Bach, Robert society as a viable market segment, because
Swinston, a longtime Cunningham dancer who of their immature understanding of the media
was also the choreographer’s assistant, held an and its dazzling impact on teen and young
abbreviated class in Cunningham technique. undeveloped brains.
(d) On July 27, the day after the choreographer (c) According to marketing research companies,
Merce Cunningham died, there was an open teenagers are important to marketers because
house at the West Village studio in which his they can spend a substantial open income,
dance company has operated since 1971. spend family money easily, and they are easily
able to influence their families to spend on large
(A)
dbac
and small household purchases that appeal to
(B)
bdca them.
(C)
bdac (d) The media is increasingly focusing on kids and
(D)
acbd adolescents to captivate with advertisements.
(E)
dbca (A)
abdc
17. (a) The fork is held with the left hand and the knife is (B)
acbd
held with the right. (C)
bdac
(b) When no knife is being used, the fork can be (D)
dbca
held with the tines up. (E)
dcab
(c) The fork is held generally with the tines down
using the knife to cut food or help guide food on 20. (a) In reality, researchers have concluded that pain
to the fork. is used as a means to a different end, that
(d) With the tines up, the fork balances on the side end being the destruction of the individual as a
of the index finger, held in place with the thumb person.
and index finger. (b) The second is that the purpose of the pain is to
elicit information.
(A)
abdc
(c) The first is that the primary purpose is to inflict
(B)
acbd
pain.
(C)
bdac
(d) There are two common misconceptions about
(D)
bdca torture.
(E)
dcab (e) Any information elicited is usually no more than a
side benefit; often the victim has no information
18. (a) Doyle was born in Edinburgh in 1859, one of
to give. (Real NMAT Question)
nine children of an alcoholic Irish artist who was
consigned, in later life, to a lunatic asylum. (A) abdce
(b) One can break the answer down into a mix of (B) cbdae
elements. (C) dcbae
441

Book 1.indb 441 30/04/2019 4:49:05 PM


NMAT by GMAC™ Official Guide 2019

(D) abcde lowering their own reproductive success for the


(E) cdbae benefit of another individual.
(c) Although they may not always play out on such
21. (a) Wine is an alcoholic beverage, typically made of an epic scale, examples of selfless behaviour
fermented grape juice. abound in nature.
(b) Wine is produced by fermenting crushed grapes (d) Humans help other humans to do everything
using various types of yeast. from obtaining food to finding mates to
(c) Yeast consumes the sugars found in the grapes defending territory. (Real NMAT Question)
and converts them into alcohol.
(A) cadb
(d) Besides grapes, other fruits such as apples and
(B) abcd
berries can also be fermented.
(C) acbd
(e) And the resultant wines are normally named after
the fruit from which they are produced and are (D) cbad
called fruit wine or country wine. (Real NMAT (E) dbca
Question)
24. (a) While a luxury one usually starts with fish—
(A) abdce pickled herring, smoked eel or hot fried plaice.
(B) edbca (b) Danish food includes a variety of open
(C) abced sandwiches.
(D) bedca (c) The sandwiches are traditionally served for the
(E) abcde mid-day meal.
(d) An ordinary mid-day meal consists of just a
22. (a) Within the span of a single year, close to 40 simple sandwich prepared during breakfast and
journalists either disappeared or were found packed in a lunch box.
killed in Mexico. (e) The cuisine of Denmark stems from the country’s
(b) In the face of this threat to lives, many print and agricultural past, as well as its geography and
television media houses opted to completely stop climate. (Real NMAT Question)
reporting on the visible and ongoing drug cartels. (A) edbca
(c) The general public, in its thirst for news and (B) abcde
information, now turns to the relative safety of
the internet for updates on the drug scene there. (C) ebcda

(d) Freedom of speech and expression has always (D) decab


been a bit of a risky business for those in the (E) bedac
journalistic field. 25. (a) Most of these places are high up on
(e) Reasons for their disappearance may be traced mountains.
to open-minded debates about the impropriety (b) This hardened ice is what comprises the
of drug use, aired on national media networks. glacier.
(Real NMAT Question)
(c) Repeated snowfalls compress the lower layers
(A) abecd of snow.
(B) adebc (d) Glaciers form in extremely cold places.
(C) ceabd (e) Finally, the lowest layer hardens and turns into
(D) daebc ice. (Real NMAT Question)
(E) dbaec (A) abedc
23. (a) An organism’s cells coordinate to keep their (B) dceba
division in check and avoid causing cancer, (C) cadeb
worker ants sacrifice their own fecundity to (D) cbdae
serve their queen and colony, female lions within
a pride suckle one another’s young. (E) daceb

(b) Even if the helpers may not necessarily be 26. (a) The hedgehog population in the UK has
putting their lives on the line, they are risking drastically fallen from 30 million in the 1950s to
about 1.5 million in 2011, and Britons may not

442

Book 1.indb 442 30/04/2019 4:49:05 PM


5.0   Language Skills Practice

see any more red squirrels in 20 years’ time, (A) cdbae


according to a report. (B) adcbe
(b) In general, according to the report, the situation (C) badce
has improved for species restricted to habitats (D) abcde
that could benefit from site-based conservation
(E) edacb
and has worsened for many widespread species
such as hedgehogs and red squirrels. 29. (a) Hydraulic fracturing occurs when the effective
(c) However, the future of otters, bats and water voles stress is reduced sufficiently by an increase in
is not as bleak as their populations have increased, the pressure of fluids within the rock, such
thanks to improving conservation efforts. that the minimum principal stress becomes
(d) The report ‘State of Britain’s Mammals 2011’ by tensile and exceeds the tensile strength of the
Oxford University’s wildlife conservation unit also material.
shows that the dwindling population of the common (b) This is particularly so in the case of “tensile”
dormouse and mountain hare is threatened. fractures, which require the walls of the fracture
(e) A ban on chemicals used in sheep dip in the to move apart, working against this confining
late 1990s, for instance, has resulted in cleaner pressure.
rivers in Britain, which in turn, has benefitted (c) Fracturing in rocks at depth tends to be
otters. (Real NMAT Question) suppressed by the confining pressure due to
the load caused by the overlying rock strata.
(A) adbce
(d) Fractures formed in this way will be oriented in
(B) adceb
a plane perpendicular to the minimum principal
(C) abced stress, and for this reason induced hydraulic
(D) abdce fractures in wellbores are sometimes used to
(E) dabce determine the orientation of stresses.
(Real NMAT Question)
27. 
(a) In fact, experts advise a comprehensive
recruitment policy so as to avoid bad hires. (A) cbad
(b) The recruitment process of any company (B) cabd
begins when the need for human resources is (C) cadb
determined. (D) abcd
(c) They then end up hiring rather than recruiting. (E) acdb
(d) This in turn leads to bad recruitment.
30. (a) Getting involved in a hobby is a better way to
(Real NMAT Question) beat stress as it provides relaxation, sharpens
(A) abcd the mind, increases creativity, and also helps in
making friends and acquiring valuable skills.
(B) acbd
(b) Regrettably, we now regularly spend our free
(C) adbc
time watching TV or surfing the internet.
(D) badc
(c) Ironically, modern life has become more
(E) bcda stressful when there is more of this free time,
and yet the best we do is lie down on the
28. (a) The colony was then named Port Cornwallis after
couch.
Admiral William Cornwallis.
(d) Just a few years back, free time or leisure time
(b) This penal colony is now known as Port Blair
was used as an outlet for doing something
(after the officer who founded it).
different, pursuing one’s passion by taking up
(c) However, disease and death in the penal colony hobbies. (Real NMAT Question)
forced the government to cease its operations in
May 1796. (A) acdb
(d) After two years, this colony moved to the (B) dbca
northeast part of Great Andaman. (C) bcda
(e) In 1789, the government of Bengal established a (D) cdba
penal colony on Chatham Island in the southeast (E) badc
bay of Great Andaman. (Real NMAT Question)

443

Book 1.indb 443 30/04/2019 4:49:06 PM


NMAT by GMAC™ Official Guide 2019

31. (a) In other words, our desire to stand out leads us (A) dabce
to do silly things and we, not satisfied with being (B) dabec
unique, try to be very unique.
(C) ecdab
(b) We sometimes go to ridiculous extents to (D) ecdba
appear unique.
(E) edabc
(c) This, of course, is a defect in character which,
in adults, stems out of some sort of insecurity 34. (a) However, at least for now, it seems that the
that desperately needs to be compensated for. nations whose people eat shark fins have the
(d) It leads to such contexts that could perhaps upper hand.
permit the use of expressions like ‘very unique’. (b) In fact, in China, due to it being so expensive,
(Real NMAT Question) serving the shark-fin soup is seen as a mark of
(A) badc respect.
(B) bdac (c) By some estimates, some 73 million sharks are
killed each year by finning fishermen leading
(C) bdca
to widespread demand from wildlife experts to
(D) abcd protect sharks.
(E) bcda (d) No wonder then, that despite shark meat itself
not being so popular, sharks are hunted for
32. (a) When an entire lump or suspicious area is
their fins, leading to the infamous ‘shark-finning
removed, it is called an excisional biopsy.
at sea’, where sharks are de-finned and thrown
(b) It involves the removal of cells or tissues to back into the sea.
examine and to determine the presence or (e) Shark fins, reportedly fetching up to $10,000 a
extent of a disease. fin, are used to create the delectable shark fin
(c) The removed tissue is examined under a soup in countries such as Hong Kong, China,
microscope by a pathologist, or is analysed Japan, Taiwan and Singapore. (Real NMAT
chemically. Question)
(d) Biopsy is a commonly used medical test these (A) abedc
days. (B) bceda
(e) But when only a sample of tissue is removed (C) dceab
with preservation, it is incisional or core biopsy.
(D) ecbad
(Real NMAT Question)
(E) ebdca
(A) debca
35. (a) In our day-to-day social and professional
(B) bedac
conversations, we tend to overuse, trivialise and
(C) dbcae devalue the word ‘great’ to the point of parody.
(D) abdec (b) It seems everything, or almost everything, has
(E) abcde an in-built potential for greatness.
(c) An ice-cream can be great, a square-cut can
33. (a) Chiropractic has caused many disabilities and
be great, a barber can be great, a hotel can be
even death in some cases.
great, and a magician can be great.
(b) Yet few chiropractors disclose to their patients
(d) So, as we commence our detailed examination
the risks of cervical manipulation.
of what constitutes greatness, we need to move
(c) This is not necessarily true. from the ridiculous to the sublime. (Real NMAT
(d) Significant risks accompany some alternative Question)
therapies.
(A) abcd
(e) Most people think a treatment is safe if it is
alternative. (Real NMAT Question) (B) bdca
(C) adcb
(D) bacd
(E) cadb

444

Book 1.indb 444 30/04/2019 4:49:06 PM


5.0   Language Skills Practice

5.2 Answer Key


The following discussion on answers and explanations is intended to familiarise you with the most
efficient and effective approaches to these kinds of questions. Remember that it is the problem solving
strategy that is important, not the specific details of a particular question.

1 Antonyms (A) Indomitable means impossible to subdue or


defeat. This is not the required antonym.
1. Antonym means a word opposite in meaning to (B) Correct. Agitated means feeling or appearing
another. Contentious is the given word whose troubled or nervous. This is an antonym of ‘stoic’.
antonym (or opposite) one has to denote.
(C) Sycophant means a person who acts obsequiously
Contentious means controversial or causing or likely
towards someone important in order to gain
to cause an argument.
advantage. This is completely unrelated.
(A) Arguable means able to be argued or asserted. It
(D) Serene means calm, peaceful, and untroubled.
is similar in meaning to ‘contentious’.
This can be negated.
(B) Correct. Agreeable means willing to agree to
(E) Impassive means not feeling or showing emotion.
something. It is an antonym of ‘contentious’.
This is also not the opposite of ‘stoic’ and thus
(C) Similar means having a resemblance in can be negated.
appearance, character, or quantity, without being
identical. This is unrelated and can be negated. The correct answer is B.

(D) Controversial means the same as ‘contentious’. 4. Antonym means a word opposite in meaning to
Hence, it is a synonym. another. Stoic is the given word whose antonym (or
opposite) one has to denote. Stoic means a person
(E) Truculent means eager or quick to argue or fight. who can endure pain or hardship without showing
This can be negated. his/her feelings or complaining.
The correct answer is B. (A) Indomitable means impossible to subdue or
defeat. This is not the required antonym.
2. Antonym means a word opposite in meaning to
another. Pejorative is the given word whose antonym (B) Correct. Agitated means feeling or appearing
(or opposite) one has to denote. Pejorative means troubled or nervous. This is an antonym of ‘stoic’.
expressing contempt or disapproval. (C) Sycophant means a person who acts
(A) Derogatory means showing a critical or obsequiously towards someone important in
disrespectful attitude. This is similar in meaning order to gain advantage. This is completely
and thus not an antonym. unrelated.

(B) Nugatory means of no value or importance. This (D) Serene means calm, peaceful, and untroubled.
is unrelated to ‘pejorative’. This can be negated.

(C) Correct. Complimentary means expressing a (E) Impassive means not feeling or showing
compliment; praising or approving. This is an emotion. This is also not the opposite of ‘stoic’
antonym of ‘pejorative’. and thus can be negated.

(D) Simple means easily understood or done. This is The correct answer is E.
unrelated and thus can be negated.
5. Antonym means a word opposite in meaning to
(E) Oratorical means relating to the art or practice of another. Insular is the given word whose antonym (or
public speaking. This can also be negated. opposite) one has to denote. Insular means narrowly
The correct answer is C. restricted in outlook or scope.

3. Antonym means a word opposite in meaning to (A) Correct. Unbiased means showing no prejudice
another. Stoic is the given word whose antonym (or for or against something; impartial.
opposite) one has to denote. Stoic means a person (B) Circumscribed means restricted (something)
who can endure pain or hardship without showing within limits. This can be cancelled out as it is
his/her feelings or complaining. not an antonym.

445

Book 1.indb 445 30/04/2019 4:49:06 PM


NMAT by GMAC™ Official Guide 2019

(C) Isolated means far away from other places, (C) Miscreant is a person who has done something
buildings, or people; remote. This can also be wrong or unlawful.
rejected. (D) A ‘hypochondriac’ is a person who is abnormally
(D) Dispassionate means not influenced by strong anxious about his health.
emotion, and so able to be rational and (E) An iconoclast is a person who attacks or
impartial. This is also not the correct answer. criticises cherished beliefs or institutions.
(E) Insouciant means showing a casual lack of
concern. This is also not an antonym of ‘insular’. The correct answer is B.

The correct answer is A. 9. Antonym means a word opposite in meaning to


another. Alacrity is the given word whose antonym
6. Antonym means a word opposite in meaning to (or opposite) one has to denote. Alacrity means ‘brisk
another. Subterfuge is the given word whose and cheerful readiness'.
antonym (or opposite) one has to denote. Subterfuge
means deceit used in order to achieve one’s goal. (A) Diffidence means shyness.
(B) Correct. Apathy means lack of interest,
(A) Servitude means the state of being a slave or
enthusiasm, or concern.
completely subject to someone more powerful.
(C) Empathy means compassion.
(B) Guile means sly or cunning intelligence.
(D) Finesse means impressive delicacy and skill.
(C) Deceit means the action or practice of deceiving
someone by concealing or misrepresenting the (E) Intrepidity means courage.
truth.
The correct answer is B.
(D) Castigation means to reprimand someone
severely. 10. Antonym means a word opposite in meaning to
(E) Correct. Guilelessness means innocence or another. Placate is the given word whose antonym (or
naiveté and is the antonym of 'subterfuge'. opposite) one has to denote. Placate means to make
(someone) less angry or hostile.
The correct answer is E.
(A) Appease is a synonym for ‘placate’.
7. Antonym means a word opposite in meaning to (B) Quell means to subdue, to put an end to (a
another. Vilify is the given word whose antonym (or rebellion or other disorder), typically by the use
opposite) one has to denote. Vilify means to speak or of force.
write about in an abusively disparaging manner. (C) Correct. Exasperate means to irritate intensely
(A) Disparage means disrespect, regard or or infuriate.
represent as being of little worth. (D) Renounce means to formally declare one’s
(B) Renege means to go back on a promise, abandonment of (a claim, right, or possession).
undertaking, or contract. (E) Reproach means to scold or to disgrace.
(C) Repudiate means to reject.
The correct answer is C.
(D) Correct. Exalt means to think or speak very
highly of (someone or something), to show 11. Antonym means a word opposite in meaning to
respect. another. Innocuous is the given word whose antonym
(E) Affront means to offend the modesty or values (or opposite) one has to denote. Innocuous means
of a person. harmless.

The correct answer is D. (A) Insolent means showing a rude and arrogant
lack of respect.
8. Antonym means a word opposite in meaning to
(B) Irreverent means showing a lack of respect
another. Tyro is the given word whose antonym (or
for people or things that are generally taken
opposite) one has to denote. Tyro means a beginner
seriously.
or a novice.
(C) Officious means assertive of authority in a
(A) Renegade means a person who deserts and
domineering way, especially with regard to trivial
betrays an organisation, country, or set of
matters.
principles.
(D) Prodigious means remarkably or impressively
(B) Correct. Virtuoso means a person highly skilled
great in extent, size, or degree.
in music or another artistic pursuit. It is the
correct answer.
446

Book 1.indb 446 30/04/2019 4:49:06 PM


5.0   Language Skills Practice

(E) Correct. Detrimental means tending to cause 15. Antonym means a word opposite in meaning to
harm. another. Daunt is the given word whose antonym (or
opposite) one has to denote. Daunt means to cause
The correct answer is E. to lose courage.
12. Antonym means a word opposite in meaning to (A) ‘Exemplify’ means to clarify by giving an
another. Prosaic is the given word whose antonym illustration.
(or opposite) one has to denote. Prosaic means dull (B) ‘Protract’ means to lengthen in time.
and unimaginative.
(C) ‘Parley’ means to hold a conference between
(A) Palpable means (of a feeling or atmosphere) so opposing sides in a dispute.
intense as to seem almost tangible. (D) ‘Disconcert’ means to unsettle or disturb one’s
(B) Regressive means returning to a former or less composure.
developed state. (E) Correct. ‘Hearten’ means to make more cheerful
or confident.
(C) Uncanny means strange or mysterious.
The correct answer is E.
(D) Correct. Imaginative means having or showing
creativity or inventiveness. 16. Antonym means a word opposite in meaning to
(E) Humdrum means lacking excitement or variety. It another. Phlegmatic is the given word whose
is a synonym for prosaic. antonym (or opposite) one has to denote. Phlegmatic
means showing little emotion or passion.
The correct answer is D.
(A) ‘Salubrious’ means promoting health.
13. Antonym means a word opposite in meaning to (B) ‘Impassive’ is a synonym for phlegmatic.
another. Counterfeit is the given word whose
(C) ‘Ignominious’ means bringing disgrace or shame.
antonym (or opposite) one has to denote. Counterfeit
means a fraudulent imitation of something else. (D) Correct. ‘Compassionate’ means feeling or
showing sympathy and concern for others.
(A) Covert means not openly acknowledged or
(E) ‘Clandestine’ means kept secret or done
displayed or hidden.
secretively, especially because it is illicit.
(B) Correct. Authentic means genuine.
The correct answer is D.
(C) Reprehensible means deserving censure or
condemnation. 17. Antonym means a word opposite in meaning to
another. Succinct is the given word whose antonym
(D) Laconic means (of a person, speech, or style of (or opposite) one has to denote. Succinct means
writing) using very few words. (especially of something written or spoken) briefly
(E) Spurious means false or fake. It is a synonym for and clearly expressed.
counterfeit. (A) ‘Laconic’ means (of a person, speech, or style of
writing) using very few words. It is a synonym for
The correct answer is B. ‘succinct’.
14. Antonym means a word opposite in meaning to (B) ‘Churlish’ means rude.
another. Tentative is the given word whose antonym (C) Correct. ‘Garrulous’ means tending to talk a
(or opposite) one has to denote. Tentative means not great deal.
certain or fixed.
(D) ‘Piquant’ means having a pleasantly sharp taste
(A) Compendious means containing or presenting or appetizing flavor.
the essential facts of something in a (E) ‘Whimsical’ means unusual and strange in a way
comprehensive but concise way. that might be funny, annoying or illogical.
(B) Sinuous means having many curves and turns.
The correct answer is C.
(C) Correct. ‘Certain’ means definite or sure.
(D) Tenuous means weak or slight. 18. Antonym means a word opposite in meaning to
(E) Tenacious means stubborn or resolute. another. Efface is the given word whose antonym (or
opposite) one has to denote. Efface means erase (a
The correct answer is C. mark from a surface).

447

Book 1.indb 447 30/04/2019 4:49:06 PM


NMAT by GMAC™ Official Guide 2019

(A) ‘Obliterate’ means to wipe out. It is related to Parsimonious means a miser or someone who
efface. doesn’t like to spend money.
(B) ‘Propitiate’ means to win or regain the favour (A) ‘Greedy’ means someone who has a very strong
of (a god, spirit, or person) by doing something wish to continuously get more of something,
that pleases them. This is unrelated. especially food or money.
(C) Correct. ‘Create’ means to bring into existence. (B) ‘Parisian’ means relating to Paris. This is
(D) ‘Jeer’ means to make rude and mocking completely unrelated.
remarks, typically in a loud voice. This can be (C) ‘Penurious’ means extremely poor, unwilling to
negated. spend money. This is similar in meaning and thus
(E) ‘Exult’ means to show or feel triumphant elation can be ruled out.
or jubilation. (D) ‘Tight-fisted’ means not willing to spend or give
The correct answer is C. much money. This is a synonym.
(E) Correct. ‘Philanthropic’ means (of a person or
19. Antonym means a word opposite in meaning to organisation) seeking to promote the welfare of
another. Denouement is the given word whose others; generous and benevolent.
antonym (or opposite) one has to denote.
Denouement means the outcome of a situation, when The correct answer is E.
something is decided or made clear. 22. Antonym means a word opposite in meaning to
(A) ‘Proposition’ means a suggested scheme or plan another. Irascible is the given word whose antonym
of action, especially in a business context. (or opposite) one has to denote. Irascible means
having or showing a tendency to be easily angered.
(B) ‘Conspiracy’ means a secret plan by a group to
do something unlawful or harmful. (A) ‘Outgoing’ means friendly and socially confident.
(C) Correct. ‘Preamble’ means an introductory fact This is unrelated and can be negated.
or circumstance; especially one indicating what (B) Correct. ‘Agreeable’ means pleasant and
is to follow. amiable. This is an antonym and the correct
(D) ‘Intrigue’ means to arouse the curiosity or answer.
interest of. (C) Friendly’ means being of a jovial, pleasant
(E) ‘Climax’ means the most intense, exciting, or nature.’
important point of something. (D) ‘Benign’ means being gentle and kind.
The correct answer is C. (E) ‘Patient’ means being tolerant and stoic.
The correct answer is B.
20. Antonym means a word opposite in meaning to
another. Betoken is the given word whose antonym 23. Antonym means a word opposite in meaning to
(or opposite) one has to denote. Betoken means be a another. Sibylline is the given word whose antonym
sign of; be evidence of. (or opposite) one has to denote. Sibylline means
(A) Correct. ‘Belie’ means (of an appearance) fail to prophetic and oracular.
give a true impression of (something). (A) Correct. ‘Unprophetic’ means failing to foresee
(B) ‘Auspicate’ means to begin or inaugurate with a or predict correctly. This is an antonym and the
ceremony intended to bring good fortune. This correct answer.
can be negated. (B) ‘Perceptive’ means having or showing sensitive
(C) ‘Foretoken’ means be a sign of (a future event). insight. This can be negated.
This is a synonym and thus can be rejected. (C) ‘Eloquent’ means fluent or persuasive in
(D) ‘Postpone’ means to put off or delay. speaking or writing. This is unrelated and is thus
(E) ‘Postpose’ means to place (a modifying word or rejected.
morpheme) after the word that it modifies. This (D) ‘Palpable’ means able to be touched or felt. This
is completely unrelated. is also not the right answer.
The correct answer is A. (E) ‘Overt’ means done or shown openly; plainly
apparent. This is an incorrect option.
21. Antonym means a word opposite in meaning
to another. Parsimonious is the given word The correct answer is A.
whose antonym (or opposite) one has to denote.

448

Book 1.indb 448 30/04/2019 4:49:06 PM


5.0   Language Skills Practice

24. Antonym means a word opposite in meaning to means to make someone unable to feel or think
another. Demean is the given word whose antonym properly or to make someone become numb.
(or opposite) one has to denote. Demean means to
(A) Dumbfound means to amaze, that is, the same
cause someone to become less respected.
as stupefy. Therefore, it is a synonym and not an
(A) ‘Incite’ means to encourage or stir up (violent or antonym.
unlawful behaviour). (B) Flabbergast means to amaze, that is, the same
(B) ‘Praise’ means to express approval of or as stupefy. Therefore, it is a synonym and not an
admiration for. This is an antonym and the antonym.
correct answer. (C) Correct. To sensitise is to make someone or
(C) ‘Humble’ means having or showing a modest or something sensitive or to make them become
low estimate of one’s importance. receptive to stimuli.
(D) ‘Disgrace’ means bring shame or discredit on. (D) To suffocate is to deprive somebody of oxygen.
(E) ‘Suppress’ means to restrain, forcibly put an end (E) To arouse is to awaken or evoke a feeling,
to something. which is much greater in the degree of intensity
conveyed, in comparison to ‘sensitise’.
The correct answer is B.
The correct answer is C.
25. Antonym means a word opposite in meaning to
another. Violation is the given word whose antonym 28. Antonym means a word opposite in meaning to
(or opposite) one has to denote. To complete the another. Imperturbable is the given word whose
process one has to know the exact meaning of antonym (or opposite) one has to denote. To
violation, which refers to break or disregard rules. complete the process one has to know the exact
meaning of imperturbable, which means calm.
(A) Offence refers to an illegal act.
(B) Rejection means dismissal or refusal of a (A) Militant means favoring confrontational or violent
proposal, idea, etc. methods in support of a political or social cause.
(C) Correct. Obedience means to be submissive to (B) Cynical means believing that people are
restraint or rules. motivated purely by self-interest; distrustful of
human sincerity or integrity.
(D) Persuasion is the act of convincing by argument.
(C) Comical is amusing.
(E) Devastation means great destruction or
damage. (D) Correct. Agitated means feeling or appearing
troubled or nervous.
The correct answer is C.
(E) Angry means feeling or showing strong
annoyance.
26. Antonym means a word opposite in meaning to
another. Profess is the given word whose antonym The correct answer is D.
(or opposite) one has to denote. To complete the
process one has to know the exact meaning of 29. Antonym means a word opposite in meaning to
profess, which means to declare or admit openly. another. Clamp is the given word whose antonym
(or opposite) one has to denote. To complete the
(A) To affirm is to confirm.
process one has to know the exact meaning of
(B) To divulge is to make public. clamp, which means to strengthen or hold things
(C) To confirm is to establish the truth or together.
correctness of something. (A) To clinch is to confirm or settle.
(D) Correct. Conceal is to hide or prevent (B) To secure is to fix or attach something firmly so
disclosure. that it cannot be moved or lost.
(E) To asseverate is to positively confirm. (C) Correct. To release is to set free from
The correct answer is D. confinement.
(D) To entwine is to interweave.
27. Antonym means a word opposite in meaning to (E) To encompass is to surround.
another. Stupefy is the given word whose antonym (or
opposite) one has to denote. To complete the process The correct answer is C.
one has to know the exact meaning of stupefy, which

449

Book 1.indb 449 30/04/2019 4:49:06 PM


NMAT by GMAC™ Official Guide 2019

30. Antonym means a word opposite in meaning to


another. Voluptuary is the given word whose antonym
(or opposite) one has to denote. To complete the
process one has to know the exact meaning of
voluptuary, which refers to a person devoted to
luxury and sensual pleasure.
(A) Austereness means harshness or severity.
(B) Spartan refers to something that lacks comfort
or luxury and is not the correct opposite.
(C) Restrained means dispassionate.
(D) Disciplined means showing a controlled form of
behaviour or way of working.
(E) Correct. Abstemious is one who is only
moderately interested in things that give you
pleasure.

The correct answer is E.

450

Book 1.indb 450 30/04/2019 4:49:06 PM


5.0   Language Skills Practice

2 Analogies

(C) A guide is in command of a tourist . Incorrect
(D) A doctor is in command of a hospital. This may
or may not be true Incorrect
1. This is an analogy based question. The idea here is
to find the option which draws a similar relationship (E) A hotel is in command of a concierge. Incorrect
to that of the given pair. Start by making a bridge The correct answer is B.
between the words given in the question stem—The
function of police is to control crime. 4. This is an analogy based question. The idea here is
to find the option which draws a similar relationship
Now, plug the bridge into the answer choices:
to that of the given pair. Start by making a bridge
(A) The function of a theft is to control watchman. between the words given in the question stem—
Incorrect (be careful as the bridge is not this— Loathe is a high degree of dislike
the job of a watchman is to control theft).
Now, plug the bridge into the answer choices:
(B) The function of eat is control food.
Incorrect. (A) Discomfort is a high degree of pain. Incorrect
(C) The function of exercise is to control weight. (B) Foolhardy is a high a degree of coward . Incorrect
Not necessarily. People may exercise for (C) Anxious is a high degree of disquiet . Disquiet is
other reasons as well, such as to improve their a feeling of anxiety but it does not relate to the
general fitness even if they are not overweight. given pair. Incorrect
(D) The function of a dam is to control floods. (D) Stress is a high degree of fear. Incorrect
Correct. (E) Ecstasy is a high degree of joy. They are
(E) The function of a football is to control play. synonyms and relate to the given pair. Correct
Incorrect.
The correct answer is E.
The correct answer is D.
5. This is an analogy based question. The idea here is
2. This is an analogy based question. The idea here is to find the option which draws a similar relationship
to find the option which draws a similar relationship to that of the given pair. In case of this question, an
to that of the given pair. Start by making a bridge altruistic person lacks selfishness.
between the words given in the question stem— A
cleaver is used by a butcher to cut meat. Now, plug the bridge into the answer choices:
(A) An enlightened person lacks wisdom. Incorrect
Now, plug the bridge into the answer choices:
(B) A befuddled person lacks clarity. Correct
(A) A screwdriver is used by a mechanic to screw
and unscrew screws and not cut . Incorrect (C) A flippant person lacks calm. This may or may
not be true. Incorrect
(B) A treadmill is used by a runner to run and not
cut. Incorrect (D) An assiduous person lacks diligence. Incorrect

(C) A pen is used by a writer to write and not cut. (E) A depressed person lacks sorrow. This does not
Incorrect relate to the given pair. Incorrect

(D) A scalpel is used by a surgeon to cut through The correct answer is B.


skin and flesh. Correct
6. This is an analogy based question. The idea here
(E) A brush is used by a painter to paint and not cut. is to find the option which does not draw a similar
Incorrect relationship to that of the given pair. Start by making
The correct answer is D. a bridge between the words given in the question
stem — Metal is a type of music. It developed in the
3. This is an analogy based question. The idea here is late 1960s and early 1970s.
to find the option which draws a similar relationship
to that of the given pair. Start by making a bridge Now, plug the bridge into the answer choices:
between the words given in the question stem— A (A) Expressionism is a type of painting. Incorrect.
captain is a person in command of a ship. (B) Risotto is a type of dish. Incorrect.
Now, plug the bridge into the answer choices: (C) Clarinet is a type of musical instrument. Incorrect.
(A) A teacher is in command of a school. This may (D) Bolero is a type of dance not dress. Correct.
be true but not necessarily. Incorrect (E) Cha cha is a type of dance. Incorrect.
(B) A manager is in command of an office. Correct The correct answer is D.

451

Book 1.indb 451 30/04/2019 4:49:06 PM


NMAT by GMAC™ Official Guide 2019

7. This is an analogy based question. The idea here Now, plug the bridge into the answer choices:
is to find the option which does not draw a similar (A) ‘Entomology’ is the study of insects. Incorrect.
relationship to that of the given pair. Start by making
(B) ‘Seismology’ is the study of earthquakes.
a bridge between the words given in the question
Incorrect.
stem— The relationship between the original pair is
that although the words in the pair mean the same (C) ‘Petrology’ is the study of rocks. Incorrect.
but their connotation is different. ‘Scrawny’ has a (D) ‘Anthropology’ is the study of mankind. Incorrect.
negative connotation and means (of a person or (E) ‘Ornithology’ is the study of birds and not fish.
animal) unattractively thin and bony. Slim refers to a Correct.
person who is slender, gracefully thin.
The correct answer is E.
Now, plug the bridge into the answer choices:
(A) ‘Nitpicking’ means giving too much attention to 10. This is an analogy based question. The idea here
details that are not important, especially as a is to find the option which does not draw a similar
way of criticising (meticulous in a negative way, relationship to that of the given pair. Start by making
very precise). Incorrect. a bridge between the words given in the question
stem— The original set of words share a ‘part to
(B) ‘Shocking’ has a negative usage and ‘surprising’
whole relationship’. ‘Canto is a part of a poem.
has a positive usage. Incorrect.
(C) ‘Economical’ means careful not to waste money Now, plug the bridge into the answer choices:
or resources and ‘miserly’ is stingy. Incorrect. (A) An ‘island’ is a part of an ‘archipelago’. Incorrect.
(D) ‘Inquisitive’ is ‘being curious’ and ‘nosey’ means (B) A ‘piston’ is a part of an engine. Incorrect.
showing too much curiosity about other people’s (C) A ‘canopy’ is an overhead roof or else a
affairs. Incorrect. structure over which a fabric or metal covering
(E) ‘Ludicrous’ and ‘absurd’ are synonyms and mean is attached which is able to provide shade
‘foolish, unreasonable, or out of place as to be or shelter from weather conditions such as
amusing’. Correct. sun, hail, snow and rain. It is not a part of rain
making. Correct.
The correct answer is E.
(D) 'Mast' is a part of a boat. It refers to a large
8. This is an analogy based question. The idea here wooden spar used to hold up other spars and
is to find the option which does not draw a similar the rigging. Incorrect.
relationship to that of the given pair. Start by making (E) 'Staircase' is an integral part of a building.
a bridge between the words given in the question Incorrect.
stem— The relationship shared between the original
pair is that the first is worn on the second. A ‘tiara’ is The correct answer is C.
worn on the hair.
11. This is an analogy based question. The idea here
Now, plug the bridge into the answer choices: is to find the option which does not draw a similar
(A) A ‘muffler’ is worn around the neck. Incorrect. relationship to that of the given pair. Start by making
a bridge between the words given in the question
(B) An ‘anklet’ is worn around the ankle. Incorrect.
stem- The relationship shared by the original pair is
(C) A ‘cravat’ is worn around the neck. Incorrect. that of ‘degree of intensity’. ‘Tepid’ means slightly
(D) A ‘girdle’ is worn around the waist and not the warm which is a mild degree of ‘boiling.’
wrist. Correct.
Now, plug the bridge into the answer choices:
(E) A ‘sash’ is worn around the waist or over one’s
(A) ‘Punctilious’ means showing great attention to
shoulder. Incorrect.
detail or correct behaviour. Incorrect.
The correct answer is D. (B) A ‘downpour’ is a heavy rainfall. Incorrect.
9. This is an analogy based question. The idea here (C) ‘Dote’ means to be extremely and uncritically
is to find the option which does not draw a similar fond of somebody. Incorrect.
relationship to that of the given pair. Start by (D) Both ‘avaricious’ and ‘rapacious’ mean to be
making a bridge between the words given in the extremely greedy. Correct.
question stem— The relationship between the (E) ‘Vainglorious’ means to be excessively vain or
original pair is that the first concerns itself with the proud. Incorrect.
study of the second. Thus, ‘paleontology’ is the
study of fossils. The correct answer is D.

452

Book 1.indb 452 30/04/2019 4:49:07 PM


5.0   Language Skills Practice

12. This is an analogy based question. The idea here is 15. This is an analogy based question. The idea here
to find the option which does not draw a similar is to find the option which does not draw a similar
relationship to that of the given pair. Start by making relationship to that of the given pair. Start by making
a bridge between the words given in the question a bridge between the words given in the question
stem- ‘Denigrate’ and ‘belittle’ are synonyms that stem- ‘Fit as a fiddle’ is a simile. A simile is a figure of
mean ‘to criticise unfairly’. The relationship shared is speech that directly compares two things.
that of synonyms.
Now, plug the bridge into the answer choices:
Now, plug the bridge into the answer choices: (A) ‘Fresh as a daisy’ is a simile. Incorrect.
(A) ‘Dearth’ and ‘scarcity’ are synonyms meaning of (B) ‘Slippery as an eel’ is a simile. Incorrect.
‘a lack of something’. Incorrect.
(C) ‘Straight as an arrow’ is a simile. Incorrect.
(B) ‘Ephemeral’ and 'transitory' are synonyms
meaning ‘lasting for a very short time’. Incorrect. (D) ‘Mad as a hatter’ is the correct simile. Correct.
(C) ‘Ineptitude’ means ‘lacking skill or ability’ making (E) ‘Quiet as a church mouse’ is a simile. Incorrect.
it an antonym for ‘competence’. Correct. The correct answer is D.
(D) ‘Indolent’ and ‘lazy’ are synonyms. Incorrect.
(E) ‘Facile’ and ‘superficial’ are synonyms meaning 16. This is an analogy based question. The idea here
of ‘ignoring the true complexities of an issue’. is to find the option which does not draw a similar
Incorrect. relationship to that of the given pair. Start by making
a bridge between the words given in the question
The correct answer is C. stem- ‘Numerophobia’ is the fear of numbers.
13. This is an analogy based question. The idea here Now, plug the bridge into the answer choices:
is to find the option which does not draw a similar (A) ‘Photophobia’ is the fear of light. Incorrect
relationship to that of the given pair. Start by making
(B) ‘Ailurophobia’ is the fear of cats and not dogs.
a bridge between the words given in the question
Correct.
stem- ‘Mozzarella’ is a type of cheese. Thus, the
relationship between the original pair is that the first (C) ‘Ophidiophobia’ is the fear of snakes. Incorrect
is a type of the second. (D) ‘Agoraphobia’ is the fear of open spaces.
Incorrect
Now, plug the bridge into the answer choices:
(E) ‘Acrophobia’ is the fear of heights. Incorrect
(A) ‘Spaghetti’ is a type of pasta. Incorrect.
(B) ‘Moccasin’ is a type of shoe not dress. Correct. The correct answer is B.
(C) ‘Macchiato’ is a type of coffee. Incorrect.
17. This is an analogy based question. The idea here
(D) ‘Viper’ is a type of snake. Incorrect. is to find the option which does not draw a similar
(E) ‘Chrysanthemum’ is a type of flower. Incorrect. relationship to that of the given pair. Start by making
The correct answer is B. a bridge between the words given in the question
stem- A ‘bristle’ is a part of a brush. Thus, the
14. This is an analogy based question. The idea here relationship between the original pair is that the
is to find the option which does not draw a similar second is a part of the first.
relationship to that of the given pair. Start by making Now, plug the bridge into the answer choices:
a bridge between the words given in the question
stem- A ‘quiver’ is a group of cobras. The relationship (A) A ‘stamen’ is a part of a flower. Incorrect.
shared between the original pair is that the first (B) A ‘pericarp’ is a part of a fruit. Incorrect.
denotes a group of the second. (C) A ‘fuselage’ is a part of an aircraft. Incorrect.
Now, plug the bridge into the answer choices: (D) An ‘awning’ is not a part of a billboard. It is a
(A) A a group of fish is called a ‘school’. Incorrect. secondary covering attached to the exterior wall
of a building. Correct.
(B) A group of pigeons is called a ‘flock’ or a flight’.
Congregation refers to a collection of people. (E) ‘Woodwinds’ is a part of a symphony
Correct. orchestra. The typical symphony orchestra
consists of four groups of related musical
(C) A group of antelopes is called a ‘herd’. Incorrect.
instruments called the woodwinds, brass,
(D) An group of caterpillars is called ‘army’. Incorrect. percussion, and strings (violin, viola, cello and
(E) A ‘float’ refers to a group of crocodiles. Incorrect. double bass). Incorrect.
The correct answer is B. The correct answer is D.

453

Book 1.indb 453 30/04/2019 4:49:07 PM


NMAT by GMAC™ Official Guide 2019

18. This is an analogy based question. The idea here 20. This is an analogy based question. The idea here is to
is to find the option which does not draw a similar find the option which draws a similar relationship to that
relationship to that of the given pair. Start by making of the given pair. Start by making a bridge between the
a bridge between the words given in the question words given in the question stem— ‘Hedonism’ refers
stem— ‘Address’ and ‘location’ are synonyms that to the pursuit of pleasure. The relationship shared is
refer to the ‘particulars of a place where someone that of a philosophy and its central belief.
lives or an organisation is situated’. The relationship
Now, plug the bridge into the answer choices:
shared is that of synonyms.
(A) Asceticism is the philosophy of self-denial. It
Now, plug the bridge into the answer choices: is characterised by abstinence from sensual
(A) The words ‘annul’ and ‘abrogate’ are synonyms pleasures and pursuit of spiritual goals.
that mean– ‘to declare an agreement, decision Incorrect.
or result as invalid’. Incorrect. (B) Altruism is the quality of unselfish concern for
(B) The words ‘bark’ and ‘snap’ are synonyms that the welfare of others. Incorrect.
mean– ‘to utter a command or question abruptly (C) Sexism refers to the discriminatory, stereotypical
or angrily’. Incorrect. and prejudiced attitude adopted towards people-
(C) The words ‘mean’ and ‘generous’ are antonyms. usually women-based on their sex. Incorrect.
‘Mean’ refers to cruel or spiteful behaviour while (D) Eidolism refers to the belief in ghosts. Correct.
‘generous’ refers to kind and magnanimous (E) Nihilism refers to the philosophy that advocates
conduct. Correct. the rejection of all religious and moral principles
(D) The words ‘die’ and ‘expire’ are synonyms that and states that life is meaningless. Incorrect
mean– ‘to perish, stop living’. Incorrect.
The correct answer is D.
(E) The words ‘current’ and ‘present’ are also
synonyms. They refer to contemporary, ongoing 21. This is an analogy based question. The idea here is
activities or events. Incorrect to find the option which draws a similar relationship
The correct answer is C. to that of the given pair. Start by making a bridge
between the words given in the question stem- A
19. This is an analogy based question. The idea here is ‘basil’ is a type of ‘herb’. Thus, the relationship
to find the option which draws a similar relationship between the original pair of words is that the first is
to that of the given pair. Start by making a an example of the second.
bridge between the words given in the question
Now, plug the bridge into the answer choices:
stem- The relationship shared between the original
pair is that the second is a protective outer (A) A ‘Ragdoll’ is a breed of cat. Correct.
case of the first. Now, plug the bridge into the (B) A ‘frigate’ is a warship and not an aircraft.
answer choices: Incorrect.
(A) The same relationship is demonstrated in (C) An ‘Alaskan Husky’ is a dog breed not related to
A where a ‘rind’ is the tough, outer skin of a fox. Incorrect.
‘lemon’. This makes it the correct answer. (D) Marmots and squirrels belong to the same family
Correct. (Sciuridae). Incorrect.
(B) A ‘marshmallow’ is a ‘confection’ or a ‘sweet (E) ‘Equine’ refers to a horse or other members of
dish’. Incorrect. the horse family. Incorrect.
(C) An ‘orange’ is a ‘citrus fruit’ which refers to a The correct answer is A.
group of plants that produces acidic fruits with a
lot of juice. Incorrect. 22. This is an analogy based question. The idea here is
(D) ‘Rosa’ is the scientific name for a ‘rose’. to find the option which draws a similar relationship
Incorrect. to that of the given pair. Start by making a bridge
(E) A ‘chocolate’ is ‘edible’; it means that it is fit to between the words given in the question stem-
be eaten. Incorrect. ‘Disheveled’ means (of a person’s hair, clothes, or
appearance) untidy; disordered. The relationship
The correct answer is A. between the original pair of words is that of antonyms.

454

Book 1.indb 454 30/04/2019 4:49:07 PM


5.0   Language Skills Practice

Now, plug the bridge into the answer choices: (D) Something ‘cold’ does not cause a scald.
(A) ‘Disdain’ means the feeling that someone or Incorrect.
something is unworthy of one’s consideration (E) ‘Sickness’ and affliction are synonyms. Incorrect.
or respect. ‘Contempt’ is a synonym of disdain.
The correct answer is A.
Incorrect.
(B) ‘Discrepancy’ means an illogical or surprising 25. This is an analogy based question. The idea here is
lack of compatibility or similarity between two to find the option which draws a similar relationship
or more facts. ‘Affront’ means insult. They are to that of the given pair. Start by making a bridge
unrelated. Incorrect. between the words given in the question stem—
(C) ‘Kindle’ and ‘ignite’ are synonyms and mean to ‘Mariticide’ means ‘killing one’s husband’.
set (something) on fire. Incorrect. Now, plug the bridge into the answer choices:
(D) ‘Dispel ‘and ‘dissipate’ are also synonyms and
(A) ‘Uxoricide’ means ‘killing one’s wife’. Correct.
mean (with reference to a feeling or emotion)
disappear or cause to disappear. Incorrect. (B) ‘Ambicide’ means ‘killing one‘s friend’. Incorrect.
(E) ‘Exasperate’ means to make (someone) (C) ‘Sororicide’ means ‘killing one‘s sister’. Incorrect.
extremely angry and impatient. ‘Please’ means (D) ‘Fratricide’ means ‘killing one’s brother’. Incorrect.
to make someone happy or satisfied. They are (E) ‘Filicide’ refers to killing one‘s son or daughter.
antonyms and thus the correct answer. Correct. Another word that is used for it is ‘Prolicide’.
The correct answer is E. Incorrect.
The correct answer is A.
23. This is an analogy based question. The idea here is
to find the option which draws a similar relationship 26. This is an analogy based question. The idea here is
to that of the given pair. Start by making a bridge to find the option which draws a similar relationship
between the words given in the question stem- The to that of the given pair. Start by making a bridge
relationship shared by the original pair is that of between the words given in the question stem—
‘degree of intensity’. ‘Trickle’ means a small flow of ‘Endorse’ is synonymous with ‘support’.
liquid which is a mild degree of ‘gush’.
Now, plug the bridge into the answer choices:
Now, plug the bridge into the answer choices:
(A) Cavil is synonymous with nitpick, which means
(A) ‘Plunder’ and 'pillage' are synonyms. Incorrect ‘make petty or unnecessary objections’. Correct.
(B) ‘Quash’ and 'revoke' are also synonyms. (B) Abrogate is not synonymous with renew.
Incorrect Incorrect. Abrogate means void which does not
(C) A ‘breeze’ is a gentle wind which is a mild degree mean the same as ‘renew’.
of ‘gale’. Correct. (C) Desiccate is not synonymous with destroy.
(D) An ‘acorn’ is a pale oval nut that is the fruit of an Incorrect. Desiccate means dried up which is not
oak tree. Incorrect similar to destroy.
(E) ‘Ebb’ is the movement of the tide out to sea. (D) Fulminate is not synonymous with fumigate.
Incorrect Incorrect. Fulminate means strong protest
whereas fumigate means to disinfect.
The correct answer is C.
(E) Hypothecate is not synonymous with guess.
24. This is an analogy based question. The idea here is Hypothecate means to pledge (money) by law
to find the option which draws a similar relationship for a specific purpose. Incorrect.
to that of the given pair. Start by making a bridge
The correct answer is A.
between the words given in the question stem-
‘Trepidation’ causes trembling. The relationship 27. This is an analogy based question. The idea here is
shared by the original pair is that of cause and effect. to find the option which draws a similar relationship
Now, plug the bridge into the answer choices: to that of the given pair. Start by making a bridge
(A) An ‘earthquake’ causes tsunami. Correct. between the words given in the question stem—
Pre-empt is to take action in order to prevent
(B) ‘Snowflakes’ do not cause a deluge. Incorrect.
something and anticipation is the expectation or
(C) ‘Acquittal’ does not cause incarceration. prediction of something occurring.
Incorrect.

455

Book 1.indb 455 30/04/2019 4:49:07 PM


NMAT by GMAC™ Official Guide 2019

Now, plug the bridge into the answer choices: (B) Drink is made of alcohol. Incorrect
(C) Hot is made of beverage. Incorrect
(A) A plan is made to prevent a debriefing. Incorrect.
(D) Fruit is made of grapes. Incorrect
(B) A recruit is hired to prevent a position. Incorrect.
(E) Wine is made of fruit. Correct
(C) A study is done to prevent an opportunity.
Incorrect. The correct answer is E.
(D) Inoculation is done to prevent vaccination.
Incorrect. 30. This is an analogy based question. The idea here is
to find the option which draws a similar relationship
(E) Correct. Quarantine is taking action in order to to that of the given pair. Start by making a bridge
prevent something (disease) and prognosis is between the words given in the question stem—Radii
a medical term that means predicting the likely is the plural form of radius.
outcome of something (effects of a disease).
Now, plug the bridge into the answer choices:
The correct answer is E.
(A) Horse is the plural form of mare. Incorrect
28. This is an analogy based question. The idea here is (B) Bees is the plural form of swarm. Incorrect
to find the option which draws a similar relationship (C) Square is the plural form of side. Incorrect
to that of the given pair. Start by making a bridge (D) Authors is the plural form of author. Correct
between the words given in the question stem—
kind and benevolent are synonyms. The relationship (E) Mathematics is the plural form of equation.
shared is that of synonyms. Incorrect

Now, plug the bridge into the answer choices: The correct answer is D.

(A) Empty and full are synonyms. Incorrect 31. This is an analogy based question. The idea here is
(B) Blemish and loyalty are synonyms. Incorrect to find the option which draws a similar relationship
(C) Absurd and rational are synonyms. Incorrect to that of the given pair. Start by making a bridge
(D) Stubborn and obstinate are synonyms. Correct between the words given in the question stem—
Diffident and shy are synonyms. The relationship
(E) Capricious and unchanging are synonyms. shared is that of synonyms.
Incorrect
Now, plug the bridge into the answer choices:
The correct answer is D.
(A) Arrogant and valiant are synonyms. Incorrect
29. This is an analogy based question. The idea here is (B) Laconic and sarcastic are synonyms. Incorrect
to find the option which draws a similar relationship (C) Garrulous and talkative are synonyms. Correct
to that of the given pair. Start by making a bridge (D) Onerous and portentous are synonyms. Incorrect
between the words given in the question stem—
Bread is made of grain. (E) Incoherent and stammering are synonyms.
Incorrect
Now, plug the bridge into the answer choices:
The correct answer is C.
(A) Many is made of berries. Incorrect

456

Book 1.indb 456 30/04/2019 4:49:07 PM


5.0   Language Skills Practice

3 Synonyms (D) Intermittent means occurring at irregular


intervals; not continuous or steady. This option
can also be rejected.
1. The purpose of the question is to find a word similar
(E) Ephemeral means lasting for a very short time.
in meaning to that of the given word. In other words
This is also not the correct answer.
one has to locate the synonym of Expendable.
Expendable means of relatively little significance, and The correct answer is B.
therefore able to be abandoned or destroyed. 4. The purpose of the question is to find a word similar
(A) Gushing means (of speech or writing) effusive or in meaning to that of the given word. In other words
exaggeratedly enthusiastic. This is unrelated and one has to locate the synonym of Trite. Trite refers to
can be negated. (of a remark or idea) lacking originality or freshness.
(B) Anomaly means something that deviates from (A) Innovative means (of a product, idea, etc.)
what is standard, normal, or expected. This can featuring new methods; advanced and original.
also be negated. This is an antonym and thus can be rejected.
(C) Dejected means sad and depressed. (B) Exiguous means very small in size or amount.
(D) Superfluous means unnecessary, especially This is completely unrelated.
through being more than enough. (C) Correct. Hackneyed means (of a phrase or idea)
having been overused. This is a synonym and
(E) Parsimonious means very unwilling to spend
thus is the correct answer.
money or use resources.
(D) Opportune means (of a time) especially
The correct answer is D.
convenient or appropriate for a particular action
2. The purpose of the question is to find a word similar or event. This can be negated as a synonym.
in meaning to that of the given word. In other words (E) Pertinent means relevant or applicable to a
one has to locate the synonym of Distraught. particular matter. This is unrelated and can be
Distraught means very worried and upset. rejected.
(A) Subservient means prepared to obey others The correct answer is C.
unquestioningly. This can be negated.
(B) Composed means having one's feelings and 5. The purpose of the question is to find a word similar
expression under control. This is an antonym in meaning to that of the given word. In other words,
and thus can be negated. one has to locate the synonym of Dwindle. Dwindle
(C) Authoritative means able to be trusted as being refers to gradually reducing in size, amount or
accurate or true. This is unrelated to ‘distraught’. strength.

(D) Dogmatic means inclined to lay down principles (A) Enhance means to increase or further improve
as undeniably true. This can be rejected. the quality or value of something. This is an
(E) Correct. Agitated means feeling or appearing antonym and thus can be rejected.
troubled or nervous. (B) Burgeon is an antonym of ‘diminish’ that
indicates rapid increase in something.
The correct answer is E.
(C) Voracious refers to consuming large quantities
3. The purpose of the question is to find a word similar of something (generally food). The word is not
in meaning to that of the given word. In other directly related to the main word.
words one has to locate the synonym of Relentless. (D) Correct. Dwindle and diminish are synonyms that
Relentless means persistent or unceasingly intense. indicate gradual reduction in something.
(A) Lenient means (of a punishment or person (E) Surge refers to a sudden, powerful movement,
in authority) more merciful or tolerant than especially by a crowd or a natural force. This is
expected. This is not the required synonym. an antonym and can be rejected.
(B) Correct. Dogged means having or showing The correct answer is D.
tenacity and grim persistence. This is a synonym
of ‘relentless’. 6. The purpose of the question is to find a word similar
(C) Painstaking means done with or employing great in meaning to that of the given word. In other words,
care and thoroughness. This can be negated. one has to locate the synonym of Loquacious.
‘Loquacious’ means tending to talk a great deal.

457

Book 1.indb 457 30/04/2019 4:49:07 PM


NMAT by GMAC™ Official Guide 2019

(A) ‘Taciturn’ means (of a person) reserved or (A) ‘Gawky’ means nervously awkward.
uncommunicative in speech. This is an antonym (B) ‘Inert’ means lacking the ability or strength to
and thus can be rejected. move.
(B) Correct. ‘Verbose’ means using or expressed in (C) Correct. ‘Ingratiating’ means intended to gain
more words than are needed. approval or favour.
(C) ‘Mercurial’ means subject to sudden or
(D) ‘Diabolic’ means relating to or characteristic of
unpredictable changes of mood or mind.
the Devil, something that is extremely wicked.
(D) ‘Placid’ means calm or serene.
(E) ‘Cantankerous’ means bad-tempered,
(E) ‘Unscrupulous’ means having or showing no argumentative, and uncooperative. This is an
moral principles. antonym and thus can be rejected.
The correct answer is B. The correct answer is C.

7. The purpose of the question is to find a word similar 10. The purpose of the question is to find a word similar
in meaning to that of the given word. In other words, in meaning to that of the given word. In other words,
one has to locate the synonym of Quibble. ‘Quibble’ one has to locate the synonym of Odious. ‘Odious’
means argue or raise objections about a trivial means extremely unpleasant.
matter.
(A) ‘Salubrious’ means healthy.
(A) ‘Proscribe’ means forbid, especially by law. The
(B) ‘Assiduous’ means showing great care and
word is not directly related to the main word.
perseverance.
(B) ‘Ingest’ means take (food, drink, or another
(C) ‘Laconic’ means (of a person, speech, or style of
substance) into the body by swallowing or
writing) using very few words.
absorbing it.
(C) Hoodwink’ means to deceive or trick. (D) ‘Restive’ means restless.
(D) ‘Elucidate’ means to explain. (E) Correct. ‘Execrable’ means extremely
unpleasant.
(E) Correct. 'Carp' means to continually complain
about trivial matters. The correct answer is E.
The correct answer is E. 11. The purpose of the question is to find a word similar
in meaning to that of the given word. In other words,
8. The purpose of the question is to find a word similar one has to locate the synonym of Serendipitous.
in meaning to that of the given word. In other words, ‘Serendipitous’ means occurring or discovered by
one has to locate the synonym of Encumbrance. chance.
‘Encumbrance’ means hindrance or an impediment or
burden. (A) ‘Extraneous’ means irrelevant or unrelated to the
subject being dealt with.
(A) ‘Machination’ is a plot or scheme.
(B) ‘Sagacious’ means wise or shrewd.
(B) Correct. ‘Obstacle’ is a hurdle. This is a synonym
(C) Correct. ‘Accidental’ means happening by
and thus is the correct answer.
chance, unintentionally, or unexpectedly.
(C) ‘Skepticism’ indicates doubt as to the truth of
(D) ‘Scathing’ means severely critical.
something.
(E) ‘Turbid’ means (of a liquid) cloudy, opaque or
(D) ‘Ruse’ is a ploy or a strategy.
thick with suspended matter.
(E) ‘Stoicism’ is the endurance of pain or hardship
without the display of feelings and without The correct answer is C.
complaint.
12. The purpose of the question is to find a word similar
The correct answer is B. in meaning to that of the given word. In other words,
one has to locate the synonym of Obviate. ‘Obviate’
9. The purpose of the question is to find a word similar means to remove a need or difficulty.
in meaning to that of the given word. In other words,
one has to locate the synonym of Obsequious. (A) ‘Relinquish’ means to voluntarily give up a thing
Obsequious means obedient or attentive to an or claim.
excessive or servile degree. (B) ‘Vacillate’ means to be indecisive.

458

Book 1.indb 458 30/04/2019 4:49:07 PM


5.0   Language Skills Practice

(C) ‘Tamper’ means to interfere with (something) in (C) Erudite means having or showing great
order to cause damage or make unauthorised knowledge or learning.
alterations. (D) Correct. Demure means (of a woman or her
(D) ‘Trounce’ means to defeat heavily in a contest. behaviour) reserved, modest, and shy.
(E) Correct. ‘Forestall’ means to prevent or obstruct (E) Fallacious means based on a mistaken belief.
(an anticipated event or action) by taking
The correct answer is D.
advance action.
The correct answer is E. 16. The purpose of the question is to find a word similar
in meaning to that of the given word. In other
13. The purpose of the question is to find a word similar words, one has to locate the synonym of Vandalize.
in meaning to that of the given word. In other words, Vandalize means to deliberately destroy or damage
one has to locate the synonym of Vituperation. (public or private property).
‘Vituperation’ means bitter and abusive language.
(A) Venerate means regard with great respect.
(A) Adulation means excessive admiration or praise. (B) Correct. Defile means to damage the purity or
This is an antonym and can be rejected. appearance of something.
(B) Correct. Opprobrium means harsh criticism or (C) Disparage means to regard or represent as
censure. being of little worth.
(C) Exaltation means a feeling or state of extreme (D) Quaver means (of a person’s voice) shake
happiness. or tremble while speaking, typically out of
(D) Ostentation means the pretentious or showy nervousness or emotion.
display of wealth and luxury, designed to (E) Fetter means to restrain with chains or
impress. manacles, typically around the ankles.
(E) Recrimination means an accusation in response
The correct answer is B.
to one from someone else.
The correct answer is B. 17. The purpose of the question is to find a word similar
in meaning to that of the given word. In other words,
14. The purpose of the question is to find a word similar one has to locate the synonym of Inscrutable.
in meaning to that of the given word. In other words, Inscrutable means impossible to understand or
one has to locate the synonym of Spooky. Spooky interpret.
means sinister or ghostly in a way that causes fear
(A) Tractable means (of a person) easy to control or
and unease.
influence.
(A) Noxious means harmful, poisonous, or very (B) Perfidious means deceitful and untrustworthy.
unpleasant.
(C) Buoyant means cheerful and optimistic.
(B) Hapless means (especially of a person)
(D) Correct. Enigmatic means difficult to interpret or
unfortunate.
understand.
(C) Correct. Eerie means strange and frightening.
(E) Vociferous means expressing or characterised
(D) Apprehensive means anxious or fearful that by vehement opinions.
something bad or unpleasant will happen.
The correct answer is D.
(E) Lamentable means (of circumstances or
conditions) very bad; deplorable. 18. The purpose of the question is to find a word
The correct answer is C. similar in meaning to that of the given word. In
other words, one has to locate the synonym of
15. The purpose of the question is to find a word similar Insouciant. Insouciant means lacking enthusiasm and
in meaning to that of the given word. In other words, determination.
one has to locate the synonym of Bashful. Bashful
(A) Staid means sedate, respectable, and
means shy.
unadventurous.
(A) Mordant means (especially of humour) having or (B) Verdant means (of countryside) green with grass
showing a sharp or critical quality. or other rich vegetation.
(B) Didactic means intended to teach, particularly in (C) Oblique means not expressed or done in a direct
having moral instruction as an ulterior motive. way.

459

Book 1.indb 459 30/04/2019 4:49:07 PM


NMAT by GMAC™ Official Guide 2019

(D) Correct. Nonchalant means (of a person or (C) Correct. Cowardly means lacking courage.
manner) feeling or appearing casually calm (D) Juvenile means for or relating to young people.
and relaxed; not displaying anxiety, lacking
(E) Illicit means unlawful.
enthusiasm and determination.
(E) Squalid means (of a place) extremely dirty and The correct answer is C.
unpleasant, especially as a result of poverty or
22. The purpose of the question is to find a word similar
neglect.
in meaning to that of the given word. In other words,
The correct answer is D. one has to locate the synonym of Mawkish. Mawkish
means sentimental in an exaggerated or false way.
19. The purpose of the question is to find a word similar
in meaning to that of the given word. In other words, (A) Correct. ‘Overly sentimental’ means overly
one has to locate the synonym of Corroborate. emotional.
Corroborate means to make more certain. (B) Melodramatic means showing much stronger
emotions than are necessary or usual for a
(A) Correct. Confirm is a synonym and the correct
situation.
answer.
(C) Nostalgic means feeling happy and also slightly
(B) Invalidate means make or prove (an argument,
sad when you think about things that happened
statement, or theory) unsound or erroneous.
in the past.
(C) Guarantee means provide a formal assurance,
(D) Tearful means crying or inclined to cry.
especially that certain conditions will be fulfilled
relating to a product, service, or transaction. (E) Cynical means believing that people are
motivated purely by self-interest.
(D) Apprise means to inform or tell (someone).
(E) Negotiate means to obtain or bring about by The correct answer is A.
discussion.
23. The purpose of the question is to find a word similar
The correct answer is A. in meaning to that of the given word. In other
words one has to locate the synonym of Prescribe.
20. The purpose of the question is to find a word similar Prescribe means to suggest or recommend. It also
in meaning to that of the given word. In other means to state authoritatively or as a rule that (an
words, one has to locate the synonym of Enthrall. action or procedure) should be carried out.
Enthrall means to capture the fascinated attention of
(A) ‘Bundle’ means tie or roll up (a number of things)
someone.
together as though into a parcel. This can be
(A) Abridge means to shorten (a book, film, speech, negated.
and so on) without losing the sense. (B) ‘Betray’ means expose (one’s country, a group,
(B) Admonish means to warn of a fault. or a person) to danger by treacherously giving
(C) Cajole means to impose on or dupe by flattering information to an enemy. This can be eliminated
speech. as the answer.
(D) Elucidate means to bring out the facts. (C) ‘Deceive’ means deliberately cause (someone)
concerning something more clearly. to believe something that is not true, especially
for personal gain. This is incorrect and can be
(E) Correct. Enchant means to charm.
rejected.
The correct answer is E. (D) Correct. ‘Stipulate’ means to say exactly how
something must be or must be done
21. The purpose of the question is to find a word similar
in meaning to that of the given word. In other words, (E) ‘Admonish’ means to reprimand firmly.
one has to locate the synonym of Craven. Craven
The correct answer is D.
means cowardly.
(A) Candid means straightforward. 24. The purpose of the question is to find a word similar
in meaning to that of the given word. In other words
(B) Defiant means characterised by bold or insolent
one has to locate the synonym of Affront. Affront
opposition. This is an antonym and can be
means an action or remark that causes outrage or
negated.
offence.

460

Book 1.indb 460 30/04/2019 4:49:08 PM


5.0   Language Skills Practice

(A) ‘Abstruse’ means difficult to understand. This 27. The purpose of the question is to find a word similar
can be negated. in meaning to that of the given word. In other words,
(B) ‘Examine’ means to inspect (someone or one has to locate the synonym of pogrom. Pogrom
something) thoroughly in order to determine is a Russian word that refers to a mob attack, either
their nature or condition. This is also incorrect approved or controlled by authorities, against the
and can be negated. persons and property of a religious, racial or national
minority.
(C) ‘Forbear’ means to refrain. This can be
eliminated. (A) Rescue operation is a procedure to bring people
(D) Correct. ‘Scorn’ means a feeling of contempt out of danger.
and disdain. This also has a negative connotation (B) Cartoon character is not a correct synonym.
and is the correct answer. (C) Eradicate is to destroy completely.
(E) ‘Yield’ means to give way to arguments, (D) New version of memory is not a correct
demands, or pressure. synonym.
The correct answer is D. (E) Correct. Massacre is an indiscriminate and brutal
slaughter of many people.
25. The purpose of the question is to find a word similar The correct answer is E.
in meaning to that of the given word. In other words
one has to locate the synonym of Elucidate. Elucidate 28. The purpose of the question is to find a word similar
means to make (something) clear; explain. in meaning to that of the given word. In other
(A) ‘Clarify’ means to make something easier to words, one has to locate the synonym of ostensibly.
understand. This is the correct answer. Ostensibly means very apparently or ‘in quite a loud
and clear way’.
(B) ‘Confuse’ and ‘befuddle’ mean (of a person)
unable to think clearly. (A) Calmly means without agitation.
(C) ‘Befuddle’ is similar in meaning to confuse. (B) Quietly means in a quiet manner.
(D) ‘Convince’ means to cause (someone) to believe (C) Actually means really.
firmly in the truth of something. (D) Correct. Apparently means clearly/openly visible.
(E) ‘Contradict’ means to deny the truth of (a (E) Although, ‘conspicuously’ also means ‘visibly’, it
statement) by asserting the opposite. conveys a lesser degree of boldness than does
The correct answer is A. ‘apparently’. Conspicuously means specifically/
pointedly visible.
26. The purpose of the question is to find a word similar The correct answer is D.
in meaning to that of the given word. In other words,
one has to locate the synonym of portray. Portray 29. The purpose of the question is to find a word similar
means to depict. in meaning to that of the given word. In other
(A) To ‘feign’ is to pretend to be affected by words, one has to locate the synonym of caliginous.
something. Caliginous means misty or dark.
(B) Correct. Depict is the correct synonym. (A) Lambent means softly bright or radiant.
(C) To ‘invoke’ is to appeal. (B) Recondite means deep or concealed.
(D) To ‘conjure’ is to cause a ghost or spirit to (C) Cretinous means foolish or stupid.
appear by means of a magic ritual. (D) Correct. Tenebrous means dark or shadowy.
(E) To ‘simulate’ is to imitate the appearance or (E) Sophomoric means conceited and overconfident.
character of somebody or something.
The correct answer is D.
The correct answer is B.

461

Book 1.indb 461 30/04/2019 4:49:08 PM


NMAT by GMAC™ Official Guide 2019

4 Fill in the Blanks (B) ‘Coloured with’ means change the colour of
(something) by painting, dyeing, or shading it.
This can be negated.
1. Keyword—While there is no doubt
(C) ‘Satiated with’ means to fill or supply beyond
Connector—whether capacity or desire, often arousing weariness.
Prediction for blank 1—superb This is also incorrect.
Prediction for blank 2—ability (D) Correct ‘Exposed to’ means the fact of
(A) ‘Boring’ means not interesting. ‘Talent’ means experiencing something or being affected by
natural aptitude or skill. These are incorrect it because of being in a particular situation or
place.
(B) ‘Good’ can be a fit here. ‘Detriment’ which
means the state of being harmed or damaged is (E) ‘Whetted in’ means increased or enhanced (the
incorrect. appetite, desire, and so on. This can also be
negated.
(C) ‘Average’ means mediocre which is incorrect.
‘Capability’ means potential which can be a fit. The correct answer is D.
(D) Correct. ‘Excellent’ is an apt fit. ‘Potential’ which
4. Keyword—not correct
means capability is also correct.
(E) ‘Upcoming’ means coming up.’ Spectators’ Connector—none
means onlookers. These can be negated. Prediction for blank 1—distinguishing
The correct answer is D. Prediction for blank 2—but
(A) ‘Universal means relating to or done by all
2. Keyword—there is no guarantee that gluten will not people or things in the world or in a particular
be present in the raw materials group. This cannot be a fit. ‘Moreover’ means ‘in
Connector—since addition’. This is also incorrect.
Prediction for blank 1— cannot, should not (B) ’Discriminating’ means discerning. This can be a
fit. ‘Furthermore’ means in addition. This is not a
Prediction for blank 2—Safe correct fit.
(A) Correct. ‘Cannot’ is the negative of the verb ‘can, (C) ‘Subjective’ means based on or influenced
‘Benign’ means not harmful in effect. by personal feelings, tastes, or opinions.
(B) ‘Should ’is incorrect as we are looking for its ‘Surprisingly’ means unexpectedly. These can be
contrast though ‘unsafe’ can be an option. negated.
(C) ‘Will not’ is incorrect here. ‘Vulnerable’ which (D) ‘Contentious’ means controversial. ‘Nonetheless’
means exposed to the possibility of being means in spite of that. These are not correct
attacked or harmed, either physically or fits.
emotionally is also incorrect usage. (E) Correct ‘Differentiating’ means to form or mark
(D) ‘Claim to’ means declare. ‘Harmless’ means differently from other such things. ‘However’
safe. The first option is incorrect though means still or but.
‘harmless’ can be an option.
The correct answer is E.
(E) ‘Are not to’ means cannot which can be a fit.
‘Inconspicuous’ which means not clearly visible 5. Keyword—worn purely for traditional or ceremonial
or attracting attention is an incorrect option. purposes
The correct answer is A. Connector—because
Prediction for blank—logical
3. Keyword— example of the process of natural (A) ‘Accepted’ means generally believed or
selection recognised to be valid or correct. This is not the
Connector—None right fit.
Prediction for blank—laid bare (B) Correct. ‘Rational’ means based on or in
(A) ‘Painted with’ means covered the surface of accordance with reason or logic.
(something) with paint. This is an incorrect (C) ‘Critical’ means expressing adverse or
option. disapproving comments or judgements. This can
be negated.

462

Book 1.indb 462 30/04/2019 4:49:08 PM


5.0   Language Skills Practice

(D) ‘Illogical’ means lacking sense or clear, sound intervals or only in a few places. This also does
reasoning. This is not an appropriate answer. not indicate contrast. Hence, the option can be
(E) ‘Peculiar’ means different from what is normal or negated.
expected; strange. This can also be cancelled (D) ‘Exiguous’ means very small in size or amount.
out. This can also be negated.
(E) ‘Munificent’ means displaying great generosity.
The correct answer is B.
This option can also be canceled.
6. Keyword—Laughing during the funeral The correct answer is A.
Connector—None
8. Keyword—wasn’t aware
Prediction for blank 1—shocking
Connector—because of
Prediction for blank 2—done
Prediction for blank 1—morals or values
(A) Correct. ‘egregious’, ‘gross’ and’ flagrant’ mean
shocking and obviously offensive. All three can Prediction for blank 2—satisfaction or
fit in the first blank. ‘Perpetrated’ means carried dissatisfaction
out or committed (a harmful, illegal, or immoral (A) ‘Turpitude’ means moral corruption. Both,
action). ‘turpitude’ and ’satisfied’ are not an appropriate fit.
(B) ‘Inconspicuous’ means not clearly visible or (B) ‘Depravity’ also means moral corruption. Both,
attracting attention. This can be cancelled out. ‘depravity’ and ’certain’ are not an appropriate fit.
(C) Though ‘gross’ can fit the first blank, the use (C) Correct. ‘Rectitude’, ‘righteousness’ and
of ‘inflicted’ is incorrect for the second blank. ‘morality’ mean morally correct behaviour or
‘Inflicted’ means caused (something unpleasant thinking. They can fit the first blank. ‘Rectitude’
or painful) to be suffered by someone or and ‘convinced’ are the only options which fit the
something blanks appropriately. A parent will usually believe
(D) Though ‘flagrant’ can fit the first blank, the use his/her child to be a morally righteous person,
of ‘executed’ is incorrect for the second blank. unless convinced otherwise.
Executed means put (a plan, order, or course of (D) ‘Coaxed’ means to influence or gently urge
action) into effect. by caressing or flattering. This is not an
(E) ‘Resplendent’ means attractive and impressive appropriate fit.
through being richly colourful or sumptuous (E) ‘Unassertive’ means (of a person) not having or
and ‘wreaked’ means caused (a large amount showing a confident and forceful personality.
of damage or harm). This is not an appropriate This is also not an appropriate fit.
answer.
The correct answer is C.
The correct answer is A.
9. Keyword—Budget travellers
7. Keyword—The vegetation
Connector—when
Connector—But (contrast)
Prediction for blank—reduce, drop
Prediction for blanks 1 and 2—lush—thin, thin-
lush (A) ‘Escalate’ means to ‘increase rapidly’. This can
be negated.
(A) Correct. ‘Luxuriant’, ‘copious’ and ‘bountiful’
mean abundant and can fit the first blank. (B) Correct. ‘Plummet’ means ‘a steep and rapid fall
‘Sparse’ means thinly dispersed and shows or drop’.
contrast. ‘Luxuriant’ and ‘sparse’ are the correct (C) ‘Descend’ means to ‘pass from a higher place or
options. level to a lower one’. This is not an appropriate
(B) ‘Profuse’ means the same as ‘copious’ and option.
cannot fit the blank. We need a set of words that (D) ‘Abate’ means become less intense or
show contrast. Thus, this option can be negated. widespread.
(C) ‘Slender’ and ‘scanty’ mean thinly dispersed (E) Diminish means dwindle or weaken gradually in
or scattered. They can also fill the first blank. size, amount, or strength.
However, ‘sporadic’ means occurring at irregular
The correct answer is B.

463

Book 1.indb 463 30/04/2019 4:49:08 PM


NMAT by GMAC™ Official Guide 2019

10. Keyword—that he knows more (C) Correct. ‘Oxymoron’ means ‘a figure of speech
in which apparently contradictory terms appear
Connector—rather
in conjunction.
Prediction for blank—overconfident
(D) ‘Antithesis’ means ‘a person or thing that is the
(A) ‘Discourteous’ means ’showing rudeness and a direct opposite of someone or something else.
lack of consideration for other people’.
(E) ‘Rhetoric’ means ‘the art of effective or
(B) ‘Uncouth’ means ‘lacking good manners, persuasive speaking or writing, especially the
refinement, or grace’. exploitation of figures of speech and other
(C) Correct. ‘Presumptuous’ means ‘(of a person or compositional techniques’.
their behaviour) failing to observe the limits of
The correct answer is C.
what is permitted or appropriate’.
(D) ‘Peremptory’ means ‘insisting on immediate 13. Keyword—written by someone else
attention or obedience’.
Connector—when
(E) ‘Arbitrary’ means ‘based on random choice
Prediction for blank—truthfulness
or personal whim, rather than any reason or
system’. (A) Correct. ‘Veracity’ means the ‘quality of being
truthful’.
The correct answer is C.
(B) ‘Candour’ means 'frankness'.
11. Keyword—The police officer cornered (C) ‘Fidelity’ means 'faithfulness to a person,
Connector—if cause, or belief, demonstrated by continuing
loyalty and support'.
Prediction for blank 1—Unstable
(D) ‘Mendacity’ means ‘untruthfulness’. This is
Prediction for blank 2—Drunk
the opposite of the word required and can be
(A) ‘Startled’ means ‘astonished’. ‘Restrained’ negated.
means ‘unemotional’. Both the words don’t fit the
(E) ‘Congruity’ is ‘a quality of agreement and
context.
appropriateness’.
(B) Correct. ‘Staggering’ means ‘walk or move
unsteadily, as if about to fall. ‘Inebriated’ means The correct answer is A.
‘drunk’.
14. Keyword—The road is quite
(C) ‘Wobbly’ means ‘tending to move unsteadily from
side to side. ‘Despondent’ means ‘in low spirits Connector—None
from loss of hope or courage’. Though the Prediction for blank 1—winding
first word can fill the blank, the second word is
Prediction for blank 2—dizzy
unsuitable.
(D) ‘Rickety’ means ‘feeble in the joints’. This is not (A) ‘Meandering’ means ‘following a winding
an appropriate fit for the context. ‘Intoxicated’ course’. This can fill the first blank. ‘Bizarre’
means ‘drunk’. This can fill the second blank. means ‘very strange or unusual. This is not an
appropriate fit.
(E) ‘Fragile’ means ‘delicate and vulnerable’. ‘Sober’
means ‘not affected by alcohol’. Both the words (B) ‘Discursive’ means ‘digressing from subject to
don’t fit the context. subject’. This can be negated. ‘Giddy’ means
‘dizzy’.
The correct answer is B.
(C) Correct. ‘Serpentine’ means ‘curving and
12. Keyword—pretty ugly twisting like a snake’. ‘Light headed’ means
‘dizzy’.
Connector—both
(D) ‘Sinuous’ means ‘having many curves and
Prediction for blank—contradictory terms turns’. This can fill the first blank. ‘Surreal
(A) ‘Sarcasm’ means ‘the use of irony to mock or means ‘strange’. This is not an appropriate fit.
convey contempt. (E) ‘Digressive’ means ‘tending to depart from the
(B) ‘Euphemism’ means ‘a mild or indirect word subject’. This can be cancelled out. ‘Nauseous’
or expression substituted for one considered means ‘ feeling inclined to vomit’.
to be too harsh or blunt when referring to
The correct answer is C.
something unpleasant or embarrassing.

464

Book 1.indb 464 30/04/2019 4:49:08 PM


5.0   Language Skills Practice

15. Keyword—the new evidence (E) ‘Impassive’ means ‘not feeling or showing
emotion’. ‘Panegyric’ means ‘a public speech
Connector—None
or published text in praise of someone or
Prediction for blank 1—clear something’. This option can be cancelled out.
Prediction for blank 2—accused
The correct answer is A.
(A) ‘Exonerate’ means ‘(of an official body)
absolve (someone) from blame for a fault or 17. Keyword—most feared animal on Earth
wrongdoing. This can be used for the first
blank. ‘Guillible’ means someone who is easily Connector—But
influenced, swayed. This is incorrect. Prediction for blank 1—Reduces
(B) Correct. ‘Vindicate’ means ‘clear (someone) Prediction for blank 2—Ominous
of blame or suspicion’. ‘Defendant’ means Prediction for blank 3—Puzzled
‘an individual, company, or institution sued or
(A) ‘Diminish’ means to ‘make or become less.
accused in a court of law’.
‘Joyous’ means ‘happy. ‘Dazed’ means to make
(C) ‘Castigate’ means to ‘reprimand (someone) someone unable to think or react properly. This
severely’. ‘Criminal’ means ‘a person who option can be negated.
has committed a crime’. This option can be
(B) ‘Contract’ means to ‘decrease in size, number,
cancelled out.
or range. This can fill the first blank. ‘Quiver’
(D) ‘Censure’ means ‘express severe disapproval means to ‘tremble or shake with a slight
of (someone or something), especially in rapid motion’. ‘Cheeky’ means ‘showing a
a formal statement’. ‘Wrongdoer’ means ‘a lack of respect or politeness in a way that is
person who behaves illegally or dishonestly’. amusing or appealing’. Both these are not an
(E) ‘Acquit’ means ‘free (someone) from a criminal appropriate fit.
charge by a verdict of not guilty’. This can be (C) ‘Stiffen’ means ‘make or become stiff or
used to fill the first blank. ‘Reprobate’ means rigid’. This is unsuitable. ‘Menacing’ means
‘an unprincipled person’. ‘threatening’. ‘Intimidate’ means to ‘frighten’.
The correct answer is B. (D) Correct. ‘Taper’ means ‘diminish or reduce in
thickness towards one end. ‘Sinister’ means
16. Keyword—the fact that the students were bored ‘giving the impression that something harmful
Connector—None or evil is happening or will happen’. ‘Bemused’
means ‘puzzle, confuse, or bewilder’.
Prediction for blank 1—Heedless
(E) ‘Subside’ means ‘become less intense, violent,
Prediction for blank 2—Lecture
or severe’. ‘Threatening’ means ‘ominous’.
(A) Correct. ‘Oblivious’ means ‘not aware of or ‘Indignant’ means ‘resentful’. None of the words
concerned about what is happening around fit the context.
one’. ‘Harangue’ means ‘a lengthy and
aggressive speech'. The correct answer is D.
(B) ‘Unheeding’ means ‘showing a reckless lack 18. Keyword—than many give it credit for
of care or attention’. Though the word fits with
respect to the meaning, its usage will render Connector—and
the sentence grammatically incorrect. ‘Rant’ Prediction for blank 1—Varied
means ‘speak or shout at length in an angry, Prediction for blank 2—Hint
impassioned way’. This can be used for the
Prediction for blank 3—Innumerable
second blank.
(A) ‘Distinct’ means ‘recognizably different in
(C) ‘Indifferent’ means unconcerned’. ‘Onslaught’
nature from something else of a similar type’.
means ‘a fierce or destructive attack’.
This is appropriate in the context. ‘Traces’
(D) ‘Impervious’ means ‘showing a reckless lack means ‘a mark, object, or other indication
of care or attention’. ‘Polemic’ means ‘a of the existence or passing of something.
strong verbal or written attack on someone or ‘Multitude’ means ‘a great number of people or
something’. thing’. This is grammatically inappropriate.

465

Book 1.indb 465 30/04/2019 4:49:08 PM


NMAT by GMAC™ Official Guide 2019

(B) ‘Disparate’ means ‘essentially different in kind’. 20. Keyword—harsh teacher


‘Vestige’ means ‘a trace or remnant of something Connector—in order to
that is disappearing or no longer exists. ‘Profuse’
Prediction for blank—command, control
means ‘abundant’. This option can be cancelled.
(A) ‘Humanity’ means the quality of being humane.
(C) Correct. ‘Diverse’ means ‘showing a great deal
of variety’. ‘Echoes’ means ‘detail or feature (B) Correct. ‘Authority’ means the power or right
which reminds you of something else’. ‘Myriad’ to give orders, make decisions, and enforce
means ‘a countless or extremely great number obedience.
of people or things’. This is the correct answer. (C) ‘Generosity’ means the quality of being kind
(D) ‘Distinguishable’ means ‘clear enough to be and generous.
recognised or identified as different’. ‘Memento’ (D) ‘Selflessness’ indicates a person who is more
means ‘an object kept as a reminder of a concerned with the needs and wishes of others
person or event. ‘Excessive’ means ‘more than than with one’s own.
is necessary’. This is not an appropriate fit. (E) ‘Worthlessness’ means the state or quality of
(E) ‘Different’ means ‘not the same as another or being unimportant or useless.
each other’. ‘Reminder’ means ‘a thing that
The correct answer is B.
causes someone to remember something’.
‘Riot’ means ‘an impressively large or varied
display of something’. 21. Keyword—idea of

The correct answer is C. Connector—none


Prediction for blank 1—significant, challenging
19. Keyword—clearly inspired by the challenge of Prediction for blank 2—barriers, hindrances
reproducing the event
(A) ‘Lame’ means (of an explanation or excuse)
Connector—and unconvincingly feeble. ‘Jinx’ means a person or
Prediction for blank 1—Calamities thing that brings bad luck. These are incorrect
options.
Prediction for blank 2—Credibility
(B) ‘Appreciable’ means large or important enough
Prediction for blank 3—Unthinkable
to be noticed. This can be a fit. 'Chaos’ means
(A) ‘Tragedy’ means ‘an event causing great and complete disorder and confusion. This is
often sudden damage’. This can be a fit. incorrect.
‘Authenticity is the quality of being genuine or
(C) ‘Presuming’ means to take for granted, This is
real. ‘Conceivable’ means ‘capable of being
incorrect. ‘Obstacles’ means a thing that blocks
imagined or grasped mentally. This is incorrect.
one’s way or prevents or hinders progress. This
(B) ‘Calamity’ means ‘a disaster’. ‘Plausibility’ can be a fit but is negated.
means ‘the quality of seeming reasonable
(D) ‘Overconfident’ means excessively confident.
or probable’. ‘Ineffable’ means ‘too great or
‘Hurdle’ means an obstacle. This can be negated
extreme to be expressed or described in
though ‘hurdles’ would have been correct.
words. This option can be negated.
(E) Correct. ‘Ambitious’ means (of a plan or piece
(C) ‘Affliction’ means ‘a cause of pain or harm’.
of work) intended to satisfy high aspirations
‘Chicanery’ means ‘the use of deception or
and therefore difficult to achieve. ‘Impediments’
subterfuge to achieve one’s purpose. ‘Improbable’
means hurdles. This is the correct answer.
means ‘not likely to be true or to happen’. These
words are not relevant in the context. The correct answer is E.
(D) Correct. ‘Disaster’ means ‘a sudden accident
or a natural catastrophe that causes great 22. Keyword—to stretch our legs
damage or loss of life. ‘Verisimilitude’ means Connector—none
‘the appearance of being true or real.
‘Inconceivable’ means ‘unbelievable’. Prediction for blank—include
(E) ‘Tribulations’ means ‘a cause of great trouble (A) Incorrect option.
or suffering’. ‘Artfulness’ means ‘slyly crafty (B) Correct.‘Factor in’ means to include something
or cunning’. ‘Unimaginable’ means ‘difficult or when you are doing a calculation, or when you
impossible to imagine or comprehend'. are trying to understand something. This is a
The correct answer is D. phrasal verb and the correct answer.

466

Book 1.indb 466 30/04/2019 4:49:08 PM


5.0   Language Skills Practice

(C) Incorrect option. (A) Correct. Derogatory means to express criticism


(D) Incorrect option. or insult. This the best adjective to describe the
comments which are unacceptable for him.
(E) Incorrect option.
(B) Erroneous means wrong.
The correct answer is B. (C) Illaudable means not laudable.
(D) Measly means ridiculously small.
23. Keyword—after being questioned
(E) Tedious means slow or monotonous.
Connector—none (same direction)
The correct answer is A.
Prediction for blank—revealed
(A) ‘Emerged’ means moved out of or away from 25. Keyword—Having been selected
something and became visible.
Connector—N/A
(B) ‘Expected’ means regarded (something) as
likely to happen. Prediction—He did something.
(C) ‘Concealed’ means hidden. (A) Correct. In the given sentence, the subject is
‘he’. ‘He’ has been selected to represent the
(D) Correct. ‘Disclosed’ means make (secret or
Association.
new information) known. The correct word
should be revealed. Disclosed matches this (B) This is a passive phrase while the sentence is in
best and should be the correct answer. active voice.
(E) ‘Threatened’ means to intimidate or frighten. (C) The subject is ‘he’ and not ‘members’.
The correct answer is D. (D) The subject is ‘he’ and not ‘members’.
(E) The subject is ‘he’ and not ‘member’.
24. Keyword—difficult to bear
The correct answer is A.
Connector—after
 Prediction—a word that doesn’t do justice to
Ashwin’s diligence

467

Book 1.indb 467 30/04/2019 4:49:08 PM


NMAT by GMAC™ Official Guide 2019

5 Cloze Test 5. Prediction for blank—generously


(A) Correct. ‘Liberally’ means in large or generous
amounts.
1. Prediction for blank—traveller
(B) ‘Munificently’ means very liberal in giving or
(A) ‘Lunatic’ means a person who is mentally ill.
bestowing.
(B) ‘Historian’ means someone who writes about
(C) ‘Indulgently’ means allowing someone to have
or studies history.
or do what they want, especially when this is
(C) Correct. ‘Wanderer’ means a person who not good for them.
travels aimlessly.
(D) ‘Reasonably’ means to a moderate or
(D) ‘Merchant’ means a person or company acceptable degree.
involved in wholesale trade, especially one
(E) ‘Judiciously’ means using or showing judgment.
dealing with foreign countries or supplying
goods to a particular trade. The correct answer is A.
(E) ‘Warrior’ means (especially in former times) a
6. Prediction for blank—records, chronicles
brave or experienced soldier or fighter.
(A) ‘Archives’ means a collection of historical
The correct answer is C. documents or records providing information
about a place, institution, or group of people.
2. Prediction for blank —showed, revealed
(B) Correct. Describes means to give a detailed
(A) ‘Repudiated’ means rejected. account of something.
(B) ‘Proclaimed’ means announced officially or (C) ‘Personate’ means to play the part of
publicly. (a character in a drama).
(C) ‘Created’ means brought (something) into (D) ‘Portray’ means to describe (someone or
existence. something) in a particular way.
(D) ‘Surfaced’ means became known. (E) ‘Render’ means to represent or depict
artistically.
(E) Correct. ‘Exhibited’ means manifested clearly
(a quality or a type of behaviour). The correct answer is B.
The correct answer is E. 7. Prediction for blank—idea, belief
3. Prediction for blank— having (as a quality) (A) ‘Genesis’ means the origin or mode of
(A) Correct. ‘Possessing’ means having something formation of something.
as a quality. (B) ‘Proposal’ means a plan or suggestion,
(B) ‘Surrendering’ means giving something that is especially a formal or written one, put forward
yours to someone else because you have been for consideration by others.
forced to do so or because it is necessary to (C) ‘Assertion’ means a confident and forceful
do so. statement of fact or belief.
(C) ‘Utilizing’ means making practical and effective (D) Correct. ‘Notion’ means a conception of or
use of. belief about something.
(D) ‘Rejecting’ means refusing. (E) ‘Allegation’ means a claim or assertion that
(E) ‘Affording’ means providing or supplying (an someone has done something illegal or wrong,
opportunity or facility). typically one made without proof.

The correct answer is A. The correct answer is D.

4. Prediction for blank—lived in, dwelt 8. Prediction for blank—discovery


(A) ‘Struggled’ means fought or grappled. (A) ‘Detection’ means the action or process
of identifying the presence of something
(B) ‘Adhered’ means believed in and followed the
concealed. Grammatically incorrect.
practices of.
(B) ‘Realisation’ means an act of becoming fully
(C) ‘Loathed’ means hated. aware of something as a fact.
(D) Correct. ‘Resided’ means lived in. (C) Correct. ‘Find’ means a discovery of
(E) ‘Labored’ means worked hard. something.
The correct answer is D.

468

Book 1.indb 468 30/04/2019 4:49:09 PM


5.0   Language Skills Practice

(D) ‘Apprehension’ means anxiety or fear that (C) ‘Introductory’ means serving as an introduction
something bad or unpleasant will happen. to a subject or topic. Though correct in
(E) ‘Cognizance’ means knowledge or awareness. meaning, this is inappropriate grammatically.
(D) ‘Opening’ means a beginning. This is an
The correct answer is C.
incorrect fit.
9. Prediction for blank—give access (E) ‘Inaugural’ means marking the beginning of an
(A) Incorrect option. institution, activity, or period of office. Though
correct in meaning, this is inappropriate
(B) Incorrect option.
grammatically.
(C) Correct. If a building, organisation, or country
‘opens its doors’, it allows people to enter, The correct answer is B.
especially for the first time. 13. Prediction for blank—present everywhere,
(D) Incorrect option. universal
(E) Incorrect option. (A) Rampant means (especially of something
The correct answer is C. unwelcome) flourishing or spreading
unchecked.
10. Prediction for blank—extend over (B) Widespread means found or distributed over a
(A) ‘Ranging’ means vary or extend between large area or number of people.
specified limits. (C) Correct. Ubiquitous means present, appearing,
(B) ‘Stretching’ means to cause something to or found everywhere.
reach, often as far as possible, in a particular (D) Prevalent means widespread in a particular
direction. area or at a particular time. Though related
(C) ‘Incorporating’ means take in or contain in meaning, its use will render the sentence
(something) as part of a whole. grammatically incorrect.
(D) Correct. ‘Spanning’ means extend across (a (E) Pervasive means widespread in a particular
period of time or a range of subjects). area or at a particular time. Though related
in meaning, its use will render the sentence
(E) ‘Traversing’ means travel across or through.
grammatically incorrect.
The correct answer is D.
The correct answer is C.
11. Prediction for blank—place of interest 14. Prediction for blank—superimpose
(A) ‘Entertainment’ means the action of providing (A) Analogy means a comparison between one
or being provided with amusement or thing and another, typically for the purpose of
enjoyment. explanation or clarification.
(B) ‘Distraction’ means a thing that prevents (B) Correct. Juxtaposition means two things being
someone from concentrating on something seen or placed close together with contrasting
else. effect.
(C) Correct. ‘Attraction’ means a place which (C) Similarity means the state or fact of being
draws visitors by providing something of similar.
interest or pleasure.
(D) Discrepancy means an illogical or surprising
(D) ‘Extravaganza’ means a large, exciting, and lack of compatibility or similarity between two
expensive event or entertainment. or more facts.
(E) ‘Amusement’ means the provision or enjoyment (E) Incongruity means unusual or different from
of entertainment. what is around or what is generally happening.
The correct answer is C. The correct answer is B.
12. Prediction for blank—First 15. Prediction for blank—unimpressive, common
(A) ‘Initial’ means existing or occurring at the (A) Exceptional means unusual and uncommon.
beginning. Though correct in meaning, this is
inappropriate grammatically. (B) Delectable means (of food or drink) delicious.
(B) Correct. ‘Maiden’ means being or involving the (C) Correct. Mediocre means average, not very
first attempt or act of its kind. good.

469

Book 1.indb 469 30/04/2019 4:49:09 PM


NMAT by GMAC™ Official Guide 2019

(D) Appetizing means something that stimulates (D) Oriented means tailored or adapted to
one's appetite. specified circumstances.
(E) Substantial means ‘of considerable (E) Highlight means to draw special attention to.
importance, size, or worth’.
The correct answer is C.
The correct answer is C.
20. Prediction for blank—guess, conjecture
16. Prediction for blank—simple, non-fussy (A) Veil means something that covers or hides like
(A) Ostentatious means characterised by a veil.
pretentious or showy display. This conveys (B) Reveal means make (previously unknown or
a meaning that is opposite of our intended secret information) known to others.
meaning and can be negated.
(C) Surpass means to exceed; be greater than.
(B) Correct. Rustic refers to furniture or material
that is plain and simple in its design and (D) Clarified means made (a statement or situation)
execution. less confused and more comprehensible.

(C) Brash means having an ostentatious or (E) Correct. Surmise means suppose that
tasteless appearance. something is true without having evidence to
confirm it.
(D) Grandiose means extravagantly or
pretentiously imposing in appearance or style. The correct answer is E.
(E) Unimpressive means not striking. 21. Prediction for blank—paradoxically (There is a
The correct answer is B contrast being brought out in the sentence)
(A) Correct. Ironically is used to draw attention
17. Prediction for blank—approach
to a situation which is odd or amusing
(A) Taste means perceive or experience the because it involves a contrast.
flavour of. (B) Naturally is used to indicate that you think
(B) Need means necessity. something is very obvious and not at all
(C) Desire means wish. surprising in the circumstances.
(D) Excess means exceeding a prescribed or (C) Fastidiously means very attentive to and
desirable amount. concerned about accuracy and detail.
(E) Correct. Access means a way or means of (D) Suspiciously means with a cautious distrust
approach or entry. or suspicion of someone or something.
(E) Contemptuously means in a scornful way
The correct answer is E. that shows disdain.
18. Prediction for blank—plot, manipulate, scheme The correct answer is A.
(A) Report means to give an account of.
22. Prediction for blank—isolation, detachment
(B) Forecast means to predict or estimate.
(C) Evaluate means to assess. (A) Elevation means promotion or advancement.
(D) Memorise means to learn by heart. (B) Correct. Alienation means the feeling that you
have no connection with the people around
(E) Correct. Manoeuvre means carefully guide or you.
manipulate (someone or something) in order to
achieve an end. (C) Allegation means a statement, made without
giving proof, that someone has done
The correct answer is E. something wrong or illegal.
19. Prediction for blank—obvious, apparent (D) Alleviation means the action or process of
making suffering, deficiency, or a problem less
(A) Show means to be visible. severe.
(B) Appeal means making a serious, urgent, or (E) Mollification means to soothe in temper or
heartfelt request. disposition.
(C) Correct. Evident means clearly seen or
understood. The correct answer is B.

470

Book 1.indb 470 30/04/2019 4:49:09 PM


5.0   Language Skills Practice

23. Prediction for blank—uncared for, disregarded 27. Prediction for blank— feel sad
(A) Discernible means able to be seen or understood. (A) Guilt means the fact of having committed a
(B) Correct. Neglected’ means not receiving specified or implied offence or crime.
proper attention. (B) Repel means drive or force back or away.
(C) Negligent means careless. (C) Offend is to cause feel upset, annoyed or
(D) Forgiven means to grant pardon to (a person). resentful.
(E) Omitted means left out or left undone. (D) Correct. The author regrets or laments not
disclosing his feelings.
The correct answer is B.
(E) Revenge means the action of hurting or harming
24. Prediction for blank—enliven, animate someone in return for an injury or wrong suffered
at their hands.
(A) Enlighten means give (someone) greater
knowledge and understanding about a subject The correct answer is D.
or situation.
28. Prediction for blank— relationship
(B) Correct. Brighten means make or become
happier and more cheerful. (A) Correct. Rapport means a close and harmonious
(C) Lighten means to reduce the burden; make relationship. The sentence talks of a positive
things easier. bond. So, ‘rapport’ is an apt word to fill the
blank.
(D) Signify means to be an indication of.
(B) Bondage means state of being a slave.
(E) Burden means to load heavily.
(C) Sympathy means feelings of pity and sorrow for
The correct answer is B. someone else’s misfortune.
(D) Compassion means sympathetic pity and
25. Prediction for blank— the past concern for the sufferings or misfortunes of
(A) Totality means whole of something. others.
(B) Addition means action of process of adding (E) Resemblance means the state of resemblance
something to someone else. or being alike.
(C) Correct. Retrospect means act of thinking about The correct answer is A.
the past. The sentence talks of what the author
should have done. 29. Prediction for blank— Positive of investigators
(D) Memorising means to learn by heart.
(A) Glued means fasten or join with or as if with
(E) Recollection refers to a memory. glue.
The correct answer is C. (B) Diligently means in a way that shows care and
conscientiousness in one’s work or duties.
26. Prediction for blank— negatively affect (C) Fascinated means strongly attracted and
(A) Sad means unhappy. interested.
(B) Burst means break open. You cannot burst a (D) Preserved means maintain (something) in its
friendship. original or existing state.
(C) Effect means to bring about. (E) Correct. Conscientious means diligent and
careful. The sentence mentions a ‘sense of
(D) Annoy means to irritate. You cannot annoy a mission’.
friendship.
(E) Correct. Regret is to feel sad or disappointed The correct answer is E.
over something that one has done or failed to
do. The sentence suggests that the author didn’t 30. Prediction for blank— Communicate information
want to negatively affect their friendship. and truth

The correct answer is E. (A) To move is to change position.

471

Book 1.indb 471 30/04/2019 4:49:09 PM


NMAT by GMAC™ Official Guide 2019

(B) To reassign is to appoint someone to a different 32. Prediction for blank— Be like other exemplary
post or role. journalists
(C) To relocate is to move to a new place and (A) Vie means compete eagerly with someone in
establish one’s home or business there. order to do or achieve something.
(D) Correct. Transmit is to communicate or convey. (B) Chase is to pursue in order to catch or catch up
The sentence talks of discovering and sharing with.
information. So, the synonym of sharing –
(C) Contend means to struggle to surmount.
‘transmit’ is the correct answer.
(D) Correct. Emulate is to match or surpass a
(E) To transport is to take or carry people or
person or achievement typically by imitation. The
goods from one place to another by means of a
sentence has a positive tone and in the given
vehicle, aircraft, or ship.
tone and context, ‘emulate’ is the best fit.
The correct answer is D. (E) Challenge is to a call to someone to participate
in a competitive situation or fight to decide who
31. Prediction for blank— Excessive demand for news is superior in terms of ability or strength.
(A) Greedy’ refers to excessive desire. The correct answer is D.
(B) Devoted does not fit the given context.
(C) Voracious means engaging in an activity with
great enthusiasm.
(D) Correct. Insatiable means impossible to satisfy.
The sentence talks of balancing an excessive
demand for news which is either difficult or
impossible to satisfy.
(E) Gluttonous is excessively greedy. It is used with
reference to food.
The correct answer is D.

472

Book 1.indb 472 30/04/2019 4:49:09 PM


5.0   Language Skills Practice

6 Identify the Error (A) Correct. ‘that is’ is grammatically incorrect. The
correct expression is ‘who is’. We use ‘who’ for
persons.
1. The aim is to find the phrase which is making the
(B) ‘for’ is correctly used.
given statement grammatically incorrect. The clue
word in the given sentence is ‘both’. The word ‘both’ (C) ‘is one of’ is correct.
vouches for two events or people like, ‘both the (D) ‘most important clients’ is also correct here.
boys… or ‘both Robin and Batman’, etc. Keeping this (E) There is an error in A.
in mind one may analyse the given sentence.
The correct answer is A.
(A) ‘to lose’ is a grammatically correct expression.
(B) ‘need to’ is also required here 5. The aim is to find the phrase which is making the
given statement grammatically incorrect. ‘Everyday’
(C) Correct. ‘Both’ always takes ‘and’ and not ‘as
is an adjective and its use is incorrect here. Keeping
well as’.
this in mind one may analyse the given sentence.
(D) The adverb ‘regularly’ is required here.
(A) ‘Hoping’ is correctly used.
(E) There is an error in C.
(B) ‘a permanent position’ is also a correct
The correct answer is C. expression.
2. The aim is to find the phrase which is making the (C) ‘put in’ is also the correct phrasal verb.
given statement grammatically incorrect. We have (D) Correct. ‘everyday’ is wrongly used here. We
two independent clauses here. Keeping this in mind need to use the noun phrase ‘every day’ (with a
one may analyse the given sentence. space in the middle).
(A) ‘artist’s most impressive’ is a grammatically (E) There is an error in D.
correct expression. The correct answer is D.
(B) ‘have been produced’ is also correct.
6. The aim is to find the phrase which is making the
(C) Correct. ‘New York, he moved there’ contains an
given statement grammatically incorrect. The use of
error. Since we have two independent clauses,
although and yet in the same sentence is redundant.
they cannot be connected using a comma. We
Keeping this in mind one may analyse the given
need to use a semi colon instead.
sentence.
(D) 'native place, India'. This is also correctly used.
(A) ‘studied’ is correctly used here.
(E) There is an error in C.
(B) ‘for the test’ is also grammatically correct.
The correct answer is C.
(C) Correct. ‘yet’ is incorrect.
3. The aim is to find the phrase which is making the (D) ‘only manage’ is correct.
given statement grammatically incorrect. The verb (E) There is an error in C.
in the second clause is not in the same form as the
first. Keeping this in mind one may analyse the given The correct answer is C.
sentence.
7. The aim is to find the phrase which is making the
(A) ‘pursue their MBAs’ is correct. given statement grammatically incorrect. Parallelism
(B) ‘completing college, while’ is grammatically has not been observed in the grammatical structure.
correct. Keeping this in mind one may analyse the given
sentence.
(C) Correct. ‘working for’ contains an error. ‘Work
for' is the correct form of the verb here. (A) ‘large and barren’ is correct.
(D) ‘before seeking’ is correctly used (B) ‘is at once’ is correctly used.
(E) There is an error in C (C) ‘climatically harsh’ is a grammatically correct
expression.
The correct answer is C.
(D) Correct. ‘the beauty of it is haunting’ is incorrect.
4. The aim is to find the phrase which is making the In order to maintain a parallel structure, the
given statement grammatically incorrect. The correct correct phrase should be ‘hauntingly beautiful’.
relative pronoun has to be used. Keeping this in mind (E) There is an error in D.
one may analyse the given sentence.
The correct answer is D.

473

Book 1.indb 473 30/04/2019 4:49:09 PM


NMAT by GMAC™ Official Guide 2019

8. The aim is to find the phrase which is making the (B) ‘of artefacts from’ is correct.
given statement grammatically incorrect. The (C) ‘period, going’ is also correct.
sentence talks about an action that started in the
(D) ‘as’ is correct usage.
past and is still carrying on. The usage of simple past
and past perfect continuous is correct. Keeping this (E) There is an error in A.
in mind one may analyse the given sentence. The correct answer is A.
(A) ‘started spewing’ is correct
12. The aim is to find the phrase which is making the
(B) ‘and’ as a connector is correct given statement grammatically incorrect. The
(C) ‘has been erupting’ is also correct. Past perfect sentence is constructed correctly. Keeping this in
tense is used for an action still carrying on. mind one may analyse the given sentence.
(D) ‘ever since’ is the correct usage. (A) ‘as to which’ is grammatically fine.
(E) There is no error in the sentence. (B) ‘would prefer’ is correct.
The sentence is correct as written. (C) ‘replied that’ is also correct.
The correct answer is E. (D) ‘was fine’ is correct construction.
(E) There is no error in the sentence.
9. The aim is to find the phrase which is making the
given statement grammatically incorrect. Since The correct answer is E.
from is used before ‘not only’, it does not have to be
used before ‘but also’. Keeping this in mind one may 13. The aim is to find the phrase which is making the
analyse the given sentence. given statement grammatically incorrect, There is
a subject-verb agreement error in the sentence.
(A) ‘items sourced’ is correct. Keeping this in mind one may analyse the given
(B) ‘not only Asia’ is correct. sentence.
(C) Correct. ‘from’ is incorrect. ‘From’ used before (A) ‘Each of the’ is correct.
‘not only’ takes care of the entire sentence. (B) Correct. ‘has a’ should be used in place of
(D) ‘Central and Latin Americas’ is correct. ‘have a’. 'Each of the’ takes a plural noun and a
(E) There is an error in C. singular verb.

The correct answer is C. (C) ‘narrate’ is also correct.


(D) ‘about the crime’ is correct as well.
10. The aim is to find the phrase which is making the (E) There is an error in B.
given statement grammatically incorrect. ‘Items’
should be compared to ‘items’ and not ‘restaurant'. The correct answer is B.
Keeping this in mind one may analyse the given
14. The aim is to find the phrase which is making the
sentence.
given statement grammatically incorrect, The
(A) ‘served in’ is correct. sentence has a redundancy error. Keeping this in
(B) ‘are far more’ is also correct. mind one may analyse the given sentence.
(C) ‘and delicious’ is correct usage. Correct. ‘Suppose’ and ‘if’ mean the same and
(A)
(D) Correct. ‘your restaurant’ is incorrect. ‘The the use of ‘if’ is unnecessary.
correct comparison should be ‘than those (B) ‘marooned’ is used correctly.
served in your restaurant’. (C) ‘what would’ is also correct.
(E) There is an error in D. (D) ‘do’ is correct as well.
The correct answer is D. (E) There is an error in A.

11. The aim is to find the phrase which is making the The correct answer is A.
given statement grammatically incorrect. The helping
verb should be plural and not singular. Keeping this in 15. The aim is to find the phrase which is making the
mind one may analyse the given sentence. given statement grammatically incorrect, There are
certain phrases that have a fixed usage. Keeping this
(A) Correct. ‘there is’ is incorrect. The correct verb in mind one may analyse the given sentence.
to refer to a large number of artefacts should be
the plural ‘are’. (A) ‘both dancing and singing’ is correct.

474

Book 1.indb 474 30/04/2019 4:49:09 PM


5.0   Language Skills Practice

(B) ‘refuses’ is correct. 19. The aim is to find the phrase which is making the
Correct. ‘Conform to’ should be used in place
(C) given statement grammatically incorrect, ‘Had’ is
of ‘conform by'. ‘Conform to’ is a fixed phrase used both as a helping verb and as a main verb.
which means comply with rules, standards, or Keeping this in mind one may analyse the given
laws. sentence.
(D) ‘wishes of’ is correct as well. (A) Correct. ‘If I had’ is incorrect. It should be ‘If I
(E) There is an error in C. had had’. The sentence is in the past perfect
tense and we use ‘had’ as both the helping verb
The correct answer is C. and the main verb.
16. The aim is to find the phrase which is making the (B) ‘have had’ is correct.
given statement grammatically incorrect, There are (C) ‘pick up’ is correct.
certain words whose usage together is redundant. (D) ‘from your’ is also fine.
Keeping this in mind one may analyse the given
(E) There is an error in A.
sentence.
The correct answer is A.
(A) ‘he heard’ is correct.
(B) ‘father’s accident’ is correct. 20. The aim is to find the phrase which is making the
(C) Correct. ‘Returned’ means to ‘come back’. The given statement grammatically incorrect, A singular
additional use of ‘back’ is unnecessary. subject takes a singular verb. Keeping this in mind
one may analyse the given sentence.
(D) ‘to his village’ is correct as well.
(E) There is an error in C. (A) ‘A major’ is correct.
(B) ‘Maratha sculptors’ is also correct.
The correct answer is C.
(C) Correct. ‘were the creation' is incorrect. ‘Was’
17. The aim is to find the phrase which is making the should be used in place of ‘were’.’Contribution’
given statement grammatically incorrect, There are is a singular subject that takes the singular verb
certain nouns that retain their construction in the ‘was’.
singular and plural forms. Keeping this in mind one (D) ‘the Buddha’s’ is correct.
may analyse the given sentence.
(E) There is an error in C.
(A) Correct. ‘Innings’ should be used in place of
The correct answer is C.
‘inning’. ‘Innings’ has an identical singular and
plural form. 21. The aim is to find the phrase which is making the
(B) ‘the match’ is correct. given statement grammatically incorrect, The phrase
(C) ‘more interesting’ is also correct. ‘one of the’ takes a plural noun and a singular verb.
Keeping this in mind one may analyse the given
(D) ‘second one’ is correct as well.
sentence.
(E) There is an error in A.
(A) ‘One of the’ is correct.
The correct answer is A.
(B) Correct. ‘commit the guru’ is the incorrect
18. The aim is to find the phrase which is making the given expression. ‘Commits’ should be used in place of
statement grammatically incorrect. The incorrect form commit.
of the adjective has been used here. Keeping this in (C) ‘as well as’ is correct.
mind one may analyse the given sentence. (D) ‘to celibacy’ is also apt.
(A) ‘Safety glass’ is correct. (E) There is an error in B
(B) ‘toughened’ is also fine. The correct answer is B.
(C) ‘times stronger’ is also correct.
(D) Correct. ‘untreating’ is incorrect. ‘Untreated’ 22. The aim is to find the phrase which is making the
should be used in its place. The ‘ing’ ending is given statement grammatically incorrect, ‘Whom’ is
not used with this adjective. used to refer to the object of a verb or preposition.
Keeping this in mind one may analyse the given
(E) There is an error in D.
sentence.
The correct answer is D.
(A) ‘One’ is correct.

475

Book 1.indb 475 30/04/2019 4:49:10 PM


NMAT by GMAC™ Official Guide 2019

(B) Correct. ‘Who’ is the incorrect pronoun. ‘Whom’ (D) ‘a while’ is fine.
should be used in its place.‘Whom’ is an (E) There is an error in C.
objective-case pronoun, which means that it
functions as an object in a sentence. The correct answer is C.
(C) ‘went’ is fine. 26. The aim is to find the phrase which is making the
(D) ‘last’ is also correct. given statement grammatically incorrect. The
(E) There is an error in B. placement of the adjective is incorrect. Keeping this
in mind one may analyse the given sentence.
The correct answer is B.
(A) Correct. ‘Retiring C.E.O’ is the incorrect phrase.
23. The aim is to find the phrase which is making the The correct construction should be ‘soon to
given statement grammatically incorrect. Most retire’ or ‘about to retire C.E.O’.
abstract nouns are uncountable, and we do not (B) ‘asked his’ is fine.
use articles when we talk about things in general.
(C) ‘interest’ is correct.
Keeping this in mind one may analyse the given
sentence. (D) ‘after his retirement’ is also apt.
(E) There is an error in A
(A) ‘If’ is correct.
(B) Correct. ‘an’ is incorrect. It should be ‘any The correct answer is A.
information’. Information’ is an abstract noun and
will not take an article. 27. The aim is to find the phrase which is making the
(C) ‘a’ is correct. given statement grammatically incorrect. We have
two independent clauses here. Keeping this in mind
(D) ‘to’ is fine.
one may analyse the given sentence.
(E) There is an error in B.
(A) ‘everybody’s’ is correct.
The correct answer is B. (B) ‘clinching’ is also correct.
24. The aim is to find the phrase which is making the (C) Correct. ‘reduced’ is grammatically incorrect.
given statement grammatically incorrect. The verb The word ‘reduced’ should be replaced by
form is incorrectly used. Keeping this in mind one ‘reducing’ to ensure parallel construction.
may analyse the given sentence. (D) ‘coveted’ is correctly used.
(A) Correct. ‘Estimating’ is wrongly used. ‘Estimated’ (E) There is an error in C.
should be used in its place which means roughly The correct answer is C.
calculated.
(B) ‘have been found’ is correct. 28. The aim is to find the phrase which is making the
(C) ‘reasonably’ is also correct. given statement grammatically incorrect. We have
two independent clauses here. Keeping this in mind
(D) ‘for use’ is fine.
one may analyse the given sentence.
(E) There is an error in A.
(A) ‘which plies’ is grammatically correct.
The correct answer is A (B) ‘to Hong Kong’ is correct.
25. The aim is to find the phrase which is making (C) ‘is the largest’ is also correct.
the given statement grammatically incorrect. An (D) Correct. ‘of’ is grammatically incorrect. ‘Of’
incorrect phrase has been used here. Keeping this in should be replaced by ‘in’ as the correct
mind one may analyse the given sentence. preposition to indicate location with respect to
the ‘world’ is ‘in’.
(A) ‘with fatigue’ is correctly used.
(E) There is an error in D.
(B) ‘begged permission’ is also correct.
(C) Correct. ‘Lay down ‘ is the incorrect expression. The correct answer is D.
‘Lie down’ should be used which means to move
into a position in which your body is flat, usually 29. The aim is to find the phrase which is making the
in order to sleep or rest. ‘Lay down’ means to given statement grammatically incorrect. We have
put or place in a horizontal position or position two independent clauses here. Keeping this in mind
of rest; set down. one may analyse the given sentence.

476

Book 1.indb 476 30/04/2019 4:49:10 PM


5.0   Language Skills Practice

(A) ‘these’ is correct. 33. The aim is to find the phrase which is making the
(B) ‘stuffed’ is grammatically correct. given statement grammatically incorrect. We have
two independent clauses here. Keeping this in mind
(C) Correct. ‘are’ is grammatically incorrect. ‘Either’
one may analyse the given sentence.
will take the singular verb, ‘is’.
(D) ‘little’ is correctly used. (A) ‘an’ is correct.
(E) There is an error in C. (B) ‘for’ is grammatically correct.
(C) ‘this’ is also correct.
The correct answer is C.
(D) Correct. ‘a’ is grammatically incorrect. M sounds
30. The aim is to find the phrase which is making the like a vowel, i.e., ‘em’. So, it should be preceded
given statement grammatically incorrect. We have by ‘an’.
two independent clauses here. Keeping this in mind (E) There is an error in D.
one may analyse the given sentence.
The correct answer is D.
(A) ‘his years’ is correct.
(B) ‘service to’ is grammatically correct. 34. The aim is to find the phrase which is making the
given statement grammatically incorrect. We have
(C) ‘and statesman’ is also correct.
two independent clauses here. Keeping this in mind
(D) Correct. ‘were honoured’ is grammatically one may analyse the given sentence.
incorrect. The sentence is talking of one person
who is both a politician and statesman and (A) ‘told me’ is correct.
hence, the phrase should have a singular verb (B) Correct. ‘it’s’ is grammatically incorrect. ‘It’s’ is
and should be ‘is honoured’. a contraction of ‘it is’ while ‘its’ is a possessive
(E) There is an error in D. determiner. In the given case, ‘its’ is the correct
word.
The correct answer is D.
(C) ‘spoke’ is grammatically correct.
31. The aim is to find the phrase which is making the (D) ‘I have’ is correctly used.
given statement grammatically incorrect. We have (E) There is an error in B.
two independent clauses here. Keeping this in mind
The correct answer is B.
one may analyse the given sentence.
(A) ‘A representative’ is correct. 35. The aim is to find the phrase which is making the
(B) ‘provide the students’ is correctly used. given statement grammatically incorrect. We have
two independent clauses here. Keeping this in mind
(C) ‘insight into’ is grammatically correct.
one may analyse the given sentence.
(D) ‘of the country’ is also correct.
(A) ‘an assignment’ is correct.
(E) Correct. There is no error in the given sentence.
(B) Correct. ‘read quickly over’ is grammatically
The correct answer is E. incorrect. The phrasal verb is ‘read over’ which
means to read something thoroughly and
32. The aim is to find the phrase which is making the carefully. The adverb ‘quickly’ should come
given statement grammatically incorrect. We have before the phrasal verb it is modifying.
two independent clauses here. Keeping this in mind
(C) ‘missed’ is grammatically correct.
one may analyse the given sentence.
(D) ‘and’ is also correct.
(A) Correct. ‘has’ is grammatically incorrect. The
(E) There is an error in B.
subject ‘plums’ is plural and should have a plural
verb too, that is, ‘have’. The correct answer is B.
(B) ‘greyish’ is correct.
(C) ‘whitish’ is also correct.
(D) ‘wipe off’ is grammatically correct.
(E) There is an error in A.
The correct answer is A.

477

Book 1.indb 477 30/04/2019 4:49:10 PM


NMAT by GMAC™ Official Guide 2019

7 Choose the Correct 3. ‘Made from’ is used when the condition of the
material changes.
Preposition The correct answer is B.

1. Option C is the right answer as ‘ebd’ is the correct 5. Option B is the right answer as ‘bac’ is the correct
sequence of prepositions to be used in the given set sequence of prepositions to be used in the given set
of sentences. of sentences.
As we can see: As we can see:
1. Krishna said that he would arrive between 2 and 1. Abstain from food or indulgence (fixed phrase)
3 pm. meaning ‘refrain from’.
2. In the newly constructed shopping mall the shops 2. Exception to (fixed phrase).
stay open from 11 am to 11 pm. 3. Accord with (fixed phrase) meaning ‘to agree
3. The formalities are all over and the rocket is with someone or something’.
ready for launch in 30 minutes.
The correct answer is B.
The correct answer is C.
6. Option C is the right answer as ‘bac’ is the correct
2. Option A is the right answer as ‘acf’ is the correct sequence of prepositions to be used in the given set
sequence of prepositions to be used in the given set of sentences.
of sentences. As we can see:
As we can see: 1. We use ‘on’ for entering a public transport
1. The administration’s new proposals have been vehicle.
met with a lot of opposition from the citizens of 2. We use ‘by’ for left or right of somebody or
the town. something.
2. The children are so excited to eat out that they 3. We use ‘different from’ to mean something that
are unable to choose between a pizza and a is not the same as another.
burger.
The correct answer is C.
3. When asked to come to the party, Ramesh
declined saying that he had decided to stay at 7. Option C is the right answer as ‘dab’ is the correct
home and study instead. sequence of prepositions to be used in the given set
The correct answer is A. of sentences.
As we can see:
3. Option C is the right answer as ‘adc’ is the correct
sequence of prepositions to be used in the given set 1. We use ‘knock over’ to indicate pushing
of sentences. someone or something that causes the person
or thing to fall.
As we can see:
2. We use ‘on foot’ to indicate walking.
1. ‘With’ is used with countable nouns for
3. We use ‘with an accent’ to indicate ‘having’.
instruments.
2. ‘In’ is used with uncountable nouns for The correct answer is C.
instruments.
8. Option D is the right answer as ‘bae’ is the correct
3. ‘Over’ is used in the context of meal/tea/lunch
sequence of prepositions to be used in the given set
and so on.
of sentences.
The correct answer is C. As we can see:
4. Option B is the right answer as ‘ade’ is the correct 1. We use ‘out of’ to mean ‘without’.
sequence of prepositions to be used in the given set 2. We use 'above' as a preposition when we want
of sentences. to indicate 'higher than' a figure or a benchmark.
As we can see: 3. ‘Care for’ means to feel affection for someone.
1. ‘Beside’ means by the side of. The correct answer is D.
2. ‘Amongst’ is used for vowel starting nouns.

478

Book 1.indb 478 30/04/2019 4:49:10 PM


5.0   Language Skills Practice

9. Option E is the right answer as ‘bcf’ is the correct 13. Option D is the right answer as ‘dca’ is the correct
sequence of prepositions to be used in the given set sequence of prepositions to be used in the given set
of sentences. of sentences.
As we can see: As we can see:
1. ‘Entrust with’ means ‘to assign the responsibility 1. ‘Persuade of’ means to make someone do or
of’. believe something by giving them a good reason
2. ‘Convict of’ means ‘to declare (someone) to be to do it or by talking to that person and making
guilty of a criminal offence by the verdict of a them believe it.
jury’. 2. ‘Under’ means according to an agreement, a law
3. ‘Persist in’ means ‘to do something repeatedly or a system.
and show no intention of stopping’. 3. We use ‘over’ to show an apparent change in
one's mood, attitude and so on.
The correct answer is E.
The correct answer is D.
10. Option B is the right answer as ‘ebd’ is the correct
sequence of prepositions to be used in the given set 14. Option E is the right answer as ‘fcb’ is the correct
of sentences. sequence of prepositions to be used in the given set
of sentences.
As we can see:
As we can see:
1. ‘Comply with’ means ‘to act in accordance with a
wish or command’. 1. ‘On’ is used to show the possession of; being
carried by.
2. ‘Want of’ means ‘lack of’.
2. We use ‘with’ to indicate a result attendant on a
3. ‘Insight into’ means the power or act of seeing
specified action.
into a situation.
3. If you are aboard a ship or plane, you are on it or
The correct answer is B. in it.
11. Option D is the right answer as ‘def’ is the correct The correct answer is E.
sequence of prepositions to be used in the given set
of sentences. 15. Option B is the right answer as ‘aeb’ is the correct
As we can see: sequence of prepositions to be used in the given set
of sentences.
1. ‘Triumph over’ means ‘to achieve victory over
someone or something’. As we can see:
2. ‘Side with’ means ‘to be on someone’s side’. 1. 'On' is used to indicate where someone or
something is hit or touched.
3. ‘Slur on’ means ‘an insinuation or allegation
about someone that is likely to insult them or 2. ‘Between’ is used to indicate two or more people
damage their reputation. or things that together produce a result or have
an effect.
The correct answer is D. 3. ‘Into’ is used to describe the state, condition, or
form of (something).
12. Option C is the right answer as ‘dae’ is the correct
sequence of prepositions to be used in the given set The correct answer is B.
of sentences.
16. Option C is the right answer as ‘bae’ is the correct
As we can see: sequence of prepositions to be used in the given set
1. ‘Over the weekend’ means the same as during of sentences.
the weekend. It is used to talk about something As we can see:
that happened (or will happen) between Friday 1. 'In' is used to indicate unspecific times during a
evening and Sunday evening. day, season, year and so on.
2. ‘On’ is used with the plural word-weekends. 2. ’To’ is used to introduce the second part of a
3. ‘In’ is used as a function word to indicate the comparison or ratio.
larger member of a ratio. 3. ‘Against’--- not to the advantage or favour of
somebody/something.
The correct answer is C.
The correct answer is C.

479

Book 1.indb 479 30/04/2019 4:49:10 PM


NMAT by GMAC™ Official Guide 2019

17. Option D is the right answer as ‘cad’ is the correct As we can see:
sequence of prepositions to be used in the given set 1. ‘Array’ takes the preposition ‘of’ meaning an
of sentences. impressive display or range of a particular type
As we can see: of thing.
1. We use ‘over’ to refer to extended periods of 2. ‘Focus on’ is a fixed phrase which means to give
time. most of your attention to someone or something
2. ‘Intended’ takes the preposition ‘to’ which means 3. ‘Push or sweep under the carpet’ is a fixed
planned or meant. phrase which means to ignore, deny, or conceal
3. ‘Extracted from’ means removed or taken out. from public view or knowledge something that
is embarrassing, unappealing, or damaging to
The correct answer is D.
one’s reputation
18. Option D is the right answer as ‘beb’ is the correct
The correct answer is C.
sequence of prepositions to be used in the given set
of sentences. 22. Option A is the right answer as ‘ace’ is the correct
As we can see: sequence of prepositions to be used in the given set
1. ‘See through’ means not be deceived by or see of sentences.
clearly from one end to another end. As we can see:
2. ‘Stand out’ means to be easily seen or noticed. 1. ‘Linger on’ means to remain at something and
3. Run through’ means to run out of something not move on.
before the stipulated time, to exhaust a resource
2. ‘Look to someone’ means expect someone to
earlier than anticipated or planned for.
provide something or do something.
The correct answer is D.
3. ‘Shiver with or in fear’ is a fixed phrase. It means
19. Option C is the right answer as ‘dfa’ is the correct to shake, quiver, or tremble, from fear.
sequence of prepositions to be used in the given set
The correct answer is A.
of sentences.
As we can see: 23. Option E is the right answer as ‘cae’ is the correct
1. ‘Leave in peace’ means to avoid disturbing or sequence of prepositions to be used in the given set
bothering someone. of sentences.
2. ‘Frown on’ means to disapprove of something or As we can see:
someone.
1. ‘At the hospital’ is a standard expression. We use
3. ‘Disapprove of’ means to have or express an ‘at’ for a point.
unfavourable opinion.
2. ‘In the hospital’ is a standard expression. We use
The correct answer is C. ‘in’ for an enclosed space.
20. Option C is the right answer as ‘fba’ is the correct 3. ‘Mad about something’ means to be crazy about
sequence of prepositions to be used in the given set someone or something.
of sentences.
The correct answer is E.
As we can see:
1. ‘Difference between’ is a fixed phrase used for 24. Option C is the right answer as ‘dba’ is the correct
contrast or dissimilarity. sequence of prepositions to be used in the given set
of sentences.
2. ‘Differ from’ is a fixed phrase which means to be
unlike, or dissimilar. As we can see:
3. ‘Designed for’ is a fixed phrase meaning do or 1. ‘under’ refers to a condition of subjection,
plan (something) with a specific purpose in mind. making it the correct word for sentence.
2. You selected ‘for’ a seat.
The correct answer is C.
3. You serve food ‘to’ someone. ‘To’ is used as
21. Option C is the right answer as ‘bde’ is the correct a function word to indicate action towards a
sequence of prepositions to be used in the given set person, place or thing
of sentences. The correct answer is C.

480

Book 1.indb 480 30/04/2019 4:49:11 PM


5.0   Language Skills Practice

25. Option B is the right answer as ‘cad’ is the correct 28. Option D is the right answer as ‘dae’ is the correct
sequence of prepositions to be used in the given set sequence of prepositions to be used in the given set
of sentences. of sentences.
As we can see: As we can see:
1. ‘from’ is used to as a function word to indicate 1. To ‘pull off’ is a phrasal verb which refers to
the starting or focal point of an activity. succeeding in doing something that is difficult.
2. ‘as’ is used to describe the purpose or quality of 2. ‘fall out’ is also a phrasal verb which means to
something or someone. stop being friendly with someone because you
3. ‘after’ refers to following in time, place, or order. have had a disagreement with them.
3. To ‘put aside’ is to not allow yourself to be
The correct answer is B.
affected by a problem, argument etc. so that
you can achieve something more important.
26. Option C is the right answer as ‘bda’ is the correct
sequence of prepositions to be used in the given set The correct answer is D.
of sentences.
29. Option B is the right answer as ‘bdb’ is the correct
As we can see:
sequence of prepositions to be used in the given set
1. The emissions result in widespread pollution. of sentences.
The correct phrasal verb to be used in sentence
1 is ‘led to’. As we can see:
2. ‘From’ refers to a point from where an action or 1. ‘for’ is used to mean towards or in the direction of.
journey starts. So, the court order was procured 2. ‘with’ means using something.
‘from’ the court. 3. ‘for’ refers to duration.
3. ‘by’ is used to refer to the instrumentality of
The correct answer is B.
some activists or through the agency of some
activists.
30. Option D is the right answer as ‘cbe’ is the correct
The correct answer is C. sequence of prepositions to be used in the given set
of sentences.
27. Option B is the right answer as ‘adb’ is the correct
As we can see:
sequence of prepositions to be used in the given set
of sentences. 1. ‘in’ refers to inside an area.
2. ‘at’ is used to show an exact position or
As we can see:
particular place.
1. ‘of’ is used to show belonging.
3. ‘to’ is used after some verbs, especially when
2. To ‘come under something’ is to experience a the action described in the infinitive will happen
bad situation later.
3. The only preposition that is used after the word
The correct answer is D.
‘subsequent’ is ‘to’.
The correct answer is B.

481

Book 1.indb 481 30/04/2019 4:49:11 PM


NMAT by GMAC™ Official Guide 2019

8 Reading Comprehension (B) It does not include ‘books by some male
authors’.
(C) The passage clearly mentions option iii in
Passage 1 the first paragraph. However, the author also
asserts that women themselves have been
Topic—Women being acknowledged only for their swayed by men’s focus on their beauty, and
appearance. have started focusing more on looking attractive
than on fulfilling a more noble purpose. Thus,
Scope—How women have unwittingly contributed to men’s
statements ii and iii are correct.
focus on their appearance to the neglect of other facets of
their personality. (D) Contains statement (i)
(E) Contains statement (i)
Passage Map
The correct answer is C.
P1—Introduces the author’s dislike about the fact that men
are obsessed only with women’s looks and women fall prey 3. (A) Large refers to ‘anything big in size’.
to this empty flattery.
(B) Deceptive means ‘anything that gives an untrue
P2—Accepts that there are some aspects in which men or wrong impression'. It can stand in place of
are superior but that does not warrant their treating women specious, as it is a synonym of misleading or
merely as objects of allure. false. Therefore, Deceptive fits this context the
best.
1. (A) Option A is not within the scope of the passage
(C) Accommodating means ‘adjusting’ or ‘making
and can be eliminated.
space for’.
(B) Option B is immediately struck out because
(D) Unconvincing means ‘lacking in an ability to win
the author is not trying to reconcile or propose
the belief of a viewer/listener’.
something.
(E) Malicious means harmful.
(C) Option C is mentioned in the passage but it is
not the main purpose of the entire passage. The correct answer is B.
(D) Option D summarises this passage the best
and should be the correct answer. The primary 4. (A) It does not include statement (ii)
purpose question can only be answered (B) Statement iii is not mentioned or implied
correctly if you have understood the passage anywhere in the passage.
properly. In the passage, the author is making
the point that women have been conditioned (C) It includes statement (iii) which in not implied
by society to focus only on looking attractive. anywhere in the passage.
However, the author argues that women should (D) The author clearly agrees in the second
cherish much nobler ambitions than just looking paragraph that men are physically superior
attractive. to women. Thus, statement (i) is correct. The
(E) Option E is again struck out because the author author also mentions towards the end of the first
is not trying to reconcile or propose something. paragraph that women get intoxicated by the
attention they get from men. Thus, statement (ii)
The correct answer is D. is also correct.
(E) It includes statement (iii)
2. (A) Parents of women are not mentioned in the
passage. The correct answer is D.

482

Book 1.indb 482 30/04/2019 4:49:11 PM


5.0   Language Skills Practice

Passage 2 2. (A) Such a statement is not mentioned in the


passage nor can it be inferred from it.
Topic – Mother’s cooking.
(B) Such a statement is not mentioned in the
Scope – What was so special about mother’s cooking. passage nor can it be inferred from it.
(C) Such a statement is not mentioned in the
Passage Map passage nor can it be inferred from it.
(D) It can be inferred from the line: “..connected to
P1 – Asks the question – what made mother’s dishes so other pleasant memories of our younger selves”.
special.
(E) Cannot be inferred.
P2 – Describes features of mother’s garden. The correct answer is D.
P3 – Describes the herbs that played a major role in
3. (A) This is not the ‘unique’ feature that has been
mother’s cooking.
mentioned.
1. (A) We cannot say that the herbs are playing a (B) “Use” of herbs is not a feature of the garden.
supporting role as the author mentions
(C) Cannot be inferred.
other members of the cast saying,“these
members play what appear to be minor (D) It doesn’t speak of the ‘garden’.
roles but leave a lasting impact in the (E) This can be inferred from the line - there
minds of the audiences”. was nothing remarkable about it, at least
at first glance. What did stand out was that
(B) There is no discussion on differences between
everything—flowers, fruits and vegetables—was
herbs and fruits/vegetables.
neatly laid out and pleasing to the eye.
(C) ‘Herbs in cooking’ is the subject of discussion as
The correct answer is E.
opposed to ‘herbs and cooking’.
(D) Can be inferred from the passage which says 4. (A) The passage refers to the herbs as ‘sweet
that it may appear that herbs are playing a herbs’ in one sentence, but this does not imply
supporting role in a dish but they are actually that the herbs actually tasted sweet. The use of
playing a vital role because it is the flavour of inverted commas around this phrase conveys
these herbs that will be remembered long after how important these herbs were in mother’s
the other ‘supposedly’ major ingredients have cooking.
been forgotten.
(B) This is mentioned in the last paragraph.
(E) This could be correct, based on the information (C) This is mentioned in the last paragraph.
in the passage but it does not address
this comparison and so it can be (D) This is mentioned in the last paragraph.
eliminated. (E) This is mentioned in the last paragraph.

The correct answer is D. The correct answer is A.

483

Book 1.indb 483 30/04/2019 4:49:11 PM


NMAT by GMAC™ Official Guide 2019

Passage 3 (B) The second paragraph starts by stating that


a rational and a highly impulsive person may
Topic—What triggers human emotions? perceive the same event differently. Thus, B
should be the answer.
Scope—To explore the relationship between impulses and
release of certain emotions. (C) Incorrect because, as the first paragraph clearly
states, a person cannot consciously control the
release or dissipation of his hormones.
Passage Map
(D) Is a distortion of facts in the first paragraph.
P1—–states that certain emotions are accompanied by (E) Is the opposite of what the passage states.
the release of hormones and these emotions cannot be
The correct answer is B.
controlled by humans.

P2—explains how the understanding of impulse triggers 3. (A) Mislead means ‘to urge/compel someone in the
and impulse control can help curb sudden emotional wrong direction’.
outbursts. (B) Misdemeanor means ‘misconduct’.
1. (A) Statement iii is factually incorrect. (C) Misguide means ‘to provide wrong/faulty
assistance to’.
(B) The passage states that the reason a person is
(D) Misinterpret means ‘misunderstand’, which is
not able to voluntarily control emotions is that
closest in meaning to ‘misconstrue’ as used
these emotions are sustained by hormones
here.
whose rate of dissipation cannot be controlled
by humans. Thus, if no such hormones were (E) Misrepresent means ‘to portray (any subject)
released, there would be no problem in the first incorrectly’.
place. So, statement i is correct. Statement ii is The correct answer is D.
also correct because by consuming this drug,
a person can increase the rate of dissipation 4. (A) Cannot be inferred.
of hormones, something he had no control
upon earlier. Statement iii is incorrect because (B) Because the word ‘unfortunately’ provides a
it specifically talks about adrenaline. Even if transition from the previous sentence, which
adrenaline is not produced, some other hormone talks about better emotional control, the next
could be produced upon whose dissipation rate sentence mentions that, unfortunately, in several
a person would have no control. cases a human cannot have conscious control
(C) Statement iii is factually incorrect. over his emotions. Thus, B is the best answer.
(D) Does not contain statement ii. (C) The definition of ‘desired state of affairs’ cannot
be inferred from the information given within the
(E) It contains statement iii. passage.
The correct answer is B. (D) No such ‘efforts’ made by researchers have
been mentioned.
2. (A) Incorrect because, as the first paragraph clearly (E) Such a connection is not what follows the said
states, a person cannot consciously control the statement.
release or dissipation of his hormones.
The correct answer is B.

484

Book 1.indb 484 30/04/2019 4:49:12 PM


5.0   Language Skills Practice

Passage 4
Topic—Grants by charitable foundations. 2. (A) It is mentioned in the last paragraph.
Scope—How charitable foundations keep a check on the (B) It is mentioned in the 3rd paragraph.
grants they make to beneficiaries. (C) It is mentioned in the 2nd paragraph.
(D) The passage never states that charitable
Passage Map organisations act as mentors to their
beneficiaries. They can act as or appoint
P1—introduces the three stages in which charitable monitors, but mentors is not mentioned
organisations keep a check on the use of their grants by anywhere in the passage.
beneficiaries. (E) It is mentioned in the 3rd paragraph.

P2—explains the first stage and how the charitable The correct answer is D.
organisation exercises control at this stage.
3. (A) Doesn’t contain statement i.
P3—explains the second stage and how the charitable (B) Contains statement iii.
organisation exercises control at this stage.
(C) Doesn’t contain statement i.
P4—explains the third stage and how the charitable (D) Contains statement iii.
organisation exercises control at this stage. (E) The question is asking you which of these
1. (A) The passage is not concerned with why it entities will not be funded by a charitable
is imperative or important for charitable organisations. While statements i and ii are
organisations to act in this manner. Rather, clearly implied in the passage, statement iii
the passage focuses on how charitable is incorrect because whatever instructions or
organisations go about imposing this control. requirements a charitable organisation has need
to be mentioned in the agreement up front.
(B) There is no such ‘argument’ in the passage. The beneficiary is not obliged to follow any
(C) This does not cover misuse of funds. instructions that the charitable organisations
may come up with later on.
(D) The passage is clearly concerned with
highlighting three different ways in which The correct answer is E.
charitable foundations try to prevent or control
the misuse of their funds by the beneficiaries 4. (A) Secrecy means ‘ furtiveness’.
of these funds. Thus, this is the correct (B) Responsibility means ‘ownership’.
answer.
(C) Authority is closest in meaning to ‘discretion’ or
(E) This does not cover ‘control of funds’. ‘choice’.
The correct answer is D. (D) Credibility means ‘trustworthiness’.
(E) Gullibility means ‘pliability’.
The correct answer is C.

485

Book 1.indb 485 30/04/2019 4:49:12 PM


NMAT by GMAC™ Official Guide 2019

Passage 5 2. While all the options contain elements mentioned in


the passage, most of them distort these elements.
Topic—The Guinness Book of World Records. (B) states what is stated in the passage verbatim and
is the correct answer.
Scope—The beginning of record taking and how the nature
of records has changed over the years with reference to the (A) The record is for the world’s most powerful (and
Guinness Book. not the largest) combustion system.
(B) Mentioned in the 3rd paragraph.
Passage Map (C) Incomplete. The record is for the world’s
heaviest train to be pulled with a beard.
P1—To provide an introduction to the Guinness Book. (D) The passage never states that this record is
actually there in the Guinness Book; it only
P2—To discuss the approach of the initial record collectors
states that the likelihood of this fact being in the
(with reference to the Guinness Book).
book is high.
P3—To discuss the Guinness Book in its current form and (E) Distortion. The record is for the world’s heaviest
how the nature of records contained in it has changed from train to be pulled by a beard and not for the
its earlier forms. world’s strongest beard.

P4—To provide a conclusion by stating a possible useful The correct answer is B.


role of the Guinness Book in the future.
3. (A) While this is true, the difference has to be in the
1. The author clearly has a neutral-to-positive tone in content and not in the presentation.
the passage, so all the options that contain negative (B) The author clearly states in the second
terms, such as ‘criticise’, can be eliminated. The paragraph that, in the current version of the
passage starts by discussing the origins of the book, entertainment-oriented records share
Guinness Book and goes on to discuss the changes space with education-oriented ones. (B) states
that have come about in the book as it has evolved this best and is the correct answer.
over the years. Note that the last paragraph ends
(C) While this is also true, this is not necessarily a
on a positive note, further reinforcing the decision to
point of difference between the two versions of
eliminate all options with negative connotations.
the book, as this was probably true for the older
(A) Conveys the main idea of the passage best and version as well.
is the correct answer. (D) The author never states that the focus of the
(B) The author never criticises anything in the book has shifted; he merely states that the book
passage. now also contains entertainment-oriented facts.
(C) The author does much more than just explain the (E) Same as (C).
origin of the book.
The correct answer is B.
(D) While the passage does mention this fact in
passing, this is not why the author wrote the
entire passage.
(E) The first part of this option about the origin
of the book is correct, but the second part
which states that the author advocates against
something is obviously incorrect.
The correct answer is A.

486

Book 1.indb 486 30/04/2019 4:49:12 PM


5.0   Language Skills Practice

Passage 6 3. (A) Statement i is clearly supported by information


in the 3rd paragraph of the passage. Option ii
Topic—Realpolitik is incorrect because, as Paragraph 4 states,
Machiavelli pays little importance to the
Scope—To explore Machiavelli’s concept of politics without presence of good laws. In the last sentence of
principles. the passage, Machiavelli states that the Prince
would not win glory because of the countless
Passage Map atrocities he had committed. Thus, iii is incorrect
as well.
P1—Introduces the term ‘realpolitik’—politics without
(B) Doesn’t contain statement i.
principles.
(C) Contains statement iii.
P2—Provides an example of how Machiavellian principles (D) Doesn’t contain statement i.
have been practiced from even before his time.
(E) Contains statement ii.
P3—Provides Machiavelli’s description of a strong person. The correct answer is A.
P4—States that Machiavellian wasn’t particularly bothered
4. (A) Sorolla believed in maintaining the absolute
about good governance.
supremacy of the king whereas Machiavelli
P5—Provides Machiavelli’s distinction between power and believed that wars needed to be won at any
glory. cost. This option can be eliminated.
(B) Sorolla believed in maintaining the absolute
1. (A) Not stated anywhere in the passage.
supremacy of the king whereas Machiavelli
(B) Not stated anywhere in the passage. believed that wars needed to be won at any
(C) Is true of realpolitik and not of Classical theory. cost. This option can be eliminated.
(D) There’s nothing in the passage to suggest that (C) Sorolla believed in maintaining the absolute
the Classical theory was eventually abandoned. supremacy of the king whereas Machiavelli
(E) The opening sentence of the passage states believed that wars needed to be won at any
that realpolitik was more shrewd and pragmatic cost. This option can be eliminated.
(practical) than the Classical political theory.
(D) Is the correct answer because both Sorella and
Thus, it can be concluded that at least some
Machiavelli believed in going to any lengths to
of the tenets of the Classical theory were not
maintain the supremacy of the king and win wars
rooted in practicality.
respectively.
The correct answer is E.
(E) It is not mentioned or implied anywhere in the
passage.
2. (A) Incorrect inference.
(B) Incorrect inference. The correct answer is D.
(C) Since the passage mentions Chapter XII to
highlight the fact that it took Machiavelli XI
chapters to talk about arms and army before he
mentioned good laws for the first time, it shows
how unimportant good laws were in Machiavelli’s
scheme of things.
(D) Opposite of what the passage says.
(E) Incorrect inference.
The correct answer is C.

487

Book 1.indb 487 30/04/2019 4:49:12 PM


NMAT by GMAC™ Official Guide 2019

Passage 7 2. (A) While the author does mention this, this is not
the main purpose of the entire passage.
Topic—Close synonyms (B) Again, the author mentions this in the last
paragraph but he did not dedicate the entire
Scope—The use of close synonyms interchangeably and the
passage to discussing this fact.
subtle differences in mood or feeling conveyed by them.
(C) The author does not prove anything in the
passage.
Passage Map
(D) The author’s underlying theme in the passage is
P1—To state that it is important for some people to that even if two words have the same meaning,
understand the subtle differences of meaning between close the feeling or mood conveyed by them could
synonyms. be different. He concludes the passage by
stating that users of English language play a
P2—To explain that there is a difference of mood conveyed very important role in sustaining this ‘feeling’ or
by close synonyms which cannot be explained by a ‘mood’ attached to a word. (D) captures this best
dictionary. and is the correct answer.

P3—To state that it is up to the users of English language (E) The author never criticises anyone in the
to ensure that the subtle differences of mood conveyed by passage.
close synonyms are passed down through generations. The correct answer is D.

1. (A) The The author clearly states in the second 3. (A) This is mentioned at the end of the second
paragraph that there is no point in having paragraph.
multiple words with the same meaning. He goes
(B) This is also mentioned at the end of the second
on to conclude later in the passage that even if
paragraph (etymology=origin of words).
two words have the same meaning, they convey
different feelings. Thus, he would most likely (C) This is also mentioned in the passage.
agree with (A) that two words can never be
(D) This is the main function of a dictionary.
complete synonyms; there will always be some
small difference between the two. (E) In the third paragraph, the author states that
(B) The author never states that the dictionary does the dictionary is of little use in deciding, out of
not contain all the meanings of a word, but that two close synonyms, which one conveys the
it cannot convey the feelings attached to a word, feeling and mood effectively. Thus (E) is the best
which are more subjective. answer.
(C) Opposite. The author, in fact, states that the The correct answer is E.
English language is very economical.
(D) Extreme option. There may be two words that
can be used interchangeably, even if they convey
different feelings.
(E) The author states at the end of the first
paragraph that he belongs to the opposite group
for whom such differences have significant
value.
The correct answer is A.

488

Book 1.indb 488 30/04/2019 4:49:12 PM


5.0   Language Skills Practice

Passage 8 2. (A) While common sense dictates that this may very
well be the case, there is nothing in the passage
Topic—Fingerprint records. to suggest this.

Scope—The criminal and civil uses of fingerprint records (B) The second paragraph states that this is not the
and how fingerprint impressions can be taken. case and that, in fact, the opposite may be true.
(C) Extreme option. We know that this is a very
important way, but we do not know whether this
Passage Map
is the most foolproof way. There could always be
P1—To state the importance/uses of maintaining fingerprint a better way.
records. (D) The first paragraph clearly states that the use of
fingerprint records results in the imposition of more
P2—To describe the civil uses of fingerprint records which equitable sentences by the judiciary. (D) states this
are often overlooked in favour of the criminal uses. almost verbatim and is the correct answer.
P3—To describe the actual mechanics of how fingerprints (E) According to the latter half of the third
are formed and how fingerprint impressions can be taken. paragraph, the opposite may actually be true.
The correct answer is D.
1. (A) This is the purpose of the entire paragraph and
not just of this sentence.
3. (A) The author is explaining a process which is
(B) Same as A. already in place; so, he is not explaining or
(C) This is the literal meaning of the sentence, but justifying the ‘need’ as such.
we need to answer why the author is stating this
(B) The author never advocates anything in the
fact.
passage.
(D) The question is asking you for the role of the
(C) This is too narrow in scope. (E) states the main
sentence, that is, why did the author put this
purpose much better.
sentence in this paragraph? Since this is a
function question, you need to answer for the (D) While the ‘advocates’ are mentioned in the
‘why’ and not the ‘what’. Note that the sentence passage, the main purpose of the passage is
starts with the term ‘however’, which immediately much more than to just state this fact.
implies that it is trying to show a contrast with (E) The passage starts by discussing the importance
the previous statement. The previous statement of fingerprints in criminal investigation, and
states that the lay mind associates fingerprints then goes on to explain their importance in civil
with being useful primarily in criminal investigation. cases as well. It finally concludes by discussing
The second sentence then creates the contrast the various modes of fingerprint collection. (E)
by stating that this is not the case and that, in summarises all of this best and is the correct
fact, there are more fingerprints in the Civil File of answer.
the FBI than in the Criminal File; thereby implying
The correct answer is E.
that fingerprints probably have a more important
or an equally important role outside of criminal
investigation as well. Thus, the function of this 4. (A) Inferred from the third paragraph.
sentence is to create this contrast between the (B) Inferred from the third paragraph.
two roles of fingerprints. (D) brings this out best (C) Inferred from the third paragraph.
and is the correct answer.
(D) Towards the end of the third paragraph, wax is
(E) Same as A. mentioned as one of the probable mediums or
The correct answer is D. backgrounds on which one can take fingerprints
and not the substance that actually needs to be
Note that in such questions the wrong answer choices will applied on the fingers. Hence, (D) is the correct
typically provide you the answer for the entire paragraph answer.
and not for the specific sentence or they will paraphrase (E) Inferred from the third paragraph by the use of
the sentence in question, that is, answer ‘what’ rather ‘saline substance emitted from the glands’.
than ‘why’.
The correct answer is D.

489

Book 1.indb 489 30/04/2019 4:49:13 PM


NMAT by GMAC™ Official Guide 2019

Passage 9 3. (A) Incorrect option.

Topic—Robotic vision (B) Correct. The second paragraph mentions


that range finders are expensive and fiddly as
Scope—How self-guided robots perceive their surroundings opposed to digital cameras which are small and
and a possible innovation with regards to the same. cheap, because of which Dr. Davison prefers the
latter.
(C) Incorrect option.
Passage Map
(D) Incorrect option.
P1—To introduce two methods by which a robot can (E) Incorrect option.
understand its environment—Stereoscopic vision and
SLAM—and state that SLAM is the preferred method these The correct answer is B.
days.
4. What is the main purpose of the author in writing the
P2 and P3—To discuss problems with using range finders passage?
and how range finders can be replaced by digital cameras.
(A) This is just a part of the passage, but the
P4—To conclude that it will not be easy to replace range passage is doing much more than just this.
finders with video cameras, given the complex nature of (B) It is Dr. Davison who is advocating this fact; the
computations that need to be done. author of the passage is neutral in his tone.
(C) The passage is only about robots and not
1. (A) Incorrect option.
computers in general.
(B) Incorrect option.
(D) The author does not praise anyone in the
(C) Correct. The first paragraph explains passage.
stereoscopic vision as integration of images
(E) The whole passage talks about computer
from two ‘eyes’.
programmes that will help in the development
(D) Incorrect option. of self-guided robots. Option (E) provides an apt
(E) Incorrect option. summary that lucidly indicates the essence of
the passage.
The correct answer is C.
The correct answer is E.
2. 
(A) This is stated in the first line of the second
paragraph.
(B) The opening lines of the passage talk about Dr.
Davison, who thinks that one eye is enough for a
robot. These robots have so far been using range
finders to see with one eye, and now Dr. Davison
wants to replace these rangefinders with digital
cameras. So, rangefinders do allow robots to see
with one eye.
(C) This can be implied from the first line of the passage.
(D) This can be inferred from these lines in the first
paragraph: For a robot to work autonomously, it has
to understand its environment.
(E) The second paragraph states that, for digital cameras
to replace range finders, it is imperative that digital
cameras be able to make 3D maps. But this does not
necessarily imply that range finders were also making
3D maps; it is possible that range finders were making
use of something entirely different. Hence, (E) should
be the answer.
The correct answer is E.

490

Book 1.indb 490 30/04/2019 4:49:13 PM


5.0   Language Skills Practice

Passage 10 2. (A) The author has discussed cyberspace with


reference to its impact and growth; its importance
Topic—Cyberspace with reference to anything else is not mentioned in
this paragraph, making this option out of scope.
Scope—What is cyberspace and how it has changed in
recent times? (B) No such threat is mentioned anywhere in the
paragraph.
(C) This is actually mentioned in the second
Passage Map
paragraph and not in the third.
P1—To describe what is cyberspace. (D) The opening sentence of the third paragraph
states that cyberspace has changed rapidly in
P2—To describe how different people have used the past 20 years and the rest of the paragraph
cyberspace for different ends. goes on to describe these changes.
P3—To describe the revolution that has taken place in (E) While this is mentioned in the last sentence
cyberspace in recent times. of the paragraph, the main purpose of the
paragraph is more than just to state this.
1. (A) The opening lines of the passage clearly state The correct answer is D.
that the term ‘cyberspace’ was coined in 1982;
so, it cannot possibly have been in use for over 3. (A) Since this is not mentioned in the passage, we
a century. have no way of figuring out whether the author
believes this is true or not.
(B) Clearly stated in the second paragraph.
(B) Note the wording of this question—which of the
(C) The third paragraph states that there is still following is not true—which is different from
no substance to cyberspace. So, even though an except question. So, you basically need to
the author believes that the cyberspace is a identify an option that states the opposite of what
separate place, it still does not have a physical the author has stated in the passage. (B) does
form or location. this best and is the correct answer, because the
(D) The second paragraph states that people have author clearly states in the third paragraph that the
regulated cyberspace. cyberspace is its own separate place.
(C) Can be inferred from the second paragraph.
(E) Stated in the last sentence of the second
paragraph. (D) Can be inferred from the third paragraph.
(E) Can be inferred from the last sentence of the
The correct answer is A. first paragraph, and also from the end of the
third paragraph.
The correct answer is B.

491

Book 1.indb 491 30/04/2019 4:49:13 PM


NMAT by GMAC™ Official Guide 2019

Passage 11 (D) In the last paragraph, the author reiterates


the fundamental unity of the country. It is also
Topic—The Indian culture and its connection with inferred by his use of the term ‘after a passivity
philosophy/spirituality of some centuries’—he concedes that spiritual
India encountered a setback due to distractions
Scope—Philosophy/spirituality, and not invasions and of war— however, the underlying unity will
foreign rules, best describe Indian culture resurface. Hence, (D) is the best option.
(E) Not mentioned—political triumph has only been
Passage Map mentioned to illustrate political instability.

P1—To state that philosophy was the biggest achievement The correct answer is D.
and also contribution, of the Indian civilisation.
3. (A) The author never compares Indian philosophy
P2—To state that the history of India can be understood with anything.
best in the spiritual context.
(B) The author has, in the passage, discussed
P3—To further build on P2 by stating that, as it has done in the philosophy of India. He has analysed the
the past, spirituality will define Indian culture and lead to its underlying factors responsible for its incredible
growth in the future as well. sustainability through the ages, across political
upheaval and geographical boundaries. Hence,
1. (A) The author has mentioned various invaders like (B) is the best answer.
the Huns, Scythians and so on. (C) The political invasions are mentioned by the
(B) The first paragraph talks of philosophy being author to reiterate the sustainability factor of
regarded as the ‘goal of all the highest ... philosophy.
activities’. (D) The unity of India is not under discussion—as
(C) In the last paragraph, the author describes per the first paragraph, the author has put
political events as ‘hurricanes’ and goes on to philosophy over and above everything else.
mention that the ‘passivity of some centuries’ is (E) The ‘rise’ is misleading—the spirituality is
temporary and did not have any impact on the assumed in the passage.
underlying spirituality of the country.
The correct answer is B.
(D) The author has reiterated the underlying unity of
Indian spirit through the ages in the passage. He 4. (A) The author never mentions that the goal of
has cited political examples which could have philosophy is to bring together different people.
broken the spirit of India, but failed. The author Hence, this is the best answer.
also mentions in the first paragraph: ‘it indicated
the ... peoples produced’. Hence, (D) is the (B) Inferred from the first paragraph.
correct answer. (C) The second paragraph has mentioned
(E) Can be inferred from the last paragraph of the categorically that it has ‘outlived’ all political
passage. changes.
(D) The first paragraph talks about spirituality being
The correct answer is D.
an integral part of philosophy.
2. (A) Not mentioned. (E) The third paragraph mentions the passivity of the
philosophy, and also predicts its rise after a ‘few
(B) Author has not mentioned Indian philosophy as a centuries’.
power—retained or lost.
The correct answer is A.
(C) Out of scope—author is discussing India’s
contribution to philosophy and not to the
progress of mankind.

492

Book 1.indb 492 30/04/2019 4:49:13 PM


5.0   Language Skills Practice

Passage 12 2. (A) A few of his characters have been mentioned


but that is not what the passage is primarily
Topic—The life and works of Charles Dickens concerned with.

Scope—The early life of Dickens, his motivation for writing, (B) The author talks about it but that is not the
his style of writing, his success and his end. central theme.
(C) The author has devoted one paragraph to this
but this is not the main idea.
Passage Map
(D) The passage is predominantly devoted to his
P1—To introduce Dickens and the simplicity in his writings. writing and not his life story.
(E) The passage focuses on the influences on his
P2 and P3—To describe how in spite of criticism from
writing, his motivation for writing, his style of
some quarters, he was hailed as the greatest instructor of
writing, etc and so option E is the best answer.
the nineteenth century.
The correct answer is E.
P4—To provide an insight into Dickens’ early life and how it
influenced his works. 3. (A) The second paragraph states that ‘his works
invariably aimed to influence the consciousness
P5—To talk about the beginning of Dickens’ literary journey of his readers’.
and his overwhelming success.
(B) Not mentioned.
P6—To describe Dickens’ criticism of American culture in (C) He returned to his original style with A Tale
his writing. of Two Cities in 1859, a historical novel,
followed by Great Expectations in 1861, widely
P7—To talk about his favourite novel which was kind of an
considered his greatest literary feat. Incorrect
autobiography.
option.
P8—To talk about the tragedies in his life, his further (D) Not mentioned.
writings and to conclude with his death. (E) A couple of his novels were based on his
childhood experiences but not all.
1. (A) Dickens says in Para 4 that he felt orphaned and
betrayed by his parents who did not take care of The correct answer is A.
him.
4. (A) The author supports this view in Para 4.
(B) Dickens wrote American Notes for General
Circulation, a sarcastic travelogue criticising (B) This has also been stated by the author in
American culture and materialism. Para 1.
(C) The author definitely supports this.
(C) The unparalleled sentimentality, rage, plight and
bitterness in his novels established Dickens as a (D) The author supports the view that these novels
spokesman for the downtrodden. were developed from Dickens’ life experiences.
(E) If this statement were to be true, it would negate
(D) To some critics, Dickens was an entertainer and
the information provided in the passage.
his novels lacked intellectual challenge, but the
London Times described the British author as The correct answer is E.
the greatest instructor of the nineteenth century
in his obituary.

(E) Correct. He returned to his original style with A


Tale of Two Cities in 1859, a historical novel,
followed by Great Expectations in 1861, widely
considered his greatest literary feat.
The correct answer is E.

493

Book 1.indb 493 30/04/2019 4:49:13 PM


NMAT by GMAC™ Official Guide 2019

Passage 13 4. (A) Incorrect option.

Topic—Computer simulations (B) The author does not sound circumspect or


guarded anywhere.
Scope—The benefits of computer simulations, their types (C) The author has maintained a positive tone
and their usefulness in analysing hypothetical situations. throughout the passage.
(D) D is the correct option as can be seen in the last
Passage Map line of the first paragraph.
P1—To introduce computer simulations and their need. (E) This is opposite of the view expressed by the
author.
P2—To exemplify the uses of computer simulations.
The correct answer is D.
P3—To explain different types of simulation models.
P4—To show the usefulness of computer simulations in
‘what if’ scenarios.

1. (A) Incorrect. The author discusses the uses of


computer simulations.
(B) Correct. The author uses this passage to
introduce simulations, discuss their uses and
advantages.
(C) The primary purpose of the passage is to
discuss the benefits of computer simulation and
not outline its history.
(D) The author only talks about the advantages.
Disadvantages are not mentioned in the
passage.
(E) The passage does not focus on one particular
application of simulation.
The correct answer is B.

2. (A) This cannot be inferred.


(B) This can also be negated.
(C) Correct. The author shows appreciation for
and maintains a positive tone about computer
simulations.
(D) This option can be rejected as there is no
distrust in the author’s tone.
(E) There is no scepticism in the author’s tone.
The correct answer is C.

3. (A) This has been mentioned.


(B) This has also been mentioned in Para 3.
(C) Correct. The author clearly states that “what
if” or hypothetical scenarios can be studied by
using computer simulations.
(D) Mentioned in Para 2.
(E) Mentioned in Para 2.
The correct answer is C.

494

Book 1.indb 494 30/04/2019 4:49:13 PM


5.0   Language Skills Practice

Passage 14 4. (A) ‘Benign neoplasms do not spread, and are


always seen to be local affairs. They are usually
Topic—Cancer surrounded by a capsule and grow slowly,
Scope—The connection between genes and cancer. seldom killing their hosts if they are removed
before they affect vital organs.’ The author
certainly agrees with this statement
Passage Map
(B) Option B is clearly contradicted by the following
P1—Asks questions pertinent to cancer
sentence in the third paragraph—“It must also
P2—Describes the types of cancer be stated here that it takes much more than
P3—Explains how a normal cell transforms into a cancerous just one mutation to change a normal cell into a
cell full-fledged cancer cell.” Thus, the author will not
P4—Gives one reason how our genes may help prevent or agree with option B.
indirectly cause cancer (C) ‘Usually carcinogens are eliminated by the
immune system or certain enzymes’. The author
1. (A) This has not been discussed.
agrees with this statement as well.
(B) B summarises the central idea identified by us (D) ‘It is well known that cancer-causing elements
in our topic and scope best and should be the or carcinogens can be found in radiation,
correct answer. mechanical trauma, certain viral infections and
(C) The author has not touched upon this topic. many chemicals (tobacco tars, saccharine). All
(D) This has also not been discussed. of these have one common factor—all of them
cause mutations, which are changes in DNA that
(E) The author has talked about this in Para 2 but it
alter the expressions of certain genes’. This is
is not the central idea.
also negated as it is supported by the author.
The correct answer is B. (E) ‘It is well known that cancer-causing elements
or carcinogens can be found in radiation,
2. (A) Not mentioned. mechanical trauma, certain viral infections and
many chemicals (tobacco tars, saccharine)’. This
(B) Not mentioned. has also been mentioned in the passage.
(C) Not the correct option.
(D) Not mentioned The correct answer is B.
(E) The answer is clearly provided in the last two
sentences of the passage—‘Whatever the
precise genetic factor at work, the seeds of
cancer do appear to be in our own genes.
Cancer is indeed intimate’.

The correct answer is E.

3. (A) The author does not say that it is only genes


that cause cancer. There could be other ways of
developing cancer as well. This can be negated.
(B) Neoplasm is an abnormal cell mass that
develops when the controls of the cell cycle and
cell division malfunction. However, all neoplasms
are not cancerous. This can also be rejected.
(C) Not mentioned.
(D) This can be concluded from our reading of the
passage especially the last paragraph. This
cannot be denied or falsified making it the
correct option.
(E) Not mentioned.

The correct answer is D.

495

Book 1.indb 495 30/04/2019 4:49:14 PM


NMAT by GMAC™ Official Guide 2019

Passage 15 3. (A) ‘The author duo finds the design or intention


of the author neither accessible nor desirable
Topic—Author’s intention in a text or poem as a yardstick for judging the accomplishment
Scope—Whether the intention of the author actually comes of a literary text.’ The author agrees with this
through in a text statement.
(B) ‘Wimsatt and Beardsley conclude that a text is
Passage Map neither the critic’s nor the author’s. It is detached
from the author at birth and goes about the
P1—Provides an introduction to Wimsatt and Beardsley and
world beyond his power of intention or ability
their point of view
to control it.’ The author subscribes to this
P2—Provides one argument of Wimsatt and Beardsley statement also.
P3—Provides another argument put forward by Wimsatt and (C) This is also mentioned in Para 2.
Beardsley (D) The author actually states the opposite of C, as
P4—Describes Wimsatt and Beardsley’s point of view on the is clear in the last paragraph of the passage.
matter (E) The author has stated this in Para 3.

1. (A) The author talks about it in Para1 but this is not The correct answer is D.
what the passage is primarily concerned with.
4. (A) This is not the correct interpretation of the
(B) B summarises our thought in the topic and statement.
scope defined earlier in the best manner and
should be the correct answer. (B) This is also not the correct interpretation.
(C) The entire point being made by the passage
(C) This has been talked about in the last Para but is
is that meaning of a poem or story is what the
definitely not the central theme.
reader ascribes to it and not what the author
(D) The author has discussed their ideas but has not intended.
talked about the significance of their ideas.
(D) Incorrect interpretation.
(E) This is one of the points discussed by the author
(E) Not the correct option.
but not the main theme.
The correct answer is B. The correct answer is C.

2. (A) This has been mentioned in the last Para.


(B) This has also been mentioned in the last Para.
(C) This is also mentioned in Para 2.
(D) Option D is not mentioned anywhere in the
passage and so should be the correct answer.
(E) Mentioned in Para 2.

The correct answer is D.

496

Book 1.indb 496 30/04/2019 4:49:14 PM


5.0   Language Skills Practice

Passage 16 (C) This mentions incorrect projection of a company


which is not referred to in the passage.
Topic—Media for job advertisements
(D) This is skewed as the passage talks of higher
Scope—Different media used to advertise a firm’s chances of poor selection which only suggests
employment needs that the chances of making a correct selection
of the vacancy would be lesser but not rarer, in
certain cases.
Passage Map
(E) Please refer to lines, “Such advertisements
P1—Importance of choosing a particular medium for generate a vast number of candidates most of
advertising jobs whom aren’t qualified, and these inquiries are
P2—Newspaper advertisements for jobs costly to process. This situation increases the
P3—Effects and popularity of newspapers for job likelihood of poor selection decisions.” This
advertisements supports option E, making it the correct answer.
P4—Advertising in journals and trade professionals - Pros The correct answer is E.
P5—Advertising in journals and trade professionals - Cons
3. (A) Incorrect as the tone of the passage is neither
1. (A) This is incorrect as the passage doesn’t focus positive nor negative.
on primary channels of advertising jobs but
(B) Incorrect as the tone of the passage is neither
mentions just two channels.
positive nor negative.
(B) This is negated as the passage doesn’t mention
(C) The author talks of analyzing the pros and cons
the various aspects of jobs but the kind of
of a medium while choosing the most effective
people who can be reached by a particular
one and he goes on to explain the same. So,
channel.
this is the correct answer.
(C) This is mentioned but it is not the main idea of
(D) Incorrect because the author doesn’t subscribe
the passage.
to a particular medium but stresses on the need
(D) This is misleading as the passage talks of for analysis.
‘extensive use of newspaper advertising in
(E) Incorrect as the tone of the passage is neither
recruiting’ but not of the importance of job
positive nor negative.
advertisements in newspapers.
(E) The passage talks of different media used to The correct answer is C.
advertise a firm’s employment needs. It talks
of pros and cons of advertising for jobs in 4. (A) This is clearly mentioned in the last para of the
newspapers and trade journals with the focus passage.
that a firm should select a medium for job (B) This can be inferred from the first para of the
advertising based on the kind of vacancies that passage.
need to be advertised.
(C) This is the correct answer because the passage
The correct answer is E. clearly states that newspaper advertising is
extensively used in recruiting.
2. (A) This is incorrect because there are other options (D) This is mentioned in the second para.
available doesn’t make newspaper advertising a
risky proposition. (E) This is also mentioned as the first line in the third
para.
(B) This is negated because the passage doesn’t
talk of the interest of the employees already The correct answer is C.
working in an organisation.

497

Book 1.indb 497 30/04/2019 4:49:14 PM


NMAT by GMAC™ Official Guide 2019

Passage 17 3. (A) The passage states that bribery is accepted as


a normal part of business life “in many of the les
Topic—Bribery developed nations”, negating this option.
Scope—Views on bribery across different cultures and (B) This is not suitably supported by the passage as
approaches toward bribery it talks of the importance of bribery for firms in
uncompetitive environments but not about big
Passage Map and small firms.
(C) This cannot be definitely inferred as the passage
P1—Effects and broad view of bribery talks of difficulty in distinguishing between a
P2—Bribery in advanced industrial countries ‘bribe’ and a ‘gift’ which doesn’t mean that there
is no or little difference between them.
P3—Why there should be a liberal approach towards bribery
(D) This can be inferred from the very first line of the
1. (A) This is incorrect because the focus is not on passage, making it the correct answer.
developed countries. (E) This is incorrect as the passage talks of
criticism of bribery but not about its increase or
(B) This is opposite to the central idea.
decrease.
(C) This only talks of the premise of the passage.
The correct answer is D.
(D) This is also narrow in scope.
(E) The passage talks of bribery and how it is 4. (A) This can be inferred from the first line of the
viewed in different cultures. It talks of its passage suggesting that bribery is a common
importance and its identification and finally and normal response to the present state of
states arguments in favour of adopting a liberal business throughout the world.
approach towards bribery. So, this is the correct
answer. (B) The author doesn’t agree with this option
because the passage clearly states that bribery
The correct answer is E. is frowned upon throughout the world, making it
the correct answer.
2. (A) This is not mentioned in the passage.
(C) This can be inferred from the passage as it
(B) This is not mentioned in the passage. states that across various nations, bribes are
(C) This is not mentioned in the passage. seen as fees by holders of public positions in
supplement to their meagre incomes.
(D) This is not mentioned in the passage.
(D) This is clearly mentioned in the last line of the
(E) Refer to the line, “Governments overlook petty
passage.
corruption on the part of junior officials… remain
in their jobs.” This supports only option E. (E) This is also stated in the first line of the second
paragraph.
The correct answer is E.
The correct answer is B.

498

Book 1.indb 498 30/04/2019 4:49:14 PM


5.0   Language Skills Practice

Passage 18 3. (A) The passage doesn’t state which goals have


been achieved. So, this cannot be definitely
Topic—Regulatory policies concluded.
Scope—Need of regulatory policies; the pros and cons (B) The passage doesn’t conclude that incentive
of command and control system and incentive system of system is better than command-and-control
regulatory policies system to achieve social goals and hence,
negating this option.
Passage Map (C) Schultze states that socially valuable behaviour
is necessary for achieving social goals but the
P1—Need of regulatory policy passage doesn’t comment on its veracity and
P2—Criticism of command and control system hence, this option cannot be concluded.
(D) This is clearly mentioned in the last line of the
P3—Incentive system
first para.
P4—Defense of command and control system (E) This is incorrect because the passage doesn’t
talk of acceptance of Schultze’s incentive
1. (A) This is incorrect because the passage doesn’t
system.
talk of different levels at which the regulatory
activities of government exist. The correct answer is D.
(B) This is incorrect because Schultze is an
advocate of the incentive system and only the 4. (A) Refer to the lines, “The government could even
pros of the system have been mentioned by him provide incentives in the form of rewards for
in the passage. Also, it is not the central idea of such socially valuable behaviour as developing
the passage. technology to reduce pollution.” This supports
option A.
(C) This is incorrect because even though the
passage does state that the old policies haven’t (B) Refer to the lines, “The government could even
worked very well, it is not the central idea of the provide incentives in the form of rewards for
passage. such socially valuable behaviour as developing
technology to reduce pollution.” This supports
(D) The passage talks of regulatory policies and the
option B.
pros and cons of command and control system
and incentive system of regulatory policies. So, (C) Refer to the fourth para. It states that EPA,
this is the best answer. which follows command-and-control regulation, is
trying to develop standards for 62,000 pollution
(E) This is incorrect because the passage talks of
sources and the incentive system suggests an
why the regulatory policies have not worked well
alternative approach to the issue. Hence, the
but not about why they have failed.
incentive system would definitely not involve
The correct answer is D. developing standards for thousands of pollution
sources, making option C the correct answer.
2. (A) This is incorrect as the passage doesn’t talk of (D) Option D is supported by the first para of the
limiting the incentive policy to a few situations. passage as it suggests that regulatory policies
(B) This is negated because the passage doesn’t have been created to address the issues
talk of what EPA and OSHA should not do. mentioned in the option.
(C) This is incorrect because it is not Schultze’s (E) Option E is clearly stated in the passage as it
claim but a general statement as mentioned in mentions that command-and-control system
the first line of the passage. punishes offenders.
(D) This is the claim of the defenders of the The correct answer is C.
command-and-control system and not of Charles
Schultze. Hence, its incorrect.
(E) Refer to the third paragraph of the passage. It
clearly supports this option.
The correct answer is E.

499

Book 1.indb 499 30/04/2019 4:49:14 PM


NMAT by GMAC™ Official Guide 2019

Passage 19 3. (A) This is incorrect as reframing and visualisation


are not forms of communication.
Topic—Self-esteem
(B) This is correct as this is the main idea of the
Scope—What is self-esteem and how it can be improved. passage.
(C) This is not at all mentioned or suggested in the
Passage Map passage.
(D) This is incorrect as the passage just states that
P1—What is self-esteem or self-worth?
self-concept can be changed but whether it
P2—Life position can be changed at any stage of life, cannot be
concluded.
P3—Intrapersonal communication and self-worth
(E) This talks of self-image while the passage talks
P4—Visualisation of self-esteem and self-concept. So, the option
P5—Reframing can be rejected.

P6—Honest source of information about yourself The correct answer is B.

P7—Change in self-concept 4. (A) This is mentioned in the passage.


P8—Professional help for deep-rooted self-image problems (B) The passage states that having at least one
person who can honestly tell you about yourself
1. (A) The passage neither talks of interpersonal will help you create a healthy self-image and
communication nor the difference between self- hence, the author will agree to option A. The
esteem and self-concept. So, this is incorrect. passage states that positive self-talk, having a
(B) The passage neither talks of interpersonal person who is completely honest with you about
communication nor the difference between self- yourselves and professional help can make a
esteem and self-concept. So, this is incorrect. positive impact on you. So, the author will agree
to option B.
(C) This is incorrect because the passage doesn’t
focus on change. (C) Refer to the line, “Having at least one other
person…positive self-image.” This supports
(D) This is negated because the passage doesn’t
option C.
generally encourage therapy but only states
that therapy will be required if the self-image (D) Option D can be inferred from the second last
problems are deeply engrained in an individual. para of the passage which states that things and
self-concept change.
(E) The passage talks of self-esteem and how it can
be improved. This makes option E the correct (E) The second last paragraph of the passage
answer. states that things and self-concept change,
which makes option E the correct answer.
The correct answer is E.
The correct answer is E.
2. (A) Self-worth depends on how you see yourself as
compared to others. It hasn’t been mentioned
as a way of enhancing self-worth. Hence, this is
incorrect.
(B) This is incorrect as the question is about self-
worth and not self-concept.
(C) This is incorrect because visualisation is different
from reframing.
(D) Refer to the lines, “Visualisation takes the
notion…some other desirable behaviour.” It
supports option D.
(E) This is incorrect as the question is about self-
worth and not self-concept.
The correct answer is D.

500

Book 1.indb 500 30/04/2019 4:49:14 PM


5.0   Language Skills Practice

9 Parajumbles only question then is where should (b) come—after


(c) or after (a). Note that there is no option that puts
1. The entire paragraph is talking about the virus, so (b) immediately after (a). Hence, cbe is the correct
(d) should be a good starting sentence. (e) makes a order.
logical connection with (d). Sentence (c) should come
The correct answer is B.
after sentence (a), since it is a logical progression of
the thought mentioned in (A). Sentence (b) should be 7. (c) is an obvious start as it introduces the topic of
the last sentence of the paragraph. deforestation. It will be followed by (d) which tells us
how agriculture leads to deforestation. (b) will follow
The correct answer is C.
(d) as it further explains the process of deforestation.
2. The entire paragraph is concerned with comparing This is followed by (a) which tells us that logging
life to a theatre performance. Thus, (b) should be operations for wood and paper products are another
a good starting sentence. (c) should come next factor for deforestation.
because it provides an example of the scholars The correct answer is B.
mentioned in (b). (a) makes sense after (c) and (d)
should then come at the end. 8. (b) is the most obvious start as it introduces the
topic of internet being a great place for shopping.
The correct answer is E.
This eliminates all options except (B) and (C). (b)
3. The entire paragraph is talking about self-driving is followed by (d) and then (c) that elaborates the
cars, so (a) should be a good starting sentence. benefits of online shopping. Apart from competitive
Then things get a little tricky because (b) and (c) pricing, another advantage of online shopping is no
both look good as the second sentence. However, if shipping charge which is mentioned in (a).
you read these two sentences in the order (cb), you The correct answer is B.
will realise that it does not make any sense because
the two sentences are talking about different things. 9. (d) is an obvious start as it introduces the topic of the
Reading these two sentences as (bc), however, mistake at the Oscars. (c) follows as it tells us what
works because of the transition word ‘moreover’ that the mistake was. (a) and (b) follow as they discuss
connects these two sentences. So, (bc) it should be, the Oscars’ host talking about it on his show a day
with (d) coming at the end. after the Oscars.
The correct answer is E.
The correct answer is D.
10. (d) is the opening sentence as it introduces the
4. The whole paragraph is concerned with dopamine
topic of obesity in children. (c) will follow as it tells
and rewards, making (b) a good starting sentence.
us about the findings of a study conducted on the
(a) needs to come next since it is also talking about
topic. (a) and (b) are a mandatory pair as (a) talks
the reward system. d-c then make a logical pair,
about the health problems associated with obesity
because the 'he' in (c) refers to the 'Berridge' in (d).
and (b) further tells us how these conditions lead to a
The correct answer is B. decrease in the life span.

5. The whole paragraph is concerned with McConnell, The correct answer is C.


so (d) should be a good starting sentence. (a) should
11. (c) is the most obvious start as it introduces the
come next followed by (e). b-c then form a logical
topic of ‘Walt Disney Company’. (d) will follow (c) as
pair, since (c) further builds on the personality of
it tells us how ‘Walt Disney’ the ‘high school dropout’
McConnell described in (b).
founded his empire on fantasy and risk taking. This
The correct answer is C. will be followed by (b) which further talks about how
he created such a huge brand with his vision. (a) is
6. Here, (d) and (a) form a logical pair since the band obviously the closing sentence as it talks about the
in (a) is introduced in (d). (e) needs to come after enormous legacy left by him.
(c), since (c) introduces the Great Barrier Reef. The
The correct answer is B.
501

Book 1.indb 501 30/04/2019 4:49:15 PM


NMAT by GMAC™ Official Guide 2019

12. (a) is an obvious start as it introduces the topic of to the time and the number of dancers performing
South Korea becoming a popular tourist destination. at the open house. (b) is followed by (c) which talks
(d) follows it as it further talks about South Korea about Cunningham’s assistant Robert Swinston giving
featuring on the top ten lists. (c) follows as it traces a class in the master’s style at the open house. (a)
the reason for South Korea’s rise after breaking from closes the paragraph by talking about how in the
military dictatorship in the 80s. (b) further talks about recent years Swinston and other senior dancers
how after the 80s hosting international events like an had been training students as the master was
Olympics and a World Cup have made it a modern incapacitated by rheumatoid arthritis.
country.
The correct answer is E.
The correct answer is E.
17. (a) is the most obvious start as it introduces us
13. (b) is the opening sentence as it introduces the topic to table manners by talking about using the left
of Republican leaders breathing easy as the senators and the right hands to hold the fork and the knife
voted in favour of the debate. This is followed by (a) respectively. This is followed by (c) which tells us how
which talks about how the Republicans need to deal the fork and the knife are used. Next are (b) and (d)
with issues within their party before the debate.(c) which tell us how the fork is held and used by itself
and (d) are a mandatory pair as (c) talks about time without the knife.
being important for Republicans and (d) elaborates
it by stating the consequence of the bill not being The correct answer is B.
passed that week.
18. (c) is the most obvious start as it poses the question
The correct answer is D. of the reason behind Holmes’ captivating quality. (b)
will definitely follow (c) as it talks about the answer
14. An understanding of vocabulary will prove beneficial to the question. (d) will follow (c) as it speaks about
in resolving this jumble. (b) is the opening sentence summarizing the life of Holmes’ creator before
as it introduces the topic of the Holy See’s answering the question in (c). The paragraph ends
rapprochement with China. This is followed by (d) with (a) as it talks about Holmes’ creator, Doyle.
that states that this is not the first time this has
happened. Journalists have been influenced by The correct answer is B.
misleading accounts of those against this alliance. (a)
19. (d) is an obvious start as it opens the topic of the
and (c) are a mandatory pair as they talk about how
media trying to attract kids with advertisements.
foreigners view the situation of Chinese Catholics and
This is followed by (b) that takes the topic further
the fact that they are unaware of the changes that
and talks about the advertising industry viewing
have affected them. (c) will precede (a) as it is more
teenagers as a viable market option. The use of
general in its meaning.
‘furthermore’ and ‘also’ in option (a) connects it with
The correct answer is C. option (c) by presenting a further expansion of the
idea.
15. (c) is the most evident start as it introduces the topic
of ‘cricket’ and ‘century’ and (a) follows it as it further The correct answer is D.
talks about the number of such centuries by great
20. The use of chronology will help us resolve this jumble
players. This is followed by (d) which talks about
easily. (d) opens the topic of the paragraph - torture
Tendulkar’s great centuries and (b) obviously follows
- more specifically the two misconceptions about
with pronouns ‘his’ and ‘him’ acting as clues.
torture.This is followed by (c), which is clearly stating
The correct answer is B. the first misconception, and (b), which is clearly
stating the second misconception. (a) presents a
16. This jumble can be resolved with the help of counter argument to those misconceptions and
chronology. (d) is the most evident start as it (e) explains the basis for the counter argument
introduces the topic of Merce Cunningham’s death presented in (a).
and an open house being organised the day after
his death. (b) will follow (d) as it gives us details as The correct answer is C.

502

Book 1.indb 502 30/04/2019 4:49:15 PM


5.0   Language Skills Practice

21. (a) introduces the subject of the paragraph (wine) and 27. d-a is a logical pair since (a) talks about the bad hires
how it is made. The pronoun 'it' in (b) connects it with mentioned in (d). This gives us the answer as option (e).
statement (a) and introduces yeast. (c) describes
The correct answer is E.
how the yeast produces the alcohol. (d) talks about
fruits other than grapes that can be used to make
28. Sentence (e) introduces a penal colony which is the
wines and then (e) provides a detail with respect to
subject of the passage, hence, making it the starting
the naming of these wines.
sentence. The only option to start with sentence (e)
The correct answer is E. is option E, making it the correct answer.

22. (d) introduces the topic of inherent risk in freedom The correct answer is E.
of speech in journalism and (a) exemplifies the risk
by presenting some statistics. A reason for these 29. Sentence (c) talks of a particular situation which is
statistics is presented in (e) followed by the resulting mentioned to as ‘this’ in sentence (b). Also, both
action in (b) and finally by a conclusive statement the sentences are in context of ‘confining pressure’.
in (c). Similarly, sentence (a) talks of a way in which
fractures occur or form which is referred to as ‘this
The correct answer is D. way’ in sentence (d). Also, both the sentences talk of
hydraulic fractures/fracturing. So, both cb and ad are
23. (c) introduces the topic of selfless behaviour in
mandatory pairs and this is present in only option A.
nature. (a) exemplifies this, followed by more in (d)
and (b) ends in a summary type statement. The correct answer is A.

The correct answer is A.


30. Sentence (d) talks of what people earlier used to do in
24. (e) should be the starting sentence since the free time and sentence (b) talks of what people now
underlying theme of all the sentences is Danish do in their free time. This makes db a mandatory pair
cuisine. (cd) is a logical pair since (c) talks about which is not present in options C and E, hence, negating
mid-day meal that (d) further elaborates upon. Option them. Sentence (c) talks of ‘this’ free time which
(c) has to be the correct answer then because it is suggests that free time has been mentioned earlier. Its
the only option that starts with (e) and contains the mentioned in sentence (d). So, sentence (c) can come
logical pair (cd). only after sentence (d). So, both options A and D are
negated, making option B the correct answer.
The correct answer is C.
The correct answer is B.
25. (a) is a logical pair since (a) describes the places
mentioned in (d). This brings us to options (d) and (e). 31. Sentence (a) rephrases sentence (d), making da a
It doesn’t make sense starting the paragraph with mandatory pair which is present in only options B
(c). The entire paragraph talks about glaciers. Thus, and E, negating the remaining options. Sentence (d)
(d) should be the perfect starting sentence, making should come after sentence (b) as sentence (b) gives
option E the correct answer. a reason for the result mentioned in sentence (d). So,
bdac is the correct answer.
The correct answer is E.
The correct answer is B.
26. a-d is a logical pair since they both highlight
problems faced by certain species. (e) and (b) need 32. ‘ae’ is a mandatory pair as both the sentences
to come after (c), since they both talk about positive talk of two types of biopsy. Sentence (e) will come
results and (c) brings out this contrast by using the later because of the use of conjunction, ‘but’. This
word however. Thus, option (b) should be the correct mandatory pair is only available in option C, making it
answer. the correct answer.
The correct answer is B.
The correct answer is C.

503

Book 1.indb 503 30/04/2019 4:49:15 PM


NMAT by GMAC™ Official Guide 2019

33. Sentence (e) introduces the topic and sentence (c)


directly negates it, making ec a mandatory pair.
So, options A, B and E are negated. Sentence (d)
explains the negation. ab is another mandatory
pair as (a) gives an example of how an alternative
therapy, that is, Chiropractic can be risky to the
patients while (b) states that despite the risks, the
patients are rarely informed about them. So, ecdba
is the correct answer.

The correct answer is C.

34. All the sentences talk about shark fin. Since


sentence (e) introduces this topic, it has to be the
opening line of the sequence. This immediately
eliminates options (A), (B) and (C). The use of
however in sentence (a) implies a contrast and this
contrast is being made with sentence (c), thereby
making c-a a logical pair.

The correct answer is E.

35. Sentence (b) makes a statement that everything


can be great. Sentence (c) states different things
that can be great. This makes bc a mandatory pair
which is present only in option A, making it the
correct answer.

The correct answer is A.

504

Book 1.indb 504 30/04/2019 4:49:15 PM


Book 1.indb 505 30/04/2019 4:49:15 PM
6.0  Logical Reasoning Review

Book 1.indb 506 30/04/2019 4:49:15 PM


6.0  Logical Reasoning Review 6.0 Logical Reasoning Review

6.0 Logical Reasoning Review


The word ‘logic’ is derived from the Greek word ‘logos’, which means a word that expresses a thought
or some information. This section, accordingly, measures your ability to comprehend and synthesise the
information given in the question, without bothering with the truth or accuracy of that information. You
will be asked to draw inferences from statements, choose between strong and weak arguments, identify
assumptions implicit within innocuous-looking statements and even arrive at decisions in light of the given
information.
You will also be required to use your lateral thinking skills on questions that require you to decode
patterns—both numerical and visual—and series, make sense of relationships and understand directions.
The key, while attempting Logical Reasoning questions, is to focus only on the information given to you
and not allow any of your own knowledge, experiences or presumptions to cloud your judgment.
The Logical Reasoning section on the NMAT by GMAC™ will test you on the following two broad
areas of reasoning:
1. Verbal Reasoning
2. Analytical Reasoning

Verbal Reasoning includes the following topics:


• Statement and Argument Questions
• Statement and Assumption Questions
• Statement and Inference Questions
• Decision Making and Course of Action
• Critical Reasoning

Analytical Reasoning includes the following topics:


• Linear and Circular Arrangements
• Classification
• Syllogisms
• Blood Relation Questions
• Coding Questions
• Series Questions
• Direction Questions
• Alphabet Test Questions
• Input Output questions
• Matrix Type Questions
• Symbol-Based Questions
• Other Reasoning Questions

507

Book 1.indb 507 30/04/2019 4:49:15 PM


NMAT by GMAC™ Official Guide 2019

6.1 Top Tips to Prepare for Logical Reasoning


1. You will get 40 questions in the Logical Reasoning section on the NMAT by GMAC™ that you will
have to attempt in 38 minutes. You will have about 1 minute for each question.
2. This section will test you on two areas of Reasoning—Verbal Reasoning and Analytical Reasoning.
3. Read every part of the question carefully. Similarly, make sure you have looked at each answer choice
before selecting your final answer. Even if you like an option at the first glance, make sure you have
taken a look at all the remaining options as well before marking your answer.
4. NEVER assume or use any information that is not provided in the question. This section is not trying
to assess your general knowledge. Consider ONLY the information given in each statement and use
this to answer the question.
5. Pay special attention to words like ‘all’, ‘some’ or ‘none’ when you read the factual information provided
in each question. Other qualifying words such as ‘other than’, ‘only’ or ‘unless’ are also important. These
words can play a critical part in precisely specifying the facts that need to be used while reasoning.
6. Try to represent the given information pictorially, especially in arrangement questions, or as a Venn
diagram in syllogism questions.
7. Logical Reasoning skills cannot be developed overnight, so make sure you spend a lot of time before
the test practising Logical Reasoning questions and analysing your mistakes.

508

Book 1.indb 508 30/04/2019 4:49:15 PM


Book 1.indb 509 30/04/2019 4:49:15 PM
6.2  Verbal Reasoning

Book 1.indb 510 30/04/2019 4:49:15 PM


6.0  Logical Reasoning Review  6.2  Verbal Reasoning

6.3 What is Measured?


The verbal reasoning section will test you on your ability to use logic to evaluate the strength of arguments,
draw assumptions and inferences from statements, evaluate good and bad decisions and so on.
This section not only tests your understanding of English; it also tests your logical reasoning ability.

6.4 Overall Test Taking Strategies


• Logic does not mean common sense. So, do not use common sense or any outside information while
answering questions. Only focus on the statement and options given to you.
• Read the question carefully and watch out for terms such as must, always, could, might and so on, as
these could define the difference between a correct and an incorrect answer choice.
• Avoid extreme options in Inference -based questions.
The next few sections will provide you with in-depth strategies for approaching each topic.

511

Book 1.indb 511 30/04/2019 4:49:16 PM


NMAT by GMAC™ Official Guide 2019

1 Introduction
Logic, which derives from the Ancient Greek word, logike refers to the systematic study of the various
kinds of arguments. A valid argument is one where there is a logical, rational correlation between the
assumptions made in the argument and the conclusions derived.
Questions based on Verbal reasoning are regularly asked in NMAT by GMAC™ and are quite challenging.
Their difficulty level is higher as compared to the other management entrance examinations and
comprehensive practice across the various question types is required in order to enhance one’s accuracy
level.

Important Learning:  Complete the practice tests to improve your comfort level with the various types
of questions. This shall also help ascertain areas that prove challenging with respect to comprehension and
analysis. Identify these question stems and practice extensively. Use the method of elimination in order to
arrive at the correct solution.

Verbal Reasoning (VR) is an integral part of competitive exams. It aims to measure a candidate’s ability to
draw logical conclusions based on statements or arguments, and to identify the strengths and weaknesses of
those arguments. It is important to keep in mind that the statements and assumptions in logical reasoning
might defy your expectations rooted in the real world.
For instance, consider the statement, eating a lot makes you lose weight. (This goes against the general
knowledge that eating a lot leads to obesity, but within the limits of logical reasoning, this is a valid argument.)
Now, if we state that Rahul has lost weight, then based on the information provided we can conclude that
Rahul eats a lot.
As the name suggests, Verbal Reasoning questions will, in part, test your language or comprehension
skills. You will only be able to ascertain the correct answer if you are able to comprehend the argument or
question correctly.
Questions based on verbal reasoning are mostly on the application of the two types of logic :
i. Deductive logic
ii. Inductive logic

Deductive logic
Deductive logic, also known as the “from the top down” approach, starts with a general idea and works
down to the details. It is a basic form of valid reasoning. As stated above, it starts out with a general
statement, or hypothesis, and examines the possibilities to reach a specific, logical conclusion.
Deductive reasoning usually follows steps. There is a set of premises followed by a conclusion. Syllogisms
constitute a common manifestation of deductive reasoning wherein two statements are given and a
conclusion is drawn on the information provided in these statements. For example, the premise “All A is B”
could be followed by another premise, “All C is A.” These statements would lead to the conclusion “All C is
B.” Syllogisms are considered an effective tool to assess the validity of an argument.
For example, “All women are intelligent. Anna is a woman. Therefore, Anna is beautiful.” For deductive
reasoning to be valid, the hypothesis must be correct. The premises are assumed to be true. Hence, the
conclusion is logical and true. In deductive reasoning, if something is true of a class of things in general, it
is also true for all members of that class. Questions based on necessary and sufficient conditions also use
deductive reasoning.
512

Book 1.indb 512 30/04/2019 4:49:17 PM


6.0  Logical Reasoning Review  6.2  Verbal Reasoning

Inductive logic
Inductive logic, also known as the “from the bottom up” approach reasoning, is different from deductive
logic in which the reasoning is based on possibilities, not on certainties. Inductive reasoning draws generic
conclusions from specific information. Essentially, broad inferences are drawn from a specific piece of data.
This is called inductive logic.
Inductive reasoning allows for the possibility of the concussion to be tentative, even though some other
conclusion that is contrapositive may also be true. For example: “Rekha is a teacher. Rekha is tall. All
teachers are tall.” This may or may not be correct.

Common types of Verbal Reasoning questions are:


• Statement and Argument Questions
• Statement and Assumption Questions
• Statement and Inference Questions
• Decision Making and Course of Action
• Critical Reasoning

2 Statement and Argument Questions


A statement that is used to support a proposition or statement in favour or against is called an argument.
It is irrelevant whether an argument is favourable or adverse to a proposition. Both favourable & adverse
arguments are considered as strong enough in their own ways.
Strong arguments are those that are directly connected to the subject matter in the given statement and
that help you strengthen or support that statement. Weak arguments, on the other hand, are either not
connected to the subject matter at all or they are connected to it in a very indirect and far-fetched manner.
Let us understand these concepts in detail.

Structure of an argument
Let us understand the structure of an argument with the help of an example:
People don’t like to visit the Evergreen wildlife park in the rainy season. This year the park authorities have
reconstructed all the roads inside the park, so people will like to visit the Evergreen Park in the rainy reason this
year.
Conclusion—This is the point of the argument and answers the question What, that is, what the argument
is basically stating—that people would like to visit the Evergreen Wildlife park in the rainy season this year.
Conclusions usually follow signalling words such as thus, so, hence, therefore, and so on. In case there are
no such words in the argument, try to paraphrase the entire argument in one line. This line would almost
always be the conclusion of the argument.
Evidence—While the Conclusion tells you What the argument is saying, the Evidence tells you Why the
argument is concluding what it is concluding. So in the above argument, why does the author conclude that
people will like to visit the Evergreen Park this year? Because the park authorities have reconstructed all the
roads inside the park, so this becomes your evidence.
Evidence usually follows signalling words such as because, since, as a result of, and so on.

513

Book 1.indb 513 30/04/2019 4:49:17 PM


NMAT by GMAC™ Official Guide 2019

So the conclusion tells you the what of the argument and the evidence tells you the why of the argument.
Another way of looking at conclusion and evidence is that a conclusion will almost always be an opinion
whereas the evidence will almost always be a fact. In the above argument it is a fact that the roads have
been reconstructed but it is the author’s opinion that people will like to visit the Evergreen Park this year.
Assumption—Now, going back to the above argument, notice that from the given evidence we cannot
necessarily arrive at the stated conclusion. The argument only states that people don’t want to visit the
Evergreen Park during the rainy season; it never states why people don’t like to do so. So the author assumes
that the only reason people don’t like to visit the park is because of the poor road conditions within the
park. If this is not assumed then the argument will fall apart.
For example, if the real reason why people do not visit the Evergreen Park was the fact that there are
hardly any animals in the park, then even if the roads were of best quality, people will not visit the park
because bad roads was not the reason for people not visiting the park in the first place. So, for the author to
conclude that people will want to visit the park this year, he has to assume that the only reason people did
not visit the park earlier was the poor road conditions inside the park.

Identifying an Argument
How to identify an argument as strong or weak?
Strong Argument: Statements that reinforce/build on the idea discussed through the use of reasons, facts,
and examples.
Strong argument provides a valid and directly related reason either in favour of or against the proposal
made.
• A strong argument provides a sensible and acceptable argument that either supports or opposes the
proposition.
• It is supported by the given facts or established notions.
• An argument based on Universal Truth is always strong.

Weak Argument: Statements that present data, facts, reasoning that either contradicts or dilutes the
argument presented are known as weak arguments. They can be personal or judgmental in nature.
• A weak argument is not directly related to the given statement.
• A weak argument does not discuss the key aspects of the statement.
• It includes limited explanations.
• Weak arguments are personal or judgmental in nature.

STRONG ARGUMENTS WEAK ARGUMENTS


• Present opinions that relate to the central idea • Present opinions that are unrelated or vaguely
being argued. connected to the subject.
• Provide facts, data, information that are relevant • They are not supported by facts or reason.
and supportive of the subject. • They might also present facts or data in relation to a
minor/marginal aspect of the argument.
• Are logical and coherent in their construction and • Are not logical or coherent in their construction.
presentation

514

Book 1.indb 514 30/04/2019 4:49:17 PM


6.0  Logical Reasoning Review  6.2  Verbal Reasoning

For example, if you are writing a persuasive note to your mother to try to convince her to give you more
pocket money, which of these two arguments do you think will be more effective?
1. If you gave me more pocket money, I will be able to buy more clothes.
2. If you gave me more pocket money, I will be able to purchase more reference books for the upcoming
examinations

The second argument is definitely the stronger one as the reasoning used to support it is likely to garner
greater support from your mother.

Important Learning: 
• An argument that addresses the given issue even by providing a partial solution should be
considered a strong argument provided the solution is relevant to the issue and does not create a
new set of problems.
• An argument that presents a positive outcome for large number of people is considered a strong
argument. For example, even if a government or an organisation's action puts some burden on the
public but is justified on the basis that it would improve the quality of the services provided or
increase the number of new services, then the argument is strong.
• An argument that presents an unethical, unrealistic or extreme solution is considered weak.
• Any argument that supports added burden on the general population, either in terms of money
or hardship, is usually considered a weak argument. For example, if an action is supported on the
ground that it would increase the revenue or profitability for an organisation/ government but add
to the burden of the general public using the service, it is not a strong argument.
• Again, if an action is opposed on the ground that it would increase the expenditure of the
government, even if the general public would have gained by the said action, it is considered a
weak argument.

Let us elucidate the above points with the help of some examples:

Example

Statement: Should the government invest in female education to improve the sex ratio in the
country?
Argument I: Yes. Higher levels of female education will help reduce female infanticide.
Argument II: No. The government does not have the resources to invest in such schemes.
Argument III: No. There is a simple solution to the problem: the extra boys should be killed.

Solution

In the case discussed above, argument I will be considered a strong argument, because, even though
it may not solve the whole problem, it addresses an important aspect of the problem. This is because
it presents a positive and relevant solution to the issue being discussed. Argument II will not be
considered a strong argument. As mentioned above, in statements involving government/public
bodies or people in authority, the cost of undertaking an activity that impacts public life is never a
strong argument for not doing it. The third argument, though presenting a direct solution to the core
problem of sex ratio imbalance, will be considered weak because it is unethical and extreme. At the
same time, it will create larger problems.

515

Book 1.indb 515 30/04/2019 4:49:18 PM


NMAT by GMAC™ Official Guide 2019

Consider another example in the light of the above.


Example
Statement: Should the government increase railway fares?
Argument I: Yes. This will help the government reduce its losses.
Argument II: No. Railway is a public service and the government should bear all the burden of it.

Solution

Note that both the arguments will be considered weak. Argument I will be considered a weak
argument as it advocates the creation of additional burden on the masses purely to reduce losses.
However, argument II is also a weak argument as it places the entire burden on the government and
does not provide any solution.

Example
Statement: Should the government increase railway fares?
Argument I: 
Yes. This will increase the revenue being earned by the government, which will be
used to improve the railways’ infrastructure.
Argument II: No. People cannot afford to pay more.

Solution

Note that the first argument also supports increase in the railway fares but provides a positive outcome
of the decision which will eventually end up helping the people. Hence, it will be considered a strong
argument. The second argument is vague and unstructured. Hence, it is a weak argument.

Types of Questions
Questions with two arguments
Statement: Should mercy killing be legalised in India?
Argument I: Yes. Mercy killing is already legalised in several other countries.
Argument II: No. Given India’s current social fabric, there is a high probability that the mercy
killing provision might end up being misused by some people for their ulterior
motives.

Solution

The stance of other countries with regards to mercy killing is not necessarily related to what India’s
stance on the matter should be, since the socio-cultural factors prevailing in those countries might
be different from those prevailing in India. Thus, Argument I is a weak argument. Argument II, on
the other hand, points out a genuine problem with regards to legalising mercy killing in the Indian
context; therefore, Argument II is a strong argument.
The correct answer is B.

Questions with more than two arguments


It is not necessary that you will always get only two arguments in the question to choose from. You may
even get four or five arguments with the question stem asking you to identify which of these are strong and
which are weak.

516

Book 1.indb 516 30/04/2019 4:49:18 PM


6.0  Logical Reasoning Review  6.2  Verbal Reasoning

Let us take a look at an example:


Statement: Indian students who have completed their education at public institutions in India,
such as the IITs, should be banned from taking up jobs abroad as India gains nothing
from their expertise.
Argument I: Yes. This is the only way to sustain the high quality of teaching in Indian educational
institutions.
Argument II: No. Eventually most of these students come back to work in India and their
experience of working abroad makes them more effective workers.
Argument III: No. Students should be free to decide where they want to work.

(A) Only argument I is strong


(B)
Only argument II is strong
(C) Only arguments I and II are strong
(D) Only arguments I and III are strong
(E)
All the arguments are weak

Solution

There is no real connection between the quality of teaching at Indian educational institutions and
which country a student chooses to work in. Thus, Argument I is weak.
Argument II gives a strong and logical point against the view given in the statement by showing that
the country may benefit in the long run. Thus, Argument II is strong.
Argument III does not address the point raised in the statement that the country gains nothing from
the expertise of these students. Thus, Argument III is weak.
The correct answer is B.

Tips for approaching argument-based questions


While attempting Argument questions, make sure you arrive at the answer based only on the information
that is given to you in the statement and in the arguments. Do not allow your preconceived notions and
biases on the subject matter in the statement come in the way of arriving at the logically correct answer.
Do not use your own presumptions while answering Argument questions. Go only by the facts given to you
in the statement.
If the argument relies on some assumption, it is a weak argument. For example:
Statement: Should doctors be banned from carrying out private practice?
Argument: Yes. This will improve the quality of service in public hospitals.
Solution: The above argument relies on the assumption that the cause of the poor quality
of service in public hospitals at present is the fact that doctors are carrying out
private practice. However, this assumption may or may not be true. Thus, this
argument is a weak one.

If the argument gives someone’s opinion, it is a weak argument. For example:


Statement: Should there be reservation for women in the Parliament?
Argument: No. The home minister does not think so.
Solution: Just because the home minister does not believe in something does not mean that
it is not the right thing to do. Thus, this argument is a weak one.
517

Book 1.indb 517 30/04/2019 4:49:18 PM


NMAT by GMAC™ Official Guide 2019

A word or phrase should be used to connote the same meaning in the statement and in the argument.
For example:
Statement: Should there be a divorce between the judiciary and the legislature?
Argument: No. The judiciary and the legislature are not a married couple.
Solution: The word ‘divorce’ is used in a different sense in the statement, whereas the
argument is using the literal meaning of the word. Thus, this argument is a weak
one.

Let us now take a look at another example:


Statement: Should people below the age of 18 be allowed to drive?
Argument I: No. People below the age of 18 do not have enough mental and physical maturity
to drive a vehicle.
Argument II: Yes. People of any age should be free to do what they want.
Solution: The first argument provides a valid reason why people below the age of 18 should
not be allowed to drive. So, it is a strong argument. Argument II on the other
hand just provides a random opinion without backing it up with any facts. Thus,
it is a weak argument.

The correct answer is A.

3 Statement and Assumption Questions


An assumption is an unstated premise that supports the conclusion. An assumption provides logical
support to the given statement of the question. In other words, we can say that an assumption is a concept,
thought or view that is considered to be true and taken for granted. An assumption is the hidden part of a
statement that is neither directly explained nor explicit. It is implicit in the flow of the argument.
Both premise and assumption are unquestionable facts but the assumption, unlike the premise, is not
explicitly stated and needs to be deciphered. Assumption is something that is taken for granted in the
context of a statement. For instance, ‘All kids are happy when they get new gadgets. Therefore, Vicky will
be happy when he gets the new mobile phone.’ In this, the assumption is that Vicky is a kid. Without this
assumption, the argument will make no sense. We can also assume that mobile phone is a gadget.
Questions with statements and assumptions are common in the logical reasoning section of entrance
examinations. An assumption question asks you to identify an unstated premise of the statement. As you
read the statement, try and identify a gap in the underlying argument. The gap can only be closed by stating
out aloud what is now being assumed.

Identifying an assumption
While saying something, a person does not state each and every aspect of his/her views. There are many
things, views, thoughts, and so on that remain unsaid. These ‘unsaid’ things, views, thoughts, and so on are
considered as ‘assumptions’.
An assumption is an important part of any argument. Almost all arguments are based on some assumption
or the other. The most important aspect of assumptions is that they are implicit, that is, they will never be
written explicitly in the statement/argument. However, the assumption must be true for the argument’s
conclusion to be true.

518

Book 1.indb 518 30/04/2019 4:49:18 PM


6.0  Logical Reasoning Review  6.2  Verbal Reasoning

Testing an assumption
One of the most effective methods to solve questions based on assumptions is the ‘negating method’. Any
communication, be it written or verbal, makes certain assumptions. These ‘certain’ assumptions are the
very foundation of the argument. This means that if we believe the argument to be valid, the assumptions
also have to hold true. Thus, in order to ascertain whether something is, in fact, an assumption, you just
have to negate it. If the negation does not impact the central argument being made by the author, the
said statement is not an assumption. If however, upon negating the statement, the logic of the argument
collapses, then the statement is a central assumption, critical to the author’s argument. For example,
consider the statements given below:
Statement: In the recently held Doctors’ conference, the session on ‘Ethics in medicine’ surprisingly
attracted a large number of participants and also received extensive media coverage.
Assumptions:
I. Media is always very positive towards the issue of ethics in medicine.
II. The medical sector in India is known for its ethical practices.
III. Such a large participation was not expected.

Now, the Statement talks about a particular session in the Doctors’ conference which got many participants
and media coverage. All of this was surprising for the author.
Let’s consider the different assumptions:
1. 'Media is always very positive towards the issue of ethics in medicine': it is tempting to select this
option as an answer because the media gave extensive coverage to the session so one might think that
the media is ALWAYS positive towards it. But that’s not true, as the author has expressed surprise
over the extensive media coverage. That means that this behaviour is new as per the author.
2. 'The medical sector in India is known for its ethical practices': we cannot take any decision with
respect to this particular assumption. On one hand, it is possible that the session is being organised
because these practices are well established, but, on the other hand, it is also likely that the session is
being conducted to increase awareness about ethics. As it’s not stated anywhere in the sentence so this
is, definitely, not the author’s assumption.
3. 'Such a large participation was not expected': this assumption is, indeed, true. Since the author is
surprised by the response (Public participation plus media coverage) then it is fair to assume that is
was not expected. Note: Use the keyword 'surprisingly' to identify the assumption.

We have just demonstrated how to attempt questions that ask you to identify the assumption underlying
the statement. Please note that it is important to stay within the confines of the argument and not apply
external knowledge to the question at hand.

Types of Questions
Questions with two assumptions
In assumption questions on the NMAT by GMAC™, a statement will be given to you and two possible
assumptions will be written below it. The answer choices will look something like this:

(A) Only I is an assumption


(B) Only II is an assumption
(C) Either I or II is an assumption

519

Book 1.indb 519 30/04/2019 4:49:18 PM


NMAT by GMAC™ Official Guide 2019

(D) Neither I nor II is an assumption


(E) Both I and II are assumptions

Let us look at an example:


Statement: Since the CEO is going on a one-month leave, he has asked the CFO to take over
the operations of the company for that period.
Assumption I: The CFO has the necessary skills to run the company effectively.
Assumption II: The CFO may not accept the request of the CEO.

Solution

If the CFO is being asked to take charge of the company, it is definitely implied that he has the skills
to do so. Thus, Assumption I is a valid assumption. Whether the CFO accepts the offer or not is in no
way implicit in the given statement. Thus, Assumption II is not a valid assumption.
The correct answer is A.

Questions with more than two assumptions


You can even get questions with more than two assumptions in them. Let us take a look at an example:
Statement: Opening a movie theatre in Mahipur will be a loss-making venture.
Assumption I: Residents of Mahipur do not like to watch movies.
Assumption II: It is very expensive to set up a movie theatre in Mahipur.
Assumption III: There will not be enough demand for the movie theatre’s offerings in Mahipur.
(A) Only I is an assumption
(B) Only I and II are assumptions
(C) Only II and III are assumptions
(D) Only III is an assumption
(E) None of them are assumptions

Solution

On the face of it, I looks like an assumption. However, there is nothing in the statement to suggest
that the residents of Mahipur do not like to watch movies. It could be that the residents love watching
movies, but the ticket prices are so high for this new theatre that they cannot afford it. Hence, I is not
an assumption.
Again, II may or may not be the case. For example, it could be possible to set up the theatre with very
little money, but the residents of Mahipur may just not be interested in watching movies in general.
III is definitely implied in the statement, as the only reason a venture turns loss making is if there
is not enough demand for its products and services. The reason for this lack of demand could be
anything. Thus, III is a valid assumption in the argument.

The correct answer is D.

Tips for approaching assumption questions


Building Proficiency in the Question Type
• Step I: Review the theoretical concepts given at the beginning of the chapter. Study the examples and
understand the application of principles.
520

Book 1.indb 520 30/04/2019 4:49:19 PM


6.0  Logical Reasoning Review  6.2  Verbal Reasoning

• Step II: After reviewing the theory, start practising the questions given in the book. At this point in
time, focus on quality and not quantity. This means, attempt a limited number of questions each day,
analyse your responses and redefine your strategy. Always try and use the ‘negation method’ for solving
these questions.
You have to work as a detective and find out the hidden premise which is the assumption through a critical
analysis of the data provided in the argument. Keep in mind these four rules before you start:
• Use only the data that is provided in the argument. Do not utilise outside knowledge unless
specifically mentioned.
• Always consider the statements and assumptions to be true. Do not judge them based on your own
conception of absurdity.
• An assumption is not a reiteration of an already stated premise.
• Choose the most appropriate answer based on a valid reasoning keeping in mind the following
elements:
1. Assumption must always be true.
2. Look out for words such as only, best, always and so on. These words reduce the chances of an
option being a valid assumption.
3. Watch out for connecting words as they could show an implicit similarity, such as similarly,
likewise, or a contrast, such as yet, however, nonetheless.
4. Most does not mean all.
5. Many does not mean most.
6. The Assumption must be implicit in the given statement. If it may or may not be implicit, then it is
not a valid assumption.

Let us look at one more example:


Statement: Organisations should promote people based only on their contribution to the
company and not on the duration of their service within the company.
Assumption I: The duration of service is not a reflection of the contribution that an employee has
made to the company.
Assumption II: It is possible to empirically measure the contribution that each employee has made to
the company.
(A) Only I is an assumption
(B) Only II is an assumption
(C) Either I or II is an assumption
(D) Neither I nor II is an assumption
(E) Both I and II are assumptions

Solution
Since the statement concludes that the length of service should not be the criteria used to determine
whether to promote a person or not, it is obviously being assumed that there is no direct correlation
between the length of service and the contribution an individual has made to the company. So I is an
assumption. Again, when the statement concludes that promotions should be based on the contribution
of each person to the organisation, it assumes that such contributions can be empirically measured.
Thus, both the assumptions are implied in the argument.

The correct answer is E.

521

Book 1.indb 521 30/04/2019 4:49:19 PM


NMAT by GMAC™ Official Guide 2019

4 Statement and Inference Questions


An inference is something that is not directly stated but can be implied from the given information. An
inference is an extension of an argument, not a necessary part of it.
An inference is similar to a conclusion. The one difference between the two, however, is that an inference
is never explicitly stated in the statement. For example, if we say that Raghav is the most intelligent student
in his class, then from this statement we can easily infer that Vivek, who studies in the same class, is not
as intelligent as Raghav. Note that we have arrived at this inference even though Vivek is not explicitly
mentioned anywhere in the statement.
The difficulty with inference questions is that we always have a tendency to over infer by reading too
much between the lines or by using our own knowledge to answer questions. You must avoid doing these
two things.

Tips for approaching inference questions


Usually in Critical Reasoning questions, the argument comprises both a premise and a conclusion.
However, in Inference Questions, a conclusion is not provided, albeit you are required to draw a conclusion
based on the information given. Generally, inference based questions adopt the following structure:

• The statements above, if true, best support which of the following as a conclusion?
• Which of the following inferences is best supported by the statement made above?
• Which of the following conclusions can most properly be drawn from the information above?
• If the statements above are true, which of the following must be true?
A key point to be remembered is that the correct answer choice is the one that extends the conclusion a
step further. Moreover, it should be in the same tone (positive, negative, neutral) that comes across in the
argument.

THINGS TO KEEP IN MIND FOR INFERENCE QUESTIONS:


1. Inferences generally test your ability to derive an additional conclusion from the stated premises.
2. Answers with extreme wordings should be avoided. Inference answers typically do not use only,
always, never, best or any strong words that present extreme scenarios. The right answers on inference
questions will generally use more qualifiers and less extreme language.
3. Try to fully understand what the passage’s point is and the exact reasoning so that if the question asks
you to extend that reasoning, you are able to do so accurately.
4. Use the process of elimination. Inference questions typically have two or three good answers. The best
way to tackle these questions is to gradually eliminate the possible answers until you have one or two
and then choose the last one by scope.
5. An option which is not a correct inference may seem very close but will challenge the premise/given
conclusion in someway or another.

Types of Questions
Questions with two inferences
Here is how the answer choices typically look in an inference question:
(A) Only I can be inferred

522

Book 1.indb 522 30/04/2019 4:49:19 PM


6.0  Logical Reasoning Review  6.2  Verbal Reasoning

(B) Only II can be inferred


(C) Either I or II can be inferred
(D) Neither I nor II can be inferred
(E) Both I and II can be inferred

Let us take a look at an example:


Statement: Jogging is good for the health.
Inference I: Walking is not good for the health.
Inference II: All healthy people jog.

Solution

Just because jogging is good for health, it does not mean that walking or some other activity is not
good for health. Thus, Inference I is not valid. Again, even though it is true that jogging is good
for the health, it is not necessarily true that all healthy people jog. Some healthy people may be
following some other exercise routine, for all you know. Thus, neither I nor II can be inferred from the
statement.

The correct answer is D.

Questions with more than two inferences


You can even get questions with more than two inferences associated with them. Let us take a look at an
example:
Statement: Most shoes available at the Broadway store are expensive.
Inference I: There are no cheap shoes available at the Broadway store.
Inference II: The Broadway store is overcharging its customers.
Inference III: Some of the shoes available at the Broadway store are not expensive.
Inference IV: People should avoid buying shoes from the Broadway store.
(A) Only I can be inferred
(B) Only II can be inferred
(C) Only III can be inferred
(D) Only I and III can be inferred
(E) Only III and IV can be inferred

Solution
According to the statement, most shoes at the Broadway store are expensive. This does not mean that
all the shoes are expensive. There may also be some cheap shoes available. In fact, there must be some
cheap shoes available, or else the statement would have used the term all instead of most. Thus, I is not
an inference in the argument and III is an inference in the argument. We have no idea whether the
Broadway store is overcharging its customers; thus, Inference II is incorrect. Again, no one is asking us
for our suggestion (implied by the use of ‘should’); thus, IV is also not a valid inference.

The correct answer is C.

523

Book 1.indb 523 30/04/2019 4:49:19 PM


NMAT by GMAC™ Official Guide 2019

Important Learning:  The strategies discussed above can also be applied for question based on
Statement and Conclusion.

5 Decision Making and Course of Action


In NMAT by GMAC™, this type of question is generally tested through course of action questions.
Decision Making is essentially a test of a candidate’s ability to apply logical and reasoning abilities to a set
of given information and arrive at a plausible course of action. The decision-making questions in NMAT by
GMAC™ present students with case studies/scenarios which need to be resolved effectively. In a nutshell,
they are conditions that require a decision to be made, albeit in a logical and rational manner.
These questions involve elements of critical and logical reasoning.
They involve taking steps to address a problem or a circumstance to improve the situation. In such type of
questions, a situation is presented within a statement and some courses of actions are suggested in the same
context.
These questions are a bit different from conventional reasoning type questions. The main feature of these
types of questions is that they are designed in such a way that the decision-making ability of the candidates
can be scrutinised. In simple words, these questions test your ability to judge a problem and thus find a
suitable course of action for it.
You have to read the statement, analyse it, identify its cause and decide on an apt course of action that
should be followed in that situation. A suggested course of action should be practical, pragmatic and
connect with the realistic aspect of life.

Structure Of The Questions


Part 1: Statement
As the name suggests, there will be a statement that will define the problem. You have to read the statement
and identify the underlying problem.
Part 2: Course of Action
This part shall contain possible solutions or decisions that can be taken with respect to the problem. The
candidate has to decide which course of action logically follows the situation and can be adopted.
Sometimes, it might be difficult to figure out the right course of action but these can be solved quite easily
with the help of a logical mind frame.

Pattern of questions
There are two types of question patterns. The first one is based on a problem and suggested courses of action
which can help in the resolution of the problem. The second one includes a fact or situation and suggested
courses of action which could improve the situation. So, while attempting this segment, first decide whether
the given statement is based on a problem or is describing a fact/situation.
In Problem and Solution based pattern, the given statement presents a problem and the suggested course
of action presents solutions which can be accepted as appropriate, if they solve or minimise the problem and

524

Book 1.indb 524 30/04/2019 4:49:20 PM


6.0  Logical Reasoning Review  6.2  Verbal Reasoning

are practically possible. We have to keep in mind that a prescribed course of action that solves the problem,
but is not practically possible is of no use and must definitely be rejected.
Statement
A large number of people in Palampur village are suffering from dengue.
Courses of Action:
I. The city municipal authority should take immediate steps to carry out extensive fumigation in
Palampur.
II. The citizens of Palampur should be advised to take proper steps to prevent mosquito bites.
Solution
Clearly, both the two Courses of Action will prevent dengue and reduce the problem. So, both the courses
follow.
In Fact and Improvement based pattern, the given statement provides a simple fact and the suggested
course of action suggests ways to improve the condition.

Types of Questions
Questions with two decisions to choose from
In this question type, you will be given a statement followed by two courses of action or decisions. You will
be required to analyse the situation and then decide which of the two decisions to take. The choices will
typically read as follows:

(A) Only I should be pursued


(B) Only II should be pursued
(C) Either I or II should be pursued
(D) Neither I nor II should be pursued
(E) Both I and II should be pursued
Let us take a look at an example:
Statement: The manager of a factory has a suspicion that two of the workers are stealing goods.
Decision I He should immediately fire the two workers.
Decision II He should search the two workers or find out in some other way whether they have
actually stolen something.

Solution

The statement clearly states that this is just a suspicion that the manager has. It does not make
any sense to fire two people just on the basis of a suspicion. So, Decision I is not a correct one. The
manager should first investigate whether the workers have actually stolen anything and only then act
against them. So, Decision II is the correct one to make.

The correct answer is B.

Questions with three decisions to choose from


You can sometimes also get three decisions to choose from. Let us take a look at an example:

525

Book 1.indb 525 30/04/2019 4:49:20 PM


NMAT by GMAC™ Official Guide 2019

Statement: Workers shifting from villages to cities in large numbers, as has been observed
recently, is an unhealthy trend.
Decision I Firms in cities should be barred from hiring rural workers.
Decision II It should be compulsory for workers in cities to also work in rural areas for some
duration.
Decision III New schemes that can help generate employment in rural areas should be launched.
(A) Only I should be pursued
(B) Only II should be pursued
(C) Only I and III should be pursued
(D) All should be pursued
(E) Only III should be pursued

Solution

Forcing someone to work in rural areas or barring rural workers from working in cities is definitely
not a solution to the problem at hand. The solution is to try to understand why this migration is
taking place in the first place—most likely because of better employment opportunities in the cities.
So, the ideal solution should be to address this problem, which only III does.
The correct answer is E.

Let us take a look at another example:


Statement: The Anti-Corruption Bureau (ACB) has received a complaint that a police inspector
is asking for a bribe to close a case.
Decision I The ACB should wait for a few more complaints to come in before acting against the
police inspector.
Decision II The ACB should immediately take action and try to catch the police inspector red-
handed while taking the bribe.

(A) Only I should be pursued


(B) Only II should be pursued
(C) Either I or II should be pursued
(D) Neither I nor II should be pursued
(E) Both I and II should be pursued

Solution
One incident of bribery should be good enough for the ACB to act against the police inspector. There
is no reason why it should wait for more complaints to come in. Thus, Decision I is a not a logical
decision. Decision II, on the other hand, makes perfect sense as the ACB should try to catch the
inspector red-handed.
The correct answer is B.

Tips for solving questions on Decision Making and Course of Action


1. Make sure you arrive at an objective decision using only the facts given to you in the statement. Do
not let your own knowledge or preconceived notions about the subject matter affect your decision
making.

526

Book 1.indb 526 30/04/2019 4:49:20 PM


6.0  Logical Reasoning Review  6.2  Verbal Reasoning

2. In such questions, one must opt for the ideal solution. Therefore, if the choice is between practical and
ideal, choose ideal.
3. The correct answers always exhibit one, many or all of the following characteristics:

• They are ethical even if they involve a cost.


• They are also practical and not merely idealistic or ‘feel good’.
• They do NOT favour a particular side in a disputed situation.
• They are balanced and seek to provide solutions where required.
• Avoid an extreme choice at all costs.
•  ese questions always work on the premise of the greater good for the greatest number involved.
Th
This is true especially when one has to choose between the benefits to organisations/institutions
over individuals. An action benefitting a larger population is a desirable action even though it may
put an extra burden on the organisation/institutions or government.
• Always employ impartial and objective thinking.
• Make decisions in sync with the way the system and public departments function.
• Existing practices are not considered an effective course of action.
• Our decisions and views should be unbiased and neutral.
• Try to select the course of action that addresses the central cause of the problem.
• Extreme or strict action is not a valid course of action.
• The actions should always be a positive.
• The selected course of action should not aggravate the problem or create a new set of problems.
• I f in any situation, more than one course of actions are possible (but are dependent on each other),
then your answer should always be “both follow” and not “either of them follows”.
•  e course of action can be said to solve/reduce the problem when it is an established fact, when
Th
it is logically possible and when it is known from experience.
• Lastly, if there are dates or figures involved, those will certainly have a bearing on the answers.

6 Critical Reasoning
Critical reasoning is a process. It involves conceptualisation, analysis and application of objective, rational
reasoning in order to arrive at conclusions.
To be skilled in critical thinking is to be able to deconstruct one’s thinking into individual arguments,
analyse each one on its on merit, assess its validity and improve upon it. The first step in this process is
gaining an understanding of the elements of reasoning.
In critical reasoning, we are given an argument (a short paragraph) and asked to answer questions based on
it. Let us first define an argument.
Arguments constitute the basis of conversation. They are combinations of facts, data, information and
opinion that aim to influence the perspective of the other person.

527

Book 1.indb 527 30/04/2019 4:49:20 PM


NMAT by GMAC™ Official Guide 2019

Structure of an Argument
All arguments follow a structure, which may either be deliberate or discovered through analysis.
At its simplest, an argument is a simple set of three things:
I.  CONCLUSION
Is the final result which an argument arrives at. It is the statement/claim which you want to convince the
other person about. A conclusion is drawn from the premises. These act as the support for the conclusion
and reinforce the argument in its favor.
In the statement, ‘I can participate in the Miss India pageant because I am 6 feet ’, the part ‘I can participate
in the Miss India pageant’ is the conclusion or result.
A useful way of spotting a conclusion is to identify its construction. It can be presented as an advice or
strong recommendation, stating a preferable course of action. It may also be persuasive in its construction,
highlighting the desirability of a particular decision, as compared to another.
Identifying a conclusion:

•  ook for conclusion either at the beginning or at the end of the passage .However, please keep in
L
mind that the conclusion can occur anywhere in a paragraph. In fact, in rare cases it may not be
explicitly stated anywhere in the paragraph.
So, in order to identify the conclusion, ask yourself:

•  hat is the main point of this paragraph?


W
• W hat does the author want to prove?
The answer to these questions is the conclusion.
• Trigger words that signal a conclusion are:

Therefore Thus
So Hence
Implies Indicates
• Look for statements that cannot stand alone.

• Or, look for facts that are part of the argument.

The conclusion is the main reason for the paragraph to be written. It is proven with the help of some
premises.
Since the premise is stated by the author to be true, you cannot question the validity of the premise.
Recognizing the conclusion correctly is imperative and any errors in this process will most likely lead you to
select an inappropriate answer choice. So spend some time to identify the correct conclusion.

II.  FACTS
Facts are also referred to as premise in logic. By now, we can recognise the conclusion in a critical
reasoning paragraph. Typically, this conclusion is proven with some information that is provided with the
conclusion—this information are the premises. A premise (or premises) of an argument is something that
is presented as being true. Although it is not proven, its truth is assumed in the given scenario.

528

Book 1.indb 528 30/04/2019 4:49:20 PM


6.0  Logical Reasoning Review  6.2  Verbal Reasoning

Premises: The people in this city are mad.


I am 5 feet tall
Identifying premises:
Premise is generally preceded by words like
Because In view of
Since Given that

How do we spot these premises? Simple, once you remove the conclusion from the paragraph, the rest of
the sentences are the premises. Another way to look at the premises is to see that they provide the ‘why’ for
a conclusion, that is, why has the author come to this conclusion?
III.  ASSUMPTION
The third part of our argument. An assumption is an unstated premise that supports the conclusion. Both
premise and assumption are unquestionable facts but the assumption, unlike the premise, is not explicitly
stated and needs to be deciphered.
So where does the assumption come in? Most arguments are not complete logically: the gap in logic is the
assumption that the author makes to prove the conclusion.
Another way to spot the assumption is to think: When does this argument not make sense? If you can
complete the next sentence then you have your assumption. The conclusion does not make sense unless
______________ is true.
Therefore, an analysis of an argument consists of:

1. Ability to identify the premises and the conclusion which they reiterate.
2. Ability to spot the unstated links in the argument while moving from facts to conclusions or vice-
versa. It is these links that identify the underlying assumptions in the argument.
Critical Reasoning questions can be broadly divided into certain categories:

• Identify the assumption


• Identify the conclusion (Inference/ conclusion)
• Strengthen/Weaken the argument
• Miscellaneous question type

Critical Reasoning is a fairly common verbal question type in various Management Aptitude Tests (in
GMAT, it constitutes approximately 30% of the verbal section). One can enhance one's accuracy in this
section through extensive practice.

Structure of a Critical Reasoning Passage


As stated above, Critical Reasoning examines the reasoning skills of the test-taker. In particular, it evaluates
the comfort level of the test taker with:
• Logical argument construction

• Argument evaluation

• Ability to list down premises that can strengthen or weaken a stated argument

529

Book 1.indb 529 30/04/2019 4:49:20 PM


NMAT by GMAC™ Official Guide 2019

Critical Reasoning questions come in the form of a paragraph (this usually presents an argument), and a
question around the paragraph. Let’s look at a typical Critical Reasoning question.

Example 1

A recent survey on the causes of stress was conducted by ABC Ltd. They worked with a sample of 100
employees of a multinational corporation and found out that people with hair loss suffer from more
stress than people without hair loss. Hence, they concluded that hair loss is one of the causes of stress.
Which of the following, if true, would most weaken the argument given above?
(A) There are other causes like financial instability that may cause more stress than hair loss.
(B) Employees have been experiencing hair loss in recent times whereas they have been experiencing
the same levels of stress for a very long time.
(C) The study should also consider people who face no stress.
(D)
Stress may have a large number of factors that cause it.
(E)
Both (B) and (C).
The above example can be broken down into three components:
• The question stem
Which of the following, if true, would most weaken the argument given above?
• The background to solve the question
A recent survey on the causes of stress was conducted by ABC Ltd. They worked with a sample of
100 employees of a multinational corporation and found out that people with hair loss suffer from
more stress than people without hair loss. Hence, they concluded that hair loss is one of the causes
of stress.
• The answer choices
(A) There are other causes like financial instability that may cause more stress than hair loss.
(B) Employees have been experiencing hair loss in recent times whereas they have been
experiencing the same levels of stress for a very long time.
(C) The study should also consider people who face no stress.
(D) Stress may have a large number of factors that cause it.
(E) Both (B) and (C).

Solving Critical Reasoning Questions


Let us try and understand the approach for solving Critical Reasoning questions. The process is simple:
1. Read the question stem.
2. Read the paragraph and analyse it keeping the question in mind.
3. State what a good answer can look like.
4. Now look at the answer choices and eliminate the incorrect choices.

Step 1: Read the question stem.


‘Which of the following, if true, would most weaken the argument given above?’

530

Book 1.indb 530 30/04/2019 4:49:20 PM


6.0  Logical Reasoning Review  6.2  Verbal Reasoning

Step 2: Read the paragraph.


The question wants us to weaken the argument, that is, the conclusion of the paragraph. Hence
while reading, you have to spot the conclusion and be aware of the premises that validate the
conclusion. In this case, the conclusion is ‘hair loss is one of the causes of stress’. The reason—
because people with hair loss suffer from more stress.
Step 3: State what a good answer can look like.
What is wrong with this argument? Well, just because hair loss and stress occur together does not
mean that hair loss leads to stress. It could easily be stress that causes hair loss or they can be two
random events that just co-incidentally occur together.
Here is an interesting fact: In the USA, it was found that beer and diapers are usually bought
together in department stores. Using this fact, if I come to the conclusion that drinking beer leads
to loss of bladder control (hence, the diapers) or worse still, wearing diapers leads to a craving for
beer, then I am obviously wrong. So, just because two things (beer and diapers) occur together
does not imply that one causes the other. This could be one of the answers.
Step 4: Eliminate the incorrect answer choices.

(A) There are other causes like financial instability that may cause more stress than hair loss.
This just shows that there are other causes; it does not negate hair loss as a cause.
(B) Employees have been experiencing hair loss in recent times whereas they have been experiencing the
same levels of stress for a very long time.
If people are experiencing the same level of stress even before hair loss, then may be hair loss does not
cause the stress. Clearly if hair loss causes stress then with the advent of hair loss there would have at
least been an increase in stress.
(C) The study should also consider people who face no stress.
If I want to explore the causes for stress then studying people who have no stress will not be very
helpful.
(D) Stress may have a large number of factors that cause it.
Like the first option, this also just shows that there are other causes; it does not negate hair loss as a
cause.
(E) Both (B) and (C).
We have negated option (C).

The correct answer is option B.


Some observations:
• W hat we thought our answer can look like and what was the actual answer were two different
things. That’s fine! Forming an approximate answer ensures that you have understood and
imbibed the paragraph.
• The key to solving a Critical Reasoning question lies in the question stem. Hence, it is important
to read it first because the question stem will dictate how you will analyse the paragraph. This
obviously leads us to the analysis of question stem.
So, what kind of question stems can we come across and what kinds of analyses are possible?

531

Book 1.indb 531 30/04/2019 4:49:20 PM


NMAT by GMAC™ Official Guide 2019

Types of questions
1. Working with Assumption(s):
• Type 1: Find the assumption that the Author makes.
Identify the gap that will takes us from the premise to the conclusion.

• Type 2: Strengthen the Conclusion that the Author makes.


If the assumption that the author makes is true, then the conclusion will be strengthened.
• Type 3: Weaken the Conclusion that the Author makes.
If the assumption that the author makes is false, then the conclusion will be weakened.
• Type 4: Find the flaw in the Author’s reasoning.
Similar to weaken the conclusion—the assumption has to be false.
• Type 5: Evaluate the argument that the Author presents.
Information about the assumption i.e. whether the assumption is True or False will help us
evaluate the argument.
• Type 6: Explain the discrepancy or paradox in the argument.
What assumption or new evidence will help explain the unlikely conclusion?
2. Working with the Structure of the Argument
• Type 1: Describe the role that the phrases in bold play.
Understand the structure of the argument
• Type 2: Identify the reasoning.
Understand the structure of the argument
• Type 3: Parallel the reasoning.
Understand the structure of the argument and spot a similar structure in the option choices.
3. Working with the conclusion
• Type 1: Identify inference.
Given the argument, which of the following conclusions has to be true?
• Type 2: Resolve the Paradox.
Which of the following statements resolves the paradox?
As we can see most of the question types involve finding assumptions that the author makes to prove her
conclusion. What is this assumption and how do we find it, if it is not there in the text? Well, that’s what
we are going to do next. Once we get comfortable with identifying the assumption, we will look at the
other question types.

Assumption questions
As the name suggests, you will be given an argument in the question stem and you will need to identify the
assumption that is made in this argument from the five options given to you. Let us take a look at an example:

532

Book 1.indb 532 30/04/2019 4:49:21 PM


6.0  Logical Reasoning Review  6.2  Verbal Reasoning

Example 2

Over the last six years, most of the students in Tupac city have regularly attended colleges in the
neighbouring Mekon city to pursue their graduate degrees. However, according to a recent change in
the education policies of Mekon city, the colleges in Mekon city are expected to increase their fees to
almost the same level as those charged by colleges in Tupac city. Therefore, it can be safely concluded
that colleges in Tupac city will see a surge in the number of students enrolling with them to pursue
their graduate degrees.
Which of the following is an assumption on which the argument depends?
(A) The teachers at colleges in Mekon city are generally considered far superior to those at colleges in
Tupac city.
(B) Tupac city does not have good quality colleges.
(C) The low fees charged by colleges at Mekon city is the primary reason why students from Tupac city
move to these colleges.
(D) Students who study at colleges in Tupac city do not perform better than those who study at colleges
in Mekon city.
(E) Mekon city does not have more colleges than Tupac city.

Solution

Always start an assumption question by paraphrasing the conclusion and the evidence.
• Conclusion (What is the author saying?)—that there will be a surge in the enrolments at
colleges in Tupac city.
•  vidence (Why is the author saying this?)—because students in Tupac city who earlier used to
E
move to Mekon city to pursue their graduate degrees will now not do so as the colleges in Mekon
city will charge them the same fees as the colleges in Tupac city do.

Note that that conclusion is an opinion of the author but the evidence is a fact because the
colleges in Mekon city are definitely looking at increasing their fees.
•  ssumption—The author must be assuming that the low fees charged by colleges in Mekon city
A
is the single most important factor why students from Tupac city have been moving to colleges in
Mekon city. If we don’t assume this, the argument will fall apart. Thus, C is the correct answer.

Strengthen and weaken the argument questions


Strengthening an Argument
The key to strengthening an argument is finding the answer choice that reinforces the premises or the
central assumption in some way. This can be done in two ways-
• First, the assumption might be rephrased and presented as an answer choice. In this case, it
becomes a strong argument.
• Second, the correct answer can be a validation of the assumption through the citing of a study,
survey or any other relevant piece of additional evidence that makes the assumption more likely to
be true.
Essentially, any information that fills the gap present in the logic of the argument with extra information
(that supports the conclusion made in the argument) strengthens it.

533

Book 1.indb 533 30/04/2019 4:49:21 PM


NMAT by GMAC™ Official Guide 2019

Weakening an Argument
Finding a statement that weakens an argument follows the same process, only backwards.
• First, any statement that rebuts the assumption shall weaken the argument.
• Second, Data, information, reasoning, facts and so on that disprove or challenge the assumptions
or premises used in the argument shall weaken it.
In these questions, an argument will be given to you and you will need to select from five options the one
option which strengthens or weakens the argument (depending on what the question asks you to do). Let’s
take a look at an example of each:

Example 3

Of all the laptops available for sale in Ireland, those manufactured by Ivy Infotech must have the
fastest processors. Over the last six months, Ivy Infotech has sold three times as many laptops as its
closest competitor. Additionally, Ivy Infotech’s order books are full for the next 12 months.
Which of the following options, if true, most strengthens the argument?
(A) Ivy Infotech is the oldest manufacturer of laptops in Ireland.
(B) Ivy Infotech has the largest market share in laptop sales in Ireland for the past five years.
(C) 
Ivy Infotech sources its processors from the company which is the world’s biggest manufacturer of
laptop processors.
(D) All the laptops available for sale in Ireland are the same in every aspect, except for their processors.
(E) 
D ue to production bottlenecks, the production of laptops by Ivy Infotech’s rival companies fell by
more than 60% in the last six months.

Solution

Since this is a strengthen question, you know that it will be worded in the form of an argument. So
the first step is to identify the conclusion and the evidence of this argument.
• Conclusion (What is the argument stating?)—Laptops manufactured by Ivy Infotech have the
fastest processors.
• Evidence (Why is the argument stating this?)—Because Ivy Infotech has sold the maximum
number of laptops in the last six months in Ireland.
But does this make sense? Can’t there be some other plausible reason why the people in Ireland
are buying Ivy Infotech’s laptops?
• Maybe these laptops have a very sleek design, maybe they have a very long battery life, or maybe
they are the cheapest laptops in the market. There can be several other reasons (other than fast
processors) why the Irish are buying laptops manufactured by Ivy Infotech. This brings us to the
assumption. Remember that since the stimulus is in the form of an argument, it must contain an
assumption.
• Assumption (the unstated evidence)—So let’s try to predict the assumption. It will be something
along the lines of ‘the only difference among the different laptops available in Ireland is the speed
of the processor; the laptops are the same in every other aspect’, because then if the people are still
buying Ivy Infotech’s laptops these laptops must have the fastest processors, else people would be
buying some other company’s laptops.

534

Book 1.indb 534 30/04/2019 4:49:21 PM


6.0  Logical Reasoning Review  6.2  Verbal Reasoning

•  trengthener—So now that we have identified the conclusion, the evidence, and the assumption,
S
the option that tells us that the assumption is true has to strengthen the argument. (D) does this
best and should be the correct answer.
• Weakener—Similarly, the option that best tells us that the assumption may NOT be true has to
weaken the argument. In essence, this option will provide us with some other reason (other than
faster processors) why the sales of Ivy Infotech’s laptops have been very high. (E) does this best as
it gives us an alternative reason why people might be buying Ivy Infotech laptops—because the
laptops of other brands are in short supply—and not because Ivy Infotech laptops have the fastest
processors.

Inference questions
Inference questions will ask you to infer or conclude something from the information given in the passage.
The literal meaning of infer is to conclude something without it being explicitly mentioned. This is exactly
what you are required to do on an inference question—arrive at an option that is not explicitly stated in the
Passage but that can easily be concluded given the information in it.

Inference vs Assumption
An inference is similar to a conclusion that can be drawn based on one or more elements of the statement.
An inference must be true based on something that you read. An assumption is a hidden but necessary
piece of evidence. An assumption is something that must be true in order for the argument to be complete
and valid.
Inference questions are diametrically opposite to assumption questions. In inference questions, you have
to conclude a statement that has to be true. It is not ‘perhaps’, ‘may be’, or ‘most likely’ true, it has to be
true. Usually, these inferences are a small step away from the conclusion and in some rare cases it can be an
assumption that has to be true. A good trick in these questions is to be biased towards options that use mild
words such as perhaps, likely, and so on instead of all, definitely, and so on.

Example 4

Last month three automobile manufacturers—Honda, Toyota and Suzuki—launched new models of
their respective sedans in Japan. The three models that were launched were similarly priced and had
similar features. However the sales of Suzuki’s new sedan have been far lower than those of Honda
and Toyota’s new sedans in the last month.
The statements above best support which of the following as a conclusion?
(A) The people in Japan prefer cars manufactured by Toyota and Honda to those manufactured by
Suzuki.
(B) The people in Japan do not like cars manufactured by Suzuki.
(C) Cars manufactured by Suzuki are notorious for their flimsy build quality and poor fuel efficiency.
(D) In the last month, Suzuki has earned less revenue from the sale of its new sedan than Honda and
Toyota.
(E) Suzuki manufactures its cars primarily for the export market and not for domestic sale.

Solution

Do remember that the stimulus of an Inference question may not necessarily be in the form of an
argument. In fact, most often the stimulus will contain a set of facts like the one above. All that the
above stimulus tells us is that Honda, Toyota and Suzuki have each launched a new sedan last month

535

Book 1.indb 535 30/04/2019 4:49:21 PM


NMAT by GMAC™ Official Guide 2019

and that the sales of Honda and Toyota’s new sedans have been far greater than the sales of Suzuki’s
new sedan. These are all facts, and the author does not provide any conclusion on the basis of these
facts.
With this background, let’s go through each of the options in the above question and see if we can
arrive at the correct answer:
(A) This seems very obvious but such inferences must be avoided. For all you know Japanese people
might actually prefer Suzuki cars the most but due to some supply constraints Suzuki’s new sedan
may not be available in the market.
(B) The earlier explanation applies to this option as well. In fact this option goes a step further by
concluding that Japanese people do not like Suzuki cars at all. Since this may or may not be the
case, this cannot be the correct answer.
(C) This looks very logical because it provides a very convincing reason why the sales of Suzuki’s cars
have been so low. But is that what we are supposed to do? Absolutely not. So this option does not
even come close to being an Inference. It merely explains why the sales of Suzuki’s new sedan may
have been low but that is not what we are required to do in the argument.
(D) The Correct answer. The argument tells us that the three new sedans are similarly priced. Then if
Suzuki has sold fewer cars (and by a large margin) than Honda and Toyota, its revenues from the
sale of this new sedan have to be lower than those of Honda and Toyota. This option must be true
in all cases and hence has to be the correct answer.
(E) Like option C, this option again provides a logical explanation for Suzuki’s low sales in Japan but
this may or may not be the case. In any case we are not required to provide an explanation in the
first place. So this cannot be a valid inference.

Miscellaneous Question Types


Explain the Paradox

Example 5
Kamlesh: It is quite surprising that junk food consumption has increased by 20% from last year.
Kavita: What is so surprising about that? As it was last year, junk food is still popular.
Kamlesh: The Government of India has been putting in sustained efforts to publicise the harmful
effects of junk food since last year. So I expected junk food consumption to go down.
Which of the following responses can Kavita make to resolve the paradox?
(A) Junk food consumption has steadily increased every year.
(B) Kamlesh and Kavita rarely eat junk food.
(C) Currently, the youth in the country are very health conscious and regularly watch what they eat.
(D) Most people eat junk food because it is served quickly and can be eaten while travelling.
(E) Both (B) and (D)

Solution

Step 1: Read the question stem.


Which of the following responses can Kavita make to resolve the paradox? Resolving paradoxes are
very similar to ‘Weakening the Argument’ questions. Think about it. The paradox exists because the
author has made some conclusion that the premise does not support.

536

Book 1.indb 536 30/04/2019 4:49:21 PM


6.0  Logical Reasoning Review  6.2  Verbal Reasoning

Step 2: As usual, we will focus on Conclusion-Premise-Assumption.


Remember that you now need to state the assumption negatively to weaken the argument.
Conclusion / What?: It is quite surprising that junk food consumption has increased by 20% from last
year.
Premise / Why?: The government of India has been putting in sustained efforts to publicise the
harmful effects of junk food since last year.
Step 3: State what a good answer can look like.
Assumption: Kamlesh is assuming that just because something has been deemed unhealthy, people
will stop eating it. We need to negate this assumption, we can do this by introducing another cause.
Step 4: Eliminate the incorrect answer choices.
(A)
Junk food consumption has steadily increased every year.
Increase in junk food do not explain why junk food consumption is rising despite campaigns to
discourage junk food consumption.
(B)
Kamlesh and Kavita rarely eat junk food.
What Kamlesh and Kavita do in their personal life cannot explain aggregate figures for junk food
consumption.
(C)
Currently, the youth in the country are very health conscious and regularly watch what they eat.
This actually intensifies the paradox because if people are health conscious then they will reduce
their junk food consumption.
(D)
Most people eat junk food because it is served quickly and can be eaten while travelling.
This introduces a new reason as to why people prefer junk food. Hence it negates the assumption
that Kamlesh makes and helps in explaining the paradox.
(E)
Both (B) and (D)
Option (D) provides a plausible resolution of the paradox.

The correct answer is option D.

Identify the Reasoning

Example 6

Project Manager: I find it very interesting that there is a debate on gender discrimination for
salaries. In our team, the two most highly paid members are both women. If there is any gender
discrimination, then I must say, it is definitely against males.
Human Resource (HR) Manager: Well! Your observation is based on a very small sample.
Gender discrimination discussions are based on a large dataset. The women in your team may be
receiving higher wages due to their educational background, their work experience or some special
programming skills that they bring to the team. Frankly, I find your argument a little facetious.
Which of the following techniques does the HR manager use while attempting to refute the
project manager’s argument?

(A) Negating the authenticity of the data that the Project Manager is using.
(B) Introducing other criteria besides gender that could have led to the higher salary.

537

Book 1.indb 537 30/04/2019 4:49:21 PM


NMAT by GMAC™ Official Guide 2019

(C) Presenting evidence of gender discrimination against females.


(D) Mentioning that the sample size used by the Project Manager is unacceptable as the basis of any
conclusion.
(E) Both (B) and (D)

Solution

Step 1: Read the question stem.


Which of the following techniques does the HR manager use in attempting to refute the project
manager’s argument?
Here we need to focus on what the HR manager is saying and his / her conclusion along with what
the Project Manager is saying and his / her conclusion.
Step 2: Focus on the premise and conclusion for both the managers.
Project Manager
Conclusion / What?: Gender Discrimination is not a valid argument.
Premise / Why?: In our team, the two most highly paid members are both women.
HR Manager
Conclusion / What: Frankly, I find your argument a little facetious.
Premise / Why?: Well! Your observation is based on a very small sample. Gender discrimination
discussions are based on a large dataset.
Step 3: State what a good answer can look like.
The project manager has concluded against gender discrimination using a small sample size. To refute
this, the HR manager mentions that gender discrimination typically uses a large database for its
finding.
Step 4: Eliminate the incorrect answer choices.
This is mostly straightforward though some of you may find option (B) also to be a good answer.
Remember the conclusions are regarding gender discrimination; they are not about explaining why
the two women have a higher salary. Hence option (B) is incorrect. Option (D) clearly mentions the
sample size as insufficient hence, it is the answer.
The correct answer is option D.

Parallel the Reasoning

Example 7

The fear of contracting swine flu is very high in Asian countries. If only these people could compare
the number of people who actually contract swine flu (very few) with the number of people who
actually contract the common cold (high),then this fear would be assuaged.
Which of the following is most closely similar to the reasoning used in the argument above?
(A) I do not understand why people fear cockroaches even if they are numerous; the real danger
lies with tigers even though they are few.

538

Book 1.indb 538 30/04/2019 4:49:21 PM


6.0  Logical Reasoning Review  6.2  Verbal Reasoning

(B) I do not understand why people fear cockroaches even if they are numerous; the real danger
lies with tigers because they are too numerous to count.
(C) I do not understand why people fear cockroaches, they are very few; the real danger lies with
tigers because they are too numerous to count.
(D) I do not understand why people fear cockroaches, they are very few; the real danger lies with
tigers because though they are few they can hide in the unlikeliest of places.
(E) I do not understand why people fear cockroaches even if they are few; the real danger lies with
tigers as they are few.

Solution

Step 1: Read the question stem.


Which of the following is most similar to the reasoning used in the argument above?
Pay close attention to how the original argument is structured, you will have to replicate the same
structure in the answer.
Step 2: Find the structure of the original argument.
The original argument states that you should not fear something because it occurs rarely, instead you
should fear the things that occur a greater number of times.
Step 3: State what a good answer can look like.
The fear for tigers is very high in Asian countries. If only these people could contrast the number of
tigers (very few) with the number of cockroaches (high) then this fear would be assuaged.
Of course, we should not have pre-conceived notions about tigers and cockroaches. Hence the answer
can easily look like this:
The fear for cockroaches is very high in Asian countries. If only these people could contrast the
number of cockroaches (very few) with the number of tigers (high) then this fear would be assuaged.
Step 4: Eliminate the incorrect answer choices.
From the above step, the answer clearly has to be (C).
The correct answer is option C.

Tips For Solving Questions On Critical Reasoning


1) Start by reading the question stem.
Doing this shall allow you to identify the type of question (Assumption, Strengthen, Weaken,
Inference, and so on). This shall let you categorise the different elements of the argument structure, for
example, the premises, the assumption and the conclusion.
2) Try to predict the answer before looking at the options.
It is not necessary to have a detailed solution, but a generic statement or a broad outline/framework
will help point you in the right direction by eliminating choices.
3) Analyse the given answer choices.
Carefully read through all 5 answer choices. While doing so, compare these choices with your
prediction. There will be certain choices that can be eliminated upfront – they will either be beyond

539

Book 1.indb 539 30/04/2019 4:49:21 PM


NMAT by GMAC™ Official Guide 2019

the scope of the argument, too narrow in their construction or too vague and so on. Once you have
eliminated 2-3 options through this process, the remaining 2-3 options can then be evaluated on the
basis of their correctness.
4) Avoid options that are extreme or exaggerated in their construction.
For an option to be the correct answer, it must always hold true within the context of the argument.
However, one should be cautious of modifiers that amplify one aspect of the premise or make
overstatements. They usually signify an incorrect answer. For example, if the argument states that
“certain phones made by the company were defective”, then an answer choice that claims that “most
of the phones made by the company were defective.” Will be incorrect. Also, beware of extreme words
like “always”, “only’, “never”, “none” and so on. Usually, these options will be incorrect.
5) Be conscious of a change in the scope of the argument.
Scope refers to the specific aspects of a topic. For example, the topic of an RC passage may be
“Bollywood” and its scope may be “the changing face of the female lead over the years”. Thus, the
scope is a more specific, narrower delineation of the topic. In critical reasoning questions, it is
extremely important to identify those answer choices that shift the scope of the argument, albeit
slightly.
For example, the passage may present a study that claims that there has been an increase in the
natural disasters plaguing the state of Uttarakhand. In this case, an answer choice that talks about the
increased number of earthquakes in the state, though looking tempting, will actually be incorrect. This
is because natural calamities are not restricted to earthquakes only. This alters the scope of the initial
argument .

540

Book 1.indb 540 30/04/2019 4:49:21 PM


Book 1.indb 541 30/04/2019 4:49:21 PM
6.5  Analytical Reasoning

542

Book 1.indb 542 30/04/2019 4:49:21 PM


6.0  Logical Reasoning Review  6.5  Analytical Reasoning

6.6 What is Measured?


The analytical reasoning section will include arrangement questions, syllogisms, codes and series, etc., and
will test you more on your lateral thinking and visualisation skills.
This section is, to a large extent, testing your ability to make sense of incomplete or abstract text and images.

6.7 Overall Test Taking Strategies


• While attempting arrangement questions, focus on the overall arrangement and not on individual
questions because once you get the arrangement right, the individual questions can be easily answered.
• Try to represent the given information pictorially or in the form of a Venn diagram.
• While attempting direction questions, remember the rules with regards to the direction in which
a shadow falls at different times of the day.
• In symbols-based problems, try to focus on what is within each shape and not just on the shape.

The next few sections will provide you with in-depth strategies for approaching each topic.

543

Book 1.indb 543 30/04/2019 4:49:21 PM


NMAT by GMAC™ Official Guide 2019

Introduction
Unlike Verbal Reasoning, Analytical Reasoning questions will test you more on your logical and lateral
thinking skills. You will need to make sense of random images, decode abstract text, understand symbols
and so on.
Analytical Reasoning will include the following question types.

1 Linear and Circular Arrangements


Arrangement questions will require you to arrange a given set of people or objects in straight lines or in a
circular manner. There will be a list of requirements or constraints given to you and you will need to make
this arrangement keeping in mind these requirements.

Linear Arrangement
Any arrangement which has a well-defined beginning and end is a linear arrangement. It may be of three
types:
1. To arrange people or objects in a straight line (row arrangement)
2. To arrange events etc. on basis of time (chronological)
3. To arrange people in other ways (like on basis of age, position, height etc.)
Let us take a look at an example:
Directions for examples 1–4: Read the following information and answer the questions that follow.

Ajay’s work requires him to travel to cities C, R, G, J and B (each city for one day and not necessarily in this
order) from Monday to Saturday, with a one-day break in the middle.
i. He visits G neither on the first nor on the last day but he visits it earlier than R.
ii. He visits J the day after he visits R.
iii. He visits C immediately before the rest day.
iv. J and B had a two day gap between their visits.
v. He visited B immediately after the rest day.
Example 1

Which of these is the rest day?


(A) Monday
(B) Tuesday
(C) Wednesday
(D) Thursday
(E) Friday
Example 2
Between Monday and Saturday, the visits to J and C have how long a gap between them?
(A) One day
(B) Two days
(C) Three days

544

Book 1.indb 544 30/04/2019 4:49:21 PM


6.0  Logical Reasoning Review  6.5  Analytical Reasoning

(D) Four days


(E) Five days
Example 3
On which day does Ajay visit B?
(A) Monday
(B) Tuesday
(C) Wednesday
(D) Thursday
(E) Friday
Example 4
Which of the following is a correct statement?
(A) Ajay visits B on Thursday
(B) Ajay visits J the day after he visits R
(C) Ajay visits C and B on consecutive days
(D) R is the last city that Ajay visits in the week
(E) The rest day is right in the middle of the week

Solution

We first need to decipher the days on which Ajay visits each of the five cities, as after doing so all the
questions can be answered easily.
Start by looking at which cities Ajay can visit on Monday.
• G is out as it cannot be the first or the last city Ajay visits.
• Since R has to come before J, J cannot be the first city.
• B cannot be the first city since it comes after the rest day.
• J is out since Ajay visits J after he visits R.
• Thus, C has to be the city that Ajay visits on Monday.

As per the instructions in the question, C has to be followed by a rest day, which is in turn followed by
B. So, we get:
Monday—C
Tuesday—Rest day
Wednesday—B
Thursday—?
Friday—?
Saturday—?

Since J and B have a two-day gap between their visits, we get:


Monday—C
Tuesday—Rest day
Wednesday—B
Thursday—?

545

Book 1.indb 545 30/04/2019 4:49:21 PM


NMAT by GMAC™ Official Guide 2019

Friday—?
Saturday—J

Since G has to come before R, the final order we get is:


Monday—C
Tuesday—Rest day
Wednesday—B
Thursday—G
Friday—R
Saturday—J

Using the above arrangement, the questions can now be easily answered.
1. (B)
2. (D)
3. (C)
4. (B)

Important Learning:  The first step in Arrangement questions is always to figure out the correct
placement of all the entities. The individual questions can be easily answered after that.

Circular Arrangement
Unlike Linear Arrangement, wherein every position is unique by itself, in Circular Arrangement questions,
all the positions are identical. Therefore, in Circular Arrangement questions, the position of an individual
depends on the neighbors to the left and right.
Let us now look at an example of a circular arrangement question:
Directions for examples 5-8: Read the following information and answer the questions that follow.

P, Q, R, S, T, U, V and W are sitting around a circular table.


i. P is second to the right of T who is the neighbour of R and V.
ii. S is not the neighbour of P.
iii. V is the neighbour of U.
iv. Q is not between S and W. W is not between U and S. S is not between W and Q.

Example 5
Who is sitting to the right of U?
(A) P
(B) U
(C) R
(D) V
(E) Q

546

Book 1.indb 546 30/04/2019 4:49:22 PM


6.0  Logical Reasoning Review  6.5  Analytical Reasoning

Example 6
How many people are sitting between R and Q in a clockwise direction?
(A) 0
(B) 1
(C) 2
(D) 4
(E) 5
Example 7
Who is sitting to the left of S?
(A) P
(B) Q
(C) W
(D) R
(E) T
Example 8
Which of the following is the correct sentence?
(A) T and S are neighbours
(B) W and T have exactly three people between them
(C) T and S have exactly two people between them
(D) P and S are neighbours
(E) W and S are not neighbours

Solution

Let us first try to put all the eight people around a circular table based on the given instructions.
Using the information in Statement (i), we get:

R/V
R/V

Note:  Do not get confused with the lefts and rights. The right of T will be your left because all the
people are sitting facing the table.

547

Book 1.indb 547 30/04/2019 4:49:22 PM


NMAT by GMAC™ Official Guide 2019

Using the information in Statement (ii) we get:

R/V
R /V

P S?

S?

S?

Using the information in Statement (iii) we get:


T

R
V

P U

S?

S?

Using the information in Statement (iv), we finally get:


T

R
V

P U

S
Q

548

Book 1.indb 548 30/04/2019 4:49:23 PM


6.0  Logical Reasoning Review  6.5  Analytical Reasoning

Using the above arrangement, we can now easily answer the questions.
5. (D)
6. (E)
7. (C)
8. (B) Note that irrespective of whether you go in the clockwise direction or in the anti-clockwise
direction, the answer remains 3. However, the answer changes in the case of option (C), which is
why (C) is incorrect as we do not know which direction to move in.

Tips for approaching arrangement questions


1. The trick to solving Arrangement questions is to identify a person or an entity for whom more than
one reference point is given in the question (e.g. J in the question we attempted above), and use this
information to freeze this person’s position. The other positions can then be fixed using this as a
reference point.
2. It helps if you can visualise the correct arrangement or pattern in your head. In fact, it is even better if
you can draw this pattern on paper to avoid confusion later.
3. Move to the answer choices only after you have read and understood the problem completely.
4. The problem has to be solved only on the basis of the given information. Do not assume any
information which does not follow from the instructions in the question.
5. Watch out for words such as all, some, none, only, unless and so on, since questions will be based on the
meaning conveyed by these words.

2 Syllogisms
A Syllogism is a logical proposition wherein a conclusion is arrived at using two or more quantified
statements. You will typically be asked to determine whether this conclusion can logically be arrived at
from the given statements or not. The primary difference between Syllogism questions and, say, Inference
questions is that Syllogism questions will always use quantifiable terms such as ‘all’, ‘none’ and ‘some’.
In Syllogism questions on the NMAT by GMAC™, you will be given two or more statements that will
be followed by two or more conclusions.
Your answer choices will be as follows:
(A) Only conclusion I follows
(B) Only conclusion II follows
(C) Either conclusion I or II follows
(D) Neither I nor II follows
(E) Both I and II follow

Let’s look at certain concepts of Syllogism before approaching an actual Syllogism question:
i. Proposition: Every syllogism question will contain two or more propositions. A proposition is
nothing but a statement providing a relation between two terms. For example, All men are mortals is a
proposition as it provides the relation between the terms men and mortal.
ii. Use of terms such as Some and All: Almost every proposition will involve the use of words such as
some, all and no/none. If a proposition says All pens are pencils, one can conclude that there will not be
a single pen that is not a pencil. If a proposition says Some pens are pencils, then there could be some
pens that are not pencils though it is not necessary. If a proposition says No pen is a pencil then there is
not a single pen that is a pencil.
549

Book 1.indb 549 30/04/2019 4:49:23 PM


NMAT by GMAC™ Official Guide 2019

iii. Use of the term possibility: Sometimes, on Syllogism questions, you will come across the term
possibility in one or more of the given conclusions. Possibility is an interesting term because it may
or may not be true. For example, if the two propositions say Some roses are violets and Some violets
are jasmine, then some roses could be jasmine. However, we cannot say this for sure because it is also
possible that no rose is jasmine. In such cases, the conclusion Some roses being jasmine is a possibility is
true because possibility does not mean certainty and there is definitely a possibility that some roses
could be jasmine. You will typically see possibility being used in the answer choices when one or more
of the propositions contain the word some.
iv. Answer choices on a Syllogism question: In a typical Syllogism question, you will be given two
statements (propositions), which will be followed by two conclusions. You will need to determine
which of these two conclusions logically follows from the given statements. The answer choices will
look like this:
(A) Only I follows
(B) Only II follows
(C) Only I and III follow
(D) All follow
(E) Only III follows
Note that the order of these answer choices might change but the wording of the five options will remain
the same. While the remaining answer choices are self-explanatory, answer choice (C) Only I and III follow
can be confusing at times. Usually, the correct answer will be from one of the other four answer choices.
However, there is one situation in which (C) can be correct—Complementary pairs. Complementary pairs
are two conclusions one of which must be true. For example, the conclusions Some dogs are cats and No dog is
a cat are a complementary pair because one of the conclusions has to be true—either some dogs will be cats
or no dog will be a cat. In such cases, the correct answer will be option (C).
v. Use of Venn diagrams to solve Syllogism questions
You can actually make use of Venn diagrams to answer Syllogism questions because representing the
information given in the statements pictorially makes it less confusing.
There are four major propositions in Syllogisms that can be represented using Venn diagrams. In all
these propositions we will be referring to two entities—A and B—representing each by a circle.
i) All A are B – There are two ways of representing this:
a) The circle for A is completely within B.

550

Book 1.indb 550 30/04/2019 4:49:23 PM


6.0  Logical Reasoning Review  6.5  Analytical Reasoning

b) The circle for A and B is the same, that is, all B are also equal to A.

As you can see from the above diagrams, there could be some B that are not A as shown in the first figure.
So, all A are B does not necessarily mean that All B are A.
ii) No A is B – This is pretty simple. You simply draw two circles representing A and B with no overlap
between them as shown below.

A B

iii) Some A are B – In this case we draw two circles with some intersecting part that represents the A
which are also B. In the below figure, Number 1 represents the part where Some A are B.

A 1 B

The information can also be represented in the below manner, with Number 2 representing the part where
Some A are B.
A

551

Book 1.indb 551 30/04/2019 4:49:23 PM


NMAT by GMAC™ Official Guide 2019

iv) Some A are not B – The Venn diagram for this proposition will look exactly the same as for the
previous proposition—Some A are B. Just the region representing the proposition changes. In the
below figure, Number 3 represents the A that are not B.

A B

The information can also be represented in the below manner, with Number 4 representing the part where
Some A are not B.
A

B 4

The above four Venn diagrams give you all possible ways of depicting propositions tested on Syllogism
questions. However, note that on an actual Syllogism question, you will have two or more propositions
(statements) given to you involving three or more entities. Thus, you will always end up making at least
three circles.
Let us take a look at an example:
Example 1
Statements: All dogs are animals. All animals are mortal.
Conclusion I All animals are dogs.
Conclusion II All dogs are mortal.
(A) Only I follows
(B) Only II follows
(C) Only I and III follow
(D) All follow
(E) Only III follows
Solution

All dogs are animals, but this does not mean that all animals are dogs. There can also be other animals
such as cats, horses and so on. Thus, Conclusion I is invalid. Conclusion II, on the other hand, is
perfectly valid because if all dogs are animals and all animals are mortals, then all dogs must be mortals.
The correct answer is B.
552

Book 1.indb 552 30/04/2019 4:49:23 PM


6.0  Logical Reasoning Review  6.5  Analytical Reasoning

It may be useful to draw Venn diagrams while answering syllogism questions. For example, in the above
question, the answer becomes immediately obvious as soon as you make a Venn diagram, using the
information given in the two statements, as shown in the following figure.

Mortals

Animals

Dogs

Representation of the above question in the form of a Venn diagram

Important Learning:  If you come across a confusing syllogism question, try to represent the given
information in the form of a Venn diagram and you will easily be able to arrive at the answer.

Let us take a look at another example:


Example 2

Statements: All capitalists are rich. No Indians are capitalists.


Conclusion I No Indians are rich.
Conclusion II Some capitalists are Indian.

(A) Only conclusion I follows


(B) Only conclusion II follows
(C) Either conclusion I or II follows
(D) Neither I nor II follows
(E) Both I and II follow
Solution

The given statements never say that only the capitalists are rich. So, it is possible for Indians to be
rich and yet not be capitalists. Thus, Conclusion I is invalid. Conclusion II clearly contradicts the fact
given in the second statement. If no Indians are capitalists, then there is no way some capitalists can
be Indians. Thus, Conclusion II is also invalid.
The correct answer is D.
You can also be tested on Syllogism questions with more than two statements and more than two
conclusions.

Let us take a look at an example:


Example 3

Statements:
(1) All the wires are cords.
(2) All the cords are cables.
(3) Some threads are cables.

553

Book 1.indb 553 30/04/2019 4:49:24 PM


NMAT by GMAC™ Official Guide 2019

Conclusion I Some cables are wires.


Conclusion II Some threads are cords.
Conclusion III All cords are wires.

(A) Only I and II follow

(B) Only I follows

(C) Only II and III follow

(D) All the conclusions follow

(E) None of the conclusions follow


Solution

If all wires are cords and all cords are cables, then all wires are cables. So, some cables must be wires.
Thus Conclusion I is correct.
All cords are cables does not mean that all cables are cords. Thus, there may be cables that are not
cords and it could be these cables that the threads overlap with. So we cannot say for sure that some
threads are cords. Thus, Conclusion II is not correct.
Similarly, all wires are cords does not mean that all cords are wires. Thus, Conclusion III is also not
correct.
The correct answer is B.

3 Classification Questions
On Classification questions, you will be given five words/numbers/letters with some relations among three
of them. You need to identify the word/number/letter that does not belong to this group. For this reason,
these questions are also called spot the odd one out questions.
Some aspects on which classification questions are designed are:
• Synonyms: In this category, all options except the odd one have similar meaning. The odd word may
appear completely irrelevant when compared to the other options.
• Odd-even: In this category, all the options except the odd one are either even digits or odd digits.
• Name of subject: In this category, all the options except the odd one belong to a similar field of study.
• Class or category: In this, all options except the odd one belong to same class or category.
• Place: In this, all options except the odd one denote similar type of places.
• Disease: In this, all options except the odd one denote similar types of diseases.

Usually, the Classification and Odd One Out sections consist of a group of topic-specific words, given in
a set of five. Four groups are similar to each other in a certain manner, while one does not share the same
characteristics as others. Thus, it provides a reason for being an odd entity. The candidate needs to classify
the similar items and find the odd one out.

554

Book 1.indb 554 30/04/2019 4:49:24 PM


6.0  Logical Reasoning Review  6.5  Analytical Reasoning

Let us understand the concept of classification by the following illustrations:


Example 1

Four of the following are alike and form a group. Which one does not belong to this group?
(A) Hen
(B) Crocodile
(C) Lion
(D) Eagle
(E) Lizard

Solution
Lion is the only mammal in the group.
The correct answer is C.

You can also get Classification questions based on meaningless letters. On such questions, look out for
presence or absence of vowels from some of the combinations of letters.
Example 2

Four of the following are alike and form a group. Which one does not belong to this group?
(A) TYP
(B) BCD
(C) MNO
(D) CJK
(E) YLP

Solution
None of the other options contains a vowel.
The correct answer is C.

You can also get Classification questions based on numbers. On such questions, look out for odd and even
numbers, prime numbers and divisibility rules for common integers.
Example 3

Four of the following are alike and form a group. Which one does not belong to this group?
(A) 123
(B) 453
(C) 625
(D) 675
(E) 870
Solution

All other numbers are divisible by 3.


The correct answer is C.

555

Book 1.indb 555 30/04/2019 4:49:24 PM


NMAT by GMAC™ Official Guide 2019

4 Blood Relation Questions


Blood relation specifies the relation between two or more persons. Questions on blood relation will give you
a list of relations among a group of people. You will be required to analyse these relations and identify how
each of the members in the group is related to the others. You will then use this solution set to select the
answers for the given questions.
To get success in the test, you should be familiar with all blood relations. The relations from the mother’s
side are known as maternal relations while the relations from the father’s side are called paternal relations.
For example, the brother of father is called uncle or paternal uncle, while the mother’s brother is called
uncle or maternal uncle.
You should remember the following relations to solve these types of problems:

Reference Relation
Father’s mother Grandmother
Father’s father Grandfather
Mother’s father
Maternal grandfather
Maternal uncle’s father
Mother’s mother
Maternal grandmother
Maternal uncle’s mother
Grandfather’s or grandmother’s son Father or uncle
Grandfather’s or Grandmother’s only son Father
Mother’s or Father’s brother Uncle
Father’s or Mother’s sister
Aunt
Uncle’s wife
Maternal uncle’s only sister
Mother
Father’s wife
Son’s wife Daughter-in-law
Daughter’s husband Son-in-law
Husband’s sister
Wife’s sister
Sister-in-law
Brother’s wife
Husband’s brother’s wife
Sister’s husband
Brother-in-law
Husband’s brother
Wife’s or Husband’s father Father-in-law
Wife’s or Husband’s mother Mother-in-law
Brother’s or Sister’s son Nephew

556

Book 1.indb 556 30/04/2019 4:49:24 PM


6.0  Logical Reasoning Review  6.5  Analytical Reasoning

Reference Relation
Brother’s or Sister’s daughter Niece
Uncle’s or aunt’s son
Cousin
Uncle’s or aunt’s daughter
Grandson’s or Granddaughter’s daughter Great granddaughter
Grandson’s or Granddaughter’s son Great grandson

Commonly three types of questions are asked on the Blood Relations topic. Let’s understand each of these.

Type 1: Based on Conversation


In these types of questions, a conversation or a dialogue is made pointing to a person. You need to decipher
the statement to conclude the relationship between the concerned persons. It can be better understood by
the following examples:
Example 1

Pointing to a girl, Rajesh said, ‘She is the daughter of the only son of my mother’. How is Rajesh
related to that girl?
(A) Uncle
(B) Father
(C) Brother
(D) Cousin
(E) Grandfather

Solution

If Rajesh’s mother has only one son, then that son has to be Rajesh himself. So, the girl must be
Rajesh’s daughter.
The correct answer is B.

Let’s look at another example:


Example 2

Pointing to a photograph, an unmarried man, who has only one sister, says, ‘The woman in the
photograph is the maternal grandmother of my niece.’ How is the woman in the photograph related
to the man?
(A) Mother
(B) Sister
(C) Daughter
(D) Mother-in-law
(E) Aunt
Solution

Since the man has only one sister, his niece must be his sister’s daughter. So, the niece’s maternal
grandmother will be the man’s sister’s mother.
The correct answer is A.
557

Book 1.indb 557 30/04/2019 4:49:24 PM


NMAT by GMAC™ Official Guide 2019

Important Learning:  The key to answering family tree questions correctly is to be aware of the various
relations that exist between family members such as first cousin, second cousin, nephew, niece, grand-
uncle and so on.

Type 2: Puzzle
In these types of questions, a blood relation is specified among two or more persons. You need to analyse
the given information and make a relation chart. In the questions related to puzzles, you may also be asked
to count the number of male and female members in the family.
Example 3

Ajay and Binod are brothers and Chaitali and Dhvani are sisters. Ajay’s son is Dhvani’s brother. How
is Binod related to Chaitali?
(A) Grandfather
(B) Brother
(C) Son
(D) Uncle
(E) Father

Solution

If Ajay’s son is Dhvani’s brother, then Dhvani is Ajay’s daughter. So, Chaitali is also Ajay’s daughter.
This makes Binod Chaitali’s uncle.
The correct answer is D.

Example 4

If Akhil is the brother of Bharati; Bharati is the sister of Chandan; Chandan is the father of Devesh.
How is Devesh related to Akhil?
(A) Son
(B) Brother
(C) Nephew
(D) Grandson
(E) Grandnephew

Solution

From the given statement, it is clear that Akhil and Chandan are brothers. So, Chandan’s son Devesh
will be Akhil’s nephew.
The correct answer is C.

Type 3: Coded Relations


In these types of questions, symbols and codes including +, ÷, =, ⋆, ×, ©, ®, ­etc. are used for representing
relationship between two persons. You need to analyse the codes and symbols and answer the questions
accordingly.

558

Book 1.indb 558 30/04/2019 4:49:25 PM


6.0  Logical Reasoning Review  6.5  Analytical Reasoning

Directions: Read the following information carefully and answer the questions, that follow.

‘A – B’ means ‘A is the father of B’


‘A + B’ means ‘A is the daughter of B’
‘A ÷ B’ means ‘A is the son of B’
‘A × B’ means ‘A is the wife of B’
Example 5

In the expression ‘A ÷ B + C – D’, how is A related to D?


(A) Nephew
(B) Son
(C) Uncle
(D) Wife
(E) Daughter

Solution
From the given information, we can draw the following family tree:

Daughter C

B Father

Son D
A
Nephew
From the above family tree, it can be concluded that ‘B’ is the sister of ‘D’. Hence, the son of ‘B’, that
is, A will be the nephew of D.
The correct answer is A.

Tips for approaching blood relation questions


1. Apply the question to yourself
The best way to solve blood relation questions is to apply the question to yourself. Try and relate every
statement to yourself. The starting name of the statement or question could be assumed as your name
or you
2. Do not assume the gender
Do not assume the gender of any person in the question just based on the names given in the
question.
3. Break the question into smaller parts
When the statement is very long, break down every statement in the question into sub statements and
solve the question.

559

Book 1.indb 559 30/04/2019 4:49:25 PM


NMAT by GMAC™ Official Guide 2019

4. Draw clear diagrams as you solve


Easiest way to solve questions on blood relation with 100% accuracy is to draw a family tree. In family
tree, people of the same generation are placed at the same level and the entire diagram is in the form
of a hierarchy.

5 Coding Questions
These questions require you to code or decode some common words such as city names. There will be a
code given to you in the question stem and, using the knowledge of how this code has been created, you
will need to create a code for the word given in the question stem.
Let us take a look at an example:
Example 1

In a certain language, CHENNAI is written as DIFOOBJ. How will MUMBAI be written in this
language?
(A) NWNBCJ
(B) NVNBCJ
(C) NWNJCB
(D) NVNCBJ
(E) NVNCBK
Solution

It is easy to see how this code has been created. Each alphabet of the original word is replaced with
the alphabet after that. So, C becomes D, H becomes I, E becomes F and so on. Using the same rule,
(D) should be the code for Mumbai.
The correct answer is D.
Common Codes + Coded sentences
Coding a message or a term means writing it in such a manner that it cannot be understood by anyone
other than the person who knows the pattern underlying the code that can be used to decode the message.
1. Letter Coding
In these questions, the letters of the given word will be replaced by other letters. You will need to identify
the pattern and answer the question accordingly. You can get two types of letter coding questions:
Scramble-type coding: In these questions the letters in the code are the same as those in the original word
but the position of these letters changes, that is, the letters get scrambled or mixed up.
Let us take a look at an example:
Example 2

In a certain code, MALICIOUS is written as ILAMCSUOI. How is ABANDONED written in that


code?
(A) BADDENONA
(B) ONABNDDAE
(C) NBDDAENOA
(D) NABADDENO
(E) ENODDABNA
560

Book 1.indb 560 30/04/2019 4:49:25 PM


6.0  Logical Reasoning Review  6.5  Analytical Reasoning

Solution

The first thing to do in letter coding questions is to check whether the question is a scramble type
or replacement type. To do this, take a quick look at the given code. If the letters in the code match
the letters in the original word, then you are looking at a scramble type code. On the other hand, if
you see new letters in the code, you are looking at a replacement or substitution type code. In this
question, the letters in ILAMCSUOI match those in MALICIOUS, so we know we are looking at
a scramble type code. It is now a simple matter of identifying how these positions are changing. The
best way to do this is to write the original word and its code in front of each other in this manner:

M A L I C I O U S

I L A M C S U O I

Using the same arrow marks, the code for ABANDONED should be

A B A N D O N E D

N A B A D D E N O
The correct answer is D.

Replacement-type coding: In these codes, the letters in the original word are replaced with new letters
according to a fixed pattern. You need to identify this pattern to crack the code.
Let us take a look at an example:
Example 3

In a certain code, VERITY is written as TCPGRW. How is FELON written in that code?
(A) JCDML
(B) DJCML
(C) DCJML
(D) MCJDL
(E) DMCJL

561

Book 1.indb 561 30/04/2019 4:49:25 PM


NMAT by GMAC™ Official Guide 2019

Solution

Just as we did for the previous question, write the original word and the coded word in front of each
other and see if you can identify some relation between each of the letters.
V - 2 places =T
E - 2 places =C
R - 2 places =P
I - 2 places =G
T - 2 places =R
Y - 2 places =W

Following the same rule, the code for FELON should be:
F - 2 places =D
E - 2 places =C
L - 2 places =J
O - 2 places =M
N - 2 places =L

Thus, the answer is DCJML.

The correct answer is C.

Some common methods used in Replacement-type coding are:


a) Moving a letter forward – ADAM becomes BEBN
b) Moving a letter backward – HEN becomes GDM
c) Moving odd letters a step forward with no change in the even letters – BOTTLE becomes
COUTME.
d) Moving odd letters a step forward with even letters a step backward or vice versa – BOTTLE
becomes CNUSMD.
e) Reversing the letters – PHONE becomes ENOHP

Let's look at another example:


   Example 4

In a certain code, if GREEK is written as HTHIP, what is the code for SPAIN?
(A) TQBLM
(B) TRDMS
(C) JYTRP
(D) LJMDI
(E) LOIEY
Solution

In this code, each letter is replaced with the letter that comes after it. However, this difference keeps
increasing by one for every subsequent letter. This is how it works:
562

Book 1.indb 562 30/04/2019 4:49:25 PM


6.0  Logical Reasoning Review  6.5  Analytical Reasoning

G—replace with next letter—H


R—replace with the letter after the next letter—T
E—replace with the letter two letters after the next letter—H
E—replace with the letter three letters after the next letter—I
K—replace with the letter four letters after the next letter—P
The correct answer is B.

2. Number Coding

In number coding, the letters in the given word are replaced by numbers in the code based on a certain
pattern. You need to identify this pattern and answer the question accordingly. Here, keep in mind that
the most common way of replacing letters with numbers is to use the position of the letter in the English
alphabet series. For example, A can be replaced with 1, B with 2, Z with 26 and so on.
Let us take a look at an example:
Example 5

In a secret language 7 is coded as E, 5 is coded as R, 2 is coded as A, 1 is coded as T, 4 is coded as O, 6


is coded as L and 3 is coded as K. What does 7256 represent?
(A) KARL
(B) AERL
(C) EARL
(D) RATE
(E) RALE
Solution

The correct answer, as can be seen by plugging in the letters for each number, is EARL.
The correct answer is C.

Let's look at another example:


Example 6

In a code language, if ALPINE is coded as 112169145, what is the code for SERPENT?
(A) 195185142
(B) 1206345762
(C) 1765478954327
(D) 195181651420
(E) 187654356782
Solution


It helps if you have a table of the number that each letter of the alphabet corresponds to in front of
you. So, A corresponds to 1, B to 2, C to 3 and so on until you reach Z, which corresponds to 26.
From this S = 19, E = 5, R = 18, P = 16, E = 5, N = 14, T = 20. So, the answer is 195181651420, that
is, (D).
The correct answer is D.

563

Book 1.indb 563 30/04/2019 4:49:25 PM


NMAT by GMAC™ Official Guide 2019

Important Learning:  If the letters in the options are the same as those in the original word, you are
looking at a Scramble-type code.

3.  Language based Coding


Coding-decoding is also tested for a group of words. To solve such type of questions, you need to find the
common words in the group of sentences to determine the code of each word.
Let us take a look at an example:
Example 7
In a code language, ‘The day looks sunny’ is written as ‘dim may zook manny’
‘Thursday was a sunny day’ is written as ‘bekbay il po manny may’
‘Sunny days are here again’ is written as ‘manny kil lop pere jukol’
What is the code for ‘sunny’?
(A) zook
(B) may
(C) dim
(D) manny
(E) bekbay
Solution

Between the first two sentences, the only common words are sunny and day and the only common
codes are ‘manny’ and ‘may’. Thus, one of these must mean sunny. Sentence 3 includes sunny but not
day and it includes the code manny but not may. Thus, the code for sunny must be manny.
The correct answer is D.

6 Series Questions
In Series questions, you will be given a series of numbers or letters with some relation between all the terms
in the series. You will need to identify this connection in the series of numbers to find the next item in the
series.
Let us take a look at an example:
Example 1
Which of the below options would come next in this series of numbers?
1, 4, 9, 16, 25, __
(A) 29
(B) 33
(C) 36
(D) 49
(E) 64
Solution
It can be easily observed that the given series is a list of squares of all numbers starting from 1 (12 = 1,
22 = 4, 52 = 25). So, the next term in the series should be 62 = 36.
The correct answer is C.

564

Book 1.indb 564 30/04/2019 4:49:26 PM


6.0  Logical Reasoning Review  6.5  Analytical Reasoning

Let’s look at another example:


Example 2

What should be the next term in the series 2, 3, 6, 6, 10, 9, 14, 12, ____?
(A) 12
(B) 15
(C) 18
(D) 20
(E) 21
Solution

This is an alternating series in which the terms at odd and even positions follow different rules.
Odd terms Starting with 2, you get the next term by adding 4 to the previous term at an odd
position. So, 2, (2 + 4 =) 6, (6 + 4 =) 10 and so on.
Even terms Starting with 3 you keep going up in multiples of 3 at every subsequent even
position. So, 3, 6, 9, 12 and so on.
Since the next term is at an odd position, it will be equal to 14 + 4 = 18.
The correct answer is C.

Some commonly tested series


• Addition series—5, 8, 11, 14, 17, __
• Subtraction series—50, 45, 40, 35, 30, 25, __
• Product series—2, 4, 8, 16, 32, 64, __
• Square series—1, 4, 9, 16, 25, 36, 49, __
• Cube series—1, 8, 27, 64, 125, __
• Fibonacci series—0, 1, 1, 2, 3, 5, 8, __

Tips for approaching series questions


1. If the series is increasing gradually, that is, the gap between successive terms is not very large, you are
most likely looking at an addition-based series.
2. If the series is increasing rapidly, that is, the gap between successive terms is very large, you are most
likely looking at a multiplication-based series.
3. If the gap between successive terms keeps on increasing as you move ahead in the series, you are most
likely looking at a series involving squared or cubed numbers.
4. If the series looks very haphazard or random, there may actually be two series within one. For
example, the odd integers might be following one pattern and the even integers might be following a
different pattern.

Important Learning:  If a series increases gradually, it is most likely addition-based, and if it rises
rapidly, it is most likely multiplication-based.

565

Book 1.indb 565 30/04/2019 4:49:26 PM


NMAT by GMAC™ Official Guide 2019

Let us take a look at an example:


Example 3

What should be the next term in the series 22, 31, 40, 49, 58, ?
(A) 67
(B) 68
(C) 71
(D) 73
(E) 77
Solution

This is simple arithmetic progression, in which you keep adding 9 to the previous number to get to the
next number. So, the next number should be
58 + 9 = 67.
The correct answer is A.

Example 4

What should be the next term in the series 5, 7, 12, 19, 31, 50, ___?
(A) 63
(B) 75
(C) 81
(D) 92
(E) 101
Solution

This is a variation of a Fibonacci sequence in which, after the first two numbers, every subsequent
number is equal to the sum of the previous two numbers.
The correct answer is C.

7 Direction Questions
Questions on directions would involve reasoning based on the eight directions on a map—north, northeast,
east, southeast, south, southwest, west and northwest. These are illustrated in the following figure.

NW NE

W E

SW SE

The eight common directions

566

Book 1.indb 566 30/04/2019 4:49:26 PM


6.0  Logical Reasoning Review  6.5  Analytical Reasoning

It helps to have some sense of direction while answering these questions. For example, you should know
which direction you will be facing if you turn 45° or 90° from your current position. It also helps if you can
create a diagram based on the instructions given to you in the question.
Let us take a look at an example:
Example 1

A man walks 5 km to the south and then turns left. He then walks for 3 km before turning right. He
walks 5 km and turns 90 degrees in the clockwise direction. Which direction is he facing at the end?
(A) South
(B) East
(C) Northeast
(D) West
(E) Southwest
Solution
Let us draw a figure based on the given instructions. It would look something like the following
figure. Remember that clockwise means from right to left, so taking a 90° clockwise turn is the same
as taking a right turn.

Starting Point

5 km S

N
3 km to left

W E
5 km right

S
90° turn

The correct answer is D.

Important Learning:  Turning 90° clockwise means turning right and turning 90° anti-clockwise means
turning left.

Tips for approaching direction questions


(a) At the time of sunrise,
i. if a man’s shadow is behind him, he is facing east
ii. if a man’s shadow is to the left of him, he is facing north

567

Book 1.indb 567 30/04/2019 4:49:27 PM


NMAT by GMAC™ Official Guide 2019

iii. if a man’s shadow is to the right of him, he is facing south


iv. if a man’s shadow is in front of him, he is facing west

(b) At the time of sunset,


i. if a man’s shadow is behind him, he is facing west
ii. if a man’s shadow is to the left of him, he is facing south
iii. if a man’s shadow is to the right of him, he is facing north
iv. if a man’s shadow is in front of him, he is facing east
(c) At 12 noon, the rays of the sun are vertically overhead, and so there will be no shadows.

Let’s look at an example of a shadow-based question:


Example 2

A and B are having a face-to-face conversation in the evening. If A’s shadow falls in front of him,
which direction is B facing?
(A) South
(B) Northwest
(C) North
(D) East
(E) West
Solution

In the evening, if your shadow falls in front of you, you are facing the east direction. So, A is facing
east. Since B is facing A, he must be facing west.
The correct answer is E.

Let's look at another example:


Example 3

One evening Suresh and Ramesh were talking to each other and Suresh’s shadow fell behind Ramesh.
Which direction was Ramesh facing?
(A) East
(B) West
(C) North-west
(D) South
(E) North
Solution

If you remember the rules about shadows, you will recall that if the shadow falls behind you in the
evening, you are facing west. You can even verify this pictorially, in case you have forgotten the rules.
The correct answer is B.

568

Book 1.indb 568 30/04/2019 4:49:27 PM


6.0  Logical Reasoning Review  6.5  Analytical Reasoning

8 Alphabet Test Questions


As the name suggests, Alphabet test questions will test you on the English alphabet series. Before we look
at the various types of questions you can get in this topic, let’s review a few alphabet related concepts.
i. There are 26 letters – A B C D E F G H I J K L M N O P Q R S T U V W X Y Z
ii. Ranking of letters in the English alphabet series

From left to right

A B C D E F G H I J K L M
1 2 3 4 5 6 7 8 9 10 11 12 13
N O P Q R S T U V W X Y Z
14 15 16 17 18 19 20 21 22 23 24 25 26

From right to left

A B C D E F G H I J K L M
26 25 24 23 22 21 20 19 18 17 16 15 14
N O P Q R S T U V W X Y Z
13 12 11 10 9 8 7 6 5 4 3 2 1

iii. Opposite of each letter in the English alphabet series

A B C D E F G H I J K L M

Z Y X W V U T S R Q P O N

iv. There are 5 vowels – A E I O U


v. There are 21 consonants - B C D F G H J K L M N P Q R S T V W X Y Z
vi. When the question stem says to the right, you have to move from left to right, that is, from A to Z
vii. When the question stem says to the left, you have to move from right to left, that is, from Z to A
viii. When the question stem says from the right, you have to move from right to left, that is, from Z to A
ix. When the question stem says from the left, you have to move from left to right, that is, from A to Z

Commonly Tested Alphabet Test Question Types


One important thing to keep in mind while attempting Alphabet test questions is that these are primarily
observation-based questions, that is, most of the time you will not really have to solve anything. You will
need to observe the letter arrangement that is given to you and answer questions from this. Let’s look at
some question types that usually appear in the Alphabet test:
1. Position based questions:  These questions will test you on the position of different letters within the
English alphabet system or within a letter arrangement that is given in the question stem.

569

Book 1.indb 569 30/04/2019 4:49:27 PM


NMAT by GMAC™ Official Guide 2019

Example 1

Which letter will come exactly between the tenth letter from the left and the seventh letter from the
right in the English alphabet series?
(A) N
(B) O
(C) P
(D) Q
(E) R
Solution

In the English alphabet series, the 10th letter from the left is J and the 7th letter from the right is T.
The letter midway between J and T is O, which is the answer.
The correct answer is C.

2. Vowel-Consonant type questions:  In these questions, you will be given an arrangement of letters
containing a mix of vowels and consonants. The questions will be based on certain arrangements or
patterns of these vowels and consonants.
Example 2

In the letter arrangement given below, how many vowels are preceded by a consonant and followed by
a vowel?
DTUHJKIUGNRFEWBKLZOANFTGDEROPLUJHYG
(A) 1
(B) 2
(C) 3
(D) 4
(E) More than 4
Solution

In such questions, starting from one end of the arrangement (ideally the left end because we read from
left to right), look for the vowels. The first vowel you will encounter is U (DTUH....)
Now check whether it satisfies the criteria given in the questions. While U is preceded by a consonant
(T), it is not followed by a vowel. Thus, it cannot be part of the answer. Similarly moving right, come
to the next vowel – I (KIUG). I is preceded by a consonant (K) and also followed by a vowel (U).
Thus, I is part of the answer. Once again, remember not to underline these terms in your book. What
you should do instead is write the entire group of three terms (the vowel, the consonant preceding
it and the vowel succeeding it) in your rough sheet (KIU) so that, in case you want to double check
your work, you don’t have to waste time finding the vowels once again in the given arrangement. Keep
moving till the end of the arrangement in the same manner. Your final answer set should look like
this: KIU ZOA.
Thus, there are two vowels that satisfy the criteria given in the question.
The correct answer is B.

570

Book 1.indb 570 30/04/2019 4:49:27 PM


6.0  Logical Reasoning Review  6.5  Analytical Reasoning

3. Make-a-meaningful-word questions:  In these questions, you will be asked to make one or more
meaningful word(s) from a given set of letters. These letters could be given to you in isolation or as part
of a bigger arrangement.
Example 3

If it is possible to form just one meaningful word using the letters A F E C, the third letter of that
word is your answer. If more than one such word can be made, your answer is ‘M’ and if no such word
can be made, your answer is ‘X‘.
(A) F
(B) C
(C) E
(D) M
(E) X
Solution

In such questions, you need to draw upon your knowledge of English words, though the words given
to you are usually short, containing only four or five letters. Using the alphabets A F E and C we
can actually make two meaningful words – FACE and CAFE. Thus, as per the instructions in the
question, the answer has to be M.
Note: The use of the term ‘meaningful’ in the question stem is very important because the words that you make
need to have some meaning. For example, a choice such as FEAC does not work in the above question because
it has no meaning.
The correct answer is D.

Rule-based questions: In these questions, you will be given a meaningful English word and you will be
asked to base your answer on specific conditions or rules given in the questions.
Example 4

How many such pairs of letters are there in the word BAROMETER each of which has as many
letters between them, (in both forward and backward directions) in the word as they have in the
English alphabetical series?
(A) None
(B) 1
(C) 2
(D) 3
(E) More than 3
Solution

In these questions, you need to check various combinations of letters (with reference to the gap
between them) as they appear in the given word as against their positions in the English alphabet
series. Let’s start from the two most left letters: BA. The gap between them is zero (since they come
right next to each other). If you look at the position of BA in the English alphabet series, it is still zero
because even in the alphabet series they come together.
Thus, BA is one possible answer so jot it down on your rough sheet. Here note that the order of the
two letters (that is which comes first in the given word) does not matter since the question stem
clearly says that you can move forward or backward.

571

Book 1.indb 571 30/04/2019 4:49:28 PM


NMAT by GMAC™ Official Guide 2019

Now look for the next combination – BR. In the given word, there is a gap of one letter between B
and R whereas in the English alphabet there is a much bigger gap (don’t bother calculating this gap
because it is obvious that this gap is more than 1 letter). Thus, BR cannot be the answer.
Similarly, now try BO, BM and so on. Once you reach the last combination with B that is BR, you
need to start making combinations with A, that is, AR, AO and so on.
Your final answer set will look like this: BA, AROME (gap of 3 letters between A and E, same as in
the English alphabet series), TER (gap of 1 letter between T and R, same as in the English alphabet
series. Thus, the answer is three.
The correct answer is D.

9 Input-Output Questions
Input-Output is a question type in which you are given a word and number arrangement. With each
subsequent operation, the arrangement of the words and numbers changes. These operations are performed
until a final arrangement is reached or is performed in a loop. You are required to identify the hidden
pattern in the rearrangement and apply it to the questions asked.
Let’s take a look at an example
Directions: A word and number arrangement machine when given an input line of words and numbers
rearranges them following a particular rule in each step. The following is an illustration of input and
rearrangement.
Input: go now 53 39 18 for again 66
Step 1 : 66 go now 53 39 18 for again
Step 2 : 66 again go now 53 39 18 for
Step 3 : 66 again 53 go now 39 18 for
Step 4 : 66 again 53 for go now 39 18
Step 5 : 66 again 53 for 39 go now 18
Step 6 : 66 again 53 for 39 go now 18
Step 7 : 66 again 53 for 39 go 18 now

As per the rule followed in the above arrangement, attempt the following question.
Input: chicken wand 24 44 57 Hexa bleach 71
Example 1

How many steps will be required to complete the rearrangement of the above input?
(A) Three
(B) Four
(C) Five
(D) Six
(E) Seven

572

Book 1.indb 572 30/04/2019 4:49:28 PM


6.0  Logical Reasoning Review  6.5  Analytical Reasoning

Solution

The trick to solving Input-Output questions is to first try and identify the structure in the original
input-output table given to you. In this question, you will notice that the numbers in the input
get arranged in descending order whereas the words get arranged as per their order in the English
dictionary. Also note that each step is moving one number or one word such that they alternate with
each other, starting with the number. Now, we can apply this same logic to the input given to us in the
above question. Here are the steps accordingly:
Step 1 : 71 C W 24 44 57 H B
Step 2 : 71 B C W 24 44 57 H
Step 3 : 71 B 57 C W 24 44 H
Step 4 : 71 B 57 C W 24 44 H
Step 5 : 71 B 57 C 44 W 24 H
Step 6 : 71 B 57 C 44 H W 24
Step 7 : 71 B 57 C 44 H 24 W
Thus the correct answer is 7 steps, that is, Option E.
The correct answer is E.

Tips for Input-Output questions


• Usually the first, second and final steps of the arrangement are enough to identify the pattern
• If there are ‘n’ words/digits in the input then at most ‘n-1’ steps are required to rearrange it
completely

10 Matrix Type Questions


Matrix test questions are a form of coding question in which some numbers and their corresponding
alphabetical codes are given in the form of two matrices. A word will be given to you in the question and
you will need to identify its code from the given matrices.
Let us take a look at an example:
Example 1

A word is represented by only one set of numbers as given in either of the matrices below. The
columns and rows of Matrix I are numbered from 0 to 4 and the columns and rows of Matrix II are
numbered from 5 to 9. A letter from these matrices can be represented first by its row and next by its
column, e.g., A can be represented by 03, 10, etc. and D can be represented by 58, 65, etc. Similarly,
you have to identify the set for the word, TEAK.
Matrix I
0 1 2 3 4
0 C B O A T
1 A C T B O
2 B O A T C
3 T C B O A
4 O A T C B

573

Book 1.indb 573 30/04/2019 4:49:28 PM


NMAT by GMAC™ Official Guide 2019

Matrix II
5 6 7 8 9
5 R E K D L
6 D L R E K
7 E K D L R
8 L R E K D
9 K D L R E

(A) 04 10 87 57
(B) 30 76 75 22
(C) 42 75 22 88
(D) 23 88 10 75
(E) 75 42 88 22
Solution
Option A:   TA, so leave this
Option B:   TK, so leave this
Option C:   TEAK, so the correct answer
The best way to answer these questions is to quickly go through all the answer choices and check
which one gives the correct answer. However, in order to save time, do not go through all the answer
choices completely. The moment you come across a word in the matrix that does not match with the
given word, skip this answer choice and move to the next one.
The correct answer is C.

Let's look at another example:


Example 2

Study the matrices and determine the digits that comprise the given word.
0 1 2 3 4 5 6 7 8 9
0 M B Q W U 5 T A B L K
1 S N I Z B 6 X I O E S
2 O T U J A 7 W U P B Q
3 R E C H T 8 I T T V M
4 B T E I S 9 S E B F U

SUBSTITUTE

(A) 10, 04, 14, 44, 21, 12, 34, 23, 55, 42
(B) 44, 99, 78, 69, 21, 43, 86, 77, 41, 31
(C) 69, 22, 01, 95, 55, 85, 41, 99, 34, 96
(D) 69, 76, 97, 95, 41, 85, 88, 04, 21, 68
(E)
95, 99, 57, 10, 34, 66, 65, 76, 87, 96

574

Book 1.indb 574 30/04/2019 4:49:28 PM


6.0  Logical Reasoning Review  6.5  Analytical Reasoning

Solution

Option C has the correct coordinates for each letter as it appears in the word. In all the other options,
some of the letters don’t match with the numbers.
The correct answer is C.

11 Symbol-Based Questions
These questions will be similar to the series-based questions we saw earlier. The only difference between
them is that, while in series-based questions we were dealing with numbers or letters, in symbol-based
problems we are dealing with abstract symbols. You will be given a series of symbols and you will be asked
to select the next symbol in that series from the given options.
Let us take a look at an example:
Example 1

..... ....
...
.. .
.. ?
Which of the following should be the next symbol in the above series?

(A) . . . .

(B) .

(C) . . .

(D) . . .

(E) ...

Solution

It is clear from the given pattern that the order of the shapes has to be square, oval, triangle, square,
oval and so on.
Thus, the next shape has to be a square. This brings us to (A) and (C). Now, look at the number of
dots within each shape in the question stem. This number keeps going down by one until it reaches 1,
and then it starts to increase by one. So, the number of dots in the next symbol should be (2 + 1 = 3).
Thus, option (C) is the correct answer.
The correct answer is C.

Important Learning:  Do not just focus on the shape of the given figures. Also, pay attention to what is
contained within those figures (such as dots in the question we saw above).

575

Book 1.indb 575 30/04/2019 4:49:29 PM


NMAT by GMAC™ Official Guide 2019

Let us take a look at another example:


Example 2

From the answer choices select the figure that will continue the series as established in the five
problem figures.

(A)

(B)

(C)

(D)

(E)

Solution

The symbols within each box move in the following order:

The correct answer is C.

576

Book 1.indb 576 30/04/2019 4:49:31 PM


6.0  Logical Reasoning Review  6.5  Analytical Reasoning

Example 3

Which of the following should be the next figure in the above series?

(A)

(B)

(C)

(D)

(E)

Solution

The figure has three rows and three columns with three arrows in each row and column.
In the second figure, the arrows in the top and bottom positions in the left column swap places. In the
third figure, the top and bottom arrows in the middle column swap places. In the fourth figure, the
top and bottom arrows in the right column swap places. This cycle will again continue starting with
the left column.
For the middle row, the arrows reverse their direction, one arrow at a time, starting from the right
arrow. This cycle will again continue from the right arrow.
The correct answer is A.

577

Book 1.indb 577 30/04/2019 4:49:32 PM


NMAT by GMAC™ Official Guide 2019

12 Other Reasoning Questions


Apart from the question types we have seen so far, you can sometimes also get questions on the NMAT
by GMAC™ that fall under the miscellaneous category, since each of these questions is different in its
own right. These could include input-output questions, matrices questions, and questions requiring you to
visualise three-dimensional figures. Let us take a look at examples of some of these questions:
Example 1

How many dots lie opposite the face having six dots, when the given figure is folded to form a cube?

(A) One
(B) Two
(C) Three
(D) Four
(E) Five
Solution

When the figure is folded to form a cube, 6 and 3 will be opposite; one and two will be opposite; four
and five will be opposite. Thus, the correct answer is (C).
The correct answer is C.

Example 2

In a group of students, Chemistry majors are represented by a circle, Mathematics majors by a


rectangle and Physics majors by a triangle. Which letter represents students who are majoring in all
three subjects?

Maths
Chem.
a

c
d
b
e

Physics

(A) a
(B) b
(C) c
(D) d
(E) e

578

Book 1.indb 578 30/04/2019 4:49:32 PM


6.0  Logical Reasoning Review  6.5  Analytical Reasoning

Solution

From the figure, it is clear that (c) is the only area where all three figures intersect. Thus, it has to be
the correct answer.
The correct answer is C.

Example 3

Which of the following diagrams indicates the best relation between tea, coffee and beverages?

(A)  

(B)  

(C)  

(D)  

(E)  

Solution

All tea and coffee are beverages but all beverages are not tea and coffee. Option D expresses this
relation correctly.
The correct answer is D.

579

Book 1.indb 579 30/04/2019 4:49:33 PM


7.0  Logical Reasoning Practice

Book 1.indb 580 30/04/2019 4:49:34 PM


7.0  Logical Reasoning Practice

7.1 Practice Questions


Solve the following questions and indicate the best of the answer choices given.

1. Verbal Reasoning
Course of Action:
I. He should get the safe checked with the help
of the staff and if required take help of the CA
Directions for Questions 1–7: In the following questions a
handling the bank’s account.
fact situation is given followed by two suggested courses. A
course of action is a step of administrative decision taken for II. He should lodge an FIR with the police and get the
improvement or follow- up action. Read the situation and then culprit arrested as soon as possible.
decide which of the given courses of action follows. 4. Statement: A popular news channel is concerned
Give answer, about the quality of its programmes, particularly in
(A) If only course of action I follows light of losing business to its competition.
(B) If only course of action II follows Course of Action:
(C) If either I or II follows I. It should plan to revise its fee structure for the
(D) If neither course of action follows artists and take various other measures to attract
talent for its programmes.
(E) If both courses of action follow
II. It should take stringent action against people
1. Statement: In the wake of the recent spate of vector involved in the programmes which lost their
borne diseases such as Dengue and Malaria, the state popularity and led to a loss in business.
government has decided to declare these as notifiable
diseases under the existing Epidemics Act. According 5. Statement: The enteric fever, a water-borne disease,
to this Act, the family members of the patient are has turned out to be a killer this year. So far it has
bound to inform the state authorities and are liable to claimed more than 500 lives across the country. In
be punished in case they do not do so. rural areas of Bihar the toll has been over 100 during
the past two weeks.
Course of Action:
Course of Action:
I. The efficacy of the government’s decision
depends on effective implementation of the Act. I. The residents of these villages should immediately
Hence, efforts should be made to implement it in stop using drinking water from open sources such
letter and spirit. as ponds and canals.
II. The government should propagate the instances II. The Government should immediately send a
of punishment through mass media to make medical team to this area to restrict spread of the
people aware of the stern action taken against the killer disease.
defaulters. 6. Statement: The companies producing fuel in India have
2. Statement: This year conjunctivitis seems to be a been found lacking in both quality and quantity of their
major epidemic witnessed after nearly four years supplies to bulk consumers, especially the thermal
even though there have been sporadic cases of power stations. This has resulted not only in low volume
conjunctivitis every year around the monsoons. of power production but also in increased cost of
production especially to the thermal power stations.
Course of Action:
Course of Action:
I. Precautionary initiatives should be taken every
four years to check this epidemic. I. The Government should slap legally enforceable
penalties on the fuel producing companies
II. People should be cautioned not to visit their
defaulting on the quality and quantity of supplies.
neighbours during the rainy season.
II. The Government should force the fuel producing
3. Statement: The bank manager of a city branch had a companies to introduce welfare measures for
foreboding that some money might have been missing their employees so that they don’t show laxity.
from the safe of the bank.

581

Book 1.indb 581 30/04/2019 4:49:34 PM


NMAT by GMAC™ Official Guide 2019

7. Statement: The Education department has come Courses of Action:


under criticism with recent revelations that some senior I. Garment manufacturers from Korea should drop
officials are suspected of involvement in providing illegal the plans of entering Latin America.
admissions in some premiere state universities.
II. Garment manufacturers from Korea should
Course of Action: improve the quality of their products so that they
I. A thorough investigation should be ordered by are able to compete better with the Chinese.
the State Government to bring all those who are 10. Statement: The influx of apples imported from
involved to book. California in Indian stores has considerably reduced
II. State Education should be kept under Central the demand for indigenous apples, which tend to be
Government’s control. of an inferior quality.
Directions:  A statement is given followed by three courses Courses of Action:
of action numbered I, II and III. A course of action is a I. The government should force Indian stores to
step or administrative decision that needs to be taken for stop selling the imported apples and incentivise
improvement, follow-up or further action in regard to a them to sell Indian apples.
problem, policy etc. On the basis of the information given II.  o help indigenous apple growers, the
T
in the statement, and assuming everything in the statement government should impose high import duties
to be true, decide which of the suggested courses of action on imported apples.
should be logically pursued. (Real NMAT Question)
11. Statement: The state owned airline has been
8. Statement: continuously making losses for the past several
The exodus of subscribers from their company is a years with no signs of profits anytime soon.
cause of major concern for MTN Telecom Pvt. Ltd. Courses of Action:
Course of Action: I. The government should provide a bailout
I. MTN should plan combo offers to retain package to the airline in order to ensure that it
customers. does not collapse.
II. MTN should lower their monthly rental of the II. The government should take steps to ensure that
subscription. the airline improves its productivity and reduces
III. MTN should conduct market research or a survey wasteful expenditure.
to find out why this exodus is happening in the Directions: Given alongside is a statement followed by a few
first place. suggested courses of action. For the given situation, select the
(A) Only I should be pursued. best course of action that follows. (Real NMAT Question)
(B) Only II should be pursued. 12. Statement: There is quiet unrest among people
(C) Only III should be pursued. today—it will be no surprise if this simmering
(D) Both I and II should be pursued. discontent turns into some kind of violent protest.
(E) Either I or II should be pursued. Courses of Action:
I. The government needs to be sharp and realise
Directions for Questions 9–11:  The following questions the mood of its people—it should take steps to
consist of a statement followed by two decisions. You have address the existing pain points and not allow
to assume everything in the statement to be true and, on the anything to snowball.
basis of the information given in the statement, decide which
II. The government should carry on with its work as
of the suggested courses of action logically follow(s). Select
usual—unrest in people is obvious when things
from the following choices:
become difficult.
(A) If only I follows
III. The government needs to focus on the
(B) If only II follows economy—everything else will fall into place.
(C) If either I or II follows (A) Only I should be pursued.
(D) If neither I nor II follows (B) Only II should be pursued.
(E) If both I and II follow (C) Only III should be pursued.
9. Statement: The Korean garment export industry (D) Both II and III should be pursued.
venturing into the Latin American markets faces (E) None should be pursued.
tough competition from the Chinese.

582

Book 1.indb 582 30/04/2019 4:49:34 PM


7.0  Logical Reasoning Practice

13. Directions: A statement is followed by three III. The water gates should be opened since loss of
courses of action numbered I, II and III. A course life, livestock and property has happened and
of action is a step or administrative decision to be the dam has to be saved since it is of national
taken for improvement, follow-up or further action importance.
regarding the problem, policy etc. On the basis of the (A) Only I follows.
information given in the statement, decide which of
(B) Only II follows.
the suggested courses of action should logically be
pursued. (Real NMAT Question) (C) Both I and II follow.
(D) All follow.
Statement:
In the last 3 months, at least 12 restaurants in the (E) None follow.
area have caught fire due to the lack of various 15. Who wouldn’t enjoy a comedy film?! (Real NMAT
safety measures. With slow emergency response Question)
times, these incidents have resulted in the death of
What are the assumptions implicit in the above
around 100 people.
exclamation?
Courses of Action: (A) Everyone watches movies.
I. The government should make fire insurance (B) Comedies are worth being enjoyed by everyone.
mandatory for all restaurants.
(C) There are at least some people who enjoy
II. The government should check the wiring of comedies.
buildings in the area.
(D) Nobody watches movies of a genre other than
III. The Fire Brigade should be made to arrive more comedy.
promptly.
(E) Comedies are the only genre on which movies
(A) Only I follows. are made.
(B) Only II follows.
16. Painting is no more an art. These days, with so much
(C) Both I and II follow. online software being available, you don’t have to
(D) All follow. have a flair with the brush. Just a few clicks of the
(E) None follow. mouse, and a few commands from the keys, and you
get a whole gamut of colours to paint with — and
14. Directions: A statement is followed by three courses then, anyone can become a flawless painter. (Real
of action numbered I, II and III. A course of action NMAT Question)
is a step or administrative decision to be taken for What does the author of this argument assume?
improvement, follow-up or further action in regard
(A) When done, a canvas art is perfect.
to the problem, policy etc. On the basis of the
information given in the statement, decide which of (B) Online painting software would be most used in
the suggested courses of action should logically be future.
pursued. (Real NMAT Question) (C) Painting when done with some other software is
a true form of art.
Statement:
On the one hand, heavy rain in the last two days has (D) To become a flawless painter, all you need is
taken the water levels way beyond the danger zone perfect online software.
and raised the risk of the dam breaking, if water is (E) All that is important in a painting is colours, the
not released. On the other hand, the water levels are subject is not important at all.
high in the entire region and opening the water gates
will only lead to flooding. 17. As part of the austerity measures, the Government
has put a ban on foreign travel unless approved
Courses of Action: by an official of the Secretary level. (Real NMAT
I. The water gates should be opened since the Question)
region is already high on water levels. Not The argument above assumes that:
opening them will only lead to breaking of the (A) many officials make unnecessary foreign visits.
dam and not save the area in any way.
(B) even essential foreign visits could be discouraged.
II. The water gates should not be opened since that
may still save the region from being flooded, but if (C) only Secretaries can approve foreign travel of
they are opened the water is sure to flood the area. officials.

583

Book 1.indb 583 30/04/2019 4:49:34 PM


NMAT by GMAC™ Official Guide 2019

(D) the Government wants to cut down on avoidable multispeciality clinic also started running into losses
expenditure, which includes travel that can be and ultimately closed down. The HOD stated that the
avoided. reason for the losses was the closure of the software
(E) due to an economic crunch, the Government is company. (Real NMAT Question)
first trying to curb avoidable foreign visits before Which of the following can be concluded from the
looking to implement other measures. passage above?
18. A company published an advertisement seeking (A) All the doctors of the multispeciality clinic left the
applications from teachers for online tutoring town once the software company closed down.
assignments. The teachers needed to be experts (B) After the closure of the software company, the
in any one of the subjects: Mathematics, Physics, multispeciality clinic was the only big employer in
Chemistry or English. They also needed to be the town.
computer and internet savvy. (Real NMAT Question) (C) The people in the town, even if unemployed,
Which of the following assumptions is implicit in the continued to be treated at the multispeciality
given information? clinic and were also paying for the treatment.
(A) The computer industry is growing at a very fast (D) The closure of the software company led to the
pace. loss of patients at the multispeciality clinic as
(B) It is possible to be a teacher, as well as an online most of the diseases originated in the software
tutor. company.
(C) Some teachers of Biology can also become (E) The chief reason for the closure of the
online tutors. multispeciality clinic was the unemployment of
the majority of the population in the town as they
(D) The company has received the maximum number
were not able to afford expensive treatments.
of applications for Mathematics.
(E) Only the four subjects mentioned in the 21. To increase aircraft occupancy, it is necessary to stop
information can be taught in the online mode. offering food on the airplane so that the prices of tickets
can be reduced without reducing the per ticket profit.
19. The Black Spider Monkey of Brazil has become
Which of the following, if true, will support/strengthen
endangered because of human activities such as,
the claim made in the argument above?
deforestation and logging, which causes destruction of
the monkey’s natural habitat. (Real NMAT Question) (A) The price of food served on the airplane is
almost equal to the price of food in cities.
Which of the following if true would weaken the above
argument? (B) Most passengers like to eat to pass their time
(A) Costa Rican government is constantly trying during flight hours.
to preserve the natural habitat of the squirrel (C) Flight journeys are generally short and it is not
monkeys in order to save them from extinction. inconvenient to avoid eating during the journey.
(B) A certain allele (alternate gene) discovered (D) Ticket price is not the only issue for people who
in a black spider monkey pointed to micro- do not prefer air travel.
evolutionary extinction setting in. (E) None of the above
(C) Habitat loss in the Congo basin has seen
no remarkable change in the count of Dryas 22. With an active mass media people have now become
monkeys in this region. aware of their rights and are willing to fight for them.
(D) Habitat loss in the Congo basin has seen a Which of the following, if true, will support/strengthen
decrease in the count of Dryas monkeys in this the claim made in the argument above?
region. (A) According to recent data collated by the
(E) WWF-funded protected areas have seen a slight Consumer forum there is a 150% rise in
revival in the number of the black spider monkey. complaints against poor services.
(B) Most discussions in mass media are celebrity-
20. A multispeciality clinic was situated in the centre of a centric.
small town X, next to a leading software development
(C) People got to know about the existence of the
company. The clinic was attending to and treating
Citizen charter for each department only through
numerous patients and was earning a good profit.
the websites of those departments.
The software development company, which employed
more than 10,000 people was abruptly shut (D) Both (A) and (C)
down due to some litigation issues. After a year, the (E) None of the above

584

Book 1.indb 584 30/04/2019 4:49:34 PM


7.0  Logical Reasoning Practice

23. To reduce losses of the fuel marketing companies, 26. Most often Scientific journals published in
the government has decided to deregulate the diesel Commonwealth countries refer to articles written in
and petrol prices. English only. However, there are many interesting
Which of the following, if true, will support/strengthen scientific developments reported in other languages.
the claim made in the argument above? So because of a basic linguistic ignorance on
the part of scientists in these countries, articles
(A) If prices of petrol and diesel are increased, the
written in other languages do not get referred . This
demand for them will fall sharply.
inevitably results in a failure to benefit from scientific
(B) Fuel consumption patterns rarely show any progress made in other non-english speaking areas.
changes in the amount consumed with fluctuation
Which of the following, if true, tends to confirm the
in price.
conclusion given above?
(C) Automobile sales have dipped rapidly.
(A) The linguistic ignorance of a scientist in these
(D) Farmers use diesel to run machines for countries is an indication of a more general
agriculture. indifference towards foreign languages among
(E) Both (B) and (C) the scientific community.
(B) Many scientific findings reported in scientific
24. Annual recruitment in public sector banks has
journals published in these countries
increased in the recent years. This will lead to
inadvertently repeat studies done in non-english
scarcity of talented candidates for private banks.
speaking countries.
Which of the following, if true, will support/strengthen
(C) All important scientific findings are translated and
the claim made in the argument above?
also published in english-language journals making
(A) As compared to public sector banks, private them available to the scientists of these countries.
banks pay higher salaries.
(D) Reporting a scientific progress is a universal
(B) There is a limited number of people who have the phenomenon and is never limited to any single
specialised skill set required for banking jobs. language of a single country.
(C) Many people do not consider jobs in public (E) Reporting of scientific development is a process
sector banks challenging. that is compulsorily to be published in journals in
(D) The services of an employee of Public Sector different languages.
banks cannot be terminated. 27. Use of credit has both advantages and disadvantages.
(E) Both (B) and (D) While one can buy goods through credit option, many
consumers may buy things they do not really require
25. I have no doubt that the Government of India should or can even afford. But it also allows them to take
make Hindi the official language of the country. How advantage of bargains when they are short of cash.
can we justify that the government spends so much Anyone who takes advantage of this latter possibility
money to print documents in difference languages saves money by buying on credit.
because of those who cannot read and/or write
Which of the following is an assumption made in the
Hindi. The government should spend tax payers’
passage above?
money for better causes. The Government of India
should enable people to learn Hindi at the earliest. (A) Anyone who wants can buy goods through credit
option at no extra cost for which there is no point
Which of the following, if true, would weaken the
in using such a facility.
speaker’s argument the most?
(B) It is difficult for anyone to show restraint while
(A) The government translates official documents
buying goods through the credit option. However,
into more than eighteen languages to reach
these goods are usually available on a bargain as
people who cannot read or write Hindi.
well.
(B) Hindi is already the most spoken language in India.
(C) The cost of credit does not exceed the saving on
(C) People who do not know Hindi also pay taxes to purchases that credit makes possible.
the Government of India.
(D) All purchases made on credit are unnecessary
(D) Many people who travel across the country do purchases for which using such a facility is a
learn Hindi within a year. waste of money.
(E) Hindi is already the official language of India and (E) A buyer who buys goods through the credit option
other languages are equally important for the will make use of this facility to even buy goods
country. which have absolutely no utility for him or her.

585

Book 1.indb 585 30/04/2019 4:49:34 PM


NMAT by GMAC™ Official Guide 2019

28. ‘When women stop reading, the novel will be dead,’ (D) There is at least a genuine scientific basis for
declared Ian McEwan in the Guardian last year. He going to the gym—i.e. getting more exercise and
reached this rather dire conclusion after venturing becoming fitter and healthier—unlike joining a
into a nearby park in an attempt to give away free cult.
novels. The result? Only one ‘sensitive male soul’ took (E) Both (C) and (D)
up his offer, while every woman he approached was
‘eager and grateful’ to do the same. Unscientific as 30. Hosting a high-profile sporting event like the Olympic
McEwan’s experiment may be, its thesis is borne Games or the World Cup can generate significant
out by a number of surveys conducted in Britain, the intangible benefits for the host city or region, whose
United States and Canada, where men account for residents are likely to derive appreciable pride and
a paltry 20 percent of the market for fiction. Unlike sense of community from hosting the event. Their
the gods of the literary establishment who remain homes are the focus of the world’s attention for
predominantly male—both as writers and critics— a brief but intense period. The planning and work
their humble readers are overwhelmingly female. required to host the event take significant time and
Which of the following most weakens the argument effort—much of which is done by volunteers—and
made in this paragraph? engender a considerable local and national sense of
accomplishment. These factors are both important
(A) The people in one park are not a good
and valuable, even though researchers find it difficult
representative of the entire readership of novels.
to place a dollar value on them.
(B) The number of men reading novels may be fewer
Which of the following, if true, does NOT weaken the
than the number of women, but men read more
argument made in this paragraph?
novels per capita.
(A) Research shows that cities in which such events
(C) Men read a variety of books, including plenty of
are hosted have fewer instances of communal
non-fiction, while women read more fiction than
or societal tensions as compared to cities that
non-fiction.
have never hosted such events.
(D) The male readership of novels is significantly
(B) The sense of community and accomplishment
higher than the female readership in places other
last only as long as the event itself lasts, which
than Britain, the United States and Canada.
may be just a few days, and have no long-term
(E) Both (C) and (D) effects.
29. To anthropologists of the future, the gym boom of the (C) The kind of attention garnered by the host city or
past few decades may look more like a sinister cult region may be more negative than positive, i.e.
than a fitness movement. Gym-going, after all, has all the rest of the world may feel itself licenced to
the basic lineaments of a religion. Its adherents are criticise the local politics and culture.
motivated by feelings of guilt, and the urge to atone (D) Both (B) and (C)
for fleshly sins. Many visit their places of worship
(E) All of the above
with a fanatical regularity: a third of LA Fitness
members, for instance, go virtually every day. Once 31. Over the last few years Gmail, like other e-mail
there, believers are led by sacerdotal instructors, services, has become very good at spotting spam. It
who either goad them into mass ecstasy during catches just about every junk message before it hits
aerobics classes, or preside over the confessional the inbox; the messages are rerouted to the spam
tête-à-tête of personal training. Each devotee has his folder, which I myself almost never open (and when I
own rituals, though most rely on the principles of self- do open it, I never notice legitimate messages marked
mortification and delayed gratification. as spam). In other words, spam—which was once the
Assuming the following are true, which of them casts great boogeyman of the Internet, a scourge that was
most doubt on the argument in this paragraph? often predicted to bring down e-mail entirely—is no
(A) Not all people turn gym-going into a ritual, or longer a problem. We have won the War on Spam.
even go regularly. Which of the following, if true, would significantly
(B) Unlike the workings of a cult, those of a gym strengthen the conclusion in this paragraph?
are above-board and meant to help people, not (A) A number of e-mail services have promised to
brainwash them. get better at catching spam in the near future.
(C) People go to the gym for many reasons, (B) While spam is no longer a problem, the number
including socializing or just feeling good about of e-mails—even legitimate ones—that people
themselves, and not for any fanatical reasons. get keeps increasing constantly.

586

Book 1.indb 586 30/04/2019 4:49:34 PM


7.0  Logical Reasoning Practice

(C) Statistics show that the amount of spam being Which of the following, if true, will bring out the flaw
sent is also going down, as the senders realise in the argument above?
that their messages are never read. (A) In developing countries, equality of opportunity
(D) The senders of spam keep coming up with a ensures equality of income but a person doesn’t
variety of tricks to bypass spam filters, but so far, get paid for his efforts.
the spam filters have kept up with those tricks. (B) The excess income in the hands of people with
(E) (A), (C) and (D) higher incomes provides less utility than extra
income in the hands of those with lower incomes.
32. Evolutionary behaviourists are trying to understand
why it is that the emotional connection between (C) High achievement in many societies is due to
siblings can last a lifetime. The prevailing theory is equality of incomes that comes from the fact
that it all comes down to maths. With our nearest that equality of opportunity ensures equality of
relatives—each parent, our full-siblings, and our income.
children, we share 50 percent of our novel genes. (D) In developing countries, gross income of most
This overlap helps explain the continual cycle of of the people is so low that if it were distributed
family love. The shared 50 percent is the basis equally, no one would save enough to provide
for our instinctive willingness to make all sorts resources for investment.
of investments and sacrifices—even perhaps the (E) In developing countries, gross income of most
ultimate sacrifice—for those with whom we are of the people is very low because of rampant
closest. On the level of the gene, it’s a good idea corruption and high illiteracy.
to ensure those most like us will spread part of our
uniqueness. It’s a banal truism that what we feel 34. Not only the third-world countries, but the most
towards our closest relatives is generally different developed economies too would get badly affected
from our feelings for people with whom we don’t if there is opposition to the open trade agreement
share this connection. But why? The researchers say being signed. (Real NMAT Question)
that the emotions of love and fidelity are nature’s way From which of the following can the statement above
of doing the maths on behalf of our genes. be most properly inferred?
Which of the following facts, if true, would strengthen (A) The survey report of the World Trade
the argument in this paragraph? Organisation is nothing but a means to protect
(A) Research shows that identical twins, who share the business interests of developed nations.
100 percent of their genes, are emotionally (B) Third-world economies are weak due to non-
closer to each other than fraternal twins, who agreement on free trade which does not allow
share only 50 percent of their genes. goods of their countries to be sold elsewhere.
(B) Research shows that identical twins, who share (C) Free trade agreement will result in two-digit
100 percent of their genes, are much more alike growth rate of third-world economies, which are
than fraternal twins, who share only 50 percent currently hindered as they are happy doing trade
of their genes. amongst themselves or not trading at all.
(C) Research shows that half-siblings, who share only (D) The worst fears of the third-world economies is
25 percent of their genes, are not as alike as full- that they would not be able to absorb the unfair
siblings, who share 50 percent of their genes. practices of the developed economies and the
(D) Research shows that half-siblings, who share only free-trade would in fact be favourable only to the
25 percent of their genes, are just as emotionally developed countries.
close as full-siblings, who share 50 percent of (E) According to a World Trade Organisation’s survey
their genes. report, the third-world countries will not achieve
(E) Both (A) and (B) a two-digit economic growth rate if they do not
sign the free-trade agreement with an open mind,
33. Ensuring equal opportunity has long been a prominent which could see their worst economic fears
goal in many countries, particularly Western Europe coming true.
and America. These countries have more equality of
income - what a person gets for his efforts and the 35. An animal conservation group in South Africa is trying to
yield of his property after taxes are deducted. Many change the long-standing image of owls as terrorising
economists in India believe this concept of equality creatures. The group argues that owls are feared and
should be implemented in developing countries in considered oppressive just because they are reclusive
order to speed up economic development. animals that work only during the night. (Real NMAT
Question)

587

Book 1.indb 587 30/04/2019 4:49:35 PM


NMAT by GMAC™ Official Guide 2019

Which of the following, if true, would most seriously (B) There has been no change in the ticket prices at
weaken the animal conservation group’s argument? movie theatres in the past few months.
(A) Owls are feared not only in South Africa but in the (C) The talk show host has also made fun of several
USA, India and Europe as well. other movies and TV programmes on his talk
(B) Raccoons and bats are also shy animals and show during this period.
are active only during the night, yet they are not (D) The talk show host makes fun of only those
feared or perceived as being oppressive. movies that perform poorly at the box office.
(C) Owls are losing out on their natural dwelling (E) The movie has been criticised by some sections
places and are being forced to move to other of the media too for being overly long and having
areas of human dwelling, which make them more a clichéd storyline.
frightening.
38. In a recently conducted survey on the quality of life
(D) Night brings a psychological effect of fear in the among the citizens of a city, surveyors spoke to
human minds and thus owls, who work during 980 citizens, a majority of whom said that they were
the night-time, also have a similar effect on the satisfied with their quality of life. Thus, it can be
human mind. concluded that the citizens of this city are satisfied
(E) More is known about the behaviour of other with their quality of life.
more-feared animals such as tigers and
crocodiles, thereby making such animals’ Which of the following options most seriously
behaviours relatively more predictable. weakens the above argument?
(A) The surveyors spoke to people from only a
36. The cost to a company of an incompetent particular economic strata.
departmental head can be far greater than that of
(B) Some of the respondents said that they were
an incompetent assembly line worker. Management
extremely dissatisfied with their quality of life.
training is seen as being so vital today that industry
devotes considerably more time, money and effort (C) The survey results are different from what have
to this level than to the training of employees. (Real been reported from other neighbouring cities.
NMAT Question) (D) The survey sample was representative of the
Which of the following assumptions is implicit in the entire population of the city.
information given above? (E) The governing body of the city is not very
(A) Training is not needed for departmental heads, different from the governing bodies of other
as the cost of training them would be very high. neighbouring cities.
(B) Senior level employees in the hierarchy can 39. Scientists have discovered that manganese
be given opportunities to train lower level containing Element Z is stronger and more flexible
employees. than ordinary manganese because Element Z
(C) Junior employees take more time in being reduces the problem of small breakages. The level
trained as compared to their senior managers or of Element Z in much of the manganese produced
leaders. in Australia is naturally high because of the natural
topography of the region.
(D) The training of higher level employees is more
important than that of people working at lower Which of the following can be correctly inferred from
levels of the organisation. the statements above?
(E) An incompetent departmental head does not (A) Manganese produced from Australian ore
respond to the training output despite spending deposits contains the highest levels of Element Z
time, money and effort on their training. found in any manganese.
37. A popular talk show host has been regularly making (B) Manganese from Australia is stronger and more
fun of a new movie on his programme for the past flexible than manganese from any other country.
few days. The director of the movie attributes the (C) Manganese that is not from Australia is very likely
poor performance of the movie to this fact. to encounter the problem of small breakages.
Which of the following options most seriously (D) Producing manganese from ore deposits
weakens the above argument? containing Element Z is the best way to make
(A) Several other film makers have also accused manganese that is strong.
the talk show host of talking poorly about their (E) Some manganese produced in Australia is less likely
movies. to develop small breakages than other manganese.

588

Book 1.indb 588 30/04/2019 4:49:35 PM


7.0  Logical Reasoning Practice

40. There are two ways of successfully completing a dwellers use for food and housing can be used by
marathon—practise running for 4 hours every day or rural dwellers for other needs.
make changes to your diet so that you are consuming Which of the following assertions is implied the most
more high energy foods. Research has clearly proved in the above statements?
that practising running is far more effective than
(A) People in both rural and urban areas tend to
making changes to your diet, as far as competing in a
spend a large proportion of their income on food
marathon is concerned. Thus, if a person consistently
and housing.
runs for at least 4 hours every day, he is doing the
most that can be done to perform well in the marathon. (B) The actual income of city dwellers is more as
compared to rural dwellers.
Which of the following options, if true, most seriously
weakens the above argument? (C) People living in rural areas have lower food and
housing costs than people living in cities.
(A) Some experienced marathon runners believe
that diet plays a more important role in their (D) While city dwellers earn more, they also spend
performance than exercise or running. more because of higher food and housing costs.
(B) The performance in the marathon can be (E) City dwellers save a larger proportion of their
improved further by employing both the income than rural dwellers.
strategies together. 43. Kavita is going to appear for an aptitude test in two
(C) Everyone who competes in a marathon does not months. To successfully clear this test, a person
necessarily do so for the purpose of winning it, needs to do two things—take 20 practice tests
but for its health benefits. and also attend 15 coaching sessions by a popular
(D) One of the most important aspects of performing teacher. Nobody who has failed to meet the previous
well in a marathon is the ability to manage your two criteria has ever cleared this test. If Kavita takes
pace. the 15 coaching sessions, she will only have enough
time to attempt 15 practice tests and if she attempts
(E) A marathon is as much a test of endurance as it
20 practice tests, she will only have enough time to
is of speed.
take 10 coaching sessions.
41. A company sells two types of mobile phones – A and The claims above, if true, most strongly support
B. The total sales of these two phones increased by which of the following conclusions?
24% between 2010 and 2015. In the same period,
(A) Kavita should take all 15 coaching sessions
however, the sales of ‘B’ type mobile phones, most
and then take as many tests as she can since
of which were sold from the company’s exclusive
tutoring is more important than practising.
outlets, grew by just 6%.
(B) As long as Kavita takes the 20 practice tests,
Which of the following conclusions about the
she will most likely be able to clear the aptitude
company’s sales from 2010-15 is best supported by
test.
the statements above?
(C) Kavita will retake the aptitude test if she is not
(A) Buyers were more likely to buy type ‘A’ mobile
able to successfully clear it the first time.
phones when they went to multi brand phone
sellers. (D) Kavita will most likely not be able to clear the
aptitude test.
(B) The prices of mobile phones purchased at the
company’s exclusive stores were higher than (E) If Kavita attends the 15 coaching sessions, she
those phones that were purchased elsewhere. will most likely be able to clear the aptitude
test.
(C) The sales of type ‘A’ mobile phones increased by
more than 24% during this period. 44. Conservation of tigers is a major concern for
(D) A majority of Type ‘B’ mobile phones were wildlife experts across the world since the number
purchased by corporate buyers who tend to of tigers left in the wild is rapidly declining. While
make bulk purchases. several countries are taking steps to counter this
fall, experts are unanimously of the opinion that if
(E) The number of people who bought mobile phones
the tiger is to survive in the wild, its best chances of
from multi brand sellers increased during this
doing so are in India.
period.
The claims above, if true, most strongly support
42. Economists have discovered that, at comparable which of the following conclusions?
income levels, people living in rural areas generally
have more purchasing power than people living in (A) The survival of the tiger is under threat in
cities. This is because some of the income that city countries other than India also.

589

Book 1.indb 589 30/04/2019 4:49:35 PM


NMAT by GMAC™ Official Guide 2019

(B) If the tiger fails to survive in India, it will become 47. India has decided to step up exports of various
extinct. consumer goods to China. (Real NMAT Question)
(C) The Indian government is not taking enough Which of the following statements strengthens India’s
steps to check the poaching of tigers. decision?
(D) The maximum number of tigers in the world are (A) China is the third largest market in the world for
found in India. these consumer goods.
(E) The number of tigers outside India is falling at a (B) The process of getting approval for trade in
faster rate than within India. China is tedious and takes a long time.
(C) China is known for manufacturing these products
45. Over the past 2 years, there has been a sharp decline at comparatively economical prices.
in the number of smokers in college campuses
across the city. Over this same period, the city’s (D) The tax paid to the Chinese government on the
governing council has spent a lot of money in coming total turnover in China has increased by 8%.
up with advertisements highlighting the harmful (E) Market barriers and cost competitiveness restrict
effects of smoking targeted at college students. The the entry and growth of companies in China.
authorities conclude that its advertisements have 48. The University has increased the pace of work for
been responsible for the drop in cigarette use at getting the NAAC accreditation, which is a must to
college campuses within the city. attract students from all over the country. It has
Which of the following options most seriously decided to get the accreditation in the next six
weakens the above argument? months. All the requirements for getting A+ grading
(A) There has been no similar decrease in the must be fulfilled. (Real NMAT Question)
consumption of alcohol among college students. Which of the following statements weakens the
(B) There has been no appreciable decrease in the decision of the University?
number of smokers within the city who are not in (A) A committee has been set up to prepare the Self
college. Study Report of the University in 2 months.
(C) The supply of cigarettes has remained (B) The required infrastructure will be built up by the
unchanged within the city. contractor in the next 2 years in a phased manner.
(D) A change in the taxation structure has led to a (C) The University Academic Council has done a
rapid increase in cigarette prices over the last detailed study of the requirements for NAAC
two years within the city. accreditation.
(E) Some studies show that there is no direct (D) The Board of Management will review the Self
relation between smoking and the susceptibility Study Report after 2 months for necessary
to diseases such as lung cancer. corrective actions.
46. The sale of badminton racquets in Chennai has tripled (E) The recruitment of the required minimum number
in the last year. Thus, it can be concluded that more of faculty members will also be done before the
and more people in Chennai have started playing inspection of the NAAC team.
badminton. 49. In the last one month, more than a dozen children
Which of the following options most strengthens the have been attacked by Rottweilers (a large dog
above argument? breed) when they stepped out of their house to
play. Thus, it is unsafe to keep dogs as pets in
(A) Chennai is famous for its badminton racquets
neighbourhoods with a large population of children.
and tourists often buy these racquets when they
visit Chennai. Which of the following options most strengthens the
above argument?
(B) The sales of badminton racquets in other cities
have not shown a similar increase. (A) Rottweilers have also occasionally attacked adults
accompanying the children and even other dogs.
(C) The majority of the badminton racquets sold in
Chennai were sold to residents of Chennai. (B) The Rottweiler is known to be a particularly
ferocious dog breed that can attack at the
(D) The sales of cricket bats and golf clubs have also
slightest of provocations.
shown an increasing trend in Chennai.
(C) Small dog breeds like the Pug are very friendly
(E) Badminton racquets are a popular gift item when
towards children and can be safely kept in
residents of Chennai visit their relatives in other
cities.

590

Book 1.indb 590 30/04/2019 4:49:35 PM


7.0  Logical Reasoning Practice

neighbourhoods with a large population of (C) The images produced by the new technology
children. require a longer time to dry and this can become
(D) Children need to be taught not to tease dogs or a deterrent to its adoption.
run when a dog comes towards them. (D) The companies had to consider the cost of
(E) The behaviour exhibited by Rottweilers towards switching from existing technology to a new one
children is representative of the behaviour of all and that cost was considerable.
dogs in general. (E) The people handling the purchase decisions are
not aware of technological developments and
50. A notice in a cinema hall read: “If viewers with a Gold hence take erroneous decisions.
ticket are found sitting in Platinum category seats,
they will be fined”. (Real NMAT Question) 53. A team leader has left the company and the manager
urgently needs to put a replacement in place. A
Which of the following statements can be inferred
precious amount of time in the project has already
from the given statement?
been lost and the product launch has already been
(A) There are only two categories of tickets in the announced by the client. Stakes are high and time is at
cinema hall. a premium. Besides, the team which comprises of very
(B) Gold and Platinum tickets cost the same. highly skilled individuals is becoming edgy due to the
(C) Viewers with Gold tickets cannot sit in the front sheer pressure.
seats. What decision should the manager take if they have
(D) Viewers with Platinum tickets can sit in the Gold to choose between a person with very high people
category seats. management and interpersonal skills and a person
known to be short-tempered but highly skilled in
(E) Inspections are carried out in the cinema hall to
multiple domains to be used in the project? (Real
check the tickets.
NMAT Question)
51. The annual report of a government agency revealed (A) Choose the person with multiple domain
that despite an 8% fall in sales volume of tobacco in expertise - different team members would need
the year 2012-13, the revenue and consequently tax support in different domains to complete the
collection from its sales, had gone up by almost 3% work.
in the same period. (Real NMAT Question) (B) Choose the person with people management
Which of the following can be inferred from the above? skills - both the team and the client would need
(A) The government wants to ban tobacco use. high levels of engagement.
(B) New tobacco users are added every year to (C) Choose the person with multiple domain
boost tobacco sales. expertise - the team would need such a person,
should anyone else leave.
(C) Tobacco consumers are not sensitive towards
the price of tobacco. (D) Choose the person with multiple domain
expertise - a lot of time has already been lost and
(D) There are no alternative products which could
time is at a premium.
satisfy the same needs.
(E) Choose neither - you need to seek HR
(E) Due to an increase in taxes, the sale of tobacco
intervention, advertise and get someone else
products in the black market has increased.
recruited at the earliest.
52. A new technology in laser printing is set to improve 54. A major part of the rabi crop in the district was
the quality of images considerably for the same cost. damaged due to unseasonal heavy rains during
However, a survey revealed that a lot of companies the last few days of winter. Many farmers were
did not plan to switch over to the new technology in committing suicide. (Real NMAT Question)
the immediate future. (Real NMAT Question)
Which of the following is implied in the above What decision do you think the head of the agriculture
statement? department should take?
(A) Some companies think that the new technology (A) The agriculture department should counsel the
might not be successful. farmers.
(B) The quality of the images is not the only thing (B) The agriculture department should update the
that the companies look for when deciding on central government and wait for the Centre’s
printing solutions. directives.

591

Book 1.indb 591 30/04/2019 4:49:35 PM


NMAT by GMAC™ Official Guide 2019

(C) It must be understood that the agriculture (E) Terminate all contracts that the company has
department could have done nothing as it was a with the tainted player with immediate effect and
force majeure. not take any athlete as a brand ambassador of
(D) The agriculture department should look at the company in future.
granting relief to the affected farmers by waiving 57. At an international athletic event, some of the female
loans and providing free seeds. athletes felt that one of their compatriots in the
(E) The agriculture department should waive loans 200 m running event was actually a male. They
and encourage the farmers to go to other parts complained to the International Body for Sports (IBS).
of the country where rains are not heavy. Consequently, IBS asked the athlete to undergo a
gender test. The sports authority of the athlete’s
55. A lot of development work is being done in the country refused to let the athlete undergo the test
hills. There are better roads being constructed and as it was forbidden to do so in their culture. They
tourism is getting a big thrust. The local residents warned that if IBS forced the test, the country
too are getting access to the latest technology along with its neighbouring countries, would end all
and getting better housing facilities. Not only is the affiliations with IBS. If that were to happen, IBS would
economy progressing but the government too is lose a lot of revenue and it would be a major setback
benefitting from tax revenues. A local resident of for all future international sports tournaments. (Real
the area is very worried about the ill effects that NMAT Question)
the development work is having on the natural
What should the IBS do?
environment. (Real NMAT Question)
(A) Get the test done at any cost as IBS is the apex
What should the local resident do to save the
body and no one can hold it to ransom.
environment?
(B) No international events should be organised in
(A) Hold meetings with other residents and try to
future so that such issues won’t occur again.
hinder this development.
(C) The athlete shouldn’t be forced to undergo the
(B) Go on a hunger strike and ask the government to
test as it would have financial implications on the
stop the development work, or else they will fast
future of sports.
until death.
(D) Cancel the ongoing event until further notice
(C) Let the development go ahead as the local
and, in the meantime, resolve the issue with the
resident does understand that development
sports authority of the athlete’s country.
happens at some cost to the environment.
(E) Talk to the sports authority of the athlete’s
(D) Meet the government officials concerned and
country to try and convince them to let the
ask them to rationalise the development so that
athlete undergo the test, and let the other events
nature is not affected.
continue as scheduled.
(E) Abduct a high ranking government official and
release him only after the government promises 58. Doors Inc. is a technology company that focuses
to stop the development work in the hills. on creating operating systems, which are software
packages that run everything on a computer. The
56. A well-known athlete is the brand ambassador of a company is releasing the newest version of its
company. Recently, the media has shown reports operating system and is deciding on the appropriate
claiming that the athlete was involved in doping and pricing strategy. The CEO favours an aggressive
cheating cases. (Real NMAT Question) market penetration strategy in order to gain market
What should the Public Relations manager of the firm do? share, even if it means pricing the operating system
(A) The media has a habit of tarnishing images of such that the company would lose money on every
renowned people, he should not pay too much sale. (Real NMAT Question)
attention to it. Which of the following, if true, would most strengthen
(B) Immediately cancel the contract of the player so the argument for adopting a price skimming strategy?
that the company’s image does not suffer. (A) Existing operating systems satisfy most
(C) Let the advertisements run and let the contract customer requirements.
be intact as the final hearings of the cases are (B) Large organisations are eager to adopt the newer
not complete. version because of improved enterprise functions.
(D) Take all the advertisements featuring the player (C) Two other competitors are expected to release
off air but not cancel the contract until the cases newer versions of their operating systems in the
and charges are settled. coming weeks.

592

Book 1.indb 592 30/04/2019 4:49:35 PM


7.0  Logical Reasoning Practice

(D) New sales are expected to be low, with most (B) The increasing population of India also allows for all
sales being of the less-profitable upgrades of the kinds of products to find a market in India.
operating system. (C) India will have the maximum number of diabetic,
(E) The majority of sales are expected to occur high blood pressure and hepatitis patients in the
through original equipment manufacturers, which future.
are companies that manufacture computers and (D) With the rise of per capita income, the spending
sell them bundled with the operating system. on drugs and pharmaceuticals will triple in the
59. The more you practise, the more proficient you will next decade.
become. (Real NMAT Question) (E) The Indian consumer currently spends very little on
drugs and pharmaceuticals and opts for traditional
Which of the following strengthens the above statement?
methods/remedies for treatment.
(A) She takes after her dad—she is as good in
badminton as he is. 63. Statement: Rebellion is known as the disobedience
(B) However much he tries to solve the questions, he to a lawful authority that is caused due to widespread
just does not seem to be getting the hang of it. discontent among certain groups of people. When a
(C) No amount of practice is helping—I seem to be government wants to tackle such a situation, it should
committing the same mistake over and over again. maintain calm and continue to urge the rebels to
hold talks with the authority in order to find a solution
(D) She must be a genius. With a hectic schedule,
together. Therefore, the government of the day should
she hardly gets time to practise and yet, she is
be mindful, tolerant, and open to peaceful negotiation.
one of the top contenders for the trophy.
Which of the following, if true, could weaken the
(E) Rajesh takes less than a minute to solve
argument above?
algebraic equations—he loves algebra and can
be seen solving the practice questions for hours. (A) When a government shows lack of understanding of
its own people, it causes discontentment among its
Directions for Questions 60–61: It is sometimes mooted population that results in defiance of its authority.
that there can be democracy in a two party system. That
(B) People in some part of any country are, by nature,
would be correct if politics were a game like cricket or
rebellious and hence cannot be made to follow
football; but politics is not sport.
discipline.
60. Which of the following would strengthen the argument? (C) Government of any country, by nature, tends
(A) Two party system is practiced in the maximum to be authoritative and may not like to be
number of democracies in the world. challenged whatever maybe its nature or method
(B) Politics is a dirty game. of governance.
(C) Two political parties limit the choice of the voters. (D) It is found that dictators with an iron hand approach
(D) Sports, just like politics needs to be fair. have dealt with rebellion successfully.

(E) A two party system is a waste of time and energy. (E) When a government shows care for its own people,
it causes contentment among its population and no
61. Which of the following would weakens the argument? talk is required for solving any problem.
(A) Two party system is practiced in the maximum
number of democracies in the world. 64. Statement: Should the Western way of life be adopted
here in India? (Real NMAT Question)
(B) Politics is a dirty game.
Arguments:
(C) Two political parties limit the choice of the voters.
I. No. The Western culture is quite different to the
(D) Sports, just like politics needs to be fair. Indian culture, hence should not be blindly followed.
(E) A two party system is a waste of time and II. Yes. We should be open to accepting the good
energy. from all cultures, and have a positive mindset.
62. By 2020, India will emerge as one of the top five III. No. The West has already spoilt our people a lot
global markets for pharma products. (Real NMAT and it’s time we turned our backs on them.
Question) (A) Only I is strong.
Which of the following statements weakens this (B) Only II is strong.
possibility for India?
(C) Only III is strong.
(A) There is a high domestic demand and increased
consumer spending on drugs in India. (D) Both I and II are strong.
(E) None of them are strong.

593

Book 1.indb 593 30/04/2019 4:49:35 PM


NMAT by GMAC™ Official Guide 2019

Directions for Questions 65–66:  Each question given Which of the following arguments strengthens the
below consists of a statement, followed by two arguments above statement?
numbered I and II. You have to decide which of the (A) Children are known to come up with dumb
arguments is a ‘strong’ argument and which is a ‘weak’ excuses.
argument.
(B) In adults, the ability to digest milk is a genetic
Give answer: adaptation.
(A) If only argument I is strong (C) Milk intake is good as it increases the calcium
(B) If only argument II is strong levels in the body.
(C) If either I or II is strong (D) Children who avoid milk face an increased risk of
(D) If neither I nor II is strong and pre-pubertal bone fracture.
(E) If both I and II are strong. (E) It has been scientifically proven that there are
lactose-intolerant people who cannot digest milk.
65. Statement: Should Drinking be prohibited?
Questions 69–70: Consist of a statement followed by two
Arguments:
arguments numbered I and II. Decide which of the Arguments
I. Yes. It is wrong to drink away one’s money. is strong. Select from the following options.
II. No. Thousands of workers in the wine industry (A) Only argument I is strong
will be rendered unemployed.
(B) Only argument II is strong
66. Statement: Should teaching of Tamil be made (C) Either argument I or II is strong
compulsory at school level in Tamil Nadu?
(D) Neither argument I nor II is strong
Arguments:
(E) Both arguments I and II are strong
I. No, where are the trained teachers to teach this
language? 69. Statement: Should all the illegal construction that
has been carried out in the city by unscrupulous
II. Yes, Tamil Nadu’s people should be proud of their
builders be demolished?
ancient language.
Arguments:
Directions:  Given alongside is a statement followed by
I. Yes. This will dissuade such builders from
three arguments numbered I, II and III. Decide which of the
carrying out such activities in future and also
given arguments is strong and is able to support the given
punish people for buying such properties.
statement. (Real NMAT Question)
II. No. There are people living in these buildings
67. Statement: Should CBSE continue with the new who will have nowhere to go.
grading system which is based upon the teacher’s
informal evaluation of pupils rather than pupil 70. Statement: Has the easy availability of consumer
examination? loans made life easier for the Indian consumer?
Arguments: Arguments:
I. Yes. Informal evaluation reduces the pressure felt I. Yes. The consumers can now buy items that they
by students. couldn’t afford earlier.
II. No. The pressure of examinations prepares II. No. The easy availability of these loans can make
pupils for the pressures they will face in later life. consumers buy unnecessary things.
III. No. Exams improve a student’s manners and Directions for Questions 71–72: Each question given
etiquettes. below consists of a statement, followed by two arguments
(A) Only I is strong. numbered I and II. You have to decide which of the
arguments is a ‘strong’ argument and which is a ‘weak’
(B) Only II is strong.
argument. Select from the following options:
(C) Only III is strong.
(A) Only argument I is strong
(D) Both I and II are strong.
(B) Only argument II is strong
(E) Neither of them are strong.
(C) Either argument I or II is strong
68. Children are often heard saying that they do not want (D) Neither argument I nor II is strong
to drink milk as they cannot digest it. (Real NMAT
(E) Both arguments I and II are strong
Question)

594

Book 1.indb 594 30/04/2019 4:49:35 PM


7.0  Logical Reasoning Practice

71. Statement: Should more new universities be set up (B) Only argument II is strong.
in India? (C) Both I and II are strong arguments.
Arguments: (D) Either I or II is strong.
I. No, the government will have to spend a lot of (E) Neither I nor II is strong.
money in setting up new universities.
75. A statement is given followed by two assumptions
II. Yes, setting up more universities will boost foreign numbered I and II. An assumption is something
interest in the country’s education system. supposed or taken for granted. Consider the
72. Statement: Should the government shut down all statement and the following assumptions and decide
loss-making public-sector enterprises? which of the assumptions is implicit in the statement.
(Real NMAT Question)
Arguments:
Statement: Mohan requested that his mother
I. No. This will result in a loss of employment for
arrange food for about 30 persons as he had invited
hundreds of people.
all his friends to celebrate his birthday.
II. Yes. The government should minimise its loss
Assumption:
and spend the amount on creating alternative
employment opportunities I. Most of Mohan’s friends may come to his house
on his birthday.
73. Directions: The given question is followed by two
II. There may be more than 30 friends who may
arguments numbered I and II. Decide which of the
attend Mohan’s birthday party.
arguments is strong and is able to support the
statement. (Real NMAT Question) (A) Only Assumption I is implicit.
Statement: (B) Only Assumption II is implicit.
Will the latest bill on curbing corruption lead to a (C) Both I and II are implicit.
blemish-free administration? (D) Either I or II is implicit.
Arguments: (E) Neither I nor II is implicit.
I. Yes, it will tighten the noose on all politicians.
Directions for Questions 76–80:  For each of the following
II. No, there will still be loopholes that politicians questions there is one argument followed by two statements.
can use. Check whether the statements are assumptions made in
(A) Only Argument I is strong. the argument to arrive at the conclusion made. Give your
(B) Only Argument II is strong. answer.
(C) Neither Argument I nor Argument II is strong. (A) If only Assumption I is implicit in the argument
(D) Either Argument I or Argument II is strong. (B) If only Assumption II is implicit in the argument
(E) Both Argument I and Argument II are strong. (C) If either Assumption I or Assumption II is implicit
in the argument
74. Directions: Given alongside is a statement followed
(D) If neither Assumption I nor Assumption II is
by two arguments, numbered I and II. Decide which of
implicit in the argument
the given arguments is strong and is able to support
the given statement. (Real NMAT Question) (E) If both Assumptions I and II are implicit in the
argument
Statement:
Is India strong enough to be a dominant power in the 76. Argument: Ravi Sharma’s new book is refreshingly
21st Century? different. It is based on the emotions that the youth
faces in today’s metros as they grow up. This book
Arguments:
will definitely be his first best-seller.
I. Yes, India can become a world power in the 21st
Assumptions:
century with the resources that it has – both,
natural and human. India is also performing well I. All books based on the emotions of the youth will
as a leader. be best-sellers.
II. No, poverty and disease is widespread. There is II. Ravi Sharma did not have a best-seller till date.
still the issue of the caste system, which controls 77. Argument: Niloufer has received the Employee of
its democracy. The country must first decide on the Year award at the Macrosoft Annual Convention.
its priorities and set its problems right. She has also clocked the most number of hours at
(A) Only argument I is strong. work compared to all other colleagues.

595

Book 1.indb 595 30/04/2019 4:49:36 PM


NMAT by GMAC™ Official Guide 2019

Assumptions: Which of the following, if true, may be the most


I. ‘Employee of the Year’ is usually awarded to plausible cause of the situation for the spread of the
employees who work longer hours. above mentioned diseases?
II. Niloufer is quite friendly with her bosses. (A) The lifestyle and food habits of people, if not
healthy, cause many diseases.
78. Argument: Madhu cleaned her room in two days
(B) People themselves are responsible for the
whereas Sapna, who is her elder sister, cleaned her
environment they live in.
room in three days. It is evident that Madhu cleans
rooms quicker than Sapna does. (C) People are aware of factors that can contribute
to lifestyle related diseases.
Assumptions:
(D) Fast pace technological advancement is rampant
I. Madhu’s and Sapna’s rooms are of the same size. these days contributing to the spread of diseases.
II. Sapna loves to clean whereas Madhu loves to (E) Cancer and AIDS are not avoidable by changing
play games. lifestyle and food habits.
79. Argument: Meena has taught in more than ten Directions for Questions 83–85:  In each question below
schools in the past fifteen years. With her diverse is given a statement followed by two assumptions numbered
experience in teaching, she should prove to be a very I and II. An assumption is something supposed or taken
good teaching resource for your new school. for granted. You have to consider the statement and the
Assumptions: following assumptions and decide which of the assumptions
I. Meena is extremely qualified for the job. is implicit in the statement.

II. Teaching in many schools for many years makes Give answer If
one a very good teaching resource. (A) Only assumption I is implicit,
(B) Only assumption II is implicit,
80. Argument: The weather department has forecast an
early monsoon this year. Advisories have been issued (C) Either assumption I or II is implicit,
from all local stations to farmers for hastening the (D) Neither assumption I nor II is implicit,
harvest of horticulture crops and preparing the land (E) Both assumptions I and II are implicit.
for sowing of grain crops.
83. Statement: Rahul’s new car ran off the road
Assumptions: causing some damage to the front part of the car.
I. The weather department’s forecast is accurate. He is claiming insurance on the repairs needed. His
insurance premium will shoot up.
II. Farmers cannot take any decisions without the
help of the weather department. Assumptions:
I. Insurance premium shoots up if you claim
81. Statement: A very reputed non-banking finance
insurance on the damage of a new car.
company has launched a loan scheme for buying new
agricultural tools. The farmers have responded well II. Insurance premium shoots up if you claim
to the scheme and it has become successful as it insurance for damage to the front part of a car.
has brought prosperity among the farmers. 84. Statement: The ‘Everybody Party’ has come up with
Which of the following assumptions is implicit in the a new sop in its election agenda. It is promising to
above statements? increase the upper limit of LPG cylinders’ ration per
(A) The prosperity of farmers is the only measure of family, from 12 to 14 per year. This sop is expected
the success of a loan scheme. to give the party a clear edge over other parties in
garnering the middle-class votes.
(B) Only tools are important for farmers to enhance
farm activities. Assumptions:
(C) Earlier there was no such loan scheme available I. The new sop is not promised by any other party.
for farmers. II. Middle-class voters will benefit from the raised
(D) Farmers do not prefer taking loans for buying ration limit of LPG cylinders.
agricultural tools on their own. 85. Statement: The new mobile phone model of ‘Same
(E) The prosperity of farmers is an important Song’- Milky Way KD4, is lighter, larger and faster
indicator for the success of a loan scheme. than the previous models. In the first week of its
82. Statement: According to a recent survey, innumerable release, the company sold a record 1.3 million sets
diseases ranging from cancer to AIDS, among others, across the world. This model will be the highest
have plagued mankind due to unhealthy habits. selling model in the sales history of the company.

596

Book 1.indb 596 30/04/2019 4:49:36 PM


7.0  Logical Reasoning Practice

Assumptions: (D) Only IV is implicit.


I. No other model of ‘Same Song’ has sold 1.3 (E) None of these assumptions is implicit.
million pieces in the first week.
89. Statement: Children less than five years of age need
II. All customers prefer lighter, larger and faster to be continually kept mentally stimulated.
models of mobile phones.
Assumptions:
Directions:  A statement is followed by some assumptions. I. A large part of the development of intelligence
An assumption is something considered true or taken for and other social skills takes place in the early
granted. Consider the statement and the assumptions and years of a child.
decide which of the assumptions is implicit in the statement.
II. 40 percent of a person’s intelligence at the age
86. Statement: The police were still not aware of the of 22 can be predicted by the age of five.
contents of the hard disk seized from the residence
of Troccoli. The experts of D-DAC, who had come (A) Only I is implicit.
from Thiruvananthapurm to help decipher the data on (B) Only II is implicit.
the hard disk, had gone back. (C) Only III is implicit.
Assumptions: (D) Only IV is implicit.
I. Troccoli is involved in terrorist activities. (E) None of these assumptions is implicit.
II. Data in hard disks are usually in code language.
90. Statement: The Apogee Laptop Company has
III. The D-DAC experts are from the police decided to increase the price of its laptops by 25
department. percent with immediate effect.
IV. The decoding of the hard-disk data could help Assumptions:
the police in their investigation. (Real NMAT
Question) I. Other laptop manufacturers will also soon raise
the prices of their respective laptops since
(A) Only I is implicit. Apogee is the market leader.
(B) Only II is implicit. II. The Apogee Company does not expect the
(C) Only III is implicit. demand for its laptops to go down considerably
(D) Only IV is implicit. after this hike.
(E) None of these assumptions is implicit. (A) Only I is implicit
(B) Only II is implicit
87. I need absolute silence for some time. Please do not
let Rakesh come anywhere near me, not for at least (C) Either I or II is implicit
an hour. (Real NMAT Question) (D) Neither I nor II is implicit
What is implied in the above? (E) Both I and II are implicit
(A) Rakesh is a small child.
Question 91: Consists of a statement followed by two
(B) Rakesh is not very popular. assumptions. You have to consider the statement and the
(C) Rakesh is a spoilt teenager. two assumptions and decide which of the assumptions is
(D) Rakesh will come with noise. implicit in the statement. Select from the following options:
(E) Rakesh is the neighbour’s noisy child. (A) Only I is implicit
(B) Only II is implicit
88. Statement: Of late, incidents of food poisoning
caused by the consumption of grains mixed with (C) Either I or II is implicit
impurities have been taking place in rural areas. (D) Neither I nor II is implicit
Assumptions: (E) Both I and II are implicit
I. There are shops selling grains mixed with 91. Statement: Suresh has applied for a loan of Rs. 60000
impurities in rural areas. from his bank to pay for his son’s educational expenses.
II. The percentage of people consuming grains is Assumptions:
higher in rural areas.
I. The bank will reject the loan because it has a policy
(A) Only I is implicit. of not awarding loans for education purpose.
(B) Only II is implicit. II. Suresh has failed to arrange for the money from
(C) Only III is implicit. all other sources available to him.

597

Book 1.indb 597 30/04/2019 4:49:36 PM


NMAT by GMAC™ Official Guide 2019

92. Directions: A statement is given followed by two 95. In the National Park, the white tigers hardly moved
assumptions numbered I and II. An assumption is around, they always looked for opportunities to lie
something to be taken for granted. Consider the around. (Real NMAT Question)
statement and the assumptions, and decide which of
From which of the following can the above statement
the assumptions are implicit in the statement. (Real
be most properly inferred?
NMAT Question)
(A) White tigers are usually known to be lazy.
Statement:
(B) The National Park makes the tigers active.
Sachin scored a century in the last match against
(C) Only white tigers are known to hunt their food
Australia.
aggressively.
Assumptions:
(D) If an active tiger is seen at the National Park, it
I. Sachin participated in the last match against must not be the white tiger.
Australia.
(E) Due to their appearance, white tigers attract
II. India may win this match against Australia. a lot of attention, which in turn makes them
(A) Only I is implicit. aggressive.
(B) Only II is implicit.
Directions for Questions 96–101:  For each of the
(C) Both I and II are implicit. following questions there is one argument followed by
(D) Either I or II is implicit. two statements. Check whether the statements are valid
(E) Neither I nor II is implicit. inferences drawn from the argument. Give your answer.
(A) If only Inference I can be inferred from the
93. Directions: Given alongside is a statement followed argument.
by two assumptions numbered I and II. An assumption
is something to be taken for granted. Consider both (B) If only Inference II can be inferred from the
the statement and the following assumptions, and argument.
decide which of the assumptions are implicit in the (C) If either Inference I or Inference II canbe inferred
statement. (Real NMAT Question) from the argument.
Statement: (D) If neither Inference I nor Inference IIcan be inferred
Some of the best restaurants in the country are found from the argument.
in Kalikapur. (E) If both Inferences I and II can be inferred from the
Assumptions: argument.
I. The country boasts of some really world class 96. Argument: The East coast of the country is prone
restaurants. to natural disasters in the form of thunderstorms
II. Kalikapur has access to some of the best chefs in and cyclones, which occur for a couple of weeks
the country. during the hot season. For most part of the year, the
weather is dry and winds are mild along that coast.
(A) Only I is implicit.
(B) Only II is implicit. Inferences:
(C) Neither I nor II is implicit. I. Hot season lasts for only two weeks on the East
coast of the country.
(D) Either I or II is implicit.
II. For most part of the year the east coast is safe
(E) Both I and II are implicit.
from disasters.
94. The Chairman, Hamid Ansari, tried to restore order 97. Argument: A number of research studies report that
in the House and called for the first listed question of the correlation between incomes and residence
the day but BJP members were unrelenting. He then in slums is nowhere near perfect. People living in
adjourned the House till noon. (Real NMAT Question) slums may actually be well above the conventional
What can be inferred from the above argument? standards of poverty as measured by urban poverty
(A) BJP members like creating nuisance. lines.
(B) No one cares what the chairman wants. Inferences:
(C) Questions that are asked in the House are listed. I. Some people live in slums despite being able to
(D) The Chairman has the authority to adjourn the House. afford a house elsewhere.

(E) Adjourning of the House becomes essential when II. Poverty lines do not measure incomes
some members are unrelenting. accurately.

598

Book 1.indb 598 30/04/2019 4:49:36 PM


7.0  Logical Reasoning Practice

98. Argument: The deforestation activities in the (C) The Government is taking the necessary steps to
Amazon have generated tremendous revenue for the facilitate female passengers on Raksha Bandhan.
small South American nations. Now the people of (D) Male passengers will be provided alternative
this region are well-fed, owing much to the state- means of travel on Raksha Bandhan to avoid
sponsored food security programmes. overcrowding on buses.
Inferences: (E) The number of female passengers shall not
I. People of the South American nations are not exceed the total available seats in the bus to
self- reliant in buying food. avoid overcrowding this year.
II. The small nations in the South American region 103. If we throw ethics to the wind and dance with the
invest a part of their revenues on food security. powerful, we must remember that we may step on the
99. Argument: Gold prices have dropped in the recent devil’s tail. So, let’s dance with caution. (Real NMAT
months due to a fall in demand for gold bars. This is Question)
contrary to the normal trend seen during the festive Which of the following is a conclusion that can be
season where demand for gold ornaments is higher drawn from the above statement?
that results in an increased demand for gold bars. (A) The powerful are devils.
Inferences: (B) The powerless are not devils.
I. The general belief is that gold prices rise during (C) Dancing with the powerful is necessary.
the festive season. (D) One has to be careful and wise while dealing with
II. Demand for gold bars is not connected with the power.
demand for gold ornaments. (E) One has to throw ethics to the wind if one wants
100. Argument: programmes on television that are to dance with the devil.
targeted at women usually revolve around recipes 104. Directions: Given alongside are a few facts.
and hints on household management. The print Based on these facts, select from among the
media also follows suite. given statements, the statement that can be best
Inferences: concluded. (Real NMAT Question)
I. Women are interested only in such matters. Facts:
II. Women are not interested in sports. Fact 1: Graphics can tell news.
101. Argument: In an IPL match held in South Africa a Fact 2: All newspapers have graphics.
team scored 198 runs of which 100 runs were made Fact 3: Some newspapers have text.
by all-rounders.
Statements:
Inferences: I. Some newspapers have both text and graphics.
I. 
More than half of the team consists of all- II. Graphics can tell news better than text can.
rounders.
III. The news in newspapers is very interesting.
II. Most of the all-rounders were spinners.
(A) Only I can be concluded.
102. The Government of Rajasthan has instructed the (B) Only II can be concluded.
State Roadways Transport Corporation not to charge
any money from female passengers in lieu of tickets (C) Only III can be concluded.
on the day of the Raksha Bandhan festival. The (D) Both II and III can be concluded.
number of buses running on different routes has (E) None of the statements can be concluded.
also been increased, as the buses were extremely
overcrowded in the previous year on Raksha 105. Directions: Three facts are given. Treat these facts
Bandhan. (Real NMAT Question) to be true and then, select from among the given
statements, the statement that can be concluded to
Which of the following inferences can be drawn from be a fact. (Real NMAT Question)
the given statement?
(A) Male passengers cannot travel on buses on Facts:
Raksha Bandhan. Fact 1: A is B’s sister.
(B) Female passengers can take their children too, Fact 2: A’s son is C.
free of cost on Raksha Bandhan. Fact 3: C’s aunt is B.

599

Book 1.indb 599 30/04/2019 4:49:37 PM


NMAT by GMAC™ Official Guide 2019

Statements: (A) Only I can be concluded.


I. A has no brother. (B) Only II can be concluded.
II. B’s nephew is C. (C) Only III can be concluded.
III. B is unmarried. (D) Both I and III can be concluded.
(A) Only I can be concluded to be a fact. (E) None of the statements can be concluded.
(B) Only II can be concluded to be a fact. 108. The Delhi Police has instructed all police stations in
(C) Both I and III can be concluded to be facts. Delhi to keep a check on cyber cafes operating in
(D) Both II and III can be concluded to be facts. their area. They have instructed that nobody should
use the internet facility at cyber cafes without showing
(E) None of the statements can be concluded to be
an identity proof. The number on the identity proof,
a fact.
such as PAN card, Vote ID card, or Driving Licence,
106. I will host a show if, and only if, the co-host is Hrithik must be noted down in a register, along with the
Roshan. (Real NMAT Question) date, time and duration of internet surf-time, and the
What can logically be concluded from the above computer used to surf the internet. This is to help
statement? reduce misuse of the internet by terrorists at these
cyber cafes. (Real NMAT Question)
(A) I have to host the show, Hrithik will co-host the
show. Which of the options listed below are conclusions that
can be derived from the given information?
(B) When I host the show, the co-host is Hrithik
Roshan. (A) The required stationery and other items will be
made available by the Police.
(C) The co-host is Ranbir Kapoor and I could be
hosting the show. (B) Cyber cafes will have to spend on photocopies of
the identity proofs of users.
(D) If the co-host is not Hrithik Roshan, my friend is
hosting the show. (C) Delhi Police is intending to create unnecessary
problems for internet users in Delhi.
(E) Both (A) and (B)
(D) Nobody will be able to access the internet at
107. Directions:  Given alongside are a few facts. cyber cafes in Delhi if he/she does not carry any
Based on these facts, select from among the given proof of identity.
statements, the statement that can be concluded to
(E) The records maintained by the cyber cafe owner
be a fact. (Real NMAT Question)
will be submitted to the respective police stations
Facts: every month.
Fact 1: ABC public school has 300 students. 109. Statement:
Fact 2: At least 1/3rd of them have chosen horse Aluminum manufactured in India is of a much better
riding as a hobby. quality than the aluminum manufactured in other
Fact 3: At least 1/6th of them have chosen art as a countries across the world.
hobby.
Conclusion I: Indian aluminum manufacturers make
Fact 4: Almost half of them have chosen creative huge profits on the sale of their aluminum.
writing as a hobby.
Conclusion II: The manufacturing process employed
Fact 5: Students are allowed to choose more than
by aluminum manufacturers in India contributes to its
one hobby only if they wish to do so.
better quality.
Statements: (A) Only I can be inferred.
I. At least one student chooses two activities. (B) Only II can be inferred.
II. The majority of the students at ABC choose (C) Either I or II can be inferred
creative writing.
(D) Neither I nor II can be inferred
III. There are 3 hobbies offered at the school - horse
(E) Both I and II can be inferred
riding, art and creative writing.

600

Book 1.indb 600 30/04/2019 4:49:37 PM


7.0  Logical Reasoning Practice

2. Analytical Reasoning

(D) Either P or T
(E) Either P or R
Directions for Questions 1–4:  Read the below information Directions for Questions 6–9:  Read the below information
and answer the questions that follow. and answer the questions that follow.
Six people: C, D, E, F, G and H are standing in a straight line Eight friends N, L, M, R, P, Y, T and Q are sitting around
facing North, not necessarily in the same order. F is standing a circular desk facing away from the centre. Each friend
second to the left of D. C is standing fourth to the left of H has a different car—Lambretta, VW, Lycan, Creta, Ariel,
and H is not standing on the extreme end of the line. D is Garmin, Punto and Scorpio, but not necessarily in that
standing second to the left of E. order. T is sitting third to the right of P. The one who
owns Lambretta is second to the left of the one who owns
1. What is the position of F with respect to E? Lycan. Y owns Lycan and is sitting exactly between P and
(A) Immediate left L. The one who owns Punto is sitting second to the right
(B) Fourth to the left of N. The one who owns Ariel is second to the right of the
person who owns Garmin. P sits third to the left of the
(C) Third to the left
person who owns VW. Neither Q nor L is the immediate
(D) Third to the right neighbour of N. Q is fourth to the left of L. N does not
(E) None of these own Creta or Ariel. The person who owns Lambretta is
sitting second to the right of the person who owns Creta.
2. Which of the following pairs represents the people
R owns Lambretta and he is not an immediate neighbour
standing at the extreme ends of the line?
of N.
(A) CE
(B) FH 6. Who sits fourth to the right of L?
(C) FE (A) P
(D) CH (B) Y
(E) None of these (C) R
(D) Q
3. Who is standing second to the right of F?
(E) None of these
(A) H
(B) D 7. Which of the following cars does N own?
(C) G (A) Punto
(D) E (B) Scorpio
(E) None of these (C) Garmin
4. Four of the following five pairs are alike in a (D) VW
certain way based on their position in the above (E) Lycan
arrangement and so form a group. Which of the
following pairs do not belong to the group? 8. What is P's position with respect to Q?
(A) GC (A) Third to the left
(B) GE (B) Fifth to the left
(C) HG (C) Second to the right
(D) DE (D) Third to the right
(E) FD (E) Fourth to the left

5. Five stations P, Q, R, S and T are located on a highway 9. Which of the following combinations is correct?
but not necessarily in the same order. Q is the middle
(A) R - Garmin
station which is towards the immediate left of R. S is
not the first station but is towards the left of Q. T is (B) Y - Creta
towards the right of S. Which station is located to the (C) N - Scorpio
immediate left of S? (Real NMAT Question) (D) T - Ariel
(A) P (E) All are true
(B) R
(C) T

601

Book 1.indb 601 30/04/2019 4:49:37 PM


NMAT by GMAC™ Official Guide 2019

Directions for Questions 10–13:   Read the following Directions for Questions 14–17:  Read the below
information and answer the questions that follow. information and answer the questions that follow.
Sarah, Caitlin, Megan, Amy, Jessica, Emma, Amber and Zoe Six friends—S, R, P, D, M and K—are sitting around a
are sitting around a circular table facing the centre. Each circular sofa. S is sitting opposite to R. P is sitting to the
one of them has a favourite fruit or vegetable—dandelion, right of R but left of D. M is sitting to the left of R. K is sitting
eggplant, apple, tomato, cherry, spinach, broccoli and to the right of S and left of M. Now, D and K interchange
beetroot. Sarah sits third to the right of the person who likes their positions and so do M and R.
tomato. Amy sits second to the left of Amber.
14. Who will be sitting second to the right of D?
Amber is not an immediate neighbour of the person who (A) S
likes tomato. Only one person sits between Caitlin who likes
(B) M
spinach and the person who likes tomato.
(C) R
The person who likes apple sits third to the right of the
(D) P
person who likes spinach. Zoe sits between the person who
likes apple and the person who likes eggplant. Jessica is not (E) K
an immediate neighbour of Zoe.
15. Four of the following five pairs are alike in a
The person who likes eggplant sits third to the right of the certain way based on their positions in the above
person who likes cherry. Only one person sits between the arrangement and so form a group. Which of the
person who likes broccoli and Emma. Jessica likes neither following pairs do not belong to the group?
broccoli nor dandelion. (A) SR
10. Who sits third to the right of Megan? (B) DM
(A) Amber (C) PS
(B) Emma (D) KM
(C) The person who likes dandelion (E) RP
(D) Zoe
16. Who will be sitting opposite to S?
(E) The person who likes beetroot
(A) R
11. Which of the following statements must be true?
(B) M
(A) Zoe likes dandelion.
(C) K
(B) The person who likes cherry sits to the
(D) D
immediate left of Sarah.
(E) P
(C) Amy sits two places to the right of Amber.
(D) Caitlin sits two places to the left of the person 17. Who will be sitting to the immediate left of K?
who likes tomato. (A) P
(E) Jessica likes broccoli. (B) D
12. What is the position of the person who likes broccoli (C) R
with respect to Zoe? (D) S
rd
(A) 3 to the left (E) None of these
(B) 4th to the right
18. Directions: Read the following information and
(C) 5th to the left answer the question. (Real NMAT Question)
(D) 4th to the left 1. P, Q, R, S and T sit around a table.
(E) 2nd to the right 2. P sits two seats to the left of R, and Q sits two
13. Which of the following combinations is not correct? seats to the right of R.
(A) Amber - cherry Which of the following cannot be the correct seating
arrangement of the five persons in either the
(B) Emma - apple clockwise direction or the counter-clockwise direction?
(C) Amy - dandelion (A) P, Q, R, S, T
(D) Zoe - beetroot (B) P, S, R, T, Q
(E) Caitlin - spinach (C) P, Q, S, R, T

602

Book 1.indb 602 30/04/2019 4:49:37 PM


7.0  Logical Reasoning Practice

(D) P, T, R, S, Q orange, red and pink, not necessarily in that order. The ground
(E) R, S, Q, P, T floor is numbered 1 and the topmost floor is numbered 9.
•  Shane owns a black coloured car and stays on an even
Directions for Questions 19–22:  Read the below
numbered floor. Richard stays on any even numbered
information and answer the questions that follow.
floor below the floor on which Shane stays. The person
“Four dogs F, G, H, J and two cats—K and M—will be who owns the orange coloured car stays on the fourth
assigned to exactly six cages numbered 1 to 6. Cage 1 floor.
faces Cage 4, Cage 2 faces Cage 5 and Cage 3 faces
•  Patrick stays on the second floor and owns the white
Cage 6.
coloured car. The person who owns a pink coloured car
The following conditions apply: The cats cannot face each stays on the third floor. Richard does not own a green
other, else they’ll start fighting. A dog must be put in Cage coloured car. There are two floors between the floors on
1. H must be put in Cage 6. J must be put in a cage whose which the people owning the red and the black coloured
number is 1 more than the number of K’s cage K and H cars stay.
cannot be opposite each other.”
•  Luke owns a grey coloured car. There are three floors
19. Which one of the following must be true? between the floors on which Luke and Jason stay. Andre
(A) F is assigned to an even numbered cage. stays on a floor immediately above Joshua’s floor. There
is one floor between the floors on which Ethan and Jason
(B) F is assigned to Cage 1.
stay.
(C) J is assigned to Cage 2 or Cage 3.
•  Ethan does not own the pink coloured car and does not
(D) J is assigned to Cage 3 or Cage 4. stay on the ground floor. The person who owns the blue
(E) K is assigned to Cage 2 or Cage 4. car stays on the top-most floor.
20. If J is assigned to Cage 3, which one of the following 23. Who stays on floor number 8?
could be true? (A) Emmanuel
(A) F is assigned to Cage 2. (B) Andre
(B) F is assigned to Cage 4. (C) Richard
(C) G is assigned to Cage 1. (D) Ethan
(D) G is assigned to Cage 4. (E) Shane
(E) M is assigned to Cage 5.
24. How many persons are staying between Jason and
21. Which one of the following must be true?
Emmanuel?
(A) A cat is assigned to Cage 2.
(A) Three
(B) A cat is assigned to Cage 5.
(B) Four
(C) K’s cage is in a different row from M’s cage.
(C) Two
(D) Each cat is assigned to an even-numbered cage.
(D) One
(E) Each dog is assigned to a cage that faces a cat’s
(E) None of these
cage.
22. If K’s cage is in the same row as H’s cage, which one 25. Who stays on the floor immediately below Joshua’s
of the following must be true? floor?
(A) F’s cage is in the same row as J’s cage. (A) Ethan
(B) F is assigned to a lower-numbered cage than G. (B) Andre
(C) G is assigned to a lower-numbered cage than M. (C) Patrick
(D) G’s cage faces H’s cage. (D) Richard
(E) M’s cage is in the same row as G’s cage. (E) Emmanuel or Luke
Directions for Questions 23–26:  Read the below 26. Who owns the yellow coloured car?
information and answer the questions that follow on the
(A) Andre
basis of this information.
(B) Ethan
Nine people, Richard, Emmanuel, Luke, Andre, Patrick,
Ethan, Jason, Shane and Joshua, stay on different floors of a (C) Emmanuel
9-storey building. All of them own one car each, and each car (D) Richard
is of a different colour: blue, white, grey, black, green, yellow, (E) Luke

603

Book 1.indb 603 30/04/2019 4:49:38 PM


NMAT by GMAC™ Official Guide 2019

Directions for Questions 27–30:  Read the following (A) 23


information and answer the questions that follow. (B) 26
Seven friends—P, T, M, J, V, R and W—are pursuing B.Com, (C) 35
B.A. and B.Sc courses. Three of them are pursuing B.Com, (D) 37
two are pursuing B.A. and two are pursuing B.Sc. Each
(E) 38
of them has a favourite musical instrument ranging from
banjo, sitar, guitar, flute, violin, saxophone and tabla but Directions for Questions 32–35:  Based on the
not necessarily in the same order. None of those pursuing information given below, answer the question that follows.
B.Com like either sitar or violin. M is pursuing B.A. and he
During the summer vacation, Manish started 4 classes
likes banjo. R is pursuing B.Sc and likes tabla. J is pursuing
between 1 pm and 8 pm on five days (Monday to Friday) of
B.Com and likes guitar. P, who does not like sitar, is pursuing
the week—Manish learns: (Real NMAT Question)
the same discipline as R. T is pursuing the same discipline
as M. V does not like saxophone. I. Swimming—all 5 days of the week at the same
time slot either from 1 pm to 3 pm or from 6 pm
27. Who among the following is pursuing B.Com? to 8 pm.
(A) J, V and W II. Music—either on Tuesday, Wednesday and Friday
(B) V, W and T from 2 pm to 4 pm or on Monday and Thursday
from 4 pm to 7 pm.
(C) J, V and T
III. Cricket—either on Monday and Thursday from
(D) J, P and R
3 pm to 5 pm or on Tuesday, Wednesday and
(E) None of the above Friday from 4.30 pm to 6 pm.
28. What is the favourite musical instrument of M? IV. Yoga—on any day of the week for one 4-hour
session or for two 2-hour sessions.
(A) Flute
(B) Sitar 32. If Manish goes for the 4-hours session of yoga, on
which day of the week does he attend this class?
(C) Guitar
(A) Friday
(D) Banjo
(B) Monday
(E) Saxophone
(C) Thursday
29. What are the favourite musical instruments of those who (D) Monday or Thursday
are pursuing B.Sc?
(E) Monday or Friday
(A) Guitar and Violin
(B) Sitar and Tabla 33. What is the maximum number of 2-hour yoga classes
that are available for Manish to attend in a week,
(C) Tabla and violin
including all scenarios?
(D) Flute and sitar
(A) 4
(E) Violin and saxophone
(B) 5
30. Which of the following combinations is correct? (C) 6
(A) J – B.A. - Guitar (D) at least 7
(B) M – B.Com - Banjo (E) at most 8
(C) T – B.A. - Tabla
34. The trainer comes to the yoga class on Tuesday;
(D) T – B.Sc - Sitar
Manish attends at least one two-hour yoga session
(E) W – B.Com - Saxophone on that day. Which of the following classes can
31. Vikas took part in the discus throw event at the Manish attend, assuming that he has to stick to the
Olympics. After the first 3 throws, Vikas analysed slots and not mix slots and days?
the leader board. He realised that the athlete from A. Cricket on Tuesday, Wednesday, Friday from
Cyprus was ranked 5 places below him and the 4:30 pm to 6 pm.
athlete from Tunisia was ranked 3 places above him. B. Music on Tuesday, Wednesday, Friday from 2 pm
The athlete from Cyprus is actually 18th from the top to 4 pm.
and the Tunisian is 17th from the bottom. How many
(A) A only
athletes took part in the competition if all the names
are on the leader board? (Real NMAT Question) (B) B only

604

Book 1.indb 604 30/04/2019 4:49:38 PM


7.0  Logical Reasoning Practice

(C) A or B (D) April


(D) A and B (E) May
(E) Cannot be determined
38. With whom will Mr Gates celebrate New Year’s Eve night?
35. If Manish attends the swimming class between 1 (A) Ambani
pm and 3 pm on all the days, when can he attend a (B) Parekh
2-hour yoga class?
(C) Aiyyar
(A) Tuesday and Wednesday
(D) Mittal
(B) Wednesday and Friday
(E) Murti
(C) Tuesday and Friday
(D) None of the above 39. Who has been scheduled for the rainy season?
(E) A, B or C (A) Parekh
(B) Ambani
Directions for Questions 36–39:  From the information
(C) Murti
provided below, answer the question that follows.
(Real NMAT Question) (D) Mittal
(E) Aiyyar
Mr Gates could schedule his business meetings with
five clients Mittal, Murti, Aiyyar, Ambani and Parekh Directions for Questions 40–43:  Read the statements
only on the 31st of different months of the same given below carefully and answer the following question by
calendar year, not necessarily in this order. choosing the correct alternative among the given five options.
1. Every season except winter has one meeting. Ten friends are sitting on two parallel lines in such a way that
2. Murti is slotted for the winter season but is on A, B, C, D, E are facing front while P, Q, R, S, T are sitting in
leave in December. the other line and facing the the formers. S is sitting second
3. Mittal meets in the first month of the only two to the right of P. T is not the neighbour of P or S. There is
consecutive months when a meeting can take one person between S and T. D is facing R. C and E are
place. neighbours. B is at the right end of the row. A is facing P. D
is not adjacent to B. E is not facing S.
4. Ambani is the last to meet Gates.
5. Aiyyar meets just before the start of the rains. 40. Which of the following are sitting on the ends of the
row facing north?
Consider the Indian seasons as: (A) A, B
Summer season—April to June (B) R, S
Rainy season—July to September (C) P, T
Winter season—October to January (D) P, Q
Spring season—February and March. (E) R, S
36. Who is scheduled for a meeting in August? 41. Who among the following in facing E?
(A) Ambani (A) P
(B) Aiyyar (B) R
(C) Mittal (C) S
(D) Murti (D) T
(E) No one (E) Q

37. For which month is Aiyyar scheduled? 42. Which of the following is true regarding C?
(A) December (A) C is facing Q
(B) February (B) Q is second to the right of the person facing C
(C) October (C) C is sitting at the centre of the line

605

Book 1.indb 605 30/04/2019 4:49:38 PM


NMAT by GMAC™ Official Guide 2019

(D) C is at the right end of the line • There is one floor between the floors on which
(E) None of these Chandu and Disha stay.
• The sum of the floor numbers on which Aman and
43. Which of the following is third to the left of T?
Bimal stay is 8.
(A) P
• Fulla stays just above the floor on which Disha stays.
(B) R
47. What is the sum of the floor numbers on which Esha
(C) S
and Chandu stay?
(D) A
(A) 4
(E) Q
(B) 5
Directions for Questions 44–46:   Read the statements (C) 7
given below carefully and answer the following question by
(D) 6
choosing the correct alternative among the given five options.
(E) None of these
Eight friends P, Q, R, S, T, U, V, and W are sitting around
a circular table. Four are not facing the centre. P is sitting 48. The number of persons between Geeta and Disha is:
fourth to the left of T. T is facing outside. S is immediate (A) One
neighbour of R. Q is second to the right of S. W is sitting
(B) Two
second to the right of U and immediate left of P. U is facing
outside and not an immediate neighbour of P. Both the (C) Three
neighbours of U face outside. S faces the centre. (D) Four
44. What is the position of Q with respect to W? (E) None of these
(A) Second to the right 49. The square of the floor number of Esha is:
(B) Third to the left (A) 1
(C) Second to the left (B) 4
(D) Third to the right (C) 9
(E) None of these (D) 16
45. How many people are there between P and T? (E) 25
(A) One 50. Aman stays just above which of the following
(B) Two persons?
(C) Three (A) Esha
(D) Four (B) Geeta
(E) Five (C) Chandu
(D) Bimal
46. Who are facing outside?
(E) None of these.
(A) P, Q, T, U
(B) Q, R, W, S Directions for Questions 51–54:  Seven students A, B,
C, D, E, F and G take an exam. They all get marks between
(C) Q, T, W, V
50% and 95%. A gets marks more than G but less than E. C
(D) P, T, V, Q gets 10% marks less than B but 10% marks more than F. G
(E) P, T, U, V does not get minimum marks. C gets marks at fourth rank
which was 70%. The highest marks is scored by E which is
Directions for Questions 47–50:  Read the following
10% more than B. Score of A is between 70% and 80%. F
information and answer the questions that follow.
scored the second lowest marks.
Seven friends Aman, Bimal, Chandu, Disha, Esha, Fulla and
Geeta, stay on different floors of a 7-storey building. The ground 51. Who scored lowest marks?
floor is numbered 1 and the topmost floor is numbered 7. (A) B
The following information is also considered: (B) D
• Bimal stays on an odd numbered floor but just (C) F
below the floor on which Disha stays.
(D) G
• Geeta stays on an even number floor but below
(E) None of these
the floor on which Bimal stays.

606

Book 1.indb 606 30/04/2019 4:49:38 PM


7.0  Logical Reasoning Practice

52. Which one of the following options is correct? 57. The sum of heights of tree F and tree G, in feet, is:
(A) The marks of B is between 70% and 75%. (A) 4
(B) The marks of C is between the score of A and E. (B) 6
(C) The marks of D is between 50% and 60% (C) 48
(D) The marks of A is between 80% and 90% (D) Cannot be determined
(E) None of these
(E) None of these
53. The marks scored by G lies between?
58. The maximum possible height of tree A is greater
(A) 50% and 70% than the least possible height of tree F by how many
(B) 60% and 70% feet?
(C) 50% and 60% (A) 5
(D) 70% and 90% (B) 6
(E) None of these. (C) 7
54. The second highest marks is scored by whom? (D) 8
(A) A (E) None of these
(B) B
Directions for Questions 59–62:  Based on the
(C) G information given below, answer the questions that follow.
(D) F (Real NMAT Question)
(E) None of these. Five people, Kuldeep, Gaurav, Susmita, Pratima, and
Directions for Questions 55–58:  Answer the questions that Devanshu are working in three branches of a company
follow based on the information given below. (Real NMAT based at Bangalore, Chennai, and Hyderabad. Two people
Question) work in Bangalore and two people work in Chennai. Of
these four employees, one works in Accounts and another
There are seven trees named A, B, C, D, E, F and G in a
works in Personnel. The remaining are in Marketing. There
garden. It is known that these trees have their heights in an
are 2 Assistant Managers, one Manager, one Director and
increasing order, which may not be the same as the order
one Supervisor.
of their names. It is also known that their heights in feet are
seven consecutive integral values, between 1 and 10 (both 1. Gaurav is the Director in the Marketing division at
inclusive). Tree A is 3 feet taller than tree D. Tree B stands in Chennai.
the middle of the row of seven. The difference in the heights 2. Pratima is the Manager at neither the Bangalore
of tree F and tree B, tree F being shorter, is same as the nor the Chennai branch. She is in the Accounts
difference between the heights of tree C and tree D, tree C department.
being taller. Tree F is shorter than tree G. 3. The person in the Personnel department is an
55. The difference in the heights of tree E and tree B is Assistant Manager in Bangalore.
the same as the difference between the heights of 4. Susmita is at the Bangalore branch working as
tree D and which tree? Supervisor and Devanshu is at the Chennai
branch.
(A) A
(B) B 59. Who is in the Personnel department?
(C) C (A) Devanshu
(D) E (B) Kuldeep
(E) None of these (C) Pratima
(D) Susmita
56. The difference in the heights of tree G and tree D, in
inches, is: (E) Gaurav
Note: 12 inches = 1 foot 60. In what capacity is Devanshu working?
(A) 12 (A) Assistant Manager in Marketing
(B) 24 (B) Assistant Manager in Personnel
(C) 48 (C) Supervisor in Marketing
(D) Cannot be determined (D) Director in Marketing
(E) None of these (E) Manager in Accounts

607

Book 1.indb 607 30/04/2019 4:49:38 PM


NMAT by GMAC™ Official Guide 2019

61. Which of the following is correct about Kuldeep? Matrix -II


Matrix -I
(A) He is the Assistant Manager in Marketing.
0 1 2 3 4 5 6 7 8 9
(B) He is the Assistant Manager in Personnel.
0 L F A T N 5 A U S E O
(C) He is the Supervisor in Marketing.
1 F A T L N 6 U E A O S
(D) He is the Director in Marketing.
2 A T N F L 7 E A U S O
(E) He is the Manager in Accounts.
3 T N F L A 8 O S E A U
62. Which one is the correct combination?
4 A T L F N 9 S E O U A
(A) Gaurav–Chennai–Accounts
(B) Pratima–Chennai–Marketing
64. FAST
(C) Pratima–Chennai–Marketing
(A) 01, 23, 20, 41
(D) Pratima–Chennai–Marketing
(B) 43, 40, 78, 98
(E) Kuldeep–Assistant Manager–Personnel
(C) 23, 20, 69, 42
Directions for Question 63: A word is represented (D) 01, 34, 69, 41
by only one set of numbers as given in any one of the
(E) None of these
alternatives. The sets of numbers given in the alternatives
are represented by two classes of alphabets as in two 65. LEAN
matrices given below. The columns and rows of Matrix (A) 33, 42, 59, 97
I are numbered from 0 to 4 and that of matrix II are
numbered from 5 to 9. A letter from these matrices can (B) 13, 66, 20, 31
be represented first by its row and next by its column, (C) 42, 97, 20, 34
e.g., ‘N’ can be represented by 10, 40 etc. and ‘S’ can be (D) 00, 85, 88, 99
represented by 14, 44 etc. Similarly, you have to identify (E) None of these
the set for the word NOISE.
Directions for Questions 66–69:  Based on the
Matrix -I Matrix -II information below, answer the question that follows.
Four people Ram, Shyam, Amit and Sumit decide to
0 1 2 3 4 5 6 7 8 9
go trekking in the Himalayas, Vindhyachal, Deccan and
0 R E O N G 5 G V E A C Aravallis, not necessarily in that order. On the first night,
1 N P V E S 6 R O N S S they decide to retire by four different lakes Nakuchiatal,
Mansarovar, Chilka and Pushkar, not necessarily in
2 M T I O N 7 M N E S I
that order. These lakes are in four different states,
3 E A I C O 8 O T I T A Uttaranchal, Madhya Pradesh, Rajasthan and Andhra
4 N T A R S 9 N S N E P Pradesh, not necessarily in that order. Further information
regarding their whereabouts is as follows: (Real NMAT
(A) 76, 85, 79, 68, 78 Question)
(B) 40, 66, 87, 79, 96 I. Shyam is staying on Chilka.
(C) 10, 24, 30, 44, 01 II. Mansarovar is on Aravallis which is in Madhya
(D) 40, 85, 79, 96, 77 Pradesh.
III. The person at Pushkar has gone trekking in Andhra
(E) None of these
Pradesh.
Directions for Questions 64–65:  In each of the IV. Sumit is at Vindhyachal.
following questions a word is represented by a set of V. Ram is trekking in Rajasthan.
numbers selected from Matrix I and Matrix II. Each number
is representing on alphabet in the matrices choose the 66. Where is Shyam trekking?
set of numbers from the alternatives the represent the (A) At the Himalayas.
each alphabet of the word. The column and rows of Matrix
(B) At Vindhyachal.
I are numbered from 0 to 4 and those of Matrix II from 5
to 9. In the matrices for N can be coded as 04, 14, 22 (C) In Madhya Pradesh.
etc. Similarly, you have to identify the correct set for the (D) In Uttaranchal.
words given in each question. (E) At Pushkar.

608

Book 1.indb 608 30/04/2019 4:49:39 PM


7.0  Logical Reasoning Practice

67. Using the information provided in the problem, which 71. What did the plastic box store?
of the following may be deduced? (A) Odd stuff
i. Ram has not gone to the Aravallis.
(B) Needles
ii. Amit has gone to Andhra Pradesh.
(C) Buttons
iii. Sumit has not gone to the Himalayas.
(D) Earrings or buttons
(A) i only
(E) Cannot be determined
(B) ii only
72. What was the size of the rust coloured box?
(C) iii only
(A) Tiny
(D) i and iii only
(B) Large
(E) ii and iii only
(C) Small
68. Which of the following is true about Pushkar? (D) Medium
(A) Amit is trekking there. (E) Cannot be determined
(B) Pushkar is in Uttaranchal. 73. Which of the following combinations is correct?
(C) Ram is trekking there. (A) White, plastic, needles, tiny
(D) Shyam is trekking there. (B) Red, brass, odd stuff, large
(E) Sumit is trekking there. (C) Rust, wood, odd stuff, large
69. Which of the following cannot be inferred on the basis (D) White, silver, earrings, medium
of the information given? (E) Black, wood, buttons, medium
(A) Which state is Chilka lake in? 74. Some friends are ranked according to their ages in
(B) Who has gone to the Himalayas? descending order. Raman is 18th from the top while
Aakash is 14th from the bottom. Anubhav is exactly
(C) Who has gone to Andhra Pradesh?
at the centre of the column and is 5th from Raman.
(D) Who has gone to Madhya Pradesh? Where is Aakash as compared to Anubhav? (Real
(E) Where is Mansarovar situated? NMAT Question)
Directions for Questions 70–73:  Based on the (A) 5th place above Anubhav.
information below, answer the questions given alongside. (B) 8th place above Anubhav.
(Real NMAT Question) (C) 9th place below Anubhav.
There were four boxes - large, medium, small and tiny of (D) 10th place below Anubhav.
different colours, white, black, red and rust in no particular order.
(E) 12th place below Anubhav.
They were each made of either plastic, silver, wood, or brass
and they stored either needles, odd stuff, earrings or buttons. 75. In the given question, there are five choices (A-E).
Some more information that was known about them is as Four of them are alike and one is different. Mark the
follows. one that is different.
1. The white box was not made of silver or wood - it (A) Typhoon
was not medium or large. (B) Volcano
2. The brass box had some odd stuff and it was not (C) Storm
white or red. (D) Hurricane
3. The wooden box was small and the tiny box (E) Cyclone
stored needles in it.
76. In the given question, there are five choices (A-E).
4. The medium, silver box that was not rust or red Four of them are alike and one is different. Mark the
coloured did not have buttons. one that is different.
70. What was the colour of the box that stored earrings? (A) 34
(A) Red (B) 102
(B) Rust (C) 68
(C) Black (D) 51
(D) White (E) 47
(E) Cannot be determined

609

Book 1.indb 609 30/04/2019 4:49:39 PM


NMAT by GMAC™ Official Guide 2019

77. In the given question, there are five choices (A-E). 83. (A) Numismatics
Four of them are alike in certain way and one is (B) Ecology
different. Mark the one that is different.
(C) Geology
(A) Kiwi
(D) Osteopathy
(B) Eagle
(E) Obstetrics
(C) Dodo
84. (A) Mortal
(D) Vulture
(B) Lethal
(E) Albatross
(C) Deadly
Directions for Questions 78–79:  In the given question, (D) Eternal
there are five choices (A – E). Four of them are alike and one
(E) Murderous
is different.
Directions for Questions 85–86:  Chose the odd words in
78. Mark the one that is different. (Real NMAT Question)
the following options:
(A) EKCI
85. (A) Liver
(B) GMEK
(B) Appendix
(C) KOIM
(C) Intestine
(D) NQLO
(D) Brain
(E) RUTW
(E) Spleen
79. Mark the one that is different. (Real NMAT Question)
86. (A) Stockings
(A) Cat
(B) Bag
(B) Horse
(C) Waistcoat
(C) Camel
(D) Dress
(D) Bullock
(E) Hat
(E) Donkey
87. (A) Dachshund
80. Directions: In the given question, there are five choices (B) Boxer
(A–E). Four of them are alike and one is different. Mark
(C) Vizsla
the one that is different. (Real NMAT Question)
(D) Shih Tzu
(A) CEOQ
(B) FHRT (E) Maine Coon
(C) HJTV 88. The following questions have a few statements
(D) LNXZ followed by conclusion. Read the statements and
(E) OQZB logically select the conclusions that follow the
statements.
Directions for Questions 81–84:  Chose the odd words in Statements:
the following options:
Some cars are trucks.
81. (A) Horrendous All trucks are airplanes.
(B) Wonderful Some airplanes are trains.
(C) Atrocious No car is a train.
(D) Detest
(E) Dreadful Conclusions:
(A) All cars are airplane
82. (A) Prodigy: Ordinary
(B) All trucks are trains
(B) Smile: Laugh (C) All trains being airplanes is a possibility.
(C) Great: Large (D) No truck is a train
(D) Draconian: Brutal (E) None of these.
(E) Laconic: Concise

610

Book 1.indb 610 30/04/2019 4:49:39 PM


7.0  Logical Reasoning Practice

Directions for Questions 89–91:  In each of the questions Directions for Questions 92–96:  In each of the
below, three statements are followed by three or four questions/set of questions below, statements are given
conclusions. Decide which of the given conclusion(s) logically followed by the conclusions numbered accordingly. You have
follow(s) from the given statements, and hence is/are true. to assume all the statements to be true even if they seem to
be at variance from commonly known facts and then decide
89. Statements:
which of the given conclusions logically follows from the
(1) Some parks are ground. information given in the statements. Give answer.
(2) No ground is sky. (A) If only conclusion I follows.
(3) All sky are walls (B) If only conclusion II follows.
Conclusion I Some walls are parks. (C) If either conclusion I or II follows.
Conclusion II Some sky are parks. (D) If neither conclusion I nor II follows.
(E) If both conclusions I and II follow.
Conclusion III No wall is a park.
92. Statements:
(A) Either conclusion I or III follows
(1) All ships are goats.
(B) Either conclusion II or III and conclusion I follows
(2) All goats are cows.
(C) Only conclusion II follows
(3) No goat is a horse.
(D) Only conclusion III follows
(E) None follows Conclusion I Some horses are cows.

90. Statements: Conclusion II No horse is a cow.


(1) Some gifts are box. 93. Statements:
(2) All cream are grapes. (1) Some stones are rocks.
(3) Some cream is box. (2) Some rocks are rings.
Conclusion I Some cream are gift. Conclusion I Some stones are rings.
Conclusion II Some grapes are box. Conclusion II Some rocks are stones.
Conclusion III No cream is gift. 94. Statements:
(A) Either conclusion I or III and II follows (1) Some grapes are strawberries.
(B) Only conclusion II follows (2) All strawberries are oranges.
(C) Conclusion III follows (3) Some oranges are apples.
(D) Either conclusion I or III follows Conclusion I All strawberries are apples.
(E) None follows Conclusion II Some grapes are not
91. Statements: strawberries.
(1) Some print are wrong. 95. Statements:
(2) All wrong are deform. (1) All sheeps are goats.
(3) No deform are right. (2) Some goats are dogs.
Conclusion I Some print are deform. (3) All cats are dogs.

Conclusion II Some print are right. Conclusion I No cat is a sheep.

Conclusion III Some deform are wrong. Conclusion II Some cats are goats.
Conclusion IV All deform are wrong. 96. Statements:
(A) Only conclusion IV follow (1) No desk is a room.
(B) Conclusions I and II follow (2) Some desks are halls.
(C) Conclusions I and III follow Conclusion I Some halls are definitely not
desks.
(D) None follows
Conclusion II No room is a hall
(E) Conclusion III follows

611

Book 1.indb 611 30/04/2019 4:49:39 PM


NMAT by GMAC™ Official Guide 2019

97. Directions: Given below are two statements followed (A) Uncle
by four conclusions (1), (2), (3) and (4). Choose the (B) Cousin
conclusion(s) which logically follow(s) from the given
(C) Father
statements. (Real NMAT Question)
(D) Grandfather
Statements:
(E) Nephew
Some doctors are musicians.
All musicians are poets. 102. Pointing towards a person, Sita told her son that
the person was the only son of his maternal
Conclusions: grandmother. Who is that person to Sita’s husband?
(1) Some doctors are poets. (Real NMAT Question)
(2) No musician is doctor. (A) Father-in-law
(3) No poet is doctor. (B) Brother-in-law
(4) All musicians are doctors. (C) Brother
(A) Only (4) (D) Father
(E) Uncle
(B) Only (3)
(C) Only (2) 103. The family residing in the flat adjacent to ours
(D) Only (1) comprises of six people. The members are Arjun,
Avinash, Sati, Savitri, Rahul and Ratul. Arjun is
(E) Both (2) and (3)
a doctor and the father of Rahul. Ratul is the
98. If P is the brother of Q, B is the brother of Q and grandfather of Avinash and an engineer. Sati is the
P is the brother of E, then which of the following grandmother of Rahul and a homemaker. There are
statements is definitely true? two couples and the professions of the members
(A) Q is the brother of B are as follows: doctor, engineer, homemaker,
consultant and two students. (Real NMAT
(B) Q is the brother of E
Question)
(C) E is the brother of B
Who is Ratul to Rahul?
(D) B is the brother of E
(A) Son
(E) None of these
(B) Father
99. Rajeev is the brother of Rishi. Karuna is the sister of Ajay. (C) Brother
Rishi is the son of Karuna. How is Rajeev related to Ajay?
(D) Grandson
(A) Son (E) Grandfather
(B) Brother
(C) Nephew 104. Sujata was showing a photograph to his friend and
pointed to a boy and told the following statement:
(D) Father
“His name is Mohan and his maternal uncle is brother
(E) Uncle of my mother’s husband”. How is Mohan related to
Sujata?
100. Pointing to Akshay, Rahul said, ‘I am the only son of one
of the sons of his father.’ How is Akshay related to Rahul? (A) Brother
(A) Nephew (B) Uncle
(C) Cousin
(B) Uncle
(D) Son
(C) Father or Uncle
(E) None of these
(D) Father
(E) Brother or Nephew 105. Based on the information below, answer the question
that follows. (Real NMAT Question)
101. Pointing to a girl Joginder said, “She is the daughter of I. ‘A × B’ means “A is mother of B”.
my father’s sister.” How is Joginder related to the girl? II. ‘A # B’ means “A is sister of B”.

612

Book 1.indb 612 30/04/2019 4:49:39 PM


7.0  Logical Reasoning Practice

III. ‘A + B’ means “A is brother of B”. 110. Kala tells Shekhar that she holds the same relation
IV. ‘A – B’ means “A is father of B”. with Shekhar as the relationship that Shekhar’s
paternal grandfather’s only daughter-in-law holds with
Which of the following means “A is the paternal uncle Shekhar’s father. How is Kala related to Shekhar?
of B”? (Real NMAT Question)
(A)
A+C–B (A) Aunt
(B) A + C × B (B) Wife
(C)
A#C–B (C) Niece
(D) A + C + B (D) Sister
(E) A − C – B (E) Mother
106. A is the daughter of Z. Q’s sister has a son D and 111. In a certain code, DONKEY is written as XDJMNC.
daughter E. T is the maternal aunt of E and the How is LION written in that code?
mother of I. I is the brother of A. How is Q related to (A) MNHK
Z? (Real NMAT Question)
(B) BHUP
(A) Mother (C) VGSW
(B) Cousin (D) LDRE
(C) Daughter (E) IYUO
(D) Grandmother
112. In a certain code, TRIFLE is written as FERTIL. How is
(E) Sister-in-law
JASPER written in that code?
107. Is B the father of D? (Real NMAT Question) (A) ASEJRP
To answer this question, which of the following
(B) APJRES
statements is sufficient?
(C) PRAJSE
I. D’s mother is A.
(D) SEJPAR
II. B is the father of C who is the son of A.
(E) PERSAJ
(A) I alone is sufficient.
113. If HIVE is coded as 6137 and BELT is coded as
(B) II alone is sufficient.
9724, how is LIVE coded?
(C) Either I or II alone is sufficient.
(A) 7213
(D) Both I and II together are sufficient. (B) 6197
(E) Both I and II together are not sufficient. (C) 7321
108. Gayatri is married to Ganesh who is the brother of (D) 3124
Geeta who is married to Gopal. If Ganesh happens (E) 2137
to be the son-in-law of Gopal’s parents, then how is
Gayatri related to Gopal? (Real NMAT Question) 114. In a certain code language, ‘456’ means ‘flavoured
orange juice’, ‘678’ means ‘mixed fruit juice’, and ‘389’
(A) Aunt
means ‘very tasty fruit’. Which of the following is the
(B) Sister code for ‘mixed’? (Real NMAT Question)
(C) Niece (A) 4
(D) Mother (B) 5
(E) Sister-in-law (C) 6
109. My son said to Titli, “My mother’s mother is also your (D) 7
mother’s mother.” How is my son related to Titli’s (E) 8
brother? (Real NMAT Question) 115. If ‘HANDBAG’ is coded as ‘37’ and ‘PURSE’ is coded
(A) Maternal uncle as ‘489’, then how will the word ‘WALLET’ be coded?
(B) Paternal uncle (Real NMAT Question)
(C) Niece (A) 43
(D) Nephew (B) 115
(E) Cousin (C) 258

613

Book 1.indb 613 30/04/2019 4:49:40 PM


NMAT by GMAC™ Official Guide 2019

(D) 342 (C) UBDMF


(E) 448 (D) UBCNF
116. In a certain code, ‘YOU’ is written as ‘DUB’. How is ‘OUR’ (E) UBCLF
written in the same code? (Real NMAT Question)
122. In a certain code language, if the word SYLLABUS
(A) TBZ is coded as TXMKBAVR, then which of the following
(B) SAX is a possible code for the word OPERATION? (Real
(C) TAZ NMAT Question)
(D) TAY (A) POQFBSJON
(E) TBY (B) POEQBSJMO
(C) POEQBSJNP
117. In a certain code, GARAGE is written as HZSZHD.
How is TRUCK written in that code? (D) POFQBSJNO
(A) BVLQU (E) PPFQBSJMO
(B) UQVBL 123. In a certain language, CRICKET is coded as
(C) SQTBJ EPKAMCV. How is HOCKEY written in the same
language? (Real NMAT Question)
(D) USVDL
(A) IPDLLFZ
(E) VSUDL
(B) JQEMGA
118. In a certain code, LEMUR is written as MGPYW. How
(C) JMEIGW
is ZEBRA written in that code?
(D) FQAMCA
(A) VFRLY
(E) FNEJGZ
(B) VGEFA
(C) AGEVF 124. If in a certain code, P is written as C, O as L, A as
W, R as K, E as X, L as Q, B as N, and T as H, then
(D) BDESC
how will PORTABLE be written using this code? (Real
(E) YRVFL NMAT Question)
119. In a certain code, VANILLA is written as XXPFNIC. (A) CLKHWNQX
How is CARPENTER written in that code? (B) CKLHOWNX
(A) NBVFGHYTR (C) CLKNHOQX
(B) RTYUHGFDI (D) CLKOHNCX
(C) OPIUJKLHB (E) KCLHKNQX
(D) MGTHYUIKT
125. In a certain language, GAMBLE is coded as HCPFQK.
(E) EXTMGKVBT How is PUNTER coded in the same language? (Real
NMAT Question)
120. What is the value of the letter M in the given problem?
(Real NMAT Question) (A) QQXWJQ
M 2 N (B) QWQXJX
x N (C) QWQXJV
2 5 5 N (D) QVQXIU
(A) 1 (E) QWQXIV
(B) 2 126. In a certain number system, there are only 5
(C) 4 numerals - 0,1, 2, 3 and 4, which are represented
by @, #, $, £ and ¥, respectively. So, numbers
(D) 5
greater than 4 are represented with the help of these
(E) 6 numerals only 5 = #@, 6 = ##, 7 = #$, 8 = #£ and
so on. (Real NMAT Question)
121. In a certain code language, if CHAIR is coded as
DIBJS, how is the word TABLE coded in that same Which of the following will represent 18?
language? (Real NMAT Question)
(A) ¥#
(A) UBCMF
(B) $$
(B) UBCMG

614

Book 1.indb 614 30/04/2019 4:49:40 PM


7.0  Logical Reasoning Practice

(C) ## (A) TGNCVKXF


(D) ££ (B) GNCKXFV
(E) ¥¥ (C) SFMBUJWG
(D) RVITALEE
127. If ‘WEDDING’ is coded as 32 16 30, ‘BELONGS’ is
coded as 19 225 40, then how is ‘STRANGE’ coded (E) RELEVITE
in the same code language? (Real NMAT Question) Directions for Questions 132–135:  Solve the questions
(A)
47 16 26 that follow on the basis of the information given below.
(B)
39 19 24 There are 3 binary digits X, Y, Z. They can only have values
(C)
57 1 26 of 0 and 1. The values have to be changed from an old
number to form a new number according to the following
(D)
57 1 24
conditions: (Real NMAT Question)
(E)
47 2 26
•  If X and Y only are 1 in the old number, then
128. If ‘Some More Is Good’ is coded as ‘ACEF’, ‘More change Z to 1 to form the new number.
Sugar Not Good’ is coded as ‘CEBD’, ‘Good More Is •  If all digits X, Y and Z are 1 in the old number,
Bad’ is coded as ‘FCEI’, ‘Bad Is More Sure’ is coded change Z to 0 to form the new number.
as ‘IFCZ’, then how is ‘More Bad Is Not Good Sugar’ •  If only X is 1 in the old number, then change Y to
coded as? (Real NMAT Question) 1 to form the new number.
(A) ZFBIDC •  F or any other old number, change all 1s to 0s
(B) BFDICE and all 0s to 1s to form the new number.
(C) ACFEIZ 132. If in the old number X is 1, Y is 1 and Z is 0, then
(D) BDIZEF what could be the new number?
(E) BIZAEF (A) X is 1, Y is 0, Z is 1
(B) X is 1, Y is 0, Z is 0
129. In a certain code, ‘Given timetable below represent’ (C) X is 0, Y is 1, Z is 0
is written as ‘ku na pa cu’, ‘Presentation represent
(D) X is 0, Y is 0, Z is 1
according timetable’ is written as ‘pa su lu cu’ while
‘Presentation below cutoff’ is written as ‘su na ja’. (E) X is 1, Y is 1, Z is 1
How will ‘according’ be coded? 133. If only Y is 1 in the old number, what must be the new
(A) su number?
(B) cu (A) X is 1, Y is 1, Z is 1
(C) su or cu (B) X is 1, Y is 1, Z is 0
(D) lu (C) X is 0, Y is 0, Z is 1
(D) X is 1, Y is 0, Z is 1
(E) None of these
(E) 1 is X, Y is 1, 1 is Z
130. If in a certain code “CETKINGMUMBAI” is coded as
128, then what is the code of “IIMAHMEDABAD” in 134. If all three digits are 1 in the new number, which of
that language? the following could have been the old number?
(A) X is 1, Y is 1, Z is 1
(A) 64
(B) X is 1, Y is 1, Z is 0
(B) 65
(C) X is 1, Y is 0, Z is 1
(C) 70
(D) X is 1, Y is 0, Z is 0
(D) 71
(E) X is 0, Y is 1, Z is 0
(E) 72
135. If X is 0 in the new number, which of the following
131. Select the correct code for RELATIVE using any of could have been the old number?
the following 4 logics that have been used for coding (A) X is 1, Y is 1, Z is 1
RELATE. (Real NMAT Question) (B) X is 1, Y is 1, Z is 0
A. TGNCVG (C) X is 1, Y is 0, Z is 1
B. RTALEE (D) X is 1, Y is 0, Z is 0
C. SFMBUF (E) X is 0, Y is 1, Z is 0
D. RILETI

615

Book 1.indb 615 30/04/2019 4:49:41 PM


NMAT by GMAC™ Official Guide 2019

136. In the land of Liliputs, ‘amu hua bia’ says ‘Gold is (A) 64
precious’ and ‘darya nadiya panu hua huju amu’ says (B) 81
‘All that glitters is not gold’. What is the code for the
(C) 125
word ‘GOLD’ in that language? (Real NMAT Question)
(D) 249
(A) Amu
(E) 625
(B) Bia
(C) Darya 141. What term should come in the blank in the below
(D) Hua series?
(E) Huju 1AZ, 4DW, ______, 10JQ
137. In a certain code language, three statements are (A) 6TG
coded as follows. (Real NMAT Question) (B) 8VW
‘Engineers like hard work’ is ‘krit prit dam sam’. (C) 7WX
‘Work is like worship’ is ‘pram drit prit krit’. (D) 7GT
‘Richa is doctor’ is ‘sanu pram kram’. (E) 8XX
Which of the following is the code for ‘like’?
142. What number should come in place of the question
(A) krit
mark?
(B) prit
(C) dam 2 3 1 2
(D) sam 3 4 4 5
(E) Cannot be determined 5 6 6 7
138. If ‘grp ptr prt’ stands for ‘mixed fruit juice’, ‘pnr ptr 30 43 51 ?
qrs nsr’ stands for ‘everybody should eat fruits’ and
‘ntr prt nts nrp’ stands for ‘I like pineapple juice’.
(A) 52
What does mixed stand for? (Real NMAT Question)
(B) 56
(A) grp
(B) ptr (C) 62
(C) prt (D) 65
(D) nsr (E) 70
(E) nts 143. What number should come in place of the question
139. What number should come next in the following series? mark?
5, 7, 12, 19, 31, 50,
(A) 63
(B) 76 6 9 21 30
(C) 81
(D) 86 14 ?
(E) 91
(A) 29
140. What number should come in place of the question
mark? (B) 37
(C) 41
(D) 47
8 27
(E) 53
2197 ?

1331 343

616

Book 1.indb 616 30/04/2019 4:49:41 PM


7.0  Logical Reasoning Practice

144. What number should come in place of the question (A) 16


mark? (B) 17
(C) 51
5 25 (D) 61
(E) 62
481 61 148. Which of the options that follow will replace the
question mark (?) in the series given below? (Real
NMAT Question)
365 113
(?) IJH NOM STR XYW
(A) ABZ
265 ?
(B) DEB
(C) EDC
(D) DEC
(A) 121 (E) EDF
(B) 144
(C) 164 149. Which number can substitute the question mark?
(D) 181 (Real NMAT Question)
(E) 221
1240 1066 ?
27 9 29
145. B A B A C C D E F K G H I H J K L Z M N O R P Q R O 31 41 13
STUPVWNXYMZ 13 17 62
In the above alphabetical series, which is the 15th
letter to the left of the 5th letter from the right? (A) 3250
(A) Z (B) 3057
(B) L (C) 2224
(C) M (D) 1728
(D) N (E) 1183
(E) K
150. Each circled number is the sum of the adjacent row,
146. Which of the following is exactly midway between the column or diagonal of the numbers in the square
sixth element from the right and the eighth element array. (Real NMAT Question)
from the left of the arrangement? (Real NMAT
7 15
Question) 4 3 X
5DEF6*I4BMA7$J#HLO@PTU2RQ 7 5
(A) # 2 5 2
(B) J 9
6 8 10
(C) $
(D) H What is value of x in the above diagram?
(E) Q (A) 7
147. Which number should replace the question mark? (B) 8
(Real NMAT Question) (C) 9
(D) 10
37 24
(E) 11
31
23 29 Directions for Questions 151–154:  Find the missing term
9 91
in each of the following series.
14 48 151. 101, 109, 136, 200, 325?
? (A) 395
15 31
(B) 409

617

Book 1.indb 617 30/04/2019 4:49:42 PM


NMAT by GMAC™ Official Guide 2019

(C) 475 157. In the figure below, what is the value of ‘X + Y’? (Real
(D) 541 NMAT Question)
(E) 561
12 8 23 3 4
152. 15W12, 16X13, 20B17, 29K26,?
9 6 7 18 10
(A) 48B43
7 11 22 3 X
(B) 45B42
14 15 8 12 1
(C) 45A42
12 13 9 Y 7
(D) 46A40
(E) None of these 8 7 12 14 9
153. 6, 6, 6, 8, 18, 11, 90, 15, ?, 20 (A) 12
(A) 630 (B) 14
(B) 720 (C) 16
(C) 750 (D) 18
(D) 840 (E) 28
(E) 729
158. The numbers given follow a particular pattern.
154. CKX, GMU, KOR, OQO, ? Complete the pattern. (Real NMAT Question)
(A) SML
(B) SSM 0
(C) SSK
4 ?
(D) SSL
(E) SLK 48 ? 180
155. Study the matrix and find the value of ‘A’. (Real 294 ? 648 ?
NMAT Question)

2 4 7 11 16 22 (A) 8, 144, 432, 864


(B) 12, 96, 394, 768
2 8 21 11 80 A (C) 15, 105, 345, 1035

(D) 16, 96, 494, 980
(A) 111
(E) 18, 100, 448, 900
(B) 120
(C) 132 159. Consider the grid given below. What is the value of Z?
(Real NMAT Question)
(D) 154
(E) 176 5
156. If each of the letters of the English alphabet is
assigned an odd numerical value, starting with A=1,
B=3, C=5 and so on, what will be the difference 0
in the total values between the words INDIAN and
1 1
GERMAN? The total value of a word is the sum of the
numbers (assigned to the letters) that comprise the 10 2 3 4 Z
word. (Real NMAT Question)
(A) 14 (A) 7
(B) 17 (B) 17
(C) 29 (C) 7 or 17
(D) 96 (D) None of the above
(E) 110 (E) Cannot be determined

618

Book 1.indb 618 30/04/2019 4:49:43 PM


7.0  Logical Reasoning Practice

160. Which number should replace the question mark? (C) OCPIQ
(Real NMAT Question) (D) PCPIR
(E) PERKR
8 5 10 7 16 2
165. C D E F K G H I Z M O S T U P V W N O R P Q H J K B
218 514 ? ABAC
7 6 9 8 11 14 In the above alphabetical series, which is the 10th
letter to the left of the 4th letter from the right?
(A) V
(A) 286
(B) N
(B) 296
(C) W
(C) 304
(D) N
(D) 324
(E) P
(E) 356
166. Vishal started to walk after sunrise. After going some
161. Which of the below options would come next in this distance, he met Sanjay who was coming from the
series of numbers? opposite direction. Sanjay’s shadow was falling to the
0, 3, 8, 15, 24, __ right of Vishal. In which direction was Vishal walking?
(A) 29 (A) East
(B) 33 (B) West
(C) 35 (C) South
(D) 48 (D) North
(E) 63 (E) South-west
162. The first three terms of a series are 3.8.39, 5.14.89 167. A man walks 3 km South and then turns to the right.
and 9.26.269. The fourth term of the series will be: After walking 8 km he turns to the left and walks 4
(Real NMAT Question) km. Then he takes a 180° turn. In which direction
(A) 13.37.917 from the starting place is he now standing?
(B) 13.50.345 (A) West
(C) 15.37.345 (B) South
(D) 17.37.548
(C) North-east
(E) 17.50.917
(D) South-west
163. Study the following arrangement and answer the (E) South-east
question given below.
4AE4KO56UH7G4F3AE8I9JN6O4E3AF6G74EH8KM3 168. Himanshu is at his home. He walks 4 km to the south
N4UT6E4$A7O4U and then turns to his right. He walks 3 km in this
direction. He turns right again and walks 8 km. If he
Which of the following should come in place of the
wishes to reach his home again, in which direction
question mark in the following series based on the
should he be walking and what distance will he have to
above arrangement?
cover?
UG38, N4AG, ?, A4AK (Real NMAT Question)
(A) 5 km in South-east
(A) EKNT
(B) 4 km in South-west
(B) EK4T
(C) 5 km in South-west
(C) HM46
(D) 4 km in South-east
(D) HMU6
(E) None of these
(E) HKNT
Directions for Questions 169–170:  Prithvi walks to his
164. In a certain code, APPLE is written as DSSOH. How is office every morning. When he begins, his shadow falls right
MANGO written in that code? ahead of him. He walks for 100 m before turning 45º in
(A) PDQJR the clockwise direction. In this direction he walks for 20 m
(B) QERKS where he reaches a junction. At the junction, he turns 90º

619

Book 1.indb 619 30/04/2019 4:49:43 PM


NMAT by GMAC™ Official Guide 2019

in the anticlockwise direction and walks straight for 30 m and walks 10 m east. How far is he from the starting
and stops. His office is to his left side. Answer the following point?
questions based on the information given above: (A) 50 m
169. In which direction is Prithvi’s office facing? (B) 30 m
(A) North (C) 40 m
(B) Northeast (D) 60 m
(C) South (E) 80 m
(D) Southwest 174. Shiba started walking towards south for 10 km,
(E) Northwest turned to her right and walked 6 km, then she turned
right and walked 10 km. Finally, she turned to her
170. What is the position of Prithvi’s shadow when he left and walked 15 km more. At what distance is she
reaches his office? (Neglect the time taken by him to from the original point?
reach his office).
(A) 21 km
(A) Right in front
(B) 20 km
(B) Exactly to his right
(C) 22 km
(C) 45º clockwise to the direction he is facing
(D) 24 km
(D) 45º anticlockwise to the direction he is facing
(E) None of these
(E) Exactly to his left
175. One morning, Piyush and Ravi were talking while
171. Bhanu begins his journey from point X. All the roads
facing each other. It was observed that Piyush’s
along his path intersect only at right angles. He starts
shadow fell to his left. In which direction was Ravi
in the morning and reaches his destination by evening
facing? (Real NMAT Question)
after taking 14 right turns and 12 left turns. At his
destination he stops and looks straight ahead admiring (A) East
the beautiful sunset happening before him. In what (B) West
direction did he take his first step of the journey? (C) South
(A) South (D) North
(B) Southwest
(E) North-East
(C) North
(D) Northeast 176. When Raman wakes up in the morning and looks
out of his window, he sees the shadow of his
(E) East
home in front of him. If the window is to his right
172. Sunaina is standing facing south. Her friend Sulekha is when he sleeps, then in which direction does
sitting right to Sunaina while Reeta is sitting in the north he keep his head while sleeping? (Real NMAT
of Sulekha. Preeti is standing in the east of Sunaina. If Question)
Sulekha is equally far from Reeta and Preeti, in which (A) North
direction is Reeta with respect to Preeti? (B) West
(A) South-West (C) North-East
(B) North-East (D) South
(C) South-East (E) South-West
(D) North-West 177. How many pairs of letters in the word 'STAIRS' have
(E) None of these as many letters between them (in either direction) in
173. A man who wants to go to hospital is facing East. the word as in the English alphabet?
He walks straight for 20 m and reaches a cross (A) None
roads. From there he walks 30 m in the south–west (B) 1
direction. Then he turns south-east and walks 20 m. (C) 2
He then walks 20 m to east. From another cross (D) 3
roads he turns north-west and then north-east walking
(E) More than 3
20 m and 30 m respectively. Finally, he turns right

620

Book 1.indb 620 30/04/2019 4:49:43 PM


7.0  Logical Reasoning Practice

Direction:  Choose the pair that best represents a similar in each step. The following is an illustration of input and
relationship to the one expressed in the original pair of rearrangement.
words/alphabets. Input: gas net 54 36 17 fan act 65
178. BDFH:: YWUS: EGIJ:? Step 1: 65 gas net 54 36 17 fan act
(A) USQP Step 2: 65 act gas net 54 36 17 fan
(B) VTRQ Step 3: 65 act 54 gas net 36 17 fan
(C) VWYZ Step 4: 65 act 54 fan gas net 36 17
(D) VTSR Step 5: 65 act 54 fan 36 gas net 17
(E) YWVU Step 6: 65 act 54 fan 36 gas net 17
Step 7: 65 act 54 fan 36 gas 17 net
179. Which one of the following option comes in the place
of question mark (?)? 182. What will be step 3 for the following input?
ACDG, CAFE, EYHC, GWJA, ? can axe 32 12 kit 57 bat 89
(A) HULY (A) 89 axe 57 bat 32 can 12 kit
(B) IVLY (B) 89 axe 57 bat can 32 12 kit
(C) IULZ (C) 89 axe can 32 12 kit 57 bat
(D) IULY
(D) 89 axe 57 can 32 12 kit bat
(E) IUMY
(E) 89 can axe 32 12 kit 57 bat
180. If a six digit number is such that A, B, C, D, E are
single digits and 183. How many steps will be needed to complete the
following arrangement?
2ABCDE
76 cot 64 dog 45 hen 54 urn
×3
(A) 1
ABCDE2 (B) 2
(C) 3
Find the value of A + B + C + D + E. (Real NMAT (D) 4
Question) (E) 5
(A) 20 184. If step 3 of an input is as given below, which of the
(B) 23 following must have definitely been the input?
(C) 25 76 gap 56 ink 14 leg 22 oxe
(D) 27 (A) oxe 56 ink 56 leg 22 gap 76
(E) 28 (B) 56 ink 56 leg 22 gap 76 oxe
181. If all the letters in the addition given below are distinct (C) 56 76 ink oxe 56 leg 22 gap
natural number digits, then what is the value of C + S (D) 22 gap 76 oxe 56 ink 56 leg
+ T? (Real NMAT Question) (E) Cannot be determined
CAST 185. What will be last step for the following input?
+CAST tie 56 mat 99 pot 33 jut 22
TESI
(A) jut 99 mat 56 pot 33 tie 22
(A) 14 or 17 (B) 99 jut 56 pot 33 mat 22 tie
(B) 15 or 18 (C) 99 tie 56 pot 33 mat 22 jut
(C) 18 or 21 (D) 99 jut 56 mat 33 pot 22 tie
(D) 21 or 23 (E) 22 jut 33 mat 56 pot 99 tie
(E) 24 or 26
Directions for Questions 186–189:  In an encoder a
Directions for Questions 182–185:  A word and number sequence of numbers is fed and the following outputs are
arrangement machine when given an input line of words obtained at different stages of the encoder. (Real NMAT
and numbers rearranges them following a particular rule Question)

621

Book 1.indb 621 30/04/2019 4:49:44 PM


NMAT by GMAC™ Official Guide 2019

Input: 14 32 99 110 88 9 11 18 Step I: all team member appeared 15 30 power 25


sentences borings 40
Stage 1: 11 14 32 99 110 88 9 18
Step II: all team power member appeared 15 25
Stage 2: 11 99 14 32 110 88 9 18 sentences borings 40 30
Stage 3: 11 99 110 14 32 88 9 18 Step III: all team power member borings appeared 15
Stage 4: 11 99 110 32 14 88 9 18 sentences 40 30 25
Stage 5: 11 99 110 32 9 88 14 18 Step IV: all team power member borings appeared
sentences 40 30 25 15
186. If the output at Stage 1 is “31 19 47 86 39 13 Step IV: is the final output of the above input.
66 69”, then what is the output at Stage 5?
Answer the questions that follow assuming that the new
(A) 31 66 47 19 86 13 39 69
input given below is processed by the machine in the same
(B) 86 31 47 39 69 13 19 66 manner.
(C) 31 86 39 47 66 13 19 69
Input: Procedures 80 doctors 50 would operation 75 45
(D) 31 13 39 47 86 66 47 69 American review
(E) Cannot be determined
190. What is the position of ‘American’ in Step V?
187. If the output at Stage 5 is “141 273 87 41 78 9 (A) Fourth from the right
319 1012”, then what is the input to the encoder? (B) Fourth from the left
(A) 141 273 78 9 87 41 319 1012 (C) Sixth from the right
(B) 319 41 273 87 9 78 141 1012 (D) Fifth from the right
(C) 141 319 78 273 87 41 9 1012 (E) None of these
(D) 41 78 273 9 141 87 319 1012
191. How many steps will be required to complete the
(E) Cannot be determined arrangement?
188. If the output at Stage 4 is “wind flows over the river (A) Three
bed cool calm”, then what is the output at Stage 2? (B) Four
(A) cool calm river bed over flows the wind (C) Five
(B) cool wind flows over the calm river bed (D) Six
(C) river bed cool calm wind flows the over (E) None of these
(D) wind flows river the over bed cool calm 192. In a certain way in Step III, ‘doctors’ is related to ‘80’
(E) Cannot be determined and ‘review’ is related to ‘75’. In the same way ‘would’
is related to which of the following?
189. If the output at Stage 2 is “system restart will happen in
(A) American
another 20 minutes”, then what is the output at Stage 5?
(B) 45
(A) another 20 minutes system restart will happen in
(C) 50
(B) system restart in happen 20 another will minutes
(D) operation
(C) happen 20 minutes in another will system restart
(E) None of these
(D) system restart will another 20 minutes in happen
(E) Cannot be determined 193. How many words/numbers are there in between
‘procedures’ and ‘50’ in step IV?
Directions for Questions 190–193:  A word and number (A) Three
arrangement machine when given an input line of words
(B) Four
and numbers rearranges them based on a certain rule. The
following arrangement illustrates the pattern of steps that (C) Five
the machine follows. (D) Six
Input: team 40 member appeared 15 all 30 power 25 (E) Seven
sentences borings

622

Book 1.indb 622 30/04/2019 4:49:44 PM


7.0  Logical Reasoning Practice

Directions for Questions 194–197:  A computer programme steps as shown below. Study the logic and answer the
converts an input series into an output in six steps using a question that follows. (Real NMAT Question)
certain logic as illustrated below. (Real NMAT Question)
   Input 9 25 49 16 1 81
Input 1234 5678 2345 6789 3456
Step I 3 5 7 4 1 9
Step 1 10 26 14 30 18 Step II 11 17 23 14 5 29
Step 2 JJ ZZ NN DD RR Step III 14 22 30 18 6 38
Step 3 1224 5652 2331 6759 3438 Step IV 30 46 62 38 14 78
Step 4 25 90 23 191 98 Step V 5 7 9 6 3 11
Step 5 Y L W I T
Step 6 1 14 3 17 6
198. If the fourth input element is 64, what is the final
output of step 5?
Use the same logic for the new input below to answer
(A) 2
the question that follows.
(B) 4
New Input 1203 3211 4522 1111 6666 (C) 6
(D) 8
(E) 10
94.
1 What is the value of the fourth element in the third step?
(A) 1078 199. If the output in step 5 of a given input is 15, then
(B) 1107 what was the initial input for this element?
(C) 3204 (A) 100
(D) 4509 (B) 123
(E) 6642 (C) 131
(D) 152
95.
1 Which element of which Step would have the value 29?
(E) 169
(A) 1st element of Step 1
(B) 1st element of Step 2 200. If the output in step 5 is 13, 15, 16, 17, 18, 19, then
the input series is:
(C) 5th element of Step 1
(A) 22, 24, 25, 26, 27, 99
(D) 2nd element of Step 3
(B) 25, 35, 67, 82, 91, 89
(E) 2nd element of Step 4
(C) 100, 103, 130, 143, 153, 200
196. If the new input element ‘1203’ was changed to
(D) 102, 145, 154, 176, 191, 201
‘3210’, what would be the 1st element of Step 5?
(E) 121, 169, 196, 225, 256, 289
(A) B
(B) C 201. Due to a machine malfunction, if Step 2 of an output
(C) D series got written backwards, with the last element
appearing first and the first element appearing last,
(D) E
and it read 44, 38, 32, 26, 14, 8, then what was the
(E) F input series?
197. If the new input series was ‘3021, 1123, 2254, (A) 196, 144, 100, 64, 16, 4
2222, 5555’, what would Step 4 be? (B) 4, 16, 36, 64, 100, 144
(A) 86, 52, 39, 31, 35 (C) 9, 81, 64, 100, 144, 196
(B) 83, 51, 39, 31, 35 (D) 4, 16, 64, 100, 144, 196
(C) 71, 83, 35, 25, 25 (E) 144, 100, 64, 36, 16, 4
(D) 35, 39, 25, 25, 84
Directions for Questions 202–205:  A computer
(E) 52, 39, 35, 25, 84 programme converts an input series into the final output in
Directions for Questions 198–201:  A computer five steps as shown below. Study the logic and then answer
programme converts an input series into an output in five the questions that follow. (Real NMAT Question)

623

Book 1.indb 623 30/04/2019 4:49:44 PM


NMAT by GMAC™ Official Guide 2019

LUMINARY
   Input 1 3 5 7 11 13
(A) 75, 78, 14, 31, 40w, 22, 40, 96
Step 1 b d f h I n
(B) 85, 78, 13, 79, 04, 97, 58, 97
Step 2 6 8 10 12 16 18
(C) 76, 10, 41, 31, 14, 22, 23, 77
Step 3 g i k m q s
(D) 85, 78, 13, 32, 04, 97, 58, 33
Step 4 7 11 15 19 27 31
(E) 85, 78, 13, 79, 04, 97, 58, 33
Step 5 g k o s a e
207. Study the matrices and determine the digits that
202. If the input series is 1, 3, 8, 7, 10, 13, then what is comprise the given word.
Step 5?
(A) a, b, c, d, e, f 0 1 2 3 4 5 6 7 8 9
(B) g, k, u, s, y, e 0 T A E B J 5 J G W A L
(C) g, g, h, u, e, e 1 S B R U E 6 A Z K F E
(D) k, u, g, y, e, s
2 D V T R L 7 J B I T I
(E) g, g, u, h, e, s
3 J P B Y S 8 P U P U N
203. Which of the following input series will give the same
4 N I J U E 9 L A G C T
output as the illustrated example?
(A) 1, 3, 7, 12, 13, 15
JUBILANT
(B) 36, 59, 87, 12, 3, 4
(A) 55, 86, 32, 41, 24, 01, 41, 99
(C) 53, 72, 12, 7, 13, 1
(B) 40, 13, 76, 79, 24, 01, 40, 00
(D) 59, 49, 39, 29, 19, 20
(C) 41, 13, 76, 14, 24, 01, 40, 00
(E) 27, 29, 31, 33, 37, 39
(D) 75, 88, 76, 77, 59, 96, 88, 99
204. If the output at Step 5 is k, o, s, w, e, i, then what is (E) 42, 13, 76, 79, 24, 01, 40, 00
the input?
208. Study the matrices and determine the digits that
(A) 3, 5, 9, 7, 15, 13
comprise the given word.
(B) 3, 5, 7, 9, 13, 15
(C) 8, 10, 12, 14, 18, 20 0 1 2 3 4 5 6 7 8 9
(D) 10, 8, 12, 18, 14, 20 0 R A E B N 5 T G L R W
(E) 11, 15, 19, 23, 31, 35 1 S C R M E 6 A Z K F E
205. If 65 is the last element of the input series, what will 2 D V T R V 7 Y L I G I
be the Step 5 output of this element? 3 T P F Y S 8 P S P T L
(A) t 9 L X G C P
4 W R B U E
(B) r
(C) m RELEGATE
(D) e (A) 12, 02, 76, 68, 78, 65, 22, 44
(E) a (B) 23, 69, 89, 02, 56, 01, 98, 44
206. Study the matrices and determine the digits that (C) 00, 69, 89, 02, 56, 01, 55, 44
comprise the given word. (D) 41, 69, 57, 14, 97, 11, 22, 69
0 1 2 3 4 5 6 7 8 9 (E) 00, 69, 89, 02, 56, 01, 55, 79
0 P L E B N 5 A G L R W
1 U C E M E 6 I Y K F M
2 N V A R Y 7 U L Y U I
3 Y I F Y S 8 L S P T P
4 R M C U E 9 N Y A C L

624

Book 1.indb 624 30/04/2019 4:49:44 PM


7.0  Logical Reasoning Practice

209. Directions: Study the matrices and determine the


digits that comprise the given word. (Real NMAT
Question) (A)

0 1 2 3 4 5 6 7 8 9
0 H P A B M 5 M U A Q O
1 O S H N T 6 R T P N H (B)
2 A U M A P 7 H M H T S
3 N R U S O 8 N U Q P A
4 M B T H T 9 A O B S B
(C)
OATH
(A) 10, 89, 43, 77
(B) 34, 20, 78, 89
(D)
(C) 59, 20, 66, 75
(D) 96, 57, 78, 56
(E) 96, 57, 78, 59
(E) None of these
irections for Questions 210–211: In the following
D
Directions for Questions 212–213:  In the following, the
questions, which one of the alternative figures is
given figures contain one question mark which is a missing
complementary figure of incomplete pattern?
link in the series. Choose the correct one among the given
options A, B, C, D, E to complete the series.
210.
212. ?

(A)
(A)

(B)

(B)

(C)

(D) (C)

(E) None of these.

(D)

211.
(E)

625

Book 1.indb 625 30/04/2019 4:49:46 PM


NMAT by GMAC™ Official Guide 2019

213. ?
(D)

(E) None of these


(A) 215.

(A)

(B)

(B)

(C)
(C)

(D) (D)

(E) None of these


216.
(E)

(A)

Directions for Questions 214–218:  In the given examples,


out of the four figures, three are similar in certain manner.
Choose that one figure which is not like the other three.
(B)
214.

(A)

(C)

(B)

(D)

(C)
(E) None of these

626

Book 1.indb 626 30/04/2019 4:49:48 PM


7.0  Logical Reasoning Practice

217. (A) 4
(B) 5
(C) 6
(A)
(D) 7
(E) 8
220. Find out the two signs to be interchanged in order to
make the following equation correct:
(B)
8 + 7 x 6 / 16 - 8 = 48
(A) + and –
(B) – and /
(C) + and x
(D) + and /
(C)
(E) None of the above
221. If × stands for 'addition', ÷ stands for 'subtraction', +
stands for 'multiplication' and ‘-‘ stands for 'division',
then 40 × 16 ÷ 16 - 8 + 10 = ?
(D) (A) 5
(B) 24
(C) 25
(E) None of these (D) 36
(E) 80
218.
Directions for Questions 222–225:  The question is
based on the following data. (Real NMAT Question)
(A)
Kiran, Lucky, Mohan, Naval, Puneet, Qadar, Rahul, Sanjay,
Ujwal and Vicky are the only 10 people working in the HR
department of a company. There is a proposal to form a
team from within the members of the department, subject to
(B) the following conditions:
I. A team must include exactly one among Puneet,
Rahul and Sanjay.
II. A team must include either Mohan or Qadar, but
not both.
(C)
III. If a team includes Kiran, then it must also include
Lucky, and vice versa.
IV. If a team includes one among Sanjay, Ujwal
and Vicky, then it must also include the other
(D) two.
V. Lucky and Naval cannot be members of the
same team.
(E) None of these VI. Lucky and Ujwal cannot be members of the same
team.
219. Find the missing number in the table given below.
222. Who cannot be a part of a three-member team?
2 3 9
(A) Naval
4 6 1 (B) Lucky
1 4 5 (C) Qadar
5 7 8 (D) Mohan
6 20 ? (E) Puneet

627

Book 1.indb 627 30/04/2019 4:49:49 PM


NMAT by GMAC™ Official Guide 2019

223. What would be the size of the largest possible team? 228. Let 56% of students drink coffee, 48% drink tea and
(A) 3 42% drink aerated beverages. If 62% of students drink
(B) 4 at least 2 drinks, then what is the difference between
the percentage of students who drink all 3 and those
(C) 5
who drink none of the 3? (Real NMAT Question)
(D) 6
(A) 16%
(E) Cannot be determined
(B) 22%
224. What could be the size of a team that includes Kiran? (C) 24%
(A) 2 or 3
(D) 26%
(B) 2 or 4
(E) 28%
(C) 3 or 4
229. Directions: Given alongside are a few facts.
(D) Only 2
Based on these facts, select from among the
(E) Only 4 given statements, the statement that can be best
225. In how many ways can a team be constituted so that concluded. (Real NMAT Question)
the team includes Naval? Facts:
(A) 2 Fact 1: Mr. Sethi has eight kids.
(B) 3 Fact 2: Two of them are studying computers.
(C) 4 Fact 3: Three of them are studying science.
(D) 5 Fact 4: Three of them are studying art.
(E) 6 Fact 5: Those who are studying science are inclined
towards mechanics.
226. A group of 75 students was surveyed about their
preferred summer vacation activities. Out of Statements:
these 75 students, 27 preferred swimming, 35 I. All of Mr. Sethi’s eight kids are studying.
preferred boxing, and 28 preferred football. 10 of II. Only three kids show an inclination towards
these students preferred exactly 2 out of the three mechanics.
activities. 5 students did not prefer any of these
III. Mr. Sethi’s kids study either science or
three activities meaning they could prefer to choose
computers or art.
any, or all, or none at any point in time. What was the
maximum number of students that preferred exactly IV. Those who are studying art might also have an
one of these three activities? (Real NMAT Question) inclination towards mechanics.
(A) 25 (A) Only (I) can be concluded.
(B) 35 (B) Only (IV) can be concluded.
(C) Both (II) and (III) can be concluded.
(C) 40
(D) None can be concluded.
(D) 55 (E) All can be concluded.
(E) 60
230. Directions: Use the following diagram to answer
227. In a group of 150 students, 80 students like pop the given question. The overlapped area suggests
songs, 40 students like classical songs while 70 the product of the numbers of each figure in the
students like old songs. 30 students like both pop diagram. (Real NMAT Question)
and classical songs, 25 students like both classical
and old songs while 20 students like pop and old
songs. What is the minimum number of students 4

that like at least one type of song? (Real NMAT


Question)
2 b c d
(A) 120
(B) 130 e a
6
(C) 135

(D) 140
(E) 150 The value of which of the following is the minimum?
(A) a

628

Book 1.indb 628 30/04/2019 4:49:49 PM


7.0  Logical Reasoning Practice

(B) b 232. Which of the following outcomes could never have


(C) c been obtained by A in any of the four rounds?
(D) d (A) 1
(E) e (B) 2
(C) 3
231. The diagram represents four types of big cats
(D) 4
overlapping each other: (Real NMAT Question)
(E) 5
Rhombus: Royal Bengal Tiger
Parallelogram: White Tiger 233. In the third round, what percentage of the round
Pentagon: Cheetah score was contributed by the outcome of B?
Square: Leopard (A) 25%
(B) 33%
(C) 50%
A
F I (D) 60%
C G E
B (E) Cannot be determined
D
H
234. It is given that a particular pair of players obtained
the same outcome in each of the four rounds. What
was the outcome obtained by the other remaining
player in the second round?
Based on the relationships given in the diagram, (A) 1
point out the letter that represents:
(B) 2
Tigers that are both Royal Bengal and White only
(C) 3
(A) C
(D) 4
(B) E
(E) Cannot be determined
(C) F
(D) H 235. In the first round, C was the first player to roll the
(E) I die. In the subsequent rounds, the die was rolled for
the first time by the player who obtained the largest
Directions for Questions 232–235:  Answer the questions number as the outcome in the previous round. In
using the information given below. the subsequent rounds, if C never got the honour of
In ‘Famous Casino’ situated in Monte Carlo, three friends A, B rolling the die for the first time, which of the following
and C are playing with a fair die, which has the numbers 1, 2, could be the sum of the four outcomes obtained
3, 4, 5 and 6 marked on its six faces. Turn by turn, each one of by C?
them rolls the die once and notes down the outcome obtained. (A) 5
At the end of the first three rolls, a ‘round’ gets completed. The (B) 6
players roll the die to complete three more such rounds. For
each round, the “round score” is defined as the sum of the three (C) 7
outcomes obtained in that round. The following chart represents (D) 8
the ‘round scores’ of the rounds expressed as a percentage of (E) None of these
the maximum possible round score. (Real NMAT Question)
236. Manage the mathematical symbol to solve this
35% 33.33%
puzzle:
30% 27.77%
16 * 8 * 2 * 3 * 4 = 8
25%
22.22%
(A)
× + ÷ –
Round Scores

20%
16.67%
15% (B)
× ÷ + –
10% (C) – ÷ + ×
5%
(D) – ÷ × +
0%
First Round Second Round Third Round Fourth Round (E)
÷ –×+

629

Book 1.indb 629 30/04/2019 4:49:50 PM


NMAT by GMAC™ Official Guide 2019

237. If ‘+’ means ‘÷’, ‘×’ means ‘+’, ‘-’ means ‘×’ and ‘÷’ questions below. The numbers around will give you
means ‘-’ then, which of the following will be the the clues you need to solve the puzzle.
correct equation? 16 ÷ 4 = 144
(A) 20 + 25 ÷ 15 × 4 – 6 =10 15 ÷ 3 = 144
(B) 20 – 5 × 10 ÷ 20 + 4 = 20 10 ÷ 2 = 64
(C) 50 + 10 – 5 ÷ 5 × 20 =40 20 ÷ 5 = ?
(D) 15 – 5 × 50 + 25 ÷ 10 =40 (A) 169
(E) None of these. (B) 100
238. If + means ×, ÷ means -, × means ÷ and - means +, (C) 196
what will be the value of 16 + 5 ÷ 9 × 3 – 20 = ? (D) 225
(A) 42 (E) 256
(B) 66 Directions for Questions 243–245:  The following table
(C) 77 represent the number of players nominated for different
(D) 97 cricket teams.
(E) 103 Team Total Player Batsmen Bowler All
239. ‘abc’ represents a three digit number such that abc nominated rounder
x abc = ...7p, where 7p are the last two digits of the A 20 8 6 6
product. Find c. (Real NMAT Question) B 15 10 4 1
(A) 1
C 18 12 3 3
(B) 3
(C) 5 243. In how many ways can a team selector select 6
(D) 6 batsmen from team A?
(E) 9 (A) 18
240. If 0, 1, 2, 3, …..9 is represented by A, B, C, …..J; (B) 20
what is the value of BAF ÷ DF + JJ ÷ D? (Real NMAT (C) 24
Question) (D) 28
(A) 35 (E) 30
(B) 36
244. In how many ways a team selector can select 10
(C) 37 players in team B where 6 are bats men, 3 are
(D) 38 bowlers and 1 is an all-rounder?
(E) 39 (A) 720
241. If in the following equation, A – B = C; D – C = F; C (B) 800
+ D = E; and 9 – D = A; then which number could (C) 840
replace D and what would be the value of (A + F)? (D) 900
(Real NMAT Question)
(E) None of these
ABC
245. In how many ways a team selector can select 11
−D09
players in team C where 8 are bats men, 2 are
DEF bowlers and 1 is an all-rounder?
(A) 1, 8 (A) 4455
(B) 3, 8 (B) 4545
(C) 4, 8 (C) 4465
(D) 5, 9 (D) 4475
(E) 6, 9 (E) None of these
242. By using your numerical and logical reasoning skills Directions for Questions 246–248:   In a company 580
please try to figure out which number is missing in the employees are working. The admin of company survey

630

Book 1.indb 630 30/04/2019 4:49:50 PM


7.0  Logical Reasoning Practice

colour for T-shirt distribution. 200 employees like red, • The ratio of number of players who play only
250 employees like pink and 350 employees like yellow. Cricket to only Football is 2:7.
80 employees like red and pink, 120 employees like pink
249. How many student play Cricket and Football?
and yellow and 60 employees like yellow and red. Every
Employee likes atleast one of the three T-shirt. (A) 100
(B) 200
246. The number of employees who like only red is:
(C) 150
(A) 60
(D) 300
(B) 50
(E) None of these
(C) 40
(D) 30 250. What is the 20% of students who play Cricket and
Hockey?
(E) 100
(A) 10
247. The number of employees who like only pink is:
(B) 20
(A) 30
(C) 15
(B) 40
(D) 30
(C) 50
(E) None of these
(D) 90
251. The number of student who play Hockey and Football
(E) None of these
is:
248. The sum of the number of employees who like only (A) 50
yellow and all three is: (B) 100
(A) 180 (C) 180
(B) 190 (D) 120
(C) 170 (E) None of these
(D) 110 252. What is the difference between those who play all
(E) None of these three games and those who play Hockey only?
Directions for Questions 249–252:  Answer these (A) 150
questions on the basis of the information given below: (B) 240
1200 students of a college are involved in playing three (C) 250
games. These students are grouped under the following (D) 300
categories.
(E) None of these
Game Number of students 253. Which of the following diagrams indicates the best
Cricket 400 relation between Doctors, Mothers and Fathers.
Hockey 600
Football 800
(A)
•  The following information is also available.
• The number of players who play Cricket and
Hockey is three times the number of players who
play all 3 games.
• The number of players who play Cricket and (B)
Football is two times the number of players who
play all 3 games.
•  300 players play only Hockey.
(C)

631

Book 1.indb 631 30/04/2019 4:49:50 PM


NMAT by GMAC™ Official Guide 2019

(D)
(C)

(E) None of these
Directions for Questions 254–255:   In each of the
following questions, the three given words are related in one (D)
of the five ways represented by the diagrams given below.
Choose the correct diagram which depicts the exact relation
among the objects. (E) None of these

254. Grandmother, Mother, Girls


Directions for Questions 256–257:   Study the following
figure and answer the questions given below.

(A) 30 Educated

50

Men 25 20 45 Government Job


15
40
(B)

256. How many educated men are in government job?


(A) 25
(B) 20
(C) (C) 50
(D) 30
(E) 35

257. Find the number of people in government job who are


(D)
not educated.
(A) 35
(E) None of these (B) 40

255. Children, Humans, Animals (C) 50


(D) 55
(E) 65

(A) 258. The difference between the minor angle made by


hour hand and minute hand at 7: 20 and 9: 30 is:
(A) 4o
(B) 2o
(C) 3o
(B) (D) 5o
(E) 6o

632

Book 1.indb 632 30/04/2019 4:49:52 PM


7.0  Logical Reasoning Practice

259. Which of the following indicates advocates who (A) 41


are politicians and are not doctors? (Real NMAT (B) 51
Question)
(C) 62
Politicians
(D) 71
1 (E) 74
Advocates
5 261. Out of 140 mechanics, 10% can repair all three
7
3
devices - Air Conditioner (AC), Television (TV) and
2 6
Doctors
Refrigerator. The number of mechanics who can
4 repair exactly 2 of 3 electronic devices is 50. The
number of mechanics who can repair only TVs is 22.
The number of mechanics who can repair only ACs
is equal to the number of mechanics who can repair
(A) 1 only Refrigerators.
(B) 2 How many mechanics can repair only ACs, if each
mechanic can repair at least 1 of the 3 electronic
(C) 3
devices? (Real NMAT Question)
(D) 4 (A) 22
(E) 5 (B) 24
260. There was a total of 72 used cars in the inventory of (C) 26
ABM motors. (D) 27
· Any customer who had exchanged the cars had (E) 29
either purchased a new car or a used car. 262. If 4th January 2003 was a Saturday, then what was
· Every customer had purchased only one car. the day on 4th March 2004? (Real NMAT Question)
· The diagram below represents the transactions of (A) Sunday
the customers. (B) Monday
(C) Wednesday
B C A: Used cars
A (D) Thursday
B: Exchanged cars
41 27 31 34
C: New cars (E) Saturday

How many used cars are present in the inventory


after these transactions? (Real NMAT Question)

633

Book 1.indb 633 30/04/2019 4:49:52 PM


NMAT by GMAC™ Official Guide 2019

7.2 Answers and Explanations


The following discussion on answers and explanations is intended to familiarise you with the most efficient
and effective approaches to these kinds of questions. Remember that it is the problem solving strategy
that is important, not the specific details of a particular question.

1. Verbal Reasoning II. 


The second course of action II is also required
as this particular area has seen the most severe
impact of the epidemic. Hence both the actions
1. I. This is an obvious course of action.
follow.
II. 
When the Government takes such an action it is
necessary that people are made aware of the The correct answer is E.
consequences they would face if they do not obey
the directive. Hence II follows. 6. I. 
The first course of action is a suitable follow up by
the Government as it would deter the defaulting
The correct answer is E. companies.
2. I. 
The intensity of the infection has been the most II. 
But the second course of action does not follow
severe in four years. The epidemic itself is not a as there is no clear and direct link between the
four-yearly feature. Hence, precautionary measures lack of welfare measures and decline in the
should be taken every year and not every four services provided.
years. Therefore, action I does not follow.
The correct answer is A.
II. 
Action II is not a preventive action against
conjunctivitis, nor is it feasible. Hence II also does 7. I.  T
 he first course of action is useful as it is a logical
not follow. follow up of the problem. Step should be taken to
ascertain the truth of such serious allegations.
The correct answer is D.
II. 
But the second action is a long drawn and indirect
3. I. 
The first course of action is logical and should be one and hence, is debatable.
followed.
The correct answer is A.
II.  T
 he bank manager has just had a premonition.
He should first confirm whether money is actually 8 I.  Since the reason of the exodus of subscribers is
missing and only then he/she should inform the unknown, we cannot directly suggest a solution.
police for taking action. This statement makes an assumption that
perhaps there is something lacking in the offers,
The correct answer is A.
hence they should try out a combo offer. So this
4. I. 
The decision to revise the fee structure for artists will not follow.
should be taken by the channel as a remedy towards II. This also makes an assumption that perhaps the
the challenging problem that has arisen before it. subscription price of the rental is the cause - what
II. 
However, taking stringent action against people if it is the service or anything else? So this will
responsible for the programmes that saw a also not follow.
decline in popularity is a negative and reactive III. This is a correct course of action as it focuses on
step that will cause more damage. In such identifying the main cause of the problem. Only
scenarios, proactive and futuristic decision after that a decision on making a change can be
making is required. So only course I follows. taken.
The correct answer is A. The correct answer is C.
5. I. 
Since the enteric fever is a water borne disease, 9. The Chinese present tough competition to the Korean
its chief source is drinking water. The first course garment export industry that is venturing into Latin
of action may help contain the infection, as it often American markets. The correct courses of action here
spreads through open sources also. will be the ones dealing with the tough competition.

634

Book 1.indb 634 30/04/2019 4:49:52 PM


7.0  Logical Reasoning Practice

I. This cannot be a desirable course of action as it So, all the courses of action will follow.
is no solution to the problem, simply quitting from
The correct answer is D.
the market may save it from the competition but
will not be a solution to the problem.
14. Opening the water gates will surely create havoc in
II. Correct. This can be a desirable course of action the region as the water levels are already high. So,
as improving quality may make the products neither I nor III follows.
more desirable, thus giving the Koreans edge
Not opening the water gates gives the people of
over the competition.
the region more time to evacuate the area and take
The correct answer is B. other suitable measures. It is also possible that
the rains may cease and the resultant floods may
10. The statement says that the imported apples are of a recede. Additionally, the breaking of the dam is only
better quality. Thus, to compete against them, Indian predicted and eventually it may or may not happen.
apple growers should take steps to improve the So, the second course of action follows.
quality of their apples.
The correct answer is B.
Course of action I is not correct because a forceful
action is always undesirable.
15. The exclamation states that anyone who watches a
Course of action II is invalid because although this comedy movie will enjoy it.
might help the indigenous growers but the core
(A) The statement doesn’t necessarily mean that
problem of quality will not be catered to.
everyone watches movies.
The correct answer is D. (B) Correct. This is correct answer because if we
negate this option, the given exclamation loses
11. Course of action I doesn’t follow because providing
its stand.
a bailout package would be similar to throwing good
money after bad. Also if there are no signs of profit (C) This is not an assumption but an inference.
anytime soon, putting more money in it will not be (D) The statement that everyone watches only
feasible. comedy movies isn’t true.
Course of action II will follow because steps must be (E) It also doesn’t suggest that there are no other
taken to try and make the airline profitable by cutting kinds of movies.
excess expenditure and improving efficiencies. This
will deal with the core problem of losses thus solving The correct answer is B.
the problem.
16. An argument is true if its negation, negates the
The correct answer is B. statement.
12. I.  This option makes sense because if the issue of (A) Its negation doesn’t affect the statement.
public discontent is ignored, it could turn into a (B) Its negation doesn’t affect the statement.
major crisis situation for the government. (C) Its negation doesn’t affect the statement.
II. The government is tasked with the responsibility
of maintaining law and order. As a result, such a (D) Its negation doesn’t affect the statement.
passive approach will prove extremely harmful- (E) Correct. The author talks only of colours and
both for the government and the community. how with the availability of a colours, anyone can
III. This proposed course of action is vague and become a flawless painter. So, we can definitely
irrelevant. infer that according to the author, only colours
are important and the other features including
The correct answer is A. subject do not matter. If we negate this option,
the statement will be negated.
13. The statement suggests that there are many
The correct answer is E.
incidences of restaurants catching fire in a certain
area. So, it should be ensured that the reasons behind
the said incidences are checked. The statement 17. An assumption is only valid if its negation, negates
also mentions that the emergency services were the given statement.
slow to respond and hence, the response time of (A) If this is negated, the rule may still hold because
such emergency services should be improved. Also, the government talks of future travels which in
considering restaurants are prone to fire accidents, no way suggests that such unnecessary foreign
proper fire insurance should be made mandatory. visits were made in the past by officials.

635

Book 1.indb 635 30/04/2019 4:49:52 PM


NMAT by GMAC™ Official Guide 2019

(B) This is incorrect because the statement doesn’t (E) This statement actually strengthens the main
talk of a blanket ban on foreign travel. argument by stating that the population of
(C) This is stated in the argument and hence, is not monkeys has revived in areas that are under
an assumption. WWF control, that is, areas that have controlled
human intervention.
(D) Correct. The Government definitely wants to
cut down on avoidable expenditure, including The correct answer is B.
avoidable travel and that is why they have taken
out a rule like this. 20. Argument Construction:
(E) This is incorrect because we are not aware of the The passage states that a multispecialty hospital in
sequence of the austerity measures undertaken a small town had to close its operations due to the
by the Government. shutting down of a software company. We have to
look for an option that establishes a link between
The correct answer is D. these two phenomena. Before we look at the options
we can ascertain that the shuttering down of the
18. (A) The advertisement mentions the requirement for
software company should have had some impact
online tutors. From this, no conclusion can be
either on the earning capacity of the people or their
drawn with respect to the growth pattern of the
employability. Note that it is a small town and the
computer industry.
shutting down of a leading software firm employ-
(B) Correct. This assumption is implicit in the ing more than 10,000 people will have a significant
argument as the argument states the condition impact on the employment rates of that town which
that the applicants should be teachers. Hence, will in-turn impact the purchasing power of the peo-
it is possible that teachers can work as online ple. Let us consider the options now.
tutors as well.
(A) This option is illogical as there is no plausible
(C) The argument states that the teachers have to relation between the doctors working at the
be proficient in different subjects: Mathematics, hospital and the software company.
Physics, Chemistry or English. Biology is not
(B) If this had been the case, then the multispecialty
mentioned and hence, goes beyond the scope of
hospital would have continued to flourish as there
the argument.
would have been no dearth of manpower.
(D) We have no data with respect to the response
(C) Again, if this statement were true, the
for the advertisement. Therefore, this option is
multispecialty hospital would have continued to
irrelevant.
function unaffected.
(E) Again, we have no data regarding the other
(D) This statement is partially correct. We have already
subjects. Hence, this option also goes against
established that there has to be some relation
the scope of the argument.
between the people working at the software
The correct answer is B. company and the success of the multispecialty
hospital. However, the reason provided for this link
19. Argument Construction: - most of the diseases originated in the software
To weaken the argument, we need to give another company, is bizarre and incoherent.
reason (apart from habitat loss) why the Black Spider (E) Correct. Logically, the only connection between
Monkey may have become endangered. the clinic closing down and the software company
(A) The statement is talking about black spider closing down has to be the fact that the employees
monkeys and not squirrel monkeys. Hence, this of the software company were the regular clientele
option is beyond the scope of the argument. of the clinic. When the software company shut
(B) Correct. This option presents another reason for down, this clientele was lost and so the clinic
the dwindling number of Black spider monkeys. also had to shut down eventually. E conveys this
Hence, it weakens the relationship established reasoning and should be the correct answer.
between human activities and its impact on the The correct answer is E.
habitat of the monkeys.
(C) Once again, this is beyond the scope of the 21. Argument Construction:
argument. The statement is talking about Black The argument establishes a relation between reduced
spider monkeys only. ticket prices and increased aircraft occupancy. It
(D) This statement also goes beyond the scope of states that one way to reduce pricing is to stop
the argument. offering in-flight food.

636

Book 1.indb 636 30/04/2019 4:49:52 PM


7.0  Logical Reasoning Practice

(A) Not related to the main argument. an automobile sales might, at best present a
(B) Not related to the main argument tangentian reason.
(C) Correct. Due to the short duration of an airplane (D) Not relevant to the argument.
flight, it is not inconvenient for passengers to (E) Only option B is correct.
avoid food. Hence, the airlines can easily avoid
The correct answer is B.
this expenditure without any impact on customer
satisfaction. This cost-saving mechanism will
24. Argument construction:
reflect in lowered flight prices, a move that will
be appreciated among all travellers and one that The author claims that the availability of talented
will entice more and more people to fly. candidates for private banks will reduce as annual
recruitment is on the up in the public sector banks.
(D) This statement shall weaken the argument as it
That means the author assumes that the talent pool
discusses other factors that influence air travel.
for banking jobs is restricted.
(E) Option C is correct.
(A) This option rather weakens the argument by
The correct answer is C. saying that private sector pays more. Had this
been true, candidates might have preferred
22. Argument Construction:  private over public.
The statement makes two claims: one, that enhanced (B) Correct. If there is a limited resource pool for
exposure to mass media is responsible for an banking jobs, then the increase in the recruitments
increased awareness amongst people regarding their being done by public banks will definitely create a
rights and second, that this has also led people to paucity of candidates for private sector jobs.
fight for their rights.
(C) This option again weakens the argument. If
(A) Option A strengthens this claim by providing data the jobs in public sector are not considered
in its support. challenging enough, then there might be a
(B) Option B weakens the argument by raising disinclination towards them.
questions on the content showed by the media. (D) Though this option presents one reason why jobs
(C) Option C also presents information that in public sector banks may be more lucrative for
strengthens the argument. candidates, it addresses only one aspect without
(D) Correct. Both options A and C strengthen the focusing on the key issue.
argument. (E) Only option (B) is correct.
(E) Option D is correct. The correct answer is B.
The correct answer is D.
25. Argument Construction:
23. Argument Construction: The author makes an argument in favour of making
Here, the government has decided to deregulate Hindi the national language of India. The reason
the prices of diesel and petrol. This means that the he gives for this suggestion is the fact that the
market forces will control the prices and not the government wastes a lot of money in translating
government. official documents into local languages; money that
can be utilised elsewhere.
(A) This statement weakens the argument. If an
increase in the prices of fuel will have a negative (A) This statement strengthens the claim of the
impact on its consumption, then the move will author by providing data that supports the main
prove counterproductive. argument.
(B) Correct. The statement says that this decision (B) This option does weaken the argument
has been taken to reduce the losses being faced tangentially – if Hindi is the most spoken
by fuel companies. Now, this will happen only if language in the country, it does not preclude
the demand for this product is not affected by the fact that there is still a percentage of
fluctuations in its price. Hence, even if the prices population that does not speak Hindi. Hence, the
go up, the government is reasonably sure that government is duty bound to cater to the needs
the demand will remain unaffected, helping the of this segment no matter how small it is.
companies increase their profits. Hence, option B (C) This option is beyond the scope of the argument.
strengthens the argument. Whether the people pay taxes or not is not the
(C) The primary reason being given for the central point being discussed. The main argument
deregulation is the refuction of losses. A dip is talking about the utilisation of people’s money.

637

Book 1.indb 637 30/04/2019 4:49:53 PM


NMAT by GMAC™ Official Guide 2019

(D) This option actually strengthens the author's credit exceeds the saving on purchases then there
claim. If people travelling accross the country is no point in using such a facility.
manage to learn Hindi within a year, it implies (D) This option is far-fetched and beyond the scope
that the language can easily be learn and is of the argument.
spoken in large parts of the country.
(E) This is merely a repetition of the main argument.
(E) Correct.This option directly contradicts the
information given in the main statement – the The correct answer is C.
passage states that Hindi should be made the
28. Argument Construction:
official language, while the option states that
Hindi is already the official language. This is a sampling assumption. The argument in
this passage is that male readership of novels is
The correct answer is E. significantly lower than the female readership (based
on surveys in Britain, the United States and Canada),
26. Argument Construction: and that if women were to stop reading them, novels
The argument states that scientists in India and other would die out.
commonwealth countries do not pay attention to (A) This is only a shortcoming of Ian McEwan’s
the studies done in non-english speaking countries. impromptu ‘survey’ (a fact mentioned in the
However, scientific studies are being conducted in paragraph itself) and not of the whole argument.
other non-english speaking countries as well.
(B) This is a tricky option. Whether or not men read
(A) This option goes beyond the scope of the more books than women per capita, the bottom
argument. The passage is specifically talking line is that they still form only 20 percent of the
about ignorance among the scientists of the market for fiction (in the stated countries, at
Commonwealth. We cannot draw generic least), so option B is not a suitable answer.
conclusions based on this.
(C) The argument is specifically about novels (i.e.
(B) Correct. If the scientific breakthroughs being non-fiction), so whether or not men read plenty of
made in the non-english speaking countries are non-fiction is irrelevant, and option C is incorrect.
not recognised by scientists in commonwealth
Correct. The argument rests on the assumption
(D) 
countries, then it can be inferred that there will
that the patterns of reading novels in the three
be an overlap between the scientific findings
countries cited is representative of a worldwide
reported in these countries and those published
pattern, but if this is not the case—as stated in
in non-english-speaking countries.
option D—then the argument is weakened.
(C) This option weakens the claim made in the passage.
(E) Only option D is correct.
(D) This is a vague and moralistic argument.
The correct answer is option D.
(E) This statement shall weaken the claim made in
the passage. 29. Argument Construction:
The correct answer is B. The argument in the paragraph is that going to the
gym is far too similar to joining a cult in the kind of
27. Argument Construction:
fanaticism it inspires in its adherents.
The passage discusses the merits and demerits of
(A) Just stating that some people are not fanatical
buying on credit. An advantage that it presents is that
about going to the gym does not change the fact
buying on credit allows people to benefit from sales
that others are. So, option A is incorrect.
and bargain offers, when they are short of cash.
(B) The paragraph is not about the workings of gyms
(A) This option merely presents an advantage of
and cults per se, but the feelings they inspire in
buying on credit.
their members, so option B is slightly tangential
(B) This option makes an erroneous connection to the point.
between credit and bargains.
(C) Option C indicates that people go to the gym for
(C) Correct. This assumption is central to the argument reasons that include socialising or feeling good
being made by the author. Using credit options to about themselves. These are not touted to be
avail oneself of bargains and sales will only prove fanatical reasons. However, this option does not
beneficial if the cost of the credit is less than the definitely rule out that the reasons are unbiased
savings that the buyer will make. If the cost of and logical. Hence, option C is incorrect.

638

Book 1.indb 638 30/04/2019 4:49:53 PM


7.0  Logical Reasoning Practice

(D) Correct. Option D casts doubt on the basic directly proportional to the degree of relatedness
argument, by stating that however fanatical between them, as measured by the percentage of
people may be about going to the gym, their genes they share.
fanaticism has a scientific basis, as opposed to (A) 
Correct. This option, which shows that twins
the irrational reasons for joining a cult. who share all their genes are emotionally closer
(E) Only option D is correct. than twins who share only half, strengthens this
argument.
The correct answer is option D.
(B) 
Option B is irrelevant as it talks about how alike
30. Argument Construction: certain siblings are, which is not an issue in this
The paragraph is making a case for the intangible argument at all.
(i.e. non-financial) benefits of hosting high-profile (C) 
Option C is also beyond the scope of the
sporting events for the host city or region. argument due to the reason provided above.
(A) Option A presents another intangible benefit of (D) 
Option D shows a contrary finding, so it weakens
hosting such events - communal harmony and the argument.
reduced friction amongst the different elements (E) Option A is correct.
of society. This strengthens the main argument
being made by the author. The correct answer is option A.
(B) Option B weakens the argument as it shows that 33. Argument Construction:
the benefits are too short-term to be of much
The argument essentially draws a parallel between
worth,
equal opportunity, equal income and economic
(C) Option C also weakens the argument as development. It states that western countries have
it shows that the intangible benefits may be strived for equal opportunity and have been able
accompanied by intangible disadvantages, which to achieve it to a large extent – by having more
may outweigh the benefits. equitable distribution of wealth. Economists in India
(D) Options B and C weaken the argument. are in favour of the equal income model as they
(E) Only option A is correct. believe that it will accelerate economic development.
(A) This option is logically contradictory – if equal
The correct answer is option A.
opportunity does result in equal income, then
31. Argument Construction: how is it possible that people don’t get paid for
their efforts.
The passage argues that the war on spam has been
won. It presents the enhanced security features of (B) The passage is not talking about the utilisation of
google as evidence for this claim. excess income.
(A) The fact that e-mail services will get better at (C) This option reiterates the main idea and
catching spam in the future does not really strengthens the argument.
strengthen this conclusion, as they are evidently (D) Correct. Economists in India are in favour of the
doing the job very well even now. So option A is equal income model as they believe that it will
ruled out. accelerate economic development. However,
(B) Option B is irrelevant, as the paragraph is about the ground reality presents a different picture. In
spam, not legitimate e-mail. developing countries, the gross income of most
of the people is so low that if it were distributed
(C) Correct. Option C strengthens the conclusion, by
equally, no one would save enough to provide
showing that even the senders of spam may be
resources for investment. This will negate the
slowly acknowledging defeat.
very rationale behind introducing the equal
(D) Option D slightly weakens the argument, as the income model.
phrase ‘so far’ implies that the spam filters may
(E) This option is beyond the scope of the argument.
not be able to keep up in the future.
(E) Only option C is correct. The correct answer is D.

The correct answer is option C. 34. Argument Construction:


We need to find a statement that supports the
32. Argument Construction:
conclusion presented in the main argument, that is, not
The basic argument in the paragraph is that the signing the open trade agreement will prove equally
degree of emotional closeness between people is harmful for the developing and developed countries.

639

Book 1.indb 639 30/04/2019 4:49:53 PM


NMAT by GMAC™ Official Guide 2019

(A) For any inference to be true, there has to be 36. Argument Construction:
ample data in the argument about it. This option The argument stresses on the importance of having
is about WTO. The main statement cannot be a competent manager. It reinforces this belief by
inferred from here. stating that more attention and resources are spent
(B) This statement is a conclusion about why the on training managers than the other employees.
third world countries are weak, this is too much (A) Option A contradicts the contention made in the
to infer from the argument as there is no data main argument by stating that training is not
about the reason. required for department heads.
(C) This statement contradicts the information
(B) Option B is beyond the scope of the argument.
provided in the main statement. If the developing
countries are being hindered due to their current (C) The argument is talking about the importance of
practices they will not shy away from doing training senior management. There is no mention
this(signing the agreement). of the time required by the various people.
(D) This statement is like a conclusion about why (D) Correct. This assumption is implicit in the
third world economies shy away from signing argument. If the cost of having an inept senior
WTO’s free-trade agreement. So it is an incorrect manager is prohibitive, it stands to reason that
option. the training of managerial staff is crucial for an
(E) E gives a better picture of the third world organisation’s success and well-being.
countries being affected badly if they do not (E) This option goes beyond the structure of the
reach the two-digit growth rate, which in turn argument. Whether a manager responds to
is affected by the signing of WTO’s free-trade training is not discussed in the passage.
agreement. Thus, the first part of the stem The correct answer is D.
statement can more or less be concluded from E
and the second part of the stem statement can 37. Argument Construction:
be inferred as global economies are interlinked.
The argument concludes that the poor performance
The correct answer is E. of the movie can be attributed to the talk show host
making fun of it.
35. Argument Construction: (A) Option A strongly supports the argument.
An animal conservation group is trying to change (B) Option B is invalid.
the image of owls by giving the argument that they
are feared because they are reclusive and work in (C) Option C does not specify whether the host is
the night. To weaken the argument, we can give responsible for the poor performance of the
examples of other animals that are not feared movie.
despite being reclusive and working in the night. (D) Correct. Option D reverses this logic by stating
(A) This option gives more data about places where that it is because the movie was performing
owls are feared so it sort of strengthens the badly that the talk show host was making fun
argument and not weaken it. of it. Thus, the talk show host could not be
responsible for the failure of the movie.
(B) Correct. By stating that raccoons and bats are
also reclusive and only come out in the dark, (E) Option E can be taken into consideration as it
it weakens the argument that owls are feared says the movie was criticised by media also but
because they are reclusive and only come out in D is the most appropriate option.
the dark - if this were the case, then raccoons The correct answer is D.
and bats too should have been feared.
(C) This statement talks of other reasons why ows 38. Argument Construction:
are feared more but doesn’t address the core A survey was conducted to know the satisfaction
argument mentioned in the question. levels among the citizen of a city. Since majority of the
(D) Just like (C) this statement also brings out other respondents said they were satisfied, a conclusion was
reasons as to why owls are feared more but drawn that the citizens as a whole were satisfies as
doesn’t do anything to weaken it. well.
(E) This statement talks about predicting the To weaken the argument, we need to find a flaw in
behavior of other animals. So it is out of the the whole survey.
scope of the argument. Correct. If the sample was taken from only
(A)
a particular economic stratum, the sample
The correct answer is B.

640

Book 1.indb 640 30/04/2019 4:49:53 PM


7.0  Logical Reasoning Practice

becomes biased. Thus, this option highlights a To weaken this argument we either say that the other
flaw in the argument. alternative is better or this way of preparing oneself
(B) Some of the residents may have said they were is not sufficient or a combination of both of them is
dissatisfied. Still the argument may be valid that even better.
as a whole the citizens are satisfied. It doesn’t (A) This option only mentions the opinions of some
weaken the argument. marathon runners. Thus, it isn’t a very plausible
(C) Comparing the results with the neighbouring option.
cities doesn’t impact the argument at all as the (B) Correct. This option says combining both the
argument is about only one city. methods is even better. So the conclusion is
(D) This statement rather strengthens the argument. By negated.
stating that the sample was representative of the (C) This argument is out of scope as mentioning the
entire population, we strengthen the argument. purpose of participation doesn’t relate to the
(E) Mentioning the governing bodies makes the conclusion drawn in the question.
argument out of scope as there is no reference (D) This option mentions pace but doesn’t refer to
made to them in the question. either of the two alternatives mentioned in the
argument. So this is an incorrect option.
The correct answer is A.
(E) This option can also be eliminated with the
39. Argument Construction: same logic that to negate the argument we
Scientists state the advantage of Element Z in need to relate the options with at least one
manganese and the argument specifies that this of the alternatives. This argument doesn’t
element is naturally high in manganese found in do that.
Australia. This certainly makes the manganese found The correct answer is B.
in Australia more desirable.
(A) This option states ‘highest’. The argument 41. Argument Construction:
doesn’t give us sufficient data where we can infer This is basically a question which tests averages. If
Australian manganese having the highest amount the average of A and B is 24%, out of which B is only
of element Z. 6%, then A has to be greater than 24%.
(B) The argument says the level of element Z is high (A) Since there is no comparison made between
in manganese found in Australia but since we A and other brands, this cannot be inferred.
cannot compare it with other countries, this is an (B) Talking about prices makes the argument out
incorrect option. of scope as there is no mention of it in the
(C) This option can also be eliminated by the same argument.
logic that since no other country is mentioned, (C) Correct. By averages, we can certainly conclude
we cannot say for sure that Manganese that is this.
not from Australia is very likely to encounter the
(D) Had this been true, the percentage of B phones
problem of small breakages.
would have been higher than A. So this cannot be
(D) There may be other ways of making manganese inferred.
strong, adding element Z being one of them. So
(E) We cannot say with certainty if the number of
this conclusion becomes uncertain.
people who buy phones may have increased. So
Correct. This statement becomes correct
(E) this is an incorrect option.
because of the word ‘some’. It is a certain
conclusion as Australian manganese has high The correct answer is C.
amounts of element Z.
42. Argument Construction:
The correct answer is E.
The author has made a claim that the reason
40. Argument Construction: behind high purchasing power in rural areas as
compared to urban areas is that some money used
The author states two things that help a person
by city dwellers on housing and food can be used
complete a marathon, either by practicing for 4
somewhere else by rural dwellers.
hours or consuming high energy foods. Out of the
two, the author prefers practising over diet. On the (A) Since the money used by urban dwellers on
basis of his preference he draws a conclusion that if housing and food is used somewhere else by the
one practises well, one can ensure one is doing the rural dwellers, this option cannot be true with
most that can be done to perform well in a marathon. reference to the rural dwellers.

641

Book 1.indb 641 30/04/2019 4:49:53 PM


NMAT by GMAC™ Official Guide 2019

(B) The actual income may be the same between (B) The question says the best chances of survival of
rural and urban dwellers, just that their purchasing tigers are in India. This doesn’t mean they cannot
power varies because of the reason mentioned in survive anywhere else. So this is an incorrect
the question. So this cannot be inferred. option.
(C) Correct. If both rural and city dwellers earn the (C) We cannot be sure if the Indian government
same income, yet the rural dwellers have a is taking ample of steps or not. May be the
greater purchasing power, it must be because government is doing something, still the
the city dwellers are spending more on food and number is declining because of other reasons.
housing, making C the answer. (D) 'The chances of the survival of tigers are the best
(D) This option can also be eliminated using the in India' doesn’t mean that the existing number of
same logic used in (B). Their incomes may be the tigers is the highest in India.
same or may vary. (E) The rate at which the number of tigers is
(E) The amount of savings will not impact the declining cannot be commented upon as it is out
argument any how as the argument talks of of the scope of the argument.
purchasing power, linking it to the expenses on
The correct answer is A.
housing and food.
45. Argument Construction:
The correct answer is C.
A city’s governing council has spent a lot of money
43. Argument Construction: on advertisements on the harmful effects of smoking
To be able to clear a particular test, a person has and these advertisements are targeted towards
to fulfil two criteria. If he isn’t able to fulfil any one college students. The authority thinks the number
of them, he wouldn’t be able to clear the test. This of smokers in college campuses has gone down
is followed by the example of Kavita who in any because of these advertisements.
scenario cannot fulfil both the criteria. (A) Since this statement mentions alcohol it can
(A) This argument is merely an opinion without any easily be eliminated as being out of scope.
logical backing. We can easily eliminate opinions (B) The question targets only college goers.
as incorrect options. Comparing it with non college goers makes this
(B) As the question says, to be able to clear the test a wrong option.
one has to fulfil both 15 coaching sessions and (C) This statement talks of the whole city and not
20 practice tests, hence, this option cannot be college. Since the point of the argument is about
inferred as it talks about completing only one colleges, this is a wrong option.
requirement. (D) Correct. The argument assumes that there
(C) We cannot be sure if Kavita would retake the test can be no other reason for the decrease in the
or not. So this is an incorrect option. number of smokers except the advertisements
(D) Correct. If under the given circumstances Kavita by the authorities. Option D weakens this by
isn’t able to fulfil both the requirements together, providing an alternative explanation for the drop
it is most likely that she will not be able to clear in the number of smokers within the city—higher
the exam as to clear it both requirements have to cigarettes prices.
be fulfilled. (E) This statement has nothing to do with the
(E) This option can also be eliminated using the argument made by the authority regarding their
same logic used in (B). Only one criterion is advertisements. So it is an incorrect option.
fulfilled here.
The correct answer is D.
The correct answer is D.
46. Argument Construction:
44. Argument Construction: Since the number of badminton racquets being sold
The number of tigers throughout the world is in Chennai has tripled in one year, the author draws a
declining. To combat this problem, experts argue conclusion that more and more people have started
that the best chances of the survival of tigers are playing badminton in Chennai. Anything that relates the
in India. conclusion to the premise will strengthen the argument.
Correct. As the question says, conservation is
(A)  (A) If tourists buy these racquets, the number of
a major concern across the world. Thus, this people playing badminton in Chennai will not
statement has to be true. increase. This is an incorrect option.

642

Book 1.indb 642 30/04/2019 4:49:53 PM


7.0  Logical Reasoning Practice

(B) We cannot comment about the sale of racquets (D) This statement also sort of weakens the
in other cities. Also, this won’t impact the argument by stating children should be taught
conclusion at all. to be safe and keep away from dogs. What is
(C) Correct. The argument is based on the assumption needed in the argument is vice versa.
that most of the badminton racquets sold are (E) Correct. The evidence in the argument mentions
actually being used by residents of Chennai. Rottweilers but the conclusion is for all dogs in
(D) There is no reference to cricket bats and golf general. Thus, the assumption has to be that
clubs in the argument. So this option is out of the Rottweilers are representatives of dogs of all
scope of the argument. breeds, making E the correct answer.
(E) E actually weakens the argument by suggesting The correct answer is E.
that the increase in sales of badminton racquets
may not lead to an increase in the number of 50. (A)  The statement doesn’t suggest that there are no
residents of Chennai taking up badminton. other categories of tickets.
The correct answer is C. (B) Platinum ticket must definitely be more expensive
because of the exclusivity status of the ticket.
47. (A) Correct. This strengthens India’s decision as (C) This cannot be inferred as the location of
such a big market should definitely be tapped. Platinum ticket seats is not given.
(B) This weakens the decision because it states that (D) This also cannot be inferred as the statement
the entry to the market is difficult. doesn’t talk of what is allowed for Platinum ticket
(C) This weakens the decision as it suggests that holders.
there isn’t much scope for exports to China. (E) Correct. This can be clearly inferred because the
given situation can be checked only in case of an
(D) This makes the exports to the country less
inspection.
favourable, hence, weakening the decision.
(E) This weakens the decision because it states that The correct answer is E.
the entry to the market is difficult.
51. The increase in revenue as well as tax considering
The correct answer is A. the drop in sales volume can only be attributed to the
increase in sale price of tobacco.
48. (A) This is in line with the decision of the University. (A) This cannot be inferred.
(B) Correct. This weakens the decision as the option (B) This is definitely wrong as the sales volume has
states that the required infrastructure will take decreased.
2 years to build. So, an accreditation cannot be (C) Correct. Since this statement does not say that
obtained in 6 months. the consumers are not sensitive only to the
(C) This is in line with the decision of the University. price of tobacco, it does not eliminate scope for
(D) This is in line with the decision of the University. the possible impact of other variable factors of
(E) This is in line with the decision of the University. influence (e.g. a change in tax rate). Therefore, it
is the correct answer.
The correct answer is B. (D) Absence of alternative products can lead to
non-sensitivity of consumers towards price
49. Argument Construction:
and hence, may ensure constant sales volume
On the basis of an instance where more than a dozen despite increase in price. The given statement
children were attacked by dogs, the author draws talks of decrease in sale volume and hence, this
a conclusion that dogs shouldn’t be kept as pets in is incorrect.
neighbourhoods with a large population of children.
(E) There is no information that suggests a change
(A) The argument relates dogs to children. Bringing in tax rate which may or may not have changed.
adults and other dogs doesn’t impact the
argument anyhow. So this is a wrong option. The correct answer is C.
(B) Rottweiler is merely an example used to represent 52. (A)  It is a possibility but cannot be definitely inferred
dogs as a whole. Giving a specific argument from the given statement.
about Rottweiler doesn’t impact the argument.
(B) Correct. This is definitely true because if
(C) This statement rather weakens the argument by quality of the image was the only feature to be
giving the example of Pugs that are very friendly considered, the companies would have definitely
and can be kept as pets. switched over to the new technology.

643

Book 1.indb 643 30/04/2019 4:49:53 PM


NMAT by GMAC™ Official Guide 2019

(C) It is a possibility but cannot be definitely inferred (E) Waiving loans is a good idea but it doesn’t make
from the given statement. sense to ask the farmers to relocate to other
(D) It is a possibility but cannot be definitely inferred parts of the country.
from the given statement. The correct answer is D.
(E) It is a possibility but cannot be definitely inferred
from the given statement. 55. (A)  Considering the positive effects of the
development work, it should not be hindered.
The correct answer is B.
(B) Considering the positive effects of the
53. Argument Construction: development work, it should not be hindered.
A replacement of a team leader is needed. The (C) Considering the positive effects of the
things that need to be kept in mind are: the project is development work, it should not be hindered.
midway, time is important, the team is edgy because (D) Correct. Having a nonchalant attitude toward the
of pressure. environmental damage is not right and hence,
The best option will be the one that deals with the option D is the best course of action that doesn’t
above mentioned problems. hinder the positive effects of the development
From the scenario, it is clear that with edgy high work.
performing individuals and a client who is eagerly (E) Considering the positive effects of the
waiting for deliveries, there is going to be high level development work, it should not be hindered.
of engagement with the team and the client - hence,
The correct answer is D.
a person with good people skills would be required.
(A) This option can be eliminated as we need 56. (A)  The image of the athlete is definitely tarnished at
somebody with good people skills. the moment and hence, no reaction to the news
Correct. Since the team and client both are edgy,
(B) will hurt the company’s image, even though it
we choose a person who can handle both of may be temporary.
them well. (B) Cancelling the athlete’s contract will be unethical
(C) This option also can be eliminated as we need as the media reports could only be speculation.
a person with people skills. Also, this option (C) The image of the athlete is definitely tarnished at
is based on an assumption that someone else the moment and hence, no reaction to the news
might leave as well. will hurt the company’s image, even though it
(D) This option gives no reason why to choose may be temporary.
whom. It merely gives an opinion. So this is (D) Correct. The contract should be put on hold till
wrong. proper evidence is presented for or against the
(E) This option doesn’t take any decision. It just athlete.
involves the HR which in any case would be done. (E) It is an extreme step.
The correct answer is B. The correct answer is D.
54. (A) The situation is quite severe as a lot of farmers
57. (A)  It is an extreme step and should be avoided.
are committing suicide. In this scenario, merely
counselling the farmers will not help because (B) It is farfetched because an eligibility issue of
their problems will still stand. a single athlete is not a big enough reason to
cancel the event.
(B) This option is quite passive in its approach. Just
updating the central government and waiting for (C) It is an extreme step and should be avoided.
their directives places the entire responsibility (D) It is farfetched because the eligibility issue of
on the central government only which is neither a single athlete is not a big enough reason to
correct nor desirable. cancel the event.
(C) The agriculture department cannot wash its (E) Correct. As checking the gender of the athlete 
hands of the problem by claiming force majeure.   is necessary, the country should be made to
(D) Correct. This is a logical course of action. understand the importance of the situation.
Waiving loans will provide much needed relief to The correct answer is E.
the farmers while the free seeds will help them
plan for the new crops.

644

Book 1.indb 644 30/04/2019 4:49:53 PM


7.0  Logical Reasoning Practice

58. Price skimming means setting a relatively higher To prove it wrong he compares politics to sports and
initial price for a product or service and then lowering says it is possible only in sports and politics is not a
the price over a period of time. The argument sport.
in favor of price skimming strategy can only be To strengthen the argument we must favour the
strengthened by a point which suggests that there author’s stance.
is another factor more important than price for the
consumer. To weaken it we must oppose.
(A) This doesn’t support the need of a newer (A) This argument weakens the statement. The
version. author’s stance is that two party system isn't
conducive to democracy. This option presents
(B) Correct. This suggests that the highest demand
facts that directly contradict the author's claim.
will be in the very beginning when the new
version is here because large organisations (B) This is merely an opinion. All the opinions are
invariably go for them. So, having a higher the first options to be removed as wrong as they
introductory price will make the most of the initial aren’t substantiated.
demand. Hence, it is a very strong argument in (C) The author disagrees with a two party system
favor of price skimming strategy. stating there cannot be democracy in it .This
(C) This will motivate the company to have a lower option also points out a flaw in the two party
price and hence, will be against the price system thus supporting the author.
skimming decision. (D) Since the argument is about a two party system
(D) The option suggests that sales will be lower but the correct option also has to revolve around
does not suggest that they will increase with time. that. Sports is merely an analogy used. So this is
So, a price skimming strategy may not be feasible. a wrong option.
(E) The option suggests that sales will be lower but (E) Although this option supports the stance of
does not suggest that they will increase with time. the author, it gives only an opinion and doesn’t
So, a price skimming strategy may not be feasible. substantiate it with reasoning. We cannot choose
The correct answer is B. it over (C) which gives a valid reason.
60. The correct answer is C.
59. Argument construction:
61. The correct answer is A.
The statement establishes a relationship between
practice and proficiency. We need to select that 62. (A)  The option suggests higher demand which may
statement as the answer, which replicates this logic. result in growth of the pharma sector.
(A) This option presents genetics as playing a key (B) The option suggests higher demand which may
role in an individual’s abilities. result in growth of the pharma sector.
(B) This option actually weakens the argument by (C) The option suggests higher demand which may
stating that practice is not helping a person build result in growth of the pharma sector.
his capabilities. (D) This is a prediction of growth of the pharma
(C) This option also weakens the argument. sector.
(D) This option also negates the impact of practice (E) Correct. This weakens the possibility as it
on capabilities as it states that the person is suggests lower demand for pharma products in
doing well inspite of not being able to practice. the future as the focus is shifting to traditional
(E) Correct. This option reinforces the relation methods/remedies of treatment.
between practice and proficiency – it presents The correct answer is E.
an example wherein a person has enhanced his
proficiency in something by regular practice. 63. Argument construction:
The correct answer is E. The passage states that in order to deal with
rebellions, governments should be mindful, tolerant,
General Solution for Questions 60–61: and open to peaceful negotiation.
(A) The given argument supports the role of a
Argument construction: tolerant, understanding and open government.
The author says that the claim people make that a This has been reiterated in option A which,
two party system can also be democratic is a farce. clearly, strengthens the given argument.

645

Book 1.indb 645 30/04/2019 4:49:53 PM


NMAT by GMAC™ Official Guide 2019

(B) Option B pertains to the general characteristics 67. Argument I: The author claims that if there is an
of man’s nature; a government’s approach is not informal evaluation system, the pressure will be
relevant to it. less as compared to regular examinations. This is
(C) Option C refers to the general attitude of a desirable action as everyone would want reduced
authorities towards those who challenge them; pressure on kids. Thus this is a strong argument.
but it does not dwell on solving the rebellion. Both Argument II: This argument presents a flipside of
options B and C have no impact on the given exams stating that if exams are conducted, a child
argument. They neither support nor contradict it. is prepared to face pressure in future life which
(D) Correct. Option D directly contradicts the view is much needed. This also makes a very strong
presented in the argument. It presents an argument.
alternate way of dealing with rebellions, through Argument III: is frivolous in light of the statement
an iron hand approach, that has been adopted being argued. There is no relation between a student’s
successfully by dictators. etiquettes and exams. So this is a weak argument.
(E) Option E also strengthens the given argument. The correct answer is D.
The correct answer is D.
68. Argument construction:
64. Argument I is weak because it does not give any The argument states that the fact that children find
reason in support of its thesis. Nobody is saying that it difficult to digest milk is a reason given by them
the Indian and Western cultures are the same, but for not wanting to drink it. We have to find an option
just because they are different is no reason to not that supports or reinforces the claim made by the
follow the Western culture. children.
Argument II is strong as it gives a logical reason for (A) This option is wrong as it is an opinion that these
accepting the Western culture. are excuses.
Argument III is weak because it is negative in (B) Since the question stem talks about children,
approach and indulges in a blame game without an option targeting adults cannot strengthen or
giving any facts to support these claims. weaken the argument in any manner.
The correct answer is B. (C) This option also doesn’t relate to the argument
at all as it presents an opinion that says milk is
65. Argument I: Though the first argument appears good but provides no reasoning for the main
morally strong, please note that in questions based argument.
on reasoning, logic, not emotion or values, should
(D) Lack of calcium might have a detrimental impact
dictate the final decision. Such statements are
on the well-being of children but this statement
‘judgements' and not ‘arguments’.
also does not connect with the main contention
Argument II: However, one also has to keep in mind being made by the author.
that the decision one makes is ethically sound. Thus,
(E) Correct. E addresses the issue of the ability of
even though the decision might render thousands
people in general (including children) to digest milk.
unemployed, it will be desirable in the larger societal
If research has proven that lactose intolerance is
context. Thus, neither arguments are strong.
present in some people, then it is possible that the
The correct answer is E. claim made by the children is correct.
The correct answer is E.
66. Argument I: The state machinery can make
provisions for the training/deputation of the required 69. Argument I says that demolition will dissuade such
staff or other necessary infrastructure that is needed builders from carrying out such activities. This is
for any initiative. Thus, claiming paucity of resources certainly a desirable action. Thus it makes a very
as a reason for not doing something makes for a weak forceful argument because punishing in this case is a
argument. necessity.
Argument II: Though it is important to have a sense Argument II is weak as though the people living there
of pride in one’s heritage, forcing people to learn may not have any place to go, one cannot just allow
the language is no guarantee of the same. Also, a the illegal construction to continue for the benefit
lot of initiatives can be undertaken to promote Tamil of few people only. It can never be a justification to
that might prove more effective. Therefore, both the allowing illegal construction.
arguments are weak.
The correct answer is A.
The correct answer is D.

646

Book 1.indb 646 30/04/2019 4:49:54 PM


7.0  Logical Reasoning Practice

70. Argument I says that with the easy availability of 75. Assumption I: Mohan has requested his mother to
consumer loans, the accessibility and purchasing arrange food for about 30 people and he has also
power of the Indian consumer has increased. This invited all his friends. We don’t know the total number of
argument is always desirable and good for the Mohan’s friends. Hence, it is not possible to ascertain
consumer. Thus, it is a strong argument. that ‘30’ constitute what percentage of this total – are
Argument II also states a very valid point that if the they ‘some’, ‘most’ or ‘all. For example – maybe the
purchasing power of the consumer has increased, total number of Mohan’s friends is 100; in this case
the consumer will buy things which he may otherwise 30 is not ‘most’ of his friends. However, if the total
won’t need, leading to unnecessary spending. Thus, number of Mohan’s friends is 50, then 30 constitute
this argument also is strong. most of his friends. But if Mohan has asked his mother
Both the arguments make logically valid points and to prepare food for about 30 people, it means that the
are strong. total number of his friends is definitely more than 30.
Therefore, the only conclusion that can be drawn is that
The correct answer is E. Mohan is expecting some of his friends to attend the
party.
71. Argument I: This is a weak argument. If an action
Assumption II: There is nothing in the statement
is opposed on the ground that it will increase the
implying that more than 30 people may attend Mohan’s
expenditure of the government, even if the general
birthday party. If that were the case, Mohan would have
public is benefitted from the action, it is considered a
asked his mother to arrange food for more people but
weak argument.
he did not do so. Thus, Assumption II is not implicit.
Argument II: This is a strong argument. Increased
foreign interest in the country’s education system is The correct answer is E.
a desirable action that will result in enhancement of
infrastructure, better facilities, research opportunities 76. Assumption I: Extreme words like only and all
and so on. Thus, argument II is strong. usually make assumptions wrong. The claim the
author makes is about Ravi Sharma’s book but we
The correct answer is B. cannot make any assumption about all books. So it is
a wrong assumption.
72. Argument I: This is a valid argument. Any argument
Assumption II: The question states that this book
that supports added burden on the general
will be his first best seller, this certainly means that
population, either in terms of money or hardship, is
he did not have a best seller till date.
usually considered a weak argument. Also, closing
a loss-making enterprise is an extreme solution The correct answer is B.
that should be preceded by measures to revive the
business. Thus, argument I is a strong argument. 77. Assumption I: Though, on a quick examination of the
argument, there does appear to be a link between
Argument II: This is also a strong argument as it
the number of hours clocked and the award, no such
provides a logical justification for taking the decision
relation has been established in the argument. The
and presents an alternative course of action that will
company may grant awards based on several other
prove beneficial for the general population in the long
parameters as well (efficiency, productivity, results
run.
and so on). Hence, assumption I is not implicit.
The correct answer is E. Assumption II: This assumption is vague and makes
moralistic judgements.
73. Both the arguments are weak because the bill may
or may not be related to administration officers or The correct answer is D.
politicians.
78. Assumption I: A comparison is being drawn
The correct answer is C.
between the time taken by two sisters to clean their
74. Argument I states that India is already performing well respective rooms. In order for the comparison to be
as a leader. The abundance of its resources, which the valid, the parameters have to be the same. In this
other world leaders lack, can ensure that India will go case, two elements are important – the size of the
on to become a world power. So, argument I is strong. rooms and the extent of dirtiness. Thus, assumption I
becomes implicit in the argument.
Argument II is also strong as it states certain issues
which might pull back India from achieving greater Assumption II: This assumption is not only beyond
heights in the world economy. the scope of the argument, but also contradicts the
information provided in the statement.
Hence, both the arguments are strong.
The correct answer is A.
The correct answer is C.

647

Book 1.indb 647 30/04/2019 4:49:54 PM


NMAT by GMAC™ Official Guide 2019

79. Assumption I: In this statement, it is apparent that 82. Argument construction:


Meena is a qualified teacher with extensive experience. The argument draws a correlation between unhealthy
However, we cannot put a value to the qualifier habits and the rampant growth of some diseases -
“extremely”. Such words always negate the assumption. like cancer and AIDS.
Assumption II: This assumption is implicit in the (A) Correct. Since it is mentioned that ‘…..have
argument. plagued mankind due to unhealthy habits’, it
The correct answer is B. gives an indication that diseases like cancer and
AIDS have some relation with a person’s lifestyle
80. Assumption I: The argument says that advisories and food habits; hence, if option (A) is true then
have been issued. This doesn’t mean that the unhealthy habits, among other things, may be
forecast made by the weather department is the cause of these diseases.
accurate. This is a precautionary step that is being (B) This is a vague statement. The argument is
suggested in order to preclude the possibility of any establishing a relation only between certain
damage. This assumption is too extreme. diseases and habits.
Assumption II: This also is a wrong assumption as (C) This is a contradiction of the main argument as it
farmers might be taking decisions without the help states that people are aware of the reasons that
of the weather department all the time. The forecast contribute to lifestyle based diseases.
made by the department might be just one factor
(D) This option presents another contributing factor
that influences their decision making. The weather
(technological advancements) for the spread of
department is also merely fulfilling its responsibility
diseases.
and not dictating the decisions of the farmer.
(E) This option actually negates the relation
The correct answer is D. established by the main argument.

81. Argument construction: The correct answer is A.

The passage is talking about the success of a 83. The premium has shot up for either one of the
particular loan scheme and uses the growing reasons. Hence, either assumption is enough to
prosperity amongst the farmers as a barometer of its arrive at the conclusion.
success. The correct answer is C.
(A) The use of the extreme word “only” renders this
option incorrect. 84. Assumption I: If a particular sop will provide an edge
(B) This option is also incorrect because of the use to this party, then it can easily be ascertained that
of “only”. this sop is not provided by any other party.
Assumption II: Also, schemes are introduced to
(C) The passage is talking about this particular loan
provide benefits to the masses, thereby increasing
scheme. From this, we cannot conclude that
the popularity of the one who introduces the scheme.
no such scheme existed earlier. Maybe, similar
Hence, both the assumptions are implicit.
schemes were there but their terms were not
favourable for farmers, or the farmers did not The correct answer is E.
know about them.
(D) This statement contradicts the information 85. Assumption I: The statement that “this model will
provided in the passage as the passage be the highest selling model in the sales history of
states that the loan scheme has become quite the company” is based on the data that the company
successful. This means that the farmers are has sold a record 1.3 million sets of the model in its
using the scheme to buy tools. first week of release. Hence, it is clear that no other
phone made by the company has achieved this feat.
(E) Correct. Agricultural tools are important for the
Assumption II: Just because this phone is selling
farmers to carry out a host of farm activities and
well, we cannot make generic assumptions about
prosperity for the farmers can only come when
customer preferences. It is possible that different
these activities are successfully carried out.
customers pay attention to different aspects of a
Therefore, the loan scheme and the prosperity of
phone. Thus, only the first assumption is implicit.
the farmers are directly linked.
The correct answer is A.
The correct Answer is E.

648

Book 1.indb 648 30/04/2019 4:49:54 PM


7.0  Logical Reasoning Practice

86. The police with the help of D-DAC tried to decipher the Assumption II is not implied since we do not know
data on the hard disc but were unable to do it. So the anything about the prediction of intelligence. Also it
police were unaware of the contents of the hard disc. has a lot of data that makes it wrong.
Assumption I: We don’t know what Troccoli is being The correct answer is A.
accused of: terrorist activities or any other crime. So
this statement cannot be assumed. 90. Assumption I is invalid because we cannot be sure
Assumption II: This also cannot be assumed of the effects this step might have. The statement is
because it is not necessary that the data is in code farfetched.
language. May be the team isn’t able to access The laptop company plans to raise the price of
it due to other reasons. So it is only a probable its laptops with immediate effect. The company
assumption. must have assumed that the demand of its laptops
Assumption III: There is nothing in the argument wouldn’t go down considerably because if that were
that suggests that D-DAC officials are from the police the case, it would not have taken the decision to
department. They might be an independent body of raise the price. So assumption II is valid.
technical experts or a separate entity altogether. The correct answer is B.
Assumption IV: This has to be a correct assumption
because if the hard-disk wouldn’t have been of any 91. Assumption I is not implicit since we have no idea
use, the police wouldn’t have made all these efforts how the bank will respond.
to decode it. Assumption II is not implicit because we do not know
whether Suresh has approached other people for this
The correct answer is D.
loan. He may have finalised taking money from the
87. (A) R akesh may or may not be a small child. We bank for various reasons; unavailability may be one
cannot infer this option. of the reasons among others.
(B) This is vague as we cannot say anything about The correct answer is D.
his popularity.
(C) This also is incorrect just like A. He may be a 92. Assumption I: Sachin played in the last match and
child, adult or teenager. We cannot be sure of it. that is how he was able to score a certain number of
(D) Correct. The requirement for silence and the runs in that match. So, assumption I is implicit.
need for Rakesh to stay away imply that he is Assumption II: The given statement doesn’t suggest
noisy. So this is the correct option. anything about the next match and hence,
(E) Just like A and C, this also cannot be inferred as assumption II is not implicit.
we cannot say anything about his age or relation The correct answer is A.
with the speaker.
The correct answer is D. 93. Assumption I: Assumption I is not implicit as the
statement talks of the best restaurants of the country
88. The situation defines the plight of rural areas where and whether these are world-class cannot be inferred.
the food grains being supplied and consumed are Assumption II: We cannot infer from the statement
impure. This is causing food poisoning. what are the features that make the mentioned
A correct assumption will be the one on which the restaurants the best in the country. So, assumption II
whole argument rests upon. is also not implicit.
I. Correct. It can certainly be assumed that impure
The correct answer is C.
grains are readily available in the rural areas.
Thus, we can also say that there are shops that 94. (A) T
 he statement merely mentions that the BJP
sell impure grains. members were creating disorder in the house.
II. Since there is no data available other than This does not mean that they like doing so.
that for rural areas, we cannot say that the (B) This is a generic and vague statement and
percentage of people consuming grains is higher cannot be inferred.
in rural areas.
(C) The statement states that the chairman tried to
The correct answer is A. ask the first listed question. This does not mean
that all the questions that are raised in the house
89. Assumption I is definitely implicit in the statement are listed. There might be different kinds of
since it is the most plausible reason for keeping questions.
children mentally stimulated.

649

Book 1.indb 649 30/04/2019 4:49:54 PM


NMAT by GMAC™ Official Guide 2019

(D) Correct. The fact that the chairman adjourned the does that. So it is only a probable inference and not
house when order could not be restored implies correct.
that he has the authority to do so. Inference II: This has to be a correct inference as
(E) Adjourning the house may be one way of the argument clearly talks about the state sponsored
restoring order. It does not follow that it is the food security programmes. This certainly means that
only way or that it is essential. the small nations invest a part of their revenue on food
security. Only then does the claim become possible.
The correct answer is D.
The correct answer is B.
95. (A) C
 orrect. A is the only option from which we can
infer that white tigers like to lie around—because 99. Inference I: When the author claims that the drop
they are known to be lazy. in gold prices is against the usual trend he certainly
(B) This statement conflicts with the main statement. means that the general belief was that the gold prices
If the national park makes tigers active, then the must have increased during this festive season like
white tigers should also be active. they normally do. So it is a correct inference.
(C) This statement again contradicts the main Inference II: This is the opposite of the claim made
statement, if white tigers hunt their food by the author. On the contrary, the author says that
aggressively, then they will not like to lie around. the increased demand for gold ornaments usually
results in an increased demand for gold bars. This
(D) This statement goes beyond the scope of the
clearly shows that both are interlinked. So II cannot
argument as no comment has been made about
be inferred.
other tigers.
(E) This statement goes beyond the scope of The correct answer is A.
the argument and has no bearing on the main
statement. 100. Inference I: This may look like a correct inference
but is incorrect because the argument says that
The correct answer is A. programmes for women are often based on topics like
recipes and household management but we cannot be
96. Inference I: The argument says that disasters such sure that women are interested only in such matters.
as thunderstorms and cyclones occur during a It is a very specific and narrow statement.
couple of weeks in hot season. This doesn’t mean
Inference II: The claim in the question is confined to
that the hot season lasts for two weeks. It may last
recipes and household management. It does not talk
longer. So this is an incorrect inference.
about sports. So we cannot infer II.
Inference II: Since the inference says ‘most’ it can
be inferred as the disasters occur only during two The correct answer is D.
weeks of the hot season.
101. Inference I: Since most of the runs were made by
The correct answer is B all-rounders, we cannot say that more than half of the
team consists of all-rounders. May be only 2 players
97. Inference I: The author makes a claim that the were all-rounders and they made these runs.
correlation between incomes and residence in slums
Inference II: This inference is out of the scope of the
is not perfect. To back it up he gives the reason that
argument as we cannot say if they were spinners
the people living there may be above the poverty
or not.
standards but they still choose to live in slums. So
we can say that despite being able to afford houses The correct answer is D.
elsewhere, some people still continue to live in
slums. 102. (A) The passage only talks of female passengers
Inference II: This is a very generic statement. We and not male passengers.
cannot say that poverty lines in general are flawed. (B) The passage only talks of female passengers
May be they are correct and the flaw is somewhere and not children.
else. So it is an incorrect inference. (C) Correct. This is clearly the answer because the
The correct answer is A. statement is a step taken by the government
in order to assist or encourage the female
98. Inference I: The argument is about small South passengers on Raksha Bandhan.
American nations. Any inference drawn about South (D) The passage only talks of female passengers
America in general will be wrong. This inference and not male passengers.

650

Book 1.indb 650 30/04/2019 4:49:54 PM


7.0  Logical Reasoning Practice

(E) This is incorrect because there is no suggestion (A) This statement says if I have to host the show,
about limiting the number of female passengers Hrithik is the co-host. This is wrong because
travelling on Raksha Bandhan. Hrithik may deny and subsequently I also will. So
it is not a necessity.
The correct answer is C.
(B) Correct. According to the statement, if I am
103. (A)   Stepping on the devil’s tail is a possibility when hosting the show, it is a necessary condition that
one both ignores ethics and deals with the the co host is Hrithik. Otherwise I wouldn’t host a
powerful. There is no direct connection suggested show.
between power and devils by the statement. (C) This is completely wrong as apart from Hrithik
(B) Stepping on the devil’s tail is a possibility when I won’t host the show with anyone else. This
one both ignores ethics and deals with the statement talks about Ranbir.
powerful. There is no direct connection suggested (D) This may be a possibility but not a certainty as we
between power and devils by the statement. cannot say anything about the friend.
(C) The statement also doesn’t suggest that dancing The correct answer is B.
or dealing with the powerful is necessary.
(D) Correct. The statement states that because it is 107. Statement I cannot be concluded in light of Fact 5 -
possible that one may end up getting involved what if none choose more than one hobby.
with the devil while dealing with power, one must Statement II cannot be concluded - even if we look at
be cautious, that is, careful. Hence, one needs to art, Fact 3 only states that ‘at least’ 50 have chosen
careful and wise while dealing with power. art - nowhere does it say, ‘at most’ how many have
(E) The statement also doesn’t suggest that the only chose art. Hence, II cannot be concluded. Same
way to dance with the devil is to ignore ethics. calculation applies for creative writing also.
The correct answer is D. In Statement III, even though the stats of only three
hobbies are provided, it cannot be concluded from the
104. Statement I can be concluded from Fact 2 and 3. given facts that only 3 hobbies are allowed in the school.
Those newspapers which have text will have both text Hence, E is correct because none of the statements
and graphics. can be concluded.
Statements II and III cannot be concluded as there is no The correct answer is E.
information about the efficacy of text in telling news.
108. The statement talks of a rule that needs to be
The correct answer is A.
followed by the cyber cafes but doesn’t talk of the
assistance that will be given by the police.
105. Refer to the following diagram:
(A) It is not clear from the given information as
− − to who will bear the costs associated with the
A B
compliance of the rule.
(B) It is not clear from the given information as to
+ who will bear the costs associated with the
C
compliance of the rule.
Statement I:  The facts do not state that B and A (C) This is incorrect as the motive behind the rule is
are the only siblings. So, statement I is incorrect. clearly mentioned.
Statement II:  This can be clearly seen from the (D) Correct. The implementation of the rule will
diagram. mean that certain details of everyone who uses

Statement III:  This cannot be concluded as there the internet at cybercafé, will mandatorily be
is no information in the given facts about the marital recorded. Hence, anybody who doesn’t carry any
status of any of the given people. proof of identity will not be able to use internet at
cyber cafes.
The correct answer is B. (E) The given information does suggest that the
106. According to the statement, the only and necessary collected information can be checked by the
condition needed for the person to host the show police but there is no confirmation if it will be
is that Hrithik Roshan is a co-host. That means he submitted monthly to the respective police
wouldn’t host the show in any other condition. Also if he stations. So, this is incorrect.
is hosting a show, it has to be with Hrithik Roshan only. The correct answer is D.

651

Book 1.indb 651 30/04/2019 4:49:54 PM


NMAT by GMAC™ Official Guide 2019

109. Conclusion I: Though it is mentioned that the Conclusion II: We have no information regarding the
Aluminum manufactured in India is of a much better reasons behind the better quality of the aluminum. It
quality than the aluminum manufactured in other might have something to do with the manufacturing
countries across the world, there is no mention of process or could be due to other factors such as the
the price or profitability. There might be other factors superior ore quality and so on. Thus, Conclusion II is
that influence the decision regarding the sale of the also not valid.
material. Thus, Conclusion I is not valid.
The correct answer is D.

652

Book 1.indb 652 30/04/2019 4:49:54 PM


7.0  Logical Reasoning Practice

2. Analytical Reasoning 7. N owns Scorpio.


The correct answer is B.
1. The final arrangement will look like this:
CFGDHE 8. P is second to the right of Q.
All the questions can now be easily answered. The correct answer is C.
The correct answer is B.
9. Among the options, combination given in option C is
correct.
2. C and E are standing at the extreme ends of the line.
The correct answer is C
The correct answer is A.
10. We have to arrange eight people in a circular
3. D is standing second to the right of F.
arrangement. It is better to start by fixing the
The correct answer is B. position of Caitlin who likes spinach. Now the position
of the person who likes apple can be fixed. Similarly,
4. Except GE all the other pairs have a gap of one place we can fill the other positions.
between them. The final arrangement will look like this:
The correct answer is B.
Sarah
5. From the given information, we can locate the five (Broccoli) Amy
stations as following: Amber
(Dandelion)
(Cherry)

P S Q R T
Jessica Emma
Hence, station P is located to the immediate left of S. (Beetroot) (Apple)
The correct answer is A.
Zoe
Caitlin
6. We need to arrange 8 people in a circular (Tomato)
(Spinach)
arrangement (facing away the Centre). We should Megan
start by fixing the position of Y (as a lot of the given (Eggplant)
information is about Y or related to Y).
The final arrangement will look as follows:
All the questions can now be easily answered.
N
(SCORPIO) The correct answer is C.
T M
(VW) (CRETA)
11. Among the options, statement given in option D must
be true.

L Q The correct answer is D.


(ARIEL) (PUNTO)
12. The person who likes broccoli is fifth to the left of
Zoe.
Y R The correct answer is C.
(LYCAN) (LAMBRETTA)
P
13. Zoe likes tomato.
(GARMIN)
The correct answer is D.
All the questions can now be easily answered.
The correct answer is D.

653

Book 1.indb 653 30/04/2019 4:49:55 PM


NMAT by GMAC™ Official Guide 2019

14. The initial arrangement before interchange will look Hence, from the given options P, Q, R, S and T
as follows: cannot be the correct seating arrangement of the
five persons in either the clockwise direction or the
S counter-clockwise direction.
K D The correct answer is A.

19. The final arrangement will look like this:


Possibility 1
1 (dog) 2 (K-cat) 3 (J – dog)
4 (M-cat) 5 (dog) 6 (H - dog)
M P Possibility 2
1 (dog) 2 (M - cat/dog) 3 (M - cat/dog)
R
4 (K-cat) 5 (J-dog) 6 (dog – H)
Thus, the position after interchange will be as follows: All the questions can now be easily answered.
S The correct answer is E.
D K
20. G is assigned to Cage 1.
The correct answer is C.

21. Option C must be true.


The correct answer is C.

R P 22. Option E must be true.

M The correct answer is E.

All the questions can now be easily answered. 23. The final arrangement will look like this:
The correct answer is B. Floor Person Car colour

15. All other pairs are arranged in an anti-clockwise 9 Emmanuel Blue


direction. 8 Shane Black
The correct answer is D. 7 Ethan Green
6 Richard Yellow
16. M is sitting opposite to S.
5 Jason Red
The correct answer is B. 4 Andre Orange
3 Joshua Pink
17. P is sitting to the immediate left of K.
2 Patrick White
The correct answer is A.
1 Luke Grey
18. From the given information, we can draw the
The correct answer is E.
following seating arrangement:

Q P 24. Three persons are staying between Jason and


Emmanuel.
The correct answer is A.
S/ T T/S
25. Patrick stays on the floor immediately below Joshua's
R floor.
The correct answer is C.

654

Book 1.indb 654 30/04/2019 4:49:56 PM


7.0  Logical Reasoning Practice

26. Richard owns the yellow coloured car. to 4 pm. Cricket also cannot happen on Monday. So,
it must happen on Tuesday, Wednesday and Friday
The correct answer is D.
from 4:30 to 6 pm.
27. We have to arrange 7 people on the basis of their 1–2 2–3 3–4 4–5 pm 5–6 6–7 pm 7–8 pm
pm pm pm pm
course, name and musical instrument and so we
need a table with 3 columns and 7 rows. We can fill Yoga Yoga Yoga Yoga Swimming Swimming

data for M and R. Now, P must be doing B.Sc as he Music Music (4:30 pm) Cricket Swimming Swimming
is doing the same course as R. Similarly, T is doing Cricket

B.A. So, J, V and W are doing B.Com. Similarly, other Music Music (4:30 pm) Cricket Swimming Swimming
Cricket
points can also be analysed and filled in the table.
Swimming Swimming
The final arrangement will look as follows:
Music Music (4:30 pm) Cricket Swimming Swimming
Cricket
Person Course Instrument
J B.Com Guitar From the above table, we can easily see that Yoga
V B.Com Flute can happen on Thursday as well.
W B.Com Saxophone If yoga doesn’t happen on Monday or Thursday, then
it will happen on Tuesday or Wednesday or Friday. In
M B.A. Banjo
that case, Music and Cricket will happen on Monday
T B.A. Sitar and Thursday with their timings clashing. So, this
R B.Sc. Tabla situation is not possible.
P B.Sc. Violin The correct answer is D.
33.
The questions can now be easily answered.
The correct answer is A. Scenario 1
Day Swimming Music Cricket Yoga
28. Sexophone is the favourite musical instrument of M.
Mon 1 to 3 4 to 7
The correct answer is D. Tue 1 to 3 4.30 to 6 6 to 8
Wed 1 to 3 4.30 to 6 6 to 8
29. Tabla and violin are the favourite musical instruments
of those who are pursuing B.Sc. Thur 1 to 3 4 to 7
The correct answer is C. Fri 1 to 3 4.30 to 6 6 to 8

30. Among the options, combination given in option E is Scenario 2


correct. Day Swimming Music Cricket Yoga
The correct answer is E. Mon 6 to 8 – – 1 to 3 and 3 to 5
Tue 6 to 8 2 to 4 4.30 to 6
31. As the athlete who is 5 places below him is at 18th
Wed 6 to 8 2 to 4 4.30 to 6
from the top, so his position must be 13th from top
and the athlete who is 3 places above him is 17th Thur 6 to 8 – – 1 to 3 and 3 to 5
from bottom. So, he must be 14th from bottom. Fri 6 to 8 2 to 4 4.30 to 6
Hence, there are 12 above him and 13 below to him
and so there must be a total 12 + 13 + 1 = 26. Possible 2 hour yoga sessions in scenario 1 = Mon
The correct answer is B. not possible, Tue 6–8, Wed 6–8, Thurs not possible,
Fri 6–8.
32. If Manish goes for 4 hours session on Monday, then In scenario 2, the 4–7 music session is not possible
Music can’t happen on Monday. So, Music must because it conflicts with swimming, but both cricket
happen on Tuesday, Wednesday and Friday from 2 options are possible. If cricket is taken 3–5, the

655

Book 1.indb 655 30/04/2019 4:49:56 PM


NMAT by GMAC™ Official Guide 2019

possible yoga sessions are Mon 1–3, Tue 4–6, Wed 38. The answer can be observed from the chart we made
4–6, Thurs 1–3, Fri 4–6. If cricket is taken 4.30–6, earlier:
the possible yoga sessions are Mon 1–3, 2–4, 3–5, Mittal – July
4–6, Tue not possible, Wed not possible, Thurs
Murti – October/January
1–3, 2–4, 3–5, 4–6 and Friday not possible. So the
total number of yoga sessions available across all Aiyyar – May
scenarios is 16. Ambani – December
The correct answer is D. Parekh – March
The correct answer is A.
34. Please refer to the two possible scenarios described
in the previous question. In the first scenario, there is 39. The answer can be observed from the chart we made
a possibility that Manish can attend a yoga session earlier:
on Tuesday whereas in the second scenario, there is
Mittal – July
no free time slot for yoga on Tuesday from 6 to 8.
Murti – October/January
The correct answer is A.
Aiyyar – May
35. If Manish attends the swimming class between 1–3 Ambani – December
pm every day, then he has to necessarily attend Parekh – March
the Music class on Monday and Thursday from 4–7
The correct answer is D.
pm. This means he has to attend cricket classes on
Tuesday, Wednesday and Friday from 4.30–6 pm.
Thus, he can attend the yoga classes on any two General Solution for Questions 40–43:  The only possible
days among Tuesday, Wednesday and Friday from seating arrangement is:
6–8 pm. S is 2nd to the right of P. So, S is either at 3rd, 4th or 5th
place. If S is at 4th place, then P will be at 2nd place, in which
The correct answer is E. case T would have to be neighbor of P or S, which is not
allowed. If S is at 4th place, then P will be at 3rd place, and
General Solution for Questions 36–39:  Following the
T also has to be at 3rd place because there is one person
above instructions, the final chart will look like this:
between S and T. Therefore, the only place for P and S is 1st
Mittal – July and 3rd place.
Murti – October/January Since B is at its leftmost place, B will be facing T. D cannot
Aiyyar – May be with B and D has to face R, so the only place for D is 2nd
Ambani – December position and so R is also at corresponding 2nd place. Since
C and E are together, they will come between B and D and A
Parekh – March
will be facing P, at 1st position. Finally, since E does not face
36. The answer can be observed from the chart we made S, E will face Q and so C will face S.
earlier:
On the basis of above, the following arrangement can be
Mittal – July made:
Murti – October/January
Aiyyar – May Facing South A D C E B
Ambani – December Facing North P R S Q T
Parekh – March
40. From the above table, the persons are P and T.
The correct answer is E.
The correct answer is C.
37. The answer can be observed from the chart we made
earlier: 41. E is facing Q.
Mittal – July The correct answer is E.
Murti – October/January 42. C is sitting at the centre of the row.
Aiyyar – May
The correct answer is C.
Ambani – December
43. R is third to the left of T.
Parekh – March
The correct answer is B.
The correct answer is E.

656

Book 1.indb 656 30/04/2019 4:49:56 PM


7.0  Logical Reasoning Practice

General Solution for Questions 44–46: In questions Facing outside Facing the Centre
where some are facing the Centre and some are facing
outside, it is better to draw a table to keep track of T S
all those facing outside and those facing centre. Make U R
preliminary entries by scanning through the data given. P S
The following is the preliminary table.
V W
Facing outside Facing the Centre
P
T S
U W Q

Beginning with the other data, we have R


•  P is opposite to T. V

•  S is adjacent to R. S
•  Q is second to the right of S.
T
•  W is second to the right of U.
We get the following possible arrangements 44. As W is facing center, so Q is second to the left of W.
P The correct answer is C.
Q W
45. From both the sides, there are three people between
them.
R
The correct answer is D.
S U 46. P, T, U and V are facing outside.
T R T The correct answer is E.
If P is facing the centre, we have General Solution for Questions 47–50: 
• From statement I and II, Bimal cannot live at floor number
P
1. Because Geeta lives below the floor number of Bimal.
U W
• From the statement “the sum of the floor numbers on
which Aman and Bimal stay is 8”.
   Aman + Bimal = 8

If Bimal’s floor number is 3, then the Aman’s floor number is


T 5. Then Disha lives on floor number 4 from statement I. Last
statement says that full stay just above the floor number of
This violates the fact that both neighbours of disha which is not possible.
U face outside. So P faces outside. Now the If Bimal’s floor number is 5, then Aman’s floor number is 3.
arrangement is: We can get the following arrangement.

P
Floor Number Person
W Q
7 Fulla
6 Disha
V R
5 Bimal
U S 4 Chandu
T 3 Aman
2 Geeta
The updated table would be:
1 Esha

657

Book 1.indb 657 30/04/2019 4:49:57 PM


NMAT by GMAC™ Official Guide 2019

47. Looking the above table, we get that Esha stays on 57. The sum of heights of tree F and tree G cannot be
floor numbered 1 and Chandu stays on floor numbered determined because the exact heights of trees F and
4. Therefore, the sum of their floor numbers is 5. G are not known.
The correct answer is B. The correct answer is D.
48. Looking at the above table, we find that there are 58. Maximum possible height of tree A is 9 feet.
three persons who stay between Geeta and Disha. And, least possible height of tree F is 2 feet.
The correct answer is C. Required difference = (9 – 2) feet = 7 feet
49. Esha stays on the floor numbered 1. Therefore, the The correct answer is C.
square of 1 is 1.
General Solution for Questions 59–62:  This is a This is
The correct answer is A.
a case of tabular arrangement where we have 4 columns—
50. Looking at the above table, we find that Aman stays Name, City, Area (of job) and Designation.
just above the floor on which Geeta stays. From 1, we know that Gaurav’s designation is Director, the
The correct answer is B. city is Chennai and the Area is Marketing.
From 2, we know that Pratima is in Hyderabad (as she is
General Solution for Questions 51–54: 
neither at the Bangalore or the Chennai branch), her area is
E>A>G Accounts and her designation is Manager.
E>B>C>F From 3, we don’t know the name, but we know there is a
Moreover, C got 70%, and so B got 80% and F got person from given 5 who is in Bangalore, his/her designation
60%. Also, E got 90%. is Assistant Manager and who works in the Personnel
Using the above information, we get: department.
E (90%) > B (80%) > A > C (70%) > G > F (60%) > D From 4, we know that Sushmita is in Bangalore, her
designation is Supervisor and Devanshu is in Chennai.
51. D scored the lowest marks.
Putting all this information in table, we get:
The correct answer is B.
Name City (of Area Designation
52. The marks of D is between 50% and 60% because branch) (department)
the lowest marks possible is 50%.
Gaurav Chennai Marketing Director
The correct answer is C.
Pratima Hyderabad Accounts Manager
53. The marks scored by G lies between 60% and 70%. Bangalore Personnel Assistant
The correct answer is B. dept. Manager

54. B scored the second highest marks. Sushmita Bangalore Supervisor


Devanshu Chennai
The correct answer is B.
General Solution for Questions 55–58: Now, we can complete the table. As we know, there is just
From the given information, the correct order of one person left whose name is Kuldeep and so he must be
trees in the garden with increasing heights from left the person who is in Bangalore. His designation is Assistant
to right is as following: Manager and he works in the Personnel department.
E F D B C A G There are two Assistant Managers, one Manager, one
Director and one Supervisor. So, Devanshu must be an
Now, all the questions can be easily answered. Assistant Manager.
55. There are two trees between E and B. Similarly, there As we know, among the four who are in Chennai or
are two trees between A and D. Bangalore, two are in Marketing, one in the Personnel
department and one is in Accounts. We don’t know for
The correct answer is A.
sure about Sushmita and Devanshu’s area, but we know
56. Difference between G and D that one of them works for marketing and the other for
= 4 feet = 4 × 12 inches = 48 inches accounts.
Now the solution table can be completed as follows:
The correct answer is C.

658

Book 1.indb 658 30/04/2019 4:49:57 PM


7.0  Logical Reasoning Practice

Name City (of Area Designation Lake Range State


branch) (department) Shyam Chilka
Gaurav Chennai Marketing Director Mansarovar Aravalli MP
Pratima Hyderabad Accounts Manager Sumit Pushkar Vindhyachal AP
Kuldeep Bangalore Personnel dept. Assistant Ram Rajasthan
Manager
Sushmita Bangalore Accounts/ Supervisor Therefore, Shyam is in Uttaranchal. Ram is on lake
marketing Nakuchiatal. We cannot find out the ‘Range’ for Ram
Devanshu Chennai Accounts/ Assistant and Shyam.
marketing Manager So, our final table is drawn below.

Lake Range State


59. Kuldeep is in Personnel Department.
Shyam Chilka Himalayas/ Uttaranchal
The correct answer is B. Deccan
60. Devanshu is working as Assistant Manager in Amit Mansarovar Aravalli MP
Marketing. Sumit Pushkar Vindhyachal AP
The correct answer is A. Ram Nakuchiatal Deccan/ Rajasthan
61. Kuldeep is the Assistant Manager in Personnel. Himalayas

The correct answer is B.


66. We can see from the table that Shyam is trekking in
62. Among the given options, the combination given in Uttaranchal.
option E is the correct.
The correct answer is D.
The correct answer is E.
63. From option (A) 76, 85, 79, 68, 78 = NOISS 67. Only statements (i) and (iii) are correct.
From option (B) 40, 66, 87, 79, 96 = NOIIS The correct answer is D.
From option (C) 10, 24, 30, 44, 01 = NNESE
68. We can see from the table that Sumit is trekking in
From option (A) 40, 85, 79, 96, 77 = NOISE Pushkar.
The correct answer is D. The correct answer is E.
64. From option (A) 01, 23, 20, 41 = FFAT 69. From the given information, we cannot find out who
From option (B) 43, 40, 78, 98 = FASU has gone to Himalayas or Deccan.
From option (C) 23, 20, 69, 42 = FASL
The correct answer is B.
From option (D) 01, 34, 69, 41 = FAST
The correct answer is D. 70. From the information provided, the following table
can be deduced:
65. From option (A) 33, 42, 59, 97 = LLOO
From option (B) 13, 68, 20, 31 = LEAN Large Medium Small Tiny
From option (C) 42, 97, 20, 34 = LOAA Colours Rust Black Red White
From option (D) 00, 85, 88, 99 =LOAA Made of Brass Silver Wooden Plastic
The correct answer is B. Content Odd Stuff Earrings Buttons Needles

General Solution for Questions 66–69: From the above table, the colour of the box that
stored earrings was black.
F rom the information given, the following table can be
immediately developed: The correct answer is C.

659

Book 1.indb 659 30/04/2019 4:49:57 PM


NMAT by GMAC™ Official Guide 2019

71. From the table, the plastic box stored needles. 80. Between the first two and the last two pairs of letters,
there is only one missing letter from the English
The correct answer is B.
alphabet series. And, between the two pairs, there
is a difference of 9 letters, except in option E where
72. From the table, the size of the rust coloured box is
there is a difference of 8 letters.
large.
The correct answer is E.
The correct answer is B.
81. Horrendous means extremely unpleasant. Atrocious
73. From the table, the combination provided in option A means extremely wicked. Detest indicates extreme
is correct. dislike. Dreadful means extremely bad. These are all
The correct answer is A. negative words. However, wonderful that means
delightful, inspiring pleasure or marvellous, has a
74. There are two possibilities: Anubhav is above Raman positive connotation and usage. It is the odd one out.
or below Raman. If Anubhav is above Raman, then The correct answer is B.
rank of Anubhav from top is 13, and since he is at
the exact centre, total number of students = 25. 82. This is an analogy based question. The idea here is to
Also, Akash is 14th from below, which means 12th find the option which has a different relationship than
from top. So, Akash is 1 rank above Anubhav. This is the others.
not available in the options.
(A) Prodigy means ‘Extra-ordinary’ which is the
If Anubhav is below Raman, then rank of Anubhav is opposite of ‘Ordinary’. Thus, they are antonyms.
23rd from top, and so there are 45 students. In that This pair is the odd one out.
case, rank of Akash from top = 45 + 1 – 14 = 32,
(B) Smile and laugh are synonyms. Incorrect.
who is 9th place below Anubhav.
(C) Great and large both indicate an extent or size
The correct answer is C. considerably above average. Incorrect
(D) Draconian means harsh which is a synonym of
75. ‘Typhoon’ refers to a tropical storm. ‘Hurricane’ refers
‘brutal’. Incorrect
to a storm with a violent wind, while ‘Cyclone’ is also
a type of storm. Hence, ‘Volcano’ is the odd one out. (E) Laconic means brief, it is also a synonym of
‘concise’. Incorrect
The correct answer is B.
The correct answer is A.
76. All are multiples of 17, except 47.
83. Ecology, Geology, Osteopathy and Obstetrics are all
The correct answer is E. branches of Science whereas Numismatics is the
study of coins. Hence, it is the odd one out. Ecology
77. Dodo is the only extinct bird in the group. is the branch of science that deals with the relations
The correct answer is C. of organisms to one another and to their physical
surroundings. Geology is the science which deals with
78. Alternate letters are one removed from each other the physical structure and substance of the earth, its
- ex. C and E have D in between, K and I have J in history, and the processes which act on it. Osteopathy
between (starting from the back) means ‘a system of complementary medicine
involving the treatment of medical disorders through
In RUTW, this is so but starting from the front. R(S)T the manipulation and massage of the skeleton and
and U(v)W rather than TWRU. musculature’. Obstetrics means the branch of medicine
The correct answer is E. and surgery concerned with childbirth and midwifery.
The correct answer is A.
79. Horse, camel, bullock and donkey are all used by
humans as modes of transport. A cat is not used by
84. Mortal, lethal, deadly and murderous are synonymous
humans for any purpose. Hence, it is the odd one out.
in meaning. They refer to anything that is liable to
The correct answer is A. perish or cause immense damage. Eternal is an

660

Book 1.indb 660 30/04/2019 4:49:57 PM


7.0  Logical Reasoning Practice

antonym that means undying or perpetual, something 89.


that lasts forever.
The correct answer is D. I. Parks Ground
sky

wall
85. Except brain, all other organs are situated in the
abdomen. So, the odd one is brain. or

The correct answer is D.

86. Except Bag, all other are items of clothing. Thus, the
odd one out is (B). II. sky
Parks Ground
The correct answer is B. wall

Wall
87. Except Maine Coon, all other are breeds of dogs.
Maine Coon is one of the largest domesticated breed
of cats. Thus, the odd one out is (E). As there is nothing said clearly about wall and park,
so, either “some walls are park” or “No wall is a
The correct answer is E.
park” is possible but false individually. So, either I &
III follows:
88.
airplanes The correct answer is A.

90. The possible Venn-diagram is:


cars trucks trains

airplanes Gifts Box cream


Or cars trucks trains
Grape

From the above venn diagram it is clear that the


conclusion II "Some grapes are box" is true. As
airplanes
there is nothing said about cream and gift in the
cars trucks trains statements, so either “Some creams are gift” or “No
cream is gift” is possible but false individually. So,
either conclusion I or III and II follows.
The correct answer is A.
Check options.
Option (A): Some cars are trucks and all trucks are 91. The possible Venn-diagram is:
airplanes. So, some cars are airplanes. But it is not
necessary that all cars are airplanes. So, (A) is not
necessarily true.
Option (B): Trucks and trains may or may not have
anything common. So, “all trucks are trains” is not Print Wrong Right
necessarily correct.
Option (C): Some airplanes are trains also means Deform
some trains are airplanes, which includes the
possibility that all trains are airplanes. So, (C) is a
So, it is clear from diagram that conclusion I is true.
correct answer.
But some print may be right or may not be roght. So,
Option (D): “No truck is a train” is a possibility. But, it conclusion II doesn’t follow clearly from the above
is not necessarily true. Venn diagram. Conclusion III follows.
The correct answer is C.

661

Book 1.indb 661 30/04/2019 4:49:58 PM


NMAT by GMAC™ Official Guide 2019

But conclusion IV doesn’t follow as “All” strongly 94. A possible Venn-diagram is as follow:
shows “Completely without failure but as “Some
deform are wrong” is true which is showing
orange
uncertainty of the extent of deform being wrong.
The correct Answer is C.

ry
er
grapes apple

wb
ra
92. A possible Venn-diagram is.

st

All strawberries may be Apples, but it is not


necessarily true. Hence conclusion I does not follow.
Ships Since it is given that “some grapes are strawberries”,
“some grapes are not strawberries” is not
Goat necessarily true, as it is quite possible that all grapes
Cows are strawberries.
The correct Answer is D.

95. A possible Venn-diagram is:


horse or Ships
horse Goat
Goat

Cows Dog
Sheep
Cat
As No horse is goat, it is clear that there may or may
not be some relation between cow and horse. So
either conclusion I or conclusion II follows.
The correct answer is C. The relation between cat, sheep and goat is not clear
from the above Venn-diagram. So, ‘No cat is a sheep’
93. The possible Venn diagrams are. and ‘Some cats are goats’ does not necessarily
follow. Hence neither I nor II follows.
The correct Answer is D
stone rock ring
96. A possible Venn-diagram is:

desk hall
stone rock

or
ring desk hall room
or

As, there is nothing is said about the relation of stone


and ring , so either “Some stones are rings “ or “No room
stone is a ring “ but individually both are false. Again
from the above Venn diagrams only conclusion II
follows. i.e. Some stones are rocks.
As ‘some’ doesn’t show the extent of certainty so
The correct answer is B. conclusion I is wrong.

662

Book 1.indb 662 30/04/2019 4:49:59 PM


7.0  Logical Reasoning Practice

There is nothing said about hall and room so, 104. Brother
conclusion II is wrong. + + −

The correct Answer is D.

97. Refer to the following Venn diagram:


+ Mohan Sujata

Sujata’s mothers’ husband is his/her father. If the


father’s brother is maternal uncle of Mohan, then the
father himself is also maternal uncle of Mohan.
Doctors Musicians Poets Therefore, Mohan is cousin of Sujata.
The correct answer is C.

105. Option A suggests that A and C are brothers and C is


the father of B. Hence, A is the paternal uncle of B.
It is clear from the diagram that conclusion 1 is true. The correct answer is A.
Conclusions 2, 3 and 4 do not follow.
106. Since I is the brother of A,Z and T are a couple and
The correct answer is D. T is the mother. Now, since T is also Q’s sister, Q
98. While we know that P, Q, B and E are siblings, we is either brother-in law or Sister-in-law and since the
don’t know the sex of E and Q. Thus, D is correct. former is not among the options given, E is The
correct answer.
The correct answer is D.
Alternatively,
99. Rajeev and Rishi are brothers. Karuna is Rishi’s Draw the family diagram.
mother and so she is also Rajeev’s mother. As Ajay
is Karuna’s brother, he is also the maternal uncle of
Rajeev. Therefore, Rajeev is Ajay’s nephew. Hence, Q
T Z
option C is correct.
The correct answer is C.
100. Akshay’s father’s son will either be Akshay himself or D E I A
Akshay’s brother. Thus, Rahul will be either Akshay’s
son or the son of Akshay’s brother.
Q could be brother-in-law or sister-in-law of Z. The
The correct answer is C. only option possible is (E).

101. Joginder’s father’s sister is Joginder’s aunt, which The correct answer is E.
makes the girl Joginder’s cousin.
107. From statement I, we do not get any relation between
The correct answer is B. B and D.
102. The maternal grandmother has to be Sita’s mother, Thus, statement I alone is not sufficient.
which makes the person Sita’s brother. Thus, he will From statement II, we again do not get any relation
be Sita’s husband’s brother-in-law. between B and D.
The correct answer is B. Thus, statement II alone is also not sufficient.

103. The two students have to be the youngest members, On combining the two statements, we get the
that is, Rahul and Avinash. Sati and Ratul then following family tree:
become their grandparents and Savitri and Arjun − +
A B
become their parents. Thus, Ratul is Rahul’s
grandfather.
+
The correct answer is E. D C

663

Book 1.indb 663 30/04/2019 4:50:00 PM


NMAT by GMAC™ Official Guide 2019

From the above family tree, it is clear that B is the 113. H = 6


father of D. I=1
Thus, both statements I and II together are sufficient V=3
to answer the question. E=7
The correct answer is D. B=9
L=2
108. From the information given, the following family tree T=4
can be drawn:
Thus, LIVE = 2137
Gopal’s parents
− + The correct answer is E.

114. 456 = flavoured orange juice (1)


678 = mixed fruit juice (2)
− + 389 = very tasty fruit (3)
Gayatri Ganesh Geeta Gopal
From statements (2) and (3), we get
8 = fruit
From the above family tree, it is clear that Gayatri is From statements (1) and (2), we get
Gopal’s sister. 6 = juice
The correct answer is B. Thus, from statement (2), we get
7 = mixed
109. Author’s son and Titli could either be siblings or
cousins. Since there is no option that suggests The correct answer is D.
siblings, the correct option is cousin. 115. Writing the place values of each of the letters of
The correct answer is E. HANDBAG, we have
H A N D B A G
110. Shekhar’s paternal grandfather’s only daughter-in-
law is Shekhar’s mother. The relationship between
Shekhar’s mother and his father is of wife and 8 1 14 4 2 1 7
husband. So, the relationship between Kala and
Shekhar is of wife and husband. Thus, Kala is The logic followed is as following
Shekhar’s wife.
8 + (1 × 14) + (4 × 2) + (1 × 7)
The correct answer is B. = 8 + 14 + 8 + 7 = 37

111. The code is created in this manner: write DONKEY in Similarly, writing the place values of each of the
reverse and move back each letter by one step. letters of PURSE, we have
Y–1=X P U R S E
E–1=D
K–1=J
16 21 18 19 5
N–1=M
O – 1= N Again, the logic followed is same:
D–1=C
16 + (21 × 18) + (19 × 5)
Thus, LION will be written as MNHK.
= 16 + 378 + 95 = 489
The correct answer is A.
Now, writing the place values of each of the letters of
112. Notice that this is a jumbled type code, since all WALLET, we have
the letters in the original word and the code are the W A L L E T
same. Thus, you simply need to make the same
changes to the placement of the letters in the given
word as have been made to TRIFLE.
23 1 12 12 5 20
The correct answer is C.

664

Book 1.indb 664 30/04/2019 4:50:01 PM


7.0  Logical Reasoning Practice

Required answer 122. The pattern followed is as below:


= 23 + (1 × 12) + (12 × 5) + 20 S Y L L A B U S

= 23 + 12 + 60 = 20 = 115 +1 −1 +1 −1 +1 −1 +1 −1

The correct answer is B. T X M K B A V R

Similarly, we have
116. The pattern followed is as following:
O P E R A T I O N
Y O U
+1 −1 +1 −1 +1 −1 +1 −1 +1
+5 +6 +7
P O F Q B S J N O

D U B
The correct answer is D.
Similarly,
O U R
123. The pattern followed is as below:
+5 +6 +7
C R I C K E T

T A Y +2 −2 +2 −2 +2 −2 +2

The correct answer is D. E P K A M C V

Similarly, we have
117. This is an alternate type code. The letters at odd
positions move ahead by one step whereas the H O C K E Y
letters in even positions move back by one step. +2 −2 +2 −2 +2 −2
The correct answer is B.
J M E I G W
118. The first letter moves ahead by 1 place, the second The correct answer is C.
letter by 2 places, the third letter by 3 places and so on.
The correct answer is C. 124. Given that P is written as C, O as L, A as W, R as K,
E as X, L as Q, B as N, and T as H, then PORTABLE
119. The letters at odd positions move ahead by 2 places. can be written as following:
The letters at even positions move back by 3 places. P O R T A B L E
The correct answer is E. ↓ ↓ ↓ ↓ ↓ ↓ ↓ ↓
C L K H W N Q X
120. There are only 3 values where N × N gives N in unit
place The correct answer is A.
1. If N is 1, but it is not possible as then we must
have got M 2 N as product 125. The pattern followed is as below:
2. If N is 5, then we get 5 in unit place and 2 as G A M B L E
carry over for 5 × 5 = 25. But, then we must +1 +2 +3 +4 +5 +6
have got 2 instead of 5 in ten’s digit at product
3. N = 6, then N × N = 36, gives 3 as carry over H C P F Q K
and we have 6 × 2 + 3 (carry over) = 15. So, we
Similarly, we have
have 5 at ten’s digit and 1 is carry over. Now, we
have 6 × M + 1 = 25. It is possible for M = 4 P U N T E R
+1 +2 +3 +4 +5 +6
The correct answer is C.
Q W Q X J X
121. Each letter of the original word is replaced by the letter
that comes immediately after it in the English alphabet. The correct answer is B.
Thus, C becomes D, H becomes I, and so on.
By the same logic, the code for TABLE should be 126. In this new number system, we use 5 = 10, 6 = 11, 7
UBCMF. = 12, 8 = 13
So, 9 = 14, 10 = 20, 11 = 21, 12 = 22, 13 = 23, 14
The correct answer is A.
= 24, 15 = 30, 16 = 31, 17 = 32 and 18 = 33
665

Book 1.indb 665 30/04/2019 4:50:03 PM


NMAT by GMAC™ Official Guide 2019

So, 18 is represented as ££ 130. The positional values of the letters in English alphabet
are given below:
The correct answer is D.
A1 B2 C3 D4 E5 F6 G7 H8 I9 J10 K11 L12 M13
127. The code is such that, A, B, C…Z are represented by Z26 Y25 X24 W23 V22 U21 T20 S19 R18 Q17 P16 O15 N14
numbers. 1,2,3…26.
When we add the values of each letter, we get:
The code has 3 numbers—first number is obtained
by adding number of first 3 alphabets, and third term CETKINGMUMBAI =3 + 5 + 20 + 11 + 9 + 14
is obtained by adding numbers of last 3 alphabets +7+13 +21 + 13 + 2 + 1 + 9 =128
and second term is obtained by squaring the number IIMAHMEDABAD = 9+ 9 + 13 + 1 + 8 + 13 + 5 +
of middle alphabet. 4+ 1 + 2 + 1 + 4 =70
By using this logic we get STRANGE as (S + T + R)
The correct answer is C.
(Square of 1) (N + G + E) = 57 (12) 26 = 57126
The correct answer is C. 131. The codes that have been used for RELATE are:
(A) Each letter becomes the 2nd letter to its right -
128. ‘Some More Is Good’ is coded as ‘ACEF’…………….i so R becomes T, E become G and so on to make
‘More Sugar Not Good’ is coded as ‘CEBD’………….ii TGNCVG.

‘Good More Is Bad’ is coded as ‘FCEI’ ……………..iii (B) Keeping the extreme letters intact, the letters are
swapped such that the 2nd letter goes into the
‘Bad Is More Sure’ is coded as ‘IFCZ’ ………………iv place of the 2nd last and vice versa and the third
From i and ii, the codes for More and Good = letter goes into the place for the third last and
between C and E vice versa to make RTALEE.
From i and iii, the code for Is = F (C) Each letter becomes the letter to its immediate
From ii and iv, the code for More = C right - so R becomes S, E becomes F and so on
to make SFMBUF.
Thus, Good = E
(D) The consonants are kept in place and the vowels
Some = A
are moved one place to the right.
Bad = I
As per the coding used in B, RVITALEE fits for
Sure = Z RELATIVE.
Sugar/Not = between B/D
The correct answer is D.
Therefore,
More Bad Is Not Good Sugar = CIFEBD (in any order) 132. Old Number:
The correct answer is B. XYZ
110
129. Putting all this information in the table, we get: New number (according to the rules):
1 Given timetable below represent ku na pa cu XYZ
111
2 Presentation represent according pa su lu cu
timetable The correct answer is E.
3 Presentation below cutoff su na ja 133. If only Y is 1, then X and Z are 0s. The last rule will
apply. All 1s will change to 0 and all 0s to 1.
From statements 1 and 3, we get: The correct answer is D.
‘below’ is coded as ‘na’.
134. Use options. In case of option A, since all the digits
Similarly, from statements 2 and 3, we get: are 1’s, then new number won’t have all digits 1.
‘Presentation’ is coded as ‘su’. In case of option B, X = 1, Y = 1, Z = 0. As per
Now, from statements 1 and 2, we get: condition 1, the new number will be 111
‘represent timetable’ is coded as ‘pa cu’. The correct answer is B.
Therefore, the remaining word ‘according’ in
135. Applying the rules, only option C will have X = 0 in the
statement 3 is coded as ‘lu’.
new number.
The correct answer is D.
The correct answer is C.
666

Book 1.indb 666 30/04/2019 4:50:03 PM


7.0  Logical Reasoning Practice

136. The common words in both phrases are “gold” and 141. The number moves ahead by 3 places in each term.
“is” - amu and hua. Next, the positions of amu and The middle value moves ahead by 3 places in each
hua suggest that amu is the word for gold. term.
The correct answer is A. The third value moves back by 3 places in each term.
The correct answer is D.
137. From the given statements it can be concluded that:
Engineers = dam/sam 142. For column 1: 52 + 32 – 22 = 30
Like = krit/prit For column 2: 62 + 42 – 32 = 43
Hard = dam/sam For column 3: 62 + 42 – 12 = 51
Work = krit/prit Similarly, for column 4: 72 + 52 – 22 = 70

Is = pram The correct answer is E.

Worship = drit 143. 12 + 5 = 6


Richa = sanu/kram 22 + 5 = 9
Doctor = sanu/kram 32 + 5 = 14
So the code for ‘like’ cannot be determined.
42 + 5 = 21
Alternatively,
52 + 5 = 30
‘is’ is the only word common in 2nd and 3rd
statements, and correspondingly, the code for ‘is’ is Therefore, 62 + 5 = 41
‘pram’. ‘Like’ and ‘work’ are common in 1st and 2nd
The correct answer is C.
statements. Correspondingly, the code for ‘like’ and
‘work’ are ‘krit’ and ‘prit’, but we don’t know which 144. 12 + 22 = 5
code is for which of these two words. So, we cannot
32 + 42 = 25
determine the code for ‘like’.
52 + 62 = 61
The correct answer is E.
72 + 82 = 113
138. grp ptr prt = mixed fruit juice (1) 92 + 102 = 181
pnr ptr qrs nsr = everybody should eat fruits (2) The correct answer is D.
ntr prt nts nrp = I like pineapple juice (3)
From statements (1) and (2), we get 145. 5th letter from the right = N
ptr = fruits 15th letter to the left of N = Z
From statements (1) and (3), we get Note that this is the same as finding out the 20th
letter from the left.
prt = juice
The correct answer is A.
Therefore, from statement (1), we get
grp = mixed 146. P is the sixth element from the right.
The correct answer is A. 4 is the eighth element from the left.
Between P and 4, there are 11 elements. Thus, the
139. This is a Fibonacci sequence in which the sum of a sixth element from either side would be the middle,
term is equal to the sum of the previous two terms. which is J.
Thus, the required term = 31 + 50 = 81.
The correct answer is B.
The correct answer is C.
147. The number in the inner circle is obtained by taking
140. The numbers are all cubes of prime numbers—23, 33
the difference between the two numbers on each
and so on.
side in the outer circle. Also, if this difference is
Thus ? = 53 = 125 greater than 10, then the order of the digits are
The correct answer is C. reversed.

667

Book 1.indb 667 30/04/2019 4:50:04 PM


NMAT by GMAC™ Official Guide 2019

For example, the difference between 37 and 24 on 18 × 5 = 90


top of the outer circle is 13, so the number on top in 90 × 7 =630
the inner circle is 31 (digits get reversed).
II. 6 + 2 =8
Using the same logic, the required number
8 + 3 = 11
= 31 - 15 = 16
11 + 4 =15
Since 16 > 10, the digits get reversed, giving 61 as
15 + 5 = 20
the answer.
The correct answer is A.
The correct answer is D.
154. The series is following the pattern given below:
148. All the three-letter words are coded in such a manner
that the third letter is the first in alphabetical order +4 +4 +4 +4
followed by the first and second letters (as in HIJ, RST C  → G  → K  → O  → S
and so on). Only option D satisfies this criterion (CDE). +2
K  +2
→ M  +2
→ O  +2
→ Q  → S
The correct answer is D. −3
X  −3
→ U  −3
→ R  −3
→ O  → L

149. 1240 = 31 × (27 + 13) The correct answer is D.


1066 = 41 × (9 + 17)
Using the same rule, 155. 2 × 1 = 2
the required number = 13 × (29 + 62) 4 × 2 = 8
= 13 × 91 7 × 3 = 21
= 1183 11 × 4 = 44
16 × 5 = 80
The correct answer is E.
Therefore, 22 × 6 = 132
150. In the second square, the bottom right corner
The correct answer is C.
number should be 3, because the sum of the row is
5. Now, the top right corner number should be 7 as
156. INDIAN would be 17 + 27 + 7 + 17 + 1 + 27
the sum of the column is 10. Therefore, value of x is
or 96
8, as the sum of the top row 15.
GERMAN would be 13 + 9 + 35 + 25 + 1 + 27
The correct answer is B.
or 110
151. 101, 109, 136, 200, 325? So the difference is 14.
3
109 – 101 = 8 = 2 The correct answer is A.
3
136 – 109 = 27 = 3
157. The sum of the values in each row is 50.
200 – 136 = 64 =43
So, X is 7 and Y is 9.
325 – 200 = 125 = 53
7 + 9 = 16
?= 325 + 216 = 541
The correct answer is C.
The correct answer is D.
158. The pattern is that the numbers are
152. 15 + 12 =16, 16 + 22 =20, 20 + 32 =29, 29 +
42 = 45 13 – 12 = 0
W + 12 = X, X + 22 = B, B + 32 = K, K + 42 = A 23 – 22 = 0
2 2 2 2
12 + 1 = 13, 13 + 2 =17, 17+ 3 = 26, 26 + 4 33 – 32 = 0
= 42
And so on.
The correct answer is C.
1 0
153. It is the combination of two series 2 4
I. 6 × 1 =6 3 18
6 ×3 = 18 4 48

668

Book 1.indb 668 30/04/2019 4:50:04 PM


7.0  Logical Reasoning Practice

5 100 162. 3, 5, 9 forms a series with difference as 2 and 4, and


6 180 so next difference will be 8. So, next term will be 9 +
8 = 17.
7 294
8, 14, 26 forms a series with difference as 6 and
8 448
12, and so next difference will be 24. So, next term
9 648 will be 26 + 24 = 50.
10 900 The only option with first term 17 and next term 50
The correct answer is E. is E. So, we need not to find the third term.
The correct answer is E.
159. The number in the rectangle represents the difference
of the numbers in the rectangles below. 163. Starting from U every third letter/number is
selected. After the formation of a tetrad, three
5 letters/numbers are skipped and the same
process is followed.
7 2/12
The correct answer is C.
7 0 2/12

8 1 1 3/13 164. The pattern followed is as below:


10 2 3 4 Z=7/17
A+3=D

P+3=S
Refer the diagram above, the first number in the
second last row will be 8; the first number in the third P+3=S
last row will be 7 and the first number in the second L+3=O
row will be 7. The last number in the second row can
be 2 or 12. E+3=H
If it is 2, then continuing like that, the value of Z will
Similarly, MANGO can be coded as PDQJR.
be 7.
If it is 12, then Z = 17. The correct answer is A.
The correct answer is C.
165. 4th letter from the right = A
160. The pattern followed is 10th letter to the left of A = W
(5 × 6 × 7) + 8 = 218 Note that this is the same as finding out the 17th
Similarly, letter from the left.
(7 × 8 × 9) + 10 = 514
The correct answer is C.
Therefore,
(2 × 14 × 11) + 16 = 324 166. N
The correct answer is D.

161. The pattern followed is as below:


12 – 1 = 0

22 – 1 = 3
W E
32 – 1 = 8

42 – 1 = 15 Vishal
2
5 – 1 = 24

So, the next term will be:

62 – 1 = 35
S
The correct answer is C.
669

Book 1.indb 669 30/04/2019 4:50:05 PM


NMAT by GMAC™ Official Guide 2019

The sun rises in the East. So the shadow of a man 169. The correct answer is E.
in the morning will always fall towards the West.
170. The correct answer is C.
Since the shadow of Sanjay is to the right of Vishal,
Vishal must be heading South. 171. He takes 14 right turns and 12 left turns. Since
The correct answer is C. all turns are at equal angles, 12 right turns get
cancelled by 12 left turns. So effectively, Bhanu has
taken 2 right turns. So he has changed his direction
167.
by 180° by the end of his journey. Since he is facing
West at the end of his journey, the first step must
have been taken in the direction of East.
3 km The correct answer is E.
8 km
172. All the friends are arranged in the following way:
4 km North

Reeta

West East

The correct answer is D. Sulekha Sunaina Preeti

168. South
A
The correct answer is D.
North
4 km 173. Man walks on the following route:

West East 3 km C A B 20m C 10m D


B Home
20m
4 km 4 km 30m 30m
South 3 km
O L

In ∆ ABC, we have: 20m 20m


2 2 2
AC = AB + BC
N
AC2 = 42 + 32 = 25 M 20m
AC = 5 km in South-east
The correct answer is A.
The correct answer is A.
North
General Solution for Questions 169–170:
When Prithvi begins in the morning, his shadow is right
in front of him. This means that he is walking in the West
direction. He then takes a 45o turn in the clockwise direction. D 15km C O
West East
The following figure depicts and the path there on: 10km 10km
B A
6km

Begin
Shadow Office 174. South

Distance from original point


From the above figure we can see that office is facing in the = OC + CD = 6 + 15 = 21 km
Northwest direction. The shadow is at an angle of 45º in the
The correct Answer is A.
clockwise direction in front of him.

670

Book 1.indb 670 30/04/2019 4:50:06 PM


7.0  Logical Reasoning Practice

175. Since it is mentioned that the two were talking in the 2ABC14
morning, the sun has to be in the East; hence, the ×3
shadow would be to the west. If the West is to the
ABC142
left of Piyush, he is facing North. This means, Ravi is
facing South - since the two are facing each other. If we think about C, it can only be 7, because only
714 multiplied by 3 gives a 4-digit number ending
The correct answer is C. with 142. Hence we have
2AB714
176. By simple visualisation, we see that Raman’s head is
towards the North, then he turns to his right towards ×3
the window to see the shadow of his house, which AB7142
means the Sun is to the left of the house, which is the If we think about B, it can only be 5, because only
East. 5714 multiplied by 3 gives a 5-digit number ending
The correct answer is A. with 7142. Hence we have
2A5714
177. The pairs of letter are ST and RS, so the answer is 2. ×3
The correct answer is C. A57142
Lastly, A can only be 8, because only 85714
178. multiplied by 3 gives a 6-digit number ending with
A1 B2 C3 D4 E5 F6 G7 H8 I9 J10 K11 L12 M13 57142.
Z26 Y25 X24 W23 V22 U21 T20 S19 R18 Q17 P16 O15 N14 Hence we have
285714
The opposite of B is Y, D is W, F is U and H is S.
×3
Similarly, the opposite of E is V, G is T, I is R and J is
Q. 857142
Now, adding up the derived values of A + B + C + D
So correct Answer is B.
+ E = 8 + 5 + 7 + 1 + 4 = 25
179. The correct answer is C.
A1 B2 C3 D4 E5 F6 G7 H8 I9 J10 K11 L12 M13 Alternatively,
Z26 Y25 X24 W23 V22 U21 T20 S19 R18 Q17 P16 O15 N14 ABCDE2 is a multiple of 3. So, 2ABCDE is also a
A + 2 = C, C+2 = E, E + 2 = G, G + 2 = I multiple of 3. Since 2ABCDE x 3 = ABCDE2, we can
say that ABCDE2 is a multiple of 9. So, A + B + C +
C – 2 = A, A – 2 = Y, Y – 2 = W, W – 2 = U D + E + 2 = 9k or A + B + C + D + E = 9k – 2. The
D + 2 = F, F + 2 = H, H + 2 = J, J + 2 = L only option is (C).
G – 2 = E,E – 2 = C, C - 2 = A, A – 2 = Y 181. S + S = S, will give the same unit’s digit only if there
Therefore, ? = IULY is a carry-over from the previous addition and no
The correct answer is D. other value except 9 can be true for this.
9 + 9 + 1 = 19
180. If we think about E, it can only be 4, because only 4
multiplied by 3 gives a 2-digit number ending with 2. CA9T
Hence, we have +CA9T
2 A B C D 4 TE9I
×3 Now as there is a carry-over from T + T which means
ABCD42 T can be 6, 7 or 8.
If we think about D, it can only be 1, because only 14 Also a C will not give a carry-over. It will be either 1,
multiplied by 3 gives a 3-digit number ending with 42. 2, 3 or 4. C cannot be 1 or 2 because then the value
Hence we have of T will be 2 or 4, which is not possible as it will not

671

Book 1.indb 671 30/04/2019 4:50:07 PM


NMAT by GMAC™ Official Guide 2019

give a carry-over in the T + T addition. Also, T can 185. Note that to get the last step, that is the final output,
be 7 when C = 3 and there is a carry-over from the we don’t need to go through all the steps. We can
addition of A + A. simply arrange the given terms in alphabetical
If C = 3, T can be 6 or 7 and if C = 4 then T = 8 order (for the words) and descending order (for the
numbers) – 99 jut 56 mat 33 pot 22 tie.
We get the following possibilities:
The correct answer is D.
3A96
+3A96 General Solution for Questions 186–189:
6E92 In this code, the numbers are basically being rearranged
Here, A can be 5, 7 or 8. in every step based on a certain predefined pattern. To
make this simpler to understand, let’s replace each of the
Or, numbers by a letter and rewrite the steps of the code as
3A97 below:
+ 3A97 A = 14; B = 32; C = 99; D = 110; E = 88; F = 9;
7E94 G = 11; H = 18
Here, A can be 5 only. Input: ABCDEFGH
Or Stage 1: G A B C D E F H
4A98 Stage 2: G C A B D E F H
+4A98 Stage 3: G C D A B E F H
8E96 Stage 4: G C D B A E F H
Here, A can be 1, 2, 3, 5 or 7. Stage 5: G C D B F E A H
Therefore, C + S + T = 18 or 19 or 21. The only We’ll be using the above stages to solve all the questions
possible option is (C). asked.
The correct answer is C. 186. As per calculation done earlier:
Stage 1 = G A B C D E F H
182. In the given example, note that in every step, the
Therefore, G = 31; A = 19; B = 47; C = 86; D = 39;
words get arranged alphabetically from left to right
E = 13; F = 66; H = 69
and the numbers get arranged in descending order.
The words and numbers also alternate with the Now, Stage 5 = G C D B F E A H
numbers occupying the first slot. Using this logic: = 31 86 39 47 66 13 19 69
Input: can axe 32 12 kit 57 bat 89 The correct answer is C.
Step 1: 89 can axe 32 12 kit 57 bat
Step 2: 89 axe can 32 12 kit 57 bat 187. As per calculation done earlier:
Step 3: 89 axe 57 can 32 12 kit bat Step 5 = G C D B F E A H
Therefore, G = 141; C = 273; D = 87; B = 41;
The correct answer is D. F = 78; E = 9; A = 319; H = 1012
183. Input: 76 cot 64 dog 45 hen 54 urn Input = A B C D E F G H
Next Step: 76 cot 64 dog 54 hen 45 urn = 319 41 273 87 9 78 141 1012
Next + 1 step: 76 cot 64 dog 54 hen 45 urn The correct answer is B.
Thus, 2 more steps are needed to complete the
188. As per calculation done earlier:
arrangement
Step 4 = G C D B A E F H
The correct answer is B.
Thus, G = wind
C = flows
184. Note that we can never arrive at an input from a
given output because there exist multiple possibilities D = over
for the input. B = the
The correct answer is E. A = river
E = bed

672

Book 1.indb 672 30/04/2019 4:50:07 PM


7.0  Logical Reasoning Practice

F = cool 192. In step III ‘doctors’ is the third element from the left
H = calm end and ‘80’ is the third element from the right end.
Now, Step 2 Similarly, ‘review’ is the second element from the left
end and ‘75’ is the second element from the right
= G C A B D E F H
end.
= wind flows river the over bed cool calm
Now, as ‘would’ is the first element from the left end,
The correct answer is D. so it will be related to the first element which is from
the right end, that is, ‘50’.
189. As per calculation done earlier: The correct answer is C.
Step 2 = G C A B D E F H 193. Looking at the step IV, we find that there are three
Thus, G = system words/numbers which are between ‘procedures’ and
C = restart ‘50’.
A = will The correct answer is A.
B = happen 194. Step 1 = Sum of digits
D = in Step 2 = Letter positions repeated
E = another Step 3 = Input - S1
F = 20 Step 4 = Add the squares of each digit in the number
H = minutes of Step 3
Now, Step 5 = G C D B F E A H Step 5 = The alphabet (go around again after count
of 26)
= system restart in happen 20 another will
Step 6 = Balance of number of letters to reach Z
minutes
The fourth element is 1111
The correct answer is B. Step 1 = 4
Step 2 = DD
General Solution for Questions 190–193:
Step 3 = 1107
The arrangement of output sequence as follow:
The correct answer is B.
In step I, we first shifted smallest word (number of alphabets
in word) to the leftmost side and the largest number to the
195.
rightmost side. In step II, second smallest word is shifted
to the right of the smallest word and the second largest New Input 1203 3211 4522 1111 6666
number is shifted to the right of the largest number. And so
on in next step. For the new input:
Step 1 = 6 7 13 4 24
Step I: would procedures doctors 50 operation 75 45
American review 80 Step 2 = FF GG MM DD XX
Step II: would review procedures doctors 50 operation 45 Step 3 = 1197 3204 4509 1107 6642
American 80 75 Step 4 = 132 29 122 51 92
Step III: would review doctors procedures operation 45 Step 5 = B C R Y N
American 80 75 50 Step 6 = 24 23 8 1 12
Step IV: would review doctors American procedures The correct answer is E.
operation 80 75 50 45
Step V: would review doctors American operation Alternatively,
procedures 80 75 50 45 Step 1 is sum of digits, none of which is 29. Step 2
is alphabets, and so cannot be 29.
190. Looking at step V, we find that ‘American’ is fourth
from the left. Step 3 will be 4-digit numbers, and so cannot be
29. The only possibility is in step 4, and so the only
The correct answer is B. option is (E).
191. There are total five steps required to complete the 196. For the new input ‘3210’:
arrangement.
Step 1 = 6
The correct answer is C.
Step 2 = FF

673

Book 1.indb 673 30/04/2019 4:50:07 PM


NMAT by GMAC™ Official Guide 2019

Step 3 = 3204 200. Use options. Also, note that input element has to
Step 4 = 29 be a perfect square. In option E, all the numbers
are perfect squares so, the option E should be
Step 5 = C
correct.
The correct answer is B. Input: 121, 169, 196, 225, 256, 289
Step I: 11, 13, 14, 15, 16, 17
197. For the new input ‘3021, 1123, 2254, 2222, 5555’:
Step II: 35, 41, 43, 47, 50, 53
Step 1 = 6 7 13 8 20
Step III: 46, 54, 57, 62, 66, 70
Step 2 = FF GG NN HH TT
Step IV: 94, 110, 116, 126, 134, 142
Step 3 = 3015 1116 2241 2214 5535
Step V: 13, 15, 16, 17, 18, 19
Step 4 = 35 39 25 25 84
Alternatively,
The correct answer is D.
Since output = Input + 2,
General Solution for Questions 198–201: So, input = (Output – 2)2
Step I: Square root of input Input series = (13 – 2)2, (15 – 2)2, and so on.
Step II: (Step I × 3) + 2 So, the input series = 121, 169, 196, 225, 256,
289
Step III: Sum of Step I and Step II
Step IV: (Step III × 2) + 2 The correct answer is E.
Step V: (Step IV) – (Step II + Step III)
201. Step II: 44, 30, 32, 26, 14, 8
198. Input = 64 Step I: 14, 12, 10, 8, 4, 2
Step I: 64 = 8 Input: 196, 144, 100, 64, 16, 4
Step II: (8 × 3) + 2 = 26 Correct Input: 4, 16, 64, 100, 144, 196
Step III: 26 + 8 = 34
The correct answer is D.
Step IV: (34 × 2) + 2 = 70
Step V: 70 – (26 + 34) = 10 202. Input: 1, 3, 8, 7, 10, 13
The correct answer is E. Step 1: The positions of the letters are 1 more than
the corresponding number from Input:
199. Step V = 15 b, d, I, h, k, n
Use options. Also, note that input element has to be Step 2: The numbers are 5 more than the Input
a perfect square. 6, 8, 13, 12, 15, 18
Take input: 169 Step 3: The positions of the letters are 1 more than
Step I: 169 = 13 the corresponding number from Step 2
Step II: (13 × 3) + 2 = 41 g, I, n, m, p, s
Step III: 41 + 13 = 54 Step 4: Obtained by adding the numbers of Step 2
Step IV: (54 × 2) + 2 = 110 and the Input
Step V: 110 – (41 + 54) = 15 7, 11, 21, 19, 25, 31
Alternatively, Step 5: The position numbers of the letters
correspond to the numbers given in Step 4
If the input number is x2, then
g, k, u, s, y, e
Step I = x
Step II = 3x + 2 The correct answer is B.
Step III = x + 3x + 2 = 4x + 2
203. The letters are taken from 1 to 26 and then the
Step IV = 2(4x + 2) + 2 = 8x + 6 numbers are restarted, that is, 27 would mean 26 +
Step V = (8x + 6) – [(3x + 2) + (4x + 2)] = x + 2 1, 29 as 26 + 3, 31 as 26 + 5 and so on.
So, if x + 2 = 15, then x = 13 Only the input given in option E will give the same
And, so input = x2 = 169 output as the illustrated example.

The correct answer is E. The correct answer is E.

674

Book 1.indb 674 30/04/2019 4:50:08 PM


7.0  Logical Reasoning Practice

204. Step 5: k, o, s, w, e, i
Step 4: 11, 15, 19, 23, 5/31, 9/35
Step 3: I, k, m, o, s, u
Step 2: 8, 10, 12, 14, 18, 20
Step 1: d, f, h, j, n, p
Input: 3, 5, 7, 9, 13, 15
Alternatively, The correct answer is C.
The inputs for ‘k, o, s’ is ‘3, 5, 7’, which is available
211. On observing the given options, it is clear that figure
only in option B.
(D) is complementary figure of the given figure as the
The correct answer is B. complete figure is:

205. Reducing 65 to the letter position of the alphabet


series, we get
Input: 65 = 26 + 26 +13 = 13
Step 1: n (next letter in the alphabet series)
Step 2: 70 (input + 5) – corresponding letter position
would be r
Step 3: s (next letter position)
The correct answer is D.
Step 4: 135 (Step 2 + Input)
Step 5: e (reducing Step 4 to a letter position in the 212. Inner shape in previous figure is outer shape in the
alphabet series) next one including new shape inside.
The correct answer is D. The correct answer is E.
206. 8
 5 = L, 78 = U, 13 = M, 79 = I, 04 = N, 97 = A,
213. In every step, one � and one is added on ACW
58 = R, 33 = Y
side and rotating at 135° and 90° W alternately.
The correct answer is E.
The correct answer is D.
207. 4
 2 - J, 13 - U, 76 - B, 79 - I, 24 - L, 01 - A, 40 - N,
00 - T 214. All pairs of spiral lines are parallel to each other
The correct answer is E. except in fig (C). The correct answer is C.
215. In all the figures, the object is rotating at 45° in
208. 00 = R 69 = E 89 = L 02 = E 56 = G 01 = A 55 = T clockwise manner. In fig (C), the shape of the object
44 = E is different from the others. Hence, the odd one out
The correct answer is C. is figure (C).
The correct answer is C.
209. 10, 34, 59, 96: All of these coordinates represent 216. Image of all the alphabets are water images of
O. them except in fig. (B), which is mirror image of
89, 20, 57: All of these coordinates represent A. alphabet G.
66 and 78 represent T but not 43. Hence, option A is The correct answer is B.
incorrect.
Again, 89, 56, and 59 do not represent H. Hence, 217. In all the figures, the object is continuously rotating
options B, D and E are all incorrect. 90° in anticlockwise manner and number of lines in
the tail is same except in figure (C).
75 represents H.
The correct answer is C.
The correct answer is C.
218. In all the figures, only one leaf of the flower is shaded
210. On observing the given options, it is clear that figure except in figure D.
(C) is the complementary figure of the given figure as
the complete figure is: The correct answer is D.

675

Book 1.indb 675 30/04/2019 4:50:08 PM


NMAT by GMAC™ Official Guide 2019

219. The pattern followed is as below: (iii) Rahul, Mohan and Naval
First Column: (iv) Rahul, Qadar and Naval
2 2 2 2
(1 + 5 ) – (2 + 4 ) = 26 – 20 = 6 (v) Sanjay, Mohan and Naval
Second Column: (vi) Sanjay, Qadar and Naval
2 2 2 2
(4 + 7 ) – (3 + 6 ) = 65 – 45 = 20 Thus, there are 6 ways of constituting a team if Naval
Similarly, is to be included in the team.
Third Column: The correct answer is E.
2 2 2 2
(5 + 8 ) – (9 + 1 ) = 89 – 82 =7
226. The given information can be represented through
The correct answer is D.  Venn diagram as following:

27
220. Interchanging ‘/’ and ‘–’, the equation becomes;
8 + 7 x 6 – 16/ 8 = 48 35

48 = 48 (LHS = RHS)
Swimming Boxing
The correct answer is B.

221. Changing the correct signs, the equation becomes:


40 + 16 – 16 ÷ 8 x 10 = ? Football
or, 56 – 20 = ?
or, ? = 36
28
The correct answer is D.

222. If Lucky is in the team then as per the rule, Kiran Total number of students who prefer at least one
should also be in the team. If Lucky and Kiran are in game = 75 – 5 = 70 (the 5 students who do not
the team, then rules I and II are not followed. Hence, prefer any will be considered later).
Lucky cannot be a part of a three-member team. n( A ∪ B ∪ C) = n( A ) + n(B) + n(C) − n( A ∩ B) − n( A ∩ C) − n(B ∩ C) + n( A ∩ B ∩
The correct answer is B.
n( A ∪ B ∪ C) = n( A ) + n(B) + n(C) − n( A ∩ B) − n( A ∩ C) − n(B ∩ C) + n( A ∩ B ∩ C)
223. For the largest team, the selection of people can be ⇒ 70 = 27 + 35 + 28 – (10 + 3x) + x,
as following where x is the number preferring all three activities.
(a) One from Puneet, Rahul and Sanjay Therefore, x = 5.
(b) One from Mohan and Qadar So, the number of students preferring at least two
(c) Either pair of Lucky and Kiran or Naval and Ujjwal activities = 10 + 5
(d) Vicky Therefore, the number of students preferring exactly
Thus, the size of the team is 5. one activity = 70 – 15 = 55
Further, 5 students who could have preferred any or
The correct answer is C.
all can be the ones who prefer exactly one activity
which would maximise the number preferring exactly
224. If Kiran is in the team then Lucky will also be in the team.
one activity.
One person should be selected from among Puneet,
Rahul and Sanjay. Again, one person should be selected So, maximum students who prefer exactly one
from Qadar and Mohan. Thus, the size of the team is 4. activity = 55 + 5 = 60
Vicky could also be there as the fifth person but that The correct answer is E.
is not provided in the list of options.
227. Let A be the set of students who like pop songs, B be
The correct answer is E.
the set of students who like classical songs and C be
the set of students who like old songs.
225. Any of the following teams can be constituted if the
team includes Naval: We know that
(i) Puneet, Mohan and Naval n( A ∪ B ∪ C) = n( A ) + n(B) + n(C) − n( A ∩ B)
(ii) Puneet, Qadar and Naval − n( A ∩ C) − n(B ∩ C) + n( A ∩ B ∩ C)

676

Book 1.indb 676 30/04/2019 4:50:09 PM


7.0  Logical Reasoning Practice

n( A ∪ B ∪ C) = 80 + 40 + 70 − 30 − 25 − 20 + n( A ∩ B ∩ C 2) 29. We can’t conclude I as some children might be


80 + 40 + 70 − 30 − 25 studying more than 1 topic and some children might
− 20 + n( A ∩ B ∩ C) not be studying anything.
n( A ∪ B ∪ C) = 190 − 75 + n( A ∩ B ∩ C)
We can’t conclude II as, while we know that 3 children
n( A ∪ B ∪ C) = 115 + n(A ∩ B ∩ C) = 115 + x, who are studying science must be inclined towards
where x is the number of students liking all the three mechanics, we don’t know whether the other children
songs. are inclined towards mechanics
Refer to the following Venn diagram. We can’t conclude III as it is possible that in the given
data 1 or more may be studying 2 or 3 topics
Pop Classical simultaneously. So, there might be some child or
30
30 -15 40 some children who are also studying some other
80 topic or topics.
0
20 25 We can conclude IV as those who are studying arts
may also have an inclination towards mechanics.
25
70 The correct answer is B.
Old
230. a = 6 × 4 = 24
Pop Classical
15
b=4×2=8
45 0 40 c = 6 × 4 × 2 = 48
80
15 d = 6 × 2 = 12
5 10
e=4
40
Thus, e is the minimum and is the correct answer.
70
Old The correct answer is E.

If x = 0, then number of students who like only 231. We need to identify the common region overlapping
classical music becomes –15, which is not possible. Royal Bengal tigers and White tigers only. Note that
Thus, the minimum value of x can be 15 and so the Royal Bengal tiger is represented by rhombus while
minimum number of students who like at least one White tiger is represented by parallelogram. Thus ‘I’
type of song is 115 + x = 115 + 15 = 130. is the common region and hence the correct answer.

The correct answer is B. The correct answer is E.

232. Given the above calculation, there is no way A (or any


228. Let C represents set of students who drink coffee,
other person) could have scored a 5 on any throw
T represents set of students who drink tea and
because we won’t be able to reach a total of 6 (the
A represents set of students who drink aerated
maximum score for the four rounds) among three
beverages.
people then. If one person gets 5, then the lowest
Consider that the total number of students is 100. So, the other two people could get is 1 and 1, which
n(C) = 56, n(T ) = 48, n( A ) = 42 would still add up to 7.

n(C ∩ T) + n(T ∩ A ) + n( A ∩ C) + n(C ∩ T ∩ A ) = 62 The correct answer is E.

Let x be the number of students who drink all 3. 233. We do not know the score of B in the third round,
According to the question, so we cannot calculate the percentage of the round
score contributed by him.
n(C ∪ D ∪ A ) = 56 + 48 + 42 − 62 − x
The correct answer is E.
n(C ∪ D ∪ A ) = 84 − x
Number of students who drink none of the 3 234. Total score in round 1 = 33.33/100 × 18 = 6
Total score in round 2 = 27.77/100 × 18 = 5
n(C ∪ D ∪ A ) = 100 − (84 − x )
Total score in round 3 = 22.22/100 × 18 = 4
So, required difference
Total score in round 4 = 16.67/100 × 18 = 3
= 100 − (84 − x ) − x = 16
Round 4 has the lowest Round score. Each player
The correct answer is A. in this Round must have scored 1. As per the

677

Book 1.indb 677 30/04/2019 4:50:10 PM


NMAT by GMAC™ Official Guide 2019

questions, two players got this score (a score of 1) 237. From option (A)
in the remaining three rounds as well. Thus, these 20 + 25 ÷ 15 × 4 – 6 = 10
two players contributed 2 points to a total score of 5
obtained in Round 2. Therefore, the third player must After changing sign,
have got a score of 5 - 2 = 3.
20 ÷ 25 – 15 + 4 × 6
The correct answer is C. 4
= – 15 + 24
5
235. Total score in round 1 = 33.33/100 × 18 = 6
Total score in round 2 = 27.77/100 × 18 = 5 4 49
= +9=
5 9
Total score in round 3 = 22.22/100 × 18 = 4
Total score in round 4 = 16.67/100 × 18 = 3 From option (B)
In the first round, the highest score by a person 20 – 5 × 10 ÷ 20 + 4 = 20
could be a 3 or 4, so C must have scored a 1 or 2.
After changing sign,
In the second round, the highest score by a person
20 × 5 + 10 – 20 ÷ 4
could be a 2 or 3. So C must have scored a 1.
= 100 + 10 – 5
In the third round, the highest score by a person has
to be 2, so C must have scored a 1 = 100 + 5 = 105
In the fourth round, each player scored a 1. From option (C)
Thus C’s total possible score can be a 4 or a 5, 50 + 10 – 5 ÷ 5 × 20
making A the correct answer. After changing sign.
The correct answer is A. 50 ÷ 10 × 5 – 5 + 20
= 5 × 5 – 5 + 20
236. Checking through options, we get:
From option (A) = 25 – 5 + 20 = 40

16 × 8 + 2 ÷ 3 − 4 From option (D)


2 15 – 5 × 50 + 25 ÷ 10
= 128 + − 4
3 After changing sign.
2 372 + 2 374 15 × 5 + 50 ÷ 25 - 10
= 124 + = =
3 3 3
= 15 × 5 + 2 – 10
From option (B) = 77 – 10 = 67
16 × 8 ÷ 2 + 3 − 4 The correct answer is C.
= 16 × 4 + 3 − 4
= 64 + 3 − 4 238. After changing signs, the given expression becomes
= 63 16 × 5 – 9 ÷ 3 + 20
= 16 × 5 – 3 + 20
From option (C)
= 80 – 3 + 20 = 97
16 − 8 ÷ 2 + 3 × 4
= 16 − 4 + 12 The correct answer is D.
= 12 + 12 239. Observing squares of numbers from 1 to 25,
= 24 the last two digits are 7p only if p = 6, which is
possible when ‘bc’ = 24 or 26. So, the value of p
From option (D) is 6.
16 − 8 ÷ 2 × 3 + 4 Moreover, if unit’s place of a square number is an
= 16 − 4 × 3 + 4 odd number, then ten’s place has to be an even
number. So, if ten’s place is 7, then unit’s place
= 16 − 12 + 4
cannot be odd. The only option is, p = 6.
= 20 − 12 = 8
The correct answer is D.
The correct answer is D.

678

Book 1.indb 678 30/04/2019 4:50:11 PM


7.0  Logical Reasoning Practice

240. BAF ÷ DF = 105 ÷ 35 = 3 245. Number of ways = 12C8 × 3C2 × 3C1


JJ ÷ D = 99 ÷ 3 = 33
= 12 × 11 × 10 × 9 × 3 × 3 = 4455
4!
BAF ÷ DF + JJ ÷ D = 3 + 33 = 36
The correct answer is A.
The correct answer is B.

241. ABC – D09 = DEF General Solution for Questions 246–248:


So, n(R È P È Y) = n(R) + n(P) + n(Y) – [n(R Ç P) + n(P Ç Y) +
10 + C – 9 = F n(R Ç Y)] + n(R Ç P Ç Y)
⇒ F = C +1 ® 580 = 200 + 250 + 350 – (80 + 120 + 60) + x
So, (B – 1) = E ® x = 40
And finally,
We can draw the following Venn diagram.
A – D = D ⇒ A = 2D
A–B=C Red
200 Pink
D–C=F=C+1 40 250
⇒ D = 2C + 1 100 90
A 40
⇒ = 2C + 1 20 80
2
⇒ A = 4C + 2 210
So, C can be only 1 and so A = 6 and hence D = 3.
Therefore, 350 Yellow
B=A–C=6–1=5
C+D=E 246. The number of people who like only Red is 100.
⇒E = 1 + 3 = 4 The correct answer is E.
F=C+1=2
247. The number of people who like only Pink is 90.
So, D = 3 and A + F = 6 + 2 = 8
The correct answer is D.
The correct answer is B.

242. 16 ÷ 4 = 144 ® 16 – 4 = 12 ® 122 = 144 248. The number of people who like only Yellow and all 3
= 40 + 210 = 250.
15 ÷ 3 = 144 ® 15 – 3 = 12 ® 122 = 144
10 ÷ 2 = 64 ® 10 – 2 = 8 ® 82 = 64 The correct answer is E.
2
20 ÷ 5 = ? ® 20 – 5 = 15 ® 15 = 225
General Solution for Questions 249–252:
The correct answer is D.  n the basis of the information given, we can draw the
O
following Venn diagram.
243. There are total 8 batsmen in team A and we want to
select 6. The required number of ways = 8C6 = 8 ! Cricket 3x
2!6! Hockey
7 400
= 8 x =28 ways. 2x 600
2 2y 300
The correct answer is D. x
x
244. Total number of ways = 10C6 × 4C3 × 1C1 2x
= 10 × 9 × 8 × 7 × 4 × 1 =840 7y
4!
800 Football
The correct answer is C.

679

Book 1.indb 679 30/04/2019 4:50:13 PM


NMAT by GMAC™ Official Guide 2019

If x is the number of players who play all three games, then Girls
number of players who play Cricket and Hockey is 3x and
therefore those who play only Cricket and Hockey is 2x. Mothers
Similarly, number of players playing Cricket and Football is
2x and so those who play only Cricket and Football is x. Grandmothers
Also, if 2y play only Cricket, 7y play only Football.

Now, 2y + 2x + x + x = 400 or y + 2x = 200, and

 umber of players playing only Hockey and Football


N
= (800 – 7y – x – x) which is same as (600 – 300 – 2x – x). The correct answer is B.
So, 800 – 7y – x – x = 600 – 300 – 2x – x or – 7y + x =
–500. 255. If one object of group. A completely belongs to
another group B but both are entirely different from
 olving, we get x = 60, y = 80. The revised Venn diagram
S group C, so they are represented as
will be
Cricket Humans
Hockey
400
120 600
160 300 Animals Children
60
60 120

560
The correct answer is C.
800 Football
256. The number of educated men who are in government
249. The number of players who play Cricket and Football job = 20 (common region to all)
= 60 + 60 = 120
The correct answer is B.
The correct answer is E.
257. The number of people in government job who are not
250. The number of players who play Cricket and Hockey educated =15 + 40 = 55
= 60 + 120 = 180 The correct answer is D.
20% of 180 = 36
258. The angle covered by hour hand at 7 o’clock
The correct answer is E.
= 30 × 7 = 210o
251. The number of players who play Hockey and Football The angle covered by minute hand in 20 minutes
= 60 + 120 = 180. = 20 × 6 = 120o
The correct answer is C. The angle covered by hour hand in 20 minutes
1
= 20 × =10o
252. The required difference = 300 – 60 = 240. 2
The correct answer is B. Angle between hour hand and minute hand at 7:20

253. Some mothers are also doctors and some fathers = 210o + 10o – 120o = 100o
are also doctors. The angle covered by hour hand at 9 o’clock
The correct answer is D. = 30o × 9 = 270o
The angle covered by minute hand in 30 minutes
254. All grandmothers are definitely mothers, and all the = 30 × 6 = 180o
mothers are girls for sure. Therefore, the Venn-
The angle covered by hour hand in 30 minutes
diagram would be:

680

Book 1.indb 680 30/04/2019 4:50:13 PM


7.0  Logical Reasoning Practice

1 260. Number of used cars left after the sale of only used
= 30 × = 15o
2 cars = 72 – 41 = 31

Angle between hour hand and minute hand at 9:30 Number of used cars after the exchange of used cars
= 31 – 27 + 27 = 31 (because each is exchanged for
= 270o + 15o – 180o = 105o another one)

Required difference = 105o – 100o = 5o Thus, number of used cars after the exchange of used
cars with new cars = 31 + 31 = 62
The correct Answer is D.
The correct answer is C.
Alternatively,
At 9:30, the angle between the two hands is 261. 10% who can repair all 3 = 10% of 140 = 14
90 + x, where x is the angle covered by hour hand in Those who can repair exactly 2 = 50 (given)
30 minutes. Number of those who can repair only ACs and only
At 7:20, the angle between the two hands is Refrigerators = x + x =2x
90 + y, where y is the angle covered in hour hand in 140 – (22 + 50 + 14) = 2x
20 minutes.
or x = 27
So, required difference = x – y = angle covered by
hour hand in 30 – 20 = 10 minutes. The correct answer is D.

1 262. 2004 is a leap year.


Since speed of hour hand is °/min, required
2 Number of days between 4th January, 2003 and 4th
1 March, 2004
difference = °× 10 = 5°.
2
= (365 – 4) + 31 + 29 + 4 = 425
259. Advocates who are politicians but not doctors is 5 and = 60 weeks + 5 odd days
can be found in the overlap of triangle (politicians) and
Thus, 4th March, 2004 will be 5th day after Saturday,
rhombus (advocates) but not circle (doctors).
i.e., Thursday.
The correct answer is E.
The correct answer is D.

681

Book 1.indb 681 30/04/2019 4:50:14 PM


NOW AVAILABLE FOR 2019

•  F ree full length online practice exam of 120 NMAT past questions with
answer explanation
•  F ollows the similar structure and feature as the actual NMAT by GMAC™
exam

•  2
 full length of 240 NMAT past questions with answer explanation and
scaled score
•  F ollows the similar structure and feature as the actual NMAT by GMAC™
exam

Chapter 7.indd 682 01/05/2019 4:38:24 PM


•  G
 et access to Practice Exam 4 through the access code in the NMAT by
GMAC™ official guide
•  Full length exam of 120 NMAT past questions with answer explanation
•  F ollows the similar structure and feature as the actual NMAT by GMAC™
exam

•  C
 heck your preparation level by taking the Free NMAT by GMAC™ All
India Mock exam
•  Full length exam of 120 NMAT past questions with answer explanation
•  F ollows the similar structure and feature as the actual NMAT by GMAC™
exam

For more information on NMAT by GMAC™ Official Prep offerings,


please visit: https://www.nmat.org.in/nmat-by-gmac-official-prep/

Chapter 7.indd 683 01/05/2019 4:38:26 PM


Official Guide 2019
Official Guide 2019 Official Guide 2019
The definitive guide from the makers of the NMATTM exam
Book + Online

Here’s what you will find inside: Book + Online


„ Information about the exam format and structure
„ Extensive review of concepts and question types of each section, including tips
The ONLY
and strategies for success
source of real
TM
„ Past NMAT
TM
questions with answer explanations
NMAT
questions
„ Practice questions with answer explanations
from past
Never before seen: 240 NMATTM questions

TM
exams
„ Other official prep materials from the house of GMAC: NMAT by GMAC Official
Prep Online Practice Exam offerings including an NMAT by GMAC™ Official Prep
Practice Exam 4 (Official Guide Access Code) exclusively with the purchase of
the Official Guide through access code provided inside. This edition includes
TM
Visit nmat.org.in, the official website of the NMAT by GMAC exam, to learn more, 240 never-before-seen questions
register and purchase official prep materials. „ Discussion of concepts of each section with relevant examples
Cover Design: Wiley Book „ 480 questions from past NMAT by GMAC
TM
exams including
+ 240 new questions, with answer keys & explanations
ISBN 978-81-265-9818-2

„ 625 practice questions with answer keys & explanations

Online
9 788126 598182

Price: `899/- Graduate Management Admission Council nmat.org.in

You might also like